BPSC Solved Papers English Medium 2022-23

You might also like

Download as pdf or txt
Download as pdf or txt
You are on page 1of 336

Youth Competition Times

BIHAR PUBLIC SERVICE COMMISSION

Preliminary Examination
GENERAL KNOWLEDGE
&

GENERAL SCIENCE
SOLVED PAPERS
Chief Editor
A.K. Mahajan
Editor
Advocate Abhishek Singh
Writers
Vishan Kumar, Aakash Singh, Abhishek Tripathi
Shivam Pandey, Shivanshu Mehta, Saurabh Mishra
Computer Graphics by
Balkrishna, Charan Singh, Ashish Giri
Editorial Office
Youth Competition Times
12, Church Lane Prayagraj-211002
Mob. : 9415650134
Email : yctap12@gmail.com
website : www.yctbooks.com
Publisher Declaration
Edited and Published by A.K. Mahajan for YCT Publications Pvt. Ltd.
and printed by Roop Printing Press, Prayagraj. In order to Publish the book,
full care has been taken by the Editor and the Publisher,
still your suggestions and queries are welcomed.
In the event of any dispute, the judicial area will be Prayagraj. Rs. : 325/-
CONTENT
38th Bihar Public Service Commission (Pre) Examination, 1992-93 .......................................... 3-12

39th Bihar Public Service Commission (Pre) Examination, 1994 .............................................. 13-27

40th Bihar Public Service Commission (Pre) Examination, 1995.............................................. 28-42

41th Bihar Public Service Commission (Pre) Examination, 1996.............................................. 43-57

42th Bihar Public Service Commission (Pre) Examination, 1997-98......................................... 58-70

43th Bihar Public Service Commission (Pre) Examination, 1999 .............................................. 71-83

44th Bihar Public Service Commission (Pre) Examination, 2000-01......................................... 84-98

45th Bihar Public Service Commission (Pre) Examination, 2002 ............................................ 99-112

46th Bihar Public Service Commission (Pre) Examination, 2003-04..................................... 113-129

47th Bihar Public Service Commission (Pre) Examination, 2004-05..................................... 130-147

48th-52th Bihar Public Service Commission (Pre) Examination, 2007-08 ............................ 148-163

53th-55th Bihar Public Service Commission (Pre) Examination, 2011.................................. 164-177

56th-59th Bihar Public Service Commission (Pre) Examination, 2015.................................. 178-193

60th-62th Bihar Public Service Commission (Pre) Examination, 2016-17 ............................ 194-212

63th Bihar Public Service Commission (Pre) Examination, 2017-18..................................... 213-230

64th Bihar Public Service Commission (Pre) Examination, 2018-19..................................... 231-249

65th Bihar Public Service Commission (Pre) Examination, 2019 .......................................... 250-266

65th Bihar Public Service Commission (Pre) Examination, (PH) Re-Exam 2019 ................ 267-284

66th Bihar Public Service Commission (Pre) Examination, 2020 .......................................... 285-300

66th Bihar Public Service Commission (Pre) Re-Examination, 2020 .................................... 301-316

67th Bihar Public Service Commission (Pre) (Cancelled) Examination, 2021-22 ................ 317-336
2
38th Bihar Public Service Commission
Preliminary Examination, 1992-93
GENERAL KNOWLEDGE & GENERAL SCIENCE
(Solved Paper with Detail Explanation)
1. The Gandhara school of Art flourished in Ans. (c) : In 1877, the UK’s conservative party’s Prime
which of the following periods? Minister Benjamin Disraeli had proclaimed Queen
(a) Kushan's Period (b) Gupta's Period Victoria as Empress of India.
(c) Akbar's Period (d) Maurayan Period 5. Whom did Mahatma Gandhi consider as his
Ans. (a) : In the first Century AD, The Gandhara school political Guru ?
of Art was developed during the reign of Kushana (a) Rabindra Nath Tagore
Emperor Kanishka. Gandhara school of art was the (b) Madan Mohan Malviya
combination of Greco-Roman and Indian ideas along (c) Dada Bhai Nauroji
with the influence of other foreign tradition such as (d) Gopal Krishna Gokhale
China and Iran. This art was closely associated with Ans. (d) : The liberal Gopal Krishna Gokhle (1866-
Mahayana Buddhism and Bodhisatvas. Gandhara school 1915) was considered as Political Guru or Mentor of
is known for the first sculptural form of the Buddha in Mahatma Gandhi. G.K. Gokhale was a renowned social
Human form. reformer of India. He has presided over the Banaras
session of INC in 1905. G.K. Gokhale had formed
2. Tipu Sultan established his capital at ?
Servant of India Society on 12th June 1905 in Pune,
(a) Srirangapattanam (b) Mysore Maharashtra.
(c) Bangalore (d) Coimbatore
6. Simon Commission first came to India in ?
Ans. (a) : Tipu Sultan was the son of Mysore ruler (a) 1926 (b) 1928
Haider Ali. Tipu is well known as the ‘Tiger of (c) 1939 (d) 1942
Mysore’. He ruled Mysore from 1782 to 1799. Tipu
Ans. (b) : The Simon Commission was formed in
Sultan built Srirangapattanam in 1784 and made it his November 1927, consisted of seven English member
capital. headed by John Simon.
The treaty of Srirangapattanam after third Anglo The Simon Commission arrived in British India on 3rd
Mysore war was signed between Tipu Sultan and Lord February , 1928. The Commission appointed to review
Cornwallis in March 1792. the Government of India Act 1919 and was formed for
3. Jantar Mantar was constructed by ? constitutional reforms and make recommendations to
(a) Akbar the Government of India. Simon Commission submitted
its reports on 27th May 1930.
(b) Shahjahan
(c) Shivaji 7. In which year Indian National Congress (INC)
passed the 'Complete Independence' resolution?
(d) Maharaja Jay Singh
(a) 1930 (b) 1929
Ans. (d) : The Historical Monument Jantar-Mantar was (c) 1917 (d) 1911
constructed by enlightened ruler Sawai Jai Singh II in
Ans. (b) : The Indian National Congress (INC), on 19th
early 18 century.
December 1929, passed the historic resolution of
Jantar-Mantar, (Jaipur) was one of the five Complete Independence (Purna Swaraj) at its Lahore
observatories built by Raja Jai Singh II, other are in session. In this session a public declaration was made
Delhi, Ujjain, Mathura and Varanasi. Jantar-Mantar of that on 26 January 1930 – a day on which the congress
Jaipur (Rajasthan) is in the list of UNESCO world party urged Indians to celebrate as ‘Independence Day’.
Heritage site. 8. The year 1919 in Indian History is related to ?
4. Queen Victoria was appointed as a empress of (a) Shift of capital from Calcutta to Delhi
India in ? (b) Jallian Wala Bagh Massacre
(a) 1858 (b) 1876 (c) Division of Bengal
(c) 1877 (d) 1885 (d) Khilafat Movement

38th BPSC (Pre) Exam. 1992-93 3 YCT


Ans. (b) : Jallianwala Bagh Massacre is also known as Ans. (d) : The ethnic tribe ‘Munda’s, which means,
Amritsar Massacre. It was an incident which occurred “headman of a village” inhabitated primarily in the
on 13 April 1919, in which British troops fired on a northeast states of Jharkhand (Before November 2000,
large crowd of unarmed Indians in an open space Bihar), Odisha & West Bengal. They are one of the
known as the Jallianwala Bagh in Amritsar in Punjab. largest scheduled tribes in India. The famous freedom
9. The writer of the song, 'Vande Mataram' is ? fighter Birsa Munda was from Munda Tribes.
(a) Bankim Chandra Chatterjee 14. Lakshadweep Island is located in ?
(b) Sarojani Naidu (a) South-Western India
(c) Rabindra Nath Tagore (b) In Southern India
(d) Jai Shankar Prasad (c) South-Eastern India
Ans. (a) : The Indian poet and Journalist Bankim (d) Eastern-Indian Near West Bengal
Chandra Chatterjee has written and composed the Ans. (a) : The Lakshadweep island is located in the
National song, Vande Mataram, in 1870. It got South-west part of the Arabian Sea. Lakshadweep is an
popularized on its inclusion in Bengali fiction novel,
archipelago of 36 Islands, and it is a Union territory of
“Anandmath’.
India. Kavaratti is the Capital of Lakshdweep.
Rabindra Nath Tagore wrote the National anthem, ‘Jana
Gana Mana’. 15. In which of the following states Iron ore is not
10. Who led the movement for separate State of found ?
Pakistan ? (a) Bihar (b) Madhya Pradesh
(a) Agha Khan (c) Orissa (d) Punjab
(b) Nawab Saleemullah Ans. (d) : Iron ore is not found in the state of Punjab.
(c) Liyaqat Alikhan India’s leading iron ore producing state is Odisha
(d) Mohammad Ali Jinnah (Singhbhum), where more than 50% of Indian iron ore
Ans. (d) : Muslim League demanded a separate State of reserves occurs. The other iron ore producing states are,
Pakistan in the Lahore session of the Muslim League Bihar, Jharkhand, Karnataka, Maharashtra, Telangana,
held in 1940. It was presided over by Mohammad Ali Andhra Pradesh and Goa.
Jinnah, where the “Two nation theory” was forwarded. 16. The country that exports the highest amount of
Muhammad Ali Jinnah led the movement for separate Petroleum ?
state of Pakistan. (a) Algeria (b) Iran
11. When was the Indian National Congress (c) Nigeria (d) Saudi Arbia
established ? Ans. (d) : Saudi Arabia is the largest Petroleum
(a) 1852 (b) 1884 exporter country. It possesses around 15-17% of the
(c) 1870 (d) 1885 world’s petroleum reserves. Saudi Arabia also exports
Ans. (d) : The Indian National Congress (INC) was Natural Gas, Gold, Iron ore and copper.
established on 28th December 1885 in Bombay by 17. The distance between the two rails of the broad
British Civil Servant Allan Octavian Hume. The first gauge railway track is ?
session of INC in 1885, was held at Bombay at the
1 1
‘Gokuldas Tejpal Sanskrit College’. The President of (a) 6 Feet (b) 5 Feet
the first INC session was W.C. Bannerjee and A.O. 2 2
Hume was the General Secretary. 1
(c) 5 Feet (d) 4 Feet
12. Which of the famous Indian leader is known as 2
the 'Frontier Gandhi' ? Ans. (b) : The distance between two rails of broad
(a) Khan Abdul Gaffar Khan gauge is 5½ feet or 1.676 meter. Railway gauge is a
(b) Abul Kalam Ajad clear minimum distance between the inner sides of two
(c) Netaji Subhas Chandra Bose tracks.
(d) Mohammad Ali Jinnah 18. Nicolaus Copernicus is famous for :
(a) Invention of Telescope
Ans. (a) : Khan Abdul Gaffar Khan was known as the,
(b) He proposed that planets revolve around the
‘Frontier Gandhi’. He was spiritual leader and
sun
politician better known for his non-violent movement (c) Discovery of Calculus
like Mahatma Gandhi, hence he was called as frontier (d) Study of surgery of Human body
Gandhi. He led the Khudai Khidmatgar movement.
Ans. (b) : Nicolaus Copernicus was better known for
13. The Munda Tribes are mostly inhabited in ? discovering that it is not sun but planets which revolve
(a) In Madhya Pradesh (b) In Uttar Pradesh around the sun. He is credited with giving Heliocentric
(c) In Assam (d) In Bihar Model (Sun is at the center) of Solar System.
38th BPSC (Pre) Exam. 1992-93 4 YCT
19. The Desert of Gobi is in? 24. At sea level, which place is closest to the center
(a) China (b) Western Africa of the earth ?
(c) Southern Australia (d) Southern America (a) North Pole
Ans. (a) : The Gobi desert is a cold desert situated in (b) Tropic of Capricorn
the Asian continent, it stretches across the Mongolia (c) Tropic of Cancer
and China. It is the fifth largest desert in the world. (d) Equator
Sahara desert of Africa is the largest desert in the world. Ans. (a) : At sea level North Pole is very nearest to the
Thar of Rajasthan is the largest desert of India. centre of the Earth, this is due to bulging tendency of
20. The state of Bihar is situated between one of earth that is bulged at sides and flat on top.
the following longitudes : 25. What is Esperanto ?
(a) About 84º East to 88º East (a) Highest mountain of Latin America
(b) About 80º East to 84º East (b) Port city of Spain
(c) About 80º East to 88º East (c) Name of a Sports
(d) None the above (d) An Artificial language developed to serve as
Ans. (d) : Bihar state is situated between the the global language
coordinates, 25o20’ N and 27o31’ N longitude and Ans. (d) : Esperanto is an artificial language developed
83o19’E to 88o17’E longitude. It is entirely land-locked to serve as a global language. It was created by polish
State of India and a subtropical region of the temperate ophthalmologist L.L. Zamenhof in 1887.
zone. Nepal shares an international border with Bihar. 26. The language which is spoken by the highest
Bihar is surrounded by Uttar Pradesh, Jharkhand and number people in India after Hindi is ?
West Bengal. (a) Marathi (b) Tamil
21. Which city does River Seine Pass through ? (c) Telugu (d) Urdu
(a) London (b) Paris Ans. (c) : As on 1992 data, Telugu was the second most
(c) Rome (d) Frankfurt spoken language in the country.
Ans. (b) : River Seine is the second largest river of As per 2019, the data of people speaking the languages
France after Loire river. It rises from North-west of is:–
Dijon and passes through Paris city of France. Seine Hindi - 52, 83, 47, 193
river fall into the English channel. Bengali - 9, 72, 37, 669
22. Suez Canal connects ? Telugu - 8, 11, 27, 740
(a) Mediterranean sea with Red Sea Tamil - 6, 90, 26, 881
(b) Mediterranean sea with Caspian Sea 27. Which of the following statement is incorrect
(c) Red sea with Indian Sea about India's Vice-President ?
(d) None of the above (a) For being eligible the candidates age must be
Ans. (a) : Suez Canal, an artificial (man made) water at least 35 years
way is located in Egypt that connects the Red sea to the (b) Vice-President is the chairman of the Rajya
Mediterranean sea. It is very important as it offers Sabha
quickest shipping route between Europe and Asia. (c) Vice-President is elected by the President of
23. Ozone layer refers to – India
(a) The Atmospheric condition of Antarctica (d) S. Radhakrishnan was the first Vice-President
(b) The recent discovery made on Saturn. of India
(c) The layer about 10-20 km above the earth's Ans. (c) : According to Article 66 of Indian
surface Constitution Vice-President is not elected by President
(d) The layer of atmosphere about 15-20 km of India. He is elected indirectly by an electoral college
above the surface of the earth method by a majority vote consisting of total member of
Ans. (d) : Ozone is a molecule that contain three Parliament (both houses and both elected and
oxygen atoms. The Ozone layer is a layer of atmosphere nominated).
which sets in the Stratosphere between 15 km to 25 km 28. If there is dispute in the election of the
above the surface of earth. Ozone layer protects from President of India, that dispute can be handed
the harmful ultraviolet radiation which comes from the over to ?
(a) Advocate General of India
sun. Most importantly, it absorbs a portion of UV light
(b) Parliament
called UV-B. UV-B that has many harmful effects on (c) Supreme Court of India
human, crops and marine life. (d) None of the above

38th BPSC (Pre) Exam. 1992-93 5 YCT


Ans. (c) : According to the Article 71, the Constitution 33. What is the sanctioned strength of Judges in
of India clearly mentioned that if there is any doubt and the Supreme Court?
dispute arising out of President or Vice-Presidential (a) 24 (b) 20
election it will be handled by Supreme Court of India. (c) 18 (d) 9
29. Indian constitution does not give one of the Ans. (a) : In 1992, there was 24 sanctioned strength
following rights to the President? seats for the Judges of Supreme Court including the
(a) To appoint the Prime-Minister Chief Justice of India. Currently, there are 34 judges,
(b) To appoint the Chief-Minister of states including CJI, in the Supreme Court of India.
(c) To be the Commander-in-Chief of defense 34. The first Indian constitutional amendment bill
forces was introduced in -
(d) To declare the emergency in any part to the (a) In 1950 (b) In 1951
country. (c) In 1955 (d) In 1958
Ans. (b) : According to the Article 164 of Indian Ans. (b) : The first amendment bill of the Indian
Constitution, the Chief Minister of the state is appointed Constitution was introduced in the year 1951. By this
by the Governor of the related State. The Governor also amendment two new Articles 31-A and 31-B and the 9th
appoints other Ministers on the advice of the Chief schedule were added to the Constitution. The Act was
Minister. The Council of Ministers is collectively passed specifically to give constitutional status validity
responsible to legislative assembly of the state. to Zamindari abolition laws, as they were challenged in
Hence, option (b). is incorrect. Supreme Court.
30. If a money bill is passed by the Lok Sabha at 35. Sikkim was made a State of India ?
most how long can Rajya Sabha keep the bill (a) Under 30th Amendment
pending ? (b) Under 32nd Amendment
(a) 6 months (b) 4 months (c) Under 35th Amendment
(c) 1 months (d) 14 days (d) Under 42nd Amendment
Ans. (d) : According to the Article 109(5) of Indian Ans. (c) : As per 35th Constitutional Amendment Act,
Constitution, Rajya Sabha is required to return Money (1974) Sikkim was a special Associate state of India.
bill passed and transmitted by Lok Sabha within a After 36th Amendment Act (1975), Sikkim became the
period of 14 days from the date of its receipt of the bill 22nd full fledged state of Union of India. The Capital of
in the Rajya Sabha secretariat and not from the date on Sikkim is Gangtok.
which it is laid on the table of Rajya Sabha.
36. Who was the chairman for the committee that
31. The bill for the amendment of Indian recommended Panchayati Raj in India
constitution can be introduced in ? (a) Balwant Rai Mehta
(a) Only in Lok Sabha
(b) B.R. Ambedkar
(b) Only in Rajya Sabha
(c) Justice Krishna Ayyar
(c) Either Lok Sabha or Rajya Sabha
(d) Jag Jeevan Ram
(d) In the Supreme Court of India
Ans. (a) : The Panchayati Raj system in India signifies
Ans. (c) : As per the Article 368 of the Constitution of
the role of self-government. The Government of India
India, the bill for the amendment of Indian Constitution
appointed a committee namely Balwant Rai Mehta
could be introduced in either House of the Parliament
committee under the chairmanship of Balwant Rai
(Rajya Sabha or Lok Sabha), but each house must pass
Mehta, the committee recommended Panchayati Raj
the bill separately. In case of a disagreement between
Institution in 1959. The committee was constituted to
the two houses, there is no provision for joint sitting of
study why Community Development Program and
the two houses.
National Extension Services failed in reaching the
32. Which of the following High Courts has
desired outcomes.
Jurisdiction over more than one State/Union
Territory ? 37. What is the importance of the Panchayati Raj
(a) Allahabad (b) Delhi System of the Government ?
(c) Guwahati (d) Maharashtra (a) There is a three tier government
Ans. (c) : Currently there are 25 High Courts in India. (b) Its objective is to provide social and
In Assam, Guwahati High Court has jurisdiction over economic Justice to villages
more than one state/UT’s which are Assam, Nagaland, (c) Some members of Parliament are the
Mizoram and Arunachal Pradesh. Before 2013, members of Zila Parishad
Guwahati has also jurisdiction over Manipur. (d) All of the above
38th BPSC (Pre) Exam. 1992-93 6 YCT
Ans. (d) : The Panchayati Raj Institutions got the Ans. (d) : Physical change is defined as a change in
constitutional status after 73rd Amendment Act, 1992. matter which does not alter the chemical properties of
The primary objective of establishing the PRIs are: the matter. The examples of physical changes are:-
Promote democratic representation through three tier Dissolution of sugar into water, Burning of coal,
government. Melting of ice cube, Boiling of water, etc.
To provide social and economic justice to village. 44. There is no reaction when steam passes over?
Effective and efficient planning. (a) Aluminum (b) Copper
(c) Carbon (d) Iron
38. Fiscal deficit for 1992-93 is about ?
(a) 500 crore (b) 1000 crore Ans. (b) : There is no reaction when steam passes over
copper because it is very low in the reactivity scale.
(c) 5000 crore (d) 2000 crore
Metals like copper, silver, lead and gold do not react
Ans. (c) : The Fiscal deficit represents borrowing by the with steam or water. Aluminum reacts with steam to
central government. In 1992-93, fiscal deficit was about form a metal oxide and hydrogen.
5000 cores, that is 8-9% of the country's is GDP. Fiscal Example :
Deficit for the year 2021-22 was 1591089 Crore and Iron (Fe) and Zinc (Zn) also react with steam.
Fiscal Deficit of 2022-23 is estimated to be 1661196 45. Pasteurization is a process, in which ?
Crore. (a) Milk is kept at very low temperature for 24
39. Which of the following is not correct about five hour.
year plan period ? (b) Milk is boiled for 8 hours
(a) First - 1951 to 1956 (c) Milk is boiled for a certain period and then
(b) Second - 1956 to 1961 suddenly cooled within a certain period of
time
(c) Third - 1961 to 1966
(d) None of the above
(d) Fourth - 1966 to 1971
Ans. (c) : Pasteurised milk can be consumed without
Ans. (d) : The period of fourth five year plan was from boiling as it is free from harmful microbes. The milk is
1969 to 1974. From 1966 to 1969 Government heated to about 70ºC for 15 to 30 seconds and then
announced the plan of holiday, it was due to Indo-Pak suddenly chilled below 36ºC so that, it prevents the
war and failure of third five year plan. growth of microbes. This process was discovered by
Hence, option (d) is not correct. Louis Pasteur. It is called pasteurisation.
40. The Primary main source of revenue for Indian 46. 'Amphibians' refer to –
Government is ? (a) A fast boat
(a) Direct Tax (b) Indirect Tax (b) Animals that lives in water alone
(c) Deficit Financing (d) Loans from RBI (c) Animals that lives on land alone
Ans. (b) : The main sources of revenue of the (d) Animal that can lives both in water and land
government of India are excise and custom taxes, which Ans. (d) : The animals who can survive on both land
come under indirect taxes. and water called Amphibians. The Amphibians are
41. How long does the Sun's light take to reach the generally cold blooded. When they live on land they
earth ?
breathe with lungs and when they lives in water, they
(a) 8 second (b) 1 minute
breathe with gills. The example of Amphibian animals
(c) 8 minute (d) 24 minute
are : Frog, Toad, Salamanders, etc.
Ans. (c) : On an average sun light takes about 8 minutes
and 20 second to travel from the sun to the earth. 47. Which of the following statements is incorrect
42. Stainless steel is an alloy of ? about Malaria ?
(a) Iron and copper (b) Iron and Zinc (a) It is a disease that is caused by parasitic
(c) Iron and Chromium (d) Iron and Graphite insects
Ans. (c) : The Stainless steel is an alloy of Iron, (b) It is spread by mosquitoes
Chromium and Nickel. Chromium plays an important (c) It mostly spreads in regions of swamp
role for making stainless steel because it provides (d) It is cured by chloroquine
hardness and toughness to the stainless steel. The Ans. (a) : Malaria is caused by Plasmodium parasites
composition of chromium in stainless steel is around (protozoan micro organism not insect). It is a mosquito
10%. borne blood disease. The parasites spread through the
43. An Example of Physical change is ? bites of infected female Anopheles mosquitoes.
(a) Blackening of silverware is Air Plasmodium vivax and plasmodium falciparum are the
(b) Burning of Candle major parasites that causes Malaria.
(c) Conversion of milk into Curd 25th April is observed as world Malaria day every year.
(d) Dissolution of Sugar into Water Hence, option (a) is incorrect and rest of all are correct.

38th BPSC (Pre) Exam. 1992-93 7 YCT


48. Who is the father of Indian Nuclear Research Ans. (b) : According to the 1991 Census report, Bihar
Programme ? was the second populated state of India after Uttar
(a) S.A. Bose (b) H.J. Bhabha Pradesh. As per 2011 census report, the total population
(c) S. Chandrasekhar (d) S.S. Bhatnager of Bihar was 104, 099, 452 which is 8.60% of the total
Ans. (b) : Dr. Homi Jahangir Bhabha is considered as a population of India and currently it is the third state in
father of Indian Nuclear Research. Dr. H.J. Bhabha India after Uttar Pradesh and Maharashtra in terms of
established the Atomic Energy Establishment Trombay
population.
(AEET) in 1954 which was later renamed as Bhabha
Atomic Research Centre (BARC). 54. Boutros Ghali, the new secretary general of
49. The Credit for Research and Development on united nation belong to?
Space Science goes to - (a) Cuba (b) Zimbabwe
(a) S.S. Bhatnagar (b) V. Sarabhai (c) Nigeria (d) Egypt
(c) S.Z. Qasim (d) C.V. Raman Ans. (d) : Boutras Ghali, the former General Secretary
Ans. (b) : The Credit for Research and Development in of United Nations General Assembly, belong to Egypt.
space science goes to Vikram Sarabhai, V. Sarabhai He was the 6th Secretary General of the United Nations
was the founder of ISRO on 15 August, 1969. The from 1992-96. Antonio Guterres is present Secretary
headquarter of ISRO is in Bengaluru, Karnataka. The General of the United Nations
current chairman of ISRO is S. Somanath. 55. Which republic of former USSR was first to
50. Fast breeder reactor is a nuclear reactor for declare itself independent ?
producing electricity that produces electricity (a) Russia (b) Estoina
by ? (c) Lithuania (d) Ukraine
(a) By the process of fission Ans. (b) : Estonia became the first nation to declare
(b) By the help of solar cell itself independence from erstwhile USSR in 1988. It
(c) Use or reprocessed fissile material which is was snatched via a troop invasion by USSR in 1940.
use it 56. In the beginning of 1992 for what reason was
(d) Use of hard water as purifier the 'Davos' in Switzerland in the news ?
Ans. (c) : A fast breeder reactor is a nuclear reactor that (a) For hosting the World Economic Forum
generates electricity by trapping and regenerating the meeting
fuel materials used by it. (b) For hosting the finance Minister's meeting of
India’s first prototype fast breeder reactor is at the European Economic community
Kalpakkam. It is constructed by Bhartiya Nabhikiya (c) Protest for Winter Olympics
Vidyut Nigam Limited (BHAVINI). (d) For organising India Week
51. In which state Kala Azar disease spread ? Ans. (a) : Davos, an important city in Switzerland is
(a) West Bengal (b) Bihar famous for hosting World Economic Forum (WEF)
(c) Odissa (d) Uttar Pradesh meetings. World Economic Forum was founded in
January 1971 and its headquarter is in Geneva,
Ans. (b) : In 1992, Kala Azar disease spreaded in Bihar. Switzerland.
Visceral Leishmaniasis (VL), also known as Kala Azar
57. Where was the first direct dialogue between
(Black fevers) is a fatal disease. Leishmania parasites Israel and PLO held ?
are transmitted through the bites of infected female (a) Washington (b) Moscow
phlebotomine sandflies which feed on blood to produce (c) Madrid (d) Lisbon
eggs. Ans. (c) : The first direct dialogue between Israel and
52. The literacy rate in Bihar is about ? PLO (Palestine Liberation Organization) was held in the
(a) 30% (b) 34% Spanish capital ‘Madrid’ in 1991-92. Russia and
(c) 38% (d) 45% especially America had played a crucial role in these
Ans. (c) : As per the census report 1991, The total talks. As a result on September 13, 1993, historic
literacy rate of Bihar was about 38%, in which male agreement was singed between Palestine and Israel in
literacy rate was 51.37% and female literacy rate was Washington D.C. called as Oslo Accord.
21.99%. 58. Indira Gandhi prize for peace disarmament
As per 2011 census report, the total literacy rate in and development for the year 1990 was given
Bihar is 69.83%, where 70.32% males and 53.57% to?
females are literate. (a) Kurt Waldheim
53. The Population of Bihar in India is ? (b) Chancellor of Kohl
(a) Largest (b) Second largest (c) Sam Nujoma
(c) Third largest (d) Fourth largest (d) Nelson Mandela
38th BPSC (Pre) Exam. 1992-93 8 YCT
Ans. (c) : Sam Nujoma had been awarded Indira Ans. (a) : Ustad Zahiruddin Dagar was an eminent
Gandhi Peace Award for Peace Disarmament and Dhrupad Singer. He was in the news for winning
Development (IGPDD) in 1990. Sam Nujoma is former Sangeet Natak Academi Award. Sangeet Natak
and first President of Namibia, from 1990 to 2005. He Academy Award is awarded for performing in arts like
was also awarded with Lenin Peace Prize and honoured music, dance and drama. This award was first awarded
with "Founding Father of Namibian Nation. Indira in 1952.
Gandhi Peace Prize for the year 2021 was given to 64. 1991 has been declared as :
NGO Pratham. (a) Environment protection year
(b) Tour of India year
59. Famous film director, Satyajit Ray has (c) Girl Child year
recently received which award ? (d) Employment for all year
(a) Dada Sahab Phalke Award Ans. (b) : The year 1991 was declared as the ‘Tour of
(b) Best Indian Film Award India’ year. It was the cricket series tour from South
(c) Honorary Oscar for lifetime achievement Africa to India.
(d) Padam Vibhusan 65. Who was given the Bharat Ratna award in the
Ans. (c) : Satyajit Ray, was awarded Honorary Oscar award declared on republic Day in 1992 ?
for lifetime achievement award at the 64th Academy (a) Only J.R.D. Rata
Award, 1992. Satyajit Ray was a renowned Indian Film (b) Only Rajiv Gandhi
Maker. He was also awarded Bharat Ratna in 1992. (c) Only Subhas Chandra Bose
60. In the beginning of 1992, JKLF was in news (d) Both J.R.D. Tata and Subhas Chandra Bose
for – Ans. (a) : In 1992, Bharat Ratna which is India’s
(a) Abduction of an Indian politician highest civilian honour, was given to JRD Tata, Satyajit
(b) Planting a bomb in Indian Embassy Ray, Maulan Abul Kalam Azad and Subhas Chandra
(c) Demonstration in Islamabad Bose (Posthumdusly) but due to opposition by some
(d) Making an attempt to cross LOC and enter members of Neta ji Subhash C. Bose family and refusal
PoK to accept the fact of his death. In 1997 as per Supreme
Ans. (d) : In 1992, JKLF (Jammu Kashmir Liberation Court verdict Bose’s award was cancelled. So
Front) was in news for making an attempt to cross LOC government of India only announced the award but not
and enter PoK. JKLF was a political Organization. conferred on him. Hence, only option (a) is correct
61. Where did the recent massacre is Bihar in answer.
which about 40 people were killed took place ? 66. Who is the Chairman of University Grant
(a) Near Gaya (b) Near Patna Commission (UGC) ?
(c) Near Ranchi (d) Near Bhagalpur (a) Prof. Yashpal (b) Prof. Rajni Kothari
(c) Prof. K.N. Khanna (d) None of the above
Ans. (d) : In 1991, extremists killed 40 people in a
village near Bhagalpur (Bihar). Such massacres have Ans. (d) : In the year 1992, the chairman of UGC
(University Grant Commission) was Prof. G. Ram
happened many times in Bihar due to land dispute.
Reddy. His tenure lasted from 1991 to 1995. Prof.
62. In the recent Punjab Assembly elections how Mamidala Jagadesh Kumar is current chairman of UGC
many seats did Bhartiya Janta Party win ? from February 2022 till now.
(a) 10 (b) 8 67. Dada Sahab Phalke Award in 1990 was given
(c) 6 (d) 3 to ?
Ans. (c) : In the 1992 Punjab Assembly elections, (a) Raj Kapoor (b) Satyajit Ray
(c) Ashok Kumar (d) A. Nageshwar Rao
Indian National Congress (INC) won 87 seats and
Ans. (d) : A. Nageswara Rao was an Indian actor and
formed the government. The Bhartiya Janta Party (BJP) producer. He was known for his work predominantly in
won 6 seats. There are 117 seats in Punjab legislative Telugu cinema. He was conferred with ‘Dada Sahab
assembly. Phalke’ Award in 1990. Actor Rajnikanth recently got
63. Ustad Zahiruddin Dagar was recently in the award as 51st person.
news for ? 68. The Noble Prize in chemistry for the year 1992
(a) For Winning Sangeet Natak Academi award was given to ?
(b) For Winning Indira Priyadarshini award (a) Erwin Neher
(c) For his performance in UNO (b) Bert Sakmann
(c) Richard R. Ernst
(d) For his performance in Republic Day
(d) Pierre Gilles de Gennes
celebration
38th BPSC (Pre) Exam. 1992-93 9 YCT
Ans. (c) : The Nobel prize in Chemistry for the year 73. The religious congregation founded by Mother
1991 had been awarded to Swiss Chemist Richard R. Teresa is known as ?
Ernst. for his contributions to the development of the (a) Sisters of charity
methodology of High resolution Nuclear Magnetic (b) Missionaries of charity
Resonance (NMR) spectroscopy. (c) Organization of love
(d) Charity for all
69. In the 1992-93 Railway Budget, the estimated
increase in the surplus revenue was Ans. (b) : ‘Missionaries of Charity’ was a religious
(a) 130 crore (b) 2090 crore congregation founded by Mother Teresa. It was
established in 1950 in Calcutta.
(c) 13000 crore (d) 20000 crore
74. Which of the following statement is true ?
Ans. (b) : The increase in additional revenue was
(a) The United States of America has federal
estimated at about 2090 crore in the railway budget of
form of Government
1992-93.
(b) India has both federal and unitary form of
70. The 1991 Davis Cup was won by ? Government
(a) Germany (b) France (c) France has a federal form of Government
(c) Sweden (d) USA (d) The prime Minister of Pakistan is appointed
Ans. (b) : Davis cup is an international Men’s tennis by its people
tournament which is organised by International Tennis Ans. (b) : According to the Article 1 of the Indian
Federation and is held every year. The 1991 Davis Cup Constitution, " India, that is Bharat, shall be a Union of
was won by the France. States". It is considered that India has a quasi-federal
The 2021 Davis Cup had been won by the Russian and a unitary system. USA has Presidential form of
Tennis federation. government.
71. Which of the following statement is correct 75. What is the significance of panchsheel ?
about the recent cricket series between India (a) Non Alignment
and Australia ? (b) Economic co-operation and development
(a) Ravi Shastri scored a century in the fifth test (c) Development of cultural relations
match (d) Friendship and peaceful coexistence among
(b) Ravi Shastri scored double century in third countries
match Ans. (d) : The Panchsheel agreement is also known as
(c) Kapil Dev scored a century in the last test the five principles of peaceful co-existence. These five
match significant principles of Panchsheel are –
Mutual respect for each other’s territorial integrity and
(d) Vengsarkar scored a century in the last test
sovereignty.
match
Mutual non-aggregation in each other's internal
Ans. (a) : Ravi Shastri had scored a century in the fifth affairs.
test of the cricket series played between India & Mutual non-interference.
Australia in 1992. Hence, according to the given Equality and co-operation for mutual benefits.
statements option (a) is the right answer. Peaceful coexistence.
72. What was the purpose of establishing SAARC It was launched as an agreement between India and
? China is 1954.
(a) Non-interference in one another issues 76. Bachendri Pal is a famous name from which of
(b) Military agreement the following field ?
(c) Scientific and cultural exchange (a) Sports (b) Music
(d) Regional Co-operation (c) Dance (d) Mountaineering
Ans. (d) : The South Asian Association Regional Ans. (d) : Bachhendri Pal is an Indian Mountaineer, she
Cooperation (SAARC) was established on 8 December, became the first Indian women who reached the peak of
1985 in Dhaka. There are 8 member countries of Mount Everest in 1984.
SAARC, that is India, Sri Lanka, Nepal, Bhutan, 77. Who gave the Slogan of 'Jai Jawan Jai Kisan' ?
Bangladesh, Afghanistan, Pakistan and Maldives. The (a) Lal Bahadur Shastri
purpose of establishment of SAARC, to regional co- (b) Subhas Chandra Bose
operation and promote the welfare of the people of (c) Devi Lal
South Asia and to improve their quality of life. (d) Morarji Desai

38th BPSC (Pre) Exam. 1992-93 10 YCT


Ans. (a) : At the public gathering at Delhi in 1965, the (c) Sarojani Naidu (d) Nayantara Sahagal
second Prime Minister of India, Mr. Lal Bahadur Ans. (b) : Abul Kalam Azad has written “India wins
Shastri gave the slogan ‘Jai Jawan, Jai Kisan’ to boost freedom”. It is his autobiography which contains his
the moral of Indian armed forces and farmers. It was personal experiences when India became independent.
given during the battle with Pakistan in 1965. Due to Abul Kalam Azad was a senior leader of INC and he
scarcity of food grains in the country. became the first Minister of Education in the Indian
78. Which of the famous Indian is known as government, in the Honour of his birth anniversary 11
'Gurudev' ? November is observed as National Education day of
(a) Rabindra Nath Tagore India.
(b) C. Rajgopalachari 84. The languages spoken in Switzerland are ?
(c) Lala Lajpat Rai (a) Only German
(d) B.R. Ambedkar (b) Only French
Ans. (a) : The famous Indian poet, social reformer, (c) Both German and French
nationalist and educationist Rabindra Nath Tagore was (d) German, French and Italian
popularly known as ‘Gurudev’. Mahatma Gandhi gave Ans. (d) : Switzerland is a landlocked European
him the title ‘Gurudev’. He became Asia’s first Nobel country. It is surrounded by France, Germany. Italy,
Laureate when he won the Nobel Prize in Literature for Austria and Liechtenstein, so the Switzerland has four
his poem ‘Gitanjali’(1913). official languages, French (23%), German (60%),
79. Who was the Prime Minister of Britain at the Italian (8%) and Romanish (0.5%).
time of second world war ? 85. The official language of UNESCO is ?
(a) Clement Attlee (b) Anthony Eden (a) Only French
(c) Winston Churchill (d) Herald Wilson (b) Only English
Ans. (c) : At the time of World War-II (1939). Neville (c) Both English and French
Chamberlain was the Prime Minister of Britain. After (d) Only English French and Russian
the resignation of Chamberlain. Winston Churchill Ans. (c) : The United Nations Educational, Scientific
became the Prime Minister of Britain in 1940. He was and Cultural Organization (UNESCO) is a specialized
the first to raise the voice against European agency of the United Nations. It was formed on 16
appeasement of Germany during Hitler reign. Hence the November 1945, the headquater of UNESCO is in
right answer from the given options will be option (c). Paris. UNESCO have six official languages, Arabic,
80. 'Operation Blackboard' is a Governmental Chinese, English, French, Russian and Spanish.
scheme for ? 86. The Author of the book 'Life Divine' is ?
(a) Stopping the terrorism in Punjab (a) S. Radha Krishnan
(b) Improving the national programme of school (b) Mulkraj Anand
education (c) Aurobindo Ghosh
(c) Improving the school education of girls (d) Swami Vivekanand
(d) Encouraging the adults to become literate Ans. (c) : The author of the book ‘Life Divine’ is Sri
Ans. (b) : After the New Education Policy (1986). The Aurobindo Gosh, published in 1939. It is based on his
Operation Black Board’ was launched in 1987. It is principal philosophical work which is a theory of
central government’s program. Operation Black Board, spiritual evolution. This book is considered as a “advent
provided compulsory and free education to all children of a divine life on the earth”. Sri Aurobindo Ghosh has
up to 14 years of age in the country. written some other books, as; Essays on the Gita.
81. 'Chipko Movement' is ? Rebirth and Karma, etc.
(a) A movement to stop untouchability 87. Which of the following pair is incorrect ?
(b) Movement to save trees (a) Bheemsen Joshi - Classical vocal music
(c) A federation for milk production (b) Allahrakha - Tabla
(d) None of the above (c) Debu Chaudhari - Sitar
Ans. (b) : Chipko movement or Chipko Andolan is also (d) M.S. Subbulakshmi - Flute
known as ‘Hug the tree’. It was a non-violent Ans. (d) : M.S. Subbulakshmi was an Indian carnatic
movement started in 1970s. Reni village in singer from Madurai, Tamil Nadu. She has been
Uttarakhand's Chamoli district, the birthplace of Chipko awarded Bharat Ratna in 1988.
movement. Chipko movement was a movement against Hence, option (d) is not correctly matched and rest of
cutting the tree and forests. Gaura Devi and Sundar Lal all are correctly matched.
Bahuguna was the famous leader of Chipko movement. 88. Which of the following journalists has won the
82. "Autobiography of an Unknown Indian" is Magsaysay award ?
written by ? (a) Khushwant Singh (b) Arun Shourie
(a) Nirad C. Chaudhuri (c) Dharamveer bharti (d) Kamleshwar
(b) Khushawant Singh Ans. (b) : Arun Shourie has won the Magsaysay award
(c) Kamala Das in 1982. He is a renowned journalist and economist who
(d) Balwant Gargi have also been an advisor/consultant to Planning
Ans. (a) : The Autobiography of “Unknown Indian” has Commission and member of Rajya Sabha.
been written by, Nirad D. Chaudhury. It was published In 2021, Magsaysay award has been awarded to
in 1951. Firdausi Qadri (Bangladesh), Muhammad Amjad Saqib
83. The writer of India wins freedom is ? (Pakistan), Steven Muncy (American), Watch Doc
(a) Jawahar Lal Nehru (b) Abul Kalam Azad (Indonesia) and Roberto Ballon (Philippines) .

38th BPSC (Pre) Exam. 1992-93 11 YCT


89. Various states and their popular dances are 94. In which field is scientist Ivan Pavlov known
given : for his work ?
A. Andhra Pradesh i. Bihu (a) In physics
B. Assam ii. Burra (b) In mathematics
C. Himachal Pradesh iii. Ghoomar
D. Rajasthan iv. Nati (c) In chemistry
Match the states with its dances correctly. (d) In clinical psychology
A B C D Ans. (d) : Ivan Pavlov was a Russian physiologist. He
(a) i ii iii iv is famous for his work in clinical psychology. He was
(b) ii i iii iv awarded Nobel prize in physiology or medicine in 1904
(c) ii i iv iii for his work on the physiology of digestion.
(d) iv i iii ii 95. Limba Ram is famous for which sport ?
Ans. (c) : States with their popular dance form are: (a) Archery (b) Swimming
State – Popular Dance (c) Table Tennis (d) Lawn Tennis
Andhra Pradesh – Burra Ans. (a) : Limba Ram is an Indian Archer. He won the
Assam – Bihu gold medal for Indian in Archery at Asian Cup (1989),
Himachal Pradesh – Nati Asian Archery Championship (1992) and Common
Rajasthan – Ghoomar \Wealth Archery Championship (1955). He was
90. Two American citizens born in India who have awarded the Padma Shri Award in 2012.
won Nobel Prize in the field of science are : 96. Thomas Cup prize is given for which sport ?
(a) Hargovind Khurana and C.V. Raman (a) Tennis (b) Football
(b) S. Chandra Sekhar and C.V. Raman (c) Golf (d) Badminton
(c) C.V. Raman and S.N. Bose Ans. (d) : Thomas Cup trophy signifies the world
(d) Hargovind Khurana and S.Chandra Sekhar supremacy in the sport of Badminton. It is also called as
Ans. (d) : Dr. Hargovind Khurana and S. Chadrasekhar World's Men's Team Championship. It is an
are the two American citizens who are born in India and international badminton competition among the teams
won the Nobel prize in the field of science. representing member nations of the Badminton World
The other Indian who won the Nobel prize : Federation (WBF). The first tournament of Thomas Cup
Rabindranath Tagore – Literature – 1913 was held in 1948-49.
C.V. Raman – Physics – 1930 97. The first Asian Games were held at ?
Mother Teresa – Peace – 1979 (a) Peking (b) Tokyo
Amartya Sen – Economics – 1998 (c) New Delhi (d) Colombo
Venkatraman – Chemistry 2009 Ans. (c) : The first Asian Games were held in New
Ramakrishnan Delhi, in 1951. The Asian games are held every four
Kailash Satyarthi – Peace – 2014 year. In that game Japan topped the medal table, total-
Abhijit Banerjee – Economics 2019 24 gold, 21 silver, and 15 bronze medals.
91. 'Kalinga Prize' is given for ? 98. The largest Stadium of the world is situated in
(a) The best film of the year ?
(b) For the popularization of science (a) Brazil (b) Czechoslovakia
(c) Development of cultural understanding (c) Germany (d) USA
(d) Extraordinary performance in sport Ans. (b) : The worlds’ largest stadium is in the Prague
Ans. (b) : The Kalinga prize is given for popularization (Czechoslovakia), known as Stockholm stadium. The
of science. It is awarded annually by Kalinga world’s largest indoor stadium is in the USA. (Michigan
Foundation Trust, Odisha. Stadium). Presently Rungrado Stadium (North Korea) is
the largest stadium in the world. At present Largest
92. The weight of an adult elephant is ? stadium of the world is Narendra Modi Stadium
(a) 500 kg (b) 1000 kg (Ahmadabad Gujarat) in India
(c) 5000 kg (d) 10000 kg 99. Prakash Padukone is famous for which sports
Ans. (c) : The elephant is considered to be the largest ?
living mammal on earth. The weight of an adult (a) Badminton (b) Hockey
elephant is around 5000 kg. (c) Football (d) Tennis
93. Scientist Albert Einstein is famous for ? Ans. (a) : Prakash Padukone is famous for Badminton.
(a) Explaining the atomic structure of Hydrogen He ranked number one in the world’s Badminton trophy
(b) For the photoelectric effect in 1980 and also won the England Open Badminton
(c) For constructing first nuclear reactor Championship in 1980.
(d) Predicting the existence of neutrons 100. Durand Cup is associated with which sport ?
Ans. (b) : Albert Einstein was a German physicist. He (a) Cricket (b) Football
is famous for the invention of the photoelectric effect, (c) Hockey (d) Table Tennis
his other inventions and contributions are – Theory of Ans. (b) : Durand Cup is the world’s third oldest
Relativity, Quantum theory of light, Avogadro’s football tournament. It is organized by Durand Football
Number and Brownian movement. He won the Nobel Tournament Society and founded in 1888. FC Goa is
prize in physics for the ‘photoelectric effect’ in 1921. the champion of Durand cup 2021.

38th BPSC (Pre) Exam. 1992-93 12 YCT


39th Bihar Public Service Commission
Preliminary Examination, 1994
GENERAL KNOWLEDGE & GENERAL SCIENCE
(Solved Paper with Detail Explanation)
1. From which book is the 'Gayatri Mantra' Ans. (c) : The inscription which proves Chandragupta’s
taken? dominance over western India is The Junagadh rock
(a) Upanishad (b) Bhagavad Geeta inscription.
(c) Rigveda (d) Yajurveda The Junagadh rock inscription of Rudradaman, also
Ans. (c) : The 'Gayatri Mantra' has taken from known as the Girnar Rock inscription of Rudradaman,
'Rigveda', it is also known as Savitri Mantra. Gayatri is a Sanskrit prose inscribed on a rock by the Indian
Mantra has been composed by Maharshi Vishwamitra. Satraps ruler Rudradaman I. It is located near Girnar hill
The Mantra is given in the third mandala of the Rigveda near Junagadh, Gujarat, India. The inscription is dated
which is dedicated to the goddess Savitri. to shortly after 150 CE.The Junagadh rock contains
2. In Sangam period, Uraiyur is famous for- inscriptions of Ashoka (fourteen of the Major Rock
(a) An important trading centre of spices Edicts of Ashoka), Rudradaman I and Skandagupta.
(b) An important trading centre of cotton
(c) An important trading centre of foreign trade 6. What was the aim of Barahiatal movement of
(d) An important trading centre of internal trade Monghyr?
(a) Demanding the return of their Bakasht land
Ans. (b) : In Sangam Period, Uraiyur is a region in
Tiruchirappalli in Tamil Nadu India. It was the capital (b) To stop exploitation of Muslim peasants
of the early cholas. Uraiyur was famous for spinning of (c) To end the zamindari system
silk and cotton clothes. There was great demand for (d) To start 'class-struggle'
these in the western world especially for the cotton Ans. (a) : The main purpose of Barhiatal movement of
clothes. munger was the demand for restoration of Bakasht
3. Who was the founder of Ajivika Cult? Land. This movement was led by Karyanand Sharma, in
(a) Upali (b) Anand 1935.
(c) Raghulabhadra (d) Makhali Gosala This movement was a collective effort by the explaited
Ans. (d) : It was founded by Goshala Maskariputra (a and oppressed farmers to take back their raiyat land
friend of Mahavira, the 24th Tirthankara of Jainism) converted in to Bakasht Land from the zamindars. The
and was contemporary of Jainism and Buddhism. term 'Bakasht' denoted that the farmers had given it to
Ajivika sect is based on the philosophy that the affairs the zamindars because not being able to pay the rent due
of the entire universe were ordered by a cosmic force to the recession.
called niyati that determined all events, including an 7. When did Bihar separate from Bengal?
individual’s fate.This school of thought gained (a) 1911 (b) 1912
prominence for some time during Mauryan rule. (c) 1913 (d) 1914
4. Which contribute maximum in development of Ans. (b) : Bihar separated from the Bengal Presidency
Bhagavat cult? on 22 March 1912 and comprised with Orissa and
(a) Parthian (b) Indo-Greek People become a single province.On 22 March 1912, it was
(c) Kushana (d) Gupta carved out, hence March 22 is celebrated as Foundation
Ans. (d) : Gupta's contribution is the largest in the Day of Bihar. Again in March 1936, Bihar and Odisha
development of Bhagwat sect. Rulers onwards from were made into separate provinces after the declaration
Chandragupta II, Vikramaditya were known as Parama of the Government Of India Act, 1935.
Bhagwatas, or Bhgavata Vaishnavas. The Bhagavata In 2000, Jharkhand was carved out of Bihar
Purana entails the fully developed tenets and philosophy 8. When did Patna High Court establish?
of the Bhagavata cult where Krishna gets fused with (a) 1916 (b) 1912
vasudeva and transcends vedic Vishnu and cosmic Hari (c) 1913 (d) 1921
to be turned into the object of Bhakti. Ans. (a) : Patna High Court is the High Court of Bihar
5. Which inscription proves that the dominance of State, Patna high court was established on February
Chandragupta was in Western India? 1916 and the later affiliated under the Government of
(a) Kalinga inscription India Act, 1915. Justice Edward Maynard Des C.
(b) Girnar inscription of Ashoka Chamier was the first chief justice of Patna High Court.
(c) Junagarh inscription of Rudradaman Hon'ble Pandit Lakshmi Kant Jha was the first Indian
(d) Sopara inscription of Ashoka Chief Justic of Patna High Court after independence.

39th BPSC (Pre) Exam. 1994 13 YCT


9. In which district did Barahi Congress succeed 14. Who wrote 'Barah Masa'?
in 1930 and 1931 to break the English (a) Amir Khusro
administration throughly? (b) Imami
(a) Madhubani (b) Bhagalpur (c) Malik Md. Jayasi
(c) Gaya (d) Monghyr (d) Raskhan
Ans. (c) : In 1930 to 1931 in Gaya district of Bihar the Ans. (c) : Barah masa is a part of the epic 'Padmavat'
Barahi Congress successfully made the British rule composed by Malik Muhammad Jayasi, It was sung by
functionless. Nagamati. Ratnasen's married wife.
10. In which inscription does Ashoka declare 15. What was the main features of fiscal system of
himself as a king? Vijaynagar Empire?
(a) Minor edict of Maski (a) Cess
(b) Rumendeyi Pillar (b) Land revenue
(c) Queen Pillar (c) Profit from ports
(d) Bhabru edicts (d) Currency system
Ans. (d) : In Bhabru edict, the Mauryan Emperor Ans. (b) : The main source of income of Vijayanagar
Ashoka refers himself as Priyadarshi king of Magadha. empire was land revenue and its administration was
In this edict, he states that his faith in the Buddha, the under a department called Athavane
Dhamma and the Sangha. Land was divided into four categories for purposes of
11. Who was appointed as the first 'Iqta' by assessment, wet land, dry land, orchards and woods.
Mohammad Ghori in India? Usually the share was one - sixth of the produce. Land
(a) Tajuddin Yaldauz revenue could be paid in cash or kind.
(b) Qutubuddin 16. What was the main cause of the victory of
(c) Shamsuddin Iltutmish Babur in the Battle of Panipat?
(d) Nasiruddin Qubac (a) His horse stirred up army
Ans. (b) : Mohammad Ghori was the first to introduce (b) his military leadership quality
the Iqta system in India in 1206 AD., Qutbuddin Aibak (c) Tulughma system
was assigned as the first iqta of India by Mohammed (d) Internal discord of Afghans
Ghori. It was lltutmish who gave it an institutional Ans. (c) : The main reason of the victory of Babur in
form. The Iqtadari system went through numerous the first battle of Panipat (April, 1526) was the new war
changes during the sultanate period. Originally, Iqta tactics introduced by Babur were the 'Tulughma and
was a productive land that was given instead of a salary. The Araba'.
However, it became hereditary during the reign of Firuz Tulughma meant dividing the whole army into various
Shah Tughlaq in 1351 AD. units, namely the left, the Right and the centre. Through
12. Under the reign of which Sultan was the large- this a small army could be used to surround the enemy
scale 'Khalisa Land' developed? from all the sides. The first Battle of Panipat, on 21
(a) Ghiyasuddin Tughlaq April, 1526 was fought between the invading forces of
(b) Alauddin Khilji Babur and the Lodi kingdom.
(c) Md. Bin Tughlaq 17. Who introduced 'Zabti system'?
(d) Ferozshah Tughlaq (a) Gayasuddin Tughlaq (b) Sikandar Lodi
Ans. (b) : During the Mughal and Sultanate period, (c) Shershah Suri (d) Akbar
there were two categories of land- Iqta, The state was Ans. (d) : Zabti system or Dahsala System, namely
parceled out into the tracts called Iqta under the (land revenue system) was introduced by Raja
supervision of Iqtadars to administrate properly. Khalsa Todarmal, who was the finance minister of Akbar. In
was a royal land under the direct supervision and this system, average produce of ten years was derived
control of the central government. It was managed from this method. One third of this average produce was
through Nobles and Amils to maintain the troops and fixed in Rupees per Bigha and fixed as share of the state
other expenses. (Mal). Rest two third share was left to the cultivators
13. Which metals were used in the coins-Tanka, (Kharaj). This remained a standard system of revenue
Shashgani and Jital-of Sultanate period? assessment during the greater part of the Mughal
(a) Silver, Copper Empire.
(b) Gold, Silver, Copper 18. 'Zawabit' was related to -
(c) Silver, Zinc, Copper (a) The law of state
(b) The law which controls the Mansab system
(d) Gold, Zinc, Copper
(c) The law related to Taksal
Ans. (a) : Iltutmish introduced silver tanka and copper (d) The tax related to agriculture
jital, the two coins of the Delhi Sultanate. The coins Ans. (a) : The Sultans of Delhi enforced the Shariah
prior to Iltutmish were introduced by the invaders which and also appointed the Ulema to profitable officers, they
bear the Sanskrit characters and even bull and also formulated their own state laws called Zawabit to
shivalinga. Iltutmish was the first to introduce a "Pure bring in a practically in solving disputes involving the
Arabic coin" in India. people.

39th BPSC (Pre) Exam. 1994 14 YCT


19. What was the main cause of struggle between Ans. (a) : Guptas have given the largest grant of
the Shah of Iran and the Mughal emperors? villages to the temples and Brahamans. Land grants
(a) Kabul (b) Qandhar given to religious institutions were called Brahmadeya,
(c) Kunduz (d) Ghazani that denoted to Brahmins Devadana denoted to Gods
Ans. (b) : The region of Qandahar was the main cause and Agrahara denoted to settlement of priests.
of conflict between the Safavi dynasty(Shah of Iran) of 25. In which period is the concept of untouchables
Iran and the Mughal emperors of India. Babur had vividly clear?
captured it in 1522 AD and for this reason the later (a) Rigvedic Period
Mughals considered it their family territory. (b) Post-Vedic Period
20. Who had started 'Do-aspa', 'Sih-aspa' (c) Post-Gupta period
tradition? (d) Dharmshashtra Period
(a) Akbar (b) Jahanhgir Ans. (b) : The caste system was found in the post vedic
(c) Shahjahan (d) Aurangzeb period believed to be developed between 1000-600 BC,
Ans. (b) : Du-aspa and Sih-aspa: These were where it was called the varna system. It classified the
features added later to the Mansabdari system by society into the four varnas:
Jahangir. This is a system whereby the selected The Brahmins : Priest, scholars and teachers.
nobles could be allowed to maintain a larger quota The Kshatriyas : Rulers, worriors and administrators.
of troopers, without raising their Zat rank. The Vaishyas : Cattle herders, agriculturists and
⦁ Du-Aspa — (a trooper with two horses) merchants.
⦁ Sin-Aspa— (a trooper with three horses) The sudras : Labourers and service providers.
This, concept of untouchability was vividly clear during
21. Saranjami System was related to -
(a) Maratha land-revenue system post-vedic period.
(b) Taluqdari system 26. Who was associated with Ramosi Peasant
(c) Qutubshahi Administration Movement?
(d) None of these (a) Justice Ranade
Ans. (a) : A Saranjami is grant of land for maintenance (b) Gopal Krishna Gokhale
of troops or for military service found in Maratha (c) Vasudev Balvant Phadke
Empire. Rajaram Bhonsle adopted the Saranjami system (d) Jyotiba Phule
as a political measure to ensure the loyalty of key Ans. (c) : Vasudev Balwant Phadke was associated with
persons to the side of the Maratha Empire. Later under the Ramosi Peasant Movement. Ramosi Uprising was a
the Peshwa the system become hereditary. peasant led revolt against the British.Failure to solve up
anti-famine measures. It was struck in a violent manner,
22. Where was the first factory founded in 1612 by
not in a peaceful manner. Phadke was moved by the
the English?
(a) Goa (b) Hooghly plight of the farmer community during British raj.
(c) Arcot (d) Surat 27. Who had withdrawn Vernacular Press Act of
Ans. (d) : The British established their first Indian 1878?
factory (trading post) at Surat in 1613 A.D. (a) Lord Ripon (b) Lord Lyton
(c) Lord Curzon (d) Lord Minto
⦁ The French set up their first factory at Surat in 1668
Ans. (a) : Lord Lyton passed the Vernacular Press Act
AD.
1878 that authorized the government to confiscate
⦁ Portuguese established their first factory in India at newspapers that printed seditious material. Lord Ripon
Calicut in 1500 AD. succeeded Lord Lytton and abolished the Act in 1881.
⦁ The Dutch established their first factory in 28. Who published journal 'Free Hindustan' in
Masulipattanam in 1605 AD. America?
23. Which of the following sources is silent about (a) Ramnath Puri (b) G.D. Kumar
the trading route of ancient India? (c) Lala Hardyal (d) Taraknath Das
(a) Sangam Literature Ans. (d) : The Free Hindustan newspaper was
(b) Milinda Panho published in April 1908 by Taraknath Das. The Free
(c) Jatak Stories Hindustan, described as "An Organ of freedom and of
(d) All of these political, social and religious reforms", published from
Vancouver.
Ans. (b) : The oldest manuscript of the pali text was
He also established the Indian Independence League in
copied in 1495. Based on references within the text
California and also helped Lala hardayal in organizing
itself, significant section of the text are lost, making the Ghadar Party Movement in the USA.
Milind Panno the only Pali text known to have been
29. Main cause of the beginning of 'Ghadar
passed down as incomplete. Movement' was -
24. Which dynasty contributes maximum in village (a) Arrest of Lala Hardyal
gift to temples and the Brahmans? (b) Kamagatamaru episode
(a) Gupta (b) Pala (c) First World War
(c) Rasthrakuta (d) Pratiha (d) Hanging of Kartar Singh Sarabha
39th BPSC (Pre) Exam. 1994 15 YCT
Ans. (b) : A ship named Komagatamaru, filled with change its attitude towards Turkey. Although Mahatma
Indian immigrants was turned back from Canada. As Gandhi was in favour of launching Satyagraha and non-
the ship returned to India several of it's passengers were cooperation against the Government on the Khilafat
killed or arrested in a clash with the British Police. This issue. In June 1920, an all-party conference met at
incident made the Ghadar Party proclaim war and Allahabad and approved a program of boycott of
inspired thousands of Indian immigrants to come back schools, colleges, and law courts.
and organize an armed rebellion against British The Khilafat Committee launched a Non-Cooperation
imperialism. However, this movement was crushed. It Movement on 31 August 1920.
was the main reason of beginning of Ghadar Movement. 34. What was the aim of Eka movement of Avadh?
30. In which session did Home Rule supporters (a) No tax to the government
succeed in their political power? (b) To protect the rights of zamindars
(a) Lucknow Session of Congress, 1916 (c) To end the Satyagraha
(b) A.I.T.U.C. Session of Congress, 1920 (d) To pay tax regular in Rabi and Kharif to the
(c) First A.U.P. Kisan Sabha of 1918 government
(d) Joint A.I.T.U.C. and N.T.U. conference of Ans. (a) : The Eka Movement was led under the
Nagpur in 1938 leadership of Madari Pasi and other low-caste leaders
Ans. (a) : The Home Rule supporters succeed in their and many small zamindars. Towards end of 1921,
power in Lucknow session 1916 of Indian National peasant discontent resurfaced in some northern districts
congress. of united provinces-Hardoi, Bahraich and Sitapur. The
⦁ There were two Home Rule leagues launched : issue involved were :
⦁ Tilak launched the Indian Home Rule league in April (i) High tax-50% higher than the recorded rates,
1916 at Belgaum. (ii) Oppression of Thikedars in charge of revenue
collection.
⦁ Annie Besant launched the Home Rule League in (iii) The practice of share-rents.
September 1916 at Madras.
35. The nature of social boycott 'Nai Dhobi Band'
31. Who was the writer of 'Indian unrest'? in 1919 was
(a) Dadabhai Naoroji (b) Annei Besant (a) Started in Pratapgarh district by the peasants
(c) Lala Lajpat Rai (d) Valentine Chirol (b) Started by 'Sadhus' for uplifting the status of
Ans. (d) : The Indian Unrest was written by Sir Ignatius lower castes
Valentine Chirol was a british journalist, prolific author, (c) A step taken by zamindars of villages to stop
historian and diplomat. He was a passionate imperialist exploitation of lower castes by thikedars
and believed that imperial Germany and Muslim unrest (d) A movement by the lower castes against
were the biggest threats to the British Empire. thikedars
Valentine Chirol gave the title, "The father of the Indian Ans. (a) : 'Nai-Dhobi Band' was a social boycott in
Unrest" to Bal Gangadhar Tilak. 1919 started by farmers in Pratapgarh district, Bihar.
32. Who participated in 1917-18 Ahmedabad 36. Whose brainchild was I.N.A. and who founded
Satyagraha led by Gandhiji? it?
(a) Peasant Class (a) Subhash Chandra Bode
(b) Industrial Workers (b) Capt. Mohan Singh
(c) People (Mass) (c) Chandra Shekhar azad
(d) Labourers (d) Bhagat singh
Ans. (b) : Ahmedabad Mill Strike of 1918 is considered Ans. (b) : The INA was first formed in 1942 under
the first hunger strike led by Gandhi. In 1918, Mahatma Mohan Singh and Japanese Major Iwaichi Fujiwara and
Gandhi intervened in a dispute between Ahmedabad comprised of Indian prisoners of war of the British-
industrial workers and millowners desired to take the Indian Army captured by Japan in the Malayan
bonus. (present-day Malaysia) campaign and at Singapore.
Workers demanded a 50% wage increase but the mill Subhash Chandra Bose came to Singapore in July of
owners were only willing to offer a 20% wage increase. 1943 and took charge of the revived Indian National
Gandhi called on workers to go on strike and demand a Army also known as the Azad Hind Fauj.
35% rise in wages rather than a 50% increase. The
strike was called off. In the end, the tribunal awarded 37. Which Executive committee of congress
the workers a 35% wage increase. adopted a policy abolish the right of land-
ownership
33. In which town was 1920's Khilafat Committee (a) Executive Committee, 1937
Conference held, which requested Gandhiji to (b) Executive Committee, 1942
lead to Non-cooperation?
(c) Executive Committee, 1945
(a) Lucknow (b) Lahore
(c) Allahabad (d) Karachi (d) Executive Committee, 1946
Ans. (c) : In early 1919, the All India Khilafat Ans. (a) : The Congress Working Committee adopted
Committee was formed under the leadership of the Ali the policy of abolishing land ownership in 1937.
brothers, Maulana Abul Kalam Azad, Ajmal Khan and Tenancy (Amendment) Act, was passed by Congress
Hasrat Mohani, to force the British Government to Government in United Province and Bihar.

39th BPSC (Pre) Exam. 1994 16 YCT


38. When was the chotanagpur Tribal-Uprising Ans. (a) : The Boghazkoi inscription (14th Century
held? B.C.) which migrated from India to Asia Minor
(a) 1807-1808 (b) 1820 (Turkey) at that early age mentions four Vedic Gods.
(c) 1858-59 (d) 1889 Indra, Varuna, Mitra and Zasatyas, proves Central Asia
Ans. (b) : The Kol Revolt, the tribal inhabitants of as Aryans homeland.
Chotanagpur comprised kols, Bhils, Ho, Munda and 44. Planning Commission is a -
Oraons. They led an independent life in 1820 when the (a) Ministry
king of porhat owed allegiance to the British and agreed (b) Administrative department
to pay huge taxes annually. (c) Advisory body
39. Which of the following is related to (d) Autonomous corporation
Champaran Satyagraha of Gandhiji? Ans. (c) : Planning Commission was an extra-
(a) Zardari (b) Tinkathia constitutional and non-statutory body formed by the
(c) Zenmis (d) None of these resolution of the Government of India in March 1950. It
Ans. (b) : Champaran satyagraha was started by was an advisory body to the Government of India on an
Mahatma Gandhi in 1917 to look into the problem of array of issues of economic development. It was headed
the Tinkathia system in which peasants of champaran by the Prime Minister as the ex-officio Chairman. It was
were forced to grow Indigo on 3/20th of the total land. dissolved in 2014 and NITI Aayog replaced this body.
Raj Kumar Shukla persuaded Gandhiji to look into the 45. The Constitution of India was adopted by -
peasant's problem in champaran, Bihar. Champaran (a) Constituent Assembly
Satyagraha was Mahatma Gandhi's first experiment of (b) British Parliament
Satyagraha in India. (c) Governor-General
40. Which of the following was related ot Ulgulan (d) Parliament of India
movement? Ans. (a) : The Republic is governed in terms of the
(a) Santhal (b) Kachcha Naga Constitution of India which was adopted by the
(c) Kol (d) Birsa Munda Constituent Assembly on 26th November, 1949 and
came into force on 26th January, 1950. The Constitution
Ans. (d) : The Ulgulan movement was a revolt against provides for a Parliamentary form of government which
the British and zamindars in the chhotanagpur plateau is federal in structure with certain unitary features
region of India. The movement was led under the
46. India is a sovereign country, because preamble
leadership of Birsa Munda proclaimed a rebellion in
of constitution starts with-
1894. He declared himself as 'God' and gathered a large
(a) Democratic
number of supporters to revolt against the land system
(b) People Democracy
and beth-begari (bonded labour).
(c) Republic of People
41. Which of the following period is known as (d) We the people of India
Indus Civilization?
(a) Historical Period Ans. (d) : The Preamble proclaims that India is a
(b) Paleolithic Period Sovereign State. 'Sovereign' means that India has its
(c) Post-historical Period own independent authority and it is not a dominion or
(d) Pre-historical Period dependent state of any other external power . Preamble
Ans. (d) : Indus Valley Civilization (sometimes, the of constitution start with ' WE THE PEOPLE OF
Harappan Civilization). This civilization existed from INDIA'.
about 3000-2500 BC to about 1500 BC, (Pre-Historic 47. Any money bill can be introduced in-
Period) which means it existed at about the same time (a) Only in Rajya Sabha
as the Egyptian and Sumerian Civilizations. (b) Only in Lok Sabha
42. The period 800 to 600 B.C. is known as - (c) In both Houses
(a) Brahmana Period (b) Sutra period (d) Joint sitting of both Houses
(c) Ramapna Period (d) Mahabharata Period Ans. (b) : Article 110 of the Constitution deals with the
Ans. (a) : The period of 800 BC to 600 BC is known Money Bills.
for-Brahman's Era- Provisions for Money Bill in India
Sutra Period - 600 to 200 BC 1. President’s recommendation is needed to introduce
Mahabharat Period- 1200 to 800 BC them
43. The importance of 'Bogazkio known is for - 2. Speaker decides whether a bill is a Money Bill or not.
(a) This inscription deals with Vedic goddesses 3. They can be introduced only in Lok Sabha.
and gods 4. It cannot be amended or rejected by the Rajya Sabha.
(b) An important trading centre between Central 5. President can either accept or reject a money bill but
Asia and Tibet cannot return it for reconsideration.
(c) The original text of Veda is written here 6. No provision for joint sitting of both the houses to
(d) None of these resolve the deadlock

39th BPSC (Pre) Exam. 1994 17 YCT


48. Fundamental Rights of Indian Citizen are 53. The election of a speaker of the Lok Sabha by -
covered in - (a) All members of the parliament
(a) Articles 12 to 35 of constitution (b) Direct election by people
(b) Articles 13 to 36 of constitution (c) All members of Lok Sabha
(c) Articles 15 to 39 of constitution (d) Members of major parties of Lok sabha
(d) Articles 16 to 40 of constitution Ans. (c) : In the Lok Sabha, the Speaker and the Deputy
Ans. (a) : In Part II of Indian constitution Article 12 to Speaker both are elected from all its members by a
35 deals with fundamental Rights. simple majority of members present and voting in the
The mainly six fundamental Rights: house. There are no specific qualifications are
1. Right to Equality -Article 14 to 18 prescribed for elected as the speaker of the house.
2. Right to Freedom- Article 19 to 22 54. The president of India can be removed by -
3. Right against exploitation- Article 23 and Article 24 (a) The Prime Minister of India
4. Right to freedom of Religion- Article 25 to Article 28 (b) Lok Sabha
5. Cultural and Educational Right-Article 29 to 30 (c) Chief justice of India
6. Right to constitutional Remedies-Article 32 (d) Parliament
49. Panchayati Raj is a subject on Ans. (d) : Correct option is (d) President of India
(a) Concurrent List removed by Parliament.
(b) Centre (union) List The President can be removed from office by the
(c) State List process of impeachment only on the grounds of
(d) List of Residuary Power violation of the Constitution. The impeachment process
Ans. (c) : Panchayati is a subject under the State List. of the President of India is a quasi-judicial process.
The Panchayati Raj was constitutionalized through the Article 61 describes the process of impeachment of the
73rd Constitutional Amendment Act (1992) and added President of India. Then, the notice is sent to the
Part IX and added a new Schedule, Eleventh. The President of India and within 14 days the process of
provision of panchayat in Indian constitution is from impeachment starts. The resolution to impeach the
Article 243 to 243 (O). In the Eleventh schedule of the President must be passed by a special majority (two-
Constitution contains 29 subjects. thirds) from any house of the parliament by levelling
50. Which of the following articles of Indian charges against him.
Constitution directs states to organise Village 55. In India, any person who is not the member of
Panchayats? either House, can continue at the post of
(a) Article-32 (b) Article-40 minister till -
(c) Article-49 (d) Article-51 (a) 3 months (b) 6 months
Ans. (b) : Article 40 of the Indian Constitution brings (c) 1 year (d) No time limit
up one of the Directive Principles of State Policy which Ans. (b) : It is stated that if a person is not a sitting
lays down that the state shall undertake necessary steps member of either House of a parliament, he can be a
to organize village Panchayats. minister for only a period of 6 months. To save his
51. Judicial review means that the Supreme Court position he must become a member of either house of a
- parliament by means of election.
(a) Has a final right over all matters. 56. The maximun time-interval between two
(b) Can frame charges against the President sessions of Parliament -
(c) Can discuss the matters decided by the high (a) 4 months
Courts (b) 6 months
(d) Can void the law made by any state. (c) 1 year
Ans. (d) : Judicial review means that the Supreme (d) Time decided by the President
Court can declare any law of the state invalid. If the Ans. (b) : According to Article 85(1) of Indian
laws made by the state violate the constitutional Constitution, the maximum time-interval between two
provisions, then the Supreme Court can declare them sessions of parliament in India should not be more than
unconstitutional and void. This jurisdiction of the court 6 months. That means, the parliament should meet at
is called judicial review. least twice in a year.
52. Principles of a Welfare State are discussed in -
57. Terrarossa is typically developed in the
(a) Directive Principles of State Policy
(b) Chapters on Fundamental Rights geographical region of -
(c) Seventh schedule of Constitution (a) Limestone (b) Cynite
(d) Preamble of Constitution (c) Granite (d) Sandstone
Ans. (a) : The principles of welfare State are found in Ans. (a) : It is type of red clay soil produced by the
the Directive Principles of State Policy in part IV of the weathering course of limestone. This type of soil found
Indian constitution from Article 36 to 51. Directive in Mediterranean region.
Principles are fundamental in the governance of the 58. Mauna loa is an example of -
country. DPSPs are non-justifiable in the court. It is (a) Active volcano (b) Dormant volcano
borrowed from the constitution of Ireland. (c) Extinct volcano (d) volcanic plateau
39th BPSC (Pre) Exam. 1994 18 YCT
Ans. (a) : Mauna Loa is the largest active volcano, The Ans. (c) : Democracy is the form of government which
word, 'Mauna Lao' means, "long mountain" Mauna Loa is by the people for the people and from the people.
is an active shield volcano in the Hawaiian Islands, are Hence, citizens are at the centre of democracy. To
of five volcanoes that form the Island of Hawaii in the increase the participation of citizens in the democracy it
USA Pacific Ocean. needs universal Adult franchise so that every citizen of
59. What are the reasons of arising war-like the country participate in decision making without any
tension between North Korea and America? bias.
(a) Flatly refusal of North Korea for the 63. The President can proclaim National
inspection of its atomic sites emergency on -
(b) Launching of patriot missiles by America at (a) Armed rebellion
North Korea (b) External aggression
(c) Economic apartheid against North Korea by (c) War
Japan and America (d) All of the above
(d) Monopoly over Korean penisula and regional
policy of America Ans. (d) : According to the Article 352 of Indian
Constitution, the president can declare a national
Ans. (a) : Flatly refusal of North Korea for the
inspection of its atomic sites was the main reason of emergency when the security of India or a part of it's
arising war-like tension between North Korea and threatened by war or external aggression or armed
United States of America. rebellion. The 42nd Amendment Act of 1976. enabled
60. Recently, thousands of devotees of Hare the president to limit the operation of a National
Krishna residing in Britain participated in emergency to a specified part of India. The
Virodh-March against the government. proclamation of Emergency must be approved by both
Virodh-March was related to - the Houses of parliament within one month from the
(a) Prohibition for the construction of a new date of its issue and President must revoke a
temple in Britain by the Hindus proclamation if the Lok Sabha Passes a resolution
(b) The order to close Bhaktivedant temple disapproving its continuation.
(c) In certain notified areas prohibition on the 64. As far as the representatives of the Lok Sabha
Hindus to meet Englishmen are concerned, which states ranked 2nd and
(d) Prohibition to build new houses by the 3rd?
Hindus for themselves (a) Bihar and Maharashtra
Ans. (b) : Thousands of devotees of Hare Krishna (b) Madhya Pradesh and Tamil Nadu
residing in Britain participated in virodh-march against (c) Madhya Pradesh and Maharashtra
the government due to the government ordered to close (d) Maharashtra, Andhra Pradesh and West
Bhaktivedant Temple at that time. Bengal
61. The book 'Tereekh-e-Mujahideen' by
Ans. (a) : The Lok Sabha (House of the People) was
Pakistani-author Dr. Sadiq Hussain provoked
a lot of emotions of the Indian people because duly constituted for the first time on 17 April, 1952
in this book - after the first General elections. The first session of the
(a) It is said that the Sikh Gurus are called first lok Sabha commenced on 13 May, 1952. In Article
thieves, wicked and plunderers 81, the Lok Sabha as per the Constitution, 530 members
(b) It is said that Indian Muslims are abused chosen by direct election from states, and 20 members
rudely from union territories.
(c) It is said that Kashmiri extremists are The representation of Lok Sabha seat, state wise:
provoked to kill Indian leaders 1. Uttar Pradesh — 80
(d) It is said that Hindus are asked to leave 2. Maharshtra — 48
Pakistan 3. West Bengal — 42
4. Bihar — 40
Ans. (a) : The Pakistani-author Dr. Sadiq Hussain
5. Tamilnadu — 38
provoked a lot of emotions of the Indian people through
his book. 'Tereekh-e-Mujahideen' because he said in his Note :-At 10th Lok Sabha general election (1991)
book that the Sikh gurus were thieves, wicked and Jharkhand (14) was the part of Bihar so then Bihar (54)
plunderers. was the second and Maharastra was on third rank.
62. The essence of the democracy of India is based 65. The sky appears blue, because -
on - (a) In sunlight blue colour is more than other
(a) Written colours
(b) There are Fundamental Rights given (b) Short wavelength gets scattered more than
(c) People have a right to choose and change the long wavelength by atomosphere
government (c) Blue colour is more soothing
(d) There are Directive Principles of State (d) Atmosphere is more absorbed by short
Policies wavelength

39th BPSC (Pre) Exam. 1994 19 YCT


Ans. (b) : The Sky colours appears blue, because the Ans. (b) : Pitchblende contains many radioactive
Rayleigh scattering. elements that can be traced back to the decay of
Rayleigh scattering is inversely dependent upon Uranium, such as Radium, Lead, Helium, In 1898 Marie
wavelength. The colours with the smaller wave lengths and Pierre Curie discovered the element Radium while
are scattered more. The violet and blue colours are working with Pitchblende, It is radioactive uranium rich
scattered through the atmosphere and fall in our eyes. mineral and one.
Hence, the blue light reaching our eyes makes the sky The Symbol of Radium-Ra, atomic number = 88
appear blue. 72. The normal dry cell has electrolyte of -
66. One micron is equal to - (a) Zinc
(a) 1/10 m.m. (b) 1/100 m.m. (b) Sulphuric Acid
(c) 1/1000 m.m. (d) 1/10,000 m.m. (c) Ammonium Chloride
Ans. (c) : As we know that - (d) Manganese Dioxide
1 meter = 100 cm or 1000 mm Ans. (c) : A dry cell is one of the electrochemical cells,
and 1 micron ( µ ) = 1/1000000 meter or 10-6 m developed by the "German scientists Carl Gassner" in
-4 -3
so 1 µ = 10 cm or 10 mm or 1/1000 mm. 1886. A dry cell uses a paste electrolyte of Amonium
chloride next to zine anode. It also consists of a carbon
67. The characteristics of superconductivity - cathode which is surrounded by a mixture of carbon and
(a) High Permeability manganese dioxide (MnO2). Dry cell produces a
(b) Low Permeability maximum of 1.5 V of voltage.
(c) Zero Permeability
73. Dynamo -
(d) Infinity Permeability
(a) It transforms electrical energy into kinetic
Ans. (c) : Super Conductivity was discovered by Heike energy
Kamerlingh Onnes in 1911. The primary characteristics (b) It transforms mechamical energy into
of super conductivity is zero permeability. Permeability electrical energy
is the measure of the resistance of a material against the (c) It transforms electrical energy into
formation of magnetic field. mechanical energy
68. Year is maximum on - (d) It generates mechanical energy
(a) Pluto (b) Jupiter Ans. (b) : A Dynamo is an electric generator that
(c) Neptune (d) Earth generates Direct Current (DC) using a commutator. It
Ans. (a) : Year is maximum on Pluto. After the converts mechanical energy into electrical energy.
discovery of Eris in 2005, Pluto was demoted in 2006 The electric dynamo uses rotating coils of wire and
from a planet to a dwarf planet by the IAU
magnetic fields to convert mechanical rotation into a
(International Astronomical Union). Pluto is ninth-
largest and tenth-most massive known object directly pulsing direct electric current through Faraday's law of
orbiting the sun. induction and Lenz's law.
69. The rounded shape of a rain drop is caused 74. The theory of an 'Atom Bomb' is based on -
becacuse of - (a) Nuclear Fusion (b) Nuclear Fission
(a) Density of water (b) Surface tension (c) Both of them (d) None of these
(c) Atmoshperic (d) Gravitation Ans. (b) : The theory of an 'Atom bomb' is based on the
Ans. (b) : The property by virtue of which liquid tries to Nuclear Fission. The fission process becomes self-
minimized its free surface area is called surface tension. sustaining as neutrons produced by the splitting of atom
Force strike nearby nuclei and produce more fission. This
Surface tension = Newton per meter. known as chain reaction and it causes an atomic
length
Due to the surface tension, a rain drop tends to assume a explosion. The isotopes of uranium-235 and Plutonium-
rounded or spherical shape to minimize the surface area. 239 are used in atomic research because they readily
70. The open door of a refrigerator in a room can- undergo fission.
(a) Cool the room to some less degree Nuclear fission of heavy elements was discovered by
(b) Cool the room up to the temperature of the German Scientist Otto Hahn, with the help of Fritz
refrigerator Strassmann.
(c) Warm the internal room to some extent 75. The formula of the Plaster of Paris is -
(d) Neither warm nor cool the room (a) CaSO4 (b) CaSO4.H2O
Ans. (c) : If a refrigerator's door is kept open, then room (c) 2CaSO4.H2O (d) CaSO4.2H2O
will become hot, because then refrigerator exhaust more Ans. (c) : The chemical formula of Plaster of Paris is-
heat into the room than earlier. In this way, temperature (CaSO4.1/2 H2O) or (2 CaSO4.H2O) and it is known as
of the room increases and room becomes hot. No calcium sulphate hemihydrate. The compound 0
Plaster of
refrigerator is efficient. Thus it exhaust more heat into Paris is prepared by heating gypsum at 120 C.
the room than it extract from it. 1 3
CaSO4 .2H 2O 1200 C CaSO4 . H 2O + H 2O
71. Which mineral is the source of radium? 2 2
(a) Limestone (b) Pitchblende (Gypsum) (POP) (Water)
(c) Rutile (d) Heamatite
39th BPSC (Pre) Exam. 1994 20 YCT
76. Lemon tastes sour, it is caused by - Ans. (d) : Cold-blooded animals can be defined as
(a) Hydrochloric Acid (b) Acetic Acid animals that cannot regulate their internal body
(c) Tartaric Acid (d) Citric Acid temperature with the change in the environment. They
Ans. (d) : The lemon contains about 5-6% of citric acid cannot survive in extreme temperature conditions.
which gives a sour taste. Citric fruit juices such as These animals also called as Poikilothermic. Such as,
orange and lemon may be useful for courling the risk of Tortoise, Fishes, Sharks, etc.
specific type of kidney stones. The value of lemon juice 82. Out of water, fish die because -
is about 2-3. (a) They get a large amount of oxygen
77. Which one is false about bleaching powder? (b) Their body temperature is increased
(a) Highly soluble in water (c) They cannot inhale
(b) Light yellow coloured power (d) They cannot move in water
(c) Oxidising agent Ans. (c) : The water acts as hydration for the fishgills.
(d) Reaction with dilute acid it gives chlorine When out of water the gills dry up and cannot functions
Ans. (a) : Bleaching powder is soluble in water but due properly longer, they suffocate and die out of water,
to the presence of impurities, we never observe a clear The oxygen present in water is dissolved oxygen. Fish
solution. So option (a) is false. need dissolved oxygen for respiration.
Bleaching powder is chemically, calcium oxychloride 83. The members of 'Bee-colony' recognise one
(CaOCl2). Bleaching powder is manufactured using another. How?
Backmann's plant in which slack lime and chlorine are (a) By smell (b) By sight
made to react to create Bleaching Powder. (c) By dance (d) By touch
It is used for bleaching dirty clothes in the laundry, as a Ans. (c) : Honeybees have evolved complex
bleaching agent for cotton and linen in the textile communication systems to maintain the colony, The
industry bees communicate continuously, inside and outside the
78. Amalgam is - nest. One specific communication behavior is the Bee
(a) Ultra coloured alloy Dance, which serves to recruit nest mates to flowers,
(b) Alloy with carbon waters or new nesting sites.
(c) Alloy with mercury 84. How many teeth develop two times in the
(d) Alloy of high resistant to abrasion lifespsn of a human?
Ans. (c) : Amalgam is an alloy of mercury with one or (a) 4 (b) 12
more metals. (c) 20 (d) 28
Mercury is a transitional metal and it is the only metal, Ans. (c) : There are about 20 teeth in humans which
which is liquid at room temperature. Silver and gold regrow twice to form the permanent set of teeth during
amalgams are used to fill tooth cavity. their life. In adult human, total number of teeth is 32.
79. The normal type of coal is - 85. The leg bones of a human body are -
(a) Bituminous (a) 8 (b) 30
(b) Semi-bituminous (c) 32 (d) 34
(c) Anthracite Ans. (b) : There are total 30 bones in one leg of an
(d) Coke adult. These bones include femur the thigh bone,
Ans. (a) : Coal is a readily combustible, black or Patella- (the knee cap), Tibia-Fibula-the bone below
brownish-black sedimentary rock and predominantly knee cap and Tarsals, Meta- Tarsals and Phalanges.
made of carbon. There are basically four type of coal 86. How many bones are present in a human skull?
found in India- (a) 8 (b) 12
⦁ Anthracite (c) 32 (d) 28
⦁ Bituminous Ans. (*) : There are 29 bones are present in the human
skull.
⦁ Sub-bituminous Cranial bones — 8
⦁ Lignite Facial bones — 14
Bituminous coal is a normal type of coal found in India. Auditory bones — 6
80. National animal (mammal) of India is - Hyoid bones —1
(a) Cow (b) Peacock All the given options are wrong.
(c) Lion (d) Tiger 87. The latest predecessor of a modern man is -
Ans. (d) : The National Animal in terms of Mammal of (a) Java man (b) Cro-Magnon man
India is Tiger, The Scientific name of Tiger is 'Panthera (c) Neanderthal man (d) Peeking man
Tigris'. The Indian Tiger is the symbol of grace, Ans. (b) : Cro-Magnon man is the nearest ancestor of
strength, ability and enormous power. humans and has close relations and similarities with
81. Which one is a cold-blooded animal present-day human beings.
(a) Penguin (b) Whale Cro-Magnon man lined during the stone age which is
(c) Otter (d) Tortoise also called the Paleolithic age.
39th BPSC (Pre) Exam. 1994 21 YCT
88. Generation of plant-like mother plant be by - 94. Plant of peas is -
(a) seeds (a) Herb (b) Flower
(b) Grafting (stem cutting ) (c) Sherb (d) None of these
(c) None of both Ans. (a) : The Pea plant is a herb. It's botanical name is
(d) Both of them Pisum Sativum, It is dicotyledonous plant. Mendel
Ans. (b) : A mother plant is a plant from which a choose garden pea plant for his experiment because the
grower cuts clones, which are new plants that are flowers of this plant are bisexual and they are self-
genetically identical to the mother plant. To create a pollinating and thus, self and cross pollination can
clone, the grower cuts a branch from the mother plant easily be performed.
and places it in a substrate, where it develops roots of its
95. The main factor of evolution is -
own.
(a) Mutation (b) Acquired character
89. In a cell, which of these is present more than (c) Sexual production (d) Natural selection
80%?
(a) proteins (b) Fats Ans. (d) : Natural selection is the main factor in
(c) Minerals (d) Water evolution, as it provides direction to the differentiation
Ans. (d) : Human body consisting more than 80% of and other acquired genes into the next generation.
water, further, it regulates the activities of fluids, Natural selection is the process that causes significant
tissues, cells, lymph, blood, glandular secretions. An changes in the frequencies of both gene forms and allele
average adult body contains 50-65% liter of water and it genes in a population of organisms.
is provided for cell functions, the decrease in our daily 96. The age of the oldest rock system term is
in take affect the efficacy of cell activity. confirmed by -
90. Insulin is extracted by - (a) K-Ar method (b) C14 method
(a) Rhizome of ginger (c) Ra-Si method (d) U-Pb method
(b) Roots of Dhalia Ans. (d) : Uranium-lead dating is one of the oldest
(c) Balsam flower method and most refined of the radio metric dating
(d) Stem of potatoes schemes. That uses the decay chain of uranium and lead
Ans. (b) : Insulin is a hormone, responsible for glucose to find the age of a rock. Uranium decays radioactively,
regulation. It is produced by beta cell of pancreas it becomes different chemical elements, until it stop at
Insulin is acquired from the Dahlia tubers. Insulin has lead.
been used in the treatment of Diabetes mellitus since 97. Air is a -
1922 when Leonard Thompson become the first human (a) Compound (b) Element
to receive an injection of man-made insulin. (c) Mixture (d) Electrolyte
91. For which invention was Hargovind Khurana Ans. (c) : Air is a mixture of the gaseous substances as
rewarded? Nitrogen, Oxygen, and smaller amount of other
(a) Synthesis of protein substances and dust particles, it was indefinite shape
(b) Synthesis of genes and volume has mass and weight. It consists of 78%
(c) Synthesis of nitrogenous bases nitrogen, 21% Oxygen 0.9%, Argon and 0.04% Carbon
(d) None of these
dioxide.
Ans. (b) : Hargovind Khurana was an Indian born
American an organic chemist who specialized in the 98. Soil conservation is a process in which-
study of protein and nucleic acids shared Nobel prize in (a) Infertile soil transforms into fertile soil
medicine with Robert W Holley in 1968. For their (b) Soil is depleted
interpretation of the genetic code and its function in (c) Soil is eroded
protein synthesis. (d) Soil is protected from erosion
92. The biggest flower of the world is - Ans. (d) : Soil conservation is a method of protecting
(a) Lotus (b) Rafflesia soil from getting eroted either by wind or by water.
(c) Big cactus (d) None of these some methods for soil conservation are:
Ans. (b) : Rafflesia Arnoldii rain forest flowers found ⦁ Afforestation
mainly in Indonesia. It can be grow to be 3 feet across ⦁ Crop Rotation
and weight up to 15 pounds. It is a parasitic plant, with ⦁ Contour ploughing
no visible leaves, roots, or stem. ⦁ Terracing and contour bunding
93. Generally stem-cutting is used for the 99. Which Indian won the recently held A round-
generation of - the-World Air Race Tournament?
(a) Banana (b) Sugarcane (a) Vijaypant Singhania
(c) Mango (d) Cotton (b) Dalpat Singhania
Ans. (b) : Stem cutting are commonly used for the (c) Dr. Karan Singh
propagation of the Sugarcane plant. It is a mode of (d) Nawal K. Tata
vegetative propagation. The stem is used to grow a Ans. (a) : The First air race held at Domingvez field
whole new plant similar to the parent plant. It is known United States in 1910. Vijaypant Singhania participated
as cloning. The stem sections should be free of diseases in Arizona in 1994. He was the first Indian who won the
and pests. competition.

39th BPSC (Pre) Exam. 1994 22 YCT


100. Which of the following statements is true about Ans. (a) : The Total area of India is 3287263 sq. km.
sedimentary rocks? which is approximately 2.2% of the total surface of the
(a) These are rocks, whose structure depends on world. India is the seventh largest country in terms of
temperature and pressure the area after Russia, Canada, United States of America,
(b) These rocks are crystalline China, Brazil, and Australia respectively.
(c) These rocks are stratified The total population of India is 17.5% (2011 census
(d) These rocks are not deposited in water report) of the total population of the world.
Ans. (d) : The statement is not true about sedimentary Note : (In 1991 census report India's population was
rocks is that these rocks are not deposited in water. 16% around of the world population).
Sedimentary rocks are formed by the deposition and Hence, right option is (a).
subsequent cementation on the material at the earth's 106. The depth of alluvial soil of the Ganga is about
surface within the bodies of water. These rocks are -
formed in: (a) 6000 metres (b) 600 metres
⦁ Deposition of weathered remains of other rocks. (c) 800 metres (d) 100 metres
⦁ Accumulation and the consolidation of sediments. Ans. (b) : The average depth of alluvial soil in the
⦁ Precipitation from solution. Gangetic plain is about 600m. Alluvial soil is mainly
101. 'Tin is obtained by - found in Northern plains. In the upper and Middle
(a) Placer deposit Ganga plain, two type of alluvial soils have been
(b) Metamorphic rocks developed, namely, Khadar (new alluvial) and Bhangar
(c) Igneous rocks containing low silica (old alluvial).
(d) All of these 107. Indian Monsoon is indicated by change of
Ans. (a) : Tin (Sn) is obtained mainly from the mineral season, it is caused by -
cassiterite, cassiterite often accumulates in alluvial (a) Temperature difference between land and
channels as Placer deposits because it harder, heavier, ocean
more chemically resistance. (b) Cool air from central Asia
(c) Uniformity of temperature
102. The south of 'Shiwalik' rock series. Bhabar
(d) None of these
region is an example of -
(a) Midland situation Ans. (a) : The factors influencing Indian Monsoon
(b) Inter mountaining situation formation due to the differential heating and cooling of
land and water which creates a low pressure on the
(c) Piedemont situation
landmass of India while the seas around experience
(d) Littoral region
comparatively high pressure.
Ans. (c) : The South of 'Shiwalik' rock series, Bhabar
108. Which river has a source near Mansarovar
region is an example of Piedmont Situation. The
Lake in Tibet?
piedmont zone is the northernmost geographic in Ganga
(a) Brahmaputra (b) Satluj-Indus
Plain.
(c) Indus (d) All of these
103. The collection of islands "Ladshadweep" is a -
Ans. (d) : Mansarovar lake in Tibet, is hear the source
(a) Coral origin (b) Volcanic origin
of the Sutlej which is the eastern most large tributary of
(c) Soil deposit (d) None of these
the Indus. Nearby the source of Brahmaputra river, the
Ans. (a) : Lakshadweep is the chain of coral Islands in Indus river, and the Kurnali, an important tributary of
the subcontinents. It was formerly known by various the Ganges. Lake Mansarovar overflow into Lake
names Laccadive, Minicoy and Amini Islands, Rakshastal which is a salt-water endorheic lake.
Kawaratti is the capital of Lakshadweep. It is separated 109. Hundru Fall is on -
from Minicoy island by 90 channel, 80 channel seperates (a) Indravati (b) Cauveri
Minicoy island to Maldives. (c) Subarnarekha (d) None of these
104. India is spread between - Ans. (c) : Hundru fall is situated on the course of the
(a) 37 17'53" N and 8 6'28" S Subarnarekha River. It is originated near Nagri village
(b) 37 17'53"N and 8 4'28" S in the Ranchi district, Jharkhand, Hundu falls is at a
(c) 37 17'53"N and 8° 2'28" S height of 320 feet, and it is the 34th highest waterfall in
(d) None of these India.
Ans. (d) : India is situated between 804' to 3706 (North 110. Which of the follwing is snow line in the
latitude) and 6807' to 97025' East longitude in south- Himalayas
eastern part of Northern Hemisphere. (a) 6400 to 6000 m in the east
(b) 4000 to 5800 m in the west
105. The area of India is 2.2% of the world, but (c) 4500 to 6000 m in the west
population is - (d) None of these
(a) 16% of the world population Ans. (c) : The Himalayas have snowline between at
(b) 17% of the world population 4500 m to 6000 m in the west region of mountain. This
(c) 18% of the world population is because the Himalayas are located in the lower
(d) 28% of the world population latitudes.
39th BPSC (Pre) Exam. 1994 23 YCT
111. The coastline of India is - Ans. (b) : Kolkata was the most populated city in India,
(a) 6,200 km long (b) 6,100 km long Kolkata is also known as the city of Joy, There are
(c) 5,985 km long (d) 6,175 km long mainly three headquarter in Kolkata that is Geological
Ans. (b) : The total coastline of India measures about survey of India, Botanical survey of India and
7516.6 (7517) km with the nine coastal states and four Zoological survey of India.
union territories. The coast of India include west coast, Note : When the question was asked Kolkata was the
east coast, coast of Lakshadweep, Andaman and most populated city but as census of now, Mumbai is
Nicobar island. States only considered about 6100 km. the most populous city.
112. Which is not included in cash crops? 118. Which continent is the biggest in area?
(a) Sugarcane (b) Cotton (a) Europe (b) Africa
(c) Jute (d) Wheat (c) North America (d) South America
Ans. (d) : Wheat is not a cash crop in India. Generally, Ans. (b) : There are seven continent in the world. Asia
plants that people grow to sell and not to eat or use is on the top in terms of both area and population.
themselves crops which are produced by farmers mainly Africa is the second-largest continent in the world with
to earn money. 6% of the earth's total geographical area and 20.4% of
Examples of cash crop is: the land area of the world. It is surrounded by
Coffee, Cocoa Tea, Sugarcane, Cotton, Jute, Tobacco, Mediterranean sea to the north, the Indian Ocean to the
Rubber and Spices. south east, the Atlantic Ocean in the west. and the Suez
canal and the Red sea in the Northeast. It has a tropical
113. India is the highest producer and consumer of - and subtropical type of climate. It is the hottest
(a) Rice (b) Tea continent in the world.
(c) Oilseeds (d) Pulse
119. The clear sky night is colder than a cloudy sky
Ans. (b) : India is the largest producer and consumer of night, due to -
tea. Tea is the highly famous and also highly consumed (a) Condensation (b) Radiation
product in India. In India, tea is produced in the large (c) Insulation (d) Conduction
amount and export to other countries. There are many Ans. (b) : Clear nights are colder than the cloudy night
tea plantations in Kerala, Nilgiri, Assam, Karnataka, due to the process of radiation, where the energy waves
Tripura etc. are some of the topmost tea production travel through vacuum as electromagnetic waves. Cloud
states in India. (*According to 1992 report) effect reduce the radiation imbalance between the
114. Which state is richest in mineral production? upward flux from the ground and downward flux from
(a) Rajasthan (b) Madhya Pradesh the atmosphere, relative to that on clear nights.
(c) Bihar (d) Odisha 120. Which of the following pairs is correct?
Ans. (d) : Odisha is one of the most mineral-rich states (a) Mediterranean region-Summer rainfall
in India. Mineral found in the State are iron, coal (b) Equatorial region-Rainfall with thunder in the
bauxite etc. Odisha has emerged as the key State with afternoon
regard to the mineral and metal-based industries. In (c) Monsoon region-Heavy rainfall in all months
2019-20, Odisha contributed to the largest share (d) Desert region-Winter rainfall
(35.3%) of mineral production in India. Ans. (b) : The equatorial region get the rainfall with
115. In India the first fertilizer plant was thunder in the afternoon because the equatorial region is
established at - the areas of abundant precipitation. The high level of
(a) Nangal (b) Sindri rainfall is due to the equatorial belt's high temperature,
(c) Alwaye (d) Trombay high humidity and highly unstable air.
Ans. (b) : The first fertilizer plant in India in Sindri, 121. Which of the following industrial clusters is on
Jharkhand (at that time, Bihar), to produce ammonium Chhotanagpur Plateau?
sulphate in 1951, urea in 1959 and double salt in 1959. (a) Bhilal (b) Ranchi
Jharkhand separated from Bihar in 15th November 2000. (c) Asansol (d) Durgapur
116. If the population of India is increased by 2% Ans. (b) : Ranchi Industrial clusters is on Chhota
annually then the population is increased to Nagpur Plateau. Chhota Nagpur Plateau covers much of
two times in - Jharkhand (at that time Bihar) Orissa, West Bengal,
(a) 25 years Bihar and Chhattisgarh.
(b) 30 years 122. Damodar river originates from -
(c) 35 years (a) Tibet
(d) 40 years (b) Chotanagpur Plateau
(c) Near Nainital
Ans. (c) : The population will be doubled than the (d) Someshwar Hills
previous government data indicator of growth rate
Ans. (b) : Damodar River originated from Chhota
immigration, Total Fertility Rate (TFR), population Nagpur Plateau. If flows across Indian states of
density and urbanization. Jharkhand and West Bengal. It is called as 'Sorrow' of
117. The most populated city India is - Bengal because of its ravaging floods in the plains of
(a) Bangalore (b) Kolkata West Bengal. Barakar is the most important tributary of
(c) Hyderabad (d) Chennai the Damodar.

39th BPSC (Pre) Exam. 1994 24 YCT


123. Which is know as the 'Mistress of the east Ans. (a) : The progress of India is satisfactory in the
ocean'? increment in the GNP (Gross National product).
(a) Sri Lanka (b) Pakistan
GNP is the value of all finished goods and services
(c) Myanmar (d) India
owned by a country's residents over a period of time.
Ans. (a) : Sri Lanka is called as 'Mistress of the East
Ocean'. Sri Lanka is an island country in the Indian 129. Which industry of India was badly affected by
Ocean. Formely known as 'Ceylon'. The capital of Sri partition?
Lanka is Sri Jayawardenpura Kotte. (a) Cotton and sugar industries
124. The forest area in Bihar is - (b) Engineering and cement industries
(a) 28 lakh hectare (b) 29 lakh hectare (c) Jute and cotton industries
(c) 30 lakh hectare (d) 64 lakh hectare (d) Paper and iron industries
Ans. (b) : The forest area in Bihar was 29 lakh hectares.
As per latest assessment 2021, Bihar has a total Ans. (c) : Jute and cotton industries were badly affected
geographical area of 94,163 Sq.km. by partition. The reason being, most of the jute and
The Total forest Area Recorded is 6,877.41 sq km, cotton industries were in West Punjab and East Bengal.
which is 7.3% of the total geographical area of the state. 130. The priority of the eighth five year plan is -
125. A steel plant of Bihar is at - (a) Employment generation
(a) Darbhanga (b) Gaya (b) Increasing import
(c) Hazaribagh (d) Jamshedpur (c) Increasing industries
Ans. (d) : The first Iron and steel plant was set up in (d) Liberty of press
India at Jamshedpur. Tata Iron and Steel Company Ans. (a) : The Major objectives of Eight five year plan
Limited (TISCO) was the first iron and steel Industry in (1992-1997) was:
India. It was located in Jamshedpur Bihar, presently ⦁ Creation of employment, check population growth,
Jamshedpur is in the Jharkhand state. The industry is and overall human development.
located on the banks of Subarnarekha and Kharkai river.
⦁ Primary health facilities, drinking water &
126. In India, planned economy is based on - vaccination in all village.
(a) Socialist system
⦁ Strengthen the basic infrastructure.
(b) Mixed economy
(c) Capitalist system ⦁ The expected growth rate of eight five year plan
(d) Gandhian system was 5.6% and attained 6.78%.
Ans. (b) : Indian economy is neither fully capitalist nor 131. In which industry are maximum people
socialist but is a mix of both systems. This is because in working in India?
India post-independence, the inadequate capacity of (a) Jute Industry (b) Iron-steel Industry
private sector resulted into state taking more initiative (c) Textile Industry (d) Sugar Industry
but post liberalisation the share of state participation has Ans. (c) : The textile industry occupies unique position
decreased whereas private/market participation has in the India economy, because it contributes
increased. significantly to industrial production (14%) and
127. In India the largest source of revenue is - employment generation (35 million persons directly the
(a) Direct tax (b) Sales tax second largest after agriculture) and foreign exchange
(c) Railways (d) Excise duties earnings (about 24.6%)
Ans. (a) : The largest source of Indian revenue was 132. The main source of national income of India is -
Direct tax. There are two type of taxes that are levied in (a) Industry (b) Agriculture
India. These are direct taxes and Indirect taxes. (c) Forest (d) Foreign trade
Direct tax : Income tax, corporate income tax, securities Ans. (a) : The main source of national income of India
transaction taxes, etc. in 1990, was industry. As of now in, 2021 Service
Indirect tax : G.S.T. custom duty etc. sector is the main source of national income.
128. The progress of India is satisfactory in - 133. According to the South Asia coalition on child
(a) Increment in gross national products
(b) Reduction of unemployment servitude report, how many captive children
(c) From, economic reforms (1991) India's GNP are there in India?
gradually increases (a) 4.5 crore (b) 5.5 crore
(d) Ralation to inequity (c) 6 crore (d) 6.5 crore

39th BPSC (Pre) Exam. 1994 25 YCT


139. The average of Monday, Tuesday and
Ans. (a) : According to the south Asian coalition on
Child Servitude Report (1991), there are 4.5 crore Wednesday was 42°C. The average
captive children in India. temperature of Tuesday, Wednesday and
134. In India, What percentage of rural population Thursday was 43°C. If the temperature of
has the avaiability of primary education? Monday was -
(a) 94% (b) 96% (a) 41°C (b) 42°C
(c) 97% (d) 98% (c) 43°C (d) 44°C
Ans. (a) :
Ans. (a) : As of 2018, 94% of rural children are
Mon+Tue+Wed
enrolled in primary education. Out of these 91% girls Tue + Wed + Thu
= 42, = 43
and 97% boys are educated in primary education. 3 3
135. "Shahtoosh" is supposed to be the finest, Mon + Tue + Wed = 126
warmest and lightest wool of the world, where
and Tue + Wed + Thu = 129
is it grown? ∵ Mon - Thu = -3
(a) Nepal (b) Uzbekistan
(c) China (d) Bngladesh Mon - 44 = -3
Mon = -3 + 44 = 410C
Ans. (c) : Shahtoosh is a very fine type of wool, It is
140. Consider the following :
also called the king of fine wools. This wool is made up
of the hairs of Chiru, which is a Tibetan Antelope. This
P : For 40% increment in any number, it is
animal lives in a very cold environment generally at an sufficient that number is multiplied by 1.4
altitude of 5000 meters. Due to the great quality of
Q : For 40% decrease in any number, it is
Shahtoosh wool. Its demand was very high during the
20th century. sufficient that number is divided by 1.4
136. Which of the following numbers is placed in the (a) P is true but Q is false
blank? 2,4,3,9,4,16,5,25,6,......... (b) Q is true but P is false
(a) 36 (b) 39 (c) Both P and Q are true
(c) 81 (d) 243 (d) Both P and Q are false
Ans. (a) : Ans. (a) : Statement P :
According to the question - 100+100×40% =140
∴ 100 × 1.4 = 140
So, P is true
Statement Q : 100-100×40% = 60
137. If HORSE is written as DRQNG in any code, and, 100 = 71.42
then how is MONKEY written in that code? 1.4
(a) XDJMNL (b) YEKNOM Hence, Q is false.
(c) ESRODH (d) GNQRDM 1
141. If 33 % of X is 11 then the value of X will be-
Ans. (a) : 3
(a) 11 (b) 22
(c) 33 (d) 44
Ans. (c) :
1
33
X × 3 = 11
100
Similarly MONKEY will be coded as XDJMNL. 100 1
⇒ X× × = 11
138. Which smallest number is multiplied to 56700 3 100
∴ X = 33
to be a perfect square?
Hence the value of X is 33.
(a) 3 (b) 6
142. Two trains running on the same day at 10
(c) 7 (d) 9 O'clock and 11 O'clock in the morning from
Ans. (c) : Prime factors of 56700 Delhi to Amritsar having their speed to 60
km/h and 75 km/h respectively, how far from
Delhi will they meet each other?
so, we see that 7 does not make any pair, so we should (a) 150 km (b) 200 km
multiply it by 7 to get a perfect square. (c) 250 km (d) 300 km

39th BPSC (Pre) Exam. 1994 26 YCT


Ans. (d): Relative speed of the train = 75 - 60 Ans. (c): Rajiv Gandhi Computer Science University
= 15 km/hr was the first university under private sector in India. It
Distance travelled by first train in 1 hour=60×1 = 60 km was established in 1984 in the Arunanchal Pradesh.
60 147. For the peaceful purpose, of which country is
time taken to meet each other = = 4 hours
15 India far ahead in the utilization of atomic
distance travelled by second train in 4 hours = 75×4 = power?
300 km (a) China (b) Pakistan
Hence, At 300 km from Delhi they will meet each other. (c) France (d) England
143. In an examination 20% students fail in English, Ans. (b) : India is an unofficial 6th nuclear power
and 35% students fail in Hindi, 27% students
country in the world. But majority of Indian nuclear
fail in both subject. If the number of successful
installation are used for civil applications i.e. electricity
students is 288, then the number of students
generation, medicinal purposes, etc. so India far ahead
sitting in the examination is?
(a) 600 (b) 500 from Pakistan in the utilization of Atomic power for
(c) 400 (d) 300 peaceful purpose.
Ans. (c) : Suppose the total number of students = 100 148. Who is the owner of star TV?
Percentage of failed students = 20 + 35 - 27 = 28% (a) Tata Group
Percentage of pass students = 100 - 28 =72% (b) Richard Attenborough
288×100 (c) Rupert Murdoch
∴ Total number of Students = = 400
72 (d) George Luka
144. My watch reads 5 minutes late at noon 2 pm on Ans. (c) : Rupert Murdoch is the owner of Star TV. He
Monday and 3 minutes ahead at noon 2 pm on is also the founder of News corp.
Wednesday. When does it show the right time? 149. Which high court gave the decision that it is
(a) 6 O'clock morning on Tuesday illegal to be divorced after the repetition of
(b) 8 O'clock morning on Tuesday word 'talak' three times at a stretch?
(c) 4 O'clock morning on Wednesday (a) Calcutta High Court
(d) 8 O'clock morning on Wednesday (b) Allahabad High Court
Ans. (b) : The time difference = 5 + 3 = 8 minutes (c) Bombay High Court
8 minute difference in 48 hours. (d) Punjab High Court
3 minute difference in = 6×3= 18 hrs. Ans. (b) : Allahabad High Court declared 'Triple Talak'
so, Right time = 2 O'clock on Monday + 18 hours.
(Muslim Divorce Practice) invalid.
= 8 O' Clock morning on Tuesday
Supreme court too in 2018 have shunned the practice
145. X, Y, Z, U are in increasing order and U, Y, W and as per Muslim women (Protection of Right of
are in decreasing order. Which of the following
Marriage) Act, 2019 it is illegal to pronounce Triple
shows neither increasing order nor decreasing
Talak.
order?
150. Under the agreement of Israel P.L.O on
(a) X, Y, Z (b) W, Y, Z
(c) X, U, W (d) U, Y, Z September 13, 1993. What regions did Israel
hand over to the Palestinians?
Ans. (c) : X,U,W shows neither in increasing order no
(a) Jerusalem and Sinal
decreasing order.
(b) Jerico and Gazapatti
146. Which university will be the first under the
(c) Jerico and Jerusalem
private sector in India?
(d) Gazapatti and Jerusalem
(a) Indira Gandhi Environment University
(b) Jawaharlal Nehru Life Science University Ans. (b) : Under the agreement of Isreal P.L.O. on 1993
(c) Rajiv Gandhi Computer Science National Israel handed over Jericho and Gazapatti to Palestine.
University The agreement is commonly known as Cairo agreement
(d) Sonia Gandhi Oceanic Science National and it ensured limited Palestine self-rule in West bank
University and Gaza strip within 5 years.

39th BPSC (Pre) Exam. 1994 27 YCT


40th Bihar Public Service Commission
Preliminary Examination, 1995
GENERAL KNOWLEDGE & GENERAL SCIENCE
(Solved Paper with Detail Explanation)
1. Who used the caves of Barabar as their shelter- Ans. (b) : Khams was a tax levied on the one-fifth of
house? the booty wealth captured during the war. It does not
(a) Ajivika (b) Tharus indicates the tax imposed on land produce.
(c) Jains (d) Tantriks Kharaj was a type of land tax on agricultural land. It
Ans. (a) : A cluster of four caves namely Lomas Rishi, was 1/10 of the produce of the land. Ushr was a land
Sudama caves, Vishvakarma Caves and Karan Chaupar tax. It levied from 10% harvests of irrigated land &
caves are known as Barabar caves is the oldest 10% on harvest from rain-watered land & 5% to the
examples of Mauryan rock-cut architecture in India. land dependent on well water.
These are supposed to have constructed by Ashoka for Muktai was a mixed system of tax assessment (Lagan).
the use of Ajivika ascetics. These caves are carved out 6. Identify the first ruler of Vijay Nagar who
from a monolithic granite rock. captured Goa from Bahmanis ?
2. Which of the following is the earliest holy book (a) Harihara I (b) Harihara II
of Jain? (c) Bukka II (d) Devraya II
(a) Twelve Angas (b) Twelve Upangas Ans. (b) : Harihara-II was the first ruler of Vijaynagar
(c) Fourteen Purvas (d) Fourteen Uparvas Empire, who had captured Goa from the Bahmanis.
Ans. (a) : Fourteen Purvas is the earliest teachings of Vijaynagar empire was founded in 1336 by two brothers
the Tirthankar as before Mahavir, which were Harihara I & Bukka Raya of Sangam Dynasty.
memorised and passed through the ages. But later is lost 7. Match the names mentioned in column I to the
on the basis of Mahavirs teaching the colonial texts of names mentioned in column II.
Jainism started which is known as Jain Agamas. In Jain Column-I Column-II
Agamas Twelve Angas is considered as earliest holy A. Nagananda 1 . Banabhatta
book of Jain.
B. Harshcharit 2. Harshavardhana
3. Gradual decline of towns is an important
C. Tughlaq Nama 3. Amir Khusro
feature of which period?
(a) Gupta period (b) Pratihara D. Tuhfat-ul- 4. Raja Ram
(c) Rashtrakuta Period (d) Satavahana Muwahhiddin Mohan Roy
E. Neel Darpan 5. Asde Malic Isasi
Ans. (a) : The decline of Gupta dynasty started during
6. Deenbandhu
the rein of Skandagupta. He was successful in
Mitra
retaliating against the Hunas & Pushyamitras but his
Codes
empire was drained of finance & resources because of A B C D E
this. Beside the Huna's invasion there was also gradual (a) 1 2 3 4 5
decline in economic prosperity as a result the towns (b) 2 1 5 3 4
which were the centers of economic activities began to (c) 1 2 3 5 6
decline. Hiuen Tsang has also talked about the gradual (d) 2 1 3 4 6
decline of major Gupta's is cities. Ans. (d) : Correct matches are as :-
4. Chinese traveller I-tsing visited Bihar around- Column-I Column-II
(a) 405 AD (b) 635 AD Nagananda — Harshavardhana
(c) 670 AD (d) None of these Harshcharit — Banabhatta
Ans. (c) : Chinese Buddhist pilgrim I-tsing visited Tughlaq Nama — Amir Khusro
Bihar around 670 AD. He left for India from Canton by Tuhfat-ul- — Raja Ram Mohan Roy
Sea in 671. After visiting the Sacred Buddhist sites in Muwahhiddin
Neel Darpan — Deenbandhu Mitra
Magadha, he resides at the Great Nalanda Monastery
for ten year (676-685AD) devoting himself to study of 8. Who of the following Englishman did Jahangir
respect with title 'Khan'?
the Vinaya. He stayed at Tamralipti for three years and
(a) Hawkins (b) Sir Tomas Roe
learned Sanskrit. (c) Edward Terry (d) None of these
5. Which of the following does not indicates the Ans. (a) : William Hawkins was a diplomat of the
tax imposed on land produce? British East India Company. He was an envoy in the
(a) Kharaj (b) Khams court of the Mughal Emperor Jahangir for two years. He
(c) Ushr (d) Muktai was called 'English Khan' by Jahangir.

40th BPSC (Pre) Exam. 1995 28 YCT


9. Identify the region from where Europeans got Ans. (d) : Revolt of 1857 was also known as First War
best shora and opium. of Independence. The English educated middleclass,
(a) Bihar (b) Gujarat Sahukar, Zamindars, rich merchants traders did not
(c) Bengal (d) Madras participated in the revolt of 1857. All the main leaders
Ans. (a) : European got best quality of shora and opium and kings of that time participated in this revolt.
form Bihar. Production of shora was also abundant in Agricultural Labour, farmers also participated in this
Bengal. revolt.
14. The revolt of 1857 affected many towns of
10. Who was the last Mughal emperor to sit on Bihar. Which of the following entires gives the
Mayur Singhasan? correct sequences of the outbreak of the revolt
(a) Shah Alam I (b) Mohammad Shah in different towns of Bihar-
(c) Bahadur Shah (d) Jahandar Shah (a) Patna, Muzaffarpur, Rohini, Jagdishpur
Ans. (b) : The peacock throne was a famous jewelled (b) Rohini, Patna, Muzaffarpur, Jagdishpur
throne that was the seats of the Mughal emperor of (c) Muzaffarpur, Patna, Jagdishpur, Rohini
India. It was constructed during the reign of Shahjahan. (d) Jagdishpur, Patna, Rohini, Muzaffarpur
Muhammad Shah was the last Mughal emperor to sit on Ans. (b) : The revolt of 1857 affected many towns of
the Mayur throne. In February 1739, Nadir Shah Bihar. The correct sequences of outbreak of the revolt in
attacked on Delhi & took away the peacock throne with different towns of Bihar is Rohini, Patna, Muzaffarpur,
him as the symbol of victory. Jagdishpur.
11. Which of the following laws was/were passed in 15. Trace the tribe that rebelled against Britishers
1856? because human sacrifice system was banned
(a) Kuke (b) Khond
(1) Lex-Loci Act (c) Oraon (d) Nackda
(2) Sati Abolition Act
Ans. (b) : Khond uprising took place in Orissa in 1846.
(3) Hindu Widow Remarriage Law
Chakra Bisoi was the leader of the Khond people. The
(4) Doctrine of Lapse main issue of Khond uprising was the banning of the
Select your answer to the following codes practice of 'Mariah'. It was a practice of human sacrifice
(a) 1 and 3 (b) 1 and 4 that was banned by Britishers. Additional reason was
(c) 3 and 4 (d) 1,2 and 4 the introduction of new taxes and influx of zamindars
Ans. (*) : Hindu Widow's Remarriage Act had legalized and moneylenders. This revolt was suppressed by
the remarriage of hindu widows on 16 July 1856. The Britishers in 1855.
Act was drafted during the tenure of Lord Dalhousie 16. Which year 'Hindustan Republican
and passed by Loard Canning. Bengal Sati Regulation Association' was set up?
Act of 1929 was passed in 1829 by the then Governor- (a) 1920 (b) 1924
General Lord William Bentinck which made the (c) 1928 (d) 1930
practice of sati illegal in all jurisdictions of India. Ans. (b) : Hindustan Republican Association (HRA)
Doctrine of lapse was an annexation policy followed was established in 1924 by Sachindra Nath Sanyal,
widely by Lord Dalhousie. It was first applied in the Narendra Mohan Sen, Pratul Ganguly. The other
year 1848 to the state of Satara. important members of HRA was Bhagat Singh, Chandra
Lex-Loci Act, 1850 was for protecting the right of a Shekhar Azad, Sukhdev, Ram Prasad Bismil, Roshan
converted to inherit ancestral property. Singh, Ashfaqulla Khan and Rajendra Lahiri. HRA was
later reorganized as the Hindustan Socialist Republican
12. Who called the revolt of 1857 a conspiracy? Association (HSRA) was established in 1928 at Feroz
(a) Sir James Outram and W. Taylor Shah Kotla in New Delhi.
(b) Sir John K. 17. First woman president of Indian National
(c) Sir John Laurence Congress was
(d) T.R. Homes (a) Sarojini Naidu (b) Pandit Ramabai
Ans. (a) : Sir James Outram & W. Taylor called that the (c) Aruna Asaf Ali (d) Annie Besant
revolt of 1857 was a result of a Hindu-Muslim Ans. (d) : Annie Besant was the first woman President
conspiracy. Outram's view was that it was a Muslim of INC who presided the Calcutta session in 1917.
conspiracy which took advantage of Hindu grievance. Sarojini Naidu was the first India women President of
V.D. Savarkar called revolt of 1857 as the First War of INC and who presided the Kanpur session in 1925.
Independence. TR Holmes expressed that it was a 18. In 1873, who founded Satyashodhak Samaj?
conflict between civilization and barbarism. (a) Gopal Krishna Gokhale
13. Which of the following groups did not (b) Jyotiba Phule
(c) Shiv Nath Shastri
participate in the revolt of 1857? (d) None of the above
(1) Agricultural Labour
Ans. (b) : Satyashodhak Samaj was founded by Jyotiba
(2) Sahukar
Phule in Pune, Maharashtra on 24th September 1873
(3) Farmers
which also means 'Truth-seeker's Society' formed in
(4) Zamindars
order to attain equal social & economical status for the
Select your answer to the following codes
lower castes in Maharashtra. He wrote a book named
(a) Only 1 (b) 1 and 2
(c) Only 2 (d) 2 and 4 Gulamgiri.

40th BPSC (Pre) Exam. 1995 29 YCT


19. Which of the following classes were mainly reform movements which aimed at reorientations of
unaffected by the Swadeshi Movement of 1905? Indian society along modern lines these movements
(1) Women mainly attracted the Intellectuals, Urban high castes,
(2) Farmers Liberal princely states.
(3) Muslims 23. Who put the demand of right to enter into the
(4) Intelligentsia temples that caused terrible riot in Trunelvali
Select your answer to following codes. in 1899?
Codes: (a) Oklinga (b) Nadars
(a) 1 and 2 (b) 1,2 and (c) Mahar (d) Pali
(c) 2 and 3 (d) 1 and 3 Ans. (b) : In 1899, Nadars of Tamil Nadu demanded the
Ans. (c) : Swadeshi movement was started against the right to enter in to temples that caused terrible riot in
partition of Bengal & got formal proclamation on 7 Trunelvali.
August 1905 at a meeting held at Calcutta Town Hall. It 24. Which reformer of Maharashtra was known of
was not only political and economic movement but also Lokhitwadi
the swadeshi leaders appeal Indian's for boycotting of (a) M. G. Ranade
foreign goods, government services, courts etc. This (b) Gopal Krishna Gokhale
movement was a great success, landlords women's & (c) Pandita Ramabai
students had taken part in this movement. However (d) Gopal Hari Deshmukh
Muslims and peasants were unaffected by this Ans. (d) : The intellectual thinker & social reformers
movement. Peasants fell that joining the movement from Maharashtra, Gopal Hari Deshmukh was born in
would not improve their conditions. Pune and popularly known as Lokhitwadi.
20. Under the provision of Regulating act a 25. Who among the following did not contribute to
provincial assembly for Bihar was set up in Home Rule League?
(a) 1772 (b) 1774 (a) Bal Gangadhar Tilak
(c) 1776 (d) 1778 (b) Annie Besant
Ans. (b) : The Regulating Act was the first (c) S. Surbramania lyer
parliamentary ratification & authorization defining the (d) T. A. Alcott
powers & authority of the East India Company with Ans. (d) : Home Rule League was started in 1915-16.
respect to its Indian possessions. The British Parliament There were two home rule league one under the
has passed this act in June 1773 and was implement in leadership of Annie Besant & other was under the
India in 1774. According to the provisions of this Act, a leadership of Bal Gangadhar Tilak. In April 1916 at
provincial assembly was setup in Bihar in 1774. Belgaum. Tilak founded the first home rule league then
21. Ninth Mandala of Rig Veda Samhita is after Annie Besant along with S. Subramania Iyer
completely dedicated to launched Home Rule league in September 1916 at
(a) Indra and his Elephant Madras. They had common objective of achieving self
(b) Urvashi and Heaven government in India. T.A. Alcott had not contributed to
(c) Gods related to plants and drugs Home Rule League.
(d) Soma and God who is named after the drink 26. Who viewed the Khilafat Movement as an
Ans. (d) : Rigveda is the oldest of the four veda opportunity for integrating Hindus and
consists of 10 Mandals and 1028 Hyms. Each Mandala Muslims which will to appear again within
is divided into Anuvakas. The Ninth Mandala is known coming hundred years?
as Soma Mandala. It is dedicated to Soma & God who (a) Ali Brothers
is named after the drink. The famous Gayatri Mantra is (b) Abul Kalam Azad
in third mandala. The Nasadiya Sukta which talk about (c) Mahatma Gandhi
the origin of the universe is in 10th Mandala. (d) Khan Abdul Gaffar Khan
22. Religious and Social Reforms Movements of Ans. (c) : Mahatma Gandhi was elected as President of
Nineteenth century attracted mainly which All India Khilafat Conference held in Delhi in 1919.
section of population? Gandhi ji saw this as an opportunity to bring Muslims
(1) Intellectuals under the umbrella of an unified national movement. At
(2) Urban High Castes the Calcutta session of INC in 1920 he convinced other
(3) Poor Common General Class leaders of the need to start a Non Cooperation
(4) Liberal Princely State Movement in support of khilafat as well as for swaraj.
Select your answer to the following codes 27. Indicate true sequence of events concerning the
(a) Only 1 (b) 1 and 2 postponement of non-cooperation movement
(c) 1,2 and 3 (d) 1,2 and 4 (1.) Police firing in Chauri-Chaura
Ans. (d) : During the 19th century, religious (2.) Incident of fire police station by annoyed
superstitions and social obscurantism was prevalent crowd
across the country. The 19th century was considered as (3.) Postponement of Movement by Gandhiji
the century of social & religious renaissance in India. (4.) Arresting of Gandhiji
The 19th century witnessed a series of socio-religious Select your answer to the following codes.

40th BPSC (Pre) Exam. 1995 30 YCT


(a) 1,2,3 and 4 (b) 2,1,3 and 4 Ans. (b) : Birsa Munda was a tribal leader & a folk hero
(c) 4,1,2 and 3 (d) 2,1,4 and 3 who belong to Munda Tribe. He became a mass leader
Ans. (a) : The Non cooperation Movement was and began to be considered as Bhagwan and Dharati
launched on 5th September 1920 by INC under the Aba by his followers. Anand Pandey was the Guru of
leadership of Gandhiji. The police fired upon congress Birsa Munda.
procession of approx 3300 personnels at Chauri Chaura 32. Identify the region where, santhals declared
in Gorakhpur. The angry crowd set the police station on his government in 1855-56
fire, killing 22 policemen on 4 February 1922, non- (a) Munger-Bhagalpur
cooperation movement was formally suspended on 10 (b) Bhagalpur-Rajmahal
March 1922 by Gandhi ji and later sentenced to 6 years (c) Gaya-Munger
(d) Shahabad-Gaya
imprisonment. He had been charged with sedition for
authoring three articles in the journal that he had started Ans. (b) : The Santhals are a tribal group concentrated
namely 'young India'. in Jharkhand and Bihar. The Santhal revolt took place
on 30th June 1855. The Santhals declared themselves
28. Who founded the organisation khudai-
free and declared their government in Munger-
Khidmatgar?
Bhagalpur in 1855-56.
(a) Abul Kalam Azad
33. Bihar Congress Socialist Party was formed in
(b) Khan Abdul Ghaffar Khan
(a) 1930 (b) 1931
(c) Inayatullah Mashriqi (c) 1934 (d) 1939
(d) Maulana Hasrat Mohani
Ans. (c) : Bihar Congress Socialist Party was formed in
Ans. (b) : Khudai Khidmatgar was a non violent 1934 when Jaiprakash Narayan convened meeting at
movement against British occupation of Indian Anjuman Islamia Hall in Patna. Acharya Narendra Dev
subcontinent. It was founder by Khan Abdul Gaffar was the first president and Jayprakash Narayan was the
Khan also known as Bacha Khan. general secretary of this party.
29. Who was the first Indian to be the member of 34. In the decade of 1830, Patna city was the hub of
Communist International. (a) Sanyasi Revolt (b) Godkhari Revolt
(a) M. N. Roy (b) Muzaffar Ahmad (c) Munda Revolt (d) Wahabi Movement
(c) S.S. Dange (d) None of these Ans. (d) : Wahabi Movement was started in around
Ans. (a) : The first Indian who was elected to the 1830. It was centered in Patna, Bihar. It was an Islamic
revivalist movement that emphasized condemning any
member of communist International in 1924 was M.N.
deviation from the original Islam & returning to its true
Roy. As a member of communist International, he spirit. The movement was led by Syed Ahmed Barelvi.
pondered Lenin on national liberation & operated in the 35. Identify the region of revolt of indigo farmer in
upper levels of International communism. Bihar in 1866-86
30. Two statements given below, first Assertion (A) (a) Muzaffarnagar and Chhapra
and second Reason (R), read both carefully. (b) Madhubani and Beguasrai
Assertion (A) : Rashtryia Swyam Sevak Sangh (c) Darbhanga and Champaran
was separated from the Quit India Movement. (d) Champaran and Muzaffarpur
Reason (R) : It thought that the freedom of Ans. (d) : Indigo revolt took place in 1859-60. It was a
India will be delayed due to this movement. revolt by the farmers against British planters who had
Concerning above mentioned both the forced them to grow indigo. Champaran &
statements which of the following is true? Muzzaffarpur were the main region of revolt in Bihar
(a) A and R both true and R explains A properly during 1866-86.
(b) A and R both true and R does not explain A 36. Best mica of the world is extracted
properly (1)from Dhanbad
(c) A is true but R is false (2) from hazaribagh
(d) A is true false but R is true (3) from Jharia
Ans. (c) : (A) is correct but (R) is false because (4) from Kanti
Rashtriya Swayam Sevak Sangh (RSS), Hindu Select your answer to the following codes.
(a) 1 and 2 (b) Only 2
Mahasabha communist party of India & Muslim league
(c) 2 and 3 (d) 2 and 4
put them separate from the Quit India Movement in of
Ans. (b) : India is one of the world's largest producer of
1942 and in the view of RSS the freedom can be
mica. Andhra Pradesh, Rajasthan and Bihar are the top
attained by defending "religion and culture" not by
3 mica producing states in the country.
fighting with British. Top quality of mica of the world in extracted from
31. Who was guru of Birsa Munda? Girdih & Koderma district of Jharkhand. Since in the
(a) Swami Sahajanand (b) Anand Pandey above given option there is neither koderma nor Girdih,
(c) Jatra Bhagat (d) M. N. Roy Hence, the most appropiate answer is Hazaribagh.
40th BPSC (Pre) Exam. 1995 31 YCT
37. Two statements are given below, first is 41. First Sugar Mill was setup in Bihar at
Assertion (A) and second is Reason (R) read (a) Marhaura (b) Betiah
carefully both. (c) Motihari (d) Patna
Assertion (A) : South bihar is more developed Ans. (a) : The first sugar mill was setup in 1904 at
than North Bihar Marhaura in the Saran district of Bihar.
Reason (R) : Development of industries 42. In production of crude copper in the country
occurred in South Bihar. Bihar ranks
With respect to both the statements, which of (a) First (b) Second
the following is true. (c) Third (d) Fourth
(a) A and R both are true and R explains A Ans. (a) : When the question was asked, Bihar was the
properly second largest producer of crude copper in the country
(b) A and R both are true and R does not explain At present Madhya Pradesh is largest producer of
A properly copper in India.
(c) A is true but R is false 43. If the Speaker of Legislative Assembly of a
(d) A is False but R is true state wants to resign, he will give his
Ans. (d) :North Bihar is more developed than South resignation to
Bihar and Development of Industries in South Bihar is (a) Chief Minister (b) Governor
more. Hence, Assertion is false but reason is true. (c) Deputy Speaker (d) President of India
38. Total geographical area of Bihar is Ans. (c) : Article 179 of the Indian Constitution is
(a) about 170 thousand km2 related to vacation and resignation and removal from
(b) about 172 thousand km2 the offices of Speaker and Deputy Speaker. It states that
(c) about 174 thousand km2 "A member holding office as Speaker or as per this
(d) about 178 thousand km2 article a member who holds office as Speaker may at
Ans. (c) : When the question was asked the total any time will give resignation to the Deputy Speaker
geographical area of Bihar was about 174 thousand and Deputy Speaker will give resignation to the
square kilometer. The total geographical area of present Speaker.
Bihar is 94, 163 km2 44. Right to dissolve Rajya Sabha is vested in
39. Which of the following rivers in Bihar merge (a) President (b) Vice-President
into the Ganges? (c) Supreme Court (d) None of these
(1) Gandak Ans. (d) : Rajya Sabha is a permanent body not subject
(2) Son to dissolution. However one third of its members retire
(3) Ghaghara every second year and are replaced by newly elected
(4) Punpun members. Each member is elected for a term of 6 year.
Codes. The Vice President of India is the ex-officio chairman
(a) 1,2,3,4 (b) 1,2,4 of Rajya Sabha.
(c) 1,3,4 (d) 2,4 45. In context of the preamble of Indian
Ans. (a) : Ganga is the main river of Bihar that enters Constitution which of the following sequence is
into the state from Chausa which formed the boundry of correct?
Bhojpur and Saran district. (a) Republic, democratic, socialist, universal
sovereign
Ghaghara river originates from the Tibetan plateau near
(b) universal sovereign, democratic, secular
Mansarovar in Nepal and enter in Bihar from
republic
Gopalganj. It joins river Ganga at Chhapra.
(c) universal sovereign, socialist, seculat,
River originates from the north of Dhaulagiri in Tibet
near Nepal. It flows in southern direction & forms a socialist republic
boundary between U.P. and Bihar. It joins the river (d) Universal sovereign, socialist, secular,
Ganga near Patna. democratic, republic
Son river originates in Madhya Pradesh. It merges into Ans. (d) : The Preamble of Indian Constitution has been
river Ganga near Patna. Amended by the 42nd Amendment Act 1976. There
Punpun river originates in the Palam district of were two changes made in the Preamble. Firstly, the
Jharkhand & joins Ganga at Fatuha, about 25 km down characterization of India as sovereign, democratic
stream of Patna. republic has been changed to sovereign socialist,
40. Chhotanagpur Plateau secular, democratic, republic and secondly the words
(a) Is a front sloping (b) Is a pitfall Unity of nation were changed to unity and integrity of
(c) Is a foothill (d) Is a peneplain the nation. Hence option (d) is in correct sequence.
Ans. (d) : Chota Nagpur plateau is located in north 46. A candidate of Lok Sabha Poll looses his
western Chhattisgarh & Central Jharkhand. It is security amount if he/she does not get votes
composed of Archean Granite and Gneiss rock with 1 1
(a) of valid votes (b) of valid votes
patches of Dharwar rock. Chhotanagpur plateau is a 3 4
peneplain. This plateau has huge reserves of iron, cocel 1
& Maganese. (c) valid votes (d) None of these
5
40th BPSC (Pre) Exam. 1995 32 YCT
Ans. (d) : As per Representation of People Act, 1951. It 51. Legislative Council in a state can be set up or
is mandatory for every contesting candidate either in abolished
Parliamentary or Assembly election to deposit a certain (a) by the Legislative Assembly of that state
amount as security. If the candidate gets lower than one- (b) by the Central Parliament
sixth of the total number of valid votes cast in the (c) by the Central Parliament on the
constituency his security deposit will be lost. recommendation of Governor
47. Council of Ministers is responsible to (d) by the President on the recommendation of
(a) President (b) Prime Minister Governor
(c) Speaker (d) Parliament Ans. (b) : Article 169 of Indian Constitution allows
Ans. (d) : As per Article 75 (3) of Indian Constitution parliament either the abolition or creation of legislative
the Council of Minister shall be collectively responsible councils in states, parliament can abolish a legislative
to the Lok Sabha. The main function of Council of council (where it already exists) or create it (where it
Ministers is to aid & advice to the President. The does not exist), if the legislative assembly of the
Council of Ministers of a state is collectively concerned states passes a resolution to that effect such a
responsible to the legislative assembly of the state. specific resolution must be passed by the state assembly
48. Presently the number of seats of each state in by a special majority.
Lok Sabha is allotted on the basis of 52. Prior to the completion of tenure, Lok Sabha
(a) Census of 1951 (b) Census of 1961 can be dissolved
(c) Census of 1971 (d) Census of 1981 (a) by the will of President
Ans. (c) : Article 81 of Indian Constitution states that (b) by Speaker
Lok Sabha shall not be more than 552 elected members (c) by the President on the recommendation of
of whom not more than 20 will be represent Union Council of Ministers
Territories. The distribution of seat in Lok Sabha on the (d) by the President on the recommendation of
basis of 1971 census and shall remain unaltered till the Speaker
first census to be taken after the year 2026.
49. Who is the leader of the Lok Sabha? Ans. (c) : Lok Sabha is not a continuing chamber. Its
(a) President (b) Prime Minister normal term is 5 years from the date of its first meeting
(c) Speaker (d) None of these after the general elections. Article 85 (2) (b) states that
Ans. (b) : Generally Prime Minister of India is the Lok Sabha can be dissolve before the completion of its
leader of Lok Sabha who are directly elected by people tenure. The President has power to dissolve the Lok
and at the time of electing Prime Minister one need not Sabha on the recommendation of Council of Ministers.
to be a member of either house, however within six 53. Legislative Council exists in which of the
months, such elected Prime Minister must become a following states?
member of either of Lok Sabha or Rajya Sabha. (1.) Kerala (2.) Himachal
50. Joint Session of both the houses of Parliament (3.) Delhi (4.) Bihar
is concerned Select your answer to the following codes
(1) for the election of the President of India (a) 1 and 4 (b) 1 and 2
(2) for the election of the Vice-President of (c) 2 and 3 (d) Only 4
India
(3) to pass the bill relating to the amendment Ans. (d) : At Present (2022) only six states that have
of constitution two houses (bicameral legislature). These are-Andhra
(4) to discuss on such a bill both the Houses Pradesh, Uttar Pradesh, Bihar, Maharashtra, Telangana
have different views and Karnataka.
Select your answer to the following codes 54. Which of the following Articles of Indian
(a) 1 and 4 (b) 3 and 4 Constitution Consists the provision of the
(c) 1 and 2 (d) Only 4 election of Legislative Assemblies of States?
Ans. (d) : As per the Indian Constitution the joint (a) Article 170 (b) Article 176
sitting of the Parliament is called by the President and is (c) Article 178 (d) None of these
presided over by Speaker or in his absence by the
Deputy Speaker or in his absence, The Deputy Ans. (a) : Article 170 of the Indian Constitution is
Chairman of Rajya Sabha. A joint sitting can be related to the configuration and provisions related to
summoned when if a bill is passed by one house and state of the legislative Assemblies. As per Article 170,
transmitted to the other house— there should be a legislative assembly in every states
1. The other House reject the bill or and the legislative assembly of state can have maximum
2. The other house do not agree on the amendments 500 constituencies and minimum 60 constituencies.
made to the bill or. These constituencies would be selected through the
3. More than 6 months lapsed with the bill being process of general election.
received by the other house without it being passed. 55. How many members of Parliament are there in
There is no provision in Indian Constitution for joint Lok Sahba from Bihar?
sitting for the election of President, Vice President and (a) 50 (b) 52
Constitutional Amendment Bill. (c) 54 (d) None of these
40th BPSC (Pre) Exam. 1995 33 YCT
Ans. (c) : When the question was asked, the number of Ans. (d) : The National Forest Policy has proposed that
MPs in the Lok Sabha from Bihar was 54. At present to strengthen the ecological balance there should be
there are 16 Rajya Sabha member & 40 Lok Sabha 33% area under the forest.
member are elected from Bihar. 63. Which country has the largest hard coal
56. Which of the following is the land-bounded reserves?
river? (a) Nepal (b) China
(a) Tapti (b) Krishna (c) New Zealand (d) India
(c) Luni (d) Narmada Ans. (b) : China has the largest hard coal reserves in the
Ans. (c) : Luni is a land-bounded river. It originates world. As of 2020 United States is the country with the
from the Naga Hills of the Aravali Range in Rajasthan's world's largest proven coal reserves.
Ajmer district. It passes through the southeastern 64. The region known as the Rice Bowl of India is
portion of the Thar desert and ends in the marshy land (a) Kerala and Tamil Nadu
of Rann of Kutch in Gujarat. (b) Krishna-Godavari Delta region
Narmada & Tapi are the major river in India that flows (c) North-East region
through rift valleys and drain into Arabian Sea. (d) Indus Gangetic plain
57. Monsoon retreat is marked by- Ans. (b) :Krishna-Godavari Delta region is called as
(1) clear sky "Rice Bowl of India" yet the same term is also used for
(2) high pressure condition in Bay of Bengal Chhattisgarh.
(3) rise in temperature on land 65. Arrange the following state of India in the
Select your answer to the following codes decreasing order of their area.
(a) Only 1 (b) 1 and 2 (1) Andhra Pradesh
(c) 1,2 and 3 (d) 2 and 3 (2) Bihar
Ans. (a) : The retreating southwest monsoon season is (3) Madhya Pradesh
(4) Uttar Pradesh
marked by clear sky and rise in temperature. The retreat
Codes:
of monsoon happen around September to December as
(a) 3, 2, 4, 1 (b) 1, 2, 3, 4
the winds which are the sources for the monsoon
(c) 4, 3, 2, 1 (d) 3, 4, 1, 2
becomes weaker and withdraw. The Sky will become
clear as the clouds slowly move away. Ans. (d) : The descending order of the states according
to area is Madhya Pradesh, Uttar Pradesh, Andhra
58. Which of the following river is longest in India? Pradesh & Bihar.
(a) Mahanadi (b) Godawari State Area
(c) Ganga (d) Narmada Madhya Pradesh 3,08,000 km2
Ans. (c) : Ganga is the longest river in India. Its origin Uttar Pradesh 2,40,928km2
is the Gangotri Glacier in Uttarakhand. It's total length Andhra Pradesh 1,60,205 km2
is 2525 km. Godavari is second largest river in India. It Bihar 94,163 km2
is popularly called as Dakshin Ganga. It is about 1465 66. Which of the following is the largest glacier?
km long. (a) Kanchenjanga (b) Rundun
59. Mormugao sea-port is situated in (c) Gangotri (d) Kedarnath
(a) Orissa (b) Tamil Nadu Ans. (c) :Among the above options, Gangotri glacier is
(c) Goa (d) Kerala the largest glacier in the Garhwal Himalaya and the
Ans. (c) : Mormugao post trust is in the coastal region source of river Ganga.
of Goa. This port is a natural open type harbour on the Siachen Glacier is the longest glacier in India. It is
mouth of Zuari River. located in the Karakoram range in the Himalayas.
60. Most important fishing industry in India is 67. On 8 October, 1991 Reserve Bank of India
(a) at deep water (b) at above sea-coast raised the bank rate from 11% to
(c) at cultural inland (d) at natural inland (a) 11.5% per annum (b) 12.0% per annum
Ans. (b) : India is one of the largest producers of fish, (c) 12.5% per annum (d) 13.0% per annum
both marine and inland. Most important fishing industry Ans. (b) : On 8 October 1991, Reserve Bank of India
in India is located at above sea coast. raised the bank rate from 11% to 12% per annum. Bank
61. Which one of the following is not a seaport rate is the rate charged by RBI for lending fund to
commercial bank.
city?
(a) Tokyo (b) Canberra 68. Most important source of income for state
(c) New York (d) London government in India is
(a) Land tax
Ans. (b) : Canberra is situated on the banks of lake (b) Agricultural Income tax
Burley Griffin. It is an inland city having no seaport. (c) Import Duty
whereas Tokyo, New York & London is a sea port city. (d) Sales tax
62. Essential to maintain ecological balance Ans. (c) : The most important source of revenue of state
minimum forest coverage is Government in India sales tax that has been replaced by
(a) 25% of total land (b) 50% of total land the Goods and services Tax (GST) started from 1 July,
(c) 40% of total land (d) 33% of total land 2017
40th BPSC (Pre) Exam. 1995 34 YCT
69. India is called an underdeveloped country Ans. (c) : As per census 1991, the population of India
because of its was 84.4 crore. As per census 2011, the total population
(1) necessity of employment of India is 121 crore.
(2) rapidly growing population 75. Average annual growth of non-development
(3) excessive dependence on agriculture outlay in Seventh Five Year Plan is
(4) slow rate industrial development (a) 12.35% (b) 1.8%
Select your answer to the following Codes: (c) 9.75% (d) 18.9%
(a) Only 1 (b) 1 and 2 Ans. (d) : Seventh Five year plan duration was from
(c) 2 and 3 (d) 2, 3 and 4 1985 to 1990 under the leadership of Rajiv Gandhi. The
Ans. (d) : India is a developing country thus it has main objectives of this plan was to establish growth in
features of an under developed economy like low per areas of increasing economic productivity, Production
capita income, rapid growth of population, excessive of food grains, and generating employment. Average
dependence on agriculture, slow rate of industrial annual growth of non-development outlay in 7th five
development, poor quality of human capital etc. year plan was 18.9%.
70. When was the Planning Commission set up? 76. Exemption limit of Income Tax in the budget of
(a) 1949 (b) 1950 1995-96 raised upto
(c) 1951 (d) 1952 (a) 35000 (b) 40000
Ans. (b) : Planning commission was established on 15 (c) 45000 (d) 50000
March 1950. It was mandated to formulate five year Ans. (b) : Exemption limit of Income Tax in the budget
plan for social & economical development in India. The of 1995-96 was raised up to ` 40,000. The basic
planning commission was replaced by newly formed personal tax exemption limit was last revised in 2014.
NITI Aayog in 2015. Present exemption limit is Rs. 2.5 lakh.
71. Who finally approves the draft of five year 77. In Union Budget of 1995-96 amount proposed
plans. for family welfare services directly mean for
(a) President rural areas is:-
(b) Planning Commission (a) 267 crore (b) 627 crore
(c) 726 crore (d) 762 crore
(c) National Development Council
(d) None of the above Ans. (c) : In the Union Budget of 1995-96, ` 726 crore
was proposed for family welfare services directly meant
Ans. (c) : National Development Council (NDC) was
for rural area.
set up on 6 August 1952. It is one of the key
organizations of the planning system in India. NDC is 78. What form of agriculture is in Bihar?
was an apex body for decision making and deliberation (1) Subsistence
of development matters in India. National Development (2) Commercial
Council (NDC) finally approved the draft of five year (3) Export-intensive
(4) Self-reliant
plans.
Select your answer to the following Codes
72. After independence India progressed (a) Only 1 (b) 2 and 3
maximum (c) Only 2 (d) Only 4
(a) In the production of rice Ans. (a) : Around 80% of population of Bihar is
(b) In the production of pulses employed in agricultural Sector. The agricultural
(c) In the production of jute economy in Bihar is still very much oriented towards
(d) In the production of wheat subsistence production.
Ans. (d) : After independence, India maximised the 79. Who is the Head of District Planning and
wheat production by adoption of Green revolution The Development Council?
green revolution in India was an effort to increase (a) Finance Minister of the State
agriculture production with the help of industrial (b) District Development Officer
agriculture technologies, such as HYV seeds, fertilizers, (c) District Planning Officer
pesticides & irrigation. (d) Chief Minister of State
73. National Income in India is calculated by Ans. (b) : District Planning and Development Council
(a) Planning Commission has been set up in all district under the deputy
(b) Finance Commission commissioner's office and is headed by the District
(c) Indian Statistical Institute Development Officer.
(d) Central Statistical Organisation 80. What does the joint sector in Indian Economy
Ans. (d) : Central Statistical Organization (CSO) under stand for?
the Ministry of Statistics and Programme (a) Dividend of Government in an enterprise is
Implementation is responsible for measuring National More than 60%
Income and other related macroeconomic aggregates. (b) A commodity is produced in both government
74. As per census of 1991, the population of India and private sectors
is (c) It is also called co-operative sector
(a) 80.2 crore (b) 82.2 crore (d) In an enterprise with combined ownership of
(c) 84.4 crore (d) 88.5 crore both government and private sectors
40th BPSC (Pre) Exam. 1995 35 YCT
Ans. (d) : The joint sector would include units in which (a) Excess deepness
both public & private investments have taken place and (b) Due to reflection of sky and scattering of light
where the State takes an active part in direction & by the drops of water
control. Maruti Udyog Limited is an example of joint (c) Because of blue colour of water
sector industry. (d) Because of upper layer of water
81. How many inducement (development) sectors Ans. (b) : The sea appears blue due to the absorption
are there in India? and scattering of light. The component of blue light has
(a) 5 (b) 6 shorter wavelength, higher frequencies & scattered
(c) 7 (d) 8 more compared to the red and other colours of the
Ans. (c) : There are 7 inducement (development) spectrum of visible light.
sectors in India. 88. Permanent hardness of water is due to
82. Sun takes time to revolve around the centre of (a) Calcium Carbonate
our galaxy is (b) Chlorides and Sulphates of Calcium and
(a) 5 crore year (b) 10 crore year Magnesium
(c) 20 crore year (d) 25 crore year (c) Chlorides and Sulphates of Sodium and
Potassium
Ans. (c) : Our Milky way galaxy is a very large. Milky (d) None of the above
way is a spiral galaxy. The diameter of Milky way is
Ans. (b) : Permanent hardness of water is due to the
around 100,000 light years. The galactic year is also
known as cosmic year, it is the duration of time required presence of chlorides and sulphates of calcium and
for the solar system to orbit around the centre of the magnesium. It can be removed by treating the hard
Milky way galaxy. The sun takes 25 crore year to water.
revolve around the centre of our galaxy. 89. The blood group of universal donor is
(a) A (b) AB
83. Which statement of the following is true for
(c) O (d) D
igneous rocks?
(a) They have very little fossils Ans. (c) : 'O–' blood group is called universal donor
(b) They are porous for water because in this blood there has no antigens present. AB+
(c) They are both crystalline and amorphous blood group is a universal acceptor because it has no
(d) These rocks have on silicas antibodies and has both antigens.
Ans. (c) : Igneous rock are formed by solidification of 90. Which of following is the Largest source of
magma (molten rock below the surface) & lava (molten alternative energy?
rock above the surface). Igneous rocks are formed from (a) Solar Energy (b) Tidal Energy
the crystalization of molten rock. These rocks are both (c) Atomic Energy (d) Land heat Energy
crystalline and amorphous. Ans. (a) : Solar Energy is the largest source of
84. Which of the foodgrains is richest in protein? alternative energy. Solar energy is capable in producing
(a) Gram (b) Peanut heat & electricity, wind, Tidal, Geothermal & Biomass
(c) Soyabean (d) Arhar energy. These are also known as non-conventional
Ans. (c) : In the given option Soyabean is the richest source of energy.
source of Protein. Soyabean is a complete protein, it 91. The bill of Electricity consumption based on
means that it contains all nine amino acids. the measurement of
85. Soil erosion can be checked by (1.) Watt (2.) voltage
(a) Excess grazing (b) Removal of plants (3.) Ohm (4.) Ampere
(c) Afforestation (d) None of the above Select your answer to the following Codes
(a) Only 1 (b) 1 and 2
Ans. (c) : Soil erosion is the natural process in which (c) 2 and 3 (d) 1 and 4
the topsoil of a field is carried away by physical process
Ans. (a) : The bill of electricity consumption is based
such as wind & water sources.
Soil erosion can be checked by aforestation. Roots of on the measurement of watt. A watt is a unit of power
trees or plant hold the soil. Thus when more trees are that measures the rate at which energy is generated or
planted, their roots do not allow the soil to be blown or consumed.
washed away thus prevent soil erosion. 92. As per present evidence, the origin of living
86. Flightless bird found in New Zealand is being on earth took place around 2 Lakh year
(a) Ostrich (b) Albetos ago. In this era protozoa, algal and bacteria
(c) Kiwi (d) None of these were-
(a) Before 20000 year
Ans. (c) : Flightless birds are principle features of New (b) Before 200000 year
Zealand's edge ecology. New Zealand has species of (c) Before 2000000 year
flightless birds including the Kiwi, several species of (d) Before 2000000000 year
Penguins, the Takahe, the Weka & the Moa and several Ans. (b) : As per present evidence, the origin of living
other extinct species. beings on earth took place around 200000 year ago. In
87. Sea appears blue- this era Protozoa, Algae and bacteria were before
due to excess depth 200000 years.
40th BPSC (Pre) Exam. 1995 36 YCT
93. Laughing gas is 101. Archaeopteryx is
(a) Nitrogen Oxide (a) Oldest bird of jurasic age
(b) Nitrogen Pentaoxide (b) Reptile of jurasic period
(c) Nitrogen Peroxide (c) Reptile of treasic period
(d) Nitrus Oxide (d) Reptile of both treasic and jurasic period
Ans. (d) : Nitrous Oxide (N2O) is commonly known as Ans. (a) : Archaeopteryx is the oldest bird of the
laughing or happy gas. It is a colorless non flammable Jurassic period, Archaeopteryx considers as a
gas. This gas is used in medical and dental procedures transitional fossil between dinosaurs and modern birds
as a sedative. It help to relieve anxiety before the with its blend of avian and reptile features, it was long
procedure and allow the patient to relax. viewed as earliest known bird.
94. Heaviest metal of following is 102. In the night wind blows fastly, dew is not
(a) Copper (b) Uranium formed because
(c) Aluminium (d) Silver (a) Rate of evaporation is fast
Ans. (b) : Among the above given options. Uranium is (b) Moisture in air is low
the heaviest naturally occurring element. It's atomic (c) Temperature is high
number in 92. (d) Sky is not clear
95. Radoactive substance emits Ans. (a) : Dew is not formed on the windy night
(a) Alpha rays (b) Beta rays because rate of evaporation is fast. The wind blow fast
(c) Gamma rays (d) All of these and the moist air does not come along enough into
Ans. (d) : Radioactive substances are atoms that decay contact with cold object to get cooled. So the
naturally. These substances are unstable and can give condensation does not take place and no dew can be
off alpha particles, beta particles and gamma radiation. formed.
96. Dolphins are classified in 103. Who won the Oscar Award of best film of
(a) Fish (b) Amphibians 1994?
(c) Reptile (d) Mammals (a) Star Van Saga (b) Fuzitiv
Ans. (d) : Dolphins are classified as cetaceans (marine (c) Zurasic Park (d) Forrest Gump
mammals) Although dolphins swim in water and appear
to be fish like compared to other animals living in Ans. (d) : Forrest Gump had won 6 Oscar Award
ocean, But they are classified as mammals because including Best Film, Best Actor in leading role and Best
dolphins also lactate and have mammary glands. Director at the 67th Academy Awards in 1994.
97. When we eat meat of goat or sheep, then we are 104. Which colour was commonly used in Harappan
(a) Primary pottery?
(b) Secondary consumers (a) Red (b) Blue-grey
(c) Tertiary consumers (c) Yellow (d) Blue
(d) None of these Ans. (a) : In the Harappan culture, the art of pottery
Ans. (b) : Goat, Sheep and other herbivores are primary making was generally made of red clay. Harappan
consumers. Animals which eat primary consumers they pottery consist of goblets, basin, flask, bowl, cylindrical
are called secondary consumers.
jars, tumblers, narrownecked vases etc.
98. Aspirin is
(a) Antibiotic (b) Antipyretic 105. Brian Lara got huge fame for crossing two
(c) Reliever (d) None of these records in world cricket. Two cricketers whose
Ans. (b) : Aspirin is a antipyretic. It is also called records he crossed are
acetylsalicylic acid. Aspirin is a common drug for (a) Garfield and Clive Loyd
relieving minor aches, pains and fevers. It is also used (b) Donald Bradman and Len Hutton
as an anti-inflammatory or a blood thinner. (c) Sunil Gavaskar and Alen border
99. What is detergent? (d) Garfield Sobers and Hanif Mohammad
(a) Soap (b) Medicine Ans. (b) : Brian Lara is a former West Indies cricket
(c) Catalyst (d) Refining agent team player. In 1994, He had scored 375 against
Ans. (d) : Detergents are refining agent. Detergent is a England and crossed the record of Sir Donald Bradman
substance or mixture containing soaps and surfactant and Len Hutton.
intended for washing & cleaning processes. 106. Full form of 'SAFTA' is
100. In view of life-cycle, the most important part of (a) South Asian Free Trade Area
plant is (b) Saarc Preferential Trading Agreement
(a) Flower (b) Leaf (c) South Asia Preferential Trade Agency
(c) Stem (d) Root (d) None of the above
Ans. (a) : From the life cycle point of view the most Ans. (a) : SAFTA stand for South Asian Free Trade
important part of a plant is flower. Flower is also called Area. It was signed in 2004 and came into force on 1
reproductive part of a plant as it carries the reproductive January 2006. It is a free trade agreement reached
organs. The main functions of flower are :- Pollination, between the members of the South Asian Association of
Reproduction and formation of fruits. Regional Cooperation (SAARC).
40th BPSC (Pre) Exam. 1995 37 YCT
107. The town of Biharsharif in medieval was 112. Which of the following statements relating to
important the composition named 'Triumph of the Spirit'
(1.) As centre of trade is true?
(2.) As centre of learning (a) Its author is Romi Dev
(3.) As centre of administration (b) It describes the sports life of Kapil Dev
(4.) As centre of religion (c) Its was formally released by famous artist M.
Select your answer to the following Codes F. Husain
(a) 1 and 3 (b) 1, 2 and 3 (d) All of the above
(c) 2, 3 and 4 (d) 2 and 4 Ans. (d) : Kapil Dev: "Triumph of the Spirit" is a book
written by Romi Dev. This book describes the sports
Ans. (d) : Biharsharif not only has a historical
life of Kapil Dev. This book was formally released by
importance for being the capital of Magadha empire renowned artist M.F. Husain, Hence all the statement
during the rule of Pala dynasty, but also for being the are correct.
educational and religious centre. On educational front, 113. Who among the following women tennis
the famous Nalanda University is situated here and on players once again ranked first recently?
religious front. It is a rehabilitation center for Sufi (a) Monica Shailes
Saints. (b) Steffi Graff
108. To which sect of Sufies was the famous saint of (c) Arancha Sanchej Vikario
Sharfuddin Maneri related? (d) None of these
(a) Chisti (b) Suhrawardi Ans. (b) : When the question was asked, Steffi Graff
(c) Firdausi (d) Kubrawi was ranked as the world No. l women tennis player. The
Ans. (c) : Shaikh Sharfuddin Yahya Maneri who is current world number one women tennis player is Iga
popularly known as Makhdoom-e-Jahan has lineage of Swiatek (Poland).
Sufi Saints. He belong to the Firdausi Sufi Silsilah. 114. Who won the best actress award in the
109. During last year, in which sector has the Filmfare Awards of 1995?
maximum expenditure of central Government (a) Madhuri Dixit (b) Dimpal Kapadia
of India- (c) Fareeda Jalal (d) None of these
(a) Defence (b) Development Ans. (a) : Madhuri Dixit had won Filfare Award in
(c) Administration (d) Subsidy 1995 in Best Actress category for the film Hum Aapke
Hai Kaun.
Ans. (b) : In Union Budget 2020-21 Defence sector has
115. What is Mohajir Qaumi Movement?
the highest allocation among the various sectors. The
(a) A terrorist organisation on Karachi
2021-22 Union Budget allocated over 13% of the total
(b) A political party of migrated muslims settled
budget to defence and it was ` 4,78,196 crore. in Pakistan
At the time, when question was asked 'Development' (c) A secret society of sunni muslims in Karachi
sector had the maximum expenditure of Government of (d) None of the above
India. Ans. (b) : Mohajir Qaumi Movement is a political party
110. Which of the following countries is the largest of migrated muslims settled in Pakistan. This party
purchaser of American weapons? claims to represent the Mohajir in Sindh, Pakistan
(a) Israel (b) Saudi Arabia whose leader is Afaq Ahmed.
(c) Taiwan (d) Pakistan 116. Who is the present chairman of Organisation
of Petroleum Exporting Countries (OPEC)?
Ans. (b) : When the question was asked, Saudi Arabia
(a) Dr. Alhaji Rilwanm Lukman
was the largest purchaser of American weapons. (b) Peter Gignau
111. What is the ratio of powers of National (c) Eda Bagas sedzave
Assembly and Senate respectively in the new (d) Dan Ateta
parliament of South Africa? Ans. (a) : Organisation of the Petroleum Exporting
(a) 400:80 (b) 410:90 Countries (OPEC) was created at the Baghdad
(c) 420:90 (d) 400:90 Conference in September 1960 by Iran, Iraq, Kuwait,
Ans. (d) : The National Assembly of South Africa Saudi Arbia and Venezuela. It aims to manage the
supply of oil in an effort to set the price of oil in the
consists of 400 members, elected by voters on the basis
world market. It is headquartered in Vienna. When
of proportional representation system. The Senate of question was asked Dr. Alhaji Rilwanm Lukman was
South Africa consist of 90 members, ten nominated by the chairman of OPEC. Prince Abdul Aziz Bin Salman
each of the nine province. is the current Chairman of OPEC.
40th BPSC (Pre) Exam. 1995 38 YCT
117. Who presided the G-7 summit held recently? Ans. (c) : Shako Asahara was the founder & leader of
(a) Jean Chretien Japanese doomsday cult known as Aum Shinrikyo. He
(b) Bill Clinton was convicted of masterminding & deadly 1995 sarin
(c) John Major gas attack on the Tokyo subway. He was also involved
(d) Jack Shirak in several other crime.
Ans. (a) : The 21st G-7 Summit was held in 1995 in 122. With which of the following countries in recent
Nova Scotia, Canada. It was presided over by the past did India sign a bilateral agreement that
aims to give protection and promotion to
Canadian Prime Minister Jean Chretien.
investment?
The 48th G-7 Summit is scheduled to be held from 26
(a) Germany (b) France
to 28 June 2022 in Germany. (c) Japan (d) Italy
118. Who was the president of jury board of Ans. (a) : On 10 December 1995 Germany & India has
'Cannes' film festival in 1995? singed the Bilateral Investment Promotion & Protection
(a) Clinton Eastwood Agreements.
(b) Catherine Denabue 123. The largest number of Tiger of the whole world
(c) Jeanne Moreau is in India. The approximate number is
(d) Shais Stone (a) 6000 (b) 16000
Ans. (c) : Jeanne Moreau was the President of jury (c) 60000 (d) 10600
board of Cannes film festival held in 1995. Ans. (*) : India is the home to nearly 70% of tigers in
In 2022, Vincent Lindon is the president of Jury board the world. As per tiger census report 2018 there are total
of Cannes film festival. 2967 Tigers in India.
124. Current winner of heavy weight category of
119. India Oceanists discovered a high mountain of
World Boxing Organisation?
height 1505 m at the bottom of Arabian sea (a) Bret Heart
about 455 km away South-West from Bombay (b) Shaun Mich
the mountain is called (c) Riddick Bowe
(a) Kailash II (d) George Lewis Gonzalekh
(b) Raman Sagar Mountain Ans. (c) : Riddick Bowe had defeated Herbie Hide and
(c) Kanya Sagar Parvat became the Heavyweight champion of world boxing
(d) Bombay Parvat organization in 1995.
Ans. (b) : Indian Oceanists discovered a high mountain 125. Who is the president of World Bank?
with a height of 1505 m at the bottom of Arabian sea (a) Lewis T. Preston
about 455 km south-west from Bombay, the mountain (b) Renarto Ragiyaro
called Raman Sagar Mountain. (c) Andru Yang
(d) Amest Stem
120. Recently which of the following states reduced
Ans. (a) : When the question was asked Lewis T
the price of coarse rice to ` 2 per kg?
Preston was the President of World Bank. He served
(a) Kerala this post from 1991 to 1995. At present David Malpass
(b) Andhra Pradesh is the President of World Bank group since 2019.
(c) Tamil Nadu 126. Recently Indian Women Cricket Team won the
(d) Jammu and Kashmir Three Nations Tournament held at
Ans. (b) : When the question was asked, Andhra (a) England (b) New Zealand
Pradesh had reduced the price of coarse rice to ` 2 per (c) Australia (d) Sri Lanka
kg. Ans. (b) : Indian Women's National Cricket team had
121. Shako Asahara has been highlighted in recent won an ODI tri-series in February 1995 by defeating
news broadcasting because New Zealand in the final.
(a) He is the representative of Japan in United This series was held in New Zealand.
Nations Organisation 127. Which of the following countries strongly
opposed the proposal of single currency for the
(b) He was chairman of Japanese Commercial
European Community?
Board talking with United States of America
(a) Germany
(c) He belongs to a religious sect of Japan who is (b) England
alleged for terrorist activities (c) France
(d) He represented Japan in G-7 Apex conference (d) Italy
40th BPSC (Pre) Exam. 1995 39 YCT
Ans. (b) : The country England had strongly opposed 132. Every year 5 Lakh Indians die of a disease.
the proposal of single currency for European Identify it-
Community. (a) Encephalitis
(b) Aids
128. Governor of Uttar Pradesh gave direction to
(c) Cancer
convene the session of Legislative Assembly of
(d) Tuberculosis
its speaker. For such action he used those
Ans. (d): Every year 5 Lakh Indian dies due to
rights which were vested in his post
Tuberculosis (TB). TB is a major health problem in
(a) By Article 172 India. It is caused by a bacterium called Mycobacterium
(b) By Article 175 tuberculosis. TB commonly affects the lungs.
(c) By Article 175(2) 133. Which of the following countries has been
(d) By Article 176 declared to be guilty of serious negligence of
Ans. (c) : Article 175(2) of Indian Constitution States human rights?
that the Governor may send message to the House or (a) Iran (b) Pakistan
Houses of the Legislature of the State, whether with (c) Turkey (d) All of these
respect to a bill then pending in the legislature or Ans. (d) : In the given options countries Iran, Turkey
otherwise, and a House to which any message is so sent and Pakistan had been declared to be guilty of serious
shall with all convenient dispatch consider any matter negligence of human rights.
required by the message taken into consideration. 134. Finally what is the irrigation capacity of Bihar
129. Green peace stands for (a) 89.20 lakh hectares
(a) an institution relating to agriculture (b) 90.30 lakh hectares
(c) 91.30 lakh hectares
(b) an international peace army
(d) 92.11 lakh hectares
(c) an international organisation of ecological
Ans. (a) : When the question was asked, the irrigation
supporters
capacity of Bihar was 89.20 lakh hectares.
(d) None of the above
135. What is the crop intensity in Bihar?
Ans. (c) : Greenpeace is an International Organization. (a) 108% (b) 118%
It was Founded in 1971. It uses non violent & creative (c) 128% (d) 138%
campaigns to highlight global environmental problems Ans. (c) : When the question was asked, Crop intensity
in order to find solution for a sustainable future for the in Bihar was 128%.
planet & the human race at large. At present the cropping intensity of Bihar is 138%.
130. Nova Scotia, the venue of recently held G-7 136. The longest living tree is
summit is located in- (a) Eucalyptus
(a) Sweden (b) Sequoia
(b) France (c) Deodar
(c) Switzerland (d) Parang
(d) Canada Ans. (b) : The longest living tree is Sequoia.
Ans. (d) : The 21 G7 summit was held in 1995 is 137. Where, the disputed Enron Project is situated?
st

Halifax, Nova Scotia, Canada. (a) Bombay


Group of Seven (G7) is an intergovernmental (b) Dabhol
organization and political forum consisting of France, (c) Agrawali
UK, Canada, Germany, Italy, Japan & U.S. are the G7 (d) Nander
Countries. Ans. (b) : The Enron project is situated at Dabhol in the
131. Which of the following books is banned by the state of Maharashtra.
Jammu and Kashmir Administration? 138. What is 'Nashak'?
(a) Muslim Law and Constitution (a) A new missile developed by India to fight in
(b) Farhange Asfia air to air
(c) Tahrike Mujahiddin (b) A new missile developed by India to fight in
(d) Atshe chinar air to surface
Ans. (c) : When the Question was asked, Tahrik-e- (c) A new missile developed by India to fight in
Mujahideen book was banned by the Jammu & Kashmir surface to surface
Administration. (d) A missile boat developed by India

40th BPSC (Pre) Exam. 1995 40 YCT


Ans. (d): INS Nashak was a Vidyut class missile boat 210000
of Indian Navy. It was commissioned on 19 March 1971 = = 26250 Rs.
8
and was decommissioned on 31 December 1990. INS 143. In a class Test X and Y obtained following
Nashak served in the Indo-Pak war of 1971. marks
139. What was the rate of land revenue as given in Subject X Y Total Marks
Dharma Shastras? English 84 92 100
1 1 Hindi 80 79 100
(a) (b)
3 4 Math 90 88 100
1 1 History 69 60 100
(c) (d)
6 8 Who obtained more marks and how much
Ans. (c) : Rate of land revenue as given in Dharma (a) X obtained 1% more marks
Shastra was 1/6 of the total produce. (b) X obtained 2% more marks
140. All India State People's Conference was set up (c) Y obtained 1% more marks
(a) In 1924 (d) Y obtained 2% more marks
(b) In 1926 Ans. (a) : Obtain total marks of X =84+80+90+69 =323
(c) In 1927 Obtain total marks of Y = 92+79+88+60 = 319
(d) In 1929 323 323
Ans. (c) : All India State Peoples Conference was a Obtain marks percentage of X = ×100 = %
400 4
conglomeration of political movements in the princely
319 319
states of British India variously called Praja Mandals or Obtain marks percentage of Y = ×100 = %
Lok Parishads. It was setup and the first session of this 400 4
organization was held in Bombay in December 1927. So, X got more mark percentage
This organization was dissolved on 25 April 1948. 323 319  323 − 319 
= − = 
141. If DECIDE is coded with 453945, then ABIDE 4 4  4 
will be coded with 4
(a) 94521 = , = 1 % more.
4
(b) 49521
144. If x% of y is y% of z, then z is equal to
(c) 12945
(a) x (b) y
(d) 49951
x y
Ans. (c) : Given, (c) (d)
DECIDE 100 100
| | | | | | Ans. (a) : According to the given conditions,
4 5 3 9 4 5 (Numeral value of letter) ⇒ x% of y = y % of z
Then ⇒ (x/100) y = (y/100)×z
ABIDE ⇒x=z
| | | | | Hence option (a) is the correct answer.
12945
145. A train X starts from a place at the speed of 50
thus, option (c) 12945 is the correct Answer. km/h. From the same place another train Y
142. Average annual income of 10 teachers is ` starts after one hour at the speed of 70 km/h.
25000 and of the two teachers, annual income How much time do the train Y will take to pass
of each is ` 20000, then average annual income the train X?
of rest teachers is (a) 3 h
(a) ` 26250 3
(b) ` 25500 (b) 2 h
4
(c) ` 23200 (c) 2 h
(d) ` 2200
1
Ans. (a) : Average income of rest teacher is – (d) 2 h
2
25000 × 10 − 2 × 20000
= Ans. (d) : Let train Y cross the train X after time 't after
8 time (t) the distance travelled by train Y = 70 × t and
250000 − 40000 distance travelled train X = 50 (t + 1) [since train X
=
8 starts 1 hours before]

40th BPSC (Pre) Exam. 1995 41 YCT


70 t = 50 (t + 1) ⇒ 70 t – 50 t = 50 15
∴ distance covered in 1 round =
m
50 1 2
20 t = 50 ⇒ t = ⇒ 2 hrs
20 2 15
hence distance covered in 7 round = ×7
146. If 10 labours dig a well in 4 days, then how 2
many labourers will be required to the same = 105 m
well in half day? 2
(a) 5 (b) 40 as per question
(c) 60 (d) 80 105
to cover m, required time is 9 sec.
Ans. (d) : As we know, 2
work done = man × day  105 
 
(W = M × D) Hence speed of train =  2  m / sec
Given, 9
work → dig a well  105 
=   m / sec
Labours (M) = 10  2×9 
Days (D) = 4 105 18
= × km / hr
If same well digs in half day (1/2 day) 2×9 5
then M = ? = 21 km/hr.
as 149. Measurement of an Wooden block is 5×10×20
W 1 = M 1 × D1 cm. How many such complete blocks will be
Since (W1 = W2) [same well] needed to make a solid cuboid of minimum
M 1 × D1 = M 2 D2 measurement?
10 × 4 = M2 × (1/2) (a) 6 (b) 8
M2 = 10 × 4 × 2 (c) 12 (d) 16
M2 = 80 Ans. (b) : Given,
Hence if same well digs is 1/2 day then there will be 80 Dimension of wooden block = 5×10×20
labours. Hence the side of smallest cube by using this wooden
147. The population of a city increases at annual block should be = 20 cm
rate of 4% and due to refugees increase at the Hence volume of cube = 20 × 20 × 20 cm3
rate of 1% additionally, then what will be the & volume of wooden block = 5 × 10 × 20
increase in population after two years? 20 × 20 × 20
(a) 10% Hence requirement of minimum block =
5 × 10 × 20
(b) 10.25%
=8
(c) 10.50%
150. During ascending on a minaret of 180 upstairs
(d) 10.75%
of a person relaxes for 2 minutes after
Ans. (b) : Percentage increase in one year = (4+1) = 5%
ascending 30 upstairs. Till he reached above
The increase in population after two years
the minaret how long did be relax?
5×5 (a) 30 min
5+5+ = 10.25%
100 (b) 12 min
148. The wheel of an engine having its perimeter (c) 10 min
1 (d) 9 min
7 m moves 7 round in 9 s, then speed of train
2 Ans. (c) : Before climbing the 180th upstairs a man rests
km per hour will be five times at an intervals of 30-30 steps i.e. after
(a) 15 (b) 21 crossing the 30th, 60th, 90th, 120th and 150th stairs the
(c) 30 (d) 35 takes a rest of 2 minute each. Hence it is clear that after
Ans. (b) : Given, reaching 180th upstairs the man will reach his
1 destination hence time taken to rest 5 times for 2
Perimeter of wheel = 7 m minutes. That is equal to 5 × 2 = 10 minutes
2
40th BPSC (Pre) Exam. 1995 42 YCT
41th Bihar Public Service Commission
Preliminary Examination, 1996
GENERAL KNOWLEDGE & GENERAL SCIENCE
(Solved Paper with Detail Explanation)
1. Which was the main cause of the failure of the Ans. (c) : In Vaikom Satyagraha (from 1924-25) which
Revolt of 1857? occurred in Travancore against untouchability, Gandhiji
(a) Lack of Hindu-Muslim unity did not participated directly though, he was invited by
(b) Lack of common strategy and central
organisation T. K Madhawan when he met Gandhiji at Tirunelveli on
(c) Its effect was within limited area 23rd Sept. 1921.
(d) Non-participation of landlords 4. Bardoli Satyagraha was led by
Ans. (b) : Causes of failure of the Revolt of 1857 (a) Mahatma Gandhi
(i)The Revolt was localized, restricted and poorly (b) Vallabhbhai Patel
organized. (c) Morarji Desai
(ii)The resources of the British Empire were far superior (d) Dr. Rajendra Prasad
to those of the rebels.
(iii)The rebels had no common goal before them except Ans. (b) : The Bardoli Satyagraha of 1928 was a
the anti-foreign sentiments. movement launched by Sardar Vallabhbhai Patel for
(iv)Lack of common strategy and central organization. the peasants of Bardoli against the unjust raising of
2. Match the names mentioned in Column I with taxes.
the names mentioned in Column II and selected 5. Indian National Congress opposed the Rowlatt
the right answer using the codes given below. Act.
Column-I (Books) Column-II (Authors) (a) to limit the individual liberty
A. History of the 1. Tara Chandra (b) to ban on Indian National Congress
Freedom Movement in
India (c) to enlarge the communal delegations
B. History of the 2. WW Hunter (d) to imprison national leaders charging them
Freedom Movement in traitors to the nation
Bihar Ans. (a) : The Rowlatt Act, 1919, was apposed by
C. Anandmath 3. Raja Ram Mohan Roy Indian National Congress as the Act gave immense
D. Precepts of Jesus 4. K K Dutta power to the British Police officers. It said that, Officers
E. Our Indian 5. Bankim Chandra
could arrest any Indian anytime and anywhere without
Musalmans Chatterji
Codes any arrest warrant. This roused inconvenience among
A B C D E the Indians and so the Rowlatt Act was opposed by
(a) 1 4 5 3 2 INC.
(b) 2 4 5 3 1 6. Swami Vivekanand established Ramkrishna
(c) 1 5 4 3 2 Mission in the year
(d) None of the above (a) 1916 (b) 1891
Ans. (a) : The correct matches are follows as:- (c) 1893 (d) 1897
(a) History of the Freedom Movement in India- Tara
Chandra Ans. (d) : Ram Krishna Mission was founded by
(b) History of the freedom movement in Bihar – K.K. Swami Vivekanand in 1897. The headquarter of
Dutta. Ramkrishna Mission is located at Belur Math in West
(c) Ananda Math- Bankim Chandra Chattopadhyay Bengal. The Mission is named after and inspired by the
(Chattergee) Indian saint Ram Krishna Paramhansa. Ramkrishna
(d) Precepts of Jesus - Raja Ram Mohan Roy. Paramhansa was the Guru (teacher) of Swami
(e) Our Indian Musalmans- William Wilson Hunter.
Vivekananda.
3. In which of the following Satyagraha
Movement did Gandhiji not participated 7. After passing the resolution of 'Quit India'
directly? Gandhiji was arrested
(a) Rajkot Satyagraha (a) in Yarvada jail
(b) Khera Satyagraha (b) in Nainy Jail
(c) Vaikom Satyagraha (c) in the fort of Ahmednagar
(d) Non-Cooperation Movement (d) in Aga Khan Palace

41th BPSC (Pre) Exam. 1996 43 YCT


Ans. (d) : On 8th August, 1942, Mahatma Gandhi gave Ans. (b) : During the Quit India movement in 1942,
a clarion call to end the British rule and launched the Mahatma Gandhi gave the slogan of "Do or Die.
Quit India Movement at the session of the All-India Gandhi ji gave the call “Do or Die’’ in his speech
Congress Committee in Mumbai. After passing the delivered at the Gowalia Tank Maidan, now popularly
resolution Gandhi, Nehru and many other leaders of the known as August Kranti Maidan.
INC were arrested by the British Government. Gandhiji 13. Who was the secretary of Mahatma Gandhi
was detained at the Agha Khan Palace in Pune. Other during Noakhali?
leaders were sent to Jail in Ahmednagar Fort. (a) Nirmal Kumar Bose
Note:- The slogan ‘Quit India’ was coined by Yusuf (b) Mahadeo Desai
Meherally. Meherally also coined the slogan “Simon Go (c) Pyare Lal
Back”. (d) Vallabh Bhai Patel
8. On which issue did the second Round Table Ans. (c) : Pyare Lal was the secretary of Mahatma
Conference fail? Gandhi during Noakhali communal violence. His sister
(a) Communal Delegation Dr. Sushila Nayar was personal doctor of Mahatma
(b) Granting Dominion Status Gandhi.
(c) The date of transfer of ruling power 14. Who was elected the president of Indian
(d) Postponement of Civil Disobedience National Congress in 1938?
movement (a) Abul Kalam Azad
Ans. (a) : The second Round table conference that took (b) Subhash Chandra Bose
place in September - December, 1931, was attended by (c) Pattabhi Sitaramayya
the Congress representative but it failed over the (d) J B Kripalani
question of Communal representation. Ans. (b) : Subhash Chandra Bose was elected as the
9. The best representative of Neo-Hinduism in President of Haripura session of Indian National
second half of nineteenth century was Congress for the year 1938. He was one of the most
(a) Ramkrishna Paramhans eminent freedom fighters of India.
(b) Swami Vivekanand 15. In which context did Jaipraksh Narayan got
(c) Bankim Chandra Chatterji the recognition of National leader?
(d) Raja Ram Mohan Roy (a) Quit India Movement
(b) Eatablishment of Congress Samajwadi Dal
Ans. (b) : Neo-Hinduism also called as Neo-Vedanta (Party)
and Hindu Universalism are the terms to characterize (c) Bhoodan Movement
interpretations of Hinduism that developed in 19th (d) has becoming the member of Congress
century. The best representative of Neo-Hinduism was Executive
Swami-Vivekananda.
Ans. (b) : In 1934, 'Loknayak' Jai Prakash Narayan
10. Home Rule Movement was symbol of starting a established the 'Socialist Party of Congress' which got
new phase of freedom movement in India him the recognition of National leader. He is also
because known as the "Hero of quit India Movement.
(a) It put a perfect project for the self- 16. Who was the last Governor-General of
government before the nation independent India?
(b) Leadership of the movement came in the (a) Lord Mountbatten (b) C Rajagopalachari
hands of Gandhiji (c) Rajendra Prasad (d) None of these
(c) Hindus and Muslims started struggle jointly Ans. (b) : Chakravarti Rajagopalachari was the first and
(d) It established co-ordination between last Governor-General of India after Independence.
extremists and moderates Chakravarti Rajagopalachari was popularly known as
Ans. (a) : In 1916, the Home rule movement was 'Rajaji'. After Independence, Rajagopalachari took over
started by Annie Besant along with Bal Gangadhar as the Chief Minister of Madras in April, 1952.
Tilak.In 1916, Annie Besant along with Bal Gangahar 17. After the failure of Civil Disobedience
Tilak started the Home Rule Movement. The Home Movement, Gandhiji gave importance to
Rule League demanded self government to the Indians. (a) constructive programmes
The correct answer is (a). (b) limited use of violence
11. Gadar Party was established in the year (c) negotiation with British
(a) 1907 (b) 1913 (d) None of the above
(c) 1917 (d) 1920 Ans. (a) : After the failure of Civil disobedience
Ans. (b) : The Gadar party was established under the movement Gandhiji gave importance to Constructive
leadership of Lala Hardayal with Baba Sohan Singh programmes.
Bhakna as it’s President. The party was formed on 18. During 1923-28, the repetition of revolutionary
November, 1913, at San Francisco in the USA. activities in Indian politics was due to
12. The slogan 'Do or die' was related with (a) rising influence of leaders like Hardayal and
(a) Lala Lajpat Rai Lajpat Rai
(b) Mahatma Gandhi (b) Postponement movement by Gandhiji
(c) Bal Gangadhar Tilak (c) influences of foreign events
(d) Subhash Chandra Bose (d) refusal of Indian demand by Britishers
41th BPSC (Pre) Exam. 1996 44 YCT
Ans. (b) : The Revolutionary movement during 1923- (a) Near by 0º latitude
28 was opposed to the generally peaceful civil (b) Near by 23º latitude
disobedience movement led by Gandhi ji. When (c) Near by 50º latitude
Gandhi, postponed the movement the repetition of (d) Near by 70º latitude
revolutionary activities occurred periodically in Indian Ans. (b) : Desert is a barren land on the earth's surface
Politics. where evaporation is greater than precipitation and most
19. Normal temperature of Human body on of the desert in the world are found at the western side
Celsius scale will be of continent. The possibility of finding a desert on the
(a) 310 degree (b) 98.4 degree Earth is more nearby 23º latitudes in both Hemisphere.
(c) 36.9 degree (d) 31.5 degree These are hot deserts, mostly found near the Tropics of
Ans. (c) : The Normal temperature of Human body on Cancer and Capricorn, between 15º-30º north and south
Celsius scale is 36.9ºC which is 98.6ºF at Fahrenheit of the Equator.
scale. 25. The height of ozone layer above the surface of
20. The largest planet of solar system is earth is
(a) Earth (b) Mars (a) 10-20 km (b) 40-50 km
(c) Jupiter (d) Saturn (c) 70-80 km (d) 110-120 km
Ans. (c) : The largest planet in our solar system is Ans. (b) : The height of ozone layer above the surface
Jupiter. Its mass is 300 times more than that of the Earth of earth is nearly 40-50 km. The ozone layer exist in the
and its diameter is 1,40,000 km about 11 times of the stratosphere of the atmosphere. Ozone layer protects us
Earth's diameter. from the dangerous UV rays coming from the sun
21. The gas, the mass of which is highest in the air, which causes skin cancer and other health damages.
is 26. In respect of women, men can suffer more of
(a) Hydrogen (b) Carbon dioxide colour blindness because
(c) Oxygen (d) Nitrogen (a) They pass more through mental tensions
Ans. (d) : The three major constituents of air are (b) They live out of home relatively more
Nitrogen, oxygen and Argon. Out of these three, (c) They have only one X-chromosome
Nitrogen gas constitutes about 78% by its volume (d) Commonly they have less fat
percentage. Hence, mass of Nitrogen gas is highest in Ans. (c) : The colour blindness is a colour vision
the air. deficiency which is unable to differentiate the colour as
22. Isotopes is that nuclei of atomic nucleus in Red & Green. The most common cause of colour
which? blindness is an inherent fault. Males are more likely to
(a) Number of neutrons is same but number of be colour blind than females because the genes
protons different responsible for colour blindness are located on the
(b) Number of protons is same but number of chromosome. Males have only one X-chromosomes.
neutrons different whereas females have two ‘X’.
(c) Number of both protons and neutron are same 27. Active chemical to remove mosquitoes is
(d) Number of both protons and neutrons are (a) Pyrethrin
different (b) Atropin
Ans. (b) : Isotopes are atoms of element with the same (c) 2-iso propoxiphinyle
number of protons but different number of neutrons and (d) Benzene Hexaclorophin
each isotope of a given element has a different Mass Ans. (a) : The active chemical used to remove
number but having same Atomic number which is mosquitoes is Pyrethrins or pyrethroids which are
determined by their number of Protons present in the chemically derived from chrysanthemum flowers that
nucleus. are toxic to insects.
23. In winter you touch a log of wood and a rod of 28. The surface of earth cover with water is
iron you will feel rod of iron cold because approximately
(a) the heat of rod of iron is less than the heat of (a) one-fourth (b) half
log of wood (c) two-third (d) three-fifth
(b) in comparison to wood, iron is good Ans. (c) : The surface of earth covered with water's
conductor of heat approximately 2/3rd part of water and 1/3rd part with
land.
(c) in comparison to wood, iron is bad conductor
of heat 29. The group of stars that indicate the direction of
pole, is
(d) the rod of iron is heavier than the log of wood
(a) Saptarishi (b) Mrig
Ans. (b) : Since, we know that Iron is a good conductor (c) Vrischik (d) Vrikh
of heat, therefore in winter a rod of iron get's more Ans. (a) : The group of stars that indicate the direction
colder than a log of wood. Because a log of wood is a of pole is Saptarishi. It is also known as Great or Big
bad conductor of heat. Bear. It Includes 7 stars that's why its name is
24. The possibility of finding a desert on earth is saptarishi. The pole star indicates always in the North
more direction.
41th BPSC (Pre) Exam. 1996 45 YCT
30. Local time of Patna is Ans. (c) : The S.I. unit of focal length of a lens is
(a) ahead of Indian Standard Time Diopter. The convex lens has positive capacity while
(b) behind the Indian Standard Time concave lens has negative capacity.
(c) same that of Indian Standard Time
(d) no related with Indian Standard Time we know that.
1 1
Ans. (a) : Local time of Patna is ahead of Indian P = ⇒ 2 = {∵ P = +2D)
Standard Time. In India, Standard Time Meridian f f
1 100
passes through 82 º E longitude which passes through ⇒ f= = 50cm
2 +2
Mirzapur district near Prayagraj in Uttar Pradesh. Since, 36. If a bulb of 100 W burns for 10 h, the
The longitude of Patna is 85º9' E longitude which is
expenditure of electricity will be
approximately 3º ahead of Indian standard time. Hence,
It is clear that the local time of Patna is ahead of Indian (a) 0.1 units (b) 10 units
1º (c) 1 units (d) 100 units
Standard Time (IST) i.e. 82 E. Ans. (c) : Power of bulb = 100 W
2
31. Constituents of atomic nucleus are Time = 10 hours
(a) electron and proton we know, Energy = Power × Time
(b) electron and neutron ⇒ E = 100 w × 10 hrs. = 1000 w = 1 kwh. = 1 unit.
(c) proton and neutron 37. At the age of 20 years, the number of bones in
(d) proton, neutron and electron human body is approximately
Ans. (c) : The main constituents of an atomic nucleus (a) 100 (b) 200
are Proton and Neutron while electrons lie in the outer (c) 300 (d) None of these
orbit of an atom. Only protons and neutrons lie inside
the nucleus. The sum of proton and neutron determines Ans. (b) : The number of bones in human body at the
the Atomic Mass number of an element. age of 20 year is about 206. In the early childhood it is
32. Heavy water is that water equal to 270. Among these stapes is the smallest bone in
(a) the temperature of which is kept at 4ºC the Human body.
(b) in which soluble salts of calcium and 38. The disease caused by HIV is
potassium are found (a) Tuberculosis (b) Dysentry
(c) in which isotopes replace hydrogen (c) Cancer (d) AIDS
(d) in which isotopes replace oxygen
Ans. (d) : The HIV stands for "Human Immuno-
Ans. (c) : Heavy water is also called deuterium oxide
and is denoted by the chemical formula, D2O. Thus, Deficiency virus. It can spread through sexual contact,
Heavy water is that water in which Hydrogen is illicit injection or sharing needles or contact with
replaced by its isotopes. It is used in nuclear reactor as a infected blood. Primarily, HIV attacks the body's
moderator to slow down the neutrons speed. immune system. If HIV is not treated, it can lead to
33. Dense forests on earth are mostly found AIDS. HIV destroy the white blood cells which play a
(a) nearby equator major role in helping your body to fight with diseases.
(b) nearby tropic of cancer 39. Which part of Bihar was unaffected in the
(c) nearby tropic of capricorn Revolt of 1857?
(d) nearby poles 1. Danapur 2. Patna
Ans. (a) : Since equatorial region receives the 3. Area 4. Muzaffarpur
maximum precipitation throughout the year that’s why 5. Munger
very dense forest are found there. Select your answer from the following codes.
34. Which one of the following is not language of (a) 4 and 5 (b) Only 5
computer? (c) Only 4 (d) 3, 4 and 5
(a) BASIC (b) C++ Ans. (a) : The part of Bihar which were unaffected of
(c) FAST (d) FORTAN the Revolt of 1857 were Muzaffarpur and Munger.
Ans. (c) : FAST is not a language of computer. A While Arrah, Patna and Danapur were the important
computer language is a medium of communication places where the revolt took place. In Bihar, this revolt
between the different components of computer and its is led by Veer Kunwar Singh from Jagdishpur.
software. Some prominent computer language are 40. Which area was affected widest with the
FORTRON, communal riots due to 'Quit India' Movement?
C, C++, PYTHON, JAVA, ALGOL etc (1.) Bihar (2.) Bengal
35. If the lens power of glass is +2 diopter, its focal (3.) Gujarat (4.) Joint State
length will be Select your answer to the following codes.
(a) 200 cm (b) 100 cm (a) 1 and 2 (b) Only 2
(c) 50 cm (d) 2 cm (c) 2 and 3 (d) 1 and 4
41th BPSC (Pre) Exam. 1996 46 YCT
Ans. (d) : Bihar and United Province area were the 46. Word Athletic Tournament of 1995 was held at
regions which were most affected by the communal (a) Moscow (b) Stockholm
riots due to 'Quit India' movement. (c) Madrid (d) None of these
The 'Quit India' movement was launched on 08 August Ans. (d) : In 1995 the World Athletic Tournament was
1942, at Gowalia Tank in Bombay, where Gandhiji held at the Ullevi stadium, Gothen burg, Sweden on 5-
proclaimed his mantra "do or die". It is also known as 13 August 1995. Recently happened in Eugene, Oregon.
August movement. It was launched at the Bombay 47. Second Lady to reach the Peak of Everest was
session of the All India Congress Committee by (a) Bachendri Pal (b) Santosh Yadav
Mahatma Gandhi. (c) M Srilekha (d) None of these
41. 'Magahar Mahotsav' was started in respect of Ans. (b) : The first Indian lady to reach the peak of Mt.
Saint Kabir in the year. Everest was Bachendri Pal in 1984. While Santosh
(a) 1987 (b) 1985 Yadav became the second Indian lady who climbed up
(c) 1990 (d) 1997 the summit of Mt Everest. She is first women in the
Ans. (c) : The Magahar Mahotsav is an annual cultural world to scale Mt. Everest twice.
festival held at Magahar in Sant Kabir Nagar, district 48. The Hindi novel selected for the Sahitya
Uttar Pradesh, since 1990, in memory of famous saint Academy Award of 1983 was
Kabir. It is held every year in January, Magahar is the (a) Ardhnarishwar
birth place of Sant Kabir. (b) Neela Chand
42. According to the final census report, the (c) Main waqt ke hun samne
population growth rate of India was (d) None of the above
(a) 129.9% (b) 84.25% Ans. (a) : Hindi novel selected for the Sahitya Academy
(c) 25.4% (d) None of these award of 1993 was 'Ardhnarishwar' written by most
prominent writer of Indian literature "Vishnu
Ans. (d) : In 1872 AD, the first non-synchronous Prabhakar". In 2021, Daya Prakash Sinha won it for
census was conducted in India under the rule of Lord 'Samrat Ashok'.
Mayo. The first synchronous census in India was 49. The award given to Gnana Rajasekaran for his
conducted in 1881 under lord Ripon. (It is done after tamil film 'Moga Mul' was
every 10 year) (a) Dada Saheb Phalke Award
According to final census report 2011, the population (b) Best Actor National Award
growth rate of India is 17.5%. (c) Indira Gandhi Award for best film
The census year of 1921 is called the year of “The Great (d) Filmfare award
divide” in the demographic history of India due to a Ans. (c) : Mogamul is a Tamil language film of 1995
decadal population decline of 0.31%. which was directed by Gnana Rajasekaran based on a
43. The country which the men's single Indonesian novel written by Thi. Jankiraman. He had won the
Open Badminton Championship of 1995 was Indira Gandhi Award for best film. In 2019, Helen
(a) Iraq (b) Russia (Malayalam) film won this award directed by 'Muthu
Kutty'.
(c) Iran (d) South Korea
50. The 73rd Amendment of Indian Constitution
Ans. (a) : In 1995, Aardi Wiranta was the winner of provides
men's single Indonesian open Badminton. (a) first time of Panchayati Raj
championship. Victor Axelsen of Denmark won it in (b) removal of administrative control over
2022. Panchayats
44.The winner of 11th Jawaharlal Nehru Soccer (c) changing of laws regarding election of
Tournament was panchayats
(a) Iraq (b) Russia (d) making panchayat election directive and
(c) India (d) Saudi Arabia equivalent
Ans. (a) : Iraq was the winner of 11th Jawahar Lal Ans. (c) : The 73rd Amendment of Indian constitution
Nehru soccer tournament. It was held in Kolkata, India. came into force on 24th April, 1993. This Act
India won it recently, hosted in New Delhi. empowered the state government to take the necessary
45. Who won the single tournament of women in steps that would lead to the formalisation of the Gram
Panchayat and to mould the method which are earlier
US Open Tennis Championship of 1996?
used in the panchayat election.
(a) Monica Seles It empowered the panchayat function as an institution of
(b) Steffi Graf self Government. This Act added a new chapter into the
(c) Martina Hingis Constitution called Part IX: "The Panchayat"
(d) Arantxa Sanchez Vicario 51. The author of 'The Moor's Last Sigh' is
Ans. (b) : Steffi Graf was the winner of singles (a) Peg tyre (b) Salman Rushdie
tournament of women in U.S. open Tennis (c) PG Woodhouse (d) Mary Higgins Clark
championship of 1996. It was held from August 26, to Ans. (b) : The Moor's Last Sigh is a historical fiction
September 08 1996. Emma Raducanu won 2021 US written by Salman Rushdie. This novel is his fifth novel
Open. and was published in 1995.

41th BPSC (Pre) Exam. 1996 47 YCT


52. From last few years Buddhist monks in Bihar Ans. (d) : The APEC stands for - "The Asia-Pacific
are agitating to have control over Economic Cooperation" is a regional economic forum
(a) Mahabodhi Temple of Gaya established in 1989 to leverage the growing
(b) Vishnupad Temple of Gaya interdependence of the Asia-Pacific countries. APEC'S-
21 member aim to create greater prosperity for the
(c) Sun Temple of Deo
people of the region by promoting balanced, inclusive,
(d) None of the above sustainable, and innovative growth by accelerating
Ans. (a) : Buddhist monks in Bihar have been agitating regional economic integration.
from last few years to have control over the Mahabodhi 58. South asian Priority Trade Agreement
temple of Gaya. This site contains a descendent of the 'SAPTA' was formally inaugurated on
Bodhi tree under which Buddha gained enlightment. (a) 7 December, 1994 (b) 7 November, 1993
(c) 7 December, 1995 (d) None of these
53. In November 1995, two serial blasts that caused
Ans. (c) : South Asia Preferential Trading Agreement
the assassination of American soldier occurred (SAPTA) came into existence in 1995. Indigenously
at SAARC members following his sixth SAARC summit
(a) Islamabad (b) Jerusalem held in Sri lanka in 1991 proposed to establish South
(c) Los Angeles (d) Riyadh Asian Preferential Trade Agreement (SAPTA). The
Ans. (a) : In November 1995, two serial blasts that framework agreement on SAPTA was approved in 1993
caused the assassination of two American soldiers and implemented in Dec. 1995.
occurred at Islamabad. Islamabad is the capital city of 59. The country won the European National Cup
(1995) of Hockey was
Pakistan. (a) Netherland (b) Spain
54. The largest scorer of medals in Asian Athletics (c) Germany (d) England
competition of 1995 was the country Ans. (c) : The European National cup of Hockey is
(a) China (b) India organised by the European Hockey Federation. It was
(c) Japan (d) South Korea held in Dublin, Ireland from 16 to 27 August 1995 and
Ans. (a) : The largest scorer of medals in Asian the winner country of this championship was Germany.
Athletics competition of 1995 was China which is 60. The author of 'Jharkhand castel over the
Graves' is
officially known as Peoples Republic of China(PRC), (a) Bhikhari Miyar (b) Bhagwan Das
which is the world's most populous country. (c) Victor Das (d) Sibu Soren
55. Indian archer Limba Ram won gold medal in Ans. (c) : The Author of "Jharkhand castel over the
(a) Federation cup tournament, 1995 grave" is 'Victor Das'. Generally it is a composite of
(b) National Archer Championship, 1995 Tribal studies of India Series.
(c) Commonwealth Archer championship, 1995 61. Indira Gandhi International Award of 1995 for
(d) all of the above peace, disarmament and progress was given to
(a) General Obasanjo (b) Toyin Raji
Ans. (a) : Indian archer Limba Ram won the gold (c) Yitzhak Rabin (d) None of these
medal in the federation cup tournament of 1995. He is Ans. (a): The Indira Gandhi Prize for Peace,
an archer by Profession who represented India in Disarmament and Development, was instituted in the
several International competition inducing three memory of the former Prime minister by a trust (Indira
Olympic Games. Gandhi Memorial Trust) in her name in 1986. It consist
56. Vyas Samman of 1995 for Hindi poem was of a monetary award of Rs. 25 Lakh along with a
given to citation. In 1995 this award was given to General
Obasanjo.
(a) Kunwar Narayan (b) Karnam Miss World 2021 - Karolina Bielawska of Poland.
Malleshwari
62. Crown of Miss World of 1995 was bestowed to
(c) Ruchitra Malhotra (d) Dinesh Singh (a) Jacqueline Anguilera (b) Aishwarya Rai
Ans. (a) : Vyas samman recognizes the outstanding (c) Manpreet Brar (d) Amico Martinovich
literary work in hindi, authored by an Indian citizen Ans. (a) : The Crown of Miss world of 1995 was given
writer. In 1995, Vyas Samman was given to "Kunwar to Jacqueline Aguilera of Venezuela. She was crowned
Narayan" for the literary work "Koi dusra Nahin" This by miss world 1994 Aishwarya Rai of India.
award was first awarded in 1991 to Ram Vilas Sharma 63. CHOGM apex conference of 1995 was held at
Its prize money is 2,50,000 Rupees. (a) Sydney (b) Stolkholm
Noted Hindi writer Asgar Wajahat will be awarded the (c) Auckland (d) Ireland
31st Vyas Samman for his play 'Mahabali'. Ans. (c) : The CHOGM summit, 1995 was held in
Auckland New Zealand between 10 Nov. 1995 and 13
57. The full form of APEC is Nov. 1995. The Commonwealth Head of Government
(a) Asia for Peace and Economic Cooperation Meeting (CHOGM) is a summit where the Common
(b) Asian Pact for Environment Control wealth leaders meet once every two years to discuss
(c) Asia Pacific Economic Control global and common wealth issues and reach
(d) Asia-Pacific Economic Cooperation consequences on future policies and initiative.
41th BPSC (Pre) Exam. 1996 48 YCT
64. Golden Triangle (Swarnim Trikon) stands for 68. During Hiuen Tsang tour in India most famous
(a) a federation of gold producing countries city for the production of cotton clothes was
(b) Bermuda triangle where ocean seems golden (a) Varanasi (b) Mathura
(c) Toxic substance producing area of Thailand (c) Pataliputra (d) Kannauj
and Myanmar Ans. (b) : Hiuen Tsang a Chinese Buddist pilgrim who
(d) Trilateral smuggling bases of Afghanistan, came to India during the reign of king Harshvardhan
Iran and Pakistan During the period of Hiuen Tsang's visit, the city most
Ans. (d): The golden Triangle is located in South East famous for the production of cotton cloth was Mathura.
Asia comprising of their contiguous countries of Laos, 69. The capital of Qutub-ud-din Aibak was
Thailand and Myanmar. (a) Lahore (b) Delhi
Golden crescent is the region of south west Asia which is (c) Ajmer (d) Lakhnauti
a principal global site for Opium production and
distribution. It comprises Afghanistan, Iran and Pakistan. Ans. (a) : Qutbuddin Aibak was the first ruler of the
Mamluk Dynasty. He was born to a Turkish family in
65. Which of the following dynasties is not Central Asia and was sold as a slave to Muhammad
mentioned in Sangams literature? Ghori, the ruler of Ghor in Afghanistan. When Ghori
(a) kadamb (b) cher
was assassinated. Aibak himself declared as Sultan of
(c) chol (d) Pandya
Delhi in 1206 and made his capital to Lahore. He
Ans. (a) : The Sangam Literature is the literature of reigned till his death in 1210.
ancient Tamil Spanning from 300 BCE to 300 CE. In
Sangam literature a brief knowledge come about only 70. First Maratha Sardar to accepts the subsidiary
three major dynasties. These were Chola, Cheras and alliance of Lord Wellesley was
Pandya. The Kadmaba dynasty has not been mentioned (a) Peshwa Bajirao II (b) Raghuji Bhosle
is Sangam literature. (c) Daulat Rao Sindhia (d) None of these
66. Four important events of Buddha's life and Ans. (a) : Subsidiary alliance was basically a treaty
thereby related four place are mentioned between the British East India Company and the Indian
below. princely states by virtue of which the Indian Kingdom
Match Schedule I and Schedule II and select had lost their sovereignty to the English. It was framed
correct answer to the codes given below. by Lord Wellesley, the Governor-General of India from
Schedule - I Schedule - II 1798 to 1805.
Peshwa Bajirao-II was the first Maratha to enter into the
A. Birth 1. Sarnath
subsidiary alliance with the British.
B. Enlightenment 2. Bodhgaya 71. Who among the following was the founder of
C. First preaching 3. Lumbini All India Muslim League?
D. Death 4. Kushinagar (a) Sir Syed Ahmed Khan
Codes (b) Sir Mohammad Iqbal
A B C D (c) Agha Khan
(a) 1 2 3 4 (d) Nawab Salimullah
(b) 2 3 1 4 Ans. (d) : Nawab Khwaja Salilmullah founded the 'All
(c) 3 2 1 4 India Muslin League' in 1906. The party advocated a
(d) None of above separate Muslim state 'Pakistan'. The party was formed
Ans. (c) : The founder of Buddism is Siddartha Gautam in Dhaka (now in Bangladesh)
popularly known as "Buddha" He was born into royal It was India's first Muslim party and this party was
family of Sakya clan who ruled from Kapilvastu, in totally responsible for the creation of Pakistan.
Lumbini which is situated near the Indo-Nepal Border. Muhammad Ali Jinnah, A.K. Fazlul Huq. Aga Khan-III
Gautama attained Enlightenment under a pipal tree at were the presiding leaders of this league.
Bodhgaya, a village in Bihar and he gave his first 72. Who among the following was the founder of
sermon in the village of Sarnath. This event is known as Atmiya Sabha?
Dharma-Chakra Pravartana and Gautam Buddha died at (a) Raja Ram Mohan Roy
the age of 80 in 483 BCE at a place called Kushinagar, (b) Swami Dayanand Saraswati
a town in U.P. (c) Swami Vivekanand
67. Chadragupta's palace situated at Pataliputra is (d) Aurobindo Ghosh
made mainly of Ans. (a) : Raja Ram Mohan Roy started Atmiya Sabha
(a) bricks (b) stones in 1815 in order to start crusade against idol worship,
(c) woods (d) clays meaningless rituals and superstitious beliefs.
Ans. (c) : The Greek historians Megasthenes described
73. Brahmo Samaj was established in the year
the Palaces of Mauryan Empire as one of greatest
creation of mankind and wood was the principal (a) 1828 (b) 1829
building material used during the Mauryan empire. (c) 1831 (d) 1843
There are found some evidences which prove that they Ans. (a) : Raja Ram Mohan Roy founded Brahmo
were made up of woods as Ashoka's Palace at Sabha in 1828, which was later renamed as Brahmo
Kumrahar and Chandragupta Maurya's palace at Patna. Samaj. It focused on prayers, meditation and reading of

41th BPSC (Pre) Exam. 1996 49 YCT


the scriptures and believed in the unity of all religions. Ans. (b):Time that Earth takes to complete one
It split into two in 1866 namely Brahmo Samaj of india revolution around the sun is about 365 days 5 hours and
led by Keshub Chandra Sen and Adi Brahmo Samaj led 59 minutes and 16 seconds or 365.25 days. Earth is the
by Debendra Nath Tagore. third planet to the sun by its distance and fifth largest
74. Out of the statements below first is Assertion planet by its size. Earth's average orbital speed is about
(A) and second is Reason (R). Carefully read 30 km/sec. by which It takes to complete one revolution
both statements. around the sun in 365.25 days.
Assertion (A): Till the end of twelfth century 79. Who among army-commanders of Alauddin
Nalanda Mahavihara lost its glory. Khilji died fighting against Mongols?
(a) Zafar Khan (b) Nusrat Khan
Reason (R): State protection ceased to be
(c) Alp Khan (d) Ulug Khan
available to Mahavihara.
Ans. (a): Zafar Khan was the General of Alauddin
With respect to the above mentioned two
Khilji of the Khilji dynasty the ruler of Delhi sultanate.
statements, which of the following is true?
Zafar Khan holds the honour of being one of the few
(a) A and R both are true and R is proper undefeated military commanders in history. He defeated
explanation of A an invading Mongol army near Jalandhar in 1297. It
(b) A and R both are true and R is not proper was led by Jafar Khan & Ulugh Khan. In 1299 about
explanation of A 200000 Mongols entered India with the intension of
(c) A is true but R is false conquest but Zafar Khan showed his bravery in battle
(d) A is false but R is true and the Mongols were defeated but Zafar Khan was
Ans. (b) : Here Both (A) and (R) are true but (R) is not killed in this Battle.
the correct explanation of (A) 80. Arrange properly as per dates of four
The downfall of Nalanda Mahavihara was over by the important battles fought by Humayun, the
end of 12th century and official protection of names of battlefields are given below
Mahavihara was to be ended are true but the reason is (a) Chausa, Devra, Kannauj, Sirhind
not correct explanation of assertion. (b) Devra, Kannauj, Chausa Sirhind
In 11th century the Pal ruler favoured Vikramshila (c) Sirhind, Devra, Chausa, Kannauj
university over Nalanda which led to its downfall. (d) Devra, Chausa, Kannauj, Sirhind
75. Where was located the mines of gold in Bihar Ans. (d) : The correct sequence of the four major
during seventeenth century? battles fought by the Mughal ruler Humayun is as
(a) Champaran (b) Patna follows:-
(c) Sasaram (d) Jharkhand 1. Devra -1532 AD
2. Chausa - 1539 AD
Ans. (b) : During seventeenth century the mines of gold 3. Kannauj - 1540 AD
in Bihar was found in Patna. Ralph Fitch has mentioned 4. Sir hind - 1555 AD
about this in his journal. 81. Who among the following historians declared
At present, the gold deposit evidences was found in the regime of Shah Jahan to be the golden Era
Gaya, Rajgir (Nalanda) and Mungher surveyed by of Mughal Period?
Geological Survey of India (GSI) (a) V A Smith (b) J N Srivastava
76. In which year Gorakshani Sabhas were (c) A L Srivastava (d) None of these
established to stop cow slaughter in Bihar? Ans. (c) : A.L. Srivastava declared Shah Jahan's reign
(a) 1877 (b) 1882 (1628-58) as the golden age of Mughal Period
(c) 1893 (d) 1897 architecture because he constructed several large
Ans. (b) : Dayanand Saraswati established a committee monuments like the Taj Mahal, the Red fort etc. and he
for the protection of cows called Gorakshni Sabhas in also did a lot of welfare work like constructing roads
1882. and canals. Moreover this, Trade and commerce also
77. In which year Odisha was separated from flourished in this era that's why this period is known as
the golden age of Mughal architecture.
Bihar
(a) 1930 (b) 1933 82. Who among the following abolished the
(c) 1936 (d) 1937 diarchy system in Mughals?
(a) Robert Clive (b) Lord Cornwallis
Ans. (c) : Odisha along with Bihar were separated from (c) Warren Hastings (d) None of these
Bengal province on 22st March 1912 and became Bihar Ans. (c) : After the Battle of Buxur in 1764. Lord Clive
and Odisha as a separate province. Once again on 1 introduced the diarchy system in Bengal but in 1772,
April 1936 the province was partitioned into Bihar and the Governor General of Bengal, Warren Hasting
the Odisha Provinces. abolished the diarchy system. Warren Hasting was the
78. Revolving round the Sun, Earth takes first Governor General of Bengal and he was appointed
approximately by Regulating Act of 1773.
(a) 365 days (b) 365.25 days 83. Ranjeet Singh got famous Kohinoor diamond
(c) 365.5 days (d) 365.75 days from
41th BPSC (Pre) Exam. 1996 50 YCT
(a) Shah Shuja (b) Zaman Shah central council of Ministers there were 7 members of
(c) Dost Mohammad (d) Sher Ali Parliament from Bihar who were selected in the central
Ans. (a): A descendant of Ahmad Shah, Shah Shuja council of Ministers. At present 4 members of
Durrani bought the Kohinoor back to India in 1813 and Parliament from Bihar is included in central council of
gave it to Ranjeet Singh. (the founder of the Sikh ministers.
Empire). In exchange Ranjeet Singh helped Shah Shuja 90. Hottest planet of solar system is
to get back his to throne of Afghanistan. (a) Mercury (b) Venus
(c) Mars (d) Earth
84. Poet Banabhatta was inhabitant of
Ans. (b) : Our Solar System contains eight planets.
(a) Pataliputra (b) Thaneshwar Among them Venus is the hottest planet of our solar
(c) Bhojpur (d) None of these system. The surface of the Venus is approximately
Ans. (b) : Banabhatta was the Sanskrit Prose writer and 465ºC, Venus is also known as morning & Evening star.
most famous poet in the court of king Harshavardhana. Since the Venus has a very dense carbon dioxide
He had written Harsha's Biography, "Harshacharita" atmosphere and energy does not escape back into the
Banabhatta was the inhabitant of Thanesar, now in Haryana. Environment that is why it is the primary cause which
85. Patna was made the capital of a state by makes it the hottest among all other planets.
(a) Sher Shah 91. The largest mountain series of the world is
(b) Alauddin Hussain Shah (a) Himalaya (b) Andes
(c) Ibrahim Lodi (c) Rockies (d) Alps
(d) Prince Azim Ans. (b) : The largest mountain range of the world is
Ans. (a) : The King Shershah Suri (1488-1545), the Andes mountain range. It is located along the western
Afghan emperor revived this capital and constructed a coast of South America and extend from the north to
fort here on a strategic location and put a boundary south direction. Its total length approximately measures
around Patna and made Patna his provincial capital. 5500 miles.
92. Which of the following countries declared her
86. Robert F Kennedy Human Rights Award of
intention that it would support the views of
1995 was given to India for the comprehensive treaty, CTBT to
(a) Kailash Satyarthi (b) Doan Viet Hoat ban atomic test?
(c) Nguyen Dan Que (d) All of these (a) Pakistan (b) Iraq
Ans. (d) : Robert F. Kennedy Human Right Award is (c) Iran (d) None of these
dedicated to empowering young human rights defenders Ans. (d) : Iran declared her intention that she would
and mobilizing a diverse community of young people to support the views of India for the comprehensive treaty
take action for social justice and human rights. (CTBT) to ban of atomic test.
In 1995 Robert F. Kennedy award was given to three 93. Who will preside over the proposed women
persons namely Kailash Satyarthi, Doan Viet Hoat and commission in Bihar?
Nguyen Dan Que. (a) Governor of Bihar
87. Latest member of Non-Aligned Movement of (b) Chief Minister of Bihar
the following is – (c) Chief Justice of Bihar
(a) Ukraine (b) Turkmenistan (d) None of these
(c) Uzbekistan (d) None of these Ans. (b) : The proposed women commission in Bihar
Ans. (b) : At present the Movement has 120 member will be presided over by the Chief Minister of Bihar
during the year 1995. The National Women
Nation. Turkmenistan was the newest country which
Commission was established in 1992.
participated in the NAM Summit at that time during
1995. The first NAM summit conference took place in 94. The largest delta of the world is formed by
(a) Ganga-Brahmaputra
Belgrade, Yugoslavia, in September 1961. (b) Missisipi-Missouri
88. Who is the present President of Algeria? (c) Yangse-Kyang
(a) Liamine Zeroual (b) Aliza Ejetvegavic (d) Hwang-Hoe
(c) Franzo Ejetvegavic (d) None of these Ans. (a) : The Ganges-Brahmaputra delta is the largest
Ans. (a) :Liamine Zeroual was the President of the delta in the world.
People's Democratic Republic of Algeria and the head The deposition of sediments that is carried by a river
of State and chief executive of Algeria during 1995. At created a landform called a river delta.
present, President of Algeria is Abdelmadjid Tebboune. The Ganges Delta also called Sunderban Delta is
89. Presently how many members of Parliament from situated in the Bengal region of India and also in
Bihar is included in central council of ministers? Bangladesh. It covers an area of 75000 sq. km. and it is
(a) 3 (b) 5 also one of the most fertile regions of the world.
(c) 7 (d) None of these 95. Raw materials upon which cotton industry is
Ans. (c): During 1991-96 the 10th Lok Sabha election based are
were held for 5 year term in the country and in the (a) Weight depreciating (b) Weight increasing
(c) Weight constant (d) None of these
41th BPSC (Pre) Exam. 1996 51 YCT
Ans. (a) : The cotton industry based on raw material Ans. (c) : Rice is grown on the largest area in India. It is
which are weight depreciating in nature. Gujarat is the a crop of tropical humid climate. It requires temperature
largest producer of cotton. It is a fibrous crop above 25ºC and rainfall above 100 cm. About 32% part
Maharashtra has the largest number of cotton mills. of arable land is used for the cultivation of Rice. India is
Mumbai is also called as "Cotton Polis of India" the second largest producer of Rice in the world after
because Mumbai has became the most important centre China.
of cotton textile in the country due to reasons like- 102. Vidarbh is a regional name of India and it is a
proximity to raw materials, favorable climatic condition part of
and good transport and Port facilities. (a) Gujarat (b) Maharashtra
96. Additional sub-continental mountains of India (c) Madhya Pradesh (d) Odisha
were formed during Ans. (b) : Vidarbha is the eastern region of Maharashtra
(a) Azoic era (b) Palaeozoic era It comprises of 11 district. The coal mines and
(c) Mesozioc era (d) Cenozoic era Mangnese mines of Maharashtra are located in this
Ans. (b) : The extra sub continental mountains of India Vidarbha region. Regionally, It occupies 31.6% of total
were formed during the Palaeozoic era. It is part of area of Maharashtra. It contain mainly two division
geological Dravidian Rock system and were formed Nagpur division and Amravati division.
about during 600-300 million years ago. 103. Cherrapunji is located in which state
97. Bhadravati steel centre is located at (a) Assam (b) Manipur
(a) River Bhadravati (b) River Bhadra (c) Meghalaya (d) Mizoram
(c) River Tung (d) River Tungabhadra Ans. (c) : Cherapunji is situated at the south of the
Ans. (b) : The Bhadravati steel plant is also known as Khasi hills in the Indian state of Meghalaya. It receives
Vishweshwaraya Iron and Steel Plant (VISL) located at average annual rainfall about 11,777 millimeters. It lies
the place Bhadravati. It is a unit of Steel Authority of on the windward side of the khasi hills and it is located
India (SAIL) and located on the banks of the river on the shillong plateau.
Bhadra in Shimoga district of karnataka. This plant gets 104. On the basis of density of population, Bihar
its water from the Bhadra River. ranks
98. The largest jute producer state in India is (a) first in India (b) second in India
(a) Andhra Pradesh (b) Bihar (c) third in India (d) fourth in India
(c) Tamil Nadu (d) West Bengal Ans. (b) : Based on recent data, Bihar at that ranks first
Ans. (d) : At present, West Bengal is the largest in the country followed by West Bengal while
producer of Jute in India. Most of the mills in West Arunachal Pradesh having lowest population density
Bengal are located in the Hugli industrial area (on both into country. The population density of Bihar is 1102
banks of the Hugli). Jute Industry is an agro-based person/sq.km. When the question was asked, Bihar was
Industry. It is weight losing industry. The first Jute mill at rank second in Population density
in India was set up at a place called 'Rishra' on the bank 105. Least populated district in Bihar is
of the Hugli River in 1885. (a) Godda (b) Gumla
99. Old Kacchari clay of gangetic plain is called (c) Araria (d) Lohardaga
(a) Bhabar (b) Banger Ans. (d) : The least populated district in Bihar is
(c) Khadar (d) Khondolyte Lohardaga According to census 1991, its population
Ans. (b) : The older alluviam soil of Gangetic Plain is was 2,88,896, Lohardaga is known for its bauxite
called Bhanger or Bangar soil. It is a higher region than mines. On Recent data base - 2011, "Shekhpura" is the
Khadar, normally the flood water does not reach this least populated district in Bihar and most populated
region. The expansion of the Bangar region is between district is Patna having its population about - 57,
two rivers (doab). The Ganga-Yamuna Doab and the 72,804.
plain of the Sutlej are its examples. 106. Iron ore is found in the district
100. Duration of Eighth Five year plan of India is (a) Lahardaga (b) Dumka
(a) 1990-95 (b) 1991-96 (c) Dhandad (d) Singhbhum
(c) 1992-97 (d) None of these Ans. (d) : Before the bifurcation of Jharkhand and
Ans. (c) : The 8th five year plan commenced in 1992 Bihar in 2000, the iron ore is extracted from Singhbhum
and carried on till 1997. Since this Eighth plan could in Bihar. At present the reserve of iron ore is found in
not take off in 1990 due to the fast changing economic Bhagalpur and Jamui district of Bihar. Among them
situation at the centre and the years 1990-91 and 1991- 'Magnetite iron ore' is extracted from the ore obtained
92 were treated as Annual plans. from Jamui and 'Hematite' from the ore obtaining from
The main objective of the Eighth five year plan are-to Bhagalpur.
prioritize the specific sectors which requires indicative 107. Main crop of Chotanagpur plateau is
investment and strengthen the basic Infrastructure. (1) rice (2) maize
101. Which crop is planted in the largest area of (3) pulses (4) wheat
India? Select the correct codes
(a) Cashew nut (b) Jowar (a) only 1 (b) 2 and 3
(c) Rice (d) Wheat (c) 1 and 3 (d) 2 and 4
41th BPSC (Pre) Exam. 1996 52 YCT
Ans. (c) : The main crop of Chotanagpur plateau are Ans. (b) : The biggest source of the Income of
maize, pulses and some other Coarse grain because Government of India is excise duties. Excise duty is a
there is lack of assured irrigation facilities here. It is for form of tax imposed on goods for their production
this region which could not get benefited from the green licensing and sale. It applies to those goods which are
revolution as did the other Indian states like Punjab and manufactured domestically in the country. After GST
Haryana.
was introduced excise duty was replaced by Central
108. The belt of Jute Industry in Bihar is located in GST because excise was levied by the central
(1) North-West plain of Bihar
government. The revenue generated from CGST goes to
(2) North-East plain of Bihar
(3) Middle-South plain of Bihar the central government.
(4) Damodar Valley region 113. Which of the following is not important
Select the correct codes directly for economic development?
(a) 1and 2 (b) only 2 (a) Quantity of money
(c) 2 and 3 (d) 1 and 3 (b) Price stability
Ans. (d) : In Bihar the Belt of Jute industry is at north- (c) Saving rate
west plain of Bihar and central south plain of Bihar. (d) Utilisation of resources
This is because most of the river flowing through this Ans. (a) : The Quantity of money is not directly
region fulfills the demand of water supply to these
important for economic development. It only ensures
industry, therefore all the Jute Industries are situated
near the bank of these rivers. liquidity in the market. Though it has implications on
109. 'Expansion of Tarai Region in' Bihar is the consumption but it does not become sole deciding
(a) parallel to North bank of Chotanagpur plateau factor in economy for their development.
(b) parallel to India-Nepal boundary 114. The exchange rate of Indian Rupee is fixed in
(c) parallel to the East bank of Rohtas plateau relation to
(d) parallel to the West boundary of Rajmahal (a) the dollar of United States of America
mountain series (b) British Pond
Ans. (b) : The Tarai region of Bihar is extended Parallel (c) Some selected currencies
to the Indo-Nepal boundary. The soil found at Tarai (d) None of the above
region is mixed with sand. It is extended through West Ans. (c) : The exchange rate of Indian currency is
Champaran, Sitamarhi, Madhubani, Supaul, Araria and determined by some selected currencies like (US)
Kishanganj. dollar, pound sterling, 'Euro' etc. An exchange rate is
110. Forests of Chotanagpur are the price of a nation's currency in terms of another
(1) Evergreen (2) Deciduous currency. Thus an exchange rate has two components
(3) Savana (4) Thorny the Domestic currency and a foreign currency.
Select you answer out of the following codes. 115. Life Insurance Corporation is an example of
(a) 1and 3 both are true (a) Commercial Bank (b) Development Bank
(b) Only 2 is true (c) Investment Bank (d) Co-operative Bank
(c) 2 and 3 both are true Ans. (c) : Life Insurance Corporation is an example of
(d) Only 4 is true investment bank. Initially. It was established as a
Ans. (c) : The forest of Chotonagpur is both deciduous oriental life insurance company in British colonial
and Savana type. The main species of deciduous forest period but on 1st September 1956, It was nationalized by
are Teak, Sal and bamboo forest etc. Indian government and thus it has become, one of the
At present times the chotanagpur region is a part of largest Investment Bank in India. Its head quarter is
Jharkhand which is best known for its mineral ores. located at Mumbai.
111. Most ancient large scale industry of India is 116. Out of two statements given below first is
(a) Iron and Steel (b) Jute Assertion (A) and second, Reason (R).
(c) Cotton (d) Paper Carefully read both.
Ans. (c) : The most ancient large scale industry of India Assertion (A): Programmes like Integrated
is the cotton industry. The first modern cotton textile Rural Development Programme did not get
mill was set up in 1818 in fort Gloster, Kolkata but it success as per expectations.
was unsuccessful. The first successful mill with Indian Reason (R): Due to centralized structure,
capital was set up in 1854 in Mumbai by Kevasji Davar. implementation was insufficient.
Maharashtra has a foremost place in cotton textile (a) A and R both are true and R explains A
industry. Here there are 190 cotton textiles mills. properly
112. The largest source of income of Indian (b) A and R both are true but R does not explain
Government is A properly
(a) Railway (b) Excise duty (c) A is true but R is false
(c) Sales Tax (d) Direct Tax (d) A is false but R is true
41th BPSC (Pre) Exam. 1996 53 YCT
Ans. (c) : 'A' is true but 'R' is false. Integrated Rural Ans. (c) : Aantyodaya Anna Yojana (AAY) was a step
development programme failed primarily because of in the direction of making Targeted Public Distribution
other reasons. Its inherent lacunae is in subsidy System aimed at reducing hunger among the poorest
mechanism which attracted large scale corruption to
certify BPL and also it did not look over other forms of segment of the BPL population. The Aantyodaya Anna
social exclusion like non-participation in community Yojana (AAY) was launched in December, 2000 for
decision making etc. one crore poorest of the poor families. Under this
117. Economic growth of India was hampered scheme one crore poorest of the poor families from BPL
mainly due to categories were benefited and provided food grains at a
(a) unskilled agricultural system highly subsidized rate of Rs. 2/-per kg for wheat and Rs.
(b) irregular industrialisation 3/per kg for Rice.
(c) Western social mentalion
122. From which of the following sources, India gets
(d) All of the above
maximum electricity?
Ans. (d) : In India the economic growth was hampered
mainly due to unskilled agricultural system, irregular (a) Hydel (b) Thermal
industrialization and western social profession. Since (c) Nuclear (d) Non-traditional
unskilled agricultural system leads to low productivity Ans. (b) : According to the latest data of 31-05-2019,
while irregular industrialization contributes to regional the thermal electricity energy being produced is about
imbalance in development. 63.4 percentage, hydroelectricity is about 12.7 percent,
118. What is the structure of Planning Commission? and the share of nuclear power is about 1.9 percent in
(a) Government Department total percentage of electricity produced in the country. It
(b) Advisory Body
(c) Autonomous Corporation is noteworthy that India is ranked at 5th position in the
(d) Portfolio of Ministry world in terms of total Renewable energy production.
Ans. (b) : The nature of planning commission is an 123. The President of India does not have the right
Advisory body. It was established on 15th march 1950 to
by then Prime Minister of India, Jawahar Lal Nehru and (a) give pardon
the Prime Minister is the ex-officio chairman of this (b) remove justice of Supreme Court
institution. The First five year plan was launched in (c) declare emergency
1951. At present it is replaced by "NITI Aayog in 2015. (d) pass ordinances
119. The Ex-officio chairman of the Planning Ans. (b) : Article-72 of the Constitution empowers the
Commission of India is President to grant pardons to persons who have been
(a) Planning Minister tried and convicted of any offences in all cases and
(b) Prime Minister Article-123 of the constitution empowers the president to
(c) President promulgate ordinances during the recess of Parliament.
(d) Governor of Reserve Bank of India Except it, constitution confers extraordinary powers on the
Ans. (b) : The ex-officio chairman of Planning president to deal with the following three types of
Commission of India is the Prime Minister. It was emergencies, as National emergency (Art.352), President's
formed on 15th March 1950. It has formulated the Rule (Art. 356 & 365) and Financial Emergency (Art.360).
India's five years plan and it also executes some other The President of India does not have the right of supreme
functions. It also has a deputy chairman who acts as the court and High court but does not have right to remove
functional head of the organisation. Its headquarter is in the justice of the supreme court.
New Delhi. At present, it is replaced By NITI Aayog in 124. Rajya Sabha is called permanent house because
2015. (a) all members are lifelong
120. Chief food crop of India is (b) cannot be dissolved
(c) some members retire after each two years
(1) Wheat (2) Rice (d) (b) and (c) are true
(3) Maize (4) Pulses Ans. (b) : The Rajyasabha (first constituted in 1952) is
Select correct codes a continuing house, that is, it is a permanent body and
(a) only 1 (b) 2 and 3 not subject to dissolution. However one-third of its
(c) 1 and 3 (d) 2 and 4 member retire every second year. Accordingly, the
Ans. (b) : The Chief food Crop of India is rice. In terms Parliament in the Representation of the People Act
of rice production the country holds the second position (1951). Provided that the term of office of a member of
all over the world. Rice is grown in approximately 32% the Rajyasabha shall be six years.
of the total arable land of the country. The important 125. As per Indian Constitution Legislature of states
consists of
rice growing regions in India are as follows-Punjab, 1. Legislative council and Governor
West Bengal, Uttar Pradesh, Odisha, Bihar, Assam and 2. Legislative Assembly and Legislative Council
Maharashtra. 3. Legislative Assembly and Governor
121. 'Antyodaya' Programme aims at 4. Governor, Legislative Assembly and
(a) development of minority Legislative Council where it exists
(b) upliftment of dalits Select your answer out of the following codes.
(c) helping poorest among the poors (a) Only 3 (b) 2 and 3
(d) complete removal poverty (c) 3 and 4 (d) Only4
41th BPSC (Pre) Exam. 1996 54 YCT
Ans. (d) : Article 168 to 212 in Part VI of the Ans. (c) : The President nominates 12-members to the
constitution deal with the organization, composition, Rajyasabha from people who have special knowledge or
duration, offices, procedures, privileges power and so practical experience in Art, Literature, Science and
on of the state legislature. The state legislature consist Social Service. The rational behind this principle of
of the governor, the legislative council and the nomination is to provide eminent persons a place in the
legislative assembly where the legislative council is the Rajya Sabha without going through the process of
upper house while the legislative assembly is the lower Election.
house. 131. Indian citizenship cannot be achieved by
126. The word 'Mantrimandal' ( Cabinet ) is (a) birth
mentioned only once in constitution and that is (b) Nationalization
in (c) absorbing any part of land
(a) Article 352 (b) Article 74 (d) depositing money in Indian Banks
(c) Article 356 (d) Article 76 Ans. (d) : The constitution deals with the citizenship
Ans. (a) : The term "Cabinet" (Mantrimandal) was first from Article-5 to 11 under part II. The Citizenship Act
inserted in Article 352 of the constitution in 1978 by the of 1955 prescribes five ways acquiring Citizenship viz,
44th constitutional Amendment Act, thus it did not find birth descent, Registration, Naturalisation and
a place in the original text of the constitution. Now also Incorporation of territory.
Article 352 only defines the cabinet saying that it is "the 132. Age of voting in general election which was 21
council consisting of the Prime Minister and other years is reduced to 18 years by
minister of cabinet rank appointed under Article-75." (a) 73rd Amendment (b) 62nd Amendment
127. Indian Constitution was adopted by the (c) 61st Amendment (d) 71st Amendment
Constituent Assembly on Ans. (c) : The 61st Amendment Act of 1988 has
(a) 26 November, 1949 (b) 15 August, 1949 reduced to voting age from 21 to 18 years. This came
(c) 2 October, 1949 (d) 15 November, 1949 into force on March 28, 1989.
Ans. (a) : The Constitution of India was adopted on
133. Which is the prominent financial institution to
November 26, 1949 contained a Preamble, 395 Articles
give industrial financial aid in Bihar?
and 8 Schedules, This is also mentioned in the preamble
(a) World Bank
as the date on which the people of India in the
(b) BISCOMAUN
Constitution Assembly adopted, enacted and gave
(c) Bihar State Finance
themselves this Constitution.
(d) State Bank of India
128. Fundamental Duties were determined by
Ans. (c) : The Bihar State Finance Corporation is a
(a) 40th Amendment (b) 43rd Amendment
prominent Institution in Bihar which provides Industrial
(c) 39th Amendment (d) 42nd Amendment
financial aid in Bihar. The Bihar State Financial
Ans. (d) : In 1976, the Congress party set up the Sardar Corporation Act 1951, shall on and from the appointed
Swarn Singh Committee to make recommendation day continue to function in those areas in respect of
about the fundamental duties, thus this committee has
which it was functioning immediately before that day.
recommended and enacted the 42nd constitutional
Amendment Act in 1976 which added a new part 134. Chotanagpur Tenancy Act was passed in the
namely Part IV A to the constitution and this new part year
consists of only one Article that is Article-51 A which (a) 1908 (b) 1902
for the first time specified a code of ten fundamental (c) 1990 (d) 1875
duties of the citizens. Ans. (a) : The Chota Nagpur Tenancy (CNT) Act,
129. Minister of Union Council of Minister is 1908, is a land rights legislation that was created to
responsible to protect the land rights of the tribal population of
(a) Prime Minister (b) President Jharkhand, instituted by the British. A major feature of
(c) Parliament (d) Only Lok Sabha the CNT Act is that it prohibits the transfer of land to
non-tribals to ensure community ownership.
Ans. (d) : The fundamental principle underlying the
working of Parliamentry system of Government is the 135. At the end of 1995 the number of districts in
principle of collective responsibility. Article-75 clearly Bihar was
state that the Council of Minister is collectively (a) 55 (b) 52
responsible to the Loksabha. It means they work as a (c) 50 (d) None of these
team and swim or sink together. Ans. (a) : At the end of 1995, the number of districts in
130. The number of members in Rajya Sabha Bihar was 55. After the formation of Jharkhand state in
nominated by the President is the year 2000, 18 districts became the part of Jharkhand
(a) 10 (b) 15 state. At present in 2022, the number of total districts in
(c) 12 (d) 20 Bihar is 38.

41th BPSC (Pre) Exam. 1996 55 YCT


136. What is the ratio of rural-urban population in violation while Directive Principle are non- justiciable,
Bihar? that is they are not legally enforceable by the courts for
(a) 40:50 (b) 20:80 their violation where Fundamental Rights promote the
(c) 50:50 (d) 80:20 welfare of the Individual, Directive principles promote
Ans. (d) : When the question was asked, the ratio of the welfare of community.
rural-urban population in Bihar was about 80 : 20. As 141. An amount with simple interest becomes ` 160
per the census 2011, the total population of Bihar is after two years and the compound interest is `
10.41 crore and Out of total population of Bihar, 11.29 164, the rate of interest is
percentage people lived in urban regions while 88.71 (a) 4% (b) 5%
percentage in Rural areas. (c) 6% (d) 10%
137. What is the percentage of Bihar to the total Ans. (b) :
mining production in the country 2 × difference of CI and SI
Rate% = × 100
(a) Approximately 36% SI
(b) Approximately 60% 2× 4
(c) Approximately 16% = ×100 = 5%
160
(d) None of these
142. The summation of three prime numbers is 100,
Ans. (c): When the question was asked, there was about in which first is 36 more than the next, the
16 percent share of Bihar in mining production to the highest prime number is:
total mining production in the country. At present, the (a) 31 (b) 67
mining production share of Bihar in the mining (c) 73 (d) 79
production of the country has reached to a very Ans. (b): We know that the smallest prime number is=2
negligible amount due to the separation of a very rich Let the smaller one of the required prime numbers be x.
mineral ore region known as "Chotanagpur plateau Then the largest prime number is x+36
region" from Bihar. adding the three,
138. What per cent of cultivating land in Bihar is 2+x+x+36= 100
irrigated? which gives,
(a) more than 50% (b) 40% to 50% 2x+38=100
(c) 30% to 40% (d) Less than 30% subtracting 38 from both sides,
Ans. (a) : The total Geographical area of Bihar is 9.36 2x=100–38=62
million hectare in which the percentage of net irrigated Dividing both sides by 2,
x=31
sown area is 62.5% till 2022.
Therefore, the largest prime number is 31+36=67
139. How many export promotion councils are there
143. If (713+1) is divided by 6, then remainder is
in India? (a) 0 (b) 1
(a) 5 (b) 6 (c) 2 (d) 3
(c) 7 (d) 8 13
 6 +1 1
Ans.(c): When the question asked the Export Promotion Ans. (c): Remainder =   +
Councils were 7. Presently there are fourteen export  6  6
13
promotion council under the administrative control of 1 1
the Department of Commerce. These councils are = +
6 6
registered as non -profit Organization under the 2
companies Act or societies Registration Act. = = 2 is correct.
6
140. How directive principles of state-policy 144. In 1930, The age of a man at the time of his
different from Fundamental Rights? death was twenty-ninth (1/29) part of the year
(a) First is for the central government and second he was born. If at the time of his death the age
for the states was in complete years, what was his age in the
(b) First is not the part of constitution whereas year 1900?
second is (a) 74 years (b) 45 years
(c) Directive Principles of state policy is not (c) 44 years (d) 54 years
executable whereas rights must be executed Ans. (c) : Let in the year 1900, his age was x years.
(d) None of the above ∴ his birth year= 1900–x
Ans. (c): Directive principles of state-policy is different 1900 – x
on the time of his death, his age =
in several ways to the fundamental right as 29
Fundamental right are justiciable, that is, they are Because on the time of his death, the age was in
legally enforceable by the court in case of their complete years.
41th BPSC (Pre) Exam. 1996 56 YCT
1900 – x Ans. (a) :
∴ should be Integer, which is divided
29
completely by 29.
∴1900 – x = 1856
∴1900 – 1856 = 44 years
145. The summation of eleven consecutive numbers Missing number = 26
is 220, the mean number is 149. If a person moved 3/4 of his usual speed, he
(a) 25 (b) 20 gets one and half hour late, what is the normal
(c) 15 (d) 10 time he takes:
Ans. (b) : Let us assume, first number is x. 1
(a) 4 hours (b) 4 hours
∴ eleven consecutive numbers 2
= x, (x+1), (x+2), (x+3)… +(x+10)
1
∴ x (x+1)+(x+2)+(x+3) ... +(x+10)=220 (c) 5 hours (d) 5 hours
or 11x+1+2+3+...+10= 220 2
or 11x +55= 220 Distance
Ans. (b) : Speed =
or 11x = 220 –55= 165 time
165 Let usual speed = 4 km/hr
∴x = = 15
11 3
∴Reduced speed = × 4 = 3km / hr
∴ mean number = 6th number 4
= (x+5) = 15+5= 20 usual speed : Reduced speed
146. The distance between two stations A and B is 4 : 3
70 Km. A train starts from A to B with speed to∵ distance is constant
80 km/hr and at the same time another train 1 1  1 
time → : = 3 : 4  1 unit = 1 hr 
from B to A starts with 1 km/min, they will 4 3  2 
meet each-other after following time – 3
(a) 60 min (b) 45 min 1unit = hr
2
(c) 30 min (d) 15 min
3 9
Ans. (c) : Converting 1km/min to km/hr. = 60 km/hr 3unit = 3 × = hr
2 2
Distance
speed = 1
usual time = 4 hr
Time 2
Time taken by both the trains to meet each other
150. In an election 20% voters did vote. Only two
Total Distance candidates were Ram and Shyam, Ram got
=
Totalspeed 42% of total votes and he defeated Shyam with
70 70 1 1 margin of 400 votes, Ram got:
= = = hr. = hr. = 30 minute (a) 4200 votes (b) 3800 votes
80 + 60 140 2 2
(c) 3360 votes (d) 300 votes
147. Three cubes of sides 3 cm, 4 cm and 5 cm are Ans. (a) :Let us assume the number of total voters is x.
made of metals: melting these one cube is 80x
made, the side of the cube is Cast votes =
(a) 6 cm (b) 51/2 cm 100
1/2 42x
(c) 4 cm (d) 4 cm Total votes of Ram =
3
Ans. (a) : Volume of formed single cube = 3 +4 +5 3 3 100
cm3 80x 42x 38x
Total votes of Shyam = – =
= 27+64+125 cm 3 100 100 100
= 216 cm3 42x 38x
According to the question – = 400
Let a be the edge of the new cube. 100 100
volume= 216cm3 4x
3 = 400
a = 216 100
3 3
a=6 ∴ 4x = 400 ×100
a= 6cm 40000
Therefore, 6 cm is the edge of cube. x= = 10000
4
148. Missing number in series 1,2,5.......677 is 42
(a) 26 (b) 37 So, Ram got votes = ×10000 = 4200
100
(c) 126 (d) 217
41th BPSC (Pre) Exam. 1996 57 YCT
42th Bihar Public Service Commission
Preliminary Examination, 1997-98
GENERAL KNOWLEDGE & GENERAL SCIENCE
(Solved Paper with Detail Explanation)
1. Which ruler made first, Pataliputra as the 5. Which one territory of the following was not a
capital of his empire? part of Ashoka's empire?
(a) Chandragupta Maurya (a) Afghanistan (b) Bihar
(b) Ashoka the Great (c) Sri Lanka (d) Kalinga
(c) Chandragupta Vikramaditya Ans. (c) : Ashoka was one of the greatest ruler known
(d) Kanishka to history. The empire that Ashoka ruled was founded
Ans. (a) : Pataliputra was originally established by by his grandfather Chandragupta Maurya. At it's zenith,
Udayin, ruler of Haryanka dynasty. He moved his Ashoka's empire stretched from Afghanistan in the west
capital from Rajgriha to Pataliputra and this status was to Bangladesh in the east. It covered almost the whole
maintained during the reign of the Mauryas and Guptas. Indian subcontinent except present-day Kerala, Tamil
Chandragupta Maurya made Pataliputra as his capital in Nadu and modern day Srilanka.
4th century A.D. He was the founder of Mauryan 6. What was the Land Revenue rate in Gupta
dynasty. The Mauryan empire was finally destroyed by period%
Pushyamitra Shunga in 185 BC. Udayin is not given in (a) one-fourth of the produce
option so Chandragupta Maurya is correct answer. (b) one-sixth of the produce
2. Why was Nalanda University famous in the (c) one-eighth of the produce
world? (d) half of the produce
(a) Medical Science Ans. (b) : During the reign of Guptas the land revenue
(b) Philosophy rate was one-sixth of the produce.
(c) Buddhist Religious Philosophy 7. Which Sultan made Alai Darwaza?
(d) Chemical Science (a) Iltutmish (b) Balban
Ans. (c) : Nalanda University was established by (c) Alauddin Khalji (d) Firoz Tughlaq
Kumar Gupta. The University attracted scholars from Ans. (c) : The Alai Darwaza was built by Alauddin
the different parts of the country as well as world. The Khalji, as the southern entrance gateway to the
university was known for Buddhist religious philosophy Quwwat-ul-Islam Mosque.
and was the centre of learning from 5th to 12th century 8. Who was the Author of Tabaqat i-Nasiri?
C.E. It was located at present day Rajgir, Bihar India. (a) Sheikh Jamaluddin (b) Al-beruni
Nalanda was one of the oldest universities of the world (c) Minhaj-i-Siraj (d) Jiyauddin Barni
and UNESCO declared the ruins of Nalanda
Mahavihara, a world heritage site. Ans. (c) : The Persian historian Minhaj-i-Siraj had
composed Tabaqat-i-Nasiri. This book elaborate history
3. Which Mandala of Rigveda is completely of the Islamic world written in Persian.
dedicated to 'Soma'? Al-Beruni had written the book, Tahqiq-i-Hind. Fatwa-
(a) 7th Mandala (b) 8th Mandala I-Jahandari was written by Ziauddin Barani.
(c) 9th Mandala (d) 10th Mandala
9. Who wrote Humayun-Nama?
Ans. (c) : Rigveda is the oldest of 4 vedas. It comprises
(a) Babur (b) Humayun
10 books also known as Mandala, which consists of
1028 hymns in about 10,600 verses. 9th Mandala is (c) Gulbadan Begum (d) Jahangir
completely devoted to Soma. The famous Gayatri Ans. (c) : Humayun-Nama is the biography of
Mantra is in 3rd Mandala. 10th Mandala contains the Humayun. It was written by Gulbadan Begum in
Purush Sukta. Persian language. Gulbadan Begum was daughter of
Emperor Babur and sister of Humayun. Humayun-
4. Famous battle of Ten Kings was fought on the
Nama provides the detailed account of the life of
bank of the river Mughal Emperor Humayun.
(a) Ganges (b) Brahamputra
10. Who made Red Fort of Delhi?
(c) Kaveri (d) Parushni
(a) Akbar (b) Nurjahan
Ans. (d) : The famous Battle of Ten kings or Dasrajna (c) Jahangir (d) Shahjahan
Yudha was faught between Sudas, a Bharata king of
Tritsu family and the confederacy of ten well known Ans. (d) : Mughal Emperor Shahjahan had built Red
tribes-Puru, Yadu, Turvasa, Anu, Druhyu, Alina, Fort of Delhi to serve as the palace fort for his Capital
Paktha, Bhalanas, Shiva & Vishanin. This battle was Shahjahanabad. He also built Jama Masjid in Delhi,
fought on the bank of river Parushni, The Bharatas Shalimar Bagh in Lahore, Rang Mahal Diwan-i Aam,
emerged victorious in this battle. and Diwan-i-Khas etc.

42th BPSC (Pre) Exam. 1997-98 58 YCT


11. Treaty of Serirangpatanam was signed between 17. Where was the birth place of Mahavira Swami
Britishers and the propounder of Jainism?
(a) Hyder Ali (b) Duplei (a) Papplivan (b) Vaishali
(c) Tipu Sultan (d) Nandraj (c) Kundagram (d) Vikramsheela
Ans. (c) : Third Anglo-Mysore war (1790-92) was Ans. (c) : Vardhamana Mahavira is considered as the
concluded with treaty of Serirangpatanam in 1792. 24th and last Tirthankara of Jainism. He was born in 599
Treaty of Serirangapatanam was signed in 1792 B.C.E. at a village named Kundagram near Vaishali. He
between Tipu Sultan and British East India company. passed away at the age of 72 years in 527 B.C.E. at
12. Who led the Revolt of 1857 in Bihar? Pavapuri in Bihar.
(a) Khan Bahadur Khan (b) Kunwar Singh 18. Which lady contributed the most in the Indian
(c) Tantya Tope (d) Rani Ramkunvari Politics after 1947?
Ans. (b) : Kunwar Singh led the revolt of 1857 in (a) Aruna Asaf Ali (b) Kamala Rani Singh
Bihar. He led revolt at Jagdishpur, Shahabad, Arrah and (c) Tarkeshwari Sinha (d) Rabari Devi
Patna in Bihar. Ans. (c) : Tarkeshwari Sinha contributed the most in
13. First conference of Indian National Congress Indian politics after 1947. She played an active role as a
held at student leader in Patna Women's college during quit
(a) Kolkata (b) Lahore India movement in 1942. After 1947, She was elected to
(c) Mumbai (d) Pune the first Lok Sabha in 1952 from Patna East Seat. She
Ans. (c) : The first session of Indian National congress was re-elected to the lower house in 1957, 1962 & 1967
was organized at Gokuldas Tejpal Sanskrit college in from Barh constituency.
Bombay in December 1885. The first session of INC 19. In which Farmer's movement did Mahatma
was attended by 72 delegates and presided over by Gandhi participate first time-
Womesh Chandra Bonnerjee. (a) Kheda (b) Champaran
14. Who was the viceroy of India during the (c) Bordoli (d) Baroda
partition of Bengal in 1905 ? Ans. (b) : In Champaran farmer's movement, Gandhi ji
(a) Lord Hardinge (b) Lord Curzon participated for first time in any movement in India
(c) Lord Lytton (d) Lord Minto after returning from south Africa.
Ans. (b) : Lord Curzon was the Viceroy of India from 20. In which century did the first Magadh Empire
1899 to 1905. The Partition of Bengal Province came rise?
into effect on 16th October 1905 under the viceroyalty (a) BC 4th Century (b) BC 6th Century
of Lord Curzon. Anti Partition Movement was initiated (c) BC 2nd Century (d) BC 1st Century
on 7 August 1905. India's first mass movement emerged Ans. (b) : Magadha empire rise to Political supremacy
in the form of Swadeshi Movement in 1905 against the began with Bimbisara, who belonged to Haryanka
partition of Bengal. dynasty and ruled for 52 years from 544 BCE to 492
15. Who is the author of 'Neel Darpan' a book BCE. He pursued a three pronged policy namely
written on the miserable condition of indigo matrimonial alliances, friendship with strong rulers and
farmers? conquest of weak neighbours to expand empire. His
(a) Bankim Chandra capital was at Rajgriha (Girivraja).
(b) Dinbandhu Mitra 21. Indian National Congress was established by
(c) Sharat Chandra Chatterji (a) A.O. Hume
(d) Rabindranath Tagore (b) Surendra Nath Benerji
(c) WC Banerji
Ans. (b) : Neel Darpan was written by Dinbandhu Mitra (d) Annie Beasant
in 1858-59. The play showed how farmers were coerced Ans. (a) : Indian National Congress was established by
into planting indigo without adequate payment. A.O. Hume, a retired English Civil Servant in 1885.
16. Which Leader of Bihar led the peasant The first session of INC was conducted in Gokuldas
movement along with Mahatma Gandhi- Tejpal Sanskrit college, Bombay in December 1885.
(a) Baba Ramchandra This session was attended by 72 delegates and presided
(b) Dr. Rajendra Prasad over by Womesh Chandra Bonnerjee.
(c) Rajkumar Shukla 22. Name the viceroy of India concerned with
(d) Rafi Ahmad Kidwai Bengal partition.
Ans. (b) : Gandhi ji was invited by Rajkumar shukla to (a) Lord Ripon (b) Lord Curzon
acknowledge the miserable condition of indigo farmers (c) Lord Lytton (d) Lord Mountbetton
of Bihar. Gandhi ji visited and met the farmers of Bihar, Ans. (b) : Lord Curzon was the Viceroy of India from
Seeing farmer's miserable condition Gandhi ji started 1899 to 1905. The Partition of Bengal Province came
Champaran Satyagraha in 1917. The prominent leaders into effect on 16th October 1905 under the viceroyalty
of Lord Curzon. Anti Partition Movement was initiated
associated with Champaran Satyagraha were Rajendra
on 7 August 1905. India's first mass movement emerged
Prasad, Anugraha Narayan Sinha, JB Kriplani and in the form of Swadeshi Movement in 1905 against the
Mahadev Desai etc. partition of Bengal.

42th BPSC (Pre) Exam. 1997-98 59 YCT


23. Song 'Bande Mataram' was written by 29. One of the founder leaders of Hindustan
(a) Rabindra Nath Tagore Socialist Republican Association was
(b) Benugopal Mishra (a) BR Ambedkar
(c) Bankim Chandra Chatterji (b) Bhagat Singh
(d) Girish Chandra Ghosh (c) Subhash Chandra Bose
Ans. (c) : In 1870s, Bankim Chandra Chatterjee wrote (d) Jai Prakash Narayan
Bande Mataram, national song of India. It was inspired Ans. (b) : HSRA was established in 1928 at Feroz shah
by the Sanyasi Revolt of late 18th century. The Kotla in New Delhi by Chandra Shekhar Azad,
constituent assembly of India declared "Bande Ashfaqulla Khan, Bhagat Singh, Sukhdev and Jogesh
Mataram" national song on 24 January 1950. In 1896, Chandra Chatterjee.
the song was first sung publicly at the INC's session by
Rabindranath Tagore. 30. "Swaraj is my birth right and I shall have it"
This statement is concerned with
24. The proposal of Purna Swaraj was passed in (a) Lala Lajpat Roy
Lahore Congress in the year (b) Mahatma Gandhi
(a) 1919 (b) 1929
(c) 1939 (d) 1942 (c) Bal Gangadhar Tilak
(d) Subhash Chandra Bose
Ans. (b) : The Indian National Congress in December
1929 passed the historic Purna Swaraj resolution at its Ans. (c) : Lokmanya Bal Gangadhar Tilak raised the
Lahore session. This session was presided over by Slogan "Swaraj is my birth right and I shall have it". He
Jawahar Lal Nehru. He has hoisted the tri-colour on the had founded the Deccan Education society in 1884 to
bank of river Ravi and demanded "Purna Swaraj" and impart quality education to Indian youth. He started
date set for independence was 26 January 1930. two weekly magazine named as Kesari and Maratha.
25. Government of India Act, 1935 abolished Kesari was in Marathi while Maratha was in English.
(a) provincial autonomy 31. Muslim League passed a proposal to establish
(b) provincial dyarchy Pakistan in the year
(c) federal structure of India (a) 1905 (b) 1917
(d) responsible central government (c) 1940 (d) 1946
Ans. (b) : British Parliament had passed the Ans. (c) : In the Lahore session of Muslim league, 1940
Government of India Act 1935 in August 1935. The Act under the Presidentship of Muhammad Ali Jinnah,
gave more autonomy to the provinces and diarchy was Muslim league passed a resolution to establish a
abolished at the provincial levels. It suggested the separate state known as Pakistan. The term Pakistan
establishment of diarchy at the centre and a 'Federation was coined by Chaudhary Rehmat Ali.
of India' consisting of the provinces of British India and 32. A separate electoral group was made by the
most of the princely states. communal Tribunal of Ramsay Mc'Donald
26. The famous slogan 'Swaraj' for the masses, not first time by the August, 1932
for the classes was given by- (a) for Muslims
(a) CR Das (b) for Indian Christians
(b) C Rajagopalachari (c) for Anglo - Indians
(c) Motilal Nehru (d) for untouchables
(d) Gopi Nath saha
Ans. (d) : British Prime Minister Ramsay MacDonald
Ans. (a) : Chittaranjan Das also called Desh bandhu
gave the famous statement ''Swaraj for the masses, not had announced the communal award in August 1932,
for the classes. He along with Motilal Nehru had which provided for Separate electorates for the
established Swaraj party in January 1923. 'Depressed classes'.
27. Indian National Congress Committee passed 33. The person not concerned with Champaran
the proposal 'Quit India' in the year movement was
(a) 1907 (b) 1917 (a) Rajendra Prasad
(c) 1930 (d) 1942 (b) Anugrah Narayan sinha
Ans. (d) : The Quit India movement resolution was the (c) JB Kripalani
result of failure of Cripps Mission. It was passed in (d) Jai Prakash Narayan
Bombay by All India congress committee on 8th August Ans. (d) : The prominent leaders like Anugrah Narayan
1942.
Sinha, J.B. Kriplani, Rajendra Prasad etc were with
28. Formation of Azad Hind Government was
Gandhiji during Champaran Satyagraha in 1917. While
declared on
(a) 26th January, 1930 Jai Prakash Narayan was not concerned with
(b) 8th August, 1942 Champaran movement . He was associated with Quit
(c) 21st October, 1943 India movement.
(d) 18th February, 1946 34. Swami Sahjanand was related with
Ans. (c) : Formation of Azad Hind Government was (a) Tribal movement in Bihar
declared on 21st October, 1943 by Subhash Chandra (b) Caste movement in Bihar
Bose, Which was recognised by the then governments (c) Farmer's movement in Bihar
of Japan and Germany. (d) Labour movement in Bihar
42th BPSC (Pre) Exam. 1997-98 60 YCT
Ans. (c) : Swami Sahjanand Saraswati was related with (a) Sachchidananda Sinha
farmer's movement in Bihar. He had formed Bihar (b) Anugrah Narayan Sinha
Provincial Kisan Sabha in 1929. He was the first (c) JB Kirpalani
President of the All India Kisan Sabha, formed in 1936. (d) Birsa Munda
35. Agitation against Chaukidari Tax in Begusarai Ans. (a) : Sachchidananda Sinha had led agitation in
was a part of Bihar for the separate state where by in the year 1912
(a) Non- cooperation movement Bihar and Orissa Province was formed.
(b) Civil disobedience movement 41. Someshwar range of Bihar is
(c) Quit India movement (a) older than Parasnath mountain series
(d) Khilafat movement (b) older than Rajmahal mountain series
Ans. (b) : Agitation against Chaukidari tax in Begusarai (c) older than Kharagpur mountain series
was a part of Civil Disobedience Movement (CDM). (d) younger than all of these
The chaukidari tax was a tax for the compensation of Ans. (d) : Shivalik range in Bihar is subdivided into
village watchman and it was collected from the poor three parts :-
farmers in Bengal Presidency. In 1930 the farmers 1. Ramanagar Doon
refused to pay this tax during Civil Disobedience 2. Somoeshwar Range
Movement. 3. Harha Valley
36. "It will be our blunder if we leave people of As Shivaliks are youngest range in India Someshwar is
Bihar and their government unsafe before the youngest among the options.
voilence and uncivilised attacks of league 42. Damoder in Bihar flows through
leaders". In the year 1946, who told this? (a) a carst area
(a) Mahatma Gandhi (b) rift valley
(b) Jawahar Lal Nehru (c) a coastal area
(c) Rajendra Prasad (d) None if these
(d) Sardar Patel Ans. (b) : Damodar River originates from Sonajhuri
Ans. (d) : Sardar Vallabh Bhai Patel told that "It will be falls of Chotanagpur plateau in Jharkhand. It passes
our blunder if we leave people of Bihar and their through Jharkhand and West Bengal. It occupies the
government unsafe before the voilence and uncivilised eastern margins of the Chotanagpur plateau where it
attacks of league leaders. flows through a rift valley and finally joins Hugli.
37. First meeting of Congress Socialist party held Damodar is also known as the 'Sorrow of Bengal'
in Patna in the year 43. The per cent of total geographical area of Bihar
(a) 1921 (b) 1934 covered by forests is
(c) 1937 (d) 1939 (a) 17% (b) 23%
Ans. (b) : Jaya Prakash Narayan convened a meeting in (c) 27% (d) 33%
Patna in May 1934, founded Bihar-Congress Socialist Ans. (a) : When the question was asked, Bihar had 17%
Party. In October 1934, Congress Socialist Party was forest cover. According to the India State of Forest
formed. Report 2021, forest cover in Bihar is 7381 sq Km which
38. On the issue of ban on cow slaughter in is 7.84% of the State's geographical area.
Shahabad, Gaya and Patna districts in Bihar, a 44. Maithon in Bihar produces
terrible Hindu-Muslim Communal riot (a) Thermal power (b) Hydel power
occurred in the year (c) Atomic power (d) Solar power
(a) 1917 (b) 1919 Ans. (b) : Maithon dam is built on the Barakar river. It
(c) 1939 (d) 1946 produces Hydel power.
Ans. (a) : There occurred communal riots between 45. In India, Bihar is the largest producers
Hindu-Muslim on the issue of cow protection in (a) of Lead (b) of Manganese
Sahabad, Gaya and Patna district in September 1917. (c) of Mica (d) of Limestone
39. Chotanagpur Tenancy Act. 1908 imposed ban Ans. (c) : When the question was asked Bihar was the
on largest producer of Mica but it has lost its position
(a) Independent industry of forestry mainly due to formation of new state of Jharkhand. At
(b) Burning forests present Andhra Pradesh is largest producer of Mica in
(c) Baith Bagdri India.
(d) Khutkatti Land system 46. After independence the irrigated areas in
Ans. (*) : The Chotanagpur Tenancy Act, 1908 restricts Bihar, has increased approximately
transfer of tribal land to non-tribals. It was an act related (a) two times (b) four times
to landlords, tenants and settlement of rents in the (c) five times (d) ten times
Chotanagpur plateau. Ans. (a) : After independence , the irrigated areas in
40. The person who led the agitation in Bihar for Bihar has increased by approximately two times. At
the separate state whereby in the year 1911 present Bihar has irrigation potential of 43.86 Lakh
new provinces Bihar and Odisha were formed
hectare.
was
42th BPSC (Pre) Exam. 1997-98 61 YCT
47. In the following production in India, the 55. The country with the largest population in the
monopoly of Bihar is on world is
(a) Limestone (b) Coal (a) India (b) USA
(c) Pyrites (d) Graphite (c) China (d) Russia
Ans. (c) : Bihar has monopoly in terms of production Ans. (c) : The five most populous countries of the
of Pyrite ore, about 95% of total Pyrite production of world (in descending order) are:- China, India, United
India comes from Bihar. Son and Vindhyan valley is States of America, Indonesia and Pakistan.
known for their Pyrite production. Asia is most populated continent followed by Africa
48. Economy in Chotanagpur was primarily based and Europe.
on 56. Johannesburg is famous for
(a) Agriculture (b) Forests (a) Gold mining (b) Tin mining
(c) Mines (d) Industries (c) Mica Mining (d) Iron- ore mining
Ans. (c) : Chota Nagpur plateau covers much of Ans. (a) : Johannesburg is Africa's biggest city. It is
Jharkhand state as well as adjacent parts of Bihar, also known as city of gold. The major gold and
Odisha, West Bengal and Chhattisgarh. The plateau is diamond mines are located in Johannesburg in South
store house of mineral resources such as iron ore, coal, Africa. Kimberley in South Africa is famous for
Bauxite etc. Hence economy of plateau is dependent on diamond.
mines.
57. Pygmies are inhabitants of
49. Bihar was separated from Bengal in (a) Africa (b) Asia
(a) 1910 (b) 1912 (c) Australia (d) South America
(c) 1921 (d) 1947
Ans. (a) : Pygmies people are scattered over a huge
Ans. (b) : Bihar together with Orissa were separated
area in Central and western Africa, these countries are -
from Bengal and became Bihar and Orissa province in
Congo, Gabon, Rawanda, Republic of Congo and
1912. On 1 April 1936 Orissa separated from Bihar and
both became separate provinces. Uganda. Bushman, Massai, Kikuyu and Fulani etc. are
some major tribes of Africa.
50. Total number of Lok Sabha seats in Bihar is
(a) 48 (b) 50 58. Highest Mountain peak in India is
(c) 52 (d) 54 (a) Mount Everest (b) Kanchanjangha
(c) Godwin Austin (d) Nanda Parvat
Ans. (d) : When the question was asked, Bihar had 54
Loka Sabha seat. But at present there are 40 Lok Sabha Ans. (c) : Mount Everest is the world's highest peak
seat and 16 Rajya Sabha seat in Bihar. Godwin-Austin or Mount K2 (8611 m), located in
51. Heavy Engineering Corporation in Bihar is Jammu & Kashmir is the highest peak in India and
situated at second highest peak in the world after Mount Everest.
(a) Jamshedpur (b) Bokaro Kanchanjunga is the second highest peak in India with
(c) Barauni (d) Ranchi an elevation of 8586 m. Nanda Devi is third highest
peak of India with an elevation of 7817 m.
Ans. (d) : Heavy Engineering Corporation Limited is
one of the leading suppliers of capital equipment in 59. Shiwalik range was formed in
India for steel, railways power, Defence, space research (a) Esozoic (b) Paleozoic
nuclear and strategic sectors. It is located in Ranchi (c) Mesozoic (d) Cenozoic
Jharkhand. Ans. (d) : Shiwalik Hills were formed during the
52. Muri is famous for Cenozoic period. Shiwalik range is a mountain range of
(a) Bauxite mining (b) Alumina plant the outer Himalayas. Longitudinal valley known as duns
(c) Alcohol Plant (d) Sponge Iron plant lies between the lesser Himalaya and Shiwaliks.
Ans. (b) : Muri is a census town in Silli CD block in the Shiwalik range extend over a width of 10-50 km.
Ranchi Sadar Subdivision of Ranchi district in 60. Regur is the name of
Jharkhand. It is famous for Hindalco's alumina refinery. (a) Red soil (b) Black soil
53. The planet which has no satellite is (c) Coastal soil (d) Laterite soil
(a) Mars (b) Mercury Ans. (b) : Black soil is also known as Regur soil. These
(c) Neptune (d) Pluto soil are of volcanic origin, generally found in Deccan
Ans. (b) : Mercury and Venus have no natural satellites. traps. Black soil is made up of finer clay materials so it
Mars has 2 natural satellites called Phobos and Deimos. has high holding capacity for moisture and best suited
Neptune currently has 14 known natural satellites. for cotton crop. Hence these soils are also called black
54. Gulf Stream is cotton soil.
(a) a river in the bay 61. First General Election of Lok Sabha was held
(b) an oceanic current in
(a) 1949 (b) 1952
(c) another name of jet stream
(c) 1950 (d) 1954
(d) an earth surfacial wind
Ans. (b) : India's first general election for Lok Sabha
Ans. (b) : Gulf stream is warm Oceanic current in were conducted in 1951-52. Indian National Congress
Atlantic ocean and meets Labrador (cold current) at the won this election and consequently Pt. Jawahar Lal
coast of North America. Nehru became India's first Prime Minister.

42th BPSC (Pre) Exam. 1997-98 62 YCT


62. Indian Constitution consists of Ans. (d) : Lok Sabha is not a continuing chamber. Its
(a) 300 Articles normal term is 5 years from the date of its first meeting
(b) 350 Articles after the general elections, after which it automatically
(c) more than 400 Articles dissolves. Under Article 85(2)(b) of Indian constitution
(d) 500 Articles only president has the power to dissolve the Lok Sabha
Ans. (c) : At present, constitution of India comprises on the advice of Council of Ministers.
more than 400 articles, 25 parts and 12 schedules. When 68. For successful functioning Panchayati Raj
the constitution was enacted. It had 395 articles in 22
part and 8 schedules. needs full co-operation of
(a) Local Public
63. In which form is India declared in the (b) Central Government
preamble of Indian constitution? (c) Bureaucrats
(a) a sovereign democratic
(b) a socialist democratic republic (d) Politicians
(c) a sovereign, socialist, secular, democratic Ans. (a) : Panchayats or village assemblies existed in
republic ancient India as self governing institution which had
(d) None of the above well defined functions for successful functioning,
Ans. (c) : Preamble of India declares India to be a Panchayati Raj needs the cooperation of the local
Sovereign, Socialist, Secular, Democratic, Republic. people.
The term Socialist, Secular and Integrity were added to 69. Whom of the following Indian constitution
the preamble through 42nd Amendment Act, 1976. vested executive powers of central
64. Which of the following is not the fundamental government?
right? (a) President of India
(a) Right to constitutional remedy (b) Prime minister of India
(b) Right to property (c) Central Cabinet
(c) Right to peaceful assembling (d) All of these
(d) Right of free movement all over the country Ans. (a) : Under the constitution of India, President is
Ans. (b) : Originally the constitution of India provided head of the executive. All the executive power is vested
7 Fundamental Rights. However the Right to Property in him and all Executive actions are taken in his name.
was deleted from the list of Fundamental Rights by the Executive power is exercised by president either
44th amendment Act, 1978. It is now a legal right under directly or through officers subordinate to him in
Article 300-A in part XII of the constitution. So at accordance with constitution.
present there are only six Fundamental Rights.
70. Prime minister of India is generally
65. Which state sends largest number of members (a) not a member of parliament
in Lok Sabha?
(a) Bihar (b) Madhya Pradesh (b) member of Lok Sabha
(c) West Bengal (d) Uttar Pradesh (c) member of Rajya Sabha
Ans. (d) : (d) member of both the Houses
States No. of Seat in No. of Seat in Ans. (b) : President of India appoints the Prime
Lok Sabha Rajya Sabha Minister to a person who is either the leader of the party
Uttar Pradesh 80 31 which hold a majority of seat in Lok Sabha or is a
Maharashtra 48 19 person who is able to win the confidence of the Lok
Bihar 40 16 Sabha by gaining the support of other political parties.
Madhya Pradesh 29 11 Generally Prime Minister of India is a member of Lok
West Bengal 42 16 Sabha.
Tamil Nadu 39 18 The Prime Minister can be a member of either Lok
66. Supreme Court in India was established Sabha or the Rajya Sabha.
(a) by an Act of Parliament in 1950 71. Session of Lok Sabha is called for atleast
(b) under India Independence Act, 1947 (a) Once in a year
(c) under Government of India Act, 1935 (b) Two time in a year
(d) by Indian constitution (c) Three times in a year
Ans. (c) : The Supreme Court of India was inaugurated (d) Four times in a year
on 28 January 1950. It succeeded the Federal Court of Ans. (b) : The President of India time to time summons
India, established under the Government of India Act of
1935. At present Supreme Court consists of one chief each houses of Parliament to meet. But the maximum
justice and thirty three other judge. gap between two session of Parliament can not be more
67. Before completing term, Lok Sabha can be than 6 months. i.e, the Parliament should meet at least
dissolved twice a year. There are usually three session in a year,
(a) by the discretion of President Viz, the Budget session, the Monsoon session and the
(b) by the discretion of Prime Minister Winter session.
(c) by the discretion of the speaker of Lok Sabha 72. Member of Rajya Sabha is elected for
(d) by the President on the recommendation of (a) 4 years (b) 5 years
Prime Minister (c) 6years (d) life long

42th BPSC (Pre) Exam. 1997-98 63 YCT


Ans. (c) : Rajya Sabha is a continuing chamber. It is not 79. Which one of the following is not the tax of
subject to dissolution. However, one-third of its central government?
members retire every second year. Accordingly, the (a) Income Tax (b) Land Revenue
Parliament in the Representation of people Act, 1951 (c) Custom Duty (d) Excise Duty
provided the term of office of a member of Rajya Sabha Ans. (b) : Land Revenue, Sales tax, Luxury tax, Motor
shall be 6 years. Vehicle tax, fair tax entertainment tax etc, are some of
73. Most of the people in India are employed in the taxes that are collected by the State Government.
(a) Industry (b) Agriculture The Central Government collect taxes such as Custom
(c) Mining (d) None of these duty, Income tax, Central Excise duty and Service tax
etc.
Ans. (b) : Disguised employment is a major type of
employment found in agricultural sector of India. One 80. Most important export item from India is
third of agriculture workers in India under disguised (a) Leather items (b) Clothes
employment. Disguised employment occurs when more (c) Tea (d) Rice
people are engaged in a job than required. Ans. (b) : Among the options textiles (clothes) are most
important export items.
74. As per latest census, average annual growth
rate of population in India 81. Reserve Bank of India helped to invest money
(a) shows the trend of increasing in India's foreign trade
(b) shows the trend of decreasing (a) by NABARD (b) by Exim Bank
(c) increased explosively (c) by IDBI (d) by SBI
(d) was constant as usual Ans. (b) : Reserve Bank of India (RBI) helped to invest
Ans. (b) : As per census 2011, India has decadal growth money in India's foreign trade by Exim Bank. Exim
Bank was established by the Government of India,
rate of 17.7% for the entire population as compared to
under the Export-Import Bank of India Act, 1981 in
21.34% as per census report 2001. This shows that the
1982. Export-Import Bank of India is leading export of
average rate of population growth in India is declining. financing institute that engages in integrating foreign
It is mainly due to control over birth rate, rising rate of trade and investment with the country's economic
literacy among the people etc. growth.
75. The aim of New Economic Policy, 1991 was 82. In Eighth Plan, GDP was aimed to raise upto
(a) Liberalisation (b) Privatisation the following level
(c) Globalization (d) All of these (a) 6.5% (b) 4.5%
Ans. (d) : The New Economic Policy (NEP) of India (c) 6.0% (d) 5.6%
was launched in 1991. It is a set of policy measures Ans. (d) : Eighth five year plan duration was from 1992
that emphasized Liberalization, Privatization and to 1997 under the leadership of P.V. Narasimha Rao. In
Globalization. this plan, the growth rate achieved was 6.8% against the
76. The largest wheat producer state in India is target of 5.6%. The plan was launched with the
(a) Punjab (b) Haryana objectives of alleviation of poverty and unemployment.
(c) Uttar Pradesh (d) Bihar 83. In Bihar, in the term of production, the second
Ans. (c) : Today India is second largest wheat producer crop after rice is:
in the world. Leading wheat producing states in India (a) Wheat (b) Maize
(according to order) are:- Uttar Pradesh, followed by (c) Gram (d) Potato
Madhya Pradesh & Punjab. Ans. (a) : In Bihar, wheat is second mostly grown crop
77. In the following which is the largest single after rice.
source from which the government of India get 84. Land Development Bank is a part of which of
maximum income in the form of revenue tax. the following?
(a) Excise duty (b) Custom Duty (a) Commercial Bank
(c) Personal Income Tax (d) Corporate Tax (b) IDBI
Ans. (d) :When the question was asked then Corporate (c) NABARD
tax was the single largest source of income for the (d) Co-operative Credit structure
Government of India. In 2020-2021 largest source of Ans. (d) : Land Development Bank are essentially co-
tax revenue was GST followed by Income tax and operative institution. All the land development banks
Corporate tax. are registered under the Co-operative Societies Act.
78. Land Reform sector in India does not consist These banks are also called Land Mortgage Bank and
(a) the elimination of intermediaries Agricultural and Rural Development Bank in some
(b) imposition of land holdings states. The first Land Development Bank was started at
(c) co-operative farming Jhang in Punjab in 1920.
(d) Home loan to farmers 85. Success of Green Revolution depends on the
availability of:
Ans. (d) : The land reforms of the Independent India (a) high yield variety seeds
has four component:- The elimination/abolition of (b) proper irrigation facilities
Intermediaries, Tenancy Reforms, fixing ceilings of (c) chemical fertilizers and insecticides
landholdings and consolidation of landholding. (d) All of the above
42th BPSC (Pre) Exam. 1997-98 64 YCT
Ans. (d) : Green Revolution is related with agricultural 92. Fatwa (Religious order) of death against
production in India, the Green revolution was led by Salman Rushdie was issued by
M.S. Swaminathan. The success of Green revolution (a) Ayatolla Khomeini
depends on availability of High Yield Variety of Seeds, (b) Rafsanjani
(HYVS), irrigation facilities, Chemical fertilizers and (c) Gulbuddin Hikamatyar
pesticides and new methods of agriculture.
(d) Nazibullah
86. Konkan Railway passed through
(a) Maharashtra, Goa, Karnataka Ans. (a) : On 14th February 1989, the Supreme leader of
(b) Maharashtra, Karnataka, Kerala Iran, Ayatollah Khomeini issued a fatwa calling for the
(c) Maharashtra, Goa, Kerala death of Salman Rushdie and his publishers over the
(d) Maharashtra, Goa, Karnataka, Kerala publication of the 'Satanic Verses'.
Ans. (a) : Konkan Railway is a vital link between 93. The secretary-general of UNO is
Mumbai (Roha) in Maharashtra and Mangalore in (a) Butras Ghali (b) Kofi Annan
Karnataka. It passes through three states Viz. (c) Perez de Cuellar (d) Uthant
Maharashtra, Goa and Karnataka.
Ans. (b) : Kofi Annan of Ghana was the 7th Secretary
87. Cauvery water dispute in between General of United Nations from 1997 to 2006. At
(a) Andhra Pradesh and Tamil Nadu
(b) Kerala and Tamil Nadu present, Antonio Guterres is the 9th Secretary General
(c) Karnataka and Tamil Nadu of the United Nations. He took office on 1st January
(d) Tamil Nadu and Pondicherry 2017.
Ans. (c) : Cauvery water dispute involves 3 states and 94. Saddam Hussein is the President of
one Union Territory they are - Tamil Nadu, Kerala, (a) Iraq (b) Iran
Karnataka and Puducherry. (c) Kuwait (d) Saudi Arabia
88. Garo Tribes are of Ans. (a) : Saddam Hussein became the President of Iraq
(a) Assam (b) Manipur in 1979. On 13th December 2003, he was captured by
(c) Mizoram (d) Meghalaya
American forces in operation Red Dawn. He was
Ans. (d) : Garo tribes are distributed over the Garo Hanged on 30th December 2006.
Hills districts of Meghalaya. Garos are second largest
tribe after Khasi in Meghalaya. 95. The name 'Agni' is related with
89. Bhakra Nangal Project is a joint venture of (a) A mache to fight with fire
(a) Haryana, Punjab and Rajasthan (b) A warfare tank
(b) Haryana, Punjab and Delhi (c) An aeroplane
(c) Himachal Pradesh, Haryana and Punjab (d) The missile prepared by India
(d) Punjab, Delhi and Rajasthan Ans. (d) : Agni missile is developed by Defence
Ans. (a) : Bhakra-Nangal Project is a joint venture of Research Development Organisation (DRDO). The first
Haryana, Punjab and Rajasthan. It is named after the Agni series missile, Agni-I was tested in 1989. Agni
two dams built at Bhakra and Nangal on Satluj River. Series missiles are ballistic missiles with different
This dam forms the Gobind Sagar reservoir. It was
designed to harness the precious water of the Satluj for ranges.
the benefit of the concerned states. 96. International Court of Justice is situated at
90. Delhi is a/an (a) Geneva (b) The Hague
(a) State (c) London (d) Paris
(b) Union territory Ans. (b) : International Court of Justice (ICJ) was
(c) Autonomous council established in June 1945 by Charter of United Nations
(d) None of these and began work in April 1946. The Seat of ICJ is in the
Ans. (b) : Delhi is a Union Territory of India containing Hague (Netherlands). ICJ one of the six principal
New Delhi, the Capital of India. The 69th Amendment organs of United Nations. It is composed of 15 Judges
Act of 1991, accorded a special status to the Union
Territory of Delhi by designating it is the National elected to 9 year terms of office by the UN General
Capital Territory of Delhi. The amendment also Assembly and the Security Council.
provided for the creation of 70- member legislative 97. The 'Statue of liberty' is located in-
assembly and 7 (10%) member as a council of ministers (a) France (b) Spain
for Delhi. (c) England (d) USA
91. The birth-place of Mother Teresa was Ans. (d) : The 'Statue of Liberty' is located on Liberty
(a) Albania (b) France Island in New York, USA. 'Statue of Liberty'
(c) Greece (d) Italy commemorates the friendship between the United State
Ans. (a) : Mother Teresa was born on 26th August 1910 and France that began during the American Revolution.
in Macedonia. She left home and arrived in India in
1929. She was the founder and was an active member of 98. Which country is not the member of South
Missionaries of charity, a Roman Catholic Asian Association for Regional Cooperation?
Congregation. She was awarded Nobel prize for Peace (a) Afghanistan (b) Bangladesh
in 1979. (c) Nepal (d) Pakistan

42th BPSC (Pre) Exam. 1997-98 65 YCT


Ans. (a) : The South Asian Association for Regional Ans. (c) : The speed at which sunlight travels is actually
Cooperation (SAARC) was established in 1985, in equal to the speed of light. The time taken by sunlight to
Dhaka. It is headquartered at Kathmandu, Nepal. At reach the surface of earth is about 8 minutes and 20
present it has 8 member countries namely India, seconds.
Bangladesh, Bhutan, Maldives, Nepal, Pakistan, 106. To make the steel hard, it is increase
SriLanka and Afghanistan. At the time when the (a) the quantity of carbon
question was asked Afghanistan was not the member of (b) the quantity of manganese
SAARC. (c) the quantity of silicon
99. The Prasar Bharti bill in 1997 was put on the (d) the quantity of chromium
table of parliament by Ans. (a) : Carbon is present in all steel and is the
(a) Jaipal Reddy (b) Sushama Swaraj principal hardening element. The hardness of steel rises
(c) Santosh Mohandev (d) L K Advani as the Carbon content of steel is increased. Chromium is
Ans. (a) : Prasar Bharti is established under the Prasar added to steel to increase resistance to oxidation.
Bharti Act, 1990 and came into existence on in 1997. 107. The technique of chromatography is used to
Prasar Bharti Bill was put on the table of Parliament in (a) identify colour substances
1997 by then the minister of Information and Broad (b) determine the structure of substance
casting, Jaipal Reddy. (c) dry distillation of colouring substances
100. River Cauvery flows through the states (d) separate the substances from a mixture
(a) Gujarat, Madhya Pradesh, Tamil Nadu Ans. (d) : Chromatography is a separation technique
(b) Karnataka, Kerala, Tamil Nadu which is used to separate the components of a mixture.
(c) Karnataka, Kerala, Andhra Pradesh 108. The nature of radiation of light is
(d) Madhya Pradesh, Tamil Nadu, Maharashtra (a) like wave
Ans. (b) : Cauvery river rises on Brahmagiri Hill of the (b) like particle
Western Ghats in southwestern Karnataka state, flows (c) like both of wave and particle
in a southeasterly direction for 800 km through the (d) like neither of wave not particle
states of Karnataka and Tamil Nadu, and descends the Ans. (c) : Light can be described both as a wave and as
Eastern Ghats in a series of great falls. a particle. The phenomenon like diffraction,
Its drainage basin includes state of Karnataka, Tamil polarisation, and interference could be explained by
Nadu, Kerala and Puducherry. considering light as a wave. The phenomenon like
101. In a car battery, electrolytes substance used is photoelectric effect is explained by assuming that light
(a) Hydrocloric acid consist of particular called Photon.
(b) Sulphuric acid 109. Dry ice is
(c) Nitric acid (a) Solid water
(d) Distilled water (b) Solid carbon dioxide
Ans. (b) : Battery electrolyte is the liquid substances (c) Waterless ice
found in most Car batteries. Electrolyte Substance used (d) Solid Hydrogen peroxide
in Car battery is Sulphuric Acid (H2SO4). It is strong Ans. (b) : Dry ice is the solid form of Carbon dioxide
Acid. CO2. Dry Ice is manufactured by compressing and
102. One picogram is equal to cooling gaseous CO2 under high pressure.
(a) 10–6gram (b) 10–9gram 110. Which gas of the following is responsible for
–12
(c) 10 gram (d) 10–15gram the erosion of Ozone Layer?
Ans. (c) : One Picogram is equal to 10–12 gram. (a) Carbon-dioxide
103. Milk is an example of (b) Carbon mono
(a) Solution (b) Colloid Solution (c) Nitrous oxide
(c) Emulsion (d) Air solution (d) Cloroflurocarbon
Ans. (c) : Milk is an emulsion of fat and water along Ans. (d) : Ozone (O3) depletion occurs when
with other components. In emulsion, one liquid is chlorofluorocarbons (CFCs) and halons are released
dispersed in the other. into the atmosphere. CFCs and halons gases formerly
104. The velocity of sound in air is approximately found in aerosol Spray cans and refrigerants.
(a) 10 km/sec (b) 10 mile/min 111. Iron is extracted from which of the following
(c) 330 m/sec (d) 3 × 1010 sec minerals?
Ans. (c) : The speed of sound in air is about 332 meter (a) Limestone (b) Pitch blend
per second at 0°C. The speed of sound in water is about (c) Monozite sand (d) Haematite
1480 m/s. Ans. (d) :
105. The time taken to reach the sunlight upto the List-I (Ore) List-II (element)
surface of earth is approximately Hematite Iron
(a) 4.2 s (b) 4.8 s Monazite Thorium, Lanthanum and cerium
(c) 8.5 min (d) 3.6 h Pitch blend Uranium
42th BPSC (Pre) Exam. 1997-98 66 YCT
112. The chemical name of Vitamin C is Ans. (d) : The chemical formula of vinegar is
(a) Ascorbic Acid (b) Thyamine CH3(COOH). Vinegar contains Acetic Acid. The
(c) Citric Acid (d) Tartaric Acid chemical formula for vinegar and Acetic Acid is same.
Ans. (a) : Chemical name of Vitamin C is Ascorbic Citric Acid is found naturally in all citrus fruits. Oxalic
Acid. Vitamin C helps in repairing tissue and the Acid are found in Spinach, Broccoli, Tomato etc.
enzymatic production of some neurotransmitters, 117. The energy on sun released due to
Vitamin C are found in Lemons, Oranges, Amla, (a) Nuclear Fission
Strawberry etc. Scurvy is caused due to the deficiency (b) Nuclear Fusion
of Vitamin C. (c) Oxidation Reactions
Vitamin Chemical Name (d) Reduction Reactions
Vitamin A Retinol
Ans. (b) : The energy is generated in the sun by Nuclear
Vitamin B1 Thiamine fusion. The fusion process occurs in the core of the Sun.
Vitamin D Calciferol Nuclear fusion is a reaction in which two atomic nuclei
Vitamin K Phylloquinone fuse together and releasing a large amount of energy. It
113. The domestic LPG Cylinders are not provided happens at very high temperature.
with barometers because.
118. Which of the following does not consist
(a) These are costly
carbon?
(b) They cannot demonstrate the quantity of gas
(a) Diamond (b) Graphite
in LPG cylinders
(c) Coal (d) None of these
(c) Their use is not safe
(d) They are choked by the LPG Ans. (d) : Graphite and Diamonds are allotropes of
Carbon. Coal is a combustible black or brownish-black
Ans. (d) : The domestic LPG cylinders are not provided
with barometers because they get choked by the LPG. Sedimentary rock with a high amount of Carbon and
LPG is comprised primarily of propane and butane. hydro carbons.
Ethyl Mercaptan is added for odor as LPG is odorless. 119. Sir C V Raman received Nobel Prize for
In LPG Cylinders gases are filled under pressure so that
Physics in the year
the gas remains in liquid.
(a) 1928 (b) 1930
114. The element found maximum in the soil-layer
is (c) 1932 (d) 1950
(a) Oxygen (b) Nitrogen Ans. (b) : The Nobel Prize in Physics was awarded to
(c) Mangnese (d) Silicon Sir C. V. Raman in 1930 to his work on the scattering of
Ans. (a) : The element found maximum in the soil layer light and for the discovery of the effect named after his
is Oxygen. The amount of oxygen in earth's crust is
name known as Raman effect.
46.1%, silicon 28.2%, aluminum 8.2% and Iron 5.6%.
115. The chemical formula of the Plaster of Paris is 120. Density of water is maximum
1 (a) at 100°C (b) at 4° C
(a) CaSO4 (b) CaSO4 H2O
2 (c) at 0° C (d) at –4° C
(c) CaSO4. H2O (d) CaSO4. 2H2O Ans. (b) : The maximum density of water is maximum
Ans. (b) : Plaster of Pairs (PoP) is prepared by heating at 4° Celsius, Water has maximum density of 1g/cm3 or
calcium sulfate dehydrate which is also known as 1000 kg/m3 at 4°C.
gypsum. Calcium sulphate with half a molecule of
water per molecule of the salt (hemi-hydrate) is called 121. Israel was established for
Plaster of Paris. So the chemical formula of PoP is (a) Budhists (b) Christians
1 (c) Jews (d) Muslims
CaSO4. H2O. PoP is used as building material, plaster
2 Ans. (c) : Israel was established in 1948 for Jewish
in orthopedics, filling material by dentist etc. people.
116. Which acid of the following is found in
122. HUDCO stands for
vinegar?
(a) Housing and Urban Development
(a) Hydrochloric acid Corporation
(b) Citric acid (b) Himachal United Development Company
(c) Oxalic acid (c) Human Urban Distribution Company
(d) Acetic acid (d) Hissar Urban Distribution Community
42th BPSC (Pre) Exam. 1997-98 67 YCT
Ans. (a) : HUDCO stands for Housing and Urban President's rule can be proclaimed under Article 356 on
Development Corporation. It is a public sector two grounds one mentioned in Article 356 itself and
company, under the Ministry of Housing and Urban another in Article 365.
Affair.
128. US Secretary of State of America is
123. Two states talking for the surrender of sandal
(a) Hillary Rodham Clinton
wood smugglar Veerappan were
(a) Andhra Pradesh and Kerala (b) Madeleine Albright
(b) Bihar and Bengal (c) Robin Cook
(c) Karnataka and Tamil Nadu (d) Tariq Altaf
(d) Gujarat and Maharashtra Ans. (b) : When question was asked Madeleine
Ans. (c) : Veerappan was charged with Sandalwood Albright was the U.S. Secretary of State. At present
smuggling and poaching of elephants in the scrub lands Antony J. Blinken is the 71st U.S. Secretary of State.
and forests in the states of Tamil Nadu, and Karnataka. 129. Chairman of Rajya Sabha is
Karnataka and Tamil Nadu Governments were talking (a) the Vice-president of India
for the surrender of Veerappan. (b) the Chief Justice of Supreme court
124. Geet Sethi is associate with which sports (c) Chief Election Commissioner
(a) Basketball (b) Chess (d) the Prime Minister of India
(c) Snoocker (d) Tennis Ans. (a) : Article 89 of Indian constitution is related
Ans. (c) : Geet Sethi is a very famous Indian Snooker with the chairman and deputy chairman of Rajya Sabha.
player. He was awarded with Major Dhyan Chand Khel Vice President of India is an ex-officio chairman of
Ratna award for the year 1992-93. He was awarded Rajya Sabha, while the deputy chairman of Rajya Sabha
Padma Shri and Arjuna Award in 1986. is one who is elected from amongst Rajya Sabha
125. Women's Day is celebrated on member. The Vice President receives a salary in the
(a) 8 March (b) 14 February capacity of the ex-officio chairman of Rajya Sabha.
(c) 10 May (d) 2 October 130. Where is the dictatorship of Khmer Rouge
Ans. (a) : Women's Day is celebrated every year on 8th (a) Cambodia (b) Indonesia
March around the world. Theme of International (c) Malaysia (d) Thailand
Women's Day 2022 was "Gender equality today for a Ans. (a) : The Khmer Rouge is the name that was
sustainable tomorrow". 14th February is known as popularly given to members of communist party of
black day for India on account of the brutal Pulwama Kampuchea and by extension to the regime through
attacks. Mother's day is celebrated on 10th May every which CPK ruled Cambodia.
year. 2nd October is celebrated as the International Day 131. Famous ecological specialist Sunderlal
of Non-violence. Bahuguna is known for his work
126. Last Kumbh Mela of 20th century was held at (a) Chipko Andolan (b) Narmada Bachao
(a) Allahabad (b) Ayodhya (c) Satyagrah (d) Anti-corruption
(c) Haridwar (d) Varanasi Ans. (a) : Chipko movement originated in 1973 in
Ans. (c) :The last Kumbh Mela of 20th century was held Chamoli district of Uttarakhand (then Uttar Pradesh).
at Haridwar in 1998. Kumbh Mela has been inscribed This movement was led by Sundar Lal Bahuguna and
on the UNESCO'S representative list of Intangible Chandi Prasad Bhatt. Medha Patekar had led the
cultural Heritage of Humanity. Narmada Bachao Andolan.
127. On whose advice president is rule is imposed in 132. The Oscar Award for best film in 1998 was
a State? given to
(a) Chief Minister (a) Fujitine (b) Jurasic park
(b) Legislative Assembly (c) Star warsaga (d) Titanic
(c) Governor Ans. (d) : 'Titanic' had won the Oscar for best film in
(d) Chief Justice of High Court 1998. While in 2022 'Coda' has won the Oscar for Best
Ans. (c) : Article 356 of the Indian constitution is film.
related to the President rule. Article 356 states that 133. Two prominent parties which largely
president's rule is imposed by the President on the contributed in the Russian Revolution of 1917
recommendation of the Governor of the state. The are
42th BPSC (Pre) Exam. 1997-98 68 YCT
(a) Congress and Janta dal 139. Which country won the Sharjah Cup in 1998?
(b) Conservative and Labour parties (a) Australia (b) India
(c) Democratic and Republic parties (c) New Zealand (d) Pakistan
(d) Bolshevik and Mensheviks parties Ans. (b) : India had won Sharjah Cup in 1998 after
Ans. (d) : Bolshevik and Menshevik were the two beating Australia by 6 wicket.
prominent parties which largely contributed in the 140. The election for 12th Lok Sabha of India held
Russian Revolution of 1917. in
134. Who is appointed as the governor of Bihar by (a) April, 1996 (b) June, 1996
Atal Bihari Vajpayee Government? (c) April 1997 (d) Feb, 1998
(a) Sunder Lal Bhandari Ans. (d) : General election of 12th Lok Sabha were held
(b) Sunder Lal Patwa in India in February 1998. The 17th Lok Sabha was
(c) Swaraj Kaushal formed by general election held in the year 2019.
(d) Pramod Mahajan 141. Find the average of first 10 Prime numbers–
Ans. (a) : Sunder Lal Bhandari was appointed by (a) 12.7 (b) 13
Governor of Bihar during the reign of Atal Bihari (c) 12.9 (d) 12.8
Vajpayee. He took the office of governor on 27th April Ans. (c) : The first 10 prime numbers are-
1998. 2,3, 5, 7, 11, 13, 17, 19, 23, 29
At Present, Phagu Chauhan is the Governor of Bihar. 2 + 3 + 5 + 7 + 11 + 13 + 17 + 19 + 23 + 29
Average =
135. Karunanidhi is the Chief Minister of 10
(a) Andhra Pradesh (b) Karnataka 129
(c) Kerala (d) Tamil Nadu =
10
Ans. (d) : M. Karunanidhi served as Chief Minister of = 12.9
Tamil Nadu for almost two decades over five terms
142. A man buys 11 books in `10 and sells 10 book
between 1969 and 2011. M.K. Stalin is the current
in ` 11. find his profit percent percent of the
Chief Minister of Tamil Nadu.
following.
136. New name of Madras is (a) 12% (b) 12.5%
(a) Chennai (b) Karmavati nagar (c) 21.5% (d) 21%
(c) Mumbai (d) Rajpura
Ans. (d) : a = 11 books
Ans. (a) : Chennai was previously called Madras. The b = 10 books
Government of Tamil Nadu officially changed the name
a 2 − b2
of Madras to Chennai in August 1996. profit percent = ×100
b2
137. Who fought against the Aparthied policy in
South Africa for longer time?  112 − 102 
= ×100 = 21%
(a) KanethKonda (b) Idi Amin  102 
 
(c) Yasir Arafat (d) Nelson Mandela
143. Measure the difference of 5 miles and 8 km in
Ans. (d) : Nelson Mandela was a South African anti yard:
apartheid revolutionary, political leader and (a) 51.1 (b) 5
philanthropist who served as president of South Africa (c) 52.2 (d) 52
from 1994 to 1999. He was sentenced to life
Ans. (a) : Since we know that,
imprisonment in 1962 for his political activism by the 1 Mile = 1760 yards. &
South African establishment. He was free from prison 1 kilometer = 1093.613 yards
after 27 years and became the face of the anti-apartheid Hence 5 Mile = 5×1760 = 8800 yards and
movement. 8 Km. = 8×1093.613 = 8748.91 yards
138. Who is the Prime Minister of England–? Thus difference = 8748.91– 8800 = 51.09 ≈ 51.1
(a) Bill Clinton (b) John F Canedy 144. If one factor of x3 – a2x + x + 2 is x – a, then the
(c) Benjamin Netanyahu (d) Tony Blair value of a is
Ans. (d) : When question was asked Tony Blair was the (a) 1 (b) 2
Prime Minister of the United Kingdom (1997-2007). (c) –2 (d) –1

42th BPSC (Pre) Exam. 1997-98 69 YCT


Ans. (c) : Given expression Ans. (c) : Given
∵ (x - a) is the factor of x3–a2x + x + 2 Sides of triangle = 11 meter
then we can say that x–a = 0 60 meter &
x=a 61 meter
(61)2 = (11)2 + (60)2
now putting the value of x = a in expression
3721 = 121 + 3600
Remainder = 0 3721 = 3721
a3– a2 × a + a + 2 =0 It means the given triangle will be a right angle triangle.
a3 – a3 + a + 2 = 0
a+2=0
a = –2
145. If A and B are such sets that 18 members are in
A ∪ B , 8 members in A and 15 members in B,
then find the number of members in A ∩ B. 1
Area of triangle = base × height
(a) 0 (b) 5 2
(c) 23 (d) 26 1
× 60 × 11 = 1/2 ×61×h
Ans. (b) : Given 2
n(A ∪ B) = 18 660
h=
n(A) = 8 61
n(B) = 15 h = 10.8
To find n(A ∩ B) = ? 149. Difference of two positive numbers is 72 and
dividend dividing second by the first is 4. Find
n(A∪B) = n(A) + n(B) – n(A∩B)
numbers.
18 = 8 + 15 –n (A∩B) (a) 40,112 (b) 24,96
18 = 23 – n(A∩B) (c) 32,128 (d) 30,102
n(A∩B) = 23 – 18 Ans. (b) :
Let assume the two positive number are = x and y
n(A∩B) = 5
and as per question the difference between them is = 72
146. Number of twin prime number pairs between Hence y–x = 72 ––––(i)
70 and 100 are and by dividing 2nd by first is
(a) 1 (b) 0 y/x = 4
(c) 2 (d) 3 thus y = 4x
now putting the value of y in equation (i)
Ans. (a) : The twin prime numbers pair between 70 and 4x–x = 72
100 are 71 and 73. 3x = 72
Hence total number of twin prime number pair between x = 24
70 and 700 is 1. and y = 4 × 24
y = 96
147. Find the sixth number in the series 1, 5, 13, 29,
Hence numbers are 24 and 96.
61.......
150. The diameter of a solid sphere of copper is 6
(a) 90 (b) 103 cm, it is homogenously moulded and bent to
(c) 109 (d) 125 make wire of diameter 0.2 cm. The length of
Ans. (d) : wire is
(a) 36 m (b) 360 m
(c) 24 m (d) 360 m
Ans. (a) : The volume of sphere having diameter 6 cm
= 4/3 πr3 = 4/3 π(3)3
Hence 6th number will be 125. The volume of sphere having diameter 0.2 cm = πr2 ℓ
= π (0.1) 2 ℓ
148. If the sides of a triangle are 11 m, 60 m and 61 as per question–
m, then find the height of the triangle in respect 4/3 π (3)3 = π (0.1)2 ℓ
of largest side. 4/3 π ×3×3×3 = π ×0.1×0.1× ℓ
(a) 10 m (b) 9 m 36 = 0.01 ℓ
(c) 10.8 m (d) 9.8 m ℓ = 3600 cm = 36 m

42th BPSC (Pre) Exam. 1997-98 70 YCT


43th Bihar Public Service Commission
Preliminary Examination, 1999
GENERAL KNOWLEDGE & GENERAL SCIENCE
(Solved Paper with Detail Explanation)
1. The author of 'Svapnavasavadattam' is Ans. (a) : Krishna Deva Raya was the greatest of
(a) Kalidasa (b) Bhasa Vijayanagar rulers. He was also known as Abhinava
(c) Bhavabhuti (d) Rajshekhara Bhoja, Andhra Pitamah and Andhra Bhoja.
Ans. (b) : He defeated the rulers of Deccan Sultanate which
created disintegration of Bahmani kingdom. Quli
Author Book
Qutubshah was the founder of Qutushahi dynasty, was
Bhasa Savpnavasavadattam, defeated by Krishna Deva Raya.
Pratijnayaugandharayana
6. Padmini is associated with Alauddin's
Kalidasa Meghaduta, Kumarasambhavam, conquest of Chittor. She was the wife of
Ritusamhara (a) Maharana Pratap Singh
Bhavabhuti Uttararamacharitam, (b) Ranjit Singh
Mahaviracharita (c) Raja Man Singh
Rajshekhara Karpuramanjari, (d) Rana Ratan Singh
Viddhasalabhanjika, Kavyamimansa Ans. (d) : In 1303 CE, Allaudin besieged Chittor.
2. King Kharavela figures in According to some Scholars, he attacked Chittor
(a) Girnar Pillar Inscription because he coveted Padmini, the beautiful queen of
(b) Junagadh Pillar Inscription Raja Ratan Singh. The Rajput women along with Rani
(c) Hathigumpha Inscription Padmini performed Jauhar. This Padmini episode was
(d) Sarnath Inscription graphically mentioned in the book Padmavath, written
Ans. (c) : Hathigumpha Inscription is the main source by Malik Muhammad Jayasi.
of information about Kalinga ruler Kharavela. 7. Moplah Rebellion in 1921 was an offshoot of
Hathigumpha inscription consists of seventeen lines (a) Khilafat Movement
incised in deep cut Brahmi letters, on a natural cavern (b) Mutiny of 1857
called Hathigumpha in the Southern side of the (c) Swadeshi Movement
Udayagiri hill, near Bhubaneswar. (d) Non-Co-operation Movement
3. The author of 'Prithviraj Raso' is Ans. (a) : Moplah Revolt was the culmination of a
(a) Kalhana (b) Bilhana series of riots by Mappila Muslims of Kerala against the
(c) Jayanaka (d) Chand Bardai British and Hindu Landlords in Malabar (Northern
Ans. (d) : Prithviraj Raso is an epic poem in Brajbhasa Kerala). In 1921, Moplah revolt was merged with the
about the life of Chahamana king Prithviraj Chauhan. It ongoing Khilafat agitation.
was composed by Chand Bardai. 8. At Wardha on 6th July, 1942, at the Working
4. Khajuraho temple architecture is associated Committee of the Congress, Mahatma Gandhi
with discussed for the first time Quit India Policy.
(a) Chandellas (b) Gurjar-Pratiharas The President of the Committee at that time
(c) Chahamanas (d) Paramaras was
Ans. (a) : The temples at Khajuraho were built during (a) Rajagopalachari
the Chandela dynasty between 950 AD and 1050 AD. (b) Maulana Abul Kalam Azad
Chandela dynasty was founded by Nannuka. He ruled in (c) Pandit Jawaharlal Nehru
the Jejakabhukti region. (d) Dr Annie Besant
5. Krishnadevaraya, the king of Vijaynagar Ans. (b) : In July 1942, Congress Working Committee
fought the war with which ruler of Golconda met at Wardha and resolved that it would authorise
(a) Quli Qutub shah Mahatma Gandhi to take charge of non violent mass
(b) Qutub-uddin-Aibak movement. The resolution generally referred to as the
(c) Ismail-Aadil Khan Quit India resolution. The President of this committee
(d) Gajpati was Maulana Abul Kalam Azad.

43th BPSC (Pre) Exam. 1999 71 YCT


9. Masnavi the collection of Muslim law written 16. The real name of the 'Frontier Gandhi' is
by Babur is Known as (a) Purushottam Das Tandon
(a) Mubaiyan (b) Netaji Subhas Chandra Bose
(b) Diwan (c) Khan Abdul Ghaffar Khan
(c) Turkey Treatise on Prosody (d) Maulana Abul Kalam Azad
(d) Baburnama
Ans. (c) : Khan Abdul Gaffar Khan was nicknamed as
Ans. (a) : Masnavi, the collection of Muslim law by Frontier Gandhi. He is also known as Bacha Khan. He
Babur known as Mubaiyan. He wrote his autobiography
founded Khudai Khidmatgar movement in 1929.
which he called Baburnama. It is written in Chagatai
Turk. 17. The Banaras Session of August 1923 of the
10. The meaning of jihad carried on by Aurangzeb Hindu Mahasabha was presided over by
is (a) Swami Shraddhanada
(a) Darul-Harb (b) Dar-ul-Islam (b) Rajendra Prasad
(c) Holy War (d) Jiziya (c) Lala Lajpat Rai
Ans. (b) : The meaning of Jihad carried on by (d) Pandit Madan Mohan Malviya
Aurangzeb was Dar-ul-Islam. In April 1679, Aurangzeb Ans. (a) : The Banaras Session of August 1923 of
made a public proclamation of Jihad against Hindus Hindu Mahasabha was presided over by Pt. Madan
with the objective of converting the land of infidels into Mohan Malviya.
Dar-ul-Islam.
18. Dayanand Saraswati Founded
11. The novel Durgeshnandini is written by (a) Brahmo Samaj
(a) Rabindranath Tagore
(b) Arya Samaj
(b) Taraknath Gangopadhyaya
(c) Prarthana Samaj
(c) Swarma Kumari
(d) Bankim Chandra Chatterjee (d) Bahujan Samaj
Ans. (d) : Durgeshnandini is a Bengali historical Ans. (b) : Arya Samaj was established by Swami
romance novel written by Bankim Chandra Chatterjee Dayanand Saraswati in 1875. Arya Samaj was a
in 1865. vigorous reform movement of modern Hinduism. It's
aim was to re-establish the Vedas as they revealed truth.
12. Nalanda University was founded during the
dynasty of Brahmo Samaj was founded by Raja Ram Mohan Roy
(a) Maurya (b) Kushan in 1828.
(c) Gupta (d) Pala Prathana Samaj was founded by Atma Ram Panduranga
in 1876.
Ans. (c) : Nalanda University was founded by
Kumaragupta –I of the Gupta Dynasty in the 5th Century 19. With the news of Gandhiji and other Congress
CE. leaders arrest in 1942, a riot in Bihar took
place, leading to a total breakdown in the rail
13. Nalanda University was destroyed by services. The most affected area was
(a) Muslims (b) Kushans (a) Munger (b) Gaya
(c) Sythians (d) Mughals (c) Patna (d) Shahabad
Ans. (a) : In 1193, Nalanda University was destroyed Ans. (c) : With the news of Gandhiji and other congress
by Muhammad bin Bakhtiyar Khilji, a Turkish Military leaders arrest in 1942, a riot in Bihar took place, leading
general of Qutub-ud-din-Aibak. to a total breakdown in the rail service. The most
14. Vikramshila, the learning centre was founded affected region was Patna.
by 20. The Mutiny of 1857 at Patna was led by a
(a) Devapala (b) Dharmapala dynamic old person known as
(c) Nayanpala (d) Narendrapala (a) Hyder Ali Khan
Ans. (b) : Vikramshila was founded by Pala emperor (b) Rajput Kunwar Singh
Dharmapala. It was established due to supposed decline (c) Judhar Singh
in scholarship at Nalanda. (d) Kusal Singh
15. Epigraphic records (Inscriptions) suggest that Ans. (b) : The Mutiny of 1857 at Patna, Bihar was led
in ancient India the rulers of Bihar had contact by Babu Kunwar Singh. He was nearly 80 years old
with when he took charge of the sepoys who were stationed
(a) Burma (b) Thailand at Danapur on 25 July, 1857.
(c) Combodia (d) Java-Sumatra 21. The Indian National Congress was formed in
Ans. (d) : Epigraphic records (Inscriptions) suggest that the year
in ancient India, the rulers of Bihar had contact with (a) 1865 (b) 1867
Java Sumatra. (c) 1885 (d) 1887
43th BPSC (Pre) Exam. 1999 72 YCT
Ans. (c) : Indian National Congress was founded by 27. In 1908, Bal Gangadhar Tilak was imprisoned
A.O Hume, a retired Civil Servant in 1885. The first for
Session of INC was held at Gokuldas Tejpal Sanskrit (a) 5 years (b) 6 years
College in Bombay in December, 1885. It was presided (c) 7 years (d) 8 years
over by Womesh Chandra Bonnerjee. Ans. (b) : In 1908, Bal Gangadhar Tilak was
22. Which one of the following was not among the imprisoned for six years and was sent to Mandalay in
moderates? Burma.
(a) GK Gokhale
28. Most of the moderate leaders hailed from
(b) Bal Gangadhar Tilak
(c) R C Dutt (a) rural areas
(d) W.C. Bonnerjee (b) urban areas
Ans. (b) : Lokmanya Bal Gangadhar Tilak belonged to (c) both rural and urban areas
the Radicals faction. He along with Lala Lajpat Rai and (d) Punjab
Bipin Chandra Pal, was part of the Lal-Bal-Pal trio of Ans. (b) : Most of the Moderates Leaders of India's
Leaders with extremist outlooks. freedom struggle may be traced to hail from urban
23. Indian National Movement came to be areas.
dominated by the extremists after 29. The centre of 1857 revolt in Bihar from 15th
(a) 1906 (b) 1909 July, 1857 to 20th January, 1858 was
(c) 1914 (d) 1919 (a) Rampur (b) Hamirpur
Ans. (a) : The Partition of Bengal in 1905 provided the (c) Dheerpur (d) Jagdishpur
flare for the rise of Extremist in the Indian National
Congress (INC). INC was dominated by extremist after Ans. (d) : The Centre of revolt of 1857 was Jagdishpur
1906. The extremist phase of Indian nationalism is from in Bihar between July 15, 1857 to January 20, 1858
1905 to 1920. The difference between the moderates where Zamindar Babu Kunwar Singh led and
and the extremist became official in the Surat Session of established his own government. The revolt of 1857 in
the INC in 1907. Bihar was suppressed by William Taylor and Major
Vincent Eyer.
24. Swaraj Party was formed after the failure of
the 30. On 6th August, 1942 the two arrested leaders
(a) Non-cooperation Movement of Hazaribag were
(b) Quit India Movement (a) Ramlal and Dvilal
(c) Civil Disobedience Movement (b) Narayan Singh and Sukhlal Singh
(d) Swadeshi Movement (c) Ramnath and Devnath
Ans. (a) : Swaraj Party (also known as Congress (d) Shiv Kumar and Ramanand
Khilafat Party) was formed on 1 January, 1923 by Ans. (b) : Quit India Movement had been approved by
Indian politicians and members of INC who had INC on 8th August 1942. On 6th August 1942 Gandhiji
opposed Mahatma Gandhi's suspension of Non- and other main Leaders of INC were arrested by British
cooperation movement on 12 February, 1922 in Government. Narayan Singh and Sukhlal Singh were
response to the Chauri Chaura incident. CR Das was the
arrested from Hazaribagh (Patna).
President and Motilal Nehru was the Secretary of this
Party. 31. The name of the wife of Shri Jagat Narayan
25. INA was formed in Lal was
(a) Burma (b) Japan (a) Mrs. Ram Pyari (b) Mrs. Sundari
(c) Singapore (d) England (c) Mrs. Bhagawati Devi (d) Mrs. Mangala Devi
Ans. (c) : Indian National Army also known as Azad Ans. (a) : Mrs. Rampyari Devi was the wife of Jagat
Hind Fauz was founded by Mohan Singh in Singapore Narayan Lal. During the Quit India Movement she had
in 1942. INA aim was to secure Indian independence addressed the assembly in Bankipur and called people
from British rule. to quit government Jobs.
26. Who was elected as President of the All India
32. Dr. Rajendra Prasad was arrested on 9th
Khilafat Conference in 1919?
August, 1942 and was sent to
(a) Mahatma Gandhi (b) Jawaharlal Nehru
(c) Shaukat Ali (d) SC Bose (a) Camp Jail (b) Hazaribagh Jail
Ans. (a) : All India Khilafat Confrence held in Delhi in (c) Bhagalpur Jail (d) Bankipur Jail
November 1919. Mahatma Gandhi was elected as the Ans. (d) : In the early hours of 9th August 1942 all the
President of All India Congress Committee. He saw in top leaders of INC were arrested and taken to unknown
the issue a platform from which mass and united non- destination. The District Magistrate of Patna W.G.
cooperation could be declared against the British Archer arrested Dr. Rajendra Prasad and he was sent to
Government. Bankipur Jail.

43th BPSC (Pre) Exam. 1999 73 YCT


33. Jai Prakash 'Divas' was celebrated in 40. Rambilas Singh, an active member of Forward
(a) January, 1946 (b) February, 1946 Bloc Belonged to the district of
(c) March, 1946 (d) April, 1946 (a) Jamshedpur (b) Darbhanga
Ans. (d) : In April 1946, Jai Prakash Diwas was (c) Patna (d) Munger
organised for the release of Jai Prakash Narayan from Ans. (d) : Rambilas Singh belonged to the Munger
prison. district of Bihar. He was an active member of Forward
34. In which year was the Indian National Army Bloc. The All India Forward Bloc was established in
established? June 1939 by Netaji Subhash Chandra Bose.
(a) 1940 (b) 1941 41. Community Development Programme was
(c) 1942 (d) 1943 started by Government of India on October, 2
Ans. (c) : Indian National Army (INA) was an armed (a) 1950 (b) 1951
force formed by Indian nationalists and Imperial Japan (c) 1952 (d) 1953
in 1942 in South East Asia during World War–II. Ans. (c) : Community Development Programme was
35. The founder of the Indian National Congress launched by Government of India on 2nd October 1952.
was a It was a multi project programme, with the aim of an
(a) Civil servant overall development of people.
(b) Scientist 42. In a recent decision of the Union Cabinet
(c) Social worker number of sub-castes from 17 states, add to the
(d) Military Commander list of OBC Group is
(a) 116 (b) 126
Ans. (a) : Indian National Congress was established by
A.O. Hume, a retired English Civil Servant in 1885. (c) 136 (d) 146
The first Session of INC was organised in December Ans. (c) : The Union Cabinet, in one of its decision had
1885, was presided over by Womesh Chandra added 136 Sub castes of 17 states to the list of Other
Bonnerjee. Backward Castes (OBC). In this the states of Rajasthan,
Bihar, U.P., M.P., Karnataka, Andhra Pradesh, Tamil
36. Bhagat singh, Rajguru and Sukhdev were
Nadu, Punjab, Maharashtra etc. and states of eastern
executed in March India were included.
(a) 1911 (b) 1921
43. Land Reform comes under the subjects of
(c) 1931 (d) 1941 (a) Union List (b) Concurrent List
rd
Ans. (c) : On 23 March 1931, Bhagat Singh, Rajguru (c) State List (d) None of these
and Sukhdev were hanged to death by the Britishers for Ans. (c) : As per Indian Constitution, Land Reform is a
assassinating John Saunders at the Lahore jail. Every part of State list, So the responsibility for bringing up
year 23rd March is observed as Martyrs Day. This day is regulations for effective land reforms lies with
also known as Shaheed Diwas or Sarvodaya Day. individual States.
37. Who had shot dead General dyer in London? 44. Bihar State has highest number of people
(a) BC Pal (b) Khudiram below the poverty line due to
(c) Madan Lal (d) Udham Singh (a) failure of land reforms
Ans. (d) : Udham Singh assassinated Michel O' Dwyer, (b) lack of political will
the lieutenant – Governor who presided over the brutal (c) large number of labour class
British Suppression of the 1919 protest in Punjab. He (d) All of the above
was hanged in 1940 for his deed. Ans. (d) : Bihar is India's poorest and one of the most
backward states. Due to lack of political will, failure of
38. The Hoz revolt took place during land reforms and large number of labour class lead the
(a) 1620-21 (b) 1720-21 Bihar state as the highest number of people below the
(c) 1820-21 (d) 1920-21 poverty line.
Ans. (c) : Hoz was a tribal revolt that took place during 45. Capital Adequacy Ratio for banks fixed by
1820-1821 in Bihar. Government of India is
39. The king of Jagdishpur was (a) 5% (b) 6%
(a) Nana Saheb (b) Tantia Tope (c) 7% (d) 8%
(c) Lakshmi Bai (d) Kunwar singh Ans. (d) : Capital Adequancy Ratio (CAR) is the ratio
of bank's capital in relation to its risk weighted assets
Ans. (d) : Babu Kunwar Singh was the king of and current liabilities. As per Basel-I Norms CAR was
Jagdishpur (Ara). He was the chief organiser of the fight 8% for the Banks. Presently Basel III have prescribed
against the British in Bihar. He led the revolt of 1857 in CAR for Bank is 8% but RBI mandates the CAR for
Bihar. Scheduled commercial banks to be 9%.
43th BPSC (Pre) Exam. 1999 74 YCT
46. Which one of the following Financial 51. In Demography, Sex ratio refers to
Institutions is not affiliated to World Bank (a) number of females in a country
Group? (b) number of males in a country
(a) IMF (b) IBRD
(c) ratio of females per thousand males
(c) ADB (d) IDA
(d) ratio of males per thousand females
Ans. (c) : Among the above given options, Asian
Development Bank (ADB) is not affiliated to World Ans. (c) : Sex ratio is defined as the number of females
Bank Group. ADB was established in 1966. It is a per 1000 males in a given population. As per census
regional development bank which is committed to 2011, in India there are 943 females per 1000 males.
achieving a prosperous, inclusive, resilient and sustain 52. Decade-wise growth rate of population in India
noble Asia and the Pacific, while sustaining its efforts to between 1981 and 1991, compared to 1971 and
eradicate extreme poverty.
1981 has marginally
47. At present country has adopted (a) increased
(a) partial convertibility of Indian Rupee
(b) remained constant
(b) full convertibility of rupee
(c) full convertibility in current account (c) decreased
(d) full convertibility in capital (d) increased at decreasing rate
Ans. (c) : Convertibility implies a system in which Ans. (c) : The average annual population growth rate
country's currency becomes convertible in foreign shot up from round 1.25% during 1941-51 to very close
exchange and vice-versa. Since August 1994, Indian to 2% during 1951-61 and to all time high of more than
Rupee has been made fully convertible in current 2.2% during 1971-81. After 1981, the trend in
account transactions related to goods and services. Population growth rate was reversed. The decline was
48. MODVAT, which is to be implemented from slow during 1981-91 but accelerated during 1991-2000.
April, 2001 replaces 53. Deregulation of Interest rates of the Banks is
(a) Sales Tax based on the recommendation of
(b) Custom Duties
(a) Chellaiah Committee
(c) State Excise Duties
(b) Dantawala Committee
(d) Central Excise Duties
(c) Narasimhan Committee
Ans. (d) : Modified Value Added Tax (MODVAT)
was introduced into the Indian Tax system in 1986 but (d) None of these
was renamed as Central Value Added Tax (CENVAT) Ans. (c) : In 1991, the first Narsimha Committee made
with effect from 1st April, 2001. MODVAT was number of recommendations in order to overcome the
restricted upto manufacturing stage and credit was also defects of the Indian banking system. The
available of only excise duty that was to be paid on recommendations of this committee are Deregulation of
manufacturing products was available. Interest rates, Reduction in SLR, Reaching of 8%
49. Main objective of monetary policy in general is Capital Adequacy Ratio, Setting up Assets
to Reconstruction Fund etc.
(a) regulate money supply
(b) regulate private banks 54. India has to have plan holiday during
(c) regulate stock market (a) 1964-69 (b) 1966-69
(d) regulate bullion market (c) 1974-79 (d) 1979-79
Ans. (a) : Modified Policy is the control of the quantity Ans. (b) : The miserable failure of the Third Five year
of money available in an economy and the channels by Plan, the government of India had introduced 'Plan
which new money is supplied. The main objective of holidays' during 1966-69. There were three annual plans
monetary policy is to regulate money supply. Other introduced between 1966 to 1969.
objective of monetary policy are– Price Stability, 55. Regional Rural Banks called Grameen Banks
Economic growth and Exchange Rate Stability. are classified as
50. Double cropping agriculture means raising of (a) Land Development Banks
(a) two crops at different (b) Branches of Commercial Banks
(b) two crops simultaneously (c) Unscheduled Banks
(c) one crop along with other crop (d) Scheduled Commercial Bank
(d) None of the above Ans. (d) : Regional Rural Bank (RRB) in India are
Ans. (b) : Double cropping in agriculture means government owned Scheduled Commercial Banks. It
planting two crops in the same area and in the same ensure adequate credit for agriculture and other rural
crop year so that the same land is used to generate more sectors. RRBs banks are under the ownership of
than one crop per year. Ministry of Finance.
43th BPSC (Pre) Exam. 1999 75 YCT
56. Five year Plans are finally approved by 61. The first Hydel Power Station in India was
(a) National Development Council started at
(b) Parliament (a) Pykara (b) Koyna
(c) Planning Commission (c) Bhakra Nangal (d) Shivasamundra
(d) Ministry of Planning Ans. (*) : India's first hydroelectric power plant was
Ans. (a) : Five year plan in India finally approved by installed at Sidrapong near Darjeeling, W. Bengal in
National Development Council (NDC). The Draft of 1897. Hydro Power Station at Sivasamudram in
Five Year Plan Prepared by the Planning Commission Karnataka was commissioned in 1902.
(now NITI Aayog) is first submitted to the Union 62. Present rate of Cash Reserve Ratio announced
Cabinet, After its approval, it is placed before NDC, for by RBI recently is
its acceptance. (a) 11% (b) 10%
57. Planning Commission is a (c) 9% (d) 8%
(a) Constitutional Body Ans. (*) : Cash Reserve Ratio (CRR) is a Certain
(b) Adhoc Body minimum amount of deposit that the commercial banks
(c) Non-constitutional Body have to hold as reserve with Central Bank (RBI). CRR
(d) None of these is used to regulate money supply. As per RBI Monetary
Ans. (c) : Planning Commission is a extra Policy, the current rate of CRR is 4.5%.
Constitutional body. On 1 January 2015, the NITI 63. Finance Commission of India deals with:-
Aayog was established as the successor of Planning (a) finances between the states
Commission. NITI Aayog is the premier Policy think (b) finances between states and the centre
Tank of the Government of India. (c) finances between the centre of local self govt.
58. Hard currency means a currency (d) None of the above
(a) which is very hard to carry Ans. (b) : Finance Commission of India is set up under
(b) which is hard to earn Article 280 of the Indian Constitution. It defines the
(c) of developed nations financial relations between the Central and State
(d) of developing countries Governments.
Ans. (c) : Hard currency is a currency that is issued by a 64. Panchayati Raj was first introduced in
nation that is seen as politically and economically (a) Andhra Pradesh (b) Uttar Pradesh
stable. Hard currencies are widely accepted around the (c) Himachal Pradesh (d) Rajasthan
world as a form of payment for goods and services and Ans. (d) : Panchayati Raj System was inaugurated by
may be preferred over the domestic currency. the Prime Minister Jawahar Lal Nehru on 2nd October
59. NABARD provides loans to 1959 at Nagaur in Rajasthan.
(a) individuals for agriculture 65. Panchyati Raj is mainly aimed at
(b) institutions for rural development (a) increasing rivalries among villagers
(c) banks for agriculture and rural development (b) giving training to villagers for fighting
election
(d) state govts for agriculture
(c) decentralisation of power to villagers
Ans. (c) : National Bank for Agriculture and Rural
(d) None of the above
Development (NABARD) came into existance on 12
July, 1982. It promote sustainable and equitable Ans. (c) : Panchayati Raj System mainly aimed at
democratic decentralization of power to villagers.
agriculture and rural development through participative
Panchayati Raj institutions are meant to promote grass
financial and non-financial interventions, innovations,
root level democracy and development. It develop local
technology and institutional development for securing self governments in districts, zones and villages.
prosperity.
66. Panchayati Raj is organised as a unit of self
60. Indian currency is printed at govt. under
(a) RBI (a) Fundamental Rights of Indian Constitution
(b) Ministry of Finance (b) Preamble of the Indian Constitution
(c) Parliament (c) Directive Principles of State Policy
(d) Nasik Printing Press (d) 73rd Amendment of the Indian Constitution
Ans. (d) : Indian currency is Printed at four currency Ans. (d) : Panchayati Raj is organised as a unit of Self
presses, they are : Nasik, Mysore, Salboni and Dewas. government under 73rd Amendment of the Indian
Coins are minted in four mint owned by Government of Constitution. 73rd Constitutional Amendment Act 1992
India. The mints are located at Mumbai, Hyderabad, had added a new part IX consisting of 16 Article and
Calcutta and Noida. the 11th Schedule to the Indian Constitution.

43th BPSC (Pre) Exam. 1999 76 YCT


67. A separate Vananchal State carved out of 73. Westward flowing rivers are
Bihar can be (i) Narmada (ii) Tapi
(a) passing the legislation in State Assembly (iii) Rapti
(b) passing the ordinance by the Governor Codes
(c) Completing constitutional formalities (a) (i) and (ii) (b) (ii) and (iii)
(d) None of the above (c) (i) and (iii) (d) (i), (ii) and (iii)
Ans. (c) : A separate Vananchal state carved out of Ans. (a) : Narmada and Tapti are major west flowing
Bihar can be made by completing constitutional rivers in Peninsular India. Narmada originates from
formalities. Article 3 of Indian Constitution deals with Maikal range near Amarkantak in Madhya Pradesh and
the formation of new States and alteration of areas, meets the Gulf of Cambay in Arabian Sea.
boundaries or names of existing states. Tapti river originates from Multai reserve forest in
68. Present Lok Sabha Deputy Speaker is Madhya Pradesh. It flows for about 724 km before
(a) Mr. PM Sayeed falling into the Arabian Sea through the Gulf of
Cambay.
(b) Mrs. Najma Heptulla
(c) Mr. GMC Balayogi 74. National Environmental Engineering Research
Institute is at
(d) None of these
(a) Bangalore (b) Chotanagpur
Ans. (a) : When the question was asked, Mr. P.M. (c) Dehradun (d) Nagpur
Sayeed was the Deputy speaker of Lok Sabha. At this
time, there is no deputy Speaker of Lok Sabha. Ans. (d) : National Environment Engineering Research
Institute (NEERI)is located at Nagpur, Maharashtra. It
69. Mango shower is is devoted to research and innovations in environmental
(a) shower of mangoes science and engineering besides solving a range of
(b) dropping of mango problems posed by industry, public and government.
(c) rainfall in March-April in Southern State of 75. The diesel prices were recently raised because
India (a) to add to the income of the government
(d) crop of mango (b) wilful decision of the government
Ans. (c) : Mango showers are used to describe the (c) to meet the International price rates as a
occurrence of pre monsoon rainfall, they help in the follow-up action of the previous government
early ripening of mangoes. These rains are also known (d) None of the above
as April rains or summer showers. These rains normally Ans. (c) : At the time when the question was asked,
occurs from March to April in Southern State of India diesel prices were raised to meet the International price
especially in Kerala, Karnataka and Parts of Tamil rate as follow up action of the previous government. At
Nadu. present, fuel prices are reviewed on daily basis by oil
70. The culminating point of Eastern Ghats and companies which depends on the crude oil Price.
Western Ghats is 76. Who was the leader of the non-alignment
(a) Anaimudi (b) Nilgiri movement?
(c) Malayagiri (d) Anaimalai (a) China (b) India
Ans. (b) : The Western Ghats and Eastern Ghats meet (c) Pakistan (d) Indonesia
at the Nilgiri Hills in Tamil Nadu. Doddabetta is the Ans. (b) : Non Aligned Movement (NAM) was
highest peak in the Nilgiri hills in Tamil Nadu. established in 1961 in Belgrade under the leadership of
71. The order of Bihar according to population the then Indian Prime Minister Pt. Jawaharlal Nehru
density in India is with the help of, Gamal Abdel, President of Egypt and
(a) second (b) third Josip Broz Tito, the President of Yugoslavia. NAM is a
(c) eighth (d) ninth major international organization in which India has
played a key role since its foundation.
Ans. (*) : As per Census 2011, Bihar has highest
population density (1106 person/sq km) followed by 77. Which country in the world has declared itself
West Bengal (1028) and Kerala (860). as 'Hindu' state?
(a) India (b) Nepal
72. India produces more than its need
(c) Sri Lanka (d) Bhutan
(a) tea (b) foodgrains
Ans. (b) : At the time when question was asked, Nepal
(c) petroleum (d) petro-chemicals had declared itself as a 'Hindu State'. Nepal was
Ans. (a) : India produces tea more than its need. India is declared a Secular country in January 2007.
second largest producer of tea in the World after China. 78. Rameshwaram is a part of which state
Assam, W. Bengal, Tamil Nadu and Kerala are the (a) Andhra Pradesh (b) West Bengal
major tea producing states in India. (c) Tamil Nadu (d) Kerala
43th BPSC (Pre) Exam. 1999 77 YCT
Ans. (c) : Rameshwaram is a town on Pamban Island in Ans. (b) : The Swadeshi Movement was launched by
the Southeast Indian state of Tamil Nadu. Murli Manohar Joshi on 10th May 1999 at Kochi
Rameshwaram is famous for the Ramanthaswamy Kerala.
Temple which is a popular pilgrimage site for the 85. The Cabinet Minister for Railways is
followers of Hinduism. (a) Nitish Kumar (b) Ram Vilas Paswan
79. The Dadasaheb Phalke award for 1998 has (c) Mamta Banerjee (d) Anant Kumar
been given to
Ans. (c) : When the question was asked. Mamta
(a) BR Chopra (b) Yash Chopra
Banerjee was the Railway Minister of India. At Present
(c) RD Burman (d) Amitabh Bachchan Ashwini Vaishnav is the current Railway Minister of
Ans. (a) : Dadasaheb Phalke Award is honoured to the India.
recipient for their outstanding contribution to the
86. 'Linux' is
growth and development of Indian Cinema. In 1998,
this award has been awarded to B.R. Chopra. (a) operating system (b) a disease
(c) a chemical (d) a computer
80. The army chief responsible for the coup d'etat
in Pakistan recently is Ans. (a) : Linux is an operating System. It was released
(a) Gen. Parval musharraf in 1991 by Linux Torvalds.
(b) Gen. Parvez Musharraf 87. Which of the following is true about the PIO
(c) Gen. Parnaz Musharraf card scheme?
(d) Gen. Khwaja Ziauddin I. It refers to persons of Indian origin settled
Ans. (b) : General Parvez Musharraf led a military coup abroad
in 1999 and overthrew the democratically elected II. PIO card holders will need a visa
government in Pakistan. He declared himself 'Chief III. It covers the individuals in Bangladesh and
Executive' of the nation. A few year later. He became Pakistan too
the President of Pakistan. IV. They can get facilities for acquisition,
81. The Ministery of Information and holding, transfer and disposal of all immovable
Broadcasting has recently banned a channel properties anywhere in India, including
telecast from........ country, terming it as agriculture and plantation properties.
'pronographic' . codes
(a) United States (b) Russia (a) (i) and (ii) (b) (i),(ii) and (iii)
(c) Pakistan (d) Britain (c) Only (i) is true (d) All four are true
Ans. (a) : The Ministry of Information and Ans. (c) : A Person of Indian Origin (PIO) refers to
Broadcasting had banned a channel telecast from United person of Indian origin settled abroad except a national
States, termed it as pornographic. of Bangladesh, Afghanistan, Pakistan, China, Iran,
82. The Vajpayee 'Bus Diplomacy' relates to which Bhutan, Srilanka and Nepal. PIO card holders do not
country? require a visa to visit India for a period of 15 years from
(a) Nepal (b) Pakistan the date of issue of the PIO card. PIO card holders can
(c) China (d) Myanmar get facilities for acquisition, holding, transfer and
Ans. (b) : Vajpayee Bus Diplomacy is related to disposal of all immovable properties anywhere in India,
Pakistan. Delhi-Lahore bus service was started during except agriculture and plantation properties.
tenure of former Prime Minister Atal Bihari Vajpayee. 88. Which one country is not a permanent member
83. Which of the following countries is not a of the United Nations Organisation?
member of SAARC? (a) France (b) Germany
(a) Afghanistan (b) Myanmar (c) China (d) UK
(c) Maldives (d) Bhutan Ans. (b) : United Nation Organisation (UNO) was
Ans. (b) : South Asian Association for Regional established on 24 October 1945. It is currently made up
Cooperation (SAARC) is an economic and political of 193 member state. China, France, Russia, United
organization of eight countries in South Asia. It was Kingdom and USA are permanent members of UN.
established in 1985. Afghanistan, Bangladesh, Bhutan,
89. Who is the only Indian to get the title of Lord
India, Maldives, Nepal, Pakistan and Srilanka are the
member States of SAARC. While China and Myanmar by Great Britain?
have observer status in SAARC. (a) Swaran Singh (b) Swaraj Lal
(c) Swraj Paul (d) Dhanraj Pal
84. The Swadeshi Science Movement was launched
by Murli Manohar Joshi on May 10th 1999 at Ans. (c) : Swraj Paul is an Indian born British business
(a) New Delhi (b) Kochi tycoon and philanthropist, who get the title of Lord by
(c) Kottayam (d) Banglore Great Britain

43th BPSC (Pre) Exam. 1999 78 YCT


90. The silicon valley of India is 97. Dry ice is
(a) Hyderabad (b) Mumbai (a) solid water
(c) Bangalore (d) Chennai (b) solid carbon dioxide
Ans. (c) : Bangalore is known as the Silicon Valley of (c) dehydrated ice
India. Bangalore is an IT hub which attracts the best (d) solid hydrogen peroxide
domestic and international technology companies. Ans. (b) : Dry ice is a solid form of Carbon dioxide. It
91. The Editor of India Today is is called 'dryice' because it does not melt like wet ice
(a) Arun Shoulie (b) Aroon Sun instead, it converts into carbon dioxide gas.
(c) Aroon Purie (d) Anil Puri 98. Which lense is used to treat farsightedness?
Ans. (c) : Aroon Purie is the founder- Publisher and (a) concave lens (b) convex mirror
editor- in- chief of India Today. India Today is a weekly (c) convex lens (d) concave mirror
Indian English language news magzine. Ans. (c) : Farsightedness (Hyperopia) is a common
92. ....... has been appointed as the Indian cricket vision condition in which we can see distant objects
team coach in October, 1999 clearly, but objects nearby may be blurry. Convex
(a) Kapil Dev (b) Anshuman Gaekwad lenses are used to correct farsightedness.
(c) Ajit Wadekar (d) Sunil Gavaskar Concave lenses are used to correct nearsightedness
Ans. (a) : Kapil Dev was appointed the Indian Cricket (myopia).
team Coach in 1999. At Present Rahul Dravid is the 99. In which direction, the rainbow is seen at 12
Head Coach of Indian Cricket Team. noon?
93. In which events of the National Athletic meet, (a) In the West (b) In the South
PT Usha has won gold medals? (c) In the East (d) can't be seen
(a) 400 meters and 200 meters race Ans. (d) : Rainbow are seen when there are water
(b) 100 meters and 200 meters race droplets in the air and the sun is shining behind us. A
(c) 200 meters and 1500 meters race rainbow is located opposite to the Sun this explain why
(d) 1500 meters and 400 meters race rainbow are not seen at noon with the sun overhead.
Ans. (b) : PT Usha has won gold medal in 100 meters 100. For shaving one uses
and 200 meters race at National Athletic Meet. (a) Concave mirror (b) plane mirror
94. The chemical formula for washing soda is (c) convex mirror (d) None of these
(a) NaOH (b) Na2CO3 Ans. (a) : Concave mirror is used as shaving mirror
(c) NaHCO3 (d) Ca(OH)2 because when Concave mirror is placed very close to
the object, a magnified and virtual image is obtained.
Ans. (b) : Washing Soda's Chemical Formula is
Na2CO3. It is known as Sodium Carbonate, and it is a 101. Alloy made by which metal is used to make
Salt of Carbonic acid. aeroplane and in compartment of train?
95. Principle of fuse is (a) Copper (b) Iron
(a) Chemical effect of electricity (c) Aluminium (d) None to these
(b) mechanical effect of electricity Ans. (c) : Alloys of Aluminium are used to make
(c) heating effect of electricity aeroplane and parts in compartment of train as they are
(d) magnetic effect of electricity strong, lightweight, durable and inexpensive.
Ans. (c) : Electric fuse is based on the Principle of 102. From which State silver is obtained in India?
heating effect of electric current. The primary use of (a) Odisha (b) Andhra Pradesh
electric fuse is to protect electrical equipment from (c) Gujarat (d) Bihar
excessive current to prevent short circuit or mismatched Ans. (d) : When the question was asked, Silver was
load. obtained from Bihar. At present Silver is found in
96. Following particles are present in the nucleus Manbhum and Singhbhum in Jharkhand. Zawar mines
of an atom in Udaipur, Rajasthan is the largest Silver producing
(a) Proton and neutron mine in India.
(b) Electron and X-particles 103. Following which gas is used as tear gas?
(c) Proton and electron (a) H2 (b) SO2
(d) Electron and neutron (c) N2 (d) Cl2
Ans. (a) : The nucleus of an atom contains two type of Ans. (b) : Tear gas is a group of substance that irritates
subatomic particles- Protons and Neutrons. The Protons the mucous membrane of the eyes This causes a
have a positive electrical charge and the Neutron have stinging sensation and tears. Sulfur dioxide (SO2) gas is
no electrical charge. used as tear gas.

43th BPSC (Pre) Exam. 1999 79 YCT


104. The speed of light in vacuum is nearly Ans. (c) : Charles Darwin published a book on
(a) 3×1010 meters/sec (b) 3×108 meters/sec evolution in 1859 titled "the Origin of Species". In the
8
(c) 3×10 km/sec (d) 3×1010 light years book, he stated the 'Theory of Evolution by Natural
Ans. (b) : Light travels with maximum speed in Selection.
vacuum. Speed of light in vacuum is 3 × 108 meter/sec. 112. In laboratory who did synthesis of DNA?
105. Which acid is considered as basic chemical in (a) Milar (b) Khurana
Industry? (c) D-Veris (d) Kelvin
(a) H2CO2 (b) HNO2 Ans. (b) : Hargovind Khurana synthesized DNA in the
(c) H2SO4 (d) HCI laboratory. However, Arthor Kornberg was the first to
Ans. (c) : Sulfuric Acid (H2SO4) is the most commonly synthesize DNA in a test tube, which earned him a
produced industrial chemical in the world. It is used in Nobel Prize in 1959.
almost all industries like. Fertilizers, Gasoline, 113. Density of water is maximum at
Automobile batteries etc. (a) 4°C (b) 4K
106. Following which rays are used in the diagnosis (c) 4°F (d) –4°C
of intestinal diseases? Ans. (a) : The density of water is maximum at 4°C (Its
(a) X-rays (b) α-rays value is 1000 K gm–3). It is because, at 4°C two
(c) β-ray (d) γ-rays opposite effects are in balance.
Ans. (a) : X rays are used in the diagnosis of intestinal 114. The prime accused in the killing of Australian
disease. W.C. Roentgen had discovered the X rays in Missionary Graham Staines and his son is
December 1885. (a) Anand Pal (b) Arvind Pal
107. Male's gene constituent is (c) Dara Singh (d) None of these
(a) XX (b) XY Ans. (c) : In January 1999, when Australian Missionary
(c) X (d) Y and his two sons were killed, Dara Singh was found to
Ans. (b) : Every normal human cell contains 23 pairs of be prime accused.
Chromosomes. In humans, males generally have X and 115. The Pokhran II test was conducted on
Y Chromosome, While females have XX chromosome. (a) 11 June, 1998 (b) 9 June, 1998
A male's X chromosomes comes from his mother and Y
(c) 11 May, 1998 (d) 29 May, 1998
comes from his father.
Ans. (c) : Pokharan – II was the series of five nuclear
108. Which of the following blood group is universal
bomb test explosions that were conducted at the Army's
donor?
Pokharan Test Range on 11th May 1998. Pokharan -II
(a) B (b) O was conducted with the assigned code name 'Operation
(c) A (d) AB Shakti'.
Ans. (b) : Blood group 'O' is Universal donor. It is 116. 'The United Nations' General Assembly
because, in 'O' blood group, red blood cells do not carry adopted the Comprehensive Test Ban Treaty
antigens. AB blood group is called Universal recipients
(CTBT) on
because this blood group has no antibodies.
(a) 10 September, 1996 (b) 24 October, 1996
109. Photosynthesis occurs in (c) 10 August, 1996 (d) 1 May, 1996
(a) night (b) day and night
Ans. (a) : Comprehensive Nuclear Test Ban Treaty
(c) day or night (d) only day
(CTBT) was negotiated at the Conference on
Ans. (d or b) : Photosynthesis is the process by which Disarmament in Geneva and adopted by the United
plants use light energy to produce carbohydrates, such Nations General Assembly on 10th September, 1996.
as glucose and oxygen from carbon dioxide and water.
Photosynthesis occurs during the day when there is 117. Name the political party which is not a part of
Sunlight. Sunlight is not available at night time. the coalition Government at the Centre but,
Photosynthesis can also occur in night under artificial whose member has been elected as the Speaker
light. of the thirteenth Lok Sabha.
110. Which gas is essential for photosynthesis? (a) Telugu Desam Party
(a) O2 (b) CO (b) Samata Party
(c) N2 (d) CO2 (c) Janta Party
Ans. (d) : Kindly refer the explanation of above (d) None of these
question. Ans. (a) : G.M.C. Balayogi was the speaker of 13th
111. Who explained first the evolution of life? Lokshabha, who was the leader of Telugu Desam Party.
(a) Newton (b) Einstein Telugu Desam Party was not a part of the coalition
(c) Charles Darwin (d) Lamarck Government in 13th Loksabha.

43th BPSC (Pre) Exam. 1999 80 YCT


118. The winner of the 1998 Wimbledon Men's title Ans. (d) : The standard time refers to the fixed time for
is places falling in the same meridian, set in a country by
(a) Goran Ivanisevic (b) Pete Sampras the law. The Standard time of the place is defined by the
(c) Richard Krajicek (d) Andre Agassi distance of the meridian of a position from the prime
Ans. (b) : The winner of the 1998 Wimbledon Men's meridian. Indian Standard Time is 5 : 30 hours ahead of
Singles was Pete Sampras. Greenwich Mean Time. (GMT).
At Present, Novak Djokovic defeated Nick Kyrgios in 124. Bodos are inhabitants of
the final to win the Men's Single Tennis title at the 2022 (a) Garo hills (b) Santhal Pargana
Wimbledon Championship. (c) Amazon Basin (d) Madhya Pradesh
119. In which state five inscriptions belonging to Ans. (a) : Bodo tribes are inhabitants of Garo hills.
Twelth Century Rashtrakuta dynasty have Bodos are single largest community among the notified
been found? Scheduled Tribes in Assam.
(a) Tamil Nadu (b) Karnataka 125. Panama Canal connects
(c) Kerala (d) Maharashtra (a) North America and South America
Ans. (b) : In Karnataka, the five inscriptions belonging (b) Pacific Ocean and Atlantic Ocean
to 12th Century Rashtrakuta dynasty have been found. (c) Red Sea and Mediterranean Sea
Dantivarman II or Dantidurga was the founder of the (d) Indian Ocean and Pacific Ocean
Rashtrakutas dynasty. Rashtrakutas dynasty empire Ans. (b) : The Panama Canal is an 82 km artificial
encompass the entire modern state of Karnataka. waterway in Panama that connects the Pacific Ocean
120. Largest city is and Atlantic Ocean. It separates North and South
(a) Dhanbad (b) Gaya America.
(c) Patna (d) Ranchi 126. Mount Etna is
Ans. (c) : In term of population, Patna is the largest city (a) a mountain (b) a mountain peak
of Bihar (c) a volcano (d) a plateau
City Population Ans. (c) : Mount Etna is an active Strato volcano. It is
Jamshedpur – 631,364 located on the island of Sicily, Italy.
Dhanbad – 1,162,472 127. The foothills region of Himalayas is
Ranchi – 1,073,427 (a) Trans-Himalayas (b) Shivalik
Gaya – 468,614 (c) Greater Himalayas (d) Aravali
Patna – 5,838,465 Ans. (b) : Shiwalik is the foothill or southernmost
121. WTO comes into existence on region of the Himalayas. The Shiwalik hill ranges is a
(a) January, 1994 (b) January, 1995 part of Sub Himalayan mountain system extending from
(c) January, 1996 (d) January, 1997 Jammu and Kashmir on the west to Uttarakhand in east,
covering Himachal Pradesh and Parts of Punjab and
Ans. (b) : World Trade Organization (WTO) deals with
Haryana.
the global rules of trade between nations. Its main
function is to ensure that trade flows as smoothly, 128. The largest rubber producer state in India is
predictably and freely as possible. It was established on (a) Andhra Pradesh (b) Karnataka
1 January, 1995. Peter Sutherland was the first President (c) Kerala (d) Tamil Nadu
of WTO. Ans. (c) : Kerala is the largest rubber producing State in
India. India is world's third largest natural rubber
122. First President of World Trade Organisation
producer after Indonesia and Brazil.
was
(a) Arthur Dunkel (b) Peter Sutherland 129. Grand Canyon is
(a) a gorge (b) a large gun
(c) Runeto Ruggerio (d) Butros Butros Ghali
(c) a river (d) an old gun
Ans. (b) : Kindly refers the explanation of above
Ans. (a) : Grand Canyon is a gorge. It is a steep-sided
questions.
canyon carved by the Colorado River in Arizona,
123. The basis of deciding standard time of any United States. Uplift of the Colorado Plateau was key
place is step in the eventual formation of Grand Canyon.
(a) Longitude 130. A river formerly known as the 'Sorrow of
(b) Latitude Bengal' rises from Bihar is
(c) International Date Line (a) Barakar (b) Damodar
(d) Prime Meridian (c) Mayurakshi (d) Subarnarekha
43th BPSC (Pre) Exam. 1999 81 YCT
Ans. (b) : Damodar River rises in the Palamau hills of 136. The number of Indian states touching the
Chota Nagpur Plateau. Damodar river was earlier boundary of Bihar is
known as 'Sorrow of Bengal' as it used to flood to many (a) four (b) five
areas of Bardhaman, Hooghly, Howrah and Medinipur (c) six (d) seven
districts of W. Bengal.
Ans. (*) : Bihar is bordered by the Indian State of Uttar
131. According to 1991 census, population-wise, Pradesh in West, Jharkhand in South and West Bengal
which is the second largest state in India? in East and in North it is surrounded by Nepal.
(a) West Bengal (b) Maharashtra 137. The growth rate of population of Bihar during
(c) Bihar (d) Uttar Pradesh 1981-91 decade is
Ans. (c) : As per census 1991, in terms of population (a) 19.76% (b) 21.33%
Bihar was the second most populous states of India after (c) 23.54% (d) 23.06%
Uttar Pradesh. Ans. (*) : As per census 2011, the decadal growth rate
As per census 2011, Bihar is third most populous state of Bihar during 2001-2011 was 25.10%. During 1981-
in India. 91, the growth rate of population of Bihar was 23.54%
132. The three thermal power stations in Bihar run 138. The descending order of states in coal
on coal are production is
(a) Chandrapur, Panchet, Barauni (a) Bihar, Madhya Pradesh, West Bengal
(b) Bokaro, Panchet, Chandrapur (b) Madhya Pradesh, West Bengal, Bihar
(c) Barauni, Chandrapur, Santaldih (c) West Bengal, Madhya Pradesh, Bihar
(d) Bokaro, Chandrapur, Santaldih (d) Bihar, West Bengal, Madhya Pradesh
Ans. (*) : Termal Power Station has been built in Ans. (*) : In financial year 2021-22 Jharkhand is the
Bokaro (Jharkhand), Chandrapur (Maharashtra), largest producer of coal in India followed by
Santaldih (W. Bengal), These thermal power plants are Chhattisgarh and Odisha.
involved in power supply to Bihar. 139. The National Highway that passes through
133. The caste- based terrorist sena politics is in the Bihar is
following state (a) NH No.3 (b) NH No.8
(a) Bihar (b) Uttar Pradesh (c) NH No.22 (d) NH No.23
(c) Andhra Pradesh (d) Maharashtra Ans. (c) : National Highway No. 22 runs from Sonbarsa
Ans. (a) : The caste based terrorist sena politics in Bihar to Chandwa in Jharkhand.
prevailed in the state of Bihar. For example– Ranvir 140. The Tata Iron and Steel Plant receives iron ore
Sena, M.C.C. Lal Sena etc. from
134. The steel plant centre in Bihar is in (a) Bailadila
(a) Dhanabad (b) Ranchi (b) Keonjhar
(c) Mithila (d) Patna (c) Mayurbhanj

Ans. (a) : When the question was asked, the steel plant (d) Singhbhum
centre was at Dhanbad and Bokaro in Bihar. At present. Ans. (d) : Tata Iron and Steel Plant receives iron ore
Dhanbad and Bokaro is the part of Jharkhand. from Singhbhum.
135. Where is the lunatic asylum situated in Bihar? 141. Choose the correct statement
(a) Dhanbad (b) Nalanda (a) Aryabhatiya is a mathematical text
(c) Patna (d) Ranchi (b) The text named Lilavati is by Brahmagupta
Ans. (*) : Mental Hospital is located at Koelwar, (c) Pancasiddhantika is a seventh century text
Bhojpur in Bihar. (d) In Ganit-Sar-Samgraha zero is not considered

43th BPSC (Pre) Exam. 1999 82 YCT


Ans. (a) : Aryabhatiya is a mathmetical text. It is the πr 2 ×120o
only known surviving work of Indian mathematician Ans. (a) : Area of sector = 360
Aryabhata.
22 × 7 × 7 ×120o
Lilavati is written Bhaskara II, also known as =
Bhaskaracharya. 7 × 360o
7
Pancasiddhantika was written by Varahamihira in the = 22 × = 51.33 cm2
th
6 century. 3
147. A man bears 20% loss on selling a cycle for
142. Which of the following statements is true? `450. Choose the selling price of the cycle at
(a) π is an irrational number 20% profit.
(b) e is a rational number (a) ` 652.50 (b) ` 765
(c) 2 is an even number (c) ` 562.50 (d) ` 675
Ans. (d) : If the selling price of cycle is 450 then loss =
(d) ∞ is a real number 20%
Ans. (a) : Irrational numbers have endless non ATQ, cost of 80% of cycle = 450
repeating digits after the decimal point So, π is an 450 × 1
Selling price of cycle at 20% profit =
irrational number, the value of π is 3.1415926535897 80
........ 450 × 120 3
cost of 120% of cycle = = 450 ×
143. If X = {a,b} ; Y = {2,3} and Z = {3,4} then 80 2
= ` 675
choose X × {Y ∩ Z} 148. Choose the value of a, so that 2x+ay=1 and 3x–
(a) {( a,3 ) , ( b,3)}
5y=7 have no solution
3 10
(a) (b) −
(b) {( 3,a ) , ( 3, b )} 10 3
10 3
(c) {( a, 2 ) , ( a, 4 ) , ( b, 2 ) , ( b, 4 )} (c) (d) −
3 10
(d) {a,b, 2,3,4} Ans. (b) : Inconsistent pair of linear equations
a b c
Ans. (a) : Y = {2, 3}, Z = {3, 4} If 1 = 1 ≠ 1 , then there will be no solution.
a 2 b2 c2
∴ (Y ∩Z) = {3}
∴ X * (Y∩ Z) = {a, b} ×{3} = {(a, 3), (b, 3)} 2 a 1
As in = ≠ , the value of (a) is nothing
3 −5 7
144. If f: {x, y, z} → {a,b} is defined by f
−10
Hence a =
= {( x,a ) , ( y, b )} then f is 3
(a) is a unitary one function 149. Choose the value of
(b) not a function 0.05 × 0.05 × 0.05 − 0.04 × 0.04 × 0.04
(c) is a interjective function 0.05 × 0.05 + 0.05 × 0.04 + 0.04 × 0.04
(a) 1.01 (b) 0.1
(d) onto function (c) 0 (d) 0.01
Ans. (c) : Since there is no image of Z in f, Hence f is Ans. (d) : Let 0.05 = x and 0.04 = y
the interjective function. Then
145. Choose the compound interest ` 4000 at 5% x 3 − y3 ( x − y ) ( x 2 + xy + y 2 )
per year for 2 years computed annually. =
x 2 + xy + y 2 x 2 + xy + y 2
(a) ` 414 (b) ` 441 = (x – y) = 0.05 – 0.04 = 0.01
(c) ` 410 (d) ` 401 150. If the radius of a circle is doubled then choose
n the percent by which its area is increased.
 r 
Ans. (c) : A = P 1 +  (a) 400 (b) 100
 100  (c) 300 (d) 200
2
 5  21 21 Ans. (c) : If radius is r
= 4000 1 +  = 4000 × × = 4410 the Area (A) = πr2
 100  20 20
If radius is doubled Then, Area (A1) = π(2r)2 = 4πr2
So, compound interest = A – P
= 4410 – 4000 = ` 410 A1 − A
Percentage increase in area = × 100
146. A sector of a circle of radius 7 cm makes an A
angle of 120°. Find the area of sector? 4πr 2 − πr 2
= × 100
(a) 51.33cm2 (b) 53.13cm2 πr 2
(c) 53.31cm2 (d) 53.03cm2 = 3×100 = 300%

43th BPSC (Pre) Exam. 1999 83 YCT


44th Bihar Public Service Commission
Preliminary Examination, 2000-01
GENERAL KNOWLEDGE & GENERAL SCIENCE
(Solved Paper with Detail Explanation)
1. Which of the following ruler of Magadha was 5. Which one of the following is not common
the contemporary of Alexander the great? between Buddhism and Jainism?
(a) Mahapadmananda (a) Ahimsa
(b) Dhanananda (b) Indifference to Vedas
(c) Sukalpa (c) transmigration of soul
(d) Chandragupta Maurya (d) Rejection of Rituals
Ans. (b) : Dhanananda was the last king of Nanda Ans. (c) : Both Buddhism and Jainism had some
similarities in their teaching for instance, both rejected
dynasty. He was the contemporary of Alexander the
the authority of Vedas, emphasised on renunciation and
great. He had a huge army, it is believed that Alexander human efforts as the means of obtaining salvation and
the great attacked Punjab during his reign but his huge established a monastic order for both men and women.
army checked them from advancing towards the They both believed in faith and karma. Both rejected the
Magadha. Chandragupta Maurya under the able notion of grand rituals along with authority of the Vedas
guidance of Kautilya had uprooted the Nanda ruler and and the priestly class. Both Buddism and Jainism also
set up the Maurya Empire. stressed the principle of non violence.
2. Which of the following dynasties succeeded Jainism was different from Buddhism on several points.
after Mauryas? such as Jainism believed in transmigration of soul
(a) Sakas (b) Kushans while Buddihism does not, Jainsim recognized the
(c) Satvahans (d) Shungas existence of God while Buddhism did not.
6. Which of the following kings is said to have
Ans. (d) : The Maurya empire declined rapidly after
founded the city of Pataliputra?
Ashoka and later kings had very short reign. The
(a) Shisunaga (b) Bimbisara
Mauryan dynasty came to an end when the last king
(c) Ajatashtru (d) Udayin
Brihadratha was killed by military commander of
Pushyamitra Shunga. Pushyamitra Shunga (ruled 185- Ans. (d) : Udayin, the Son of Ajatshatru, had founded
149 BCE) was founder of Shunga dynasty. He was the the new capital at Pataliputra (modern Patna) situated at
first ruler of Shunga dynasty. The Maurya rule thus the confluence of Ganga and Son river.
came to an end and the Sunga Dynasty was established 7. The first Buddhist Council was held in which of
as the rulers of Magadha. the following cities?
(a) Nalanda (b) Gaya
3. 'Surya Temple' is situated in which of the
(c) Rajgir (d) Bodhgaya
following cities?
(a) Gaya (b) Bodhgaya Ans. (c) : After Buddha's demise, the first Buddist
Council was held in 483 BC at Saptaparni cave in
(c) Deo (d) Ranchi Rajagriha. It was presided over by the Mahakasyapa.
Ans. (c) : Sun Temple is situated in Deo city. It was During this council the teachings of Buddha were
built during the 8th century by Bhairavendra Singh Sun devided into three Pitakas named as Suttas, Vinaya and
Temple at Deo in Aurangabad district in Bihar is Abhidhamma pitakas.
famous for its antiquity. 8. Nizamuddin Auliya refused to meet, which one
4. In which of the following cities did Japanese of the following sultans?
build a 'World Peace Stupa'? (a) Jalaluddin Khilji
(a) Deo (b) Rajgir (b) Alauddin Khilji
(c) Gaya (d) Bodhgaya (c) Ghiyasuddin Tughlaq
(d) Mohammad - bin - Tughlaq
Ans. (b) : Vishwa Shanti Stupa (World Peace Stupa) is
Ans. (c) : Sheikh Nizamuddin Auliya was Sufi saint of
a large white stupa in Rajgir, Nalanda district of Bihar. Chisti order. He refused to meet Ghiyas-ud-din
It was built by a Japanese Budhist Monk close to where Tughlaq. Sheikh Nazamuddin Auliya and Ghiyasuddin
Lord Buddha delivered the perfection of wisdom sutra. Tughlaq did not have good relationship with each other.
This is incredible peaceful place and an important Ghiyas-ud-din had a dispute with Nizamuddin Auliya
pilgrimage site for faithful Buddhist. and his disciple, who he later overthrew.

44th BPSC (Pre) Exam. 2000-01 84 YCT


9. Which of the following rulers assumed to the 14. Which one of the following was the immediate
title of 'Hazrat-I-Ala' and afterwards 'Sultan'? cause of the First Carnatic War?
(a) Bahlol Lodi (b) Sikandar Lodi (a) Anglo - French Rivalry
(c) Sher Shah Suri (d) Islam Shah Suri (b) Austrain War of Succession
Ans. (c) : Sher Shah Suri was the founder of Suri (c) Issues of Carnatac Succession
dynasty. He defeated Nusrat Shah, Sultan of Bengal, (d) Capture of French ships by the British
after he attacked south Bihar become the vertual ruler of
Ans. (b) : The First Carnatic War was fought between
Bihar and assumed the title of Hazarat-i-Ala. He the English and French Forces during 1746-48. It was
adopted the title of Sultan-i-Adil after defeating an extension of the Anglo-French war in Europe which
Humayun in the Battle of Chausa in 1539. was caused by the Austrian war of Succession. The First
10. Which of the following sultans founded a town Carnatic War ended in 1748 when the Treaty of Aix-
where now stands Agra? Lachapelle was signed bringing the Austrian war of
(a) Mohammad -bin - Tughlaq Succession as a conclusion.
(b) Feroz Shah Tughlaq 15. Which of the following Acts empowered Lord
(c) Bahlol Lodi Cornawallis to over - rule the decision of his
(d) Sikandar Lodi Council?
Ans. (d) : Agra was founded by Sikandar Lodi in 1504, (a) Regulating Act (b) Act of 1786
as a base from where to control Malwa & Rajasthan. In (c) Charter Act of 1793 (d) Charter Act of 1813
1506 A.D Sikandar Lodhi shifted is capital from Delhi Ans. (b) : In 1786 Lord Carnwallis was appointed as
to Agra. the Governor General of Bengal and the Act 1786
11. Which one of the following Sultans of Delhi enabled the Governor-General to have the union power
introduced measures for improving the quality of Governor General and Commander-in-chief. Now
of fruits? Lord Carnwallis had the power also to override his
(a) Mohammad - bin - Tughlaq council in cases of necessity.
(b) Feroz Shah Tughlaq 16. Which of the following Indian leaders attended
(c) Bahlol Lodi the First Round Table Conference in London?
(d) Sikandar Lodi (a) Maulana Mohammad Ali
Ans. (b) : Firoz Shah Tughlaq had planted around 1200 (b) Maulana Abul Kalam Azad
orchards in Delhi and its adjoining areas to improve the (c) Mahtma Gandhi
quality of fruits and increase the production of fruit. (d) Pt. Jawaharlal Nehru
12. Which one of the following Indians was Ans. (a) : First Round Table Conference was held in
appointed as Deputy Diwan of Bihar by Robert London between November 1930 and January 1931. It
Clive? was chaired by Ramsay MacDonald. It was attended by
(a) Omi Chand (b) Manik Chand B.R. Ambedkar, Muhammad Ali Jinnah, Aga Khan III
(c) Rai Durlabh (d) Shitab Rai B.S. Moonje, Tej Bahadur Sapru, N.M. Joshi etc. The
Ans. (d) : Raja Shitab Rai was appointed as Deputy congress and some prominent business leaders refused
Diwan of Bihar by Robert Clive, Governor of Bengal to attend this conference.
presidency. Mohammad Raza Khan was appointed as 17. Which of the following leaders presided over
Deputy Diwan of Bengal, after the treaty of Allahabad. the Congress Session at Calcutta in 1906?
13. Which of the British officials defeated (a) B G Tilak (b) G K Gokhale
Portuguese at Sowalley? (c) Aurobindo Ghosh (d) Dadabhai Naoroji
(a) William Hawkins (b) Thomas Best Ans. (d) : The Calcutta Session of Indian National
(c) Thomas Roe (d) Josiah Child Congress in 1906 was presided over by the Dadabhai
Ans. (b) : Battle of Sowalley or Suvali was fought on Naoroji. In this session INC adopted the swaraj as the
29th November 1612 between the East India Company goal of Indian people.
under the leadership of Thomas Best and Portuguese. 18. Which one of the following periods is also
This battle marks the end of pourtugal's commercial known as Chalcolithic Age?
monopoly over Surat city in Gujarat, India. East India (a) Old Stone Age (b) New Stone Age
Company established over the Indian coast. (c) Copper Age (d) Iron Age

44th BPSC (Pre) Exam. 2000-01 85 YCT


Ans. (c) : Chalcolithic Age is called the first metal age Ans. (a) : In March 1931, Mahatma Gandhi and Lord
of India. This periods is also called Stone-Copper Age. Irwin (Viceroy of India 1926-31) entered into a Pact,
The Chalcolithic Age is a denotation of the period of called Gandhi-Irwin pact, by which the Congress called
transition from the use of native copper to the deliberate off the Civil Disobedience Movement and agreed to
use of bronze, a tin copper alloy. The first place in the participate in the Round Table Conference.
world that seems to have developed into the Copper 24. Jayprakash Narayan was associated with
Age in Middle East particularly in the eastern regions of (a) Champaran Movement
the Mediterranean sea. (b) Non- Cooperation Movement
19. Bihar became a separate State during the (c) Individual Civil Disobedience
British rule in the year (d) Quit India Movement
(a) 1905 (b) 1912
Ans. (d) : Jay Prakash Narayan was associated with
(c) 1936 (d) 1946 Quit India Movement, 1942. He made a successful
Ans. (b) : On 22 March 1912 Bihar and Orissa carved escape from the Hazaribagh Central Jail in 1942. He
out of Bengal presidency as separate province through along with Ram Manohar Lohia, Aruna Asaf
the Bengal, Bihar, Orissa and Assam Laws Act, 1912. Ali,Sucheta Kriplani, Biju Patnaik, R.P. Goenka and
On 1 April 1936 Bihar and Orissa became separate Achyut Patwardhan was associated with under ground
provinces under the Government of India Act, 1935. In movement and revolutionary activities in support of
2000, Jharkhand was carved out of Bihar. Quit India Movement.
20. Western Ghats in Maharashtra and Karnataka 25. Birsa Munda was in favour of
is known for (a) Jharkhand (b) Uttranchal
(a) Nilgiri Mountian (b) Sahyadri (c) Chhattisgarh (d) None of these
(c) Deccan Plateau (d) None of these
Ans. (d) : Munda tribe is found in the Chotanagpur
Ans. (b) : Western Ghat in Maharashtra and Karnataka Plateau area of Jharkhand. Birsa Munda is known to
is known as Sahyadri. It is also known as Nilgiri hills in have mobilised the tribal community against the British
Karnataka and Tamil Nadu, Analmalai hills and and had also forced the colonial officials to introduce
Cardamom hills in Kerala. Anamudi is the highest peak
laws protecting the land rights of the tribals.
of western Ghats, It covers Gujarat, Maharashtra, Goa,
Karnataka, Tamil Nadu and Kerala. 26. Who started the Individual Civil Disobedience?
(a) Vinoba Bhave (b) Jawaharlal Nehru
21. Which of the following Mughal emperors was
not allowed to enter into Delhi by Wazir (c) Sardar Patel (d) Saukat Ali
Ghaziuddin? Ans. (a) : Individual Civil Disobedience Movement
(a) Alamgir II (b) Shah Alam II arose directly as a result of the August offer in 1940.
(c) Akbar II (d) Bahadur Shah II This was a Movement that sought not only
Ans. (b) : Shah Alam II, also known as Ali Gauhar was independence but also the right to free expression.
the 17th Mughal Emperor and the son of Alamgir II. Vinoba Bhave was the first person and Jawaharlal
During his reign, the Mughal Empire was stretched Nehru as the second person chosen to offer an
from Delhi to Palam individual satyagraha by Gandhiji.
22. With whom is the statement "Swaraj is my Brahma Dutt was the third person chosen to offer an
birthright and I shall have it" associated? individual satyagraha.
(a) Birsa Munda 27. Dandi March was undertaken in
(b) Anugraha Narayan Sinha (a) 1932 (b) 1931
(c) Sacchidananda Sinha (c) 1929 (d) 1930
(d) None of these Ans. (d) : The Dandi March, also known as the Salt
Ans. (d) : "Swaraj is my birth right, and I shall have it", March and the Dandi Satyagraha, was an act of
this slogan was coined by Lokmanya Bal Gangadhar nonviolent civil disobedience led by Gandhiji. On 12th
Tilak. He announced this famous slogan in Bombay March 1930 Gandhiji set out from Sabarmati with 78
State Conference held at Belgaum in 1916. followers on a 241-mile march to the coastal town of
23. Gandhi - Irwin Pact was signed in Dandi near the Arabian Sea, There, Gandhi and his
(a) 1931 (b) 1935 supporters were to defy British policy by making salt
(c) 1942 (d) 1919 from seawater.

44th BPSC (Pre) Exam. 2000-01 86 YCT


28. Indian National Congress was founded by 33. Non - Cooperation Movement was launched in
(a) A O Hume (b) Mahatma Gandhi (a) 1920 (b) 1930
(c) Annie Besant (d) Lala Lajpat Rai (c) 1917 (d) 1921
Ans. (a) : Indian National Congress was founded in Ans. (a) : Non Cooperation Movement was led by
1885 by A.O. Hume, a retried English civil servant. He Mahatma Gandhi after the Jallianwala Bagh Massacre.
organized the first session of INC in Gokuldas Tejpal It involved using swadeshi and boycott of foreign
Sanskrit College, Bombay in December 1885. It was goods, especially machine made cloth. It was launched
attend by 72 delegates and presided over by Womesh on 1th August 1920. This movement was called off
Chandra Banerjee. because of Chauri Chaura incident in 1922.

29. Bengal was partitioned in 34. Rajendra Prasad belonged to


(a) U P (b) Bihar
(a) 1905 (b) 1909
(c) Madhya Pradesh (d) None of these
(c) 1906 (d) 1919
Ans. (b) : Dr. Rajendra Prasad was the first President of
Ans. (a) : Lord Curzon was the viceroy of India from
India from 1952 to 1962. He was born in Ziradei
1899 to 1905. The partition of Bengal province came
village, of Siwan in Bihar on 3 December, 1984. He
into effect during his viceroyalty on 16 October 1905. joined INC in 1911. He was imprisoned by British
This day was observed as day of mourning through out authorities during salt Satyagraha of 1931 and Quit
Bengal, people tied Rakhis on each other's hand as a India Movement of 1942.
symbol of unity of the two halves of Bengal.
35. Mahatma Gandhi returned India in December,
30. Where was the first session of Indian National 1931 empty Handed from
Congress held? (a) London (b) Moscow
(a) Delhi (b) Lahore (c) Washington (d) Tokyo
(c) Mumbai (d) Patna Ans. (a) : Second round table conference was held in
th st
Ans. (c) : The first session of Indian National Congress London from 7 September to 1 December 1931.
was held at Gokuldas Tejpal Sanskrit College in Gandhiji was choosen as the sole representative of the
Bombay in December 1885. It was attended by 72 Indian National Congress for this conference. The
conference was declared unsuccessful due to some
delegates and presided over by Womesh Chandra
dissensions and Gandhiji returned India in December
Bannerjee.
1931 empty handed.
31. Who viewed "the Cripps" proposals as a post -
36. Among the following who not an extremist?
dated cheque upon a crashing bank"?
(a) Bal Gangadhar Tilak (b) Madan Lal
(a) Mahatma Gandhi (b) Maulana Azad (c) Udham Singh (d) G K Gokhale
(c) J B Kripalani (d) Jayprakash Narayan Ans. (d) : Gopal Krishna Gokhale was not an extremist
Ans. (a) : In March 1942 a mission headed by Stafford leader. He was one of the leaders of the moderate of
Cripps was sent to India with constitutional proposal to Indian National Congress. He had established the
seek Indian support for the World War II. The Cripps Servants of India Society in 1905.
mission proposal failed to satisfy Indian Nationalist 37. Who formed the Forward Block?
Ganghiji described the Cripps proposal as a post dated (a) Shubhash Chandra Bose
cheque upon a crashing bank. (b) Rajendra Prasad
(c) Bhagat Singh
32. Quit India Movement was launched in
(d) B R Ambedkar
(a) 1940 (b) 1941
Ans. (a) : Netaji Subhas Chandra Bose had resigned
(c) 1942 (d) 1945 from Indian National Congress Presidentship in 1939
Ans. (c) : The Quit India Resolution was ratified at the and organized the All India Forward Bloc, a faction
Congress meeting at Gowalia Tank Maidan Bombay on within the INC in Bengal.
8 August, 1942. The Quit India resolution was proposed 38. The Korba Coalfield is located in
by the Jawahar Lal Nehru and seconded by Sardar Patel. (a) Orissa (b) Chhattisgarh
Gandhiji was named the leader of the struggle. (c) West Bengal (d) Assam
Incidentally, the slogan, Bharat Chhodo (Quit India) Ans. (b) : Korba Coalfield is located in Kobra district
was coined by Yusuf Mehar Ali and it found the of Chhattisgarh. This coalfield is located in the basin of
approval of Gandhiji. Hasdeo River, a tributary of Mahanadi.

44th BPSC (Pre) Exam. 2000-01 87 YCT


39. Gaddis are inhabitants of 45. The highest mountain peak in India is
(a) Madhya Pradesh (b) Himachal Pradesh (a) Mount Everest (b) Kanchenjunga
(c) Arunachal (d) Meghalaya (c) Godwin Austen (d) Nanga Parbat
Ans. (b) : Gaddis tribes reside in Mandi, Angra and Ans. (c) : Godwin-Austen also known as Mount K2 is
Bilaspur district of Himachal Pradesh. Guryar, Swangla, the highest peak of India. It is located in Jammu and
Lahaula, Khampa etc are others tribes found in Kashmir (LAC). It is 2nd highest peak of the world.
Himachal Pradesh. Mount Everest is the highest peak in the world.
40. Manas river is a tributray of 46. Pygmies are inhabitants of
(a) Godavari (b) Mahanadi (a) Africa (b) Asia
(c) Krishna (d) Brahmaputra
(c) Australia (d) South America
Ans. (d) : Manas river is a trans-boundry river in
Ans. (a) : Pygmies are the extremely short statured
Himalayan foothills between southem Bhutan and India.
people found through out the central Africa with smaller
It is the right bank tributary of the Brahmaputra river. It
numbers in south-east Asia.
joins the river Brahmaputra near Jogighopa in Assam.
Lohit, Dhansri, Torsa, Sankosh, Desang, Testa, Some of the major tribes of Africa are:-Zulu, Bushman,
Dikhow, Kopili, Subansiri are principle tributaries of Maasi, Himba etc.
Brahmaputra. 47. The most important food crop in India in terms
41. Eskimos are inhabitants of of area under cultivation is
(a) Canada (b) Mangolia (a) wheat (b) maize
(c) Malava (d) Sri Lanka (c) barley (d) rice
Ans. (a) : Eskimo or Inuit is a commonly used term Ans. (d) : Rice is the most cultivated crop in India. It is
referring to the native peoples of Alaska and other Sub- grown about 34% of total cropped area of country. Rice
Arctic regions including Canada, Siberia and Green is predominantly a Kharif crop of India. Wheat is
land. second most important crop of India.
42. Gulf Stream is 48. Of the following, the State which receive
(a) A river in the gulf rainfall in winter season is
(b) An ocean current (a) Kerala (b) Tamil Nadu
(c) Another name of jet stream (c) West Bengal (d) Odisha
(d) A surface wind Ans. (b) : Tamil Nadu receives a major portion of its
Ans. (b) : Gulf Stream is a warm and swift Atlantic rainfall during October and November. Tamil Nadu
Ocean current. It originates in the Gulf of Mexico and receives most of its rain from north east and retreating
stretches to the tip of Florida and follows the eastern monsoons during Winter season, moreover many low
coastlines of the United States and Newfoundland pressure system develops in the Bay of Bengal during
before crossing the Atlantic Ocean. It is responsible for this period and moves towards the east coast of Tamil
mild climate at the Eastern cost of North America as Nadu causing heavy rainfall.
well as Europe. It brings warm water from the Gulf of 49. Fiji Island is located in
Mexico into the Atlantic Ocean.
(a) Atlantic ocean (b) Pacific ocean
43. Which is known as 'Roof of the World'? (c) Indian ocean (d) Arabian sea
(a) Aravali (b) Satpura
Ans. (b) : Fiji is an island country in the South Pacific
(c) Pamir (d) Myanmar
Ocean. It's capital is Suva. Suva has been the capital
Ans. (c) : Pamir plateau is an intermontane plateau. It is since 1882.
a vast elevated plateau in Central Asia and East Asia. It
50. Paradip is located in the State of
is called the "Roof of the World" because it is at high
altitude. (a) Kerala (b) Maharashtra
(c) Odisha (d) Andhra Pradesh
44. 'Regur' is the name of
(a) red soil (b) alluvial soil Ans. (c) : Paradip Port is located in Odisha. It is
(c) black soil (d) laterite soil situated at the confluence of Mahanadi river and the
Ans. (c) : Black soil is also known as "Regur Soil" Bay of Bengal. Late Biju Patnaik, the then Chief
These soils are best suited for cotton crop, hence these Minister of Odisha, is the founder father of Paradip
soils are also called black cotton soil. port.

44th BPSC (Pre) Exam. 2000-01 88 YCT


51. Hirakund dam is built on the river 57. Malacca Strait facilitates movement from
(a) Narmada (b) Mahanadi (a) Indian ocean to South China sea
(c) Godavari (d) Chambal (b) Red sea to Mediterranean sea
Ans. (b) : Hirakund dam is built on the river of (c) Atalantic ocean to Pacific ocean
Mahanadi, Odisha. Its height is about 60.9 m and length (d) Mediterranean sea to Black sea
is about 4.8 km. The dam was inaugurated on 12 Ans. (a) : The Strait of Malacca, which flows between
January 1957 by then Prime Minister Pandit Nehru. Indonesia, Malaysia and Singapore. It connects the
52. The largest percentage of forest area is in Indian Ocean with Pacific Ocean through the South
(a) Arunachal Pradesh (b) Himachal Pradesh China Sea.
(c) Mizoram (d) Nagaland Suez Canal connects Mediterranean Sea with Red Sea.
Ans. (c) : As per forest Survey Report 2021, In terms of The Strait of Magellan links the Pacific Ocean &
forest cover as percentage of total geographical area, the Atlantic Ocean.
top five States are:- 58. Kilimanjaro is a
Mizoram (84.53%), Arunachal Pradesh (79.33%), (a) volcano (b) island
Meghalaya (76.00%), Manipur (74.34%) and Nagaland (c) peak (d) river
(73.90%). Ans. (c) : Mount Kilimanjaro is the highest peak in
53. The Pat region is located in Africa (about 5895m). It is a stratovolcano or composite
(a) Bihar (b) Jharkhand volcano and is made up of three cones- Kibo, Mawenzi
(c) Madhya Pradesh (d) Meghalaya and Shira. It is located in Tanzania.
Ans. (b) : Mesas are hills with flat and regular top 59. International Date Line passes through
surface. When they are reduced in size due to (a) Africa (b) Asia
continuous weathering and errosion, they are called (c) Pacific ocean (d) Atlantic ocean
buttes. Mesas are locally called as "pats" or patland on
Chotanagpur plateau of Jharkhand. Ans. (c) : The International Date Line passes through
the mid Pacific Ocean and roughly follows a 180 degree
54. The largest producer of cotton in India is
(a) Maharashtra (b) Gujarat longitude north-south line on the Earth. It was
established in 1884. It is located halfway around the
(c) Punjab (d) Haryana
world from the Prime Meridian.
Ans. (b) : As of recent data, Gujarat is the leading
cotton growing state followed by Maharashtra, 60. The Shivanasamudra falls is located on the
Telangana and Karnataka. India is the largest producer river
of cotton in the world. (a) Cauvery (b) Krishna
55. The river which flows through a riftvalley is (c) Godavari (d) Mahanadi
(a) Narmada (b) Sone Ans. (a) : The Shivanasamudra falls is located on
(c) Godavari (d) Cauvery Cauvery (Kavery) River. It has an average width of
Ans. (a) : Narmada river flows westwards through a rift 305m, a height of 98m & an average volume of 934
valley between the Vindhyan Range on the north and cubic meters per second.
Satpura Range on the south. Kunchikal falls in Shimoga district of Karnataka, is the
Narmada rises from Maikal range near Amarkantak in highest waterfall in India.
Madhya Pradesh.
61. The largest city located on the bank of river
Tapi, Mahi and Damodar are some rivers in India, that
Ganga is
flow through Rift valley.
(a) Varanasi (b) Patna
56. Sone, Narmada and Mahanadi originate from
(a) Palamau hills (b) Amarkantak (c) Kanpur (d) Allahabad
(c) Eastern Ghat (d) Aravali Ans. (c) : Kanpur is the largest city located on the bank
Ans. (b) : Sone river originates from Amarkantak of river Ganga. Kanpur is also known by the name of
highlands in hills of Maikal range in Chhattisgarh. It is "Manchester of the East" It is famous for leather &
important right bank tributary of river Ganga. Narmada textile goods of fine quality. Prayagraj is situated at the
river originates from Maikal range near Amarkantak. It confluence of the Ganga, Yamuna and Saraswati.
is largest west flowing river of peninsular region. 62. The percentage of urban population to total
Mahanadi river originates from a place near Nagri town population in India is
in Raipur district the State of Chhattisgarh to the south (a) 25.72 (b) 31.86
of Amarkantak. (c) 20.28 (d) 22.52
44th BPSC (Pre) Exam. 2000-01 89 YCT
Ans. (a) : As per census 1991, Urban Population as 67. Under which Article of the Constitution of
percentage of total population was 25.72%. However as India, Fundamental Rights are provided to
per census 2011, Urban population as percentage of citizens?
total population is 31.2%. (a) Article 112 to 135 (b) Article 12 to 35
63. Regur soil is most widespread in (c) Article 222 to 235 (d) None of these
(a) Maharashtra (b) Tamil Nadu Ans. (b) : The Fundamental Rights are enshrined in Part
(c) Andhra Pradesh (d) Jharkhand III of the Indian Constitution from Article 12-35. Indian
Ans. (a) : Regur soils are also known as Black soil and Constitution provides 6 fundamental rights, viz.
black cotton soil. It is mostly found in Maharashtra. It (i) Right to equality (Article 14-18)
spreads across the plateau of Saurashtra and Malwa. (ii) Right to freedom (Article 19-22)
The other states having black soil are Chhattisgarh, (iii) Right against exploitation (Article 23-24)
Gujarat, Madhya Pradesh, Andhra Pradesh. (iv) Right of freedom of religion (Article 25-28)
64. The State with highest density of population is (v) Cultural and education rights (Article 29-30)
(a) Kerala (b) Uttar Pradesh (vi) Right to constitutional remedies (Article 32-35)
(c) West Bengal (d) Maharashtra 68. The 42nd Amendment Act (1976) has
Ans. (c) : According to census 2011, the population incorporated into the Constitution of India a
density of India was 382 persons per square kilometer. new chapter on
Bihar is most densely populated state (1106 (a) Administration of Union Territories
persons/sq.km.) followed by West Bengal (1028 (b) Formation of Inter - State Councils
person/Sq.km.) and Kerala (860 persons/sq.km.) The (c) Fundamental Duties
population density of Uttar Pradesh is 829 (d) None of the above
persons/sq.km. and Maharashtra (365 persons/sq.km.) Ans. (c) : Fundamental Duties were added by the 42nd
Hence among the above given options, option (c) will Constitutional Amendment Act of 1976 on the
be correct. recommendation of the Swaran Singh Committee.
65. Which is not among the powers of Panchayati Originally 10 Fundamental Duties were in number, one
Raj more duty was added through the 86th Constitution
(a) Implementation of Land Reform Amendment Act, 2002. All the eleven duties are listed
(b) Judicial Review in Article 51-A of the Constitution.
(c) Implementation of Poverty Alleviation 69. Who holds the power to increase the number of
Programme Judges in the Supreme Court?
(d) None of the above (a) Prime Minister (b) Resident
Ans. (b) : The 73rd Constitutional Amendment Act, (c) Parliament (d) Ministry of Law
1992, has added a new part IX to the Constitution of Ans. (c) : Article 124 to 147 in Part-V of the Indian
India. This part is entitled as the 'Panchayats' & consists Constitution deals with the organization, independence,
of provision in Article 243. In addition, the Act also jurisdiction, power, procedures of the Supreme Court.
added a new 11th schedule to the Constitution. This In Indian Constitution, Parliament is authorized to
Schedule contains 29 functional. Subjects of the increase number of the Judges in the Supreme Court of
Panchyats which includes Agriculture, implementation India. At present the Supreme Court consists of thirty
of land reforms, Animal husbandry, poverty alleviation 34 (including one Chief Justice and 33 other Judges).
programme, Libraries, Market & fairs etc are functional Supreme Court Judges retire after the attaining the age
items of Panchayati Raj institution. Whereas the right to of 65 year.
judicial review is possessed by both the Supreme Court 70. Who advise the Government of India on legal
& the High Courts of the country. matters?
66. Panchayati Raj in India represents (a) Attorney General
(a) decentralisation of powers (b) Chief Justice of the Supreme Court
(b) participation of the people (c) Chairman, Law Commission
(c) community development (d) None of the above
(d) All of these Ans. (a) : Article 76 of Indian Constitution has
Ans. (d) : Panchayati Raj in India signifies the system provided for the office of the Attorney General of India.
of rural local self government. It has been established in He is the highest law officer in the country. He gives
all states by the Act of the State legislatures to build advice to Government of India upon such legal matters,
democracy at the grass root level. It represents which are referred to him by President. The President
decentralization of powers, participation of people and has assigned the duties to Attorney General to appear on
community development. It was constitutionalised behalf of the Government of India in all cases in the
through the 73rd Constitutional Amendment Act of Supreme Court in which the Government of India in
1992. concerned.
44th BPSC (Pre) Exam. 2000-01 90 YCT
71. Who appoints the Chairman of Union Public Ans. (c) : The Service Sector, also known as Tertiary
Service Commission? Sector, is the largest sector of Indian Economy. This
(a) Parliament sector accounts for 53.89% of total India's Gross value
(b) President Added (GVA). Tertiary sector of Indian Economy
(c) Chief Justice of India contributes largest to Gross National Product.
(d) Selection Committee 76. Which is the highest bank in the banking
Ans. (b) : Article 316 : Appointment and term of office structure of India?
of members of UPSC as well as SPSC. (a) State Bank of India
The President of India appoints the Chairman and other (b) Reserve Bank India
members of the Union Public Service Commission (c) Central Bank of India
(UPSC). Any members of UPSC shall hold office for a (d) Industrial Development Bank of India
term of 6 years or till the age of 65 years, whichever is Ans. (b) : Reserve Bank of India is the Central Bank of
earlier. The Chairman or any other member of UPSC India. It is the highest bank in the banking structure of
shall only be removed from his/her office by the order India. RBI was established on 1st April 1935 under the
of the President of India. RBI Act of 1934.
72. Which is the tax that is imposed by the Central 77. During which Five Year Plan, the price level in
Government of India but its revenue is divided Indian economy showed decline?
between Centre and States? (a) Plan I (b) Plan II
(a) Custom Duty (b) Central Excise Duty (c) Plan IV (d) Annual Plans
(c) Corporation Tax (d) None of these
Ans. (a) : During First Five Year plan 1951-56 the price
Ans. (b) : Central Excise Duty is a form of indirect tax level in Indian Economy showed a decline because of
that is levied by the Central Government of India but its rapid agricultural development and measures to control
revenue is divided between the Central Government and inflation as there was disequilibrium due to Second
State Government. World war and partition of economy.
73. In India National Account Statistics are 78. In which year the Indian rupee was devaluated
collected by two times within a month?
(a) Indian Statistical Institute (a) 1990 (b) 1991
(b) Central Statistical Organisation (c) 1994 (d) 1999
(c) Planning Commission Ans. (b) : In 1991 Indian rupee was devaluated two
(d) Federation of Indian Chamber of Commerce times within a month. Devaluation is the process by
and Industry which Central Bank of an economy reduces the value of
Ans. (b) : The Central Statistical Organisation (CSO), a currency with respect to other currencies.
in the Ministry of Statistics and Programme The Indian Rupee was devaluated in three instances.
Implementation, is responsible for the compilation of The Indian Rupee was devalued for the first time in
National Accounts Statistics (NAS). 1949. Later it was devalued 1966 and finally the Indian
CSO is responsible for the coordination of statistical Rupee was devalue in 1991. In 1991, the Indian Rupee
activities in India and evolving and maintaining was devalued in two steps, first on 1st July and again on
statistical standards. 3rd July.
74. The birth rate in Indian Economy as per census 79. Which is not a source of Agricultural Finance
1991 was- in India?
(a) 30.0 per thousand (b) 27 per thousand (a) Co-operative Societies
(c) 19 per thousand (d) 10.0 per thousand (b) Commercial Banks
Ans. (a) : As per census 1991, crude birth rate in India (c) Regional Rural Banks
was 30.0 per thousand. According to census of India
(d) None of these
2011, India has a crude Birth Rate of 21.8 per
thousand. Ans. (d) : Agriculture is the key sector of Indian
75. Which sector of the Indian Economy Economy in view of its contribution to employment and
contributes largest to the Gross National GDP. Agricultural Credit plays a vital role in farm
Product? sector development and facilitates adoption of new
(a) Primary Sector (b) Secondary Sector technologies. The two main sources of finance in
(c) Tertiary Sector (d) Public Sector agriculture are institutional and non institutional

44th BPSC (Pre) Exam. 2000-01 91 YCT


sources. Institutional sources consists of the 84. The song 'Vande Mataram' was written by
Government and co-operative societies, commercial (a) Jayprakash Narayan (b) Rajendra Prasad
bank including Regional Rural Bank, Lead Bank. Non (c) Birsa Munda (d) None of these
Institutional Sources include money lenders, traders &
Ans. (d) : Vande Mataram National song of India was
commission agents, relative and landlords.
written by Bankim Chandra Chatterjee. It is composed
80. Which is not a source of Industrial Finance in in Sanskrit. It was first sung by Rabindra Nath Tagore
India?
in the 1896 session of the Indian National Congress.
(a) Industrial Finance Corporation of India
85. Who is the Executive Head in India?
(b) NABARD
(a) President
(c) State Financial Corporation
(d) Unit Trust of India (b) Prime Minister
(c) Leader of the Opposition
Ans. (b) : National Bank for Agriculture and Rural
Development (NABARD) is not a source of Industrial (d) Chief Secretary, Government of India
Finance. It is an apex financial institution in India that Ans. (a) : Under the Article 53 of Indian Constitution,
provides rural and agricultural financing. the head of the executive is the President. All executive
Industrial Finance Cooperation of India provides power is vested in him and all executive actions are
medium & long term loan and advances to industrial taken in his name. The executive power shall be
and manufacturing concern. exercised by the President either directly or through
State Financial Corporations are the state level financial officers subordinate to him in accordance with the
institutions which play a vital role in the growth of constitution.
small medium enterprises in the concerned states. 86. In India, the President is elected by
Unit Trust of India provides the investor with a safe (a) Direct Election
return of the investment whenever there in a
(b) Single Transferable Vote System
requirement of funds.
(c) Proportional Vote System
81. Which Constitution Amendment granted
(d) Open Ballot System
Constitutional Status to Panchyati Raj
Institutions? Ans. (b) : Article 54 of Indian constitution is related
(a) 73rd Amendment (b) 71st Amendment with election of President. The President's election is
(c) 74th Amendment (d) None of these held in accordance with the system of proportional
Ans. (a) : Panchyati Raj Institutions is a system of rural representation by means of single transferable vote and
local self government in India. It was constitutionalized the votting is by secret ballot. The President is elected
through the 73rd Constitutional Amendment Act, 1992. by members of electoral college consisting of:-
The Act added Part IX to the Constitution and 11th
(i) The elected members of both the House of
Schedule which consists of the 29 subjects of the
Panchayats. Parliament
82. Sacchidanand Sinha was associated with (ii) The elected members of the legislative assemblies of
the States and
(a) Quit India Movement
(b) Dandi March (iii) The elected members of the legislative assemblies
(c) Civil Disobedience Movement of the union territories of Delhi and Puducherry.
(d) None of the above 87. Which is the Upper Chamber of State
Ans. (d) : Dr. Sachchidanand Sinha was the first Legislatures in India?
interim President of the Constituent Assembly. He was (a) Legislative Council
born in Arrah District, Bihar. (b) Legislative Assembly
83. Who was associated with Bihar Peasant (c) Governor's Office
Movement. (d) None of these
(a) Rajendra Prasad (b) CR Das
(c) Motilal Nehru (d) Bhagat Singh Ans. (a) : In the states having bicameral system, the
state legislature consists of the Governor, the
Ans. (a) : Dr. Rajendra Prasad was associated with
Bihar Peasant Movement. While Mahatma Gandhi was Legislative Council and the Legislative Assembly. The
on fact finding mission in Champaran district of Bihar Legislative Council (Vidhan Parishad) is the upper
to address grievances of local peasants, he called Dr. house, while the Legislative Assembly (Vidhan Sabha)
Rajendra Prasad to come to Champaran with volunteers. is the lower house.

44th BPSC (Pre) Exam. 2000-01 92 YCT


88. In India, the system of note Issue is based on 93. The percentage of land area of old Bihar that
(a) Proportional Reserve System forms the State of Jharkhand is
(b) Minimum Reserve System (a) 38.40 (b) 45.85
(c) Fixed Exchange Rate System (c) 42.35 (d) 51.72
(d) Full Convertibility System
Ans. (b) : Jharkhand is created on 15 November 2000,
Ans. (b) : In India currency notes issue system is based from what was previously the southern half of Bihar.
on Minimum Reserve System. The Minimum Reserve
Jharkhand covers on area of 79,710 km2 which is
system which requires the RBI to keep a minimum
reserve of ` 200 crore comprising foreign currencies, 46.85% of land area of old Bihar. Before the formation
gold coin and gold bullion. There is no limit for the RBI of Jharkhand total area of Bihar was 173877 km2.
to issue currencies by keeping this minimum reserve. 94. The percentage of literacy in Bihar is
89. Who holds the power to appoint the nominees (a) 42.85 (b) 37.85
from Anglo- Indian Community in the Lok (c) 52.51 (d) 43.56
Sabha?
Ans. (d) : As per census 2001, the percentage of
(a) Minorities Commission
literacy in Bihar was 47%. However, As per census
(b) President of India
2011, the total literacy rate of Bihar is 61.80% in which
(c) Prime Minister
(d) Vice-President male literacy rate was 71.20% and female literacy was
51.50%.
Ans. (b) : Under Article 331 of Indian Constitution,
President of India nominates 2 members of the Lok 95. Of the following rivers, the maximum shifting
Sabha from the Anglo-Indian community. However the of course has taken place in
reservation for Anglo-Indian in the form of nomination (a) Sone (b) Gandak
is expired on 25 January 2020 by 104th Constitutional (c) Kosi (d) Ganga
Amendment Act.
Ans. (c) : Kosi river is also known as "Sorrow of Bihar"
90. The first meeting of the Congress Socialist
This river is known for changing its route. It flows
Party was held in
(a) Delhi (b) Nasik through Nepal, Bihar & causes immense flooding due to
(c) Patna (d) Lahore frequent shifting of its course that is attributed to the
youthful topography of the river.
Ans. (c) : Congress Socialist Party was established
within the INC on 17 May 1934 in Patna with Acharya 96. Most significant oil refinery of Bihar is
Narendra Deva as President and Jay Prakash Narayan as (a) Singhbhum (b) Rudrasagar
general secretary. The first All India Congress Socialist (c) Barauni (d) Ranchi
Conference was held at the Anjuman Islamia Hall,
Patna on 17 May 1934. Ans. (c) : Most significant oil refinery of Bihar is
Barauni. It is second refinery of the country. This oil
91. The Saran irrigation canal is drawn from the
river refinery was set up in collaboration with Soviet Union
(a) Sone (b) Ganga at a cost of ` 49.4 crore and went on stream in July
(c) Kosi (d) Gandak 1964. Barauni Oil Refinery is an integrated refinery
Ans. (d) : A 743 metre long dam has been built across owned by Indian oil.
the Gandak at Valmiki Nagar from where two canals 97. Palamau (Ranchi) is famous for which
have been taken off. The Western canal is called Tirhut mineral?
and the eastern canal is known as Saran Canal. Saran (a) Iron ore (b) Copper ore
Canal irrigates 7-6 hectares of saran district of Bihar (c) Manganese ore (d) Bauxite ore
and Deoria and Gorakhpur district of U.P.
Ans. (d) : Palamu (Ranchi) is famous for Bauxite ore.
92. Of the following the largest district in Bihar is
(a) Patna (b) Gaya Iron ore, Maganese ore, Nickel ore are found in
(c) Darbhanga (d) Samastipur Singhbhum.
Ans. (b) : District Area 98. The total population in the State Bihar is
Gaya 4976 km2 (a) 64.434 million (b) 86.374 million
Patna 3202 km2 (c) 72.325 million (d) 70.562 million
Samastipur 2904 km2 Ans. (*) : As per census 2001, total population of Bihar
Darbhanga 2279 km2 was 82,998,509 in which males were 43,243,795 while
In terms of Area, West Champarn (5228 sq km) is the females were 39,754,714. As per census 2011, total
largest district of Bihar and Sheohar (349 sq km) is the population of Bihar is 104, 099, 452 of which males
smallest district. were 54,278,157 and females were 49,821,295.

44th BPSC (Pre) Exam. 2000-01 93 YCT


99. The percentage of urban population to total 105. Which of the district mentioned here is not
population in Bihar. included in the Jharkhand State?
(a) 10.76 (b) 13.14 (a) Godda (b) Ranchi
(c) 12.86 (d) 21.56 (c) Bokaro (d) None of these
Ans. (a) : As per census 2011, out of total population of Ans. (d) : All the above districts are included in
Bihar, 11.29% people live in Urban region. The total Jharkhand. Ranchi is the capital of Jharkhand.
population living in Urban area is 11,758,016 of which 106. The descending order of the states producing
6,204,307 are males and while remaining 5553, 709 are sugarcane is
females. (a) Uttar Pradesh, Maharashtra, Andhra Pradesh,
100. The State of Jharkhand was formed in Punjab
(a) 1997 (b) 1998 (b) Maharashtra, Uttar Pradesh, Bihar, Andhra
(c) 1999 (d) 2000 Pradesh
(c) Uttar Pradesh , Maharashtra, Tamil Nadu,
Ans. (d) : Jharkhand was formed by the Bihar
Karnataka
Reorganisation Act on 15 November 2000 as the 28th
(d) Maharashtra, Uttar Pradesh, Bihar, Tamil
State of India.
Nadu
101. Which city in Bihar is famous for Wildlife
Ans. (c) : As sugarcane is an important Kharif Crop.
Sanctuary and National Parks? The climate of country supports the plantation of
(a) Hazaribagh (b) Nalanda sugarcane throughout the year. India is the second
(c) Gaya (d) Jamshedpur highest producer of sugarcane in world. U.P. is India's
Ans. (a) : Valmiki National Park is a Tiger Reserve in largest Sugarcane producing State. When the question
West Champaran district of Bihar. At present it is only was asked, the descending order of States producing
national park in Bihar. When the question was asked, sugarcane was Uttar Pradesh > Maharashtra > Tamil
there was only one Wildlife Sanctuary & National Park Nadu > Karnataka but currently the correct sequence is
located in Hazaribagh, Bihar which went to Jharkhand Uttar Pradesh > Maharashtra > Karnataka > Tamil
after the State partition. Nadu.
102. Total forest cover as percentage of area in 107. The planet which has no satellite is
Bihar is (a) Mars (b) Mercury
(a) 32% (b) 21% (c) Naptune (d) Uranus
(c) 19% (d) 31% Ans. (b) : Mercury and Venus are the only planets in
the Solar System which do not have any natural
Ans. (*) : As per India State of Forest Report 2019,
satellite. Mars has two satellite i.e. Phobos & Deimos.
Bihar has 7.76% of forest cover which includes land Neptune has 14 known satellites and Uranus has 27
having trees in more than one hectare area with tree known satellites.
canopy density of 10%. 108. Solar energy is obtained from which of the
103. What is the proportion of urban population in following?
Bihar according to 1991 Census? (a) Moon (b) Ocean
(a) 13.14% (b) 14.13% (c) Sun (d) Wind
(c) 10.03% (d) 23.17% Ans. (c) : Solar energy is obtained from Sun. Solar
energy is created by nuclear fusion that takes place in
Ans. (a) : As per census 1991 Urban population as
the Sun. Fusion occurs when protons of hydrogen atom
percentage of total population of Bihar was 13.14% violently collide in the Sun's core and fuse to create a
However as per census 2011, out of total population of helium atom.
Bihar 11.29% people live in Urban region. 109. The process of conversion of solid camphor to
104. What is the main cash crop of Bihar State? camphor vapours is called
(a) Tea (b) Rubber (a) evaporation (b) freezing
(c) Sugarcane (d) Coconut and Coffee (c) melting (d) sublimation
Ans. (d) : Solid camphor directly converts into vapours
Ans. (c) : Sugercane and Jute are two main cash crops
on heating. This process in which a solid changes
of Bihar. Apart from Sugarcane and jute, potato, directly to vapour is known as sublimation. This process
tobacco, oilseeds, onion, chillies are also main cash takes place when the boiling point is less than the
crops of Bihar. melting point.

44th BPSC (Pre) Exam. 2000-01 94 YCT


110. The process by which plants produce food is 115. Which one of the following metals is used as
called semiconductor in transistors?
(a) Carbohydrolysis (b) Metabolic synthesis (a) Copper (b) Germanium
(c) Photosensitisation (d) Photosynthesis (c) Graphite (d) Silver
Ans. (d) : Photosynthesis is the process by which plants Ans. (b) : Germanium metal is used as a semiconductor
makes their own food in the presence of sunlight. in transistors and various other electronic devices. A
Carbon dioxide present in air, water, miniral and semiconductor transistor is a device used to amplify or
chlorophyll present in leaves. switch electric impulses. Gallium, Arsenide,
In this process plants and other organisms transform Germanium and Silicon are some of the most
light energy into chemical energy. commonly used semiconductors.
111. Which one the following is called dry ice? 116. Substances that are responsible for catalysing
(a) Dehydrated Ice chemical reactions in living systems are
(b) Snow falling on the mountains (a) Bacteria (b) DNA
(c) Solid carbon dioxide (c) Enzymes (d) Proteins
(d) Solid carbon monoxide Ans. (c) : An enzyme is a substance that acts as a
Ans. (c) : Dry ice is the common name for solid carbon catalyst in living organisms. Enzymes are large
dioxide (CO2). It gets this name because it does not melt biomolecules that are responsible for many chemical
into a liquid when heated, instead it changes directly reaction that are necessary to sustain life. Enzymes are
into a gas through the process of sublimation. proteins made from amino acid.
112. Which one of the following is hardest? 117. Which one of the following metals is liquid at
(a) Diamond (b) Glass room temperature?
(c) Quartz (d) Platinum (a) Lead (b) Mercury
Ans. (a) : Diamond is the hardest naturally occurring (c) Nickel (d) Tin
substance found on Earth. Diamonds are made from Ans. (b) : Mercury is the only metal which is liquid at
carbon that has been under high pressure and heat found room temperature. Rest all the given metals are in solid
in the depths of the Earth. The heat and pressure cause form at room temperature.
the diamond molecule to change so that they bond
118. Lack of which substance causes diabetes in a
together. The molecules can't move from a rigid bond,
person?
which makes the Diamond hard.
(a) Glycine (b) Haemoglobin
113. Which one of the following is used in the
(c) Histamine (d) Insulin
manufacture of soaps?
Ans. (d) : Diabetes is caused by deficiency of Insulin.
(a) Vegetable oil (b) Mobil oil
Insulin is a hormone secreated by the beta cells of the
(c) Kerosene oil (d) Cutting oil
pancreas. It is commonly associated with an ability to
Ans. (a) : Vegetable oil is used in the manufacturing of regulate glucose metabolism. Insulin controls the
soaps. Vegetable oils such as Coconut oil, Soybean oil,
amount of sugar in the blood by moving it into the cells,
Almond oil etc are used in soap owing to its fatty acid
where it can be used by the body for energy.
content which helps to make soap hard, sudsy and
conditioning for skin. 119. The vitamin which is found in citrus fruits and
is necessary for maintaining healthy skin is
114. Cooking gas is Mainly
(a) Vitamin A (b) Vitamin B
(a) Carbon dioxide
(c) Vitamin C (d) Vitamin D
(b) Carbon monoxide
(c) Methane Ans. (c) : Vitamin C is found in citrus fruits. It plays an
important role in maintaining skin health. Vitamin C is
(d) Nitrogen and oxygen mixture
also known as Ascorbic Acid. It is water soluble, lack of
Ans. (c) : Cooking gas mainly contain Methane.
vitamin C is caused Scurvy.
Methane is produced by the breakdown or decay of
organic material and can be introduced into the 120. The process by which blood is purified in
atmosphere either by natural process or human human body is called
activities. This gas called Marsh gas because it is found (a) dialysis (b) haemolysis
at the surface of marshy places. (c) osmosis (d) paralysis

44th BPSC (Pre) Exam. 2000-01 95 YCT


Ans. (a) : The process by which blood is purified in the 126. Laughing gas used as anesthesia by doctors is
human body is called dialysis. Dialysis is a treatment (a) nitrogen (b) nitrogen oxide
that filters and purifies the blood using a machine. The (c) nitrous oxide (d) nitrogen dioxide
kidneys filters our blood by removing waste and excess Ans. (c) : Nitrous Oxide (N2O) is also known as
fluid from our body. Dialysis treatment helps to keep laughing gas. It is clinically used as a safest or Local
our fluids and electrolytes in balance when the kidneys anesthetic. It is also used before dental procedures to
can't do their function. reduce anxiety and promote relaxation.
121. Which of the following metals is used as 127. Which one of the following substances is used
filament in lighting bulbs? in the preservation of food stuff?
(a) Iron (b) Molybdenum (a) Citric acid
(c) Silver (d) Tungsten (b) Potassium chloride
Ans. (d) : Tungsten metal is used as filament materials (c) Sodium benzoate
in lighting bulb because it has a very high melting point. (d) Sodium chloride
Due to this high melting point, the tungsten filament can Ans. (c) : Sodium Benzoate is used in preservation of
be kept white hot without melting away. Tungsten has food stuff. In food industry, it is used to prevent
very high resistivity so it does not burn easily at room spoilage from harmful bacteria, yeasts and molds. It
temperature. also helps maintain freshness in food by helping to slow
122. Which of the following is/are largely mined in or prevent changes in colour, flavour, pH and texture.
Bihar? 128. When was 'Damodar Valley Corporation'
I. Coal II. Iron launched?
III. Manganese IV. Petroleum (a) 1950 (b) 1954
Choose the correct answer from the following (c) 1948 (d) 1947
Code Ans. (c) : The Damodar Valley Corporation was
(a) Only I. (b) I and II established in 1948. It was the multipurpose river valley
(c) I and III (d) I, II and IV project of independent India. Under this project four
Ans. (b) : Mainly Coal, Iron and Copper are found in dams were constructed namely:-
Bihar, apart from these, Bauxite, Pyrite, Limestone, Tilaiya dam, Konar Dam, Maithon Dam and Panchet
Graphite, Manganese, Nickel and Chromite are also Dam.The Damodar river originates in the Palamu hills
found in Bihar, but after the formation of Jharkhand, at of Chhota Nagpur plateau in Jharkhand.
present the mineral sector in Bihar has became 129. Area under cultivation in the State of Bihar is
negligible. (a) 95 lakh hectare (b) 85 lakh hectare
123. Turbines and dynamos are used to convert (c) 89 lakh hectare (d) 89.9 lakh hectare
which energy to electrical energy? Ans. (a) : When the question was asked the net
(a) Chemical energy (b) Solar energy cultivated area of Bihar and Jharkhand is 56.03 lakh
(c) Mechanical (d) Magnetic energy hectare and 38.0 lakh hectare respectively. Hence most
Ans. (c) : Turbines and Dynamos are used to convert appropriate answer will be option (a).
mechanical energy into electrical energy. A wind 130. Which country has been the largest source of
turbine transforms the mechanical energy of wind into bilateral foreign aid for India?
electrical energy. (a) USA (b) Japan
Dynamo is an electrical generator that create direct (c) OPEC Countries (d) England
current using a commutator. It uses rotating coils of Ans. (a) : At present United States of America has been
wire and magnetic fields to convert mechanical the largest source of bilateral foreign aid for India.
rotations into a pulsing direct electric current. 131. The railways in India was first opened in
124. Glasses to used in reading are made from (a) 1853 (b) 1854
which type of lenses? (c) 1855 (d) 1859
(a) Concave (b) Convex Ans. (a) : On 16 April 1853 the first passenger train ran
(c) Plane (d) (a) or (b) between Bori Bunder (Bombay) and Thane, a distance
Ans. (d) : Reading glasses are made from both concave of 34 km. It was operated by three locomotives named
and convex lenses. Concave lenses are used for Shahib, Sultan and Sindh and had 13 carriages. It was
nearsighted and convex lenses are used for farsighted. dedicated by Lord Dalhousie.
125. The polluting gas emitted by vehicles is mainly- 132. When was first Indian Satellite 'Aryabhatta'
(a) carbon dioxide (b) carbon monoxide launched?
(c) Marsh gas (d) nitrogen oxide (a) 10 July, 1992
Ans. (b) : Pollutants produced by vehicles include (b) 19 April, 1975
Carbon Monoxide, Hydrocarbons, Nitrogen oxides (c) 1st November, 1966
particles, volatile organic compound & Sulfur dioxide. (d) 2nd December, 1985
Carbon Monoxide is odourless, colourless and Ans. (b) : The Aryabhatta was India's first satellite. It
poisonous gas is formed by the combustion of fossil was completely designed and fabricated in India and
fuels such as gasoline and is emitted from motor launched by a Soviet Kosmos-3M rocket from Kapustin
vehicle. Yar on 19 April 1975.

44th BPSC (Pre) Exam. 2000-01 96 YCT


133. When was 'INSAT -2A' launched? 140. For one revolution around the Sun, Saturn
(a) 4th February, 975 (b) 18th May, 1974 takes
(c) 10th July, 1992 (d) 19th April, 1975 (a) 18.5 years (b) 36 years
Ans. (c) : INSAT-2A was the first Indian multipurpose (c) 29.5 years (d) 84 years
satellite. It was launched on 10 July 1992 from French Ans. (c) : Saturn revolves or orbits around the Sun once
every 29.5 Earth year.
Guyana Site.
141. Choose the correct statement.
134. The envelope of gases surrounding the Earth is
known as (a) Z1 + Z2 ≤ Z1 + Z2 , Z1 Z2 ∈ C
(a) lithosphere
(c) atmosphere
(b) water vapour
(d) hydrosphere
(b) x = { x,x ≥0
− x,x <0 , x ∈ R

Ans. (c) : The Atmosphere is the thin enevolope of (c) i = −1 is real number
gases surrounding the Earth. It is held down by Earth's (d) 0.1234567.............is a rational number
gravitational pull. Lithosphere is rocky outer part of the Ans.(b):Since when
Earth. It is made up of brittle crust and the top part of 2 2 2
the upper mantle. Z1 + Z 2 = Z1 + Z 2 + 2 Z1 Z 2 cos(θ1 − θ 2 )
Hydrosphere is the total amount of water on a planet. then cos(θ1 − θ 2 ) ≤ 1
135. For one rotation around the Sun, Uranus takes 2 Z1 Z 2 cos(θ1 − θ 2 ) ≤ 2 Z1 Z 2
(a) 84 years (b) 36 years
(c) 18 years (d) 48 years (Multiplying on both sides) 2 Z1 Z 2
Ans. (a) : Uranus makes a complete orbit around the either
2 2
Z1 + Z 2 + 2 Z1 Z2 cos(θ1 − θ 2 )
Sun in about 84 Earth years. Uranus travels at an 2 2
average speed of 15,290 miles per hour in its orbit ≤ Z1 + Z 2 + 2 Z1 Z 2
around the Sun. ⇒ Z1 + Z 2
2
≤ [ Z1 + Z 2 ]2
136. Which planet is known as 'Evening Star'?
⇒ Z1 + Z 2 ≤ Z1 + Z 2
(a) Mars (b) Jupiter
(c) Venus (d) Saturn Hence option (a) is not true.
The real value of x when a real number is defined (b)
Ans. (c) : Venus is often called as 'Earth's twin' because
x
they are similar in size & structure. Venus is also called
as "the Morning Star or "the Evening Star" because it is  x, if x ≥0
the first celestial body to become visible in the sky at x =
 − x, if x <0
evening and is the last one to disappear from the sky at
sunrise. So, x = 0 in addition to x ∈ R
137. Which scientist has modified the 'erosional or x > 0
cycle'?
(a) Penck (b) Davis Then x = 0 (only for x = 0)
(c) Hutton (d) Dutton Hence (b) is true
Ans. (a) : Cycle of Erosion was first set forth by 142. If a=13, b=14, c=15, then choose the value of
William M. Davis. Walther Pench is noted for C
tan .
criticizing key element of the Davisian cycle of erosion, 2
conducing that the process of uplift and denudation 1 2
occurrs simultaneously, at gradual and continuous rates. (a) (b)
2 3
138. Oil and Natural Gas Commission was 7 1
established in (c) (d)
(a) 1856 (b) 1914 4 7
(c) 1936 (d) 1956 Ans. (a) : a = 13, b =14, c = 15
a+b+c 13 + 14 + 15 42
Ans. (d) : Oil and Natural Gas Commission was setup ∴s = = = = 21
in 1956 to search for the deposit of mineral oil and 2 2 2
natural gas and to increase their production. It is under c s(s − a)(s − b)(s − c)
∵ tan =
the ownership of Ministry of Petroleum and Natural Gas 2 s(s − c)
of Government of India. c (s − a)(s − b)
or, tan =
139. How many constellations are in our Space? 2 s(s − c)
(a) 87 (b) 88
(21 − 13)(21 − 14) 8× 7 4 2
(c) 89 (d) 90 = = = =
21(21 − 15) 21 × 6 9 3
Ans. (b) : The International Astronomical Union (IAU)
recognized the 88 constellations in our space. The 143. Which of the following statements is false?
constellations are groups of stars Orion, Ursa Major, (a) The square of each negative number exists
Gemini, Aquarius, Leo and Cancer etc are some name (b) The square root of each negative number does
of constellations. not exist
44th BPSC (Pre) Exam. 2000-01 97 YCT
(c) −∞ is a negative number xi (xi- X ) yi (yi- Y ) (xi- X )
(d) π is a transcendental number (yi- Y )
Ans. (c) : −∞ is not a negative number so statements 1 -4 ⋅ 5 6 -1 ⋅ 4 +6 ⋅ 3
(c) is incorrect 2 -3 ⋅ 5 9 +1 ⋅ 6 -5 ⋅ 6
dy 3 -2 ⋅ 5 6 -1 ⋅ 4 +3 ⋅ 5
144. y = x ( x ≠ 0 ) , then choose the value of
dx 4 -1 ⋅ 5 7 -0 ⋅ 4 +0 ⋅ 6
x x 5 -0 ⋅ 5 8 +0 ⋅ 6 -0 ⋅ 3
(a) (b) 1 (c) (d) –1
x x 6 +0 ⋅ 5 5 -2 ⋅ 4 -1 ⋅ 2
7 +1 ⋅ 5 12 +4 ⋅ 6 +6 ⋅ 9
Ans. (b) : y = x , (x ≠ 0)
8 +2 ⋅ 5 3 -4 ⋅ 4 -11 ⋅ 00
or y = x
9 +3 ⋅ 5 17 +9 ⋅ 6 +33 ⋅ 6
dy
∴ =1 10 4⋅5 1 -6 ⋅ 4 -28 ⋅ 8
dx
145. Choose the wrong statement. Σ(x i − X)
(a) The mathematical Text Laghu-Bhaskariya is (yi − Y) = 4 ⋅ 0
written by Bhaskara
(b) The Bakshali's Manuscript contains Patiganit
also
(c) Decimal system was invented by Europeans
1 1
(d) Ganit tilak is an eleventh century text ∴ σ 2 = ∑ (x i − X )(yi − Y ) = × 4 ⋅ 0 = 0 ⋅ 4
n 10
Ans. (c): Decimal system was invented by Europeans is
wrong statement. The origin of decimal number system 149. Choose the value of 13°12' from the following.
with base-10 positional number was invented by Indians. 11 13
and popularized by Arabs and it's still called Hindu- (a) π (b) π
150 150
Arabic number system. 6 13
(c) π (d) π
146. The set {x : x = x + 1} is 125 45
(a) a set of real number  120  0
(b) a set of complex numbers Ans. (a) : 13012' = 130 +   = 130 + 1
 60  5
(c) a set of prime numbers  
(d) all empty set 10  66 
0
= 130 + = 
Ans. (d):The set {x : x = x + 1} is an empty set (since 1 ≠ 2) 5  5 
147. Which of the following statements is false? ∵ 1800 = π
(a) In any right angled the two acute angles are 0
 66  π 66 11
complementary ∴  = × = π
 5  180 5 150
(b) The diagonals of a rhombus bisect one
another at right angle 150. Equations x=5+3cosA and y=7+3sinA
(c) If the diagonals of a quadrilateral bisect each combinedly represent a
other the figure is a parallelogram (a) parabola (b) circle
(d) Every triangle must have at the most one (c) hyperbola (d) pair of straight lines
acute angle Ans. (b) : x = 5+3 cos α
x −5
Ans. (d) : Every triangle must have at the most one or x-5 = 3cos α or cos α =
acute angle is the false statements. Since triangle must 3
2
have atleast two acute angle.  x−5
or cos 2α =   ...........(i )
148. For the data (1, 6),(2, 9),(3, 6),(4, 7),(5, 8),(6,  3 
5),(7, 12)(8, 3),(9, 17),(10, 1) choose the  y−7
covariance. and y =7 + 3 sin α or, sin α =  
 3 
(a) 0.4 (b) 0.6 (c) 4 (d) 6
2
 y−7 
Ans. (a) : Covariance 1
∴ sin 2 α =   .............(ii )
(σ ) 2 = ∑ (x i − X )(y i − Y )  3 
n
by adding (i) and (ii)
2 2
1 + 2 + 3 + 4 + 5 + 6 + 7 + 8 + 9 + 10 55  x−5  y −7 
 +  = 1 since, Sin α + Cos α = 1
2 2
X= = = 5⋅5 
10 10  3   3 
6 + 9 + 6 + 7 + 8 + 5 + 12 + 3 + 17 + 1 74 (x-5)2 + (y-7)2 = (3)2
Y= = = 7⋅4 This is equation of the circle whose center is (5,7) and
10 10
and n = 10 radius = 3

44th BPSC (Pre) Exam. 2000-01 98 YCT


45th Bihar Public Service Commission
Preliminary Examination, 2002
GENERAL KNOWLEDGE & GENERAL SCIENCE
(Solved Paper with Detail Explanation)
1. 'Kumarsambhava', an epic poem was 6. Which one of the following dynasties ruled
composed by over India after the invasion of Timur?
(a) Banabhatta (b) Chandra Bardai (a) Lodi dynasty (b) Syed dynasty
(c) Harisena (d) Kalidasa (c) Tughlaq dynasty (d) Khilji dynasty
Ans. (d) : Kumarsambhava in an epic poem written by Ans. (c) : In 1398, Timur invaded Northern India,
the classical Sanskrit writer Kalidasa, in Sanskrit. It is
attacking the Delhi Sultanate ruled by Sultan
the story of how the young God Kumara was born as
Nasiruddin Mahmud Shah Tughlaq of the Tughlaq
result of the union of Lord Shiva and Parvati and their
boundless love. Dynasty. However Tughlaq dynasty continued to rule
over Delhi Sultanate till 1414 when succeeded by Syed
2. Kautilya's Arthashastra deals with the aspects
of dynasty.
(a) Economic life (b) Political policies 7. Which of the following Sultans had greatest
(c) Religious life (d) Social life number of slaves in his court?
Ans. (a) : The Arthashastra is an Indian treatise on (a) Balban
politics, economics, military strategy, the function of (b) Alauddin Khilji
the state, and social organization written by Kautilya. (c) Mohammad Bin Tughlaq
Kautilya was instrumental in establishing the reign of (d) Feroz Shah Tughlaq
the Chandragupta Maurya, founder of the Mauryan
empire. Ans. (d) : Feroz Shah Tughlaq ruled over the Delhi
Sultanate from 1351 to 1388. He succeeded his cousin
3. Gautam Buddha was elevated to the position of
God by the time of Mohammad – bin Tughlaq following the latter’s death.
(a) Ashoka It is Feroz Tughlaq who had the largest number of
(b) Kanishka slaves in his court. He founded several cities around
(c) Chandragupta Vikramaditya Delhi including Jaunpur, Ferozpur, Ferozabad and
(d) Harsha Fatehabad.
Ans. (b) : Kanishka ruler of Kushan ruler is believed to 8. Which one of the following sultans provided
bring the Mahayan imagery in Buddhism which led to employment to the unemployed?
the idol worship and elevated the Buddha to the level of (a) Balban
Hindu God. (b) Alauddin Khilji
4. Which of the following dynasties succeeded (c) Mohammad Bin Tughlaq
Sungas? (d) Feroz Shah Tughlaq
(a) Satvahanas (b) Kushanas
Ans. (d) : Firoz Shah Tughlaq, the sultan of Delhi,
(c) Kanvas (d) Guptas
established an employment bureau that ensured that all
Ans. (c) : The last Sunga ruler was Devabhuti. He was unemployed people found their desired jobs.
killed by his own minister, Vasudeva Kanva in around
73 B.C. He then established Kanva dynasty at Magadha. 9. Which one of the following Mughal Emperors
5. Which one of the following leaders belonged to gave an important 'Farman' to the English for
the Extremist wing of the Congress? facilitating their trade in India?
(a) Aurobindo Ghosh (b) Dadabhai Naoroji (a) Bahadur Shah I (b) Farrukh Siyar
(c) GK Gokhale (d) SN Banerjee (c) Shah Alam II (d) Bahadur Shah II
Ans. (a) : Aurobindo Ghosh belonged to the extremist Ans. (b) : Farrukh Siyar's Farman (1717) was a grant
wing of the Congress while Dadabhai Naoroji, G.K. giving concessions to the English East India company
Gokhale and S.N. Banerjee belonged to the Moderate and allowing them tax-free monopoly trade especially
wing. in the Mughal subah of Bengal, Bihar and Orissa.

45th BPSC (Pre) Exam. 2002 99 YCT


10. Mahavir Jain breathed his last at Ans. (c) : In 1503 AD, Francisco de Almedia was
(a) Rajgir (b) Ranchi appointed as the first Portuguese viceroy of India. He
(c) Pawapuri (d) Samastipur was known to introduced the blue water policy in India
Ans. (c) : Lord Mahavira died at Pawapuri. Pawapuri is and erected many forts at Cochin, Cannanore (Kannur)
a holy site for Jains located in the Nalanda district in the and Anjediva.
state of Bihar. 16. Who was appointed Deputy Diwan of
Murshidabad by Robert Clive after the
11. A Buddhist Council during the reign of Ashoka
Allahabad Treaty?
was held at
(a) Mohd. Raza Khan
(a) Magadh (b) Pataliputra
(b) Shitab Rai
(c) Samastipur (d) Rajgir
(c) Rai Durlabh
Ans. (b) : The third Buddhist Council was held at (d) Syed Ghulam Hussain
Patliputra in the Magadha empire. It occured under the
Ans. (a) : Mohammad Raza Khan was appointed as the
patronage of emperor Ashoka and presided over by
Deputy Diwan of Bengal by Robert Clive after the
Moggaliputta Tissa.
Allahabad Treaty. The company appointed two deputy
12. Mahabodhi Temple has been built at Bodh diwans for the exercise of Diwani Functions, i.e.
Gaya where Mohammad Raza Khan for Bengal and Raja Sitab Roy
(a) Gautam Buddha was born for Bihar.
(b) Gautam Buddha received enlightenment 17. Which one of the following commissions is
(c) Gautam Buddha gave his first sermon associated with the Army Re-organisation after
(d) Goutam Buddha passed away the suppression of the Revolt of 1857?
Ans. (b) : The Mahabodhi Temple complex is one of (a) Public Service Commission
the four holy sites related to the life of the Lord Buddha. (b) Peel Commission
The site marks the location where the Buddha is said to (c) Hunter Commission
have attained enlightenment. The first temple was built (d) Simon Commission
by emperor Ashoka in the 3rd century B.C. which have Ans. (b) : There was a demand from British Politicians
today disappeared. to Revamp Indian Army even before revolt of 1857 but
13. Kunwar Singh held the revolt of 1857 in after revolt of 1857 Peel Commission was setup. It was
asked to provide recommendations for army
(a) Punjab (b) Bengal reorganization after the suppression of revolt.
(c) Bihar (d) Maharashtra
18. Kuka Movement was organised by
Ans. (c) : Kunwar Singh was a leader during the Indian (a) Guru Ram Das (b) Guru Nanak
Rebellion of 1857. He belonged to a family of the (c) Guru Ram Singh (d) Guru Gobind Singh
Ujjainiya clan of the Parmar Rajputs of Jagdishpur, Ans. (c) : Kuka movement was started as a group for
currently a part of Bhojpur district-Bihar. At the age of religious purification in Sikhism under Guru Ram
80, he led a selected band of armed soldiers against the Singh. Later movement became political with
troops under the command of the British East India established aim of restoring Sikh rule in Punjab and
Company. ousting the British rule. The movement is known to
have used non-cooperation and boycott as tool for first
14. Mundas raised revolt in
time.
(a) 1885 (b) 1888
19. Who was the founder of the India Association?
(c) 1890 (d) 1895 (a) Dadabhai Naoroji
Ans. (c) : Munda Revolt is one of the prominent 19th (b) Bal Gangadhar Tilak
century tribal rebellions in the sub-continent. Birsa (c) AO Hume
Munda led this movement in the region south of Ranchi (d) Surendra Nath Banerjee
in 1899-1900. The revolt in local language is known as Ans. (d) : The Indian Association of Calcutta was
the 'Ulgulan', which sought to establish Munda Raj and founded by Surendranath Banerjee and Anand Mohan
Independence. Bose in 1876. The Association was founded to fight
against injustice. It demanded equal treatment of the
15. Who was the first Portuguese viceroy in India?
Indians with the Europeans while making recruitment to
(a) Diaz (b) Vasco-di-Gama the Indian Civil service. By 1886, this association was
(c) Almeida (d) Albuquerque merged with the Indian National Congress.

45th BPSC (Pre) Exam. 2002 100 YCT


20. By whom was General Dyer shot dead? (c) MG Ranade
(a) Khudiram (b) Bhagat Singh (d) Gopal Krishna Gokhale
(c) Madan Lal Dingra (d) Udham Singh Ans. (a) : Bal Gangadhar Tilak was extremist while
Dadabhai Naoroji, MG Ranade and Gopal Krishna
Ans. (*) : General Dyer, responsible for the Jallianwala
Bagh massacre in Amritsar, died of cerebral Gokhale were moderates.
haemorrhage and arteriosclerosis in 1927. Michael 26. Where was Azad Hind Fauz set up?
O’Dwyer, Lieutenant Governor of the Punjab in India (a) Japan (b) Burma
from 1912 until 1919, who had endorsed general Dyer’s (c) Singapore (d) England
action regarding the Amritsar massacre was
Ans. (c) : Indian National Army or Azad Hind Fauz
assassinated by Sardar Udham Singh in 1940 in
was first established by Mohan Singh in 1942 in
London.
Singapore. Later on in 1943 it was revived by Subhas
21. Quit India Movement was led by Chandra Bose during second World War to secure
(a) BR Ambedkar (b) Jawaharlal Nehru India's complete independence from British Raj.
(c) Mahatma Gandhi (d) None of these 27. Bhagat Singh, Rajguru and Sukhdev were
Ans. (c) : The quit India movement was launched by hanged on
Mahatma Gandhi on 8 August, 1942 at meeting of All (a) March 23, 1931 (b) March 23, 1932
India Congress Committee (AICC) held at Gowalia (c) March 23, 1933 (d) March 23, 1934
Tank Maidan, Bombay.
Ans. (a) : Bhagat Singh, Sukhdev and Rajguru were
The famous 'Do or Die' speech marks the beginning of
sentenced to death in Lahore conspiracy case. They
Quit India Movement were hanged in Lahore jail on 23rd March 1931.
22. The slogan 'Do or Die' was led by
28. Cabinet Mission was presided over by
(a) PC Roy (b) JC Bose
(a) Lord Attlee (b) Strafford Cripps
(c) CV Raman (d) Mahatma Gandhi
(c) Clement Attlee (d) Sir P Lawrence
Ans. (d) : Slogan ‘do or die’ is associated with
Ans. (d) : Sir P Lawrence presided over the cabinet
Mahatma Gandhi. This slogan came into existence
mission which was formed at the initiative of Clement.
during the Quit India Movement in a speech delivered Atlee, the then Prime Minister of the United Kingdom.
in Bombay at the Gowalia tank Maidan on 8 August,
Its other members were Stafford Cripps and A.V.
1942.
Alexander.
23. The second session of the Indian National
29. Who gave the 'Inquilab Zindabad' slogan?
Congress was presided over by
(a) Iqbal (b) MK Gandhi
(a) Ganesh Agarkar
(c) Bhagat Singh (d) SC Bose
(b) Surendra Nath Banerjee
Ans. (c) : The slogan ‘Inquilab Zindabad’ was coined
(c) Dadabhai Naoroji
by Maulana hasrat Mohani however it was popularized
(d) Feroz Shah Mehta
by Bhagat Singh. The meaning of Inquilab is revolution.
Ans. (c) : The second session of the Indian National
30. Where was Shri Yogendra Shukla brought on
Congress held on 27-30 December, 1886 at Calcutta
the 7th December, 1942
was presided over by Dadabhai Naoroji.
(a) Ranchi (b) Munger
24. The Partition of Bengal came into effect to (c) Patna (d) Bhagalpur
(a) 15th August, 1905
Ans. (c) : Yogendra Shukla, escaped from Hazaribagh
(b) 15th September, 1905
th Jail with Jai Prakash Narayana. He was arrested on 7
(c) 16 October, 1905
December, 1942 in Muzaffarpur and then bought to
(d) 15th November, 1905
Patna and finally sent to Buxar jail.
Ans. (c) : Partition of Bengal announced on 19 July
1905 came into effect on 16 October, 1905 during the 31. Which of the following newspapers used to be
viceroyalty of Lord Curzon (1899-1905) proved to be a published from Patna?
momentous event in the history of modern Bengal. (a) Indian Nation (b) Punjab Kesari
25. Who among the following was not known as a (c) Prabhakar (d) Don
moderate in the National Movement? Ans. (a) : 'Indian Nation' Newspaper was established by
(a) Bal Gangadhar Tilak maharaja of Darbhanga, Kameshwar Singh in 1931. The
(b) Dadabhai Naoroji newspaper was published from Patna.

101
32. Jagat Narayan Lal was sent to legislature, executive and judiciary is all bound by the
(a) Bankipur Jail (b) Camp Jail constitution, and nobody, is above or beyond the
(c) Bhagalpur Jail (d) Hazaribagh Jail constitution".
Ans. (d) : Jagat Narayan Lal was one of the most active 39. Who is the executive head of the State
leader during Quit India Movement in Bihar. He was Government?
imprisoned during Non-Cooperation Movement, during (a) The Chief Minister
Civil Disobedience Movement and Quit India (b) The Governor
Movement. He was sent to Hazaribagh Central Jail. (c) The Secretary to the Chief Minister
Thus option (d) is correct. (d) The Chief Secretary
33. Jai Prakash Narayan belonged to which party? Ans. (b) : The Governor is the executive head of the
(a) Congress (b) Kisan Sabha State Government. As per article 154 of constitution
(c) Socialist (d) Raivadi executive power of state shall be vested in the Governor
Ans. (*) : Jai Prakash Narayan joined Indian National and shall be exercised by him either directly or through
Congress in 1929. In 1934, he also played a key role in officers subordinate to him in accordance with
the formation of the Congress Socialist Party. constitution.
Thus both (a) and (c) are correct answer.
40. Who advise State Government on legal
34. Ram Chandra Sharma belonged to the village matters?
(a) Gogri (b) Indrapur (a) Attorney General
(c) Pema (d) Amhara
(b) Advocate General
Ans. (d) : Ram Chandra Sharma belonged to Amhara
(c) Solicitor General
Village.
(d) Chief Justice of the High Court
35. Shri Narsingh Narayan was
(a) Socialist (b) Nationalist Ans. (b) : Advocate General is responsible to advise the
(c) Internationalist (d) Communist state government on the legal matters that the governor
refers to him. He also perform all the duties, of legal
Ans. (a) : Shri Narsingh Narayan was a socialist and he
character as are assigned to him by the governor.
was related to Bihar socialist party.
41. In Indian Polity, the executive is subordinate to
36. The Revolt of 1857 was witnessed by the poet
the
(a) Mir Taqi Mir (b) Zanq
(a) Judiciary
(c) Ghalib (d) Iqbal
(b) Legislature
Ans. (c) : Mirza Asadullah Khan Ghalib, one of the (c) Election Commission
greatest poets, was in Delhi when the uprising of 1857
(d) Union Public Service Commission
took place.
Ans. (b) : In Indian political system executive is
37. Who of the following was the bitterest enemy of
subordinate to legislature as legislature keeps check on
the British during the Revolt of 1857?
executive by various tools like asking questions,
(a) Maulvi Ahmadullah Shah adjournment motions etc.
(b) Maulvi Imdadullah
42. Zero hour in political Jargon refers to
(c) Maulana Faiz-ul-Haq Khairabadi
(a) day when no business in Parliament is done
(d) Nawab Liaquat Ali
(b) suspended motion
Ans. (a) : Ahmadullah Shah, born in 1787, more (c) adjourned time
famously known as Maulavi of Faizabad, was one of the
(d) question-answer session
leading figures of the great Indian revolt of 1857. In the
Awadh region, Maulavi Ahmadullah Shah was known Ans. (d) : In political terminology 'Zero Hour' refers to-
a question - answer session in which issues of urgent
as the ‘Lighthouse of Rebelion’.
public importance can be raised by members. It starts
38. In Indian Polity, which one is Supreme? immediately after question hour at 12 Noon. In zero
(a) The Supreme Court (b) The Constitution hour matter can be raised on very short notice.
(c) The Parliament (d) Religion 43. By what term, the ten commands of 42nd
Ans. (b) : In India Constitution is Supreme opposed to Constitutional Amendment are called?
USA where Supreme court is Supreme and UK where (a) Fundamental Rights
Parliament is Supreme. The constitutional supremacy (b) Fundamental Duties
was explicitly reiterated in the Minerva Mills case (c) Panchayati Raj Principles
whereby the Supreme Court held that "government, (d) Directive Principles of State Policy

45th BPSC (Pre) Exam. 2002 102 YCT


Ans. (b) : The ten commands of 42nd constitutional 49. Relative Poverty refers to
amendment are called as fundamental duties. Ten (a) Bankruptcy
fundamental duties were added to constitution by 42nd (b) trade cycles
Amendment Act. It was recommended by Swaran Singh (c) minimum needs of life
Committee.
(d) economic inequalities
44. Panchayati Raj System in India is laid down
under Ans. (d) : Relative poverty is the condition in which
(a) Fundamental Rights people lack the minimum amount of income needed in
(b) Fundamental Duties order to maintain the average standard of living in the
(c) Directive Principles of State Policy society in which they live. Relative poverty is
(d) All of the above considered the easiest way to measure the level of
poverty in an individual country. Relative poverty also
Ans. (c) : Panchayati Raj system of India is laid down
under the Directive Principles of State Policy which are changes over time. As the wealth of a society increases,
enumerated in Part IV of the constitution. Article 40 so does the amount of income and resources that the
directs the state to organize Village Panchayats and society deems necessary for proper conditions of living.
endow them with necessary powers and authority to 50. National income estimates in India are
enable them to function as units of self government. compiled by
45. Which article of the Constitution of India gives (a) Planning Commission
precedence to constitutional provisions over the (b) RBI
laws made by the Union Parliament/ State
(c) Central Statistical Organisation
Legislatures?
(a) 13 (b) 32 (d) Indian Statistical Institute
(c) 245 (d) 326 Ans. (c) : National Income estimates in India is
Ans. (a) : Article 13 of the constitution of India gives prepared by Central Statistical Organisation (CSO).
precedence to constitutional provisions over the laws Presently CSO has been renamed as National Statistical
made by the Union Parliament/State legislature. Office.
46. Who is the Chairman of Rajya Sabha? 51. Which important industry is located at Muri?
(a) The President (b) The Vice-President (a) Aluminum manufacturing
(c) The prime Minister (d) The Speaker (b) Copper manufacturing
Ans. (b) : Article 63 of the constitution of India (c) Copper manufacturing
provides that there shall be a vice-president of India.
(d) Chemical Fertilizers
Article 64 and 89(1) provide that the Vice-President of
India shall be ex-officio chairman of the council of Ans. (a) : The plants for the extraction of alumina from
states i.e. Rajya Sabha and shall not hold any other bauxite are located at Muri (Jharkhand) near the Bauxite
office of profit. mines. India's first alumina refinery was commissioned
47. What is the minimum age laid down for a at Muri.
person to seek election to the Lok Sabha? 52. Maithon, Belpahari and Tilaiya dams are
(a) 18 years (b) 21 years constructed on the river
(c) 25 years (d) None of these (a) Damodar (b) Barakar
Ans. (c) : Article 84(b) of constitution of India provides (c) Konar (d) Kokaro
that the minimum age for becoming a candidate for Lok
Sabha election shall be 25 years. Ans. (b) : Maithon, Belpahari and Tilaiya dams are
constructed on river Barakar.
48. Main objective of IRDP is
(a) Export Promotion 53. Triveni canal receives water from river
(b) Sound Fiscal Management (a) Sone (b) Kosi
(c) Banking Reforms (c) Gandak (d) Mayaurakshi
(d) Rural Development and Poverty Alleviation Ans. (c) : Triveni Canal gets water from the river
Ans. (d) : The main objective of IRDP is to enable Gandak. Gandak river is also known as Narayani river.
identified rural poor families to augment their income It is one of the tributaries of the Ganga river.
and cross the poverty line through acquisition of credit 54. Which the Indian State having the maximum
based productive assets. Assistance is given in the form population density?
of subsidy by the government and term credit by the (a) Delhi (b) West Bengal
financial institutions for income generating activities.
(c) Kerala (d) Uttar Pradesh
103
Ans. (b) : As per census 2011, Bihar has the highest 61. Which new state was recently carved out of
population density of 1106 Persons Per sq. km. Bihar?
followed by West Bengal with a population density of (a) Vananchal (b) Vidharbha
1028 Persons Per sq. km. (c) Jharkhand (d) Chhattisgarh
Note : When the question was asked West Bengal was Ans. (c) : The State of Jharkhand was carved out of
the highest in population density. Bihar on 15th November, 2000.
55. Which is the maximum wheat producing state 62. After the formation of Jharkhand, how many
of India? districts were left with Bihar?(Nearest figure)
(a) Uttar Pradesh (b) Punjab (a) 37 (b) 65
(c) Haryana (d) Madhya Pradesh (c) 60 (d) 62
Ans. (a) : As of 2020-21 Uttar Pradesh is the maximum Ans. (a) : After the formation of Jharkhand, 37 districts
wheat producing state of India, followed by Madhya were left with Bihar. At present there are 38 districts in
Pradesh & Punjab. Bihar.
56. Lakshadweep consists of how many Islands? 63. The principal foodgrain crops of Bihar
(a) 17 (b) 27 (a) rice, wheat and maize
(c) 37 (d) 47 (b) sugarcane, tea and barley
Ans. (*) : India’s smallest union territory Lakshadweep (c) groundnut, coffee and wheat
is an archipelago consisting of 36 islands with an area (d) None of the above
of 32 sq.km. It is a uni-district Union Territory and is Ans. (a) : Rice, wheat, maize and pulses are the
comprised of 12 atolls, - three reef, five submerged principal foodgrain crops of Bihar.
banks and ten inhabited islands.
64. Which of the following is true?
57. One of the highest falls in Asia, 'Hundru' is (a) Bihar has been the leading producer of coal
situated near and mica
(a) Ranchi (b) Hazaribagh (b) Bihar was separated from Odisha in 1911
(c) Jamshedpur (d) Bodh Gaya (c) Bihar shares its border with Odisha after the
Ans. (a) : The Hundru falls Ranchi is created on the formation of Jharkhand State
course of the Subarnarekha River, where it falls from a (d) Bihar's legislature is unicameral
height of 320 feet creating the highest waterfalls of the
Ans. (a) : When the question was asked “Bihar was the
state.
leading producer of coal and mica.
58. Which is the best suited crop for dryland?
65. Which city of Bihar became the capital of the
(a) Sugarcane (b) Jute Jharkhand State?
(c) Wheat (d) Groundnut (a) Buxar (b) Dhanbad
Ans. (d) : Among the options ground nut is relatively (c) Ranchi (d) Hazaribagh
tolerant to drought thus best suited crop for dryland. Ans. (c) : Ranchi became the capital of the Jharkhand
59. Approximately how much distance per minute State, when it was carved out of Bihar in 2000.
the Earth is rotating? 66. The word 'Bihar' originally means
(a) 28 km (b) 59 km (a) Buddhist Monastery
(c) 69 km (d) None of these (b) Land of Angels
Ans. (a) : At the equator the rotation speed of earth is (c) Arya Pradesh
about 1670 km/hr. which is approximately equal to (d) Land of Greenery
28km/min. Therefore, it covers 28 km in one minute Ans. (a) : Bihar is derived from a Pali word ‘Vihara
while rotating. which means ‘abode’ and has a Sanskrit origin.
60. Highest peaks of the world are mostly found in Vihara means a Buddhist monastery.
which type of mountains? 67. The total length of National Highways in Bihar
(a) Old folded mountains is
(b) Young folded mountains (a) 1913 km (b) 1088 km
(c) Residual mountains (c) 1313 km (d) 1371 km
(d) Block mountains Ans. (a) : As per data of NHAI the total length of
Ans. (b) : Highest peaks of the world are mostly found National Highways in Bihar is 5358 km. when the
in Young folded mountains. For example Himalayas. question was asked it was 1913 km.

45th BPSC (Pre) Exam. 2002 104 YCT


68. What is the percentage of Bihar's population in 75. Which of the following places is famous for its
the total Indian population? wildlife in Bihar?
(a) 9.33 % (b) 10.21 % (a) Bhagalpur (b) Bodh Gaya
(c) 16.44% (d) 8.04% (c) Patna (d) Hazaribagh
Ans. (d) : Bihar population is 8.60% of the India’s Ans. (c) : Patna is famous for its wildlife in Bihar as it
population according to 2011 census. When question has Sanjay Gandhi Biological Park, Patna. It has great
was asked it was 8.04%. one horned rhino, hippopotamus and alligators etc.
69. Jamshedpur is located on the bank of which 76. Three-tier system of Panchayati Raj consists of
river? (a) Gram Panchayat, Panchayat Samiti, Block
(a) Damodar (b) Brahmani Samiti
(c) Subarnarekha (d) Ajay (b) Gram Panchayat, Block Samiti, Zila Parishad
Ans. (c) : Jamshedpur city is located on the banks of the (c) Block Samiti, Zila Parishad, Panchayat Samiti
river Subarnarekha. The city is known to have founded (d) Gram Panchayat, Panchayat Samiti, Zila
by Jamshedji Tata. Parishad
70. Name the most important iron ore producing Ans. (d) : The three tier Panchayat institutions in Bihar
district of Jharkhand. consists of a gram panchayat at the lowest level,
(a) Daltanganj (b) Hazaribagh Panchayat Samiti at the block level and Zila Parishad at
(c) Dhanbad (d) Singhbhum the district level. The chairman of gram Panchayat is
called Mukhiya, the Chairperson of the Panchayat
Ans. (d) : Singhbhum is the most important iron are
Samiti is called Pramukh and the chair person of zila
producing site in Jharkhand.
Prishad is called as Adhyaksha.
71. Who among the following is the common
77. 73rd Constitutional Amendment relates to
Governor of Bihar and Jharkhand?
(a) Impeachment of the President
(a) Vinod Kumar Pandey
(b) Appointment of Election Commission
(b) Vimal Kumar Dubey
(c) Reservation of seats in Educational
(c) Vishnu Kant Shastri
Institutions
(d) Girish Chaubey
(d) Panchayati Raj System
Ans. (a) : Vinod Kumar Pandey was the common
Ans. (d) : 73rd Constitutional Amendment is related to
governor of Bihar and Jharkhand. He served as
Governor of Bihar (1999-2003) and Jharkhand (briefly the Panchayati Raj System. It added 11th Schedule to
the constitution.
during 2002).
72. The outstanding liability of the State of Bihar is 78. Which two of the following taxes are indirect
taxes?
(a) 37653 crore (b) 67653 crore
(a) Sales tax and Income tax
(c) 87663 crore (d) 90000 crore
(b) Income tax and Wealth tax
Ans. (*) : The outstanding liability of the Bihar in 2002
(c) Sales tax and Excise duty
was 34140 crore.
(d) Income tax Excise duty
73. Who was Bihar State Election Commissioner?
Ans. (c) : Indirect tax is defined as the tax imposed by
(a) M. L. Majumdar (b) Renuka Bajaj
the government on a tax payer for goods and services
(c) L. M. Singhvi (d) None of these
rendered. Sales tax and excise duty are indirect tax.
Ans. (a) : M.L. Majumdar was Bihar State election
79. Devaluation of Rupee was done last time on
commissioner during 2000-2003. Currently Bihar State
(a) July, 1991 (b) July, 1993
Election Commissioner is Deepak Prasad. (c) July, 1997 (d) July, 2001
74. Which of the following districts of Bihar Ans. (a) : Last time India rupee was devalued in 1991
transferred some of its areas to west Bengal? amidst economic crisis. India was facing Balance of
(a) Jamshedpur Payment crisis and had very limited foreign reserves to
(b) Bhagalpur and Samastipur carry out international trade.
(c) Purnea and Manbhum 80. Agriculture sector directly employs how many
(d) Rajgir percentage of labour force in India?
Ans. (c) : Some parts of Purnea and Manbhum districts (a) 20% (b) 30%
of Bihar were ceded to West Bengal. (c) 40% (d) 65%

105
Ans. (d) : When the question was asked, agriculture 86. The dispute relating to the Babri
sector directly employed 65 percentage of labour force Masjid/Ramjanambhoomi before the
in India. As per Periodic Labour Force Survey 2020 Allahabad High Court (Lucknow Bench) is in
almost 49% Labour force is employed in agriculture the nature of a
sector. (a) Writ Petition
81. 9th Five Year Plan in India covers the period of (b) Title Suit
(a) 1992-1997 (b) 1997-2002 (c) Claim for Compensation
(c) 1992-1995 (d) None of these (d) Judicial Review Petition
Ans. (b) : The Ninth Five Year Plan, launched in (1997- Ans. (b) : The dispute relating to the Babri
2002) the 50th year of India’s Independence with the Masjid/Ramjanam Bhoomi before the Allahabad High
focus “Growth with Social Justice and Equity”. The 9th Court (Lucknow Bench) was in the nature of a Title
five year plan failed to achieve the growth target of 6.5 suit.
percent and achieved a growth rate of 5.4 percent. 87. The first President of Independent India hails
82. Which sector of Indian Economy has shown from
remarkable expansion during the last decade? (a) UP (b) Andhra Pradesh
(a) Primary Sector (b) Secondary Sector (c) Bihar (d) Tamil Nadu
(c) Tertiary Sector (d) Mining Sector Ans. (c) : Dr. Rajendra Prasad was the first President of
Ans. (c) : Tertiary sector of Indian Economy has Independent India who hails from Bihar. He held office
shown remarkable expansion during the last decade. of President for two terms, he stayed in office from
The economy is divided into three sector on the basis of 1950 to 1962.
activities primary, secondary and tertiary. The tertiary 88. Name of the First Private University of India
sector covers a wide variety of activities from (a) IGNOU
commerce to administration, transport, financial and (b) JNU
real estate activities, business and personal services, (c) Amity Information Technology University
education health and social work. (d) Aligarh Muslim University
83. Per capita income of a country is derived from Ans. (c) : Among the mentioned options only Amity
(a) Population Information Technology University is private
(b) National income University.
(c) National income and population both 89. Who, among the following is elected to the
Rajya Sabha from the Bihar?
(d) None of the above
(a) Sunil Dutt (b) Rabri Devi
Ans. (c) : The per capital income of a country is derived (c) Sahib Singh Verma (d) None of these
from both National Income and population. Per capita
Ans. (d) : Among the following no person has been
income is a measure of the amount of money earned per
elected to the Rajya Sabha from the Bihar.
person in a nation or geographic region. It is obtained
by dividing national income by the total population of 90. Who is the US Envoy appointed to ease tension
the country. in West Asia?
(a) Anthony Zini (b) Louis Pischer
84. At present India's largest trading partner is
(c) Kissinger (d) Tom Lavis
(a) Russia (b) Britain
Ans. (a) : Anthony Zini was a former United States
(c) Australia (d) USA
Marine corps general and a former Commander in Chief
Ans. (d) : When the question was asked, U.S.A. was of United states Central Command. From 2001–2003,
India’s largest trading partner. Presently (June 2022), as he served as a special US envoy appointed to ease
per data released by the Ministry of Commerce USA is tension in West Asia.
India's largest trading partner followed by China.
91. In total, there are how many Nationality
85. Which of the following payment instruments Highways in India and approximately what is
introduced by the banks is known as Plastic their total length?
money?
(a) 34 and 16000 km (b) 44 and 24000 km
(a) Bearer cheques (b) Credit cards
(c) 54 and 32000 km (d) 64 and 40000 km
(c) Demand drafts (d) Gift cheques
Ans. (d) : When the question was asked the total no. of
Ans. (b) : Plastic money is the use of plastic cards like National Highways were 64 & approximately total
debit or credit card in the form of electronic length was 40,000 km. As per NHAI data presently
transactions. there are 599 highways.
45th BPSC (Pre) Exam. 2002 106 YCT
92. How much is the difference between Indian 98. Where is the governing centre of reflex action
Standard Time (IST) and Greenwich Mean located?
Time (GMT)? (a) In cerebrum (b) In cerebellum
1 1
(a) +4 h (b) +5 h (c) In spinal cord (d) In nerve cells
2 2
Ans. (c) : Governing centre of reflex action is located in
1 1
(c) −5 h (d) −4 h spinal cord.
2 2
99. An element X has four element in its outermost
Ans. (b) : The difference between Indian standard time
1 orbit. What will be the formula of its
(IST) and Greenwich Mean Time (GMT) is +5 h . compounds with hydrogen?
2
(a) X4H (b) X4H4
93. What is the percentage of forest area in total
geographical area of India? (c) XH3 (d) XH4
(a) 20.7% (b) 21.7% Ans. (d) : An element X having four elements in its
(c) 22.7% (d) 23.7% outer most orbit means the element X has its four
Ans. (c) : When question was asked forests accounted valency to add four atoms of other element having the
for 22.7% area of total geographical area of India. valency one. It may be CH4 (XH4).
According to the latest edition of India’s ‘State of 100. What is the shape of sulphur hexafluoride
Forest Report’ for the year 2021, the percentage of
forest area in total geographical area of India is 24.62% molecule?
94. The Bhakra-Nangal Dam is constructed over (a) Trigonal pyramid (b) Octahedral
which river? (c) Planar (d) Tetrahedral
(a) Ravi (b) Indus Ans. (b) : Sulfur hexafluoride molecule or SF6 is an
(c) Chenab (d) Sutlej inorganic, greenhouse gas. It is non-flammable,
Ans. (d) : Bhakra Dam is located in Bhakra village in odorless, and colourless, and is an excellent insulator.
Bilaspur district of Himachal Pradesh near the border of
Punjab and Himachal Pradesh. Nangal Dam is another SF6 molecule has an octahedral shape. The central atom
dam downstream of Bhakra Dam. Both the dams are bonds with six fluorine atoms and make 8 faces thus
called Bhakra-Nangal Dam though they are two the shape of SF6 is octahedral.
separate dams. 101. Which scientist discovered telescope?
95. Assam is surrounded by how many States and
(a) Newton (b) James Watt
Union Territories?
(a) 6 (b) 7 (c) Rutherford (d) Galileo
(c) 8 (d) 9 Ans. (d) : It is not clear who invented telescope
Ans. (b) : Assam is surrounded by seven States. No however first person to apply for a patent on telescope
Union Territory shares border with Assam. Assam is was Hans Lippershey in the Netherlands in 1608.
situated in the North-East of India and is the largest
After learning the new device, the great Italian scientist
northeastern state in terms of population while second
in terms of area. Galileo Galilei designed and built his own telescope.
96. Who invented aeroplane? Thus among the options, (d) can be considered as
(a) Wright Brothers (b) James Watt correct answer.
(c) Humphrey Davy (d) Von Brown 102. Which hormone stimulates the thyroid gland to
Ans. (a) : Wright Brothers invented aeroplane. On 17th secrete Thyroxine?
December, 1903, the Wright brothers made first (a) TSH (b) FSH
successful flight in history.
(c) LTH (d) ACTH
97. How many pairs of nerves arise from the spinal
cord? Ans. (a) : Thyroid Stimulating Hormone (TSH),
(a) 13 (b) 31 stimulates the thyroid gland to secrete thyroxin.
(c) 33 (d) 12 103. Where does fertilization occur?
Ans. (b) : The spinal cord is an extension of the Central (a) In uterus (b) In oviduct
Nervous system (CNS), which consists of the brain and (c) In ovary (d) In Vagina
spinal cord. The spinal cord begins at the bottom of the
Ans. (b) : The fertilization of sperms & egg take places
brain stem (at the area called the medulla oblongata)
and ends in the lower back. There are 31 pairs of spinal in oviduct that is the part of fallopian tube. Once
nerves, located at the cervical, thoracic, lumbar, sacral fertilization takes place this newly fertilized cell is
and coccygeal. called zygote.

107
104. Which physical quantity is represented by the 110. What is the usual property of oxides of Group
ratio of momentum and velocity of the body? III and IV?
(a) Velocity (b) Acceleration (a) Basic and Acidic (i.e., Amphoteric)
(c) Mass (d) Force (b) Basic
(c) Acidic
Ans. (c) : Mass is represented by the ratio of (d) Neutral
momentum and velocity of the body.
Ans. (a) : The usual characteristics of oxides of Group
P III and IV of the periodic table is that they are
Since, P = MV → Hence, =M Amphoteric i.e. they can able to react both as base and
V
acid.
105. Force is the product of 111. Which of the following carries a negative
(a) mass and velocity charge?
(b) mass and acceleration (a) X-rays (b) Alpha particles
(c) weight and velocity (c) Beta particles (d) Gamma rays
(d) weight and acceleration Ans. (c) : Beta particles (β) are high energy, high speed
– T
Ans. (b) : Force can be defined as the product of mass electrons (e ) or positrons (β ) ejected from the nucleus
and acceleration. The product of mass (m) of an object during radioactive decay.
However X ray, and Gamma rays are neutral and alpha
and the acceleration (a) of the body is called force.
particles carry positive charge.
F = ma
112. What is the mass number of an element, the
106. Which of the following quantities is not a atom of which contains two protons, two
vector? neutrons and two neutrons and two electrons?
(a) Displacement (b) Velocity (a) 2 (b) 4
(c) Force (d) Volume (c) 6 (d) 8
Ans. (d) : The physical quantities displacement, Ans. (b) : The mass number is defend as the total
number of protons and neutrons of an atom.
velocity & force are called vector quantities but volume
Therefore, the mass number of this element is = 2
is scalar quantity. protons + 2 neutrons = 4.
107. A distance of 1 km means 113. What is the distance between two successive
(a) 100 m (b) 1000 m crests or successive troughs called?
(c) 100 m (d) 100cm (a) Amplitude (b) Wavelength
Ans. (b) : Distance of 1 kilometer = 1000 meter. (c) Frequency (d) None of the above
108. What is the correct equation for finding the Ans. (b) : The distance between two successive crests
and successive troughs is called as wavelength.
acceleration?
114. Who accords recognition to various political
v−u
(a) a = (b) a = u + vt parties in India as National or Regional
t Parties?
v+u v+u (a) The Parliament
(c) a = (d) a =
t 2 (b) The President
Ans. (a) : If t is time taken, v is final velocity and u is (c) The Election commission
initial velocity then the acceleration is given by the (d) The Supreme Court
formula : Ans. (c) : The election commission accords recognition
to various political parties in India as National or State
⇒ v = u + at parties.
⇒ at = v – u 115. Which one of the following is a Fundamental
v−u Duty in Indian Constitution?
⇒ a=
t (a) Separation of Judiciary from executive
(b) To value and preserve the rich heritage of our
109. Which of the following is the electronic composite culture
configuration of a metallic element? (c) Free and compulsory education for children
(a) 2,8 (b) 2,8,7 (d) All of the above
(c) 2,8,8 (d) 2,8,8,2 Ans. (b) : As per Article 51(a) of the constitution of
Ans. (d) : Metals usually have 1, 2 or 3 electrons in India it shall be the fundamental duty of every citizen of
their valence shell. Electronic configuration of Calcium India to value and preserve the rich heritage of our
(metallic element) is 2, 8, 8, 2. composite culture.

45th BPSC (Pre) Exam. 2002 108 YCT


116. Who got the 1998 Nobel Prize in Economics? British writer. ‘Business as Usual’ is novel written by
(a) Solow (b) Marshall Jane Oliver and Ann Stafford. ‘A Tear and a Smile’ is
(c) Amartya Sen (d) Paul Samuelson written by Kahlil Gibran.
123. When is Samajik Adhikarita Smriti Diwas
Ans. (c) : The Sveriges Riksbank Prize in Economic
celebrated?
Sciences in Memory of Alfred Nobel 1998 was awarded (a) March, 18 (b) March, 19
to Amartya Sen “For his contribution to welfare (c) March, 20 (d) March, 21
economics.”
Ans. (c) : Samajik Adhikarita Smriti Diwas is
117. the Liberhan Commission of enquiry is related celebrated on 20th March every year.
to 124. World Consumers Rights Day is celebrated on
(a) Demolition of Babri Masjid (a) March, 13 (b) March, 14
(b) Construction of Ram Temple (c) March, 15 (d) March, 16
(c) Hawala Scam Ans. (c) : World Consumers Rights Day is celebrated
(d) Bihar Fodder Scam on 15th March every year as a means of raising global
awareness about consumer rights and needs.
Ans. (a) : Liberhan Commission headed by Justice M.S.
Liberhan was set up on 16 December, 1992. The 125. Chief Minister of which of the following States
has launched the Annadanam Scheme at 63
Commission probed the sequence of events that led to
selected temples?
the Babri Masjid demolition in Ayodhya.
(a) Kerala (b) Andhra Pradesh
118. Capital of Rajasthan is (c) Karnataka (d) Tamil Nadu
(a) Jaipur (b) Udaipur Ans. (d) : The Annadhanam scheme was launched by
(c) Jodhpur (d) Ajmer the Chief Minister of Tamil Nadu. The scheme aims to
Ans. (a) : Jaipur is the capital of Rajasthan. Rest all are help the devotees to get easy food in the temples.
cities in Rajasthan. 126. The biggest US ground operation in
Afghanistan is Known as
119. Capital of Gujarat is
(a) Operation Anaconda
(a) Godhra (b) Baroda
(b) Operation Osama Bin Laden
(c) Gandhi Nagar (d) Ahmedabad (c) Operation United States
Ans. (c) : Gandhi Nagar is the capital of Gujarat. (d) Operation Blue Star
120. The native African tribe 'Pygmies' are found in Ans. (a) : ‘Operation Anaconda’ was the biggest US
which of the following river basins? ground operation in Afghanistan. It was launched by the
(a) Niger (b) Congo President of US, George W. Bush in attempt to destroy
Al-Qaeda and Talibani Forces after 9/11 attacks on the
(c) Nile (d) Zambezi
worlds Trade Center and the Pentagon in US.
Ans. (b) : Pygmies are native African tribe found Operation Neptune Spear was launched by US in May,
mainly in Congo river basin. Their average height is 2011 to capture Osama Bin Laden in Abbotabad,
Unusually short. Pakistan.
121. Which country of the world is the largest ‘Operation Blue Star (1984) was the mission undertaken
producer and exporter of soft timber and wood by the Indian Army during Indira Gandhi’s government
pulp? against the Sikh Militants who were accumulating
(a) USA (b) Norway weapons in the Golden Temple.
(c) Sweden (d) Canada 127. Identify the number of wards in the Patna
Municipal Corporation
Ans. (d) : When question was asked the largest
(a) 57 (b) 72
producer and exporter of soft timber and wood pulp was
Canada. But now, China ranks first. (c) 64 (d) 59
Ans. (*) : Patna Municipal Corporation is divided into
122. Anita Roddick is the author of
total 75 wards.
(a) The Untold Story
(b) Business as Usual 128. Olympics for the disabled is known as
(a) Simpilympics for (b) Combilympics
(c) Business as Unusual
(c) Andilympics (d) Abilympics
(d) Tears and Smiles
Ans. (d) : Olympics for the disabled is known as
Ans. (c) : ‘Business as Unusual’ book is written by ‘Abilympics’. ‘Abilympics’ (Olympics of Abilities) are
Anita Roddick. It is her autobiography that charts the vocational skills competitions specifically designed for
story of Roddick and her company the Body Shop. persons with disabilities to enable them to expose their
‘Untold Story’ is a novel written by Monica Ali, a
unique talents.
109
National Abilympic Association of India (NAAI), 134. Which of the following countries is hosting the
headquartered in New Delhi, is the nodal agency in new ICC Champions Trophy in September,
India responsible to organize vocational skill 2002?
competitions for disabled. (a) Sri Lanka (b) Pakistan
129. What is the name of Japan's Crown princess? (c) India (d) West Indies
(a) Toko (b) Tono Ans. (a) : The International Cricket Council (ICC)
(c) Aiko (d) Piko Champions Trophy 2002 was hosted by Sri Lanka in the
Ans. (c) : This was Princess Aiko, who was born on 1st month of September, 2002. India and Sri Lanka were
December, 2001, to then Crown Prince Naruhito and declared joint champions of ICC Trophy 2002.
Crown Princess Masako. 135. Which of the following films is awarded the
130. Who among the following is convicted by the Oscar 2002, in the category of Best Foreign
Supreme Court for contempt of Court in Language Film?
March 2002? (a) Lagaan
(a) Shabana Azmi (b) No Man's Land
(b) Arundhati Roy (c) A Beautiful Mind
(c) Arun Jaitley (d) The Lord of the Rings
(d) Keshavananda Bharti Ans. (b) : Bosnia and Herzegovina’s No Man’s Land
Ans. (b) : For speaking against the Narmada Dam directed by Danis Tanovic, was awarded Oscar 2002, in
project in 2002, Booker Prize-Winning writer and the category of Best Foreign Language Film.
activist Arundhati Roy was sentenced to one day of 136. CHOGM-2002 with 51 members was held at
simple imprisonment in Tihar Jail, as a ‘symbolic’ (a) Coolum (b) Copenhagen
punishment for contempt of court. (c) Harare (d) Dublin
131. Who is the Air Chief Marshal of India? Ans. (a) : The meeting of the Commonwealth Heads Of
(a) S. Krishnaswamy (b) Krishna Kant Government (CHOGM) took place between the 2nd and
(c) Vishnu Kant (d) Rama Reddy 5th March 2002, in Coolum on the Sunshine Coast, in
Ans. (a) : S. Krishnaswamy born, in 1943, was the Air Queensland, Australia.
Chief Marshal of India during the period of 31st 137. Who represented India at the Second SAARC
December 2001- 31st December 2004. Information Ministers' Conference in
132. What is the name of the new superfast train Islamabad in March, 2002?
between New Delhi and Patna introduced with (a) Murli Manohar Joshi
effect from February 2,2002? (b) Uma Bharti
(a) Sabarmati Express (c) Sushma Swaraj
(b) Sampoorna Kranti Express (d) Mamta Banerjee
(c) Patliputra Express Ans. (c) : The then Minister of Information and
(d) Shalimar Express Broadcasting Smt. Sushma Swaraj represented India at
Ans. (b) : Sampoorna Kranti Superfast Express was the Second SAARC Information Ministers’ Conference
introduced between New Delhi and Patna on 17th in Islamabad in March, 2002.
February 2002. 138. The Chairman of the Constitutional Review
Sabarmati Express runs between Ahmedabad (Gujarat) Commission set up in February, 2000 is
and Varanasi (Uttar Pradesh). The Express is related to (a) M N Venkatachaliah (b) R S Sarkaria
the Godhra train buring case, happened in February, (c) Krishna lyer (d) Fatima Bibi
2002. Patliputra Express was started in 1999 to Ans. (a) : The National commission to Review the
transport passengers from Mumbai to Patna. Shalimar working of the Constitution was set up by the
Express runs daily between Delhi and Jammu Tawi. Government of India in February, 2000, under the
133. Which of the following actors has been chairmanship of Justice M.N. Venkatachaliah.
conferred the honorary degree (Honoris cousa) The Commission emphasised the vast potential of
by the Aligarh Muslim University in February, article 263 of Indian Constitution and that has not been
2002? fully utilized for resolving various problems concerning
(a) Amitabh Bachchan (b) Amir Khan more than one state.
(c) Shatrughan Sinha (d) Dilip Kumar 139. Who is elected for the fourth consecutive term
Ans. (d) : The legendary Bollywood actor Yusuf Khan as General Secretary of the CPI (M) at the 17th
better known as Dilip Kumar, was conferred the Party Congress at Hyderabad in March, 2002?
honorary degree (Honoris cousa) by the Aligarh Muslim (a) Chandrababu Naidu
University in February, 2002. (b) Harkishan Singh Surjeet
45th BPSC (Pre) Exam. 2002 110 YCT
(c) Khushwant Singh
(d) Mamta Banerjee
Ans. (b) : Harkishan Singh Surjeet was elected as
General Secretary of the Communist Party of India
(Marxist), at the 17th Party Congress, held at Hyderabad
in March, 2002.
140. Who is the Chairman of the National Human
Rights Commission?
In triangle DEA (AD = DE)
(a) J S Verma (b) P N Bhagwati
(c) Ahmadi (d) H L Dattu ∴θ + θ + 150o = 180o
Ans. (a) : The National Human Rights Commission 2θ = 30o
(NHRC) of India was established on 12th October, 1993, θ = 15o
under the Protection of Human Rights Act (PHRA),
Hence, ∠DAE = 15o
1993. Justice Shri Ranganath Mishra was the first
Chairman (1993-1996) of NHRC. Justice Shri J.S. xe x - log(1 + x)
143. lim equals
Verma has been the chairman of NHRC during the term x →0 x2
of (November, 1999 – January, 2003). Justice Shri H.L. 1 3
Dattu’s term was (2016 - 2020). Justice Shri Arun (a) (b)
2 2
Kumar Mishra is the Chairman of NHRC from June, 5
2021. (c) 0 (d)
2
141. The price of a commodity increased by 5%
xe x − log (1 + x )
from 1995 to 1996, 8% from 1996 to 1997 and Ans. (b) : lim equals
77% from 1997 to 1998. Then, the average x →0 x2
price increase from 1995 to 1998 is Applying L 'Hospital's rule we get,
(a) 24% (b) 32% 1
xe x + e x −
(c) 26% (d) 30% (1 + x )  0 
lim  0  form
Ans. (c) : Price in 1995 = 100 x → 0 2x  
5 again applying L' Hospital rule, we get
Price in 1996 = 100 + 100 × = 105 1
100 xe x + 2e x −
8 (1 + x )2
Price in 1997 = 105 + 105 × = 113.4 lim
100 x →0 2
2 +1
Price in 1998 = 113.4 + 113.4 ×
77
= 200.718 =
100 2
= 3/2 ans.
increment in price = 200.718 – 100 = 100.718 So option (b) is true.
then Average increment -
n2 −1 2
=
100.718
= 25.1795 = 26% (approx)
144. The series ∑ n2 −1
x , x > 0 is
4
(a) convergent for x ≥ 1
142. In the figure given below, ABCD is a square (b) convergent for x > 1
and DEC is an equilateral triangle. The points (c) divergent for x ≥ 1
A and E are joined by a straight line. (d) divergent for x > 1
Ans. (a) : convergent for x ≥ 1
145. Which statement among the following is
correct?
(a) If x < 0, then x2 > x
(b) If x2 > 0, then x > 0
The, ∠DAE equals (c) If x2 > x, then x < 0
(a) 15° (b) 10° (d) If x2 > x, then x < 0
(c) 30° (d) 20° Ans. (a) : If x < 0, then x2 > x
2
Ans. (a) : Here ABCD in a square and △DEC is an for example (–1) > (–1)
equilateral triangle. ⇒ 1 > (–1)

111
146. A car driver travels from a plane to hill station x − x 
which are 200 km apart at an average speed of  x x < 0

40 km/h. In the return trip, he covers the same f ( x ) =  
distance at an average speed of 20 km/hr. The  x + x x ≥ 0
average speed of his car in the entire distance  x 
of 400 kms is – L.H.L lim f (x)
x → 0−
(a) 25 km/h (b) 30 km/h
−h − | h |
(c) 24.33 km/h (d) 26.67 km/h lim f (0 − h) = lim
h →0 h →0 ( − h )
Ans. (d):
Distance −h − h −2h
We know speed = lim = lim
time h → 0 −h h → 0 −h
200 200 lim 2 = 2
Total time = + h →0
40 20 R.H.L.
= 5 + 10 lim f (x)
= 15 hr x → 0+
Total Distance h− | h |
Average speed = = lim f (0 + h) = lim
Total time h → 0 h → 0 h
200 + 200 h − h 0
= = lim = lim
h →0 h h →0 h
15
400 lim 0 = 0
= h →0
15 Clearly lim− f (x) ≠ lim+ f (x)
= 26.67 km / hr x →0 x →0
So lim f (x) does not exist
Hence option (d) is correct. x →0
147. Let f (x) = ax + bx + cx – 5, where a, b and c ∴ It is discontinuous at x = 0
7 3

are constants. If f (–7) = 7, then f (7) equals f(x) is continuous for all x except at x = 0
(a) –17 (b) –7 149. For the curve x2y2 = a2 (x2 + y2), the asymptotes
(c) 14 (d) 17 parallel to the coordinates axes are
Ans. (a) Here the given function (a) x = ± a, y = 0 (b) x = ± a, y = ± a
f(x) = ax7 + bx3 + cx – 5 (c) x = a, y = 0 (d) x = – a, y = a
putting –7 in the place of x as given
f(–7) = 7 Ans. (b) : x2 y2 = a2 (x2 + y2)
so, x2 (y2 –a2) – (y2 a2) = 0
f(–7) = a (–7)7 + b (–7)3 + c (–7) – 5 = 7
Highest
– 77a –73b – 7c – 5 = 7
power of x is x2
– (77a + 73b + 7c + 5 ) = 7
coefficient of x2 is y2 –a2
77a + 73b + 7c + 5 = –7
The asymptotes parallel to the x-axis are given by
77a + 73b + 7c = –12
y2 –a2 = 0
Now put
y=±a
f(7) = a(7)7 + b (73) + c (7) – 5
7 3 Highest power of y is y2 and its coefficient is x2–a2
f(7) = 7 a + 7 b + 7c – 5
The asymptotes parallel to the y-axis are given by
f(7) = –12–5
x2 –a2 = 0
f(7) = –17
Hence option (a) is correct. x=±a
Hence option (b) is correct.
x− x
148. Let the function f (x) defined as f (x) = , 150. For the curve f (x) = x3, x = 0 is
x (a) the global minimum point
then (b) the local minimum point
(a) the function is continuous everywhere (c) an extreme point
(b) the function is not continuous (d) a point of inflexion
(c) the function is continuous when x<0 Ans. (d) Curve f(x) = x3
(d) the function is continuous for all x except The curve f(x) have both negative and positive value
zero x = 0 is a point of inflexion for curve f(x) = x3
x− | x | [∵ A point of inflexion is a point of curve at which
Ans. (d) f (x) =
x curvature changes sign]

45th BPSC (Pre) Exam. 2002 112 YCT


46th Bihar Public Service Commission
Preliminary Examination, 2003-04
GENERAL KNOWLEDGE & GENERAL SCIENCE
(Solved Paper with Detail Explanation)
1. Chandragupta Maurya figures prominently in Ans. (b) : Chandragupta was the first Mauryan ruler.
the book of The Mauryan empire includes the present day state of
(a) Bhasa (b) Sudraka Bihar and good portion of Orissa and Bengal and
(c) Vishakhadatta (d) Ashvaghosha western India and the Deccan.
Ans. (c) : Chandragupta Maurya figures prominently in 7. In how many categories did Megasthenese
the book of Vishakhadatta. His book name is
"MudraraRakshasha". The Mudrarakshasa is a Sanskrit divided the Indian Society?
historical play by Vishakhadatta that tells the story of (a) Four (b) Five
king Chandragupta Maurya’s rise to power in India. (c) Six (d) Seven
2. The description of the administration of Ans. (d) : The division of Mauryan Society into Seven
Pataliputra is available in categories particularly mentioned in the Indica of
(a) Divyavadan Megasthenese. Megasthenese influenced by Herodotus’
(b) Arthashastra classification of the Egyptian society into seven classes
(c) Indica
(d) Ashoka's inscriptions based on the professional criterion, mentioned existence
Ans. (c) : The description of the administration of of seven Caste in India.
Pataliputra is available in the Book “Indica”. This book 8. The list of Sixteen Mahajanapadas is available
was written by Megasthenese who was sent to India as in
an ambassador in Pataliputra by Greek ruler Seleucus (a) Mahabharat
Nikator. This book has not survived but its fragments (b) Anguttara Nikaya
are preserved in later Greek and Latin work of (c) Chhandogya Upanishad
Diodurus. Megasthenes’Indica along with Kautilya’s (d) Samyukta Nikaya
Arthashastra provides valuable insight on the most Ans. (b) : According to Buddhist text, Anguttara
powerful and prominent dynasties of that time, the Nikaya (The land between Himalayas and Narmada)
Mauryan dynasty. was divided into 16 independent states known as
3. By which ruler, Pataliputra was chosen for the Mahajanapadas. It should be noted that the most
first time as a capital? powerful states in the 6th century were Magadha,
(a) Ajatashatru (b) Kalasoka Koshla, Vatsa and Avanti.
(c) Udayin (d) Kanishka 9. 'Sita' land in Arthashastra stands for
Ans. (c) : Udayin had firstly founded the new capital at (a) Uncultivable waste land
Patliputra (Modern Patna), situated at the confluence of (b) Land owned by the Brahmins
Ganga and Son rivers. Patliputra had very strategic (c) Land cultivated by tribes
location as it lay at the centre of the Magadha Kingdom. (d) Forest land
4. Language used in the inscriptions of Ashoka is Ans.(c) : Kautilya's Arthashastra mentions that Sita
(a) Sanskrit (b) Prakrit land was land owned by State and Slaves, paid
(c) Palli (d) Hindi labourers and tribes were employed to cultivate Sita
Ans. (b) : Most of Askoka's inscriptions are in dialects land.
of Prakrit. Which was the language of Political Power in 10. An advanced water management system of
subcontinent for several countries. There are total 33 Harappan
inscriptions among them there are 14 major Rock Edicts (a) Dholavira (b) Lothal
of Ashoka. (c) Kalibangan (d) Alamgirpur
5. Where was the third Buddhist Council held?
(a) Vasta (b) Pataliputra Ans. (a) : The remains of a highly advanced water
(c) Kaushambi (d) Kashmir management system of Indus Valley Civilization have
Ans. (b) : The third Buddhist Council was held in been discovered in Dholavira. A sophisticated water
Pataliputra during the reign of the emperor Ashoka in conservation system is one of the unique features of
250 B.C. It was held under the presidency of Dholavira. Giant water reservoir was also found there.
Moggaliputta-Tissa. The council is recognized and The Dholavira site was discovered in 1967-68 by Indian
known for the origin of Theravada school. archaeologist J.P. Joshi. It is located in Gujarat.
6. Who amongst the following Mauryan rulers 11. Where was the first Republican System of the
did conquer the Deccan? world in sixth century BC?
(a) Ashoka (b) Chandragupta (a) Vaishali (b) Athens
(c) Bindusara (d) Kunala (c) Sparta (d) Pataliputra
46th BPSC (Pre) Exam. 2003-04 113 YCT
Ans. (a) : Vaishali, is situated in Vaishali district in 17. The Muslim deputation meets Minto in 1906 at
Bihar, is believed to be the world’s first republic’s and Shimla and Pleaded for
is currently an archaeological site in India. Vaishali was (a) Separate electorate for Muslims
established in 6th century BCE as a Republic i.e. before (b) A composite electorate
the birth of Gautama Buddha thereby making it the (c) Higher representation to the Hindus
world’s first republic. (d) Special representation to Muslim by
12. Chinese traveller Hiuen-Tsang studied at the nomination
University of Ans. (a) : The Muslim deputation was a gathering of 35
(a) Taxila (b) Vikramshila prominent Indian Muslim leaders led by the Aga Khan
(c) Magadh (d) Nalanda III at Shimla in 1906. The deputation aimed to convince
Ans. (d) : Chinese traveller Hiuen-Tsang studied at the Lord Minto, the then viceroy of India to grant a separate
University of Nalanda. Nalanda University was the first electorate for Muslims and there was also a demand to
ever university of India which was founded in the 5th be given three more seat in the central legislature or to
century but got abandoned in 13th century. The grants Muslims greater representation in politics.
Vikramshila University was established at Bhagalpur in 18. The system of competitive examination for
Bihar. It was founded by the king Dharmpal. Civil services was accepted in principle in the
13. Who did illustrate "Dastan-I-Amir Hamza"? year
(a) Abdus Samad (b) Mansoor (a) 1833 (b) 1853
(c) Mir Sayeed Ali (d) Abul Hasan (c) 1958 (d) 1882
Ans. (a) : Abdus Samad had illustrated “Dastan-I-Amir Ans. (b) : The Charter Act of 1853 introduced open
Hamza", was the most renowned Persian painters. competition system for recruitment of Civil services. It
Humayun instructed Mir Sayeed Ali and Khawaja was opened for Indian the first time. Accordingly,
Abdus Samad to illustrate the Dastan-I-Amir Hamza, Macaulay committee (the committee on Indian Civil
the Renewed Persian book. service) was appointed in 1854. The Aitchison
14. "When he attained kingship, he was quite commission had recommended that the minimum and
independent of rules and orders of Shariat". maximum age limits to appear for exam were to be 19
For which Sultan AI-Baruni made this and 23.
statement? 19. Which is the painting school of Bihar?
(a) Iltutmish (a) Madhubani (b) Srirangam
(b) Balban (c) Pichhawai (d) Mughal
(c) Alauddin Khilji
(d) Mohammad Tughlaq Ans. (a) : Madhubani is the famous painting school of
Bihar. These paintings were usually created by women
Ans. (c) : Al-Baruni had written about the Alauddin on walls and floors of homes during festival,
Khilji that “when he attained the kingship, he was quite ceremonies or special occasions. Having originated in
independent of rules and orders of Shariat. Alauddin the Mithila region in Bihar, this form of painting, also
Khilji was Jalauddin’s ambitious nephew and son-in- known as Mithila art. It has been practice in areas
law. In 1294 A.D he had led the first Turkish expedition around Bihar and Nepal.
to southern India and plundered Devagiri.
15. In the Sultanate time, the highest rural 20. At the time of Transfer of Power, which one of
authority for land revenue was the following states possessed a well developed
(a) Chaudhary (b) Rawat Congress Party Organisation, which was
(c) Malik (d) Patwari (a) Hyderabad (b) Awadh
Ans. (a) : In the Sultanate time the highest Rural (c) Mysore (d) Junagadh
authority for land revenue was known as Chaudhary. Ans. (c) : Mysore has possessed a well developed
The revenue system of Delhi Sultanate was based on the Congress Party Organization at the time of Transfer of
theory of taxation as propounded by the Hanafi school Power. There was a common idea among the all
of thought which prescribed the levy of taxes i.e. Zakat, presidencies that they all accepted the sovereignty of
Kharaj, Khams and Jaziya. Land revenue was the main British rule-Mysore was the capital of Kingdom of
source of income during the sultanate period. Mysore until 1950, except in the 18th century when
16. In the Mughal adminstration "Madad-I-Mash" Haidar Ali and Tipu Sultan were the defacto rulers for a
indicates 40 year period.
(a) Toll Tax 21. The Congress in Travancore launched a Civil
(b) Revenue free land given to learned persons Disobedience Movement against the autocratic
(c) Pension to army persons Government of the Dewan of Travancore State,
(d) Cultivation tax who was
Ans. (b) : Sometimes, the Mughal administration (a) C P Ramaswami Iyar (b) J Krishnaswamy
issued grants as Madad-I-Mash to pious or otherwise (c) P K Warrier (d) Bhaskaran Nair
worthy recipients charity. The spirit of composite
culture and harmony expressed in revenue-free-grants Ans. (a) : Under the leadership State Congress, the
(Madad-I-Mash) which the nobles extended to the people of Travancore started an agitation for a
support of saints maintenance of religious institution, responsible government. C.P. Ramaswami Iyar, the
religious place and for Mashaiksh. Such type of land Dewan of Travancore, decided to suppress the agitation.
become generally unfertile. Which were given to On 26th August 1938 he banned the state congress
learned person. which then organized a civil disobedience against him.
46th BPSC (Pre) Exam. 2003-04 114 YCT
22. The 1937 session of the Indian National 28. The spread of Socialist ideology in Bihar
Congress was held in Bihar at during pre-independence period can be traced
(a) Haripura (b) Patna in
(c) Ranchi (d) Bhagalpur (a) Western education of Jai Prakash Narayan
Ans. (*) : The Indian National Congress session of and B P Sinha
1937 was held at Faizpur in Maharastra. This session (b) International Socialist Movement
was presided over by Jawahar Lal Nehru. It was the first (c) Radical Congressmen
session to be held in a village and the session of 1938 of (d) Labour Party in Britain
Indian National Congress was held at Haripura in Ans. (c) : The origins of the Socialist Party in Bihar can
Gujarat. This session was presided over by Subhas be traced back to the mass movement stage of the some
Chandra Bose. young men of the Indian National movement in the pre-
23. In pre-independent Bihar for the large independence era. The Congress Socialist Party (CSP)
landlords, Political power was a mean to was formed within the Congress in 1934 by a group of
protect their Radical young leaders who wanted a more radical
(a) States resources (b) Social status egalitarian congress. Jayaprakash Narayan, Achyut
(c) Economic power (d) Culture Patwardhan, Ashoka Mehta, Acharya Narendra Dev,
Ans. (c) : In pre-independent Bihar for the large land Ram Manohar Lohia and S.M. Joshi were the prominent
lord political power was a mean to protect their leaders among the leaders of Socialist Party.
economic power. 29. Bengal was Partitioned in 1905 AD following
24. During the 1930's, in which one of the following the resulting protest it was repartitioned in
towns, the Civil Disobedience movement of the (a) 1906 AD (b) 1916 AD
congress was so successful that for a few (c) 1911 AD (d) 1909 AD
months, all trace of British Raj seemed to have Ans. (c) : The Bengal was Partioned in 1905. It was
disappeared? Partioned by then Viceroy “Lord Curzon, the
(a) Patna (b) Sahibganj Partitioned was announced on 20th July 1905 and came
(c) Barhee (d) Muzaffarpur into effect on 16th October, 1905. But the Partition of
Ans. (c) : During the Civil Disobedience movement (in Bengal was revoked in 1911 on the occasion of arrival
year 1930) the control of British Raj ended in Barhee as of king George-V. A new scheme of Partition of Bengal
agitators captured the Eastern India and was the main was implemented on linguistic rather than on religious
site of the campaign for non-payment of taxes. People grounds, Hindi, Oriya, and Assamese speaking areas
were refused to pay for non-payment of taxes. were separate administrative units. The administrative
25. Which one of the following was not a dominant capital of British India shifted from Kolkata to Delhi in
caste in Bihar during the pre-independent 1911.
period?
(a) Bhumihar (b) Rajputas 30. One of the following sought to have the
(c) Kayasthas (d) Kurmi provisions of the wartime Defence of India Act,
used against the Gadar Movement in Punjab in
Ans. (d) : Bhumihar, Rajputas and Kayasthas were 1915, maintained in peace time also
dominated caste in Bihar before independence but (a) Lord Carmichaeal
Kurmi caste was not a dominant casts in Bihar during (b) Sir Reginald Craddock
the pre independence period.
(c) Lord Hardinge
26. The absence of relative national activity in (d) William Bentinck
Bihar before Mahatma Gandhi can be traced
to the lack of Ans. (c) : Lord Hardinge was sought to have the
(a) Union in community provision of the wartimes Defence of India Act, used
(b) Militant group against the Gadar Movement in Punjab in 1915, had
(c) Creative regional elite maintained in the peace time also.
(d) Dominant middle class The Defence of India Act. also referred to as the
Ans. (a) : Prior to Mahatma Gandhi, the absence of Defence of India Regulation Act, was an emergency
relative national activity in Bihar was seen as the lack of criminal law enacted in 1915 by the Governor. General
Union in community. Mahatma Gandhi went to Bihar at of India. The Main intention was to curtail the
the behest of Rajkumar Shukla and started the first nationalist and revolutionary activities of the Indian
Satyagraha on the land of Bihar in India. independence movement during and in the aftermath of
World War-I.
27. The Bihar Provincial Kisan Sabha was formed
in 31. Agrarian classes in Bihar during British rule
(a) Feb, 1931 (b) Jan, 1933 were placed in which one of the following
(c) March, 1929 (d) April, 1932 categories
Ans. (c) : Sahajanand Saraswati formed the Bihar (a) Five-fold division (b) Four-fold division
Provincial Kisan Sabha (BPKS) in March 1929 in order (c) Three-fold division (d) No division
to mobilize peasant grievances against the Zamindari Ans. (a) : Agrarian classes in Bihar during British rule
attacks on their occupancy rights and thus sparking the were placed in the four fold or categories.
farmer’s movement in India. Sahajanand Saraswati who 32. The massacre of the crowd at Jallianwala Bagh
was the pioneer of Bihar Kisan Sabha movement and at Amritsar took place on
founder of Bihar Provincial Kisan Sabha (BPKS) was (a) 1st June, 1918 (b) 13th April, 1919
the first president of All India Kisan Sabha. (c) 14th April, 1920 (d) 6th July, 1921
46th BPSC (Pre) Exam. 2003-04 115 YCT
Ans. (b): The Jallianwala Bagh massacre, also known 37. Mahatma Gandhi's close English companion
as the Amritsar massacre took place on 13th April, 1919. during the Freedom Movement was
A large but peaceful crowd had gathered at the (a) Reverend Thomas Moore
Jallianwala Bagh in Amritsar, Punjab to protest against (b) A O Hume
the arrest of pro-Indian independence leaders Dr. (c) Reverend Charles Andrew
Saifuddin Kitchlew and Dr. Satya Pal. After blocking (d) William Wavell
the exit with his troops in Jallianwala Bagh, the British Ans. (c) : Reverend Charles Andrew was the close
Brigadier-General R.E.H. Dyer surrounded the bagh English companion of Mahatma Gandhi during the
with his soldier and ordered them to shot at the crowd. Freedom Movement of 1942 Quit India Movement. He
33. The Second Round Table Conference at is also known by the nicknames of Deanbandhu.
London was held in the backdrop of the 38. Mahatma Gandhi held that Participation in the
(a) Emerson-Gandhi Pact Second World War meant a violation of the
(b) Hailey-Gandhi Pact principle of
(c) Irwin-Gandhi Pact (a) Justice (b) Equity
(d) Gandhi-Simon Pact (c) Socialism (d) Non-violence
Ans. (c) : The Second Round Table Conference was Ans. (d) : Truth and non-violence were the two basic
held in London from September 7, 1931 to December 1, principles of Mahatma Gandhi, Following the principle
1931. Congress boycotted 1st round table conference, to of non-violence, he withdrew the non-cooperation
ensure participation of congress the then viceroy Irwin movement after being hurt by the Chauri-Chaura
signed Gandhi Irwin pact which paved the way for incident on 5th Feb, 1922. During the freedom
participation of congress in 2nd Round Table
movement and in his personal life, he followed the
Conference.
principle of duality like truth and non-violence
34. The first Indian to hold office as Governor- throughout his life. He know very well that after
General of independent India was participation in world war-II, there will be a violation of
(a) Jamnalal Bajaj (b) C Rajagopalachari the Principle of Non-violence.
(c) Dr, Rajendra Prasad (d) M A Ansari
39. The Chairman of the Joint Parliamentary
Ans. (b) : The first Indian Governor General of
Independent India was C. Rajagopalachari. He was Committee of the 1935 bill that led to the
popularly known as Rajaji, he was also the last framing of the Government of India Act of
Governor General of India. Lord Mountbatten was the 1935 was
last viceroy and was also the first Governor General of (a) Lord Linlithgow (b) James Mcdonald
Independent India. (c) Winston Churchill (d) Clement Attlee
35. At the Congress Working Committee meeting Ans. (a) : The Government of India Act was passed by
at Allahabad (April- May, 1947), who the the British parliament in 1935 and came into effect in
following members was able to secure a 1937. It was based on a report by a joint select in
majority for his plan of a guerilla war against committee led by Lord Linlithgow, set up the two
the Japanese due to India's unequivocal stand houses of the British parliament. The report in turn was
against Nazism, Fascism and Imperialism? the result of the joint committees’ scrutiny of the
(a) Subhash Chandra Bose
(b) Sarojini Naidu “White Paper” – a scheme of constitutional proposals
(c) Maulana Azad prepared by the British government close on the heels
(d) Jawaharlal Nehru of the round table conference.
Ans. (d) : Jawaharlal Nehru was able had became able 40. The Poona Pact aimed
to secure a majority for his plan of a guerilla war (a) To Hindu-Muslim Unity
against the Japanese due to India’s unequivocal stand (b) To represent to lower caste
against Nazism, Fascism and Imperialism at the (c) To privileges to princes
Congress Working Committee meeting at Allahabad (d) To review the Diarchy
(April-May, 1947).
Ans. (b) : The Poona Pact was an agreement between
36. In 1932 with the Passage of one of the following M.K. Gandhi and B.R. Ambedakar signed in the
orders, the All India Congress Committee was
declared unlawful. Yerwada Central Jail, Poona on September 24th 1932 on
(a) Unlawful Association Ordinance behalf of the depressed class for reservation on the
(b) Violation of Peace Law electoral seats in the legislature of the British
(c) Executive Power Ordinance government but Gandhi was against this idea. Because
(d) Viceroy's Special Order he did not want to view the untouchable as being
Ans. (a) : In 1932, with the passage of Unlawful outsides the folds of Hinduism.
association ordinance, the Britishers declared unlawful 41. The objective of Butler Committee of 1927 was
organization to the All India Congress Committee. On to
January, 1932, under the unlawful association ordinance the
Congress and its allied organsation were declared illegal (a) Modernise the Indian Military
and their buildings, offices and property were occupied. (b) Modernise the Indian agriculture
Sticks and bullets were fired at the protesters. Despite the (c) Impose censorship on National press
repressive action, the movement continued through boycott (d) Improve the relationship between
of foreign goods and ceased to pay taxation. Government of India and Indian states
46th BPSC (Pre) Exam. 2003-04 116 YCT
Ans. (d): Butler Committee was founded under the 46. Who is not entitled to take part in the activities
chairmanship of Sir Harcourt Butler in 1927. It was of Lok Sabha?
aimed to investigate and clarify the relationship (a) The Comptroller and Auditor General of
between Government of India and the India Princely India
States and suggested the ways and means for more (b) The Attorney General of India
satisfactory adjustment of existing relations between (c) The Solicitor General of India
them and the British India. (d) The Secretary to President of India
42. The cause of decline of textile industries in 18th Ans. (d) : The Secretary to the President of India can
century in Bengal was not take part in the proceedings of the Lok Sabha,
(a) Decline in quality of production whereas the other three given in the option can take part
(b) Non-availability of raw material in the proceedings of the Lok Sabha. The Comptroller
(c) High tariff rates on exports to Britain and Auditor General of India (Art. 148) is appointed by
(d) Non-availability of craftsmen the President and enjoy his office till the pleasure of
Ans. (c) : The cause of decline of textile industries in President.
18th century in Bengal was high tariff rates on exports to 47. In which Amendment, the words 'Social
Britain. There were some others reason which were Secular' and 'Unity and Integrity of the Nation'
responsible for the decline as follows: were added to the Constitution of India?
(i) Imposition of free trade system in India. (a) 42nd (b) 44nd
(ii) Export of raw materials from India. (c) 52nd (d) None of these
(iii) To Compete with cotton goods from Lancashire Ans. (a) : By the 42nd Constitutional Amendment Act of
and Manchester. 1976, the words Socialism, Secularism, and Integrity
43. Who was the Congress President at the time of were added to the Constitution of India.
transfer to power in 1947 to India by the 48. Which Lok Sabha Election was held in four
British? phases?
(a) Jawaharlal Nehru (b) Rajendra Prasad (a) 11th (b) 12th
(c) Abul Kalam Azad (d) J B Kriplani (c) 13th (d) 14th
Ans. (d) : On 1947, J.B. Kriplani presided over the Ans. (d) : Election for the 14th Lok Sabha Election were
Meerut session in 1947. He was one of the most ardent held in four phase; 20-21, 26 April and were completed
disciples of Mahatma Gandhi and was president of INC on 5th May 2004. On 22nd May Dr. Manmohan Singh
during the transfer of power from Britain to India in
1947. In the election of the Congress President in 1950, had sworn as the 14th Prime Minister of the country.
Kriplani was defeated by Purushottam Das Tandon. Currently Narendra Modi is the 15th Prime Minister of
44. Who used the word 'Sarvodaya' for the first India in the order of person.
time? 49. Whose pre-consent is essential for Finance bill?
(a) Mahatma Gandhi (a) The Prime Minister
(b) Pandit Nehru (b) The Finance Minister
(c) Vinoba Bhave (c) The President of India
(d) Jai Prakash Narayan (d) None of the above
Ans. (a) : The word Sarvodaya was first coined by Ans. (c) : Indian Parliament is composed of Lok Sabha,
Gandhi to communicate his philosophy of service to all
humankind. He also translated the John Ruskin’s book Rajya Sabha and the President. Every bill passed by two
“Unto the Last” under the same title because this book Houses becomes a law only after assent of President
had influenced his thinking and gradually the term and hence, President is an integral part of Parliament.
Sarvodaya became associated with his movement. However there are certain bills that require prior
45. Which principle was propounded by Nehruji recommendation of President before introduction in
for establishing World Peace and Mutual Co- Parliament these includes:–
operation? Money bill under Article 110 and Financial bills under
(a) Non-alignment Article 117(1). Under Article 274(1), Prior
(b) Panchsheel recommendation of President required to Bills affecting
(c) Peaceful Co-existence taxation in which States are interested.
(d) None of these 50. Which is the House where the Chairperson is
Ans. (b) : India gives special importance to the peaceful not the member of that house?
coexistence. In such sequence, Pt. Jahawarlal Nehru (a) Lok Sabha
propounded the Panchsheel principle in June 1954. (b) Rajya Sabha
Which were as follows : (c) Legislative Assembly
I. Territorial integrity to each other territorial integrity (d) Legislative Council
and Sovereignty.
II. Assurance of mutual non-aggression. Ans. (c) : The Vice-President of India is the ex-officio
III. Mutual non-interference in each other’s internal chairman of Rajya Sabha. The chairman of Rajya Sabha
affairs. is not the member of the house. The maximum strength
IV. Equality and mutual benefits. of Rajya Sabha is 250 of which 238 are to be elected
V. Peaceful co-existence and 12 are to be nominated by the President of India.

46th BPSC (Pre) Exam. 2003-04 117 YCT


51. With which the 73rd Amendment of the 56. Consider the following statements
Constitution is related? Assertion(A): Of the fourteen Public sector
(a) Municipal Corporation refineries of Indian seven are shorebased.
Reason(R) : Large quantities of crude petroleum are
(b) House Rent Act available from onshore and offshore oil fields.
(c) Panchayti Raj Act (a) Both A and R are true and R is correct
(d) Increase in Salary and DA of MPs explanation of A
Ans. (c) : The 73rd Constitutional Amendment Act. (b) Both A and R are true but R is not correct
1992 deals with Panchayati Raj System. explanation of A
The 73rd amendment envisages the Gram Sabha as the (c) A is true but R is False
establish of the Panchayati Raj system which carries out (d) A is False but R is true
the function and powers assigned by the state Ans. (a) : At present the number of oil refineries in the
public sector is 18. Whereas there are 2 other joint
legislature. The 73rd amendment of 1992 added a new
venture refineries at Bina and Bathinda and there are
part IX to the constitution tilled “Panchayat” which also 3 refineries in private sector. Most of the public
includes provisions from Art 243 to 243 (0) and new sector oil refineries are located on the shores therefore
Eleventh schedule covering 29 subjects within the most of oil production comes from these on shore as
function of Panchayat. well as offshore oil fields. Hence both assertion and
52. In Indian Constitution, Article 370 is related reason are true. The reason explains the correct fact for
with which? given assertion.
(a) Jammu and Kashmir State 57. Which one of the following would you not
(b) Reservation of Scheduled Castes and include in the physical divisions of Bihar?
(a) Chhota Nagpur Plateau
Scheduled Tribes
(b) North Ganga Plain
(c) Punjab and Haryana States (c) South Ganga Plain
(d) Karnataka and Tamil Nadu States (d) East Ganga Plain
Ans. (a) : Article 370 of the constitution deals with the Ans. (d) : There are two major physical division of
state of Jammu and Kashmir. It is known that on 5th Bihar as – Gangatic Plain and Chotanagpur region and
August, 2019, through Jammu and Kashmir State the Gangetic plain is further divided into two Part – The
Reorganization Bill Passed by the parliament, the Northern Gangetic Plain and the southern Gangetic
special status was abolished as well as the Jammu and plain. East Ganga plain is the part of West Bengal.
Kashmir and Ladakh was given the status of a Union 58. In Bihar, the rivers belonging to which one of
Territory. the following groups join the Ganga or its
tributaries flowing from the North?
53. Who among the following Presidents held office (a) Ghaghara, Kosi, Punpun
for two consecutive terms? (b) Kosi, Mahananda, Karmnasha
(a) Dr. Rajendra Prasad (c) Gandak, Kamla, Baghmati
(b) Dr. S Radhakrishnan (d) Sone, Kosi. North Koel
(c) Dr. Zakir Hussain
(d) (a) and (b) Ans. (c) : The Ghaghara (Saryu), Gandak, Budhi
Gandak, Baghmati, Kamala and Mahananda, Kosi
Ans. (a) : Dr. Rajendra Prasad was the first President of rivers join the Ganges from north which also flows
India. He held office for two consecutive term from through Bihar.
1952 to 1962. He was elected as President of India in
1952 and 1957. 59. Consider the following Statements regarding
the vivid geographical aspects of Bihar.
54. What is the sequence of Dr. Manmohan Singh I. Of the total irrigated land in Bihar nearly
as Prime Minister of India? 37% is irrigated by canals.
(a) 7th (b) 11th
II. The North Ganga plain in Bihar extends
(c) 14th (d) 16th
over an area of 33620 km2.
Ans. (c) : Prime Minister of India is the head of the III. Kishanganj and West Champaran receive
central government. He is appointed by the President. the highest annual rainfall in Bihar.
Dr. Manmohan Singh was the 14th Prime Minister of IV. The total length of National Highways in
the country in order of person. His term was from 22nd Bihar is 2,461 km.
May, 2004 to 26th May, 2014. Which of these statements are correct?
55. Who is the Chief Election Commissioner of (a) I and II (b) II and III
India at present? (c) II and IV (d) I, III and IV
(a) N. Gopalaswamy (b) BB Tandon Ans. (b): Bihar nearly 23% of the total irrigated land in
(c) TS Krishnamurthy (d) TN Sheshan irregated by canals. The Northern Gangetic plain in
Ans. (c) : When the question was asked, T.S. Bihar is extended over 56980 sq. km. area. Which is
Krishnamurthy was the Chief Election Commissioner of spread over 32.37% area of the state and the Southern
India. The Chief Election Commissioner is appointed by Gantetic plain is spread over 33620 sq. km area. When
the President. The first Chief Election Commissioner the question was asked the total length of National
was Sukumar Sen and the present Chief Election highway in Bihar was 2461 km. Hence, options I, II and
Commissioner is Rajiv Kumar. IV are correct.

46th BPSC (Pre) Exam. 2003-04 118 YCT


60. In Bihar, Bangar soil occurs in (b) East Champaran, Patna, Madhubani,
(a) Gaya-Bodh Gaya-East Champaran-Bhagalpur Muzaffarpur
area (c) Patna, Muzaffarpur, East Champaran,
(b) Purnea-Saharasa Darbhanga- Muzaffarpur Madhubani
area (d) Muzaffarpur, Patna, Madhubani, East
(c) Gaya-Nalanda-Both Gaya-Sasaram area Champaran
(d) East Champaran-Nalanda-Gaya-Sasaram area Ans. (a) : The correct decending order of population of
Ans. (b) : The extension of Bangar soil in Bihar is the districts of Bihar, as per census 2001are as follows-
found to the north-western region of Purnea, Saharasha. District Population
Darbhanga, Muzaffarpur and Champaran district. This Patna - 4718592
soil is older alluvium, and is a major categories of Soil East Champaran - 3939773
in Gangetic plain. Muzaffarpur - 3746614
61. Correlate List I (Industries) with List II Madhubani - 3575281
(Cities) in Bihar and select the correct answer 64. When was Jharkhand separated from Bihar?
using the codes given below the lists. (a) 15th November, 2000 (b) 25th August, 1999
(c) 2nd August, 1998 (d) 5th July, 1997
List-I (Industries) List-II(Cities) Ans. (a) : Jharkhand was carved out from the southern
A. Paper and Pulp 1. Dalmianagar part of Bihar on 15th November, 2000 by a law named
B. Cigarette 2. Dilawarpur “Bihar Reorganization Act, 2000”. This law was passed
C. Plywood 3. Hajipur by the parliament of India in 2000. It was introduced by
D. Jute 4. Purnea the NDA government under the leadership of Atal
Codes Bihari Vajapayee. The whole state is almost plateau
A B C D region as borders its boundary with Bihar, West Bengal,
(a) 3 4 1 2 Odisha, Chattisgarh and Uttar Pradesh.
(b) 1 2 3 4 65. How many universities does Bihar have?
(c) 2 1 4 3 (a) 9 (b) 11
(d) 4 3 2 1 (c) 12 (d) 13
Ans. (b) : The Correct matching of the above is as Ans. (c) : When the question was asked the total
follows :- number of universities in Bihar were 12. While at
List I List II present there are total 24 Universities in Bihar out of
(Industries) (Cities) which 22 are traditional Universities. One is Open
Paper and Pulp - Dalmianagar (Sasaram) University and one Private University.
Cigarette - Dilawarpur (Munger) 66. Who is known as 'Lok Nayak'?
Plywood - Hajipur (a) Mahatma Gandhi
Jute - Purnea (b) Subhash Chandra Bose
62. Corelate List I (Minerals) with List II (Places (c) Jai Prakash Narayan
of Occurrence) in Bihar and select the correct (d) Bal Gangadhar Tilak
answer using the codes given below the lists. Ans. (c) : Jai Prakash Narayan is also known as Lok
List-I List-II Nayak, born in Bihar. Jai Prakash Narayan was one of
(Minerals) (Places of Occurrence) the prominent leaders of Indian National Movement and
A. Mica 1. Munger was a member of the Indian National Congress. He had
B. Gold 2. Gaya established the Socialist Congress Party in Bihar in the
C. Dolomite 3. Kishanganj year of 1934. He had participated in the fight for
D. Petroleum 4. Rohtas freedom. In the year 1942, He led the Quit India
Codes Movement in the absence of big leaders.
A B C D 67. Who is the Governor of Bihar at present?
(a) 3 4 1 2 (a) Shri Nawal Kishore Sharma
(b) 1 2 3 4 (b) Justice M Rama Jois
(c) 2 1 4 3 (c) Shri Madan Lal Khurana
(d) 4 3 2 1 (d) Justice Anshuman Singh
Ans. (c) : The correct matching of the above is as Ans. (b) : When the question was asked Justice M.
follows :- Rama was the Governor of Bihar while the current
List I List II governor of Bihar is Phagu Chauhan.
Mica - Gaya 68. What is the population of Bihar as per 2001
Gold - Karmalia village of Munger district census?
Dolomite - Bajari village of Rohtas District (a) 85 Million (b) 86 Million
Petroleum - Kishanganj (c) 83 Million (d) 82 Million
63. Which one of the following groups of districts Ans. (c) : The total population of Bihar as per census
in Bihar indicate the correct descending order 2001 was about 83 Million. In which male populations
of population as per 2001 census? were 43, 243, 795 while females were 39, 754, 714. The
(a) Patna, East Champaran. Muzaffarpur, total population growth in this decade was 25.42
Madhubani percent.
46th BPSC (Pre) Exam. 2003-04 119 YCT
69. How Many members are there in Bihar 75. When was Indian Parliament attacked by
Legislative Assembly at present? terrorists?
(a) 240 (b) 243 (a) 24th December, 1999
(c) 245 (d) 250 (b) 13th December, 2001
Ans. (b) : The number of members in the Bihar (c) 14th January, 2002
Legislative Assembly is 243 and the numbers of (d) 12th January, 2002
members in the Legislative Council is 75. It is Ans. (b) : On December 13 in 2001 the Indian
noteworthy that from Bihar 40 members are elected for Parliament was attacked by the terrorists. Two Pakistan-
the Lok Sabha and 16 for the Rajya Sabha. based groups, namely Lashkar-e-Taiba (LeT) and Jaish-
e-Mohammed (JeM), were responsible for this horrific
70. Who has written 'Jhanda Geet'? attack.
(a) Ravindranath Tagore
76. Which one of the following groups of Indian
(b) Bandim Chandra Chattarjee States indicates the correct decending order of
(c) Maithilisharan Gupta population density as per 2001 census?
(d) Shyamlal Gupta (a) Punjab, Maharashtra, Tamil Nadu, Haryana
Ans. (d) : The famous song “Jhanda Geet” was written (b) Tamil Nadu, Punjab, Maharashtra, Haryana
by Shyamlal Gupta. He wrote this song in 1924. The (c) Punjab, Tamil Nadu, Haryana, Maharashtra
National Anthem is written by Rabindranath Tagore and (d) Haryana, Punjab, Maharashtra, Tamil Nadu
was first sung by Indian National Congress in its Ans. (c) : According to census 2001, the correct
Calcutta session held on 27 Dec. 1911. sequence of states in descending order of population
71. Who said 'Swaraj is my birthright and I shall density is as follows- Punjab (484 person/sq. km.)
have it'? Tamil Nadu (480 person/ sq.
(a) Mahatma Gandhi km.)
(b) Bipin Chandra Pal Haryana (478 person/ sq. km.)
(c) Gopal Krishna Gakhale Maharashtra (315 person/sq.km.)
(d) Bal Gangadhar Tilak It is noteworthy that West Bengal has the maximum
population density as per census, 2001 among all states
Ans. (d) : In December, 1916 the Indian National followed by Bihar. (881).
Congress session was held in Lucknow under the
chairmanship of Ambica Charan. In this session Bal 77. Consider the following statements about the
agri-trade of India during 2001-02 and 2002-
Gangadhar Tilak gave the slogan “Swaraj is our birth 03.
right and I shall have it". I. Agri-imports contributed nearly 12% of the
72. How many women Chief Ministers are there in total imports of India.
India at present? II. Edible oils contributed nearly 62% of the
(a) 5 (b) 3 value of total agri-imports of India.
(c) 7 (d) 4 III. Agri-exports contributed nearly 4% of the
Ans. (d) : When the question was asked, there were total exports of India.
four state in India where the Chief Minister were IV. Marine products contributed nearly 23%
female. Whereas since 2019, the West Bengal Chief of the value of total agro-exports of India.
Minister Mamata Benerjee is the only female Chief Which of these statements are correct?
Minister in the country. (a) II and IV (b) I and II
(c) II and III (d) I, III and IV
73. Which incident led Gandhiji to withdraw Non-
Ans. (a) : Presently (2018-19) the contribution of
Cooperation Movement? service sector in total exports is 38.4% while India’s
(a) Kakori incident share of global agricultural export is 2.2 percent. As per
(b) Chauri-Chaura incident the latest figures for the year 2017-18, the share of
(c) Jallianwala Bagh incident marine products in the total exports is 2.7 percent.
(d) Muzaffarpur incident 78. India is located between
Ans. (b) : The Non-cooperation movement was called (a) Lat. 8° 4' S to 37°6' N and Long. 68° 7' W to
off by Gandhiji after the incident of Chauri Chaura in 97° 25' E
Uttar Pradesh. On 5th Feb 1922, the Protest became (b) Lat. 8° 4' S to 37°6' S and Long. 68° 7' E to
violent when protesting people caught fire a police 97° 25' W
station in which 23 policemen trapped inside and burnt (c) Lat. 8° 4' N to 37°6' N and Long. 68° 7' E to
alive. He felt the movement was turning violent in many 97° 25' E
places and now felt that Satyagrah is needed to be (d) Lat. 8° 4' S to 37°6' S and Long. 68° 7' W to
trained properly for Mass Movement. 97° 25' W
74. How many women parliamentarians are there Ans. (c) : India is located latitudinally in the Northern
in fourteenth Lok Sabha? hemisphere and longitudinally in the Eastern
(a) 48 (b) 52 hemisphere. It’s latitudinal extent is from 8o45’N to
(c) 43 (d) 56 37o6N latitudes and longitudinal extent is from 68o7’E
to 97o25’E longitude. The shape of India is not totally
Ans. (b) : In 14th Loksabha election a total of 52 triangular but Quadrangular. It measures 2933 km from
women were elected to the parliament. Where as in the east to west and 3214 km from North to South. Its total
current 17th Loksabha election (2019). A total of 78 land area is 32, 87, 263 sq. km. It has a land frontier of
women MPs have been elected. 15200 km and coastline of 7516.6 km.

46th BPSC (Pre) Exam. 2003-04 120 YCT


79. The Trans-Himalayan rivers are Ans. (a) : Water has acquired as a potential in the
(a) Sutlej, Sindhu, Ganga power sources at present that is why hydroelectricity is
(b) Branmaputra, Sutlej, Sindhu considered as the white coal. Asia and Africa have the
(c) Brahmaputra, Sindhu, Ganga highest potential of hydropower, Initially most of the
(d) Sutlej, Brahmaputra, Yamuna development of hydropower has taken place in North
Ans. (b) : The Trans-Himalay is situated to the north of America and Europe. But today China has the highest
the great Himalaya. It has Karakoram, Ladakh, Zaskar potential of hydroelectricity which has been constructed
and Kailash Mountain ranges. There are several rivers on any river. There are some greatest worlds’ potential
power river basins are :– Cango, Colombia, Yangtze
originating from the Trans- Himalaya as –
etc. The potential hydropower of Congo river is in
Brahmmputra, Sultej and Sindhu (Indus) etc.
African continent.
80. Approximately how many million years ago 83. In which one of the following principal
were the great fold mountains of the world agglomerations do we find a beehive
formed? concentration of two-thirds of the World's
(a) 400 (b) 320 population?
(c) 220 (d) 30 (a) North-East USA (b) Central Europe
(c) South and East Asia (d) North-West Europe
Ans. (d) : The world’s great fold mountains were
Ans. (c) : Two third population of the world’s total
formed about thirty million year ago. The fold population lives in the south and south east Asia. The
mountain are formed in such a shallow but long and countries of south Asia are India. Pakistan, Bangladesh
narrow sea by the sedimentary deposition. It continues Srilanka, Nepal, Afghanistan etc. and Myanmar,
till the formation of mountains. The formation of fold Malasia, Indonesia and Vietnam etc, come under the
mountain is a slow process which continues for millions East Asian countries. These two covers almost two-third
population of the world’s total population.
of years. A Huge mountain like Himalaya which is also
84. The World's greatest gold fields are located in
an example of folded mountain of world. There are which one of the following mountain ranges of
several fold mountains, For example – The Himalayas, South Africa?
The Andes, The Rockies, The Pyrenees, The Alps and (a) Roggeveldberg (b) Groot Swarberg
The Elburs etc. (c) Witwatersrand (d) Drakensberg
81. 80% of the world's active volcanoes exist in Ans. (c) : Witwatersrand, also called the ‘Rand’ ridge
of gold-bearing rock mostly in Gauteng province, South
which of the following Belts?
Africa. Its name means “Ridge of white water”. Its rich
I. Atlantic Ocean Belt gold deposits occurring in conglomerate beds known as
II. Mid-continental Belt reefs were discovered in 1886.
III. Circum-Pacific Belt 85. Which is not the Economic Infrastructure?
IV. Indian Ocean Belt (a) Roads
Codes (b) Railways
(a) II and III (b) I and II (c) Education and Health
(d) Power
(c) III and IV (d) I and III
Ans. (c) : Infrastructure is the general term for the
Ans. (a) : 80% of the world's active volcanoes exist in basic physical systems of a business, region, or nation.
Gram Pacific Belt and Mid-central Belt. The Circum- Infrastructure is generally understood as the basic
Pacific Belt also referred to ‘Ring of Fire’ is a path building block required for an economy to function
along pacific ocean characteristic by active volcanoes efficiently.
and frequent earthquakes. The majority of earth’s The infrastructure sector primarily comprises of
volcanic eruption and earthquake take place along the electricity, roads, telecommunication, railways,
Ring of Fire. The Ring of Fire is stretched to irrigation, water supply and sanitation, ports and
airports, storing facilities and oil and gas pipelines. But
approximately 40000 km (24,900 mile) tracing
education & health is not considered under Economic
boundaries between several tectonic plates including the Infrastructure.
pacific, Juan de Fuca, Cocos, Indian- Australian, Nazca, 86. Which is not included in Social Infrastructure?
North-American and Philippine plates. The Ring of Fire (a) Education
is home to more than 450 active and dormant volcanoes. (b) Health
(75% Earth’s total volcanoes) forming a semicircles (c) Telecommunication
around the rim of Pacific Ocean. (d) Housing
82. Some of the world's greatest potential water- Ans. (c) : Social Infrastructure are services that are
power river basins are established to improve the quality of the state of living
of the people. Social infrastructure contributes
(a) Congo, Columbia, Yangtze indirectly to the economic growth. Healthcare,
(b) Congo, Amazon, Narmada education and housing are the component of social
(c) Colorado, Narmada, St. Lawrence infrastructure whereas telecommunication come under
(d) Nile, Amazon, Ob economic Infrastructure.
46th BPSC (Pre) Exam. 2003-04 121 YCT
87. What are the Core Industries in India? List-I List-II
(a) Electricity (b) Coal (Forest Types ) (Regions)
(c) Petroleum Products (d) All of these A. Tropical Ever- 1. Chhotanagpur
green Plateau, Shiwaliks
Ans. (d) : The main or the key industries constitute the
B. Monsoon 2. Shillong, Plateau
core sector of an economy is known as core industries. Sahyadris
In India, there are eight sectors that are considered as C. Temperate 3. West Bengal, Terai
the core sectors. The eight core sectors of the Indian D. Mangrove 4. Eastern and Western
economy are – Electricity, Steel, Refinery Products, Himalayas, Nilgiris
Crude oil, Coal, Cement, Natural gas and Fertilizer. Codes
These sectors have a major impact on the Indian A B C D
economy and significantly affect most to the other (a) 4 2 3 1
industries as well. (b) 1 3 2 4
(c) 2 1 4 3
88. Copper-gold-iron-coal in India are connected (d) 3 4 1 2
with which one of the following sets of places? Ans. (c) : The correct matches are as follows:-
(a) Khetri-Kolar-Kudremukh-Jharia List-I (Forest Types ) List-II (Regions)
(b) Kolar-Khetri-Kudremukh-Jharia Tropical evergreen - Shillong Plateau, Sahyadris
(c) Jharia-Kolar-Kudremukh-Khetri forest
(d) Khetri-Kudremukh-Kolar-Jharia Monsoon forest - Chhotangpur Plateau, Shiwalik
Temperate forest - East & West Himalaya and
Ans. (a) : The region which is most known for their Nilgiris
production as follows : Kutch (Mangrove) - West Bengal, Tarai Region.
Production Region forest
Copper - Khetri 91. What is the Tenth Plan Period?
Gold - Kolar (a) Year 2002 to 2007
Iron - Kudremukh (b) Year 2000 to 2005
Jharia - Coal (c) Year 1999 to 2003
(d) None of these
89. Consider the following statements about
Ans. (a) : The Tenth five year plan period was from
India's fishing resources 2002 to 2007. The tenth plan was launched by Atal
I. India is the sixth inland largest producer of Bihari Vajpayee Government on December 21, 2002.
fish in the world and second in inland fish The economy accelerated in the tenth plan period
production. (2002-2007) to record an average growth rate of 7.7%,
II. India has 20 lakh km2 of Exclusive the highest in any plan period so far. The main objective
Economic Zone for deep sea fishing extending of the tenth five year plan to attain 8% GDP growth per
320 km of sea from the coast line. year.
III. Fisheries contribute about 1% of India's 92. What of the following state has no territorial
GDP. boundary link with foreign country?
(a) Rajasthan (b) Punjab
IV. India's present annual fish production is (c) Bihar (d) Haryana
about 30 million tonnes of which more than Ans. (d) : The state of Bihar shares its border with
60% is exported. Which of these statements are Nepal as its international boundary, Punjab shares its
correct? border with Pakistan and Rajasthan also shares its
(a) I and II (b) II and IV border with Pakistan as international boundary. Haryana
(c) III and IV (d) I, II and III does not share its any border with any foreign countries.
Ans. (d) : The fisheries sector has been recognized as a 93. 'Shakti Sthal' is related to whom?
powerful income and employment generator as it (a) Smt. Indira Gandhi
(b) Rajiv Gandhi
stimulates growth of a number of subsidiary industries. (c) Mahatma Gandhi
In present India is the second largest fish producing (d) Pt. Jawahar Lal Nehru
country in the world accounting for 786 million tonnes Ans. (a) :
of global production and contributing about 1.24% to Memorial Site Leaders
the coutnry’s gross value added. Export earnings from Shakti Sthal St. Indira Gandhi
the Fisheries sectors has been Rs. 46, 662.85 crore. Shantivana Pt. Jawaharlal Nehru
During 2019-20. India has an exclusive economic area Veer Bhumi Mr. Rajiv Gandhi
of 20 lakh sq. km for deep sea fishing which extends Rajghat Mahatma Gandhi
Vijaya Ghat Lal Bahadur Shastri
200 nauticle mile or 320 km from the coast to the sea. It
is important to note that India ranks at first in the 94. Which of the following statements is not true
for Dr. Manmohan Singh?
production of Shrimps in the world. (a) Former Finance Minister of India
90. Correlate List I (Forest Types ) With List II (b) Former Governor of Reserve Bank of India
(Regions) in India and select the correct answer (c) Former Chairman of UGC
using the codes given below the list. (d) Former Director of IMF
46th BPSC (Pre) Exam. 2003-04 122 YCT
Ans. (d) : Dr. Manmohan Singh is an eminent (c) Life Expectancy at Birth
economist and is the former Prime Minister of India. He (d) Infant Mortality Rate
became 13th Prime Minister of India. During 1982-1985 Ans. (d) : The Human Development Index (HDI) is a
he was the Governor of Reserve Bank of India and in summary measure of average achievement in key
1991 became the chairman of the University Grant dimensions of human development: a long and healthy
Commission and also in 1991, he became the Finance
Minister of India in P.V. Narasimha Rao’s Government life, being knowledge and have a decent standard of
and remained in the post from 21 June, 1991 to 15 May living.
1996. The HDI is the geometric mean of normalized indices
95. Which sequence is true for the following posts for each of three dimensions.
in Indian Constitution? Dimensions Indicators Dimension
(a) Chief Justice of India → Chief Election Index
Commissioner → Governors of States Long & Life expectancy in Life
(b) Governor of States → Chief Justice of India healthy life birth expectancy
→ Chief Ministers of States → Chief Election index
Commissioner Knowledge • Expected year of Education
(c) Governor of States → Chief Minister of schooling Index
States → Chief Justice of India → Chief
Election Commissioner • Mean years of
(d) Chief Justice of India → Governor of States schooling
→ Chief Election Commissioner → Chief A decent GNI per Capita GNI Index
Ministers of States Standard of
Ans. (b) : The order of Precedence is a protocol list in living
which the functionaries and authorities are recorded by 99. Which one is not the component of the
rank and office in the Government of India. It is meant Pradhanmantri Gramodaya Yojana?
for state and ceremonial occasion. The order is as (a) Elementary Education
follows : 1 – President, 2 – Vice President, 3 – Prime (b) Primary Health
Minister 4 – Governor, 5 – Former President. 6 – (c) Nutrition
Deputy Prime Minister, 7 – Chief Justice of India, 8 – (d) Rural Road
Chief Minister 9-Justice of Supreme Court, 10- Chief
Election Commissioner etc. Ans. (c) : ‘Nutrition’ is not the component of the
96. Recommendations to the President of India on Pradhan Mantri Gramodaya Yojana. Pradhanmantri
the specific Union-State fiscal relations are Gramodhya Yojana aims is –
made by the I. Meeting rural needs like Primary education, health
(a) Finance Minister care, drinking water, housing, rural road.
(b) Reserve Bank of India II. Alleviating poverty in rural areas.
(c) Planning Commission III. Generating employment in rural areas.
(d) Finance Commission IV. Strengthening Panchayati Raj system in rural areas.
Ans. (d) : Article 280 of the Constitution of India 100. Which is known as the third pillar in
provides for a Finance Commission as a Quasi judicial
International Economic Relations?
body. It is constituted by the President of India every
fifth year or at such earlier time as he considers (a) IMF (b) World Bank
necessary. The Finance Commission is required to make (c) WTO (d) GATT
recommendations to the President of India on the Ans. (c) : The World Trade Organization (WTO) came
distribution of net proceeds of taxes to be shared into being on January 1, 1995 which holds a great
between the Centre and the States and the allocation promise for the entire world economy in respect of
between the states of the respective shares. International trade. It was earlier known with a name
97. Who is the Deputy Chairman of the Planning 'General Agreement on Tariffc and Trade (GATT)
Commission? which was replaced by WTO on 1st January, 1995. The
(a) Shri K C Pant WTO is a watchdog of International trade. Besides the
(b) Dr. Montek Singh Ahluwalia World Bank and the IMF, the World Trade
(c) Dr. Rangarajan Organization (WTO) is now being considered as the
(d) Dr. Bimal Jalan third pillar in the post-war International Economic
Ans. (b) : When the question was asked, the Deputy Relation. The headquarter of WTO is in Geneva,
Chairman of the Planning Commission was Dr. Montek Switzerland.
Singh Ahluwalia. The Planning Commission of India 101. Which is not included in the Millennium
was replaced by the NITI Aayog from 1 January 2015.
Development Goals?
98. Which one is not a component to construct (a) To eradicate extreme poverty and hunger
"Human Development Index in India" under (b) To achieve Universal Primary Education
UNDP? (c) To promote Gender Equality and empower of
(a) Real GDP per capita in US Dollar Women
(b) Literacy Rate (d) To achieve Double Digit Growth Rate
46th BPSC (Pre) Exam. 2003-04 123 YCT
Ans. (d) : The Millennium Development goals are an Ans. (c) : When the question was asked, Bihar was
ambitious agenda for reducing poverty and improving ranked 12th in terms of area but after bifurcation of
Life Sustainability. There are eight millennium Telangana from Andhra Pradesh, Bihar has became 13th
Development goals – State of India in terms of area. The total area of Bihar is
1. Eradicate extreme poverty and hunger 94, 163.03 sq. km. which is 2.86 percent of the total
2. Achieve universal primary education. area of India.
3. Promote gender equality and empower women. 107. What is the female literacy rate in Bihar as per
4. Reduce child mortality. 2001 census?
5. Improve maternal health (a) 60.32% (b) 53%
6. Combat HIV/AIDS, Malaria and other diseases. (c) 33.56% (d) 47.53%
7. Ensure environmental sustainability. Ans. (c) : According to the 2001 Census, the literacy
8. Develop a global partnership for development. rate in India was 65.38% in which male literacy rate
102. For what purpose, social security scheme for was 75.85% and female literacy rate was 54.16%.
workers of the unorganised sector is launched? As per Census 2011, Female literacy rate in Bihar is
(a) Old Age Pension 51.5%.
(b) Health Insurance 108. What is the sex ratio in India as per census
(c) Personal Accident Insurance 2001?
(d) All of the above (a) Per 1000 males 933 females
Ans. (d) : The central government on January 7, 2004 (b) Per 1000 males 945 females
launched a pilot project to provide social security like (c) Per 1000 males 964 females
medical, and pension to 37 crore workers of the (d) Per 1000 males 985 females
unorganized sector. Under this scheme a monthly Ans. (a) : As per census 2001 sex ratio was 933 and as
pension of Rs. 500 will be given to the workers on per 2011 census sex ration is 943.
retirement or completely disabled and personnel 109. When did India's population touch 100 crore
accident insurance will be provided to the workers as mark?
due to which in case of death or complete (a) May, 2000 (b) May, 2001
incapacitation. (c) May, 2002 (d) May, 2003
103. Who is the Chairman of Twelfth Finance Ans. (a) : India’s population reached 1 billion or 100
Commission? crore on 11th of May 2000.
(a) Prof. DT Lakadwala 110. On September 11, 2001 which building in the
(b) Dr. Rangarajan USA was hit by terrorist attack?
(c) Shri Digvijay Singh (a) White House (b) Capital Hill
(c) Pentagon (d) State Department
(d) Shri KC Pant
Ans. (c) : On September 11, 2001 the terrorist attack
Ans. (b) : The Twelth Finance Commission (TFC) was was done on the World Trade Center (WTC) and the
appointed by the President on 1st November 2002 under Defence office “Pentagon” which were the two major
the Chairmanship of Dr. C. Rangarajan. institution of United States of America. This attack is
At present, N.K. Singh is the current (15th) Chairman commonly known as 9/11 attack.
Finance Commission of India. 111. The charter of UNO was signed in
104. What is the rank of Bihar in view of population (a) Geneva Conference
in India? (b) San Francisco Conference
(a) 2nd (b) 3rd (c) Dumberton Oak's Conference
(d) Yalta Conference
(c) 5th (d) 11th
Ans. (b) : The Charter of the United Nation is the
Ans. (b) : When the question was asked, Bihar was founding document of United Nations. It was signed on
ranked at third in India in terms of population. June 26, 1945 in San Francisco, at the conclusion of the
According to the 2011 census, the most populous state United Nations Conference on International
of India were – Uttar Pradesh, Maharashtra, Bihar and Organization and came into force on 24 October 1945.
West Bengal. The United Nations can take action on a wide variety of
105. What is the following country has territorial issue due to its unique international character and the
boundary link with Bihar State? powers vested in its charter which is considered as an
international treaty.
(a) Bhutan (b) Nepal
(c) China (d) Bangladesh 112. At present, how many members does NATO
have?
Ans. (b) : Bihar shares its border to the north with (a) 26 (b) 15
Nepal and to the south with Jharkhand state. West (c) 18 (d) 12
Bengal is situated in the eastern side of Bihar and Uttar Ans. (a) : When question was asked, The North
Pradesh in the west. Atlantic Treaty Organization (NATO) was established
106. What is the rank of Bihar in India in view of in 1949. It is a group of 30 countries from Europe and
Geographical Territories? North America that exists to protect the people and
(a) 2nd (b) 4th territory of its members, When the question was asked,
there were 26 member in NATO.
(c) 12th (d) 9th
46th BPSC (Pre) Exam. 2003-04 124 YCT
113. Where is the permanent headquarter of Ans. (c) : On 9th August 1974, Gerald Ford became the
International Court of Justice? 38th President of the United States despite never having
(a) The Hague (b) Geneva contested a Presidential election. At present Joe Biden
(c) Moscow (d) Washington is the 46th president of United States of America. He
Ans. (a) : The headquarter of the International Court of belongs to Democratic Party.
Justice is in the Hague. (Netherland). It is noteworthy 119. Most commonly used safety fuse wire is made
that in the six prominent organs of the UNO, the of
International Court is a important one. The six organs of (a) Tin
UNO are – General Assembly, Security Council, Trust (b) Lead
Council Ship, Economic & Social Council, the (c) Nicked
International Court of Justice, and the UN Secretariat. (d) An alloy of tin and lead
114. The Charter of Indian Ocean Rim Association Ans. (d) : A fuse in an electrical safety device that
for Regional Co-operation (IORARC) was operates to provide over current protection of an
signed on electrical circuit. It is used in a series of equipment as a
(a) April, 1993 (b) March, 1997
safety device. The material used for making a fuse wire
(c) April, 1999 (d) March, 2002
is become an alloy of lead and tin which has very low
Ans. (b): Indian Ocean Rim Association for Regional melting point.
Co-operation (IORARC) is a regional cooperation
120. A rock is through from the surface of the Moon
initiate for the Indian Ocean Rim countries, was
established at Mauritius in March 1997 with the aim of to Earth. Then,
promoting economic and technical cooperation. (a) Its mass will change
IORARC is the only pan-Indian ocean grouping. At (b) Its weight will change but not mass
present the total members of (IORARC) are 23. (c) Both mass and its weight will change
(d) Neither mass nor weight would change
115. When did India examine its first nuclear
explosion? Ans. (b) : The weight of an object is defined as the
(a) 15th May, 1964 (b) 18th May, 1974 force of gravity on the object. i.e. w = mg.
(c) 11th May, 1989 (d) 13th May, 1998 Where m = mass of the object
g = acceleration due to gravity
Ans. (b) : India conducted its first nuclear test on 18th
If a rock is brought from the surface of the moon to the
May 1974 at Pokhran in Jaisalmer district of Rajasthan.
earth, its weight will change but mass remains
Smt. Indira Gandhi was the Prime Minister of India
during first nuclear test. Once again India had unchanged because mass of a body is always constant.
conducted its 2nd nuclear test after Pokhran I, during 121. Sweetest among all sugars is
reign of Prime minister Atal Bihari Vajpayee on 11th (a) Fructose (b) Glucose
and 13th May 1998. (c) Sucrose (d) Lactose
116. When was Soviet Union disintegrated? Ans. (a) : Fructose is the sweetest naturally occurring
(a) December, 1989 (b) August, 1990 sugar, estimated to be twice as sucrose. Fructose is also
(c) December, 1991 (d) January, 1994 derived from the digestion of sucrose, a disaccharide
consisting of glucose and fructose that is broken down
Ans. (c) : The Dissolution of the Soviet Union on
to glycoside hydrolyses enzymes during digestion.
December 25, 1991, marked the end of the Cold War.
Hence, fructose is the sweetest of all carbohydrates that
The disintegration of the USSR began in the late 1980s
with growing unrest in the various constituent republics do occur naturally.
and ended on December, 26, 1991, when Supreme 122. You are standing in a lift. The force on the
Soviet voted to dissolve. floor of the lift due to your weight will
(a) Become zero when the lift moves up with an
117. Who is the most powerful leader of Communist
acceleration of 9.8 ms–2
China at present?
(b) Become zero when the lift moves down with
(a) Hu Jintao (b) Jiang Zemin
an acceleration of 9.8 ms–2
(c) Deng Xioping (d) Zhao Ziyang
(c) Become zero when the lift moves with an
Ans. (a) : When question was asked, Hu Jintao was the
acceleration of 9.8 ms–2 in any direction
General Secretary of the ruling Communist Party of
China. He was President of the Republic of China and (d) Remain unchanged whatever the rate of
also the Chairman of the Military Commission (CMC). acceleration of the lift be
At present Xi Jinping is the most powerful leader of the Ans. (b) : When the lift falls freely, the force on the
Republic of China (ROC). floor of the lift due to your weight will becomes zero
118. Only unelected President in the history of USA when the lift moves down with an acceleration of 9.8
was ms–2. In such case, the lift falls freely under gravity,
(a) Munro (b) Nixon then a = g, the net downward force on the person is R =
(c) Ford (d) Carter m(g-g) = O.

46th BPSC (Pre) Exam. 2003-04 125 YCT


Thus the apparent weight of the man become zero this is Ans. (c) : In the process of electroplating a utensil with
because both the man and the lift are moving downward zinc, the utensil is made as cathode charge and pure
with same acceleration. Hence, a person develops a zinc is made as the anode charge so that the zinc layer
falling of weightlessness. can be easily electroplated on the utensil. Thus zinc
123. The metal ion present in chlorophyll is serves as a sacrificial anode so that it catholically
(a) Iron (b) Magnesium protects exposed still.
(c) Zinc (d) Cobalt 128. The noble gas not present in the air is
Ans. (b) : Chlorophyll plays an important role in the (a) Helium (b) Neon
process of Photosynthesis. Since it allows the plants to (c) Argon (d) Radon
absorb energy from the sunlight. The element, Ans. (d) : Radon is the only radioactive out of all noble
Magnesium is an essential component of the green gases. Radon it is not present in the atmosphere. Radon
pigment present in the plants called Chlorophyll. is usually isolated from the radioactive decay of the
dissolved radium, thorium or uranium compounds.
124. Which of the following is not a vestigial organ
129. One volume of zinc (density 7100 kg-m-3) is
in man?
mixed with two volumes of copper (density
(a) Nictitating membrane
8900 kg-m-3) to make brass. the density of
(b) Auricular muscles of Pinna
brass is
(c) Incisors (a) 8000 kg-m-3 (b) 8300 kg-m-3
(d) Vermiform appendix (c) 9500 kg-m-3 (d) 9800 kg-m-3
Ans. (c) : Vestigial organs are those organs which are Ans. (b) : The weight of one volume of zinc is
no longer in use but are still present in our body. A few = 7100kg / m× = 7100kg
example of Vestal organs in human are outer rim of the
ear, the Vermiform appendix wisdom tooth and and weight of two volumes of copper is
Nictitating Membrane. Incisors in not a vestigial organ = 8900kg / mx2 = 17800kg.
in human body. On mixing two − ( 7100 + 17800 ) kg = 24900kg.
125. What is dry ice? 24900
(a) Solid ice Therefore density of brass should be = kg / m3
3
(b) Solid carbon dioxide
(c) Solid ammonia 8300kg / m3 Ans.
(d) Solid sulphur dioxide 130. Which of the following vitamins is necessary
Ans. (b) : Dry ice is just cooled and condensed carbon for clotting of blood?
dioxide. It sublimates directly into carbon dioxide gas (a) Vitamin A (b) Vitamin C
when it reaches room temperature and pressure and (c) Vitamin E (d) Vitamin K
expands. Dry ice is manufactured by compressing and Ans. (d) : Vitamin K is the fat soluble vitamin. Vitamin
cooling gaseous CO2 under high pressure to initially K plays an important role in the coagulation of blood or
produce liquid CO2 and pumping it into a storage tank, in blood clothing, therefore vitamin k is essential for
where it freezes at -109oF and is compacted into solid blood clotting.
ice. 131. A metal when left exposed to air for sometime,
126. Evolution proceeds on account of gets coated with green basic carbonate. The
(a) Fossils metal is
(b) Struggle for existence (a) Copper (b) Silver
(c) Adaptation through generations (c) Nickel (d) Zinc
(d) Inheritance Ans. (a) : When copper is exposed to atmosphere, a
Ans. (c) : The reason for Evolution is – orderly changes green coating of basic copper carbonate gets deposited
the process of origin of new organism, species, classes over its surface. This is called corrosion of copper the
etc. From ancient organism through which as a result Reaction is a as follows : -
more complex organism are born from simpler 2Cu + CO 2 + H 2 O + O 2 → CuCO3 Cu ( OH ) 2
organism or it results out by the adoption through copper from a rich variety of compounds with oxidation
generations, and inheritance – transmission of traits states +1 and +2 which are often called cuprous and
from parents to offspring or new generation body. cupric respectively.
127. In the process of electroplating a utensil with 132. The gas used for artificial ripening of fruits is
zinc (a) Ammonia (b) Acetylene
(a) The utensil is made the cathode (c) Carbon monoxide (d) Ethylene
(b) Pure zinc is made the anode
Ans. (b) : Acetylene gas is used for artificial repining of
(c) The utensil is made the cathode and pure zinc
fruits.
is made the anode
(d) The utensils made the anode and pure zinc is Ethylene is also a natural plant hormones, it accelerates
made the cathode repining in climacteric fruits.

46th BPSC (Pre) Exam. 2003-04 126 YCT


133. The explosive nitroglycerine is Ans. (c) : Insulin is a harmone. It is to be noted that in
(a) A salt the pancreas gland, there are several iselets of
(b) A nitro hydrocarbon longerhans found whose beta cells secrete the insulin
(c) A complex hydrocarbon harmone in human body. It is important to note that the
(d) An ester Diabetes disease is caused by the deficiency of Insulin.
Ans. (d) : Nitroglycerin is an ester. Esters are chemical 139. Hydrofluoric acid is not kept in glass bottles
compounds that are derived from acids (organic or because it reacts with
inorganic). Trinitroglycerin is a colourless oily liquid. It (a) Visible light
is used to make dynamite. It is prepared by reacting (b) Sodium oxide of glass
concentrated sulphuric acid and concentrated nitric acid (c) Aluminum oxide of glass
with glycerin. (d) Silicon dioxide of glass
134. How many members does UNO have at Ans. (d) : Hydrofluoric acid is normally handled in
present? metal apparatus made of copper and is always stored in
(a) 189 (b) 191 plastic bottles. Therefore hydrofluoric acid (HF) cannot
(c) 185 (d) 193 be stored in the glasses because they corrode the
Ans. (b) : When question was asked, there were 191 silicates of glass and dissolve in the hydrofluoric acid.
members countries in UNO. United Nations 140. Bihar is 'great religious centre' for
Organization (UNO) is an international organization. It (a) Sikhs (b) Jains
was founded in 1945 after the World War II to promote (c) Buddhists (d) All of these
international co-operation. Currently, United Nations is Ans. (d) : Bihar is significant Center of Hinduism,
made of 193 members countries. When the question Jainism, Buddhism and Sikhism. The 10th Guru of Sikhs
was asked, the members of UNO were 191. is Guru Govind Singh was born at Patna in 1866.
135. While tinning of brass utensils the ammonium Mahatma Buddha have attained enlighten in Bodh Gaya
chloride powder used to clean the hot utensil, in Bihar. Pawapuri in Bihar is near holy sites for Jains
produces fumes of Mahavira was born in Vaishali near Hajipur.
(a) Ammonia 141. The radius of the circle, which touches the line
(b) Carbon monoxide x – y = 0 and is concentric with the circle x2 + y2
(c) Hydrochloric acid –6x – 2y – 3 = 0 is
(d) Ammonia and hydrochloric acid 1
Ans. (d) : While tinning of brass utensils the (a) 2 (b)
ammonium chloride powder used to clean the hot 2
utensil, produces fumes of ammonia and hydrochloric (c) 1 (d) 2
acid. Ans. (a) : The center of given circle
136. The American Spaceship Columbia met with x2 + y2 –6x – 2y – 3 = 0
tragedy in can be find by changing it into standard equation of
(a) 2000 AD (b) 2001 AD circle.
(c) 2002 AD (d) 2003 AD (x2 – 6x + 9) – 9 + (y2 – 2y + 1) – 1 – 3 = 0
Ans. (d) : The space shuttle Columbia broke apart on
(x – 3)2 + (y – 1)2 – 13 = 0
Feb.1, 2003 while re-entering the Earth’s atmosphere,
killing all seven crew members including the scientist of  X = (x – 3) 
(x – 3) 2 ( y – 1)
2
Indian origin Kalpana Chawala. The tragedy occurred 13  
2
+ 2
= 2  Y = (y – 1) 
over Texas and only minutes before Columbia was X Y A  2 
scheduled to land at the Kennedy space centre.  A = 13 
137. The end product of alcoholic fermentation is Hence, the centre of the circle
(a) Pyretic acid (b) Acetaldehyde X = 0, Y = 0
(c) Ethyl alcohol (d) Formic acid x – 3 = 0, y–1=0
x = 3, y = 1
Ans. (c) : Fermentation is anaerobic respiration and
It means the centre of the circle = (3, 1)
occurs in the absence of oxygen. During alcoholic
The radius of circle which touches the line x – y = 0
fermentation, pyruvic acid (the end product of
should be equal normal on line (x– y = 0) from centre,
glycolysis) is first converted into acetaldehyde by which is–
pyruvate decarboxylase and CO2 is released.
Acetaldehyde is further reduced to alcohol by the 3 –1 2
= = = 2
2 2
enzyme alcohol dehydrogenate. Therefore the end 1 +1 2
products of alcoholic fermentation are carbon dioxide 1 −1 1 α 
and ethanol (CO2 and C2H5OH), Ethanol is also known 142. Let matrices A= 2 2  , B= β 1 
as ethyl alcohol.    
138. Out of the following the hormone is if (A+B)(A–B)=A2–B2, then
(a) RNA (b) Invertase (a) α = β = –1 (b) α = β = 0
(c) Insulin (d) Ascorbic acid (c) α = β = 1 (d) α =1, β = –1

46th BPSC (Pre) Exam. 2003-04 127 YCT


Ans. (b): = 1+ 2, 1 – 2
 1 −1  1 α  2 α − 1
( A + B) =  + = 
r = 2 +1
 2 2  β 1  2 + β 3 
n2n
1 −1 1
( A − B) = 
α  0 − (1 + α )  144. Series ∑ nn
is
−  = 
 2 2  β 1  2 − β 1  (a) convergent
2
(b) divergent
1 −1 1 −1 1 −1  −1 −3 (c) divergent and divergent
A2 =   = × =  (d) None of these
2 2   2 2  2 2   6 2 
n2n
1 α 
B2 =   =
2
1 α   1 α  1 + αβ
 ×  =
2α  Ans. (a) : ∑ nn
β 1  β 2  β 1   2β αβ + 1
n2 n
n + 1 2n +1
∵ ( A + B )( A − B ) = A 2 − B2 an = a n +1 =
 2 α − 1  0 − (1 + α )   −1 −3 1 + αβ 2α 
nn ( n + 1)n +1
∴  =  −  2β n + 1 2n +1 n n
 2 + β 3  2 − β 1   6 2   αβ + 1 a n +1
= .
( α − 1)( 2 − β ) −2 (1 + α ) + ( α − 1) 
an ( n + 1)n +1 n2n
∴ 
 6 − 3β − (1 + α )( 2 + β ) + 3 ( n + 1) 2. n n
=
 − ( αβ + 2 ) −2 ( 3 + 2α )  ( n + 1)n +1
=
 ( 6 − 2β ) (1 − αβ )  ⇒
a n +1
= 2.
nn
by matrix rule, an ( n + 1)n
6 − 3β = 6 − 2β..................(i) ⇒ β = 0 n
 
− (1 + α )( 2 + β ) + 3 = (1 − αβ ) ........(ii)  n   1 n

= 2  = 2 
by equation (ii)  n +1   1 + 1 
– 2 −2 − β − 2α − αβ + 3 = 1 − αβ  n 
1 = β + 2α + 1  
n

2α + β = 0 (∵ β = 0 ) ⇒ lim
an +1  1
= 2 lim 


so, 2α + 0 = 0 n →∞ a n n →∞
 1 + 1 
α=0  n 
 
Hence, α = β = 0  1 
lim n  −1
n →∞  1 
143. The area of a circle is doubled when its radius  1+ 
⇒ 2e  n 
is increased by 1. The radius of the circle is
(a) 2 2 (b) 2 2 − 1 ( ) 
 −
1
lim  n. n
1



n →∞  
(c) 2 − 1 (d) 2 + 1  1+
= 2e  n 
Ans. (*) : Let assume the radius of circle = r  
then the area = πr2  −1 
lim  
as per question, when radius is increased by 1, its area is n→∞ 
 1+
1 
doubled – = 2e  n 
2πr2 = π(r + 1)2 2
2πr2 = π(r2 + 1 + 2r) = 2e−1 = <1
2r2 = r2 + 2r + 1 e
r2 – 2r – 1 = 0 a +1
ie lim n <1
n →∞ a n
( 2) – 4×1× ( –1)
2
+2 ±
r= n2n
+2 ± 4 + 4
2×1 so ∑a = ∑ n
nn
is convergent
= Hence option (a) is true.
2
145. If, a, b are roots of the equation x2+11x+50=0,
2+ 8 α β
= then + equal to
2 β α
2±2 2 (a) 0.21 (b) 0.42
=
2 (c) 0.84 (d) 1.0
46th BPSC (Pre) Exam. 2003-04 128 YCT
Ans. (b): The equation x2 + 11x + 50 = 0  1+ x 
Sum of the roots (α + β) = –11 ie f −1 ( x ) = 5  
product of the roots (α β) = 50  1− x 
–1
now so domain of f is R –{1}
so option (b) is correct.
α β α 2 + β2
+ = 1
β α αβ 148. The value of lim 1
is
x →0 −
we know 1− e x

( α + β ) = α 2 + β2 + 2αβ
2 (a) 1 (b) 0
(c) –° (d) °
α 2 + β2 = ( α + β ) − 2αβ
2
1
= (–11)2 – 2 × 50 Ans. (a) : lim
x →0 1 − e −1/ x
= 121 – 100
1
α 2 + β2 = 21 lim
α β 21
x →0 e −1/ x
(e
1/ x
)
−1
+ = 1/ x
then β α 50
= 0.42
lim
x →0 e1/ x
−1
e
(e1/ x
= e∞ = ∞ )
so option (b) is true. Applying L Hospital Rule
g {f ( −1)} 1
 1 
2
146. If f(x)=x+1 and g(x)=x –3, then is ex . −
 
f {g ( −1)}  x2 
=
equal to –  1 
(a) 0 (b) 1 e1/ x  − 2  − 0
 x 
(c) 2 (d) 3 =1
Ans. (d): Given equations 149. If x = a cos q + b sin q and y = a sin q – q cos q,
f (x) = x + 1 ............(1) then x2 + y2 is equal to
g(x) = x2 – 3 ........... (2) (a) a2–b2 (b) b2–a2
putting x = –1 in eqn (1) we get 2
(c) a +b 2
(d) a2+2ab
f (–1) = –1 + 1
=0 Ans. (c) : x = a cos q + b sin q .................(1)
putting x = f(–1) = 0 in eqn (2), we get y = a sin q –b cos q ................ (2)
g (f (–1)) = 02 – 3 squaring both sides,
= 0 –3 x2 = a2 cos2 q + b2 sin2 q + 2ab cosq sinq....................(3)
y2 = a2 sin2q + b2 cos2 q – 2ab cosq sinq ................ (4)
=–3 adding equation (3) and (4)
Similarly g (–1) = (–1)2 – 3 = –2 x2 + y2 = a2 cos2 q + b2 sin2q + a2 sin2q + b2 cos2 q +
f(g(–1)) = – 2 + 1 2ab sin q cos q – 2 ab sinq cos q
=–1
= a2 (sin2 q + cos2q) + b2 (sin2q + cos2q)
g {f ( −1)}
then
f {g ( −1)}
=
−3
−1
=3 x 2 + y 2 = a 2 + b2 (∵ sin 2
)
q + cos 2 q = 1
so option (c) is correct.
so option (d) is true.
150. The number of solutions of the equation
x −5
147. If f(x)= , x ≠ –5,then the domain of 2
x − 3 x + 2 = 0 is
x +5
f–1(x)is (a) 2 (b) 3
(a) R (b) R–1 (c) 4 (d) 5
(c) (–∞,1) (d) (1,∞) Ans. (c) : |x|2 –3 |x| + 2 = 0
x −5 if x ≥ 0, | x |= x
Ans. (b) : if f ( x ) = , x≠5
x +5 the given equation can be written as
x −5 x2 – 3x + 2 = 0
y= ⇒ (x –1) (x –2) = 0
x+5
⇒ xy + 5y = x − 5 ⇒ x = 1, 2
⇒ xy − x = −5y − 5 similarly for x < 0 , ie
|x| = –x
−5 ( y + 1) x2 + 3x + 2 = 0
⇒x=
y −1 (x + 1) (x + 2) =0
⇒ x = –1, –2
x=5
(1 + y ) Hence 1, 2, –1, –2 are four solutions of the given
(1 − y ) equation .

46th BPSC (Pre) Exam. 2003-04 129 YCT


47th Bihar Public Service Commission
Preliminary Examination, 2004-05
GENERAL KNOWLEDGE & GENERAL SCIENCE
(Solved Paper with Detail Explanation)
1. Where was Mahavir Swami born? Ans. (c) : The most famous educational centre during
(a) Kundagram (b) Patliputra the period of Mauryan age was the Takshila University,
(c) Magadh (d) Vaishali which is still considered to be the oldest university in
Ans. (a) : Mahavira Swami is considered the 24th and the world with its presence dating back to 5th or the 6th
last Tirthankara of Jainism. He was born in 599 BCE at century.
a village named Kundagram near Vaishali. His father, 5. At which place in Bihar was the session of
Siddhartha was a Kshatriyas King from the Ikshvaku Indian National Congress of 1922 held?
dynasty & his mother Trishala was Lichchhavi's King (a) Harpura (b) Patna
Chetaka's sister. He passed away and became a Siddha (c) Gaya (d) Ramgarh
at the age of 72 years in 527 BCE at Pavapuri near
Patna. Ans. (c) : Indian National Congress session of 1922 was
held at Gaya, Bihar. This session was presided over by
2. Where did Mahatma Buddha's
Deshbandhu Chittaranjan Das. In this session C.R. Das
'Mahaparinirvana' take place?
and other leaders break away from INC.
(a) Lumbini (b) Bodh Gaya
(c) Kushinagar (d) Kapilvastu 6. Which was the first capital of Magadh?
(a) Patliputra (b) Vaishali
Ans. (c) : Gautam Buddha died at the age of 80 in 483
BCE at a place called Kushinagar a town in UP. The (c) Girivraja(Rajgriha) (d) Champa
event is known as Mahaparinirvana. Ans. (c) : Magadha was an ancient Indian kingdom. It
Siddhartha Gautam, born in circa 563 BCE in royal was counted among one of the 16 Mahajanpadas. Its
family of Sakya clan who ruled from Kapilvastu, in first capital was Rajgriha which was also known as
Lumbini which is situated near the Indo-Nepal Girivraja in southern Bihar. Rise to political supremacy
Border.At the age of 29, Gautama left home and after began with Bimbisar, who belonged to Haryanka
49 consecutive days of meditation, Gautama dynasty. The city of Rajgriha, historically known as
attained Bodhi(enlightenment) under a pipal tree Girivraja was an ancient city which is located in
at Bodhgaya a village in Bihar. Buddha gave his first Nalanda district of Bihar. It was the first capital of
sermon in the village of Sarnath, near the city of Magadha. Bimbisara's grand son Udayin had founded
Benares in UP. This event is known as Dharma-Chakra- the new capital at Pataliputra. Shishunaga had
Pravartana (turning of the wheel of law). temporarily shifted capital to Vashali.
3. At which place did Mahatma Buddha give his 7. Which dynasty ruled over Magadh after Nanda
first 'Dharmachakra Pravartan'? dynasty?
(a) Lumbini (b) Sarnath (a) Maurya (b) Shunga
(c) Patliputra (d) Vaishali (c) Gupta (d) Kushana
Ans. (b) : Gautam Buddha delivered his first sermon on Ans. (a) : Dhanananda was last king of Nanda dynasty.
deliverance from suffering to his five former He is believed to be an arrogant and oppressive ruler
companions in a deer park at Sarnath (Varanasi). This who imposed heavy taxes on common man. His greed
event is known as ' Dhammachakka - Pavattana and exploitation made him quite unpopular among the
meaning 'turning the wheel of dharma.' Buddha gave his masses & ultimately Chandragupta Maurya under the
first sermon in the village of Sarnath, near the city of able guidance of Kautilya, took the advantage of this
Benares in UP. This event is known as Dharma-Chakra- public resentment and uprooted the Nanda rule and
Pravartana (turning of the wheel of law). setup the Mauryan empire.
4. The most famous educational centre during the Thus option (a) is correct.
Period of Mauryan age was 8. Who was the writer of 'Mudrarakshas'?
(a) Vaishali (b) Nalanda (a) Vishakhadatta (b) Kautilya
(c) Takshila (d) Ujjain (c) Bana (d) Kalhana

47th BPSC (Pre) Exam. 2004-05 130 YCT


Ans. (a) Mudrarakshas was written by Vishakhadatta. 13. Who was the ruler of Devagiri at the time of
Authors Theirs Book Alauddin Khilji's invasion?
Kautilya Arthashashtra, Sampurna Chanakya Neeti (a) Ramchandra Dev (b) Prataprudra Dev
Banabha Harshacharitra, Kadambari (c) Malik Kafoor (d) Rana Ratan Singh
tta Ans. (a) : Alauddin Khalji was Jalaluddin's ambitious
Kalhana Rajatarangini nephew and son - in - law. After winning forts of
Harsha Ratnavali, Nagananda, Priyadarshika. Chittor and Ranthambore, He desired the conquest of
9. Who has written the 'Harshacharit'? Deccan. He sent a huge army under Malik Kafur in
(a) Kalidasa (b) Banabhatta 1306 - 1307 CE against Rai Ramachandra Dev, the ruler
(c) Vishnugupta (d) Parimalgupta of Devagiri, who had earlier promised to pay tribute to
Sultan but did not pay. Ramachandra surrendered after
Ans. (b) : Harshacharit was written by Banabhatta.
little resistance to Malik Kafur & was treated
10. A Buddhist Council during the reign of
honourably.
Kanishka was held at
(a) Magadh (b) Patliputra 14. In the Mughal administration 'Muhtasib' was
(c) Kashmir (d) Rajgriha (a) Military Officer
Ans. (c) : (b) Head of Foreign Affairs
Buddhist Year Venue Chairman Royal (c) An Officer-in-charge of public morals
Council Patron (d) Officer of Correspondence Department
First 483 Rajgriha Mahakasyapa Ajatashatru
BCE Ans. (c) : In mughal administration Muhtasibs were the
Second 383 Vaishali Sabakami Kalashoka
censors of public morals who were appointed to ensure
BCE the general observance of the rules of morality. They
Third 250 Pataliputra Moggaliputta Ashoka also used to examine weights and measures and enforce
BCE Tissa fair prices etc.
Fourth 75AD Kashmir Vasumitra Kanishka
centuray 15. A common feature to all Bhakti Saints was that
CE they
11. What is the name of Megasthenese's book? (a) composed their verses in the language
(a) Arthashashtra (b) Rigveda understood by their followers
(c) Purana (d) Indica (b) rejected the authority of the priestly class
Ans. (d) :Megasthenes, ambassador of Seleucus I (c) encouraged women to go to the temples
Nicator,sent to the court of Chandragupta Maurya (d) encouraged idol worship
between 302 to 298 BC. He witnessed the reign of Ans. (a) : The Bhakti reformers preached the principles
Chandragupta and wrote about it in his book ‘Indica’. of monotheism and criticized idol workship. The Bhakti
Accounts of the king’s administration can be found in Saints usage of vernacular or regional languages of the
the book of Megasthenes in great detail. According to masses, as opposed to the sacred language 'sanskrit' of
his accounts, the army that built that empire was heavily the elite priests. They composed their poems in popular
reliant on the supply of troops, oxen, horses, and languages and dilects spoken across India which further
elephants. Megasthenes also praised the wooden enabled them to transmit their ideas among the various
architecture of the Mauryan empire. According to him, lower classes and masses.
the Mauryan palace at Pataliputra was as magnificent as 16. 'Satyartha Prakash' was written by
the one in Iran’s capital. He also mentions the wooden
constructions at Pataliputra, known as Palibothra, to (a) Raja Ram Mohan Roy
Greek and Latin writers. (b) Mahatma Gandhi
12. Who was the first ruler of the Slave dynasty? (c) Swami Vivekananda
(a) Iltutmish (d) Swami Dayanand Saraswati
(b) Qutub-ud-din Aibak Ans. (d) : Swami Dayanand Saraswati wrote 'Satyartha
(c) Razia Prakash' Dayananda's views were published in his
(d) Balban famous work, Satyarth Prakash. He had setup the first
Arya Samaj Unit at Bombay in 1875 and later the head
Ans. (b) : The slave dynasty was also called the
Mamluk dynasty. Qutub - ud - din Aibak was the quarters of the Arya Samaj were established at Lahore.
founder of slave dynasty. He was a Turkish slave of 17. Raja Ram Mohan Roy established Brahma
Muhammad Ghori. He is known as 'Lakh Baksh'. He Samaj in
had built Quwat - ul - Islam (first mosque in India) and (a) 1816 AD (b) 1820 AD
Adhai din ka Jhopara in Ajmer. (c) 1828 AD (d) 1830 AD
47th BPSC (Pre) Exam. 2004-05 131 YCT
Ans (c): Raja Ram Mohan Roy founded Brahmo Ans. (a) : Abul Kalam Azad was the Indian National
Sabha on August 20, 1828, in Calcutta which was later Congress president who conducted negotiations with
renamed as Brahmo Samaj. Its aim was the worship of Cripps Mission in 1942 and Lord Wavell at Shimla
the eternal God. It was against priesthood, rituals and conference. Abul Kalam Azad had presided over INC
sacrifices. It focused on prayers, meditation and from 1940 to 1945.
reading of the scriptures. It was the first intellectual 23. How many seats were given to depressed
reform movement in modern India. It split into two classes under Communal Award and Poona
part in 1866 namely Brahmo Samaj of India led Pact?
by Keshub Chandra Sen and Adi Brahmo Samaj led (a) 74 and 79, respectively
by Debendranath Tagore. (b) 71 and 147, respectively
18. The first President of "All India Trade Union (c) 78 and 80, respectively
Congress' was (d) 78 and 69, respectively
(a) VV Giri (b) SA Dange
Ans. (b) : The communal Award was announced by
(c) Pt. Nehru (d) Lala Lajpat Rai Ramsay Mac Donald on August 16, 1932.
Ans. (d) : The All India Trade Union Congress Poona Pact was signed by B.R. Ambedkar on behalf of
(AITUC) was founded on October 31, 1920. Lala depressed classes on 24 September 1932. The poona
Lajpat Rai was elected as the first President of AITUC pact abandoned the idea of seperate electorates for the
and Dewan Chaman Lal as the first general secretary. depressed classes. But the seats reserved for the
19. The Prime Minister of England at the time of depressed classes were increased from 80 to 147 in
Quit India Movement was provincial legislature. The poona pact was accepted by
(a) Chamberlain (b) Churchill the government as an amendment to the communal
(c) Clement Attlee (d) Mc Donald award.
Ans. (b) : Sir Winston Churchill was the British Prime 24. Who among the following capitalists served as
Minister during the Quit India Movement. AICC Treasurer for a long period and went to
Clement Attlee was the Prime Minister of England at jail in 1930?
the time of Indian Independence. (a) GD Birla (b) Jamnalal Bajaj
Quit India Resolution was ratified at the congress (c) JRD Tata (d) Walchand
meeting at Gowalia Tank, Bombay on 8 August 1942. Hirachand
20. The 'Anushilan Samiti' was Ans. (b) : Jamnalal Bajaj was an Indian industrialist and
(a) Dedicated to the upliftment of women freedom fighter of India. He served as All India
(b) Promoting widow remarriage Congress Committee Treasurer in 1920 and remained
(c) Interested in labour welfare on this post all his life. He went to jail in 1930. He was
considered to be the 5th son of Gandhi Ji.
(d) A revolutionary organisation
25. Which British Commander was defeated by the
Ans. (d) : Anushilan Samiti was formed by Barinder
Santhals is 1855?
Kumar Ghosh and Jatindernath Banerji. On 24 March
(a) Captain Nek Feville
1902. It was headed by Barindra Kumar Ghosh. It was a
(b) Lt. Bastain
secret revolutionary organisation. It Promoted
revolutionary violence as a way to end British rule in (c) Major Burrough
India. (d) Colonel Whicte
Ans. (c) : Major Burrough was defeated by the Santhals
21. 'Ring fence' policy was associated with
in 1855. On 30th June 1855 Santhal brothers Sidhu and
(a) Warren Hastings (b) Lord Dalhousie Kanhu Murmu organised 10,000 Santhals and
(c) Henry Lawrence (d) Lord Hastings proclaimed a rebellion against the British. The tribals
Ans. (a) : Warren Hastings followed a policy of ring - took an oath to drive away the British from their
homeland. The Murmu brothers’ sisters Phulo and
fence which aimed at creating buffer zones to defend
Jhano also played an active part in the rebellion.
the company is frontiers. Santhals are a tribal group concentrated in the state of
The subsidiary alliance system was used by lord Jharkhand. This was the first peasant revolt that
Wellesley. occurred in India. The revolt can be attributed to the
Doctrine of Lapse is attributed to lord Dalhousie, but he introduction of the Permanent Land Settlement of 1793.
was not its originator. 26. Who presided over the first session of All India
Kisan Sabha?
22. Which Congress President negotiated with
(a) Swami Sahjananda
Cripps Mission (b) Indulal Yajnik
(a) Abul Kalam Azad (b) Jawaharlal Nehru (c) NG. Ranga
(c) JB Kirpalani (d) C Rajagopalachari (d) PC Joshi
47th BPSC (Pre) Exam. 2004-05 132 YCT
Ans. (a) : The All India kisan Sabha was founded in (b) Ajay Ghosh
Lucknow in April 1936 with Swami Sahajanand (c) Jyotindranath
Saraswati as the president and N.G. Ranga as the (d) Bhagat Singh
General secretary. A kisan manifesto was issued and a Ans. (a) : Under the leadership of Chandrashekhar
periodical under Indulal Yagnik was started. Azad, the name of Hindustan Republic Association
(HRA) was changed to Hindustan Socialist Republican
27. Who was the leader of revolt of the Mundas in Association (HSRA). The participants included Bhagat
1899-1900? Singh, Sukhdev, Bhagwati Charan Vohra from Punjab,
(a) Sibhu Phanindranath Ghosh from Bihar and Bejoy Kumar
(b) Budhu Bhagat Sinha, Shiv Verma, Jaidev Kapur from the United
(c) Birsa Munda Provinces. The HSRA decided to work under a
(d) Sambhudan collective leadership and adopted Socialism as its
official goal.
Ans. (c): Munda Rebellion was one of the most
important tribal movement. It was led by Birsa Munda 31. When was the new Enfield Rifle with greased
cartidges introduced in the British Indian
in south of Ranchi in 1899 - 1900. The cause of this
Army?
revolt was the unfair land grabbing practices by colonial (a) November, 1856 (b) December, 1856
and local authorities that demolished the tribal (c) January, 1857 (d) February, 1857
conventional land system. Birsa Munda also known as Ans. (b) : The British East India company decided to
'Dharti Aba'. He started a movement called 'Ulgulan' or use new Enfield Rifle instead of old Brown Bess gun in
the Great Tumult'. December 1856. The report about the mixing of bone
28. Who presided over the Delhi session of Indian dust in olta (flour) & the introduction of enfield rifle
National Congress in 1947? enhanced the sepoy growing disaffection with the
(a) JB Kriplani government. The greased wrapping paper of the
(b) Rajendra Prasad cartridge of new rifle had to be bitten off before loading
& grease was reportedly made of beef and pig fat. The
(c) Abul Kalam Azad
cow was Sacred to the Hindus while the pigs was taboo
(d) Jawaharlal Nehru for the Muslims. The Army administration did nothing
Ans. (b) : In Delhi Indian National Congress session on to allay these fears, and the sepoy felt their religion was
14th June 1947, the resolution for India's partition was in grave danger.
passed, The Delhi session was presided over by Dr. 32. When was All India Trade Union Congress
Rajendra Prasad, Maulana Abul Kalam Azad had established in Bombay?
presided our the Ramgarh session of INC in 1940. (a) 1920 (b) 1925
Jawahar Lal Nehru had presided over the Faizpur (c) 1929 (d) 1935
session of INC in 1937. Ans. (a) : The All India Trade Union Congress was
J.B. Kriplani had, presided over the Meerut session of established on 31 October 1920. Lala Lajpat Rai was
INC in 1946. elected as the first president of AITUC and Diwan
Chaman Lal as the first general Secretary.
29. When was Kunwar Singh, leader of revolt of
33. Name the Deoband Scholar who played
1857 in Bihar, breathed his last
significant role in the Freedom Movement.
(a) 26th April, 1858 (b) 17th June, 1858 (a) Abul Kalam Azad
(c) 9th May, 1858 (d) 20the June, 1858 (b) Mohammad Ali Jinnah
Ans. (a) : Kunwar Singh was a notable freedom fighter (c) Badruddin Tyabji
during the revolt of 1857. He belonged to a royal (d) Chirag Ali
Ujjainiya Rajput house of Jagdishpur, currently a part of Ans. (a) : The Deoband Movement was begun at the
Bhojpur district, Bihar. He was the chief organiser of Darul Uloom, Deoband in Saharanpur district in 1866
fight against the British in Bihar. On 22 and 23 April by Muhammad Qasim Nanautavi and Rashid Ahmad
1858 being injured he fought bravely against the British Gangohi to train religious leaders for the Muslim
Army & with the help of his army drove away the community, Maulana Abul Kalam Azad was the most
British army, brought down the union Jack from prominent of the tradtional Muslim Scholars of the
Deoband School who held the view that there was no
Jagdishpur fort and hoisted his flag. He returned to his
conflict between Muslims and nationalism.
palace on 23 April 1858 and soon died on 26 April
1858. The great revolt of 1857 occurred amidst the 34. Who was Shardamani?
(a) Wife of Raja Ram Mohan Roy
report of introduction of greased cartridges to be used in
(b) Wife of Ramakrishna Paramahansa
Enfield rifles in December 1856. (c) Mother of Vivekananda
30. Name of the revolutionary from Bihar who (d) Daughter of Keshab Chandra Sen
attended the meeting of Hindustan Socialist Ans. (b) : Shardamani Mukhopadhyay is also known as
Republican Army in September, 1928. Sarda Devi. She was married to Ramkrishna
(a) Phanindra Nath Ghosh Paramhansa at the early age of five in May 1859.
47th BPSC (Pre) Exam. 2004-05 133 YCT
35. Who founded the 'Amrit Bazar Patrika'? 40. Which state in India produces maximum mica?
(a) Girish Chandra Ghosh (a) Madhya Pradesh (b) Bihar
(b) Harish Chandra Mukherjee (c) Odisha (d) Jammu & Kashmir
(c) SN Banerjee Ans. (b) : When the question was asked, Bihar was
largest Mica Producing state of India.
(d) Sisir Kumar Ghosh At present Andhra Pradesh is the largest producer of
Ans. (d) : Amrit Bazar Patrika was a Bengali daily mica in India. In Andhra Pradesh, Nellore district
newspaper. It was started by Sisir Kumar Ghosh, produces the best quality mica.
Motilal Ghosh in 1868. It was used as tool to expose 41. Which state has the lowest per capita income in
British agent's Cruel deeds of tyranny. India at current price?
36. Who said that, "Congress Movement was (a) Bihar (b) Odisha
neither inspired by the people nor devised or (c) Rajasthan (d) Gujarat
Planned by them"? Ans. (a) : Bihar has lowest per capita income in India at
current price while Goa has the highest per capita
(a) Lord Dufferin (b) Sir Syed Ahmed income at current Price.
(c) Lord Curzon (d) Lala Lajpat Rai Per capita income is measure of the amount of income
Ans. (d) : Lala Lajpat Rai said that, "Congress earned per person in a nation or geographic region.
Movement was neither inspired by the people nor 42. Which state has the lowest literacy rate
devised or planned by them. according to 2001 census report?
37. Who propounded the doctrine of 'Passive (a) Arunachal Pradesh (b) Madhya Pradesh
(c) Bihar (d) Haryana
Resistance'?
Ans. (c) : As per census 2001 Bihar has lowest literacy
(a) Mahatma Gandhi
rate 47.53%, along with the lowest Literacy rate for
(b) Bipin Chandra Pal males at 60.32% and for females, at 33.57%.
(c) Bal Gangadhar Tilak 43. According to year 2002, Birth Rate of Bihar
(d) Aurobindo Ghosh (per 1000) is
Ans. (d) : The theory of 'Passive Resistance' was (a) 7.9 (b) 30.9
propounded by Aurobindo Ghosh, who is known as (c) 20.9 (d) 26.9
father of Indian Extremism. In a place of "Prayer", Ans. (b) : As per year 2002 Birth rate of Bihar was
Petition", 'Protest" and "Please", he advocated the more 30.9. In 2018, Birth rate for Bihar was 26.8 births per
radical policy of boycott. He wrote two books i.e. 'New thousand inhabitants.
Lamp for old' and the other being, "Savitri". 44. Percentage share of Agricultural labour in total
workers of Bihar is
38. Who played an important role in the signing of (a) 40.18 (b) 48.18
Gandhi-Irwin Pact? (c) 29.17 (d) 46.18
(a) Motilal Nehru Ans. (b) : When the question was asked, the percentage
(b) Madan Mohan Malaviya share of Agricultural labour in total workers of Bihar
(c) Tej Bahadur Sapru was 48.18
(d) Chintamani 45. Coal Reserves in Bihar on January 1st, 2003 is
(in million tonne)
Ans. (c) : Tej Bahadur Sapru was the member of Indian (a) 260 (b) 360
Liberal Party. He along with other members of Indian (c) 160 (d) 210
liberal party had apealed to Gandhi Ji to talk with the Ans. (c) : Coal Reserves in Bihar on 1 January 2003
Viceroy. Gandhi and Irwin reached a compromise was 160 million tonnes. At present, Jharkhand has
which was known as the Gandhi - Irwin Pact (the Delhi highest coal reserve followed by Odisha &
Pact). This Pact was signed on 5 March, 1931, as a Chhattisgarh.
result of this pact Indian National Congress had 46. Which one of the following statements about
participated in the second Round Table conference irrigation in Bihar is not correct?
which was held in 1931 and during September - (a) Of the total area in Bihar nearly 46% is
December. irrigated
39. Which region of India was known for the (b) Of the total irrigated area in Bihar nearly 30%
is irrigated by canals and 39% by tubewells
cultivation of poppy plant during British rule?
(c) the maximum area irrigated by canals is
(a) Bihar (b) South India
found in Bhojpur, Aurangabad, West
(c) Gujarat (d) Assam Champaran, Rohtas and Munger districts
Ans. (a) : Bihar was known for the cultivation of poppy (d) The maximum area irrigated by tubewells is
plant during British rule. British companies extracted found in Samastipur, Sitamarhi, Begusarai,
huge profit by smuggling opium trade out of Calcutta to Muzaffarpur, Gopalganj and Khagaria
China. districts
47th BPSC (Pre) Exam. 2004-05 134 YCT
Ans. (b): When the question was asked, of the total Ans. (c) : At the time when the question was asked,
irrigated area in Bihar nearly 27% was irrigated by Agriculture sector contributes nearly 47% to the Bihar's
canals and 6% by tube wells. Rest all the statements are GDP. whereas about 47 Lakh acres of land in Bihar get
correct. At present , Bhojpur district of Bihar has the flooded every year, while 10 lakh acres remain
maximum number of canal irrigation facility. permanently waterlogged and silk industry in Bihar had
47. Nearly 55% of the total population of Bihar provided self employment to over 150000 people.
survive in absolute poverty because.
50. Rivers belonging to which one of the following
I. Bihar lacks in natural resources, both
groups form the boundary between Bihar and
existing and potential.
Uttar Pradesh?
II. Bihar has the fastest total fertility rate in
India. (a) Kamla, Sone and Baghmati
III. Bihar has very unfavourable agro-climatic (b) Budi Gandak, Kosi and Ganga
conditions. (c) Karmanasha, Gandak and Ghaghra
IV. Bihar lacks in quality infrastructure and (d) Narth Koel, Ajay and Punpun
business friendly environment. Ans. (c) : Karmanasha, Gandak and Ghaghra rivers
Which of these reasons are correct? forms the boundary between Bihar and Uttar Pradesh.
(a) I and II (b) II and III The river Karmanasha originates from the Kaimur range
(c) II and IV (d) I, III and IV (Sarodgarh) in Mirzapur. It makes boundary of Bihar &
Ans. (d) : About 55% of total population of Bihar U.P. It joins Ganga near Chausa in Bihar. Gandak river
survive in absolute poverty because Bihar lacks in originates from Himalaya in Nepal & Joins the Ganga at
natural resources after the bifurcation of state in both
Sonpur, east of Patna. The river Ghaghara originates
existing and potential. This state has very unfavorable
agro climatic condition mainly in northern Bihar due to from the Mapcha chung Glacier Situated in the plateau
devastating flood every year . Bihar lacks in quality of Tibet. It joins Ganga near Chapra.
Infrastructure and business friendly Environment. As 51. Correlate List I (Characteristics of national
per census 2001 the total fertility rate in Bihar was not Highways) with List II (Number of National
the fastest, Hence option 'd' is correct. Highways) and select the correct answer using
48. Which one of the following groups of districts the codes given below the lists.
in Bihar indicates the correct descending order List-I List-II
of population density as per the 2001 census?
(Characteristics of (Numbers of National
(a) Darbhanga, Samastipur, Siwan, Saran
national Highway) Highways)
(b) Siwan, Saran, Darbhanga, Samastipur
(c) Saran, Darbhanga, Samastipur, Siwan A. National 1. National Highway
(d) Darbhanga, Saran, Samastipur, Siwan Highway that No. 12
passes Through
Ans. (*) : As per census 2001, Darbhanga district
density was at 1446 people per sq.km. Saran district Bihar
density was at 1230 people per sq.km. Samastipur B. National 2. National Highway
district density was of 1169 people per sq.km. and Highway that No. 87
Siwan district density was at 1223 people per sq.km. does not pass
The correct descending order of population density as through Bihar
per census 2001 will be Darbhanga, Saran, Siwan,
C. National 3. National Highway
Samstipur hence none of the above option is correct.
Highway having No. 2
49. Consider the following statements regarding
the vivid geographical aspects of Bihar maximum
I. Agriculture contribute nearly 67% to Bihar's length in Bihar
domestic product. D. Proposed 4. National Highway
II. Silk industry in Bihar provides self- National No. 31
employment to over 150000 people. Highway not to
III. About 47 lakh acres of land in Bihar get serve Bihar
flooded every year, while 10 lakh acres remain Codes
permanently waterlogged. A B C D
IV. 85% to 90% of Bihar's rural households (a) 3 1 4 2
own less than 5 acres of land each.
(b) 4 2 3 1
Which of these statements are correct?
(a) II and IV (b) I and II (c) 1 3 2 4
(c) II and III (d) III and IV (d) 2 4 1 3
47th BPSC (Pre) Exam. 2004-05 135 YCT
Ans. (a) : correct match are as follows :- Ans. (b) : Leader newspaper was published by a
List-I List-II moderate nationalist Pt. Madan Mohan Malviya from
(Characteristics of (Numbers of National Prayagraj. This newspaper was essentially the
mouthpiece of the policies of liberals. Young India was
national Highway) Highways)
a weekly journal, published by Mahatma Gandhi. New
A. National Highway 1. National Highway India newspaper was published by Annie Besant.
that passes through No. 2
54. Who appoints the Governor of Jammu &
Bihar
Kashmir?
B. National Highway 2. National Highway (a) The Chief Minister of Jammu & Kashmir
that does not pass No. 12 (b) The Chief Justice of High Court of Jammu &
through Bihar Kashmir
C. National Highway 3. National Highway (c) The Prime Minister of India
having maximum No. 31 (d) The President of India
length in Bihar Ans. (d) : The President of India on the advice of the
D. Proposed National 4. National Highway Central Government appoints the Governor of Jammu
Highway not to No. 87 & Kashmir With the revoking of Article 370, Jammu &
serve Bihar Kashmir is divided into two union Territories, Jammu &
52. Correlate List I (Agro-climatic) with List II Kashmir and Ladakh. At present Manoj Sinha is the
(Percentage of Area) and select the correct Lieutenant Governor of Jammu & Kashmir.
answer using the codes given below the lists. 55. The decision to conduct Panchayat Elections is
List-I List-II taken by which of the following?
(a) The Central Government
(Agro-climatic) (Percentage of Bihar's
Area) (b) The State Government
(c) The District Judge
A. North and 1. 29.87%
(d) The Commission
North Eastern
Bihar Plains Ans. (b) : The superintendence, direction and control
election of the preparation of electoral rolls and the
B. South Bihar 2. 23.71% conduct of all elections to the panchayats shall be
Plains vested in the State Election commission. The decision
C. Chhota Nagpur 3. 26.14% to conduct Panchayat Election is taken by the State
Plateau Government.
D. Chhota Nagpur 4. 20.28% 56. Which of the following Articles of the
Hills Constitution of India is related to liberty of
press?
Codes
(a) Article 19 (b) Article 20
A B C D
(c) Article 21 (d) Article 22
(a) 3 4 1 2
Ans. (a) : Freedom of the press is implicit under the
(b) 1 2 3 4 Article 19(1)(a) of the Indian constitution, which
(c) 2 1 4 3 provides that every citizen has the right to express his
(d) 4 3 2 1 view, opinions, belief and convictions freely by word of
mouth, writing, printing, picturing or in any other
manner.
Ans. (b) : The correct match are as :-
57. Which of the following Constitutional
List-I List-II Amendment Acts was said to be a Mini-
(Agro-climatic) (Percentage of Constitution?
Bihar's Area) (a) 42nd (b) 44th
th
A. North and North Eastern 1. 29.87% (c) 46 (d) 50th
Bihar Plains Ans. (a) : The 42nd Amendment Act, 1976 is the most
B. South Bihar Plains 2. 23.71% comprehensive amendment made so far to the
constitution of India. This amendment is known as
C. Chhota Nagpur Plateau 3. 26.14% "Mini constitution".
D. Chhota Nagpur Hills 4. 20.28% 58. The statewise allocation of seats in Lok Sabha
53. Which of the following papers was essentially is based on the 1971 census. Upto which year
the Mouthpiece of the policies of liberals? does this remain intact?
(a) New India (b) Leader (a) 2031 (b) 2026
(c) Young India (d) Free Press Journal (c) 2021 (d) 2011
47th BPSC (Pre) Exam. 2004-05 136 YCT
Ans. (b) : The 42nd Amendment Act of 1976 Froze the 63. Who was the President of the Committee on
allocation of seats in Lok Sabha till year 2000 at the whose recommendation was established
1971 level. This ban on readjustment was extended for Panchayati Raj in India?
another 25 years that is upto 2026 by the 84th (a) Ashok Mehta (b) Dr. Iqbal Narayan
Amendment Act of 2001, with the same objective of (c) Balwant Rai Mehta (d) Jeevraj Mehta
encouraging population limiting measures. Ans. (c) : Panchayati Raj was constitutionalized
59. The Panchayati Raj System was first started in through the 73rd constitutional Amendment Act of 1992.
India in the States of Rajasthan and In January 1957, the Government of India appointed
(a) Haryana (b) Gujarat Balwant Rai Mehta committee. The committee
(c) Uttar Pradesh (d) Andhra Pradesh submitted its report in November 1957 and
Ans. (d) : Rajasthan was the first state to establish recommended the establishment of a three tier
Panchayati Raj. This scheme was inaugurated on 2nd Panchayati Raj System. L.M. Singhvi committee in
October 1959, in Nagaur district of Rajasthan and was 1986 had suggested that the Panchayti Raj institution
followed by Andhra Pradesh, which adopted the should be constitutionally recognized, protected and
Panchayati Raj System in 1959. Thereafter, most of the preserved.
States adopted the system. 64. Which state has lower literacy rate than the
60. How many members are nominated by the country's literacy rate according to 2001
President in the Rajya Sabha? census?
(a) 2 (b) 10 (a) Maharashtra (b) Gujarat
(c) 12 (d) 15 (c) Odisha (d) West Bengal
Ans. (c) : The maximum Strength of Rajya Sabha is Ans. (c) : As per census 2001, Odisha had lower
fixed 250, out of which 238 are to be the representatives literacy rate than the country's literacy rate. As per
of the states and Union Territories (elected indirectly) census 2011, The literacy rate in country is 74.04%. As
and 12 are nominated by the President of India. The per census 2011, total ten states have literacy below the
President nominates 12 members from people who have national average of literacy. These states are : - Bihar
special knowledge or practical experience in art, Arunchal Pradesh, Jharkhand, Rajasthan, Andhra
Literature, Science and Social services.
Pradesh, Uttar Pradesh, Madhya Pradesh, Chhattisgarh,
61. By which of the following can the President of Assam & Odisha.
India be impeached?
(a) By the Lok Sabha 65. When development in economy takes place, the
(b) By the Parliament share of tertiary sector in National Income?
(c) By the Chief Justice of India (a) First falls and then rises
(d) By the Prime Minister (b) First rises and then falls
Ans. (b) : Under Article 61 of Indian constitution, The (c) Keeps on increasing
President can be removed from office by a process of (d) Remains constant
impeachment for violation of the Constitution. The Ans. (c) : The composition of national income keeps
impeachment charges can be initiated by either House changing with growth in economy when development in
of Parliament. These charges should be signed by one - the economy takes place, the share of tertiary sector in
fourth members of the House, and a 14 days notice National Income keeps on increasing because tertiary
should be given to the President. After the impeachment sector is involved into services within outside the
resolution is passed by a majority of two-thirds of total country.
member of that House, it is sent to other House. If the 66. The outline of Second Five Year Plan was made
other house also sustains the charges and passes the by
impeachment then the president stands removed from (a) V N Gadgil (b) V K R V Rao
his office. (c) P C Mahalnobis (d) C N Vakil
62. Which of the following subjects is in the
Ans. (c) : Second five year plan (1956 - 61) main goal
Concurrent list ?
was to put India on a fast track to industrilization. This
(a) Agriculture (b) Education
plan was drafted by Statistician P.C. Mahalanobis &
(c) Police (d) Defence
also called Mahalanobis plan. He gave the highest
Ans. (b) :The 42nd Amendment Act of 1976 transferred priority to strengthening the industrial base of the
five subjects to concurrent list from state list that is (a) economy. In the second five year plan, steel mills were
education (b) forests (c) weights and measures, (d) built in Bhilai, Durgapur and Rourkela.
protection of wild animals and birds, and (e)
administration of justice. At present, union list has 100 67. NABARD was established in the
subjects, State lists has 61 subjects and Concurrent list (a) Fourth five Year Plan
has 52 subjects. Agriculture, police, prisons are the (b) Fifth Five Year Plan
subjects of states list. Defense, banking, insurance etc. (c) Sixth Five Year Plan
are the subjects of union list. (d) Eighth Five Year Plan

47th BPSC (Pre) Exam. 2004-05 137 YCT


Ans. (c) : National Bank for Agricultrure and Rural Ans. (d) : National Income estimate in India are
Development (NABARD) is an apex financial prepared by central statistical organisation. CSO is
institution that provides finance for Agriculture and responsible for coordination of Statistical activities in
rural development. Its headquarter is at Mumbai. This India, & evolving and maintaining statistical standard.
institution was set up on the recommendation of the B.
73. Who is called the 'Father of Indian Economic
Sivaraman committee on 12 July 1982 under the sixth
Reform'?
five year plan.
(a) Jawaharlal Nehru (b) Indira Gandhi
68. The main source of National Income in India is
(c) Manmohan Singh (d) PV Narasimha Rao
(a) Service Sector (b) Agriculture
(c) Industrial Sector (d) Trade sector Ans. (d) : P. V. Narasimha Rao who was ninth Prime
Minister of India is called as the Father of Indian
Ans. (a) : The services sector is the largest sector of
India. Gross Value Added (GVA) at current prices for Economic Reform.
the services sector is estimated at 96.54 lakh crore INR 74. Finance Minister of India is
in 2020-21. The services sector accounts for 53.89% of (a) Jaswant Singh (b) Yaswant Sinha
total India's GVA of 179.15 lakh crore Indian rupees. (c) P Chidambaram (d) None of these
With GVA of Rs. 46.44 lakh crore, the Industry sector Ans. (c) : When the question was asked, P
contributes 25.92%. While Agriculture and allied sector
Chitambaram was the Finance Minister of India. At
share is 20.19%.
present, Nirmala Sitharaman is the Finance Minister of
69. Economic Planning is in India.
(a) Union List
75. Which of the following direct taxes gives
(b) State List
maximum net revenue to the government?
(c) Concurrent
(a) Corporate Tax (b) Income Tax
(d) not any specified list
(c) Wealth Tax (d) Gift Tax
Ans. (c) : Economic Planning is the subject of
concurrent List under the seventh schedule of the Ans. (a) : At the time when the question was asked,
constitution of India. Corporate tax was the single largest source of income to
70. According to 2001 census the State having the the government of India. However As per Union Budget
highest urban population is 2022 - 23, both Corporate tax and Income tax
(a) Uttar Pradesh (b) Maharashtra contributed 15 paise to each rupee collection.
(c) Tamil Nadu (d) Kerala 76. From the following which one is not a tool of
Ans. (b) : As per census 2001, Maharashtra had the Fiscal Policy?
highest number of urban population. As per census (a) Taxation (b) Public Expenditure
2011, in terms of the absolute number of persons living (c) Interest Rate (d) Public Debt
in Urban areas, Maharashtra continues to lead with 50.8 Ans. (c) : Interest rate is not a tool of fiscal policy
million people. where as taxes, public expenditure, public debt and a
71. When was the Planning Commission nation's budget are tools of fiscal policy.
established? 77. Which is the "Year of Great Divide" in the
(a) 10th March, 1950 history of population growth in India?
(b) 15th March, 1950
(a) 1951 (b) 1991
(c) 16th March, 1951
(c) 2001 (d) 1921
(d) 20th march, 1950
Ans. (d) : The year 1921 is often referred to as the "year
Ans. (b) : Planning commission was established on 15th
of the Great Divide" with regard to population growth
March, 1950 by an executive resolution of the
Government of India on the recommendation of the in India because from that year onwards the population
Advisory Planning Board constituted in 1946, Under the of India has been significantly rising every decade.
chairmanship of K.C. Neogi. On 1 January 2015, the 78. Which state ensures the maximum production
NITI Aayog was established as the successor to the of Coffee in India?
planning commission. (a) Maharashtra (b) Karnataka
72. National income estimates in India are (c) Tamil Nadu (d) Kerala
prepared by Ans. (b) : Karnataka is the largest producer of coffee in
(a) Planning Commission India. The two varieties of coffee grown in India are
(b) RBI Arabica and Robusta. In India coffee is traditionally
(c) Finance Ministry grown in Western Ghats spread over Karnataka, Kerala
(d) Central Statistical Organisation and Tamil Nadu.

47th BPSC (Pre) Exam. 2004-05 138 YCT


79. Consider the following statements and select Ans. (a) : Himalaya is a tertiary folded mountain range.
the correct answer using the codes given below. It was formed as a result of the collision between the
Assertion (A) : until the end of eighties of the Indian plate and Eurasian plate. Himalayan Mountain
last century, the Indian industries showed a have formed of a great geosyncline called the Tethys
clear attitude of structural shift from Sea and that uplift has taken place in different phases
consumer goods to basic and capital goods hence, Statement(iv) is false and rest all the other
industries. Statements are correct.
Reason (R) : The demand for consumer goods 82. Correlate List I (Railways) with List II
declined during the period. (Connecting Places) and select the correct
Codes : answer using the codes given below the lists.
(a) Both A and R are true, but R is the correct List-I List-II
explanation of A (Railways) (Connecting places)
(b) Both A and R are true, but R is not the correct
A. European 1. Paris to Istanbul
explanation of A
trans-
(c) A is true, but R is False continental
(d) A is false, but R is true Railways
Ans. (c) : In the late eighties, Indian industries had B. Trans-Andean 2. Leningrad to
shown a clear trend of structural shifting from consumer Railway Vladivostok
goods, industries to basic and capital goods industries,
C. Trans-Siberian 3. Buenos Aires to
but during this period the demand for consumer goods
Railway Valparasio
had not decreased. Hence Assertion is true but Reason
is false. D. Orient Express 4. Paris to Warsaw
80. Some iron and steel plants have been planned Codes
along the Western Coast of India. What is the A B C D
major reason for this locational shift of this (a) 3 2 4 1
industry? (b) 4 3 2 1
(a) Increased nuclear power generation in the (c) 2 3 1 4
Western Coastal region (d) 1 4 3 2
(b) Occurrence of high grade iron ore deposits in
Goa and parts of Madhya Pradesh and the Ans. (b) : The correct matches are as :-
comparative ease of exporting steel from here
List-I List-II
(c) Decline in international demand for Indian
(Railways) (Connecting places)
iron ore from the Western Coastal region
(d) Adoption of sponge iron technology A. European trans- 1. Paris to
continental Railways Warsaw
Ans. (b) : Some Iron and Steel plants have been
established mainly along the western coast of India due B. Trans-Andean 2. Buenos Aires
to the availability of high grade iron are deposits in Goa Railway to Valparasio
and Some part of Madhya Pradesh in India and the C. Trans-Siberian 3. Leningrad to
comparative ease of exporting steel from this region. Railway Vladivostok
81. Consider the following statements regarding D. Orient Express 4. Paris to
formation of landforms in India. Istanbul
I. Structurally, the Meghalaya plateau is an 83. Which one of the following statements about
extended part of the Deccan plateau. the economy of selected countries is not
II. The Valley of Kashmir was formed in a correct?
synclinorium (a) USA accounts for more than 50% of the
III. The Gangetic plain was formed in a fore word's production of maize but only 3% of
deep. the word's export
IV. The Himalayas originated as a result of (b) No country produces and exports more wool
triangular convergence of the Indian plate, the than Australia
European plate and the Chinese plate. (c) India figures among the leading producers of
Which of these statements are correct? dairy products in the world
(a) I, II and III (d) With 4% of the world's sheep population,
(b) I, III and IV New Zealand accounts for two-thirds of the
(c) I and III mutton and one-sixth of the wool exports of
(d) It and IV the world
47th BPSC (Pre) Exam. 2004-05 139 YCT
Ans. (d) : When the question was asked, In the total Ans. (c) : In 2004, about 3 Lakh people died due to
production, corn is the largest crop of USA. USA Tsunami. Hence option C is not correct. Rest all the
accounts for more than 50% of the world's production other options are correct. Malabar coast was most
of Maize but only 3% of the world's export. Australia is affected by Tsunami in 2004.
the world's largest wool producer and exporter,
producing around 25% of greasy wool to the world 87. Which one of the following statements about
market. India is the world's largest milk producer, with energy production and consumption in India is
22% of global production. One third of the world's not correct?
sheeps are found in Australia, which is largest in the (a) During the last decade energy production in
world. The Murray Darling Basin is very suitable for India has shown a declining trend
sheep farming in Australia. Australia produces about (b) The per capita energy consumption in the
one quarter of the world's wool. Since 2017 Australia world is lowest in India
has overtaken New Zealand as the world's top supplier (c) The non-conventional sources of energy
of Sheepmeat to global market. contribute less than 1% of the total
84. A ridge, 64000 km long and 2000 km to 2400 commercial energy produced in India
km wide, runs downs the middle of the North (d) Industry is the major energy consuming
and the South Atlantic Ocean basins, into the sector in India
Indian Ocean basin, then passes between
Australia and Antarctica to the South Pacific Ans. (d) : The major energy consuming sector in India
Ocean basin. The ridge is ? is Industry (about 58%). There has been a gradual
(a) Socotra-Lakshadweep-Chagos ridge increase in energy production in India over the past
(b) Pacific-Challenger ridge decades. At present India is 5th largest energy consumer
(c) Dolphin-Challenger ridge in the world. The non conventional source of energy
(d) Mid-Oceanic ridge contributes more than 1% of the total commercial
energy produced in India. Hence only option (d) is
Ans. (d) : Mid - Oceanic Ridge is 64000 km long and
correct.
2000 km to 2400 km width passing through North &
South Atlantic Oceanic basins enters into the South 88. What is true about the Second Green
pacific Oceanic basin through India Ocean and then Revolution in India?
through the middle of Australia & Antarctica. I. It aims at further increasing production of
85. Israel has common borders with wheat and rice in areas already benefited from
(a) Lebanon, Syria, Jordan and Egypt Green Revolution.
(b) Lebanon, Syria, Turkey and Jordan II. It aims at extending seed-water-fertilizer
(c) Lebanon, Turkey, Jordan and Egypt technology to areas which hitherto could not
(d) Lebanon, Syria, Iraq and Egypt benefit from Green Revolution.
Ans. (a) : Israel has border with lebanon on the north, III. It aims at increasing yields of crops other
Syria and Jordan on East and Egypt on the South side. than those used for Green Revolution in the
The Mediterranean Sea is the western border of Israel. beginning.
86. Which one of the following statements about IV. It aims at integrating cropping with animal
the 2004 Indian ocean earthquake and the husbandry, social forestry and fishing.
resulting Tsunami is not correct? Select the correct answer from the codes given
(a) The earthquake originated due to slipping of below.
about 1200 km of fault line by 15 m along the (a) I and II (b) II and III
subduction zone where the Indian plate (c) II and IV (d) I and IV
subducts the Burma plate at the Sunda Trench
Ans. (c) : The second green revolution aims at creating
(b) The resulting Tsunami devastated the shores
sustainable agriculture by leveraging advancement in
ranging from the coast of Indonesia as far as
technology. Second green revolution aims to supply
the East Coast of Africa, some 8500 km away
seed, water, fertilizer technology to areas that have not
from the epicentre
yet benefited from the Green revolution. It also aims at
(c) As per the current estimates the earth quake-
integrating cropping with animal husbandry, social
generated Tsunami killed more than 50 lakh
forestry and fishing.
people, in addition to unaccounted dead
bodies swept out to sea 89. Who represented India in the Second ASEAN
(d) Bangladesh had very few casualities because Summit held in October, 2003?
the quake affected fault line was in nearly (a) The Prime Minister
North-South orientation, the greatest strength (b) The President
of the Tsunami waves was in an East-West (c) The Vice-President
direction (d) None of these
47th BPSC (Pre) Exam. 2004-05 140 YCT
Ans. (*) : Second ASEAN summit was held in Kuala 96. The Prime Minister of India is the head of the
lumpur on 5 August 1977. Hence question is not (a) State Government (b) Central
appropriate. Government
90. The delegation of both the Houses of Indian (c) Both (a) and (b) (d) None of these
Parliament led by the Speaker which visited Ans. (b) : Prime Minister of India is the head of the
China in early January, 2003 consisted of Central Government. As India follows a Parliamentary
(a) 22 members (b) 12 members system of government modelled after the westminster
(c) 16 members (d) None of these system, most of the executive powers are exercised by
Ans. (a) : 22 member delegation of both Houses of the Prime Minister. He acts as an advisor to the
Indian Parliament led by the then speaker Murali President and is the leader of the council of Ministers.
Manohar Joshi visited China in January 2003. 97. Goa Ministry was dismissed by the
91. The Group-7 rich countries meet at Deauville (a) Governor (b) Chief Minister
in France was held in (c) President of India (d) None of these
(a) March, 2003 (b) April, 2003 Ans. (a) : Goa Ministry was dismissed by Governor.
(c) May, 2003 (d) None of these The Governor dissolves the state legislative assembly in
Ans. (d) : The G7 Summit was held in Deauville in case of breakdown of constitutional machinery in a
France in June 2003. Group of seven (G7) is an inter state. In a parliamentary form of government, the
governmental organisation which was formed in 1975. Governor has the right to dismiss the state cabinet.
UK, Canada, France, Germany, Italy, Japan and the US 98. Which of the following states did not go to polls
are G7 countries. The 48th Summit will be held from 26 in February, 2005?
to 28 June 2022 in Germany. (a) Bihar (b) Haryana
92. Nanavati Commission was set up in the year (c) Jharkhand (d) Punjab
(a) 1984 (b) 1994 Ans. (d) : Among the above given options, Assembly
(c) 1999 (d) 2001 Election was not held in Punjab in February 2005.
Ans. (*) : Nanavati Commission was appointed by 99. State Assembly Elections of February, 2005
National Democratic Alliance (NDA) government in were held in
May 2000, to investigate the "Killing of innocent sikhs" (a) one phase (b) four phases
during 1984 anti-sikh riots, also Nanavati Commission (c) two phases (d) None of these
was setup in 2002 following the burning of Sabarmati Ans. (d) : In February 2005, Assembly Elections were
Express near Godhra Station on 27 February 2002 in held in three phase on 3, 15 and 23 February 2005 in the
which 59 died. State of Haryana, Bihar and Jharkhand.
93. Which state had a tussle with Election 100. Final phase of elections in February, 2005 in
Commission on the counting date in February, Bihar was for
2005 elections? (a) 93 seats (b) 83 seats
(a) Bihar (b) Punjab (c) 73 seats (d) None of these
(c) Jharkhand (d) None of these
Ans. (a) : In February 2005, the last phase of election
Ans. (d) : Haryana Government in Assembly Election was held for 93 Seats in Bihar. At Present Nitish Kumar
in February 2005 had dispute with Election commission is the Chief Minister of Bihar.
on the date of counting of Votes.
101. Who is the Chief Election Commissioner of
94. The Governor of Jharkhand is India?
(a) Buta Singh (b) Syed Sibtey Razi (a) TM Longdoh (b) MS Gill
(c) Ved Marwah (d) TV Rajeshwar (c) SL Sakdhar (d) None of these
Ans. (b) : When the question was asked Syed Sibtey Ans. (d) : Article 324 of the Indian constitution is
Razi was the Governor of Jharkhand (2004 - 2009) At related to the Election commission. When the question
present, Ramesh Bais is the 10th Governor of Jharkhand was asked BB Tandon was the 14th Chief Election
since 14th July 2021. commissioner of India from May 2005 to June 2006. At
95. Supreme Leader of China is present Rajiv Kumar is the 25th Chief Election
(a) Jiang Zemin (b) Hu Jintao Commissioner of India.
(c) Mao Zedong (d) None of these 102. Justice UC Banerjee Commission was
Ans. (b) : When question was asked, Hu Jintao was the constituted to
Supreme leader of China. He served as General (a) Probe the 2002 Godhra incident
Secretary of the Chinese Communist party from 2002 to (b) Probe Sikh riots
2012. At present Xi Jin Ping is the paramount leader in (c) Probe fodder scam
China. (d) None of the above
47th BPSC (Pre) Exam. 2004-05 141 YCT
Ans. (a) : In September 2004, The Ministry of Railway Ans. (b) : The correct matches are as :-
had appointed a committee headed by retired Supreme List-I List-II
court Judge UC Banerjee to Probe the Ghodhra train (Natural Hazards) (Regions)
burning incident on 27 February 2002.
A. Floods 1. Plains of Uttar Pradesh
103. Head of which country was the chief guest to and Bihar
grace the Republic Day celebration this year
B. Earthquakes 2. Himalayan Foothill
(2005)?
Zone
(a) China (b) Bhutan
C. Droughts 3. Mid-Eastern India
(c) Nepal (d) Sri Lanka
D. Cyclones 4. Jharkhand and Northern
Ans. (b) : In 2005, king of Bhutan Jigme Sinhhye
Wangchuck was the Chief guest on the occasion of Orissa
Republic Day celebration. In 2020 Brazilian President 106. Consider the following statements about the
Jair Bolsonaro was the Chief guest on the occasion of Population characteristics of India.
Republic Day in 2020. In 2021 and 2022 there were no I. During 1991-2001 decade, the population
chief guest due to Covid-19 Pandemic. grew at the rate of nearly 21%
104. Consider the following statements about II. In 2001 the gap between male and female
India's export-import trade. literacy has widened compared to 1991.
I. The share of textile sector in India's export is III. As per the 2001 census there are 35 large
21% and jewellery and ornaments 17%. cities in the country which accommodate nearly
II. Western Europe and North America 48% of total urban population.
together account for 53% of India's exports IV. The 1991 census reflected a decline in the
and 40% of its imports. number of unproductive consumers.
III. Latin America, sub-Saharan African and Codes
Middle East countries offer the most potential (a) I and II (b) II and III
markets for Indian export trade. (c) I and IV (d) II, III and IV
IV. India's share in world exports is a little Ans. (c) : During the 1991 - 2001 decade the population
over 2%. growth rate was 21.54%. In 2001 the gap between male
Which of these statements are correct? and female literacy has decreased as compared to 1991.
(a) I and II (b) II, III and IV The gap between male and female literacy was 24.8% in
(c) III and IV (d) I, II and III 1991 which decreased to 21.6% in 2001. As per census
2001, there were 35 large cities in country which
Ans. (d) : When the question was asked, India's share in
the world exports was not more than 2%. So statement accommodate nearly 48% of Urban Population. The
(IV) is incorrect and rest all other statements are correct. 1991 census reflected a decline in the number of
Unproductive consumers. Hence correct option will be
India's share of merchandise exports is about 17% of the
total global export in 2019. The correct answer for this (c).
question will be option (d). 107. The function of bureaucracy is
105. Correlate List I (Natural Hazards) with List II (a) to implement the government policies
(Regions) and select the correct answer using (b) to run administration
the codes given below the lists. (c) to help the ministers
List-I List-II (d) All of the above
(Natural Hazards) (Regions) Ans. (d) : Bureaucracy is a way of administratively
A. Floods 1. Himalayan organizing large number of people who need to work
Foothill Zone together. The function of Bureaucracy is to implement
government policies, to observe the law and decisions
B. Earthquakes 2. Jharkhand and made by elected officials and put them into practice, to
Northern Orissa help the minister and to run administration etc.
C. Droughts 3. Plains of Uttar 108. The Speaker of the Lok Sabha is
Pradesh and Bihar
(a) Shiv Raj Patil (b) Somnath Chatterjee
D. Cyclones 4. Mid-Eastern India (c) Manohar Joshi (d) None of these
Codes Ans. (b) : When the question was asked, Somnath
A B C D Chatterjee was the speaker of 14th Lok Sabha from 2004
(a) 3 1 2 4 - 2009. At present, Om Birla is the speaker of 17th Lok
(b) 3 1 4 2 Sabha.
(c) 2 3 1 4 109. Elections for President of Afghanistan were
(d) 4 2 3 1 held on
47th BPSC (Pre) Exam. 2004-05 142 YCT
(a) October 9th, 2004 114. Aspirin is obtained from
(b) November 10th, 2004 (a) petroleum
(c) September 8th 2004 (b) earth
(d) None of these (c) tree
(d) chemical reaction of acids
Ans. (a) : Elections for the president of Afganistan were
Ans. (d) : Aspirin is one of the group of drugs called
held on 9 October 2004, in which Hamid Karzai had non-steroidal anti - inflammatory. It is used to reduce
won the election. He served as the President of pain and high temperature. Aspirin is prepared by
Afghhanistan from 22 December 2001 to 29 September chemical synthesis from salicylic acid through
2014. acetylation with acetic anhydride.
110. People get skin burns in a swimming pool due 115. For wheat cultivation which fertilizer is used?
to (a) Nitrogen (b) Potassium
(a) infrared ray (b) chlorine (c) Copper (d) Iron
(c) heat (d) ultraviolet ray Ans. (a) : Nitrogen fertilizer is used for wheat
cultivation. Granular Urea and Urea ammonium nitrate
Ans. (b) : Skin burn in Swimming pool is caused due to solution are the two most common nitrogen fertilizer
presence of Chlorine in water . used for wheat cultivation. Nitrogen is one of the
Chlorine is a chemical widely used in personal and Primary nutrients for plant growth.
public swimming pools. Chlorine is added to the water 116. Which metal can be used for producing
to kill germs. When it is added to swimming pool, it electricity?
forms a weak acid called hypochlorous acid that kills (a) Uranium (b) Iron
bacteria like Salmonella and E Coli, as well as germs (c) Copper (d) Aluminium
that cause viruses such as diarrhea & swimmer's ear. Ans. (a) : Nuclear energy is generated by the splitting
of Uranium atoms, this process is called fission. This
111. When electrical energy is converted into
generates heat to produce steam, which is used by a
motion turbine generator to produce electricity.
(a) there is no heat loss (b) heat loss is 50% 117. The time-period of a pendulum
(c) heat loss is 30% (d) heat loss is 80% (a) Depends on the mass
Ans. (a) : As per principle of Conservation of Energy (b) Depends on its length
"energy can neither be created nor be destroyed, but (c) Depends on time
only be transferred from one form to another. Hence (d) Depends on temperature
when electrical energy is converted into motion then Ans. (b) : The time period of simple pendulum depends
there is no loss of heat. the length of the pendulum.
Dynamo converts mechanical energy into electrical ℓ
energy. Time period of simple pendulum (T) = 2π
g
112. Fermentation of sugar leads to Where, ℓ = length of pendulum string
(a) Ethyl alcohol (b) Methyl alcohol g = acceleration due to gravity
(c) Acetic acid (d) Chlorophyll
So, ⇒ T ∝ ℓ
Ans. (a) : Fermentation is an anaerobic process in
118. Brass is made from
which energy can be released from glucose even if
(a) Copper and Nickel
Oxygen is not available. Fermentation of Sugar
(b) Nickel and Zinc
(glucose) leads to Ethyl alcohol (ethanol). Ethanol
(c) Magnesium and Zinc
Fermentation converts glucose (sugar) into cellular
(d) Copper and Zinc
energy, Producing ethanol and carbon dio-oxide as a by
- product. Ans. (d) : Brass is an alloy of Copper and Zinc. The
composition of Brass, generally is 66% Copper and
113. Myoglobin contains the metal 34% Zinc. Bronze is an alloy of 88% Copper and 12%
(a) Copper (b) Silver Tin.
(c) Gold (d) Iron 119. Which chemical substance is used for making
Ans. (d) : Hemoglobin is a red substance in the blood rat poison?
that carries oxygen from the lungs to tissues and organs (a) Ethyl alcohol (b) Methyl isocyanate
in the body and carries carbon dioxide back to the (c) Potassium cyanide (d) None of these
lungs. Iron is an essential element for blood production, Ans. (c) : Potassium cyanide is also known as Zinc
and about 70% of our body's iron is in red blood cells Phosphide. It is a highly toxic chemical which is used as
called hemoglobin. rodenticide a poison to kill the mouse.

47th BPSC (Pre) Exam. 2004-05 143 YCT


120. White light in a tube is produced by Ans. (a) : Winter solstice experiences the shortest day
(a) heating a copper wire and longest night of the year. It occurs on 23rd
(b) heating a fibre December. Winter Solstice occurs when North pole is
(c) exciting the atoms tilted farthest from the Sun. Whereas summer solstice
(d) oscillating the molecules bring the longest day in the Northern hemisphere. It
Ans. (b) : White light is emitted by a variety of sources occurs on 21st June.
such as tungsten lamps, which are frequently labeled On 21st March and 23rd September, the whole earth
incandescent because they radiate light when heated experiences equal day and equal night. This is called
filament by electrical energy. White light may also Equinox.
come from a fluorescent source in which the light is 126. The solar eclipse occurs when
generated as a result of electrical current travelling (a) Moon is in the centre
through a charge gas. (b) Earth is in the centre
121. Gold is dissolved in (c) Sun is in the centre
(a) sulphuric acid (d) Sun, Moon and Earth are in a straight line but
(b) nitric acid Jupiter is not on this line
(c) mixture of sulphuric and nitric acids Ans. (a) : A solar eclipse occurs when the moon comes
(d) hydrochloric acid in the between the earth and sun. Blocking the Sun's
Ans. (*) : Since Gold is a less reactive metal, Hence it light and casting a shadow over certain area on Earth.
can only be dissolved in Aqua regia. Aqua regia is a This can only happen when sun, moon and Earth are
mixture of concentrated Hydrochloric acid and nearly aligned in a straight line. Solar eclipse occurs on
concentrated Nitric acid in the ratio 3 : 1. a New moon Day. Whereas lunar Eclipse occurs when
122. Acid rain is caused by the earth comes in between the moon and sun.
(a) Industries (b) Petrol 127. The high tide in the ocean is caused by
(c) Burning coal (d) Wood (a) Earthquake (b) Sun
Ans. (a) : Acid rain is defined as falling of water (c) Stars (d) Moon
droplets which contains Sulphur oxides and Nitrous Ans. (d) : The periodic rise and fall of the sea level due
Oxides, if the pH Value is less than 5.6 then it is to the gravitational pull of the Moon and Sun, once or
considered as acid rain. So2 and Nox released into the twice a day is called tides. High tides are caused by the
air mainly by industries such as oil refineries, power gravitational forces exerted on the earth by the Moon,
plants, etc, Vehicle, burning of fossil fuel are the and to a lesser extent, the Sun, When the highest point
biggest cause of acid rain. in the wave, or the crest reaches a coast, the coast
123. Light is made of seven colours. What is the experiences a high tide.
method of separating the colours? 128. The magnetic needle points to
(a) The colours can be separated by a prism (a) East (b) West
(b) The colours can be separated by a filter (c) North (d) South
(c) The colours can be separated by plants
Ans. (c) : Magnetic needle always points towards the
(d) The colours cannot be separated
North direction. A compass needle is a bar Magnet
Ans. (a) : A prism separates white light into a group of which is pointed towards north pole. Since it is attracted
seven colors called a spectrum. These seven colors are to the magnetic South pole so, magnetic needle points in
always in the same order. The colors of the spectrum the North direction.
are red, orange, yellow, green, blue, indigo, and violet.
129. Burning of hydrogen produces
124. Cooling is done by
(a) Oxygen (b) Ash
(a) Flow of water
(c) Soil (d) Water
(b) Release of compressed gas
(c) Cooking gas Ans. (d) : Hydrogen fuel is zero emission fuel burned
(d) Melting the solid with oxygen. Hydrogen burns in presence of oxygen to
form water.
Ans. (b) :A room can be cooled by release of
compressed gas. Most of the air conditioners use 2H2 + O2 → 2H2O + Heat.
compressed gas, which can cool the room or other 130. The main ideas behind 'globalisation' is
closed places. Compressed gas is released by absorbing (a) Attaining friendship among nations
surroundings heat or temperature. Hence causes (b) Solving international disputes
cooling. (c) Working together with the entire international
125. The smallest day occurs on community with a shared common
(a) 23th December (b) 23th September perspective
th
(c) 23 June (d) 23th April (d) None of the above
47th BPSC (Pre) Exam. 2004-05 144 YCT
Ans. (c): Globalisation is a process of interaction and 136. When was the 13th Non Aligned Movement
integration among the people, companies and (NAM) countries meet held in Kualalumpur?
governments of different nations. It includes the (a) January, 2001 (b) January, 2002
creation of networks and pursuits transforming social, (c) February, 2003 (d) None of these
economical and geographical barriers. The main idea
behind globalisation is working together with the entire Ans. (c) : The 13th Non Aligned Movement (NAM)
international community with a shared common countries Summit was held in Kuala Lumpur, Malaysia
perspective. on 24-25 February 2003. NAM was formed in 1955 at
131. Which one of the following countries is not a the Asia-Africa Bandung conference held in Indonesia.
permanent member of the UN Security After the 18th summit conference held at Baku in
Council? December 2019 Azerbaijan will hold the NAM
(a) Japan (b) USA presidency for 3 year until the 19th Summit in 2022.
(c) Britain (d) France 137. In November, 2002 meeting of the leaders of
Ans. (a) : The United Nations Security Council is a North Atlantic Treaty Organisation (NATO)
global body for maintaining international peace and was held at
security. The UN security council is composed of 15 (a) New Delhi (b) Prague
UN members, in which 5 are permanent members viz;
(c) Paris (d) None of these
China, France, Russia, the United kingdom and the
United States. 5 permanent members are collectively Ans. (b) : In November 2002, North Atlantic Treaty
known as the P5 Organisation (NATO) leaders summit was held in
132. Who is the Secretary General of the UNO? Prague, Czech Republic. On 29 - 30 June 2022, NATO
(a) Kofi Annan (b) U Thant members countries Summit will be held in Madrid,
(c) Kurt Waldheim (d) None of these Spain.
Ans. (a) : When the question was asked, Kofi Annan 138. Who represented India in the Indo-EU Summit
was the 7th Secretary General of United State from 1997 held in November, 2004?
to 2006. At Present, Antonio Guterres is the 9th (a) The President
Secretary - General of UN. He took office on 1st (b) The Foreign Minister
January 2017.
(c) The Prime Minister
133. Malaysian Prime Minister visited India, on
(d) The Defence Minister
(a) December 19-23, 2004
(b) November 18-22, 2004 Ans. (c) : The 5th India - EU Summit was held on 8th
(c) December 20-24, 2003 November 2004 in the Hague. The then Indian Prime
(d) January 5-9, 2005 Minister Manmohan Singh had led the Indian
Ans. (a) : When the question was asked Prime Minister delegation.
of Malaysia, Mr. H.E. Dato Seri Abdullah had visited 139. Which of the following is not a factor
India from 19 - 23 December 2004, This was his first contributing to the international relations?
visit to India after he assumed office in October 2003. (a) Technological Advancement
134. The King of Nepal declared emergency on (b) Spirit of Aggressive Nationalism
(a) February 1st, 2005 (b) January 1st, 2005 (c) Development of International Trade
(c) December 1st, 2004 (d) None of these (d) Development of Industries
Ans. (a) : On 1st February 2005, King Gyanendra of
Ans. (b) : Technological advancement, development of
Nepal had dismissed multiparty government and
International trade and industries etc. have contributed
declared state of emergency. He had sacked Prime
Minister Sher Bahadur Debua and took power directly to development of International relations where as the
and declared a State of emergency. spirit of aggressive nationalism is not a factor
135. Palestinian leader is contributing to the International relations. It creates
(a) Yasir Arafat (b) Mahmoud Abbas enmity towards each other and nations become at war
(c) Sharon (d) Mahmood Abbas with each other.
Ans. (b) : Mahmoud Abbas was a Palestinian politician 140. "prohibition of traffic in human beings", which
who was elected as President to the Palestinian National of the following article of constitution of India
Authority on 9 January 2005. deals with this?
Yasser Arafat was the first President of Palestinian (a) Article 21 (b) Article 22
National Authority from 1994 to 2004. (c) Article 23 (d) Article 24

47th BPSC (Pre) Exam. 2004-05 145 YCT


Ans. (c) : Article 23(1) of Indian constitution states that 40
or x= or x = 10 …(iii)
the trafficking in human beings and beggary and other 4
similar forms of forced labour shall be an offence Substituting the value of x in equation (ii)-
punishable in accordance with the law.
10+y = 60 or y= 50
141. A cyclist cover distances of 15 km, 20 km and z= 50x+15y or z= 50×10+15×50
25 km. at the speeds of 15 km/h, 25 km/h and
or z= 500 + 750 or z = 1250
15 km/h respectively. His average speed will be.
(a) 30 km/h (b) 20 km/h 144. Health Department knows that in a large
population 20% individuals suffer from a
(c) 22.5 km/h (d) 21.4 km/h
particular disease. The probability that 2
Totaldistance individual in a sample of 5 individuals selected
Ans. (*) : Average speed =
TotalTime from this population will be suffering from the
Total Distance = 15+20+25= 60 km disease is
Distance 15 20 25 (a) 0.2548
Total time = = + + (b) 0.0512
Speed 15 25 15
(c) 0.2048
75 + 60 + 125 260 52
= = = (d) 2,0204
75 75 15
Ans. (c) : Here n=4
60 60 × 15
∴Average speed = = = 17.3km / hr Probability of suffering from a particular disease,
52 52
15 20 1
p= = = 0.2
142. Let the consumer price index number with 100 5
respect to a certain base year be 325. The q=1–P
current year wage of a worker, having ` 240 as
= 1 – 0.2 = 0.8
wage in the base year, in order to maintain the
same standard of living, will be 5
× ( 0.8 ) × ( 0.2 )
3 2
Now 5 C 2 q5– 2 × p 2 =
(a) ` 780 (b) ` 1000 2 3
(c) ` 1560 (d) ` 1500
= 10×0.02048= 0.2048
Ans. (a) : According to the question,
145. The value of k so that the points (1, 2, 3),(k, 0,
In base year CPI= 100 4) and (-2, 4, 2) are collinear is
In current year CPI= 325 (a) 4 (b) 3
In base year CIP is 240, the current year (c) 1 (d) 2
325 Ans. (a) : Points are collinear if
CPI would be 240 × = 780
100 1 2 3
143. The maximum value of Z = 50x + 15y subject to k 0 4 =0
the constraints 5x + y ≤ 100, x + y ≤ 60, x ≥ 0, y –2 4 2
≥ 0 is
0 4 k 4 k 0
(a) 1500 (b) 1250 =1 –2 +3 =0
4 2 –2 2 –2 4
(c) 900 (d) 1000
1(0–16)–2(2k+8)+3(4k+0)=0
Ans. (b) : Putting the maximum value of x, y: = –16–4k–16+12k=0
5x+y =100 …(i) = 8k –32=0⇒8k=32⇒k=4
x+y=60 …(ii) 1
146. If f(x) is ≡ , the value of f(f(f(x))) is -
Subtracting equation (ii) from equation (i) 1− x
5x + y = 100 1
(a) (b) x2
x + y = 60
_________ (1 − x) 2
4x = 40 (c) (1-x) (d) x

47th BPSC (Pre) Exam. 2004-05 146 YCT


1 1 149. The demand function of a commodity is x = ,
Ans. (d) : f(x)= f (f ( x )) =
1– x 1 where x is the number of units demanded and p
1–
1– x is per unit price. The value of p, which
1 maximizes the revenue, is
⇒ f (f (x)) =
1 − x −1 (a) 4
1− x
(b) 8
− (1 − x )
f (f (x)) = (c) 6
x
(d) 5
Ans. (c) :
1 1 x x
Now f(f(f(x))) = = = =
1 – x x +1 – x x +1 – x 1 1
1+ x= ( 24 – 2P ) ⇒ 3x = 24 – 2P ⇒ 2P = 24 – 3x
x x 3
=x 24 – 3x 3
⇒P= = 12 – x
147. The acute angle between the normal's to the 2 2
planes 2x – y + z = 6 and x + y + 2z = 7 is
π π 3x 2
(a) (b) R=(x)–P(x)=12x–
2 4 2
π π Differentiating the equation w.r.t.x
(c) (d)
3 6 dR  dR 
= 12 – 3x  let = 0
Ans. (c) : Let the angle between two planes be θ. dx  dx 
a1x+b1y+c1z=d1
then, 12–3x=0
a2x+b2y+c2z=d2
a1a 2 + b1b 2 + c1c2 3x=12⇒x=4
cos θ =
a1 + b12 + c12 × a 22 + b 22 + c22
2
d2R
Again, = –3
2.1 – 1.1× 1.2 dx 2
=
22 + ( –1) .12 × 12 + 12 + 2 2
2
Therefore, R(x) is max at x=4

3 3 1 12
= = = P = 12 – =6
6× 6 6 2 2
1 π 150. The market value of a share of a company with
⇒cosθ= ⇒ cos θ = cos
2 3 face value ` 10 is ` 25. The company declares
π 10% dividend. The dividend on 1500 shares of
⇒θ=
3 the company will be
148. If nC1+ nC2+.....+ nCn= 255, then the value of n is (a) ` 3750
(a) 6 (b) 8 (b) ` 15000
(c) 4 (d) 10 (c) ` 7500
Ans. (b) : According to the question- (d) ` 1500
n
C1 + n C 2 + .......... + n C n = 225 Ans. (a) : The market value of share
n
C 0 + n C1 + n C 2 + ....... + n C n = 2 n ] = 1500 × 25 = 375000
∴ C1 + C 2 + ....... + C n = 2 – 1
n n n n
(∵ cn
0 =1 ) and declares 10% dividend
again 2 –1 = 255
n
37500 × 10
=
∴ 2 = 255 + 1 = 256
n 100
∴ 2n = 28 = 3750
n=8 So option (a) is correct.

47th BPSC (Pre) Exam. 2004-05 147 YCT


48th-52th Bihar Public Service Commission
Preliminary Examination, 2007-08
GENERAL KNOWLEDGE & GENERAL SCIENCE
(Solved Paper with Detail Explanation)
1. Smt. Pratibha Patil is the________ President of Ans. (c) : The Attorney General of India is the central
the Republic of India. governments chief legal advisor. He has the right to
(a) 10th (b) 11th speak and to take part in the both house of parliament
(c) 12th (d) 13th and in the joint sittings while not being a member of
Ans. (c) : Smt. Pratibha Patil assumed office as the 12th either house. He is appointed by the president of India
President of India on July 25, 2007. She was the first
under article 76(1) of the constitution and holds office
women to be elected as the President of India. At
during the pleasure of the president. he can also take
Present Draupadi Murmu is elected as 15th president of
India who defeated UPA's candidate Yashvant Sinha. Part in any of parliamentary committees but has no right
2. A bill presented in Parliament becomes an Act to vote.
after- 6. Fundamental Duties were incorporated in the
(a) It is Passed by both the Houses Constitution of India by the?
(b) The President has given his assent (a) 32nd Amendment Act
(c) The Prime Minister has signed it (b) 42nd Amendment Act
(d) The Supreme Court has declared it to be (c) 15th Amendment Act
within the competence of the Union (d) 46th Amendment Act
Parliament.
Ans. (b) : The fundamental duties were incorporated in
Ans. (b) : A bill passed by the parliament becomes an
the Indian constitution on the recommendation of
act as soon as the president gives his/her assent or
Swaran Singh committee. They are in part IVA of the
approval.
Indian constitution and article 51 A. The fundamental
3. According to our Constitution the dissolution
of Rajya-Sabha occurs after how many years? duties were added in the year 1976. Through 42nd
(a) It dissolved after two years Constitutional Amendment Act. The constitution of
(b) It dissolved every five years India adopted the fundaments Duties from the
(c) It dissolved every seven years constitution of USSR.
(d) It doesn't subject to dissolution 7. How many languages are reorganized as
Ans. (d) : According to Article 83(a) the council of regional languages in the Constitution?
states or Rajysabha shall not be subject to dissolution. (a) 12 (b) 13
However , one third of its members retire every second (c) 14 (d) 18
year and are replaced by newly elected members.
Ans. (*) : There are 22 official languages in India and
4. A High Court Judge addresses his letter of they are covered under the 8th schedule of the India's
resignation to- constitution, initially, there were 14 official languages
(a) The President
in the Indian constitution, 'Sindhi' language was added
(b) The Chief Justice of India
by the 21st Amendment Act of 1967. 'Konkani',
(c) The Chief justice of High Court
"Manipuri and Nepali were included by the 71st
(d) The Governor of the State
Amendment Act of 1992 and At the last 'Bodo', 'dogri',
Ans. (a) : A judge of the high court can resign from his 'Maithili' and 'Santhali' were added by the 92nd
office by writing to the president. However the tenure of Amendment act of 2003 which came into force in 2004.
high court judge is not fixed by the constitution but they
8. Consider the following words :
hold office until they attain the age of 62 year. A. Socialist
5. As a Non-Member, who can Participate in the B. Democratic
proceedings of either House of Parliament? C. Sovereign
(a) Vice-President D. Secular
(b) Chief Justice Arrange these words in sequence according to
(c) Attorney General the preamble of India. Choose the correct
(d) Chief Election Commissioner option.

48th-52th BPSC (Pre) Exam. 2007-08 148 YCT


(a) C, A, D, B (b) C, D, A, B, together are called a development block. The Panchayat
(c) C, D, B, A (d) D, A, C, B Samiti is the link between the Gram Panchayat and the
Ans. (a) : The preamble of India's constitution : WE district administration (Zila Parishad). The common
THE PEOPLE OF INDIA, having solemnly resolved to department in the samiti are General Administration,
constitute India into a SOVEREIGN, SOCIALIST, Finance, Public works, Agriculture, Health, Education
SECULAR, DEMOCRATIC, REPUBLIC and to secure Social Welfare, Information technology & others.
to all its citizens. The Terms 'Socialist', 'Secular' and 13. According to the World Bank's latest
Integrity were added to the Preamble through 42nd development report, the position of the India
Amendment act, 1976. economy in the world is the-
9. In which House is the Presiding Officer not a (a) Largest (b) Smallest
member of that House? (c) Second Largest (d) Seventh Largest
(a) Lok Sabha (b) Rajya Sabha Ans. (*) : Presently India is the third largest economy
(c) Vidhan Sabha (d) Vidhan Parishad on the basis of Purchasing Power Parity (PPP) and fifth
Ans. (b) : The vice-president of India is an ex-officio largest economy in the world on the basis of GDP.
chairman of Rajya sabha and become Presiding officer 14. If the cash reserve ratio is lowered by the RBI,
but not a member of Rajya sabha. its impact on credit creation will-
According to Article 63 of Indian constitution. It state (a) Increase it (b) Decrease it
that "there shall be a Vice-President of India. The Vice- (c) Be nil (d) None of these
President act as President in the absence of the Ans. (a) : Credit creation will increase, if Cash Reserve
President due to death, resignation, impeachment or Ratio (CRR) decrease. Cash Reserve Ratio (CRR) refers
other situation. to the amount of money, banks have to keep with the
10. By which constitutional amendment Bill was central bank. So when the amount required under CRR
the voting age reduced from 21 years to18 is reduced, bank have more money at their disposal
years? which is to be then used for credit creation in the
(a) 48th (b) 57th economy.
(c) 61st (d) 63rd 15. Indian Development Forum (IDF) was earlier
Ans. (c) : In India, the voting age has been reduced known as
from 21 year to 18 years by the 61st constitutional (a) Aid India Consortium (b) Aid Indian Bank
amendment act 1988. (c) World Bank (d) None of these
11. If the positions of the president and Vice- Ans. (a) : Indian Development Forum was earlier
President are vacant, who officiates as the known as Aid India Consortium. The Aid-India
President of India? consortium was organised in 1958 as an international
(a) The Prime Minister scheme to support the economic development in India
(b) The Chief justice of India and led by the world Bank.
(c) The Speaker of Lok Sabha 16. It will be true to India as–
(d) None of these (a) A food-deficit economy
Ans. (b) : In case the President's office falls vacant and (b) A labour-surplus economy
the office of Vice-president is also vacant. In this case (c) A trade surplus economy
Chief Justice of India will officiate, till the new (d) A capital surplus economy
President is elected. This provision was made in 1969 Ans. (b) : India is the second largest populous country
by the parliament to enable Chief Justice Hidayatullah in the world and it can be defined as a labour surplus
to officiate when president Zakir Hussain had died and economy in view of the large working class polulation.
vice-president V.V Giri resigned. 17. Amended VAT is related to-
(a) Sales Tax (b) Wealth Tax
12. A panchayat samiti at the block level is-
(c) Excise Tax (d) Income Tax
(a) An advisory body
Ans. (a) : VAT was introduced as an indirect tax in the
(b) An administrative authority
Indian taxation system to replace the existing general
(c) A consultant authority
sales tax. The value added Tax Act (2005) and
(d) A supervisory authority associated VAT rules came into effect in beginning of
Ans. (*) : A Block level institution is called the April 1, 2005 in many India's states. VAT is a simple
Panchayat Samiti. Panchayat Samiti is a local indirect tax which is levied at every point exchange of
government body at the Tehsil or Taluka level in India, goods and services from primary product to final
it works for the villages of the Tehsil or Taluka that consumption goods.

48th-52th BPSC (Pre) Exam. 2007-08 149 YCT


18. The Eleventh Five Year Plan's Objective is 23. The Second Green Revolution will be related
(a) Removal of poverty with
(b) Inclusive growth (a) HYVS (b) Wheat
(c) growth with social justice (c) Rice (d) Bio-technology
(d) Development of minorities
Ans. (d) : A conference for the second green revolution
Ans. (b) : The main objectives of the eleventh five year
plan (2007-2012) as 'Inclusive Growth'. Dr. Manmohan was held in New Delhi on November 22-24, 2005. The
Singh was the prime minister during the eleventh five second green revolution is related to the use of
year plan and the chairman of the Planning Biotechnology.
Commission. The document entitled "TOWARDS 24. Fiscal Deficit is
FASTER AND MORE INCLUSIVE GROWTH." (a) Total expenditure-Total receipt
19. Rainbow revolution is related with (b) Revenue expenditure-Revenue
(a) Green Revolution (b) White Revolution (c) Capital expenditure-Receipt-Borrowing
(c) Blue Revolution (d) All of these (d) Sum of budget deficit and government's
Ans. (d) : The rainbow revolution is related to market borrowings and liabilities
agricultural industries. It includes 'Green Revolution'
blue revolution and white revolution etc. It is an Ans. (d) : Fiscal deficit is the difference between the
integrated development of crop cultivation, horticulture, total income of the government (total taxes and non
forestry, fishery, poultry, and animal husbandry. It debt capital receipt) and its total expenditure while
includes the integration of all other agricultural calculating Fiscal deficit the total revenue, borrowings
revolution. The main objectives of the rainbow are not included.
revolution in Agriculture practices have to be improved Hence, Fiscal Deficit is (Budget expenditure - Budget
to maintain environmental sustainability and the receipts excluding barrowings)
sustainability of resources.
25. The infant mortality rate of Bihar, is-
20. The one rupee note bears the signature of the-
(a) More than Jharkhand (b) Equal to Jharkhand
(a) Secretary, Ministry of Finance
(b) Governor, RBI (c) Less than Jharkhand (d) Equal to all India
(c) Finance Ministry Ans. (c) : In Bihar, the infant mortality rate in rural
(d) None of these areas is 63 (per thousand live births) while in urban area
Ans. (a) : The one Rupee denominated note bears the infant mortality rate is 52 and As a whole, It is equal
signature of the secretary, Ministry of Finance. While to 61 (per thousand live birth). While the infant
all denominations of currency note are signed by the mortality rate in the state of Jharkhand is 92 (per
RBI governor. thousand live birth) which is much greater than Bihar.
21. The period of plan holiday in India was-
26. In the history of Bihar for the first time, the
(a) 1962-65 (b) 1966-69
economic survey of the State was presented by :
(c) 1968-72 (d) 1972-75
(a) Nitish Kumar (b) Lalu prasad
Ans. (b) : The period between 1966-69 is referred as (c) Sushil Modi (d) None of them
plan holiday in India as five year plans could not be
Ans. (c) : The state government of Bihar presented it's
implemented during this period. Instead of five year
first economic survey during 2006-07. It was presented
plans, there were three annual plans implemented
by the then deputy Chief Minister and Finance Minister
during this period .It was happened due to miserable
Sushil Modi in the legislative assembly.
failure of the third plan, the government of India was
forced to declare, "plan holidays" form 1966 to 1969 for 27. As per 2006-2007 Development Report. Bihar's
a revision objective and targets. HDI (Human Development Index) is being
lower than the National HDI by:
22. The "Rolling Plan" concept in national (a) 15 percent (b) 20 percent
Planning was introduced by- (c) 25 percent (d) 10 percent
(a) Indira Gandhi
(b) The National Front Government Ans. (b) : The 2006-07 report indicates Bihar had
(c) The Janta Government lagged behind other state in all the development indices.
(d) Rajiv Gandhi The state ranks at the bottom with respect to Human
Ans. (c) : The rolling plan in India was started in 1978 Development Index (HDI) with Bihar being about 20
during the government of Janata Party. This rolling plan percent lower than the National HDI.
concept was coined by Gunner Myrdal, the main 28. In the year 2006-07, the development
advantage of the rolling plan is that they are flexible. expenditure of Bihar was____ of its total
The rolling plan is a plan in which every year expenditure?
performances of the plan is assessed and a new plan is (a) About 60 percent (b) 67 percent
made next year based on this assessment. (c) 56 percent (d) 10 percent
48th-52th BPSC (Pre) Exam. 2007-08 150 YCT
Ans. (a): In the year 2006 -07 , the development 33. A coin is tossed upwards from the ground with
expenditure of Bihar was about 60% of it's total a velocity of 9.8 m/sec. Then it rises to a height
expenditure. of
(a) 9.5 m (b) 10 m
29. At present which is the main Source of wealth (c) 4.9 m (d) 49 m
in Bihar? Ans. (c) : Given,
(a) Industry (b) Agriculture Initial velocity = u = 9.8 m/sec
(c) Natural Resources (d) Minerals v2 – u2 = 2as [v = final velocity will be zero at
Ans. (b): After the Bifurcation of Bihar into the state of maximum height]
Jharkhand, agriculture has became the main source of ⇒ thus 0 – (9.8)2 = 2 (–9.8)h
wealth in Bihar. At present most natural resources and ⇒ h = 4.9 m
mineral wealth with major industry have gone to the maximum height reached = 4.9 m
state of Jharkhand which were earlier part of Bihar. 4
34. The expression has the value-
30. What percentage of its total expenditure is
spent by the government of Bihar on salary and
3
(
9 − 3 3 +1 )
1 1
pension alone?
(a) 40 percent (b) 50 percent (a) 33 −1 (b) 33 + 1
1 1
(c) 60 percent (d) 46 percent
(c) 3 2 + 1 (d) 3 2 − 1
Ans. (d) : During the question hour, the Bihar
government had used to spend 46 percent of its total 4
Ans. (c) :
revenue receipt on salary and Pension.
3
9 − 3 3 +1
31. The maximum value for 7 + 10x – 5x2 is 4
(a) 6 (b) 8 =
32 / 3 − 31/ 3 + 1
(c) 10 (d) 12
3 +1
Ans. (d) : To make the expression a function of a =
perfect square. 32 / 3 − 31/ 3 + 1
7 + 10x– 5x2 a 3 + b3
= 7 + 5– 5 + 10x – 5x2 The given expression as
a − ab + b2
2
= 12– 5(x2 – 2x +1) Where a = 3 1/3
and b = 1
= 12 – 5(x –1)2
Observe that the maximum value of expression when
=
(3 ) + 11/ 3 3 3

value of –5(x –1)2 should zero and it is possible be


when (x–1) is 0. (or when x = 1) (3 ) − 3 + 1
1/ 3 2 1/ 3

∴ The maximum value is = 31/3+1


12 – 0 = 12 35. For every integer x the expression x(x2 – 1)(3x
32. The point on the curve y = 1 + 2x – 3x2 at which +2) is divisible by
the tangent makes an angle of 45° with x-axis (a) 13 (b) 15
is? (c) 24 (d) 25
1 5 1 5 Ans. (c) : P(x) = x(x2–1) (3x+2)
(a) , (b) , P(0) = 0
2 4 3 4 P(1) = 0
1 4 1 5 P(2) = 2×3×8 = 24×2
(c) , (d) ,
3 5 6 4 P(3) = 3×8×11 = 24×11
dy Hence expression P(x) = x(x2–1) (3x+2) for every
Ans. (d) : Slope at any point, = 2 − 6x integer is divisible by 24.
dx
36. The triangle joining the points (2, 7), (4, –1), (–
Let at (a,b) the curve makes angle 45º
2, 6) is
1 (a) Equilateral (b) Right-angled
⇒ 2– 6a = tan (45º) = 1 ⇒ a =
6 (c) Isosceles (d) None of these
2
1 1 Ans. (b) :
∴ b = 1 + 2a – 3a2 = 1 + 2 × − 3 ×  
6 6
1 1 5
= 1+ − =
3 12 4
1 5
∴ Required point is  , 
6 4

48th-52th BPSC (Pre) Exam. 2007-08 151 YCT


39. the number of ways of dividing 15 men and 15
length of AB = (4 − 2) 2 + (−1 − 7) 2 = 4 + 64 = 68
women into 15 couple each, consisting of a man
length of BC = (4 + 2) 2 + (−1 − 6)2 = 36 + 49 = 85 and a woman is :
(a) 1240 (b) 1840
length of AC = (2 + 2) 2 + (7 − 6)2 = 16 + 1 = 17 (c) 1820 (d) 2005
∵ BC2 = AB2 + CA2 Ans. (a) : The pairing of a man and a woman is like this
So, it is right angled 15 15
37. The radius of a circle is increasing uniformly at 14 14
the rate of 3 cm/sec. At what rate is the area 13 13
increasing when the radius is 10 cm? and so on
2
(a) 6π cm /sec 2
(b) 10π cm /sec ⇒ 15×15+14×14+13×13…..1×1 = 1240
2
(c) 30π cm /sec 2
(d) 60π cm /sec 40. The probability that a student is not a swimmer
is 1/5. The probability that out of 5 students,
Ans. (d) : Area of circle = πr2 where r is radius exactly 4 are swimmers is
So area of circle = 100π (a) (4/5)3 (b) (4/5)4
4
A = πr 2 (c) 5(4/5) (d) None of these
Ans. (b) :
 dA 
  = 2rπ 1
 dr 3 The probability that the student is not a swimmer =
5
so it is increasing at the rate of 3cm/s
1 4
dA Probability of student being a swimmer = 1– =
= (2 × 10 × π)3 5 5
dr According to Bernoulli's theorem - If the probability (p)
= 60π of an event occurring (success) in one trial is (p) , then
Hence the area of the circle is increasing at the rate of the probability that will occur exactly r times in a total
of n trials are as following
60 cm/sec
p(r) = nCr pr qn-r
38. A family spend 30% of their income on food, where q is the probability that the event will not occur
10% on clothes, 18% on house and 7% on the According to question
other things. If the monthly income of the 4 1
p = , q = , n = 5, r = 4
family is `4000/- then the family can save 5 5
`15000/- in : probability that four out of five students are swimmers.
4 5−4
(a) 25 months (b) 30 months 4 1
= 5 C4    
(c) 11 months (d) 50 months 5 5
Ans. (c) : 4
4 1
Monthly income = 4000 rs. = 5 C4    
5 5
30 4
Food cost = 4000 × = 1200 rs. 5 4 1
100 =    
41 5   5 
10
Spending on clothes = 4000 × = 400 rs. 4
544 1
100 =    
4  5 5
18
House cost = 4000 × = 720 rs. 4
4 1 4
4
100 = 5    =  
7 5 5 5
Other expenses = 4000 × = 280 rs. 41. What is the total population of India?
100
(a) 1,027,015,247 (b) 1,02,710,1012
Monthly total expenses = 120 + 400 + 720 + 280 (c) 110,12,52,751 (d) None of above
= 2600 rs. Ans. (a) : According to the census of 2001 the
Monthly savings = 4000 – 2600 = 1400 rs. population of India was 1027015247. While As per
1400 rs. Saves in a month. 2011 census, India's total population in 1210.2 million
15000 150 in which the number of males was 623.7 million and the
15000 rs. time taken to save = = = 10.7 population of female stood at 586.5 million. There was
1400 14
an increase in the population during decade 2001-2011
about 11 months. which is equal to 17.64%.

48th-52th BPSC (Pre) Exam. 2007-08 152 YCT


42. When was the foundation of BSF laid? 47. What is the name of China's News Agency?
(a) 1965 (b) 1970 (a) China News
(c) 1910 (d) 1930 (b) China Times
st (c) New China News Agency
Ans. (a) : On 1 December, 1965 India's first line of
Defence the Border Security Force (BSF) was formed. (d) New China Report
The BSF is India's primary border guarding force this Ans. (c) : The China's new agency is "New China News
paramilitary force is one of the central armed police Agency" It is also known as xinhua news agency and
force in India. BSF was established as a response to the the News agencies of India are - PTI (Press Trust of
1965 war with Pakistan when that country had tried to India), UNI (United News of India), News Bharati,
expand its own borders at India's territory. The BSF is Indian News Service etc.
under ministry of home affairs of the Government of 48. Which country's government report is known
India. as a yellow book?
43. Where is Bheel Tribe found? (a) France (b) Britian
(a) Assam (b) Jharkhand (c) Italy (d) Germany
(c) West Bengal (d) Maharashtra Ans. (a) : Yellow book is a report or publication of the
Ans. (d) : The Bhil are one of largest tribal groups French government. Blue book is a report or Publication
living mainly in Chhattisgarh, Gujarat, Karnataka, of the British government. The Grey book is a report or
Madhya Pradesh, Maharashtra, Andhara Pradesh and publication of the Government of Japan and Belgium.
Rajasthan. The name 'Bhil' is derived from the word White paper is a government report of India.
'Bhil' which means bow. 49. When was Indian Life Insurance Corporation
44. Where is the Naval Air Station "Garud" established?
situated? (a) 1956 (b) 1544
(a) New Delhi (b) Cochin (c) 1950 (d) 1947
(c) Chennai (d) Dehradun Ans. (a) : India life insurance corporation which is also
Ans. (b) : INS Garud is an Indian Naval air station known as LIC. It was established in the year 1956. The
located near Kochi (Cochin), in the state of Kerala. It main slogan of LIC is "Yogakshemam Vohamyahan"
was commissioned on 11 May, 1953 and it is the oldest meaning 'Your Welfare is our responsibility".
operating air station of Indian Navy. 50. The first law University in India was
45. Where is Bharat Dyanamic Ltd. situated? established in august 1887 in
(a) Calcutta (b) Hyderabad (a) Trivandrum (b) Ahmedabad
(c) Chennai (d) Delhi (c) Bangalore (d) New Delhi
Ans. (b) : Bharat Dynamics Limited is situated in Ans. (c) : The first law University in India was
Hyderabad district of Telangana, is one of the leading established in august 1887 by the name as " The
defence PSU in India engaged in the manufacture of National law School of India university". (NLSIU)
surface to Air Missiles (SAMs) and Anti tank Guided Which was established in Bangalore (Karnataka).
Missiles (ATGMs), underwater weapons launchers and 51. Mophlah Revolt of 1921 took place in -
test equipment. It is the sole manufactures in India for (a) Kashmir (b) Bihar
SAMs, Torpedoes, ATGMs to the India Armed forces. (c) Kerala (d) Assam
46. Which of the country is not the member of G-8 Ans. (c) : The Mophlah Revolt of 1921 which is also
Association? known Mappila rebellion took place in Kerala. It was
(a) India (b) Germany the culmination of a series of riots by Mappila Muslims
(c) Japan (d) Canada of Kerala in the 19th and early 20th centuries aganist the
Ans. (a) : The Group of Eight (G-8) refers to the group British and the Hindi landlords in Malabar. It was an
of eight highly industrialized nations-France, Germany, armed revolt led by Ali Musliyar Variyankunnath
Italy, The United Kingdom, Japan, the United States, Kunjahammad Haji.
Canada and Russia. The group of G-8 was formed in 52. Who was the inventor of radar?
1975 with initially 6 countries as its members :-France (a) Robert Watson (b) Amenometer
West Germany, Italy, Japan, U.K. and The United (c) Bush Wall (d) Austin
States. It is noteworthy that Presently this group is Ans. (a) : RADAR stands for "Radio Detecting and
known as G-7 due to the suspension of Russia's Ranging". RADAR was first invented by Robert
membership after the succession of Crimea by Russia in Watson in 1919. It can be used to detect aircraft, ships,
2014. spacecraft guided missiles etc.

48th-52th BPSC (Pre) Exam. 2007-08 153 YCT


53. Which of the following instruments measures 58. The book "Nineteen Eighty Four" is written
blood pressure? by-
(a) Spherometer (a) R.K. Narayan (b) Khushwant Singh
(b) Amenometer (c) George Orwell (d) None of the above
(c) Sphygmomanometer Ans. (c) : George Orwell is the author of the book
(d) Barometer "Nineteen Eighty Four". It was published in 1949 as a
Ans. (c) : A Sphygmomanometer is an instrument that warning against totalitarinism. The novel serves a
is used to measures arterial blood pressure. It also sobering reminder of the evils of unaccountable
consists of a mercury or aneroid manometer to measure government.
the pressure. It is also known as blood pressure gauge, 59. Which of the following vitamins contains
blood pressure monitor and blood pressure meter. cobalt?
54. Among the following foreigners who was (a) Vitamin B6 (b) Vitamin B2
associated with Bharat Ratna? (c) Vitamin B1 (d) Vitamin B12
(a) Nelson Mandela (b) Bill Clinton Ans. (d) : Vitamin B12 is chemically named as
(c) Adolf Hitler (d) Boris Yetsin Cyanocobalamine which contains Cobalt. It is also an
Ans. (a) : Bharat Ratna :- The Jewel of India is the essential trace element for humans. It is found at the
highest civilian award of the country. It is conferred for centre of Vitamine B12 and a range of other co-enzymes
exceptional service to the nation in various fields such called Cobalamines
as science, arts, literature and recognition of public 60. Which of the following human organs is
service of the highest order. The first two non- Indians affected by the consumption of aflatoxin a food
who received the Bharat Ratna are Khan Abdual Gaffar adulterant?
Khan (1987) and Nelson Mandela (1990). There is no (a) Heart (b) Lungs
written provision that Bharat Ratna should be awarded (c) Kidney (d) Liver
to Indian citizens only. Ans. (d) : Aflatoxin food poisoning in humans
55. With which of the following fields Pulitzer commonly affects the liver.
award associated? 61. Which of the following is not a permanent
(a) Journalism (b) Science member of the U. N. Security Council?
(c) Sports (d) Industry (a) Germany (b) France
Ans. (a) : Pulitzer prize is awarded for Journalism. It (c) Great Britian (d) China
was established in 1917 and is administered by Ans. (a) : The permanent members of the united
Columbia University and Pulitzer Prize board. It was
Nations Security council are the - U.S.A. Britain,
awarded in the name of Joseph Pulitzer a, newspaper
France, Russia and China.
publisher who gave money in his will to Columbia
University to launch a Journalism school and establish 62. 'ASEAN' stands for
the prize. (a) Academy of South-East Asian Nations
(b) Association of South-East African Nations
56. Itimad-ud-Daula's tomb at Agra was built by?
(c) Association of South-East Asian Nations
(a) Akbar (b) Jahangir
(c) Noorjahan (d) Shahjahan (d) None of the Above
Ans. (c) : The tomb of Itimad-ud-Daula's is a Ans. (c) : The Association of South-East Asian Nations
mausoleum situated on the bank of river Yamuna just ASEAN, was established on 8 August 1967 in
outside Agra. It was built by Noorjahan, the wife of Bangkok, Thailand with the signing of the ASEAN
Jahangir for the father Mirza Ghiyas Beg between 1622 declaration by founding father of ASEAN namely
and 1628 Noorjahan took an interest in architecture and Indonesia, Malaysia, Philippines, Singapore and
was a prolific planner as well as a sponsor of several Thailand, Brunei, Darussalam then joined on 7 January,
important building. 1884 Vietnam on 28 July, 1995, Lao PDR and
57. Which of the following games is Radha Mohan Myanmar on 23rd July, 1997 and Cambodia on 30 April
Cup associated With?
1999, making up what is today the ten member states
(a) Polo (b) Football/Soccer
(c) Cricket (d) Tennis ASEAN.
Ans. (a) : The Radha Mohan cup is related to the Polo. 63. According to the census 2001 the density of
There are some other important tournaments related to population in India (per sq km) is -
Polo are following : (a) 304 (b) 324
Ezar Cup, Prithvi Pal Singh Cup, Classic cup etc. (c) 344 (d) 364
48th-52th BPSC (Pre) Exam. 2007-08 154 YCT
Ans. (b) : According to census 2001, the population Ans. (a) : The 123 agreement is a bilateral cooperation
density of India was 324 person per square Kilometer. as on nuclear energy cooperation between India and the
According to the recent census data as, 2011, the U.S.A.
population density of India is 382 person/sq km. 69. After the amendment in Indian constitution by
64. At one time, the Naxalities were limited to only Parliament on 14 August 2007, the total
west Bengal and Andhra Pradesh in India. number of castes is?
Now, they have extended their activities in (a) 607 (b) 1206
these states also? (c) 1416 (d) 1500
(a) Orissa, Chhattisgarh, Jharkhand, Bihar Ans. (b) : After the amendment in the constitution of
(b) Bihar, Chhattisgarh, Tamil Nadu, Uttar India by the Parliament on August 14, 2007 now the
Pradesh number of scheduled castes in the list of scheduled
(c) Madhya Pradesh, Maharashtra, Jammu castes is 1206. According to the latest data (April 12,
Kashmir, Rajasthan 2018) of the Ministry of Social Justice and
(d) Jharkhand, Bihar, Karnataka, Gujarat Empowerment, the total number of scheduled castes at
Ans. (a) : Naxalism in India which had spread in west- present is 479. Arunachal Pradesh, Mizoram,
Bengal and Andhra Pradesh in the past years, has now Meghalaya, Nagaland and Lakshadweep are such Indian
spread in Orissa, Chhattisgarh, Jharkhand, Bihar also, It states in which the number of scheduled caste is zero.
is important to note that some districts of Uttar Pradesh 70. Mr. Mohammad Hamid Ansari is the _____
are also affected by Naxalism. Vice-President of India.
65. Which one of the following cosuntries is not a (a) 10th (b) 11th
member of the G-8 Group? (c) 12th (d) 13th
(a) France (b) Italy Ans. (d) : Mr. Mohammad Hamid Ansari was the 12th
(c) Spain (d) Germany Vice-President of India. Jagdeep Dhankhar is elected as
Ans. (c) : See explanation of Question No.46. 14th Vice President of India. He took Oath on 11th
66. "LTTE" stands for- August 2022.
(a) Lankan Tigers for Tamila Eelam 71. The Eighth world Hindi Conference in 2007
(b) Liberation Tigers for Tamil Eelam was organized at
(c) Lankan Troops for Tamil Empire (a) Delhi (b) London
(d) None of the above (c) Tokyo (d) New York
th
Ans. (b) : LTTE stands for Liberation Tigers for Tamil Ans. (d) : The 8 world Hindi conference was
Eelam, was a guerrilla organisation that sought to organised in New York on 13 July, 2007.
establish an independent Tamil state, Eelam in northern 72. The mobile court in India is the brainchild of
and eastern Sri Lanka. The LTTE was established in (a) Justice Bhagwati
1976 by Velupillai Prabhakaran as the successor to an (b) Mr. Rajeev Gandhi
organisation he had formed earlier in the 1970s (c) Dr. A.P.J. Abdul Kalam
67. G-15 is - (d) Mrs. Pratibha Patil
(a) An organisation of the developed countries of Ans. (c) : The mobile court in India is the Brain child of
the world former President Dr. A.P.J. Abdul Kalam. In India the
(b) An organisation of the developed countries of first mobile court was established in the year 2007 from
Europe Indian states of Haryana.
(c) An organisation fo the developing countries
73. According to Fortune Magazine, the richest
of Asia
man in 2007 was
(d) An organistion of the developing countries of
(a) Carlos Slim Helu (b) Bill Gates
the world
(c) Laxmi Mittal (d) Warren Buffett
Ans. (d) : The group of fifteen (G-15) was set up by
Ans. (a) : According to the list of the world's richest
fifteen developing countries in 1989 at the ninth NAM
people released by fortune magazine on August 6, 2007
summit in Belgrade as an Action group with the role of
the Mexican telecoms businessman Carlos Slim was at
a catalyst for the third world. the first G-15 summit was
the first place in whole richest man of the world. It is
held in Malaysia in 1990.
noteworthy that Tesla's CEO Elon Musk is the richest
68. The 123 Agreement is between-
person in the world with a net worth of $ 239.3 billion
(a) Indo-US (b) Indo-Russia
as of 9 February, 2022.
(c) Indo-Pak (d) Indo-China
48th-52th BPSC (Pre) Exam. 2007-08 155 YCT
74. Leeds Metropolitan University has awarded an 80. Match correctly :
honorary doctorate degree on July 18, 2007 to A. The hottest 1. Chile
this Indian woman for her contribution to B. The coldest 2. Cherrapunji
cultural diversity. Name the woman. C. The wettest 3. Antarctica
(a) Arundhati Roy (b) Kiran Bedi D. The driest 4. Sahara
(c) Sharmila Tagore (d) Shilpa Shetty (a) A-1; B-4; C-2; D-3 (b) A-4; B-3; C-2; D-1
Ans. (d) : In 2007 actor Shilpa Shetty received an (c) A-3; B-2; C-1; D-4 (d) A-4; B-2; C-1; D-3
Honorary Doctorate degree by the Leeds Metropolitan Ans. (b) : The correct matching of the above is as
University, U.K. for her Outstanding contribution of follows -
cultural diversity. The hottest - Sahara
75. At which place did Pakistan's Ex-Prime The coldest - Antarctica
Minister Benazir Bhutto die? The wettest - Cherrapunji
(a) Lahore (b) Rawalpindi The driest - Chile
(c) Karachi (d) Islamabad 81. What is the percentage of seats reserved for
Ans. (b) : The former Prime Minister of Pakistan women in the Parliament of Bangladesh?
Benazir Bhutto had died in Rawalpindi. she belongs to (a) Nil (b) 15 percent
the peoples party of Pakistan. The current prime (c) 30 percent (d) 45 percent
minister of Pakistan is Shehbaz Sharif. Ans. (*) : The Reservation for women members in the
76. Which American Act was in dispute in Parliament of Bangladesh at the national level is 50 seat
relations to Indo- US Nuclear agreement? out of a total of 350 seat. As per the 15th amendment
(a) Kissinger Act (b) Hyde Act Act of the constitution of Bangladesh, the percentage of
(c) Munro Act (d) Bush Act women's reserved seat is therefore roughly 14 percent
Ans. (b) : The Hyde act has been made in America in of the total seats. The current prime minister of
relation to the Indo-US nuclear treaty. U.S. senator Sir Bangladesh is Sheikh Hasina. She belongs to Awami
Henry Hyde gave the shape to this law, Hence the Hyde league party.
act is named after him. 82. In which year did Railway Ministry announce
77. Which group was permitted at investment to start the project known as 'Village on
Nandi Gram area under the SEZ policy? Wheels'?
(a) Tata Group (b) Birla Group (a) 2004 (b) 2005
(c) Salim Group (d) Wipro Group (c) 2006 (d) 2007
Ans. (c) : Salim group of Indonesia was permitted at Ans. (c) : The announcement of starting a project
investment Nandi Gram area under the SEZ policy. named "Village on wheels" by the ministry of railways
78. In which year was the new currency 'Euro' was made in the year 2006.
introduced? 83. When was the Lok Sabha Election 2004
(a) 1996 (b) 1997 cancelled and re-voting done at Chhapra
(c) 1998 (d) 1999 constituency in Bihar?
Ans. (d) : The Euro is the official currency of 19 out of (a) 10 May and 15 July (b) 10 May and 31 July
the 27 member states of European Union. It was (c) 15 May and 31 May (d) 25 May and 10 June
introduced as a new currency in the year 1999. Ans. (c) : In the loksabha elections 2004, the polling in
79. According to the report of World Bank, more Chhapra constituency of Bihar, which was to be held on
than 50 percent poor persons in India live in 15 May, was cancelled and on 31th May 2004 re-
these four states? polling was conducted for this seat.
(a) Bihar, Uttar Pradesh, Madhya Pradesh, Orissa 84. Match correctly :
(b) Rajasthan, Uttar Pradesh, Madhya Pradesh, List-I List-II
Orissa A. Bardoli 1. Maharashtra
(c) Bihar, Uttar Pradesh, Jammu & Kashmir, B. Chauri Chaura 2. Gujarat
Orissa
C. Yerwada 3. West Bengal
(d) Bihar, Orissa Madhya Pradesh, Jammu and
Kashmir D. Noakhali 4. Uttar Pradesh
(a) A-1; B-2; C-3; D-4
Ans. (a) : When question was asked more than 50
(b) A-2; B-1; C-4; D-3
percent poor persons in India live in these four states
(c) A-2; B-4; C-1; D-3
Orissa, Bihar, Uttar Pradesh, and Madhya Pradesh.
(d) A-4; B-3; C-2; D-1
48th-52th BPSC (Pre) Exam. 2007-08 156 YCT
Ans. (c) : The correct match of the above question as 90. The captain of India Hockey team which won
follows – Asia cup 2007 was
Bardoli - Gujarat (a) Dilip Tirkey (b) Prabhjot Singh
Chauri Chaura - Uttar Pradesh (c) Baljit Singh (d) Varinder Singh
Yerwada - Maharashtra Ans. (*) : The 2007 men's Hockey Asia cup was the
Noakhali - West Bengal
seventh tournament of the hockey Asia cup for men. It
85. At which place of Bihar did Gandhiji start was held from August 31 to September 9, 2007 in
Satyagraha movement for the first time in
Chennai, India. India won defeating Korea in the finals
India?
(a) Patna (b) Gaya (7-2). Prabodh Tirkey was the captain of Indian Hockey
(c) Madhubani (d) Champaran team, while Prabhjot Singh was forwards player of this
Ans. (d) : The Champaran Satyagraha of 1917 was the hockey team.
first Satyagraha movement led by Gandhi in India and 91. The longest day in southern hemisphere is -
is considered a historically important revolt in Indian (a) 22 June (b) 22 December
Independence movement. It was a farmer's uprising that (c) 21 March (d) 22 September
took place in Champaran district of Bihar Ans. (b) : The longest day in southern hemisphere
86. The 'Bodhi Vriksh' in Bodh Gaya belongs to occurs on 22nd December. It is known as winter solstice.
_____ generation of its family. In the southern hemisphere, it is summer season.
(a) Third (b) Fourth
(c) Fifth (d) Sixth 92. 'Great Barrier Reef' is situated near -
(a) South America (b) Europe
Ans. (c) : The 'Bodhi Vriksh' in Bodh Gaya belongs to
fifth generation of its family. Gautam Buddha attained (c) Australia (d) Africa
enlightenment under this tree. The Bodhi Vriksh is a Ans. (c) :The Great barrier reef a complex of coral
sacred place to Buddhist. reefs, shoals and islets in the pacific ocean of the
87. Where is 'Vishwa Shanti Stupa' located in northeastern coast of Australia that is the longest and
Bihar? largest reef complex in the world. The Great Barrier
(a) Vaishali (b) Nalanda reef extends roughly in northwest to southeast direction
(c) Rajgir (d) Patna for more than 1900 km at an offshore distance ranging
Ans. (c) : The Vishwa Shanti Stupa is located at Rajgir from 16 to 160 Km. It covers an area of some 350000
in Bihar on a 400 m high Ratnagiri hill. It is also known square km.
as peace Pagoda. The stupa is built of marbles and has 4
93. 'Dakshin Gangotri' is located in
statues of Buddha on the 4 corners of stupa representing
(a) Uttarakhand (b) Arctic
his life period of birth, enlightenment, preaching and
death. There is also a Ropeway to reach the top of the (c) Himalayas (d) Antarctica
hill. Ans. (d) : Dakshin Gangotri was the first scientific base
88. 'Nav Nalanda Mahavir' is famous for station of India situated in Antarctica, part of the Indian
(a) Hiuen-Tsang memorial Antarctic programme. It is located at a distance of 2500
(b) Birth place of mahaver km from the south pole. It was established during the
(c) Pali Research Centre third Indian expedition to Antarctica in 1983-84. While
(d) Musuem the second and third research centres are Maitri and
Ans. (c) : Nav Nalanda Mahavir is famous for Pali Bharati Respectively. As of today India has two
Research Centre Institute. The university was operational research stations in Antarctica named -
established in 1951. Dr, Rajendra Prasad laid the Maitri and Bharati.
foundation of the university. It became a deemed
94. A mountain range of Europe is -
university in 2006.
(a) Alps (b) Himalayas
89. By whom was the first republic of the world
established in Vaishali (c) Andes (d) Rocky
(a) Maurya (b) Nanda Ans. (a) : An important range of Europe is- " The Alps.
(c) Gupta (d) Licchavi This mountain is extended in so many countries namely
Ans. (d) : Vaishali, the world's first republic was Germany, Austria, Slovenia, Monaco, Italy, Switzerland
founded by Licchavi. Vaishali is considered to be world and France. Mont Blanc is the highest peak of the Alps
first republic and currently an Archaeological site. It is mountain range having an elevation of 4807.81m above
situated in the district with the same name as Vaishali in sea level.
the Bihar state of India. Vaishali is mentioned in Jain 95. The longest river of South America is -
and Buddhist's text. (a) Nile (b) Amazon
(c) Mississippi (d) Ganga
48th-52th BPSC (Pre) Exam. 2007-08 157 YCT
Ans. (b) : The Amazon river is the largest and second Ans. (a) : The National Highway carries about 2
longest river in the world. It originates from the Andes percent of the total road network but carry 40 percent of
mountain and drain into the Atlantic ocean, passing the road traffic in India. Out of the total length of
through the Brazil. The Amazon river basin stretched national highways, 32 percent is single lane or
from the Brazilian highland in the south to the Guiana intermediate lane, 56 percent is 2-lane standard. There
highlands in the north. Its area is around 2.7 million has been a consistent increase in the construction of
square miles. National Highways with 13, 327 km being constructed
96. The share of agriculture and allied sectors in in 2020-21 from 4410 km of NH construction in 2014-
the Gross Domestic products of India is - 15
(a) 22 percent (b) 80 percent
102. The 47.48 percent irrigation potential of India
(c) 33 percent (d) 12 percent
is created through -
Ans. (a) : During the question hour, the contribution of
(a) Major Projects
agriculture and allied sector in India's GDP was 22
percent, whereas in the present, It is equal to 15.4 (b) Minor and Major Projects
percent, and except it the contribution of the service (c) Minor Projects
sector is 54.40 percent while the contribution of (d) Medium Projects
manufacturing sector is 29.73 percent. Ans. (b) : The maximum part of the total irrigated land
97. The increase in literacy rate in India during in the country is irrigated through minor and major
1991-2001 is projects. The planning commission had divided the
(a) 10.8% (b) 12.6% irrigation planning in India into three kinds –
(c) 14.3% (d) 15.5 % a. Major Irrigation Plane :- Often large canals and
Ans. (b) : The increase in literacy rate in India during multipurpose river valley projects are included in it,
1991-2001 is 12.6%. having cultivable command areas of 10000 hectare or
98. The three largest states of India in order of more.
area are - b. Medium Irrigation plan :- There irrigation projects
(a) Rajasthan, Madhya Pradesh, Maharashtra are included in it which has cultivable common areas of
(b) Madhya Pradesh, Rajasthan, Maharashtra 2000 hectare .
(c) Maharashtra, Rajasthan, Madhya Pradesh c. Minor Irrigation plan :- Having cultivable common
(d) Madhya Pradesh, Maharashtra, Rajasthan areas of less than 2000 hectare.
Ans. (a) : When the question was asked, the five largest 103. India gets maximum rainfall mainly from-
states of India are - Rajasthan , Madhya Pradesh,
(a) North-East Monsoon
Maharashtra, Andhra Pradesh and Uttar Pradesh. At
(b) Retreating Monsoon
present the correct order is Rajasthan, Madhya Pradesh,
(c) South-West Monsoon
Maharashtra, Uttar Pradesh and Andhra Pradesh.
(d) Convectional rainfall
99. Ranchi city is located in -
(a) Bihar (b) Madhya Pradesh Ans. (c) : In India 75% of the total rainfall is caused by
(c) Orissa (d) Jharkhand southwest-monsoon (June to September), Remaining
Ans. (d) : The Ranchi city is located in the states of 13% of it by northeast monsoon (October to December),
Jharkhand. The city 'Ranchi' is the capital of the Indian 10% of it By Pre-monsoon cyclonic rainfall (mainly in
states of Jharkhand. April and May) and 2% of it by western disturbances
100. Coal accounts for _____ percent commercial (December to February). Hence it is clear from above
energy requirement of India that India gets maximum rainfall from South-west
(a) 76 (b) 56 monsoon.
(c) 67 (d) 52 104. The estimated coal reserves in Bihar is
Ans. (c) : When the question was asked, the coal used (a) 1260 Crore tonnes (b) 303 Crore tonnes
to get 67 percent of India's commercial energy whereas (c) 25310 Crore tonnes (d) 16 Crore tonnes
at present about 54 percent of the India's energy is Ans. (d) : After the bifurcation of Jharkhand from the
state of Bihar almost all the natural resources have gone
obtained from the coal based thermal power station.
to the state of Jharkhand. A few of them left in Bihar.
101. ____ carries 40 percent of road traffic of India. According to the latest report, the proven reserve of coal
(a) National Highway (b) State Highway in Bihar are zero, while that estimated reserve are 160
(c) District Highway (d) Rural Highway million tonnes.

48th-52th BPSC (Pre) Exam. 2007-08 158 YCT


105. The railway zone headquarters Hajipur is (a) Maharashtar (b) Madhya Pradesh
located in (c) Karnataka (d) Bihar
(a) Chhattisgarh (b) Uttar Pradesh Ans. (d) : According to the population in India, Bihar is
(c) Jharkhand (d) Bihar at the third place in the country and after coming into
Ans. (d) : The East Central Railway (ECR) is one of the existence of Telangana, Bihar, currently ranks at
18 railway zones in India. It is headquartered at Hajipur thirteenth place in terms of area. It is noteworthy that
and comprises Sonpur, Samastipur, Danapur, Pt. Deen Uttar Pradesh is at the top in terms of population and
Dayal Upadhayaya, and Dhanabad divisions. Rajasthan is at the top state in terms area.
106. The percentage of net cultivated area out of the 111. In how many Adhikarnas is the Kautilya's
total area in Bihar is- Arthashastra?
(a) 60 (b) 40 (a) 11 (b) 12
(c) 80 (d) 70 (c) 14 (d) 15
Ans. (a) : The total geographical area of Bihar states is Ans. (d) : Kautilya's Arthashastra is considered one of
93.60 lakh hectare in which the net cultivated area is the most sophisticated and detailed treatise to have ever
56.03 lakh hectare. The district with the highest net been produced in India. Kautilya's Arthashastra is
sown or cultivated area in Bihar is Buxar. consists of 15 Adhikaranas (books) out of which the
107. Which is a joint irrigation project of Bihar and first five deal with tantra (internal administration) and
Uttar Pradesh next eight with avapa (inter-state) relations, the final
(a) Damodar Valley Project two books deal with other miscellaneous topics.
(b) Kosi Project 112. Who identified the name Sandrokottos as
(c) Son Barrage Project Chandragupta Maurya?
(d) Gandak Project (a) William Jones (b) V.A. Smith
Ans. (d) : The Gandak Project is a joint project of Bihar (c) R. Mukherjee (d) Dr. B.R. Ambedhar
and Uttar Pradesh along with Nepal. The Ans. (a) : Chandragupta Maurya was first identified by
Hydroelectricity is produced at Suratpura (Nepal) on the William Jones as 'Sandrokottos'. The mauryan dynasty
river Gandak. The Dam has been built at Bhaisalotan was founded by Chandragupta Maurya with the help of
(Valmiki nagar) in Bihar, The river Gandak originates his mentor Chanakya or Kautilya.
from the Himalayas in Nepal. It joins the Ganga at 113. Who deciphered the Brahmi inscription of
Sonpur east of Patna. Ashoka for the first time?
108. According to 2001 census _____ percent (a) James Prinsep (b) H.D. Sankalia
population of India lives in Bihar state. (c) H.D. Sankalia (d) V.N. Mishra
(a) 8 (b) 10 Ans. (a) : Ashoka used several languages in his
(c) 11 (d) 12 inscriptions, such as Brahmi, Kharosthi, Prakrit,
Ans. (a) : According to the data of census 2001 the total Aramaic and Greek languages. It is noteworthy that the
population of Bihar was 8,29,98,509, which was 8.07 Brahmi inscription was first deciphered by James
percent of the total population of India. According to Princep in 1837 CE.
the current data of census, 2011, the population of Bihar
114. The last Mauryan emperor was?
is 10,40,99,452 which is 8.6 percent of the total
(a) Jalok (b) Avanti Varman
population of the country.
(c) Nandi Varmana (d) Briha Dratha
109. Bihar is the ___ largest state of India according
Ans. (d) : The Mauryan emperor declined rapidly after
to 2001 census.
(a) Second (b) Third Ashoka and the later kings had very short reigns. The
(c) Fourth (d) Fifth emperor became weak and fragmented and the mauryan
Ans. (b) : According to the census, 2001 Bihar was the dynasty came to an end when the last king Brihadratha
third largest in terms of total population after Uttar was killed by his military commander, Pushyamitra
Pradesh and Maharashtra. According to the current data Sunga, in 187 BCE.
of 2011 census, the population of Bihar is 10,40,99,452 115. Iltutmish appointed ____ his first subedar in
which also stands at third ranks after Uttar Pradesh and Bihar -
Maharashtra in Terms of Population. (a) Aivaj
110. In India the third largest state according to (b) Nasiruddin Muhammad
population and the twelfth largest state in area (c) Alimaran
is (d) Malik Jani

48th-52th BPSC (Pre) Exam. 2007-08 159 YCT


Ans. (d) : Iltutmish appointed Malik Jani as the Ans. (a) : The Partition of Bengal was cancelled in
Governor or Subedar of Bihar. In 1225 Iltutmish 1911. It was cancelled by Lord Hardinge-II. It is
ordered his army to reoccupy Bihar and Bengal. As a important to note that the Bengal had partitioned in the
result his army reoccupied Bihar and was prepared to year of 1905. It was partitioned by Lord Curzon. The
attack Bengal, Sultan Ghiyasuddin pleaded with partition was enacted on 20th July 1905 and came into
Iltutmish not to attack Bengal and in return he will existence or effect 16th October 1905. Swadeshi
accept the sovereignty of Delhi. Iltutmish accepted the movement was one of the main protests against the
offer and ordered his army to return to Delhi and partition of Bengal.
appointed Malik Jani as the Governor of Bihar. 121. When did the British Government Appoint
116. When did Humayun first attacked Chunar Saddler University Commission for reforms in
fort? education?
(a) 1532 A.D. (b) 1531 A.D. (a) 1919 (b) 1917
(c) 1533 A.D. (d) 1536 A.D. (c) 1921 (d) 1896
Ans. (a) : In 1532 CE at Daurah he defeated the Afghan Ans. (b) : Saddler commission was set up in 1917, to
forces which had conquered Bihar and Jaunpur. Then he study and report on problems of Calcutta University. It
besieged the powerful fort of Chunar from the Afghan reviewed the entire field from school education to
Sardar Sher khan which commanded the land and the university education. It held the view that for the
river route between Agra and the east end. Which was improvement of university education, improvement of
known as the gateway of eastern India. secondary education was a necessary precondition.
117. The meaning of word Bantai during medieval 122. Which person of Jagdishpur took the lead of
period was? revolutionaries during the revolt of 1857 A.D.?
(a) Religious Tax (a) Kunwar Singh (b) Chandra Shekhar
(b) System of calculating revenue (c) Tirat Singh (d) Ram Singh
(c) Wealth tax Ans. (a) : Babu Kunwar Singh was the leader of
(d) Property tax revolution in Bihar during the Revolt of 1857. He was
Ans. (b) : The meaning of word " Bantai" during the associated with a family of the Ujjainiya clan of the
medieval period was a system of calculating revenue. In Parmar Rajputs of Jagdishpur, currently a part of
this system produce was divided between the peasants Bhojpur district, Bihar, India.
and the state in fixed proportions. 123. Emperor Shah Alam II gave the diwani of
118. Akbar merged Bengal and Bihar in his empire Bengal-Bihar and Orissa to East India
in: Company on-
(a) 1590 A.D. (b) 1575 A.D. (a) 12 August 1765 (b) 18 August 1765
(c) 1576 A.D. (d) 1572 A.D. (c) 19 August 1765 (d) 21 August 1765
Ans. (b) :. In 1576 CE Akbar along with Khan-i- Ans. (a) : Battle of Buxar ended with the signing of
Khanan Munaim Khan, marched toward Bihar. In a Treaty of Allahabad between Shah Alam II and Robert
short time he captured Hazipur, Patna and Gaur Clive of East India company on 12th August 1765. It
(Bengal). gave Diwani rights or right to collect taxes of Bengal,
119. When did Queen Victoria declare the taking Bihar and Orissa to the East India company.
over of Indian administration under the British 124. The first railway lines were laid down in India
Crown? under British Governor-
(a) 1 November 1858 (b) 31 December 1975 (a) Lord Dalhousie (b) Lord Curzon
(c) Lord Wellesley (d) Lord Lytton
(c) 6 January 1958 (d) 17 November 1859
Ans. (a) : Lord Dalhousie (1848-56) the Governor-
Ans. (a) : Queen Victoria's proclamation (1858) was
General of India under whom the first railway line
declared openly by Lord canning on November 1, 1858.
connecting Bombay to Thane was laid in 1853. Though
The act of proclaiming Queen Victoria as the Queen of
it was fully extended in all over the British India during
India is popularly known as the Government of India the period of Governor-General Lord Curzon.
Act, 1858. Lord Dalhousie also passed the post office act in 1854.
120. In which year did Lord Hardinge cancel the 125. When did the attempt of the murder of
partition of Bengal? Kingsford made at Muzzaffarpur?
(a) 1911 (b) 1904 (a) 1908 (b) 1909
(c) 1906 (d) 1907 (c) 1907 (d) 1911
48th-52th BPSC (Pre) Exam. 2007-08 160 YCT
Ans. (a) : Khudiram Bose and Prafulla Chaki the young 130. Which was the working area of Birsa Munda?
India freedom fighter went to Muzzaffarpur, Bihar to (a) Champaran (b) Ranchi
assassinate Kingsford, Calcutta presidency Magistrate (c) Balia (d) Alipur
On 30 April 1908 they throw the bombs and blew the Ans. (b) : Birsa Munda was active in the area of
carriage, unfortunately the vehicle was not carrying Ranchi, Birsa Munda was an Indian tribal freedom
Kingsford and instead two innocent British ladies were fighter, religious leader and folk hero who belonged to
killed. Prafulla committed suicide when cornered by the Munda tribe.
police at the Samastipur railway station and Khudiram 131. Jai Prakash Narayan Belonged to -
was later arrested and hanged. (a) Congress Party (b) Communist Party
126. Prabhawati Devi was the Freedom fighter of- (c) Socialist Party (d) Kisan Sabha
(a) Champaran (b) Patna Ans. (c) : Jai Prakash Narayan was born in 1902 in
(c) Bhagalpur (d) Shahabad Sitab Diara, Saran district, Bihar. He was influenced by
Ans. (b) : Prabhawati Devi Narayan was an Indian marxist ideas in the USA and Gandhian ideology. In
Independence activist from the present day state of1934 he formed the Congress socialist party with the
Bihar and she was wife of Indian independence and following members Acharya Narendra Deva, Ram
social activist, Jayprakash Narayan, Prabhawati Devi
Manohar Lohia, Minoo Masani , Achyut Patvardhan
belongs to the Patna region of Indian independence and Ashok Mehta.
region. He was popularly known as Lok Nayak as he was
127. When did Baldev Sahai resign from the post of considered the people's leader.
Advocate General? 132. The leader of the revolt of 1857 in Bihar was -
(a) 1942 (b) 1943 (a) Maulvi Ahmedullah (b) Tatya Tope
(c) 1913 (d) 1911 (c) Nana Sahib (d) Kunwar Singh
Ans. (a) : When the Quit India movement started on Ans. (d) : The Rebellion in Bihar was led by Babu
August 9, 1942, the Britishers acted brutally against the Kunwar singh. He was nearly 80 year old when he took
Indian people as a result top leaders were arrested form the charge of the sepoys who were placed at Danapur on
the country, In against of this protest the advocate
general of Bihar, Baldev Sahai resigned from his post in 25 july 1857. He was distinguished leader during the
1942. Indian Rebellion of 1857. In march 1858, Babu Kunwar
128. In which jail was Dr. Rajendra Prasad Singh conquered Azamgarh (now in U.P.).
Prisoned in the movement of 1942? 133. Why did Mahatma Gandhi support the
(a) Bankipur Jail (b) Hazaribagh Jail Khilafat movement?
(c) Camp Jail (d) Bhagalpur Jail
(a) The Khalifat had given shelter to Indian
Ans. (a) : Dr. Rajendra Prasad was arrested several
times during the National struggle and imprisoned by revolutionaries
British Government during the Salt Satyagraha of 1931 (b) Gandhiji wanted to win the support of the
and the Quit India movement of 1942. He was very Indian Muslims against the Britishers
much active in Quit India movement in 1942 and led (c) The Khalifa was support Indian struggle for
protest demonstration in Bihar. He was arrested in freedom
Sadaqat Ashram, Patna and sent to Bankipur Central
(d) The Khalifa was personal friend of Gandhiji
Jail in 1942.
129. Who was the national leader of Champaran Ans. (b) : Mahatma Gandhi supported the Khilafat
Indigo movement ? movement in order to unite the people of Hindu and
(a) Mahatma Gandhi (b) Birsa Munda Muslim religion and revolt against the British empire.
(c) Baba Ram Chandra (d) Ram Singh Hence, it is correct that Gandhiji wanted to win the
Ans. (a) : Mahatma Gandhi was the national leader of support of the Indian Muslim against the Britishers.
Champaran Indigo movement. Mahatma Gandhi was 134. The method of moderate leaders of the
persuaded by Pandit Rajkumar Shukla one such congress was-
oppressed indigo cultivator to visit the region. As a (a) Non Co-operation
(b) Constitutional Agitation
result Gandhi visited and the champaran sathyagraha
(c) Passive Resistance
began. (d) Civil Disobedience

48th-52th BPSC (Pre) Exam. 2007-08 161 YCT


Ans. (b) : The national leaders like Dadabhai Naoroji, (a) Bengal British India Society
Firoz Shah Mehta, D.E. Wacha, W.C. Bonnerjee, S.N. (b) East India Association
Banerjee who dominated the congress policies during (c) Young Bengal Association
the early period (1885-1905) were believers in (d) Indian Association of Calcutta
Liberalism and Moderate politics and therefore came to Ans. (d) : The Indian Association of Calcutta was
labeled as Moderates. The constitutional agitation was founded by Surendranath Banerjee and Ananda Mohan
the primary methods of these moderate leaders, they Bose. the Indian Association of Calcutta was founded in
also used the method of "Prayer and petition." 1876. The objectives of the Association were
135. Why did people gather to demonstrate at "Promoting by every legitimate means the political,
Jallianwala Bagh? intellectual, and material advancement of the people.
(a) To protest against the arrest of Gandhiji and 139. Who was the founder of Forward Block?
Lajpat Rai (a) Jai Praksah Narayan
(b) To protest against the arrest of Kitchlew and (b) C.R. Das
Satyapal (c) Moti Lal Nehru
(c) To offer prayers on the day of Baisakhi (d) Subhash Chandra Bose
(d) To protest against the inhuman acts of the Ans. (d) : All India Forward Block (AIFB) is a left
Punjab government wing nationalist political party founded by Subhas
Ans. (b) : People were gathered at Jallianwala Bagh in Chandra Bose in West Bengal in 1939. On the
Amritsar to protest against the arrest of Saifuddin formation of this party Netaji said that who all were
Kitchlew and Satyapal on 13th April, 1919. joining the forward Block, they has to never turn their
136. The women's movement in India started back to the Britisher's camp and must fill the Oath form
largely under the inspiration of? by cutting their finger and signing it with their blood in
(a) Ramabal Ranade (b) Annie Besant the form. All India Conference of forward Black held in
(c) Sarojini Naidu (d) Jyotiba Phule Nagpur 1940.
Ans. (d) : Jyotiba Phule was a great social reformer of 140. Who was the first President of All India Trade
the 19th century and he was a strong supporter of Union congress?
women's education with the help of his wife Savitri (a) B.T. Ranadev (b) Satya Bhakta
Devi. He established a school in Pune in 1948 for the (c) Lala Lajpat Rai (d) N.M. Joshi
education of girl child. He also supported widow Ans. (c) : The All India Trade Union Congress
remarriage as well as he established a maternity home (AITUC) is the oldest trade union federations in India.
for poor or helpless women. He has written many books It is associated with the Communist Party of India. It
against the social evils in which "Gulamgiri" is
was founded on 31st October 1920 with Lala Lajpat Rai
prominent one. He laid the foundation of "Dean Bandhu
as it first President.
Sarvajanik Sabha".
141. Where was the first All India Kisan
137. Who among the following was the first Indian
Mahasabha formed?
to start an agitation on political reforms?
(a) Calcutta (b) Madras
(a) Surendranath Banerjee
(c) Lucknow (d) Patna
(b) Dadabhai Naoroji
(c) Ram Mohan Rai Ans. (c) : The All India Kisan Sabha (also known as the
(d) B.G. Tilak Akhil Bhartiya Kisan Sabha) is the peasant or farmers
wing of the Communist Party in India, an important
Ans. (*) : Raja Rammohan Roy was the first Indian to
peasant movement founded in 1936 by Sahajanand
start an agitation for political reform. He advocated the
Saraswati during the Indian National congress
substitution of English in place of Persian language in
courts of law also for employment of India personnel in Lucknow session in 1936. Swami Sahajanand was
civil services. He also advocated the separation of elected as its first President.
offices of judges & revenue commissioners. 142. The revolt of 1857 at Lucknow was led by:
138. The most important of the pre-congress (a) Begum of Awadh (b) Tatya Tope
nationalist organization was the - (c) Rani Lakshmibai (d) Nana Saheb

48th-52th BPSC (Pre) Exam. 2007-08 162 YCT


Ans. (a) : Begum Hazrat Mahal led the revolt of 1857 Ans. (a) : In July 1931, Jay Prakash Narayan, Phulan
in Lucknow. She was also called Begum of Awadh. Prasad Verma and Ganga Sharan Sinha had established
Begum Hazrat Mahal Seized the centre of Lucknow and the "Bihar socialist party therefore Jayprakash was
declared her son as the ruler. considered as the founder of "Bihar Socialist Party"
143. When was The Rowlatt Act passed? while congress socialist party was founded in Patna
(a) 1919 (b) 1920 ( Bihar) on 17th May, 1934.
(c) 1921 (d) 1922 148. Who founded Patliputra?
Ans. (a) : The Rowlatt act (Black Act) was an act (a) Udayin
passed by the Imperial legislative council in Delhi on (b) Ashoka
21st March 1919. It was enacted by British to discourage (c) Bimbisar
Indians from rising against them by suppressing (d) Mahapadmananda
revolutionary groups and depriving Indians of their Ans. (a) : Patliputra or Modern Patna was originally
Right to personal expression and liberty. built by Udayin in the 5th century. Udayin was the son
144. Gandhiji launched the Non-cooperation and successor of Haryanka King Ajatshatru. It evolved
movement in - as the capital of leading power in historical India, such
(a) 1920 (b) 1919 as the Nanda Emperor the Maurya Emperor and the
(c) 1921 (d) 1922 Gupta dynasty etc.
Ans. (a) : Gandhiji launched formally the non- 149. Which one of the following is correct about the
cooperation movement in August 1920 at the Nagpur permanent settlement introduced in Bihar?
session of Indian National Congress, the programme of (a) The Zamindars were deprived of the
non-cooperation was endorsed and an important change ownership of the land
was made in the congress creed, now instead of having (b) The right of ownership of land was made
the attainment of self government through constitutional
hereditary transferable for the Zamindars
means as its goals, the congress decided to have the
(c) Land revenue was constitutional
attainment of Swaraj through peaceful and legitimate
(d) Abolition of Zamindari
means.
Ans. (b) : Permanent settlement Act was introduced by
145. Quit Indian movement began on :
Governor General Lord Cornwallis in 1793. It was first
(a) 8th August 1942 (b) 10th August 1942
introduced in Bengal and Bihar. The permanent
(c) 15th August 1942 (d) 16th August 1942
settlement act of 1793 had the following features :-
Ans. (a) : on August, 8 1942, Gandhi Ji gave a clarion
• Zamindars were given hereditary right for succession
call to end the British rule and launched the Quit India
of land their ownership.
movement at the session of the all-India Congress
Committee in Mumbai. • The Zamindar who were earlier only tax collectors
becomes landlords under this system.
146. After the revolt of 1857 the British recruited
the soldiers from the - • Zamindars had the right to transfer or put himself the
(a) Brahmins of U.P. & Bihar property.
(b) Bengalis and Oriya people from the East 150. The Revolutionary phase of the trade union
(c) Gurkhas, Sikhs and Punjabis in the North movement in India was -
(d) Madras presidency and Marathas (a) 1939-45 (b) 1926-39
Ans. (c) : After the revolt of 1857 the British recruited (c) 1918-26 (d) 1917-18
the soldiers in the British Indian Army from Gorakhas, Ans. (b) : The Revolutionary phase of the trade union
Sikhs and Punjabis in the North on a large scale and
movement would be traced back to 1926-39. A strong
declared them as a Martial communities.
communist influence on the movement laid a militant
147. Bihar socialist party was founded by -
(a) J.P. Narayan and revolutionary extent to it. In 1928 there was a six
(b) Satya Bhakta month long strike in Bombay textiles mills led by the
(c) M.N. Roy Girni Kamgar Union. The whole of 1928 witnessed of
(d) Subhash Chandra Bose unprecedented industrial unrest.

48th-52th BPSC (Pre) Exam. 2007-08 163 YCT


53th-55th Bihar Public Service Commission
Preliminary Examination, 2011
GENERAL KNOWLEDGE & GENERAL SCIENCE
(Solved Paper with Detail Explanation)
1. In which country is 'Takla Makan' desert Ans. (c): Orogenic force is the mountain building force.
situated? It occurs when two tectonic plates Collide with each
(a) Kazakhstan (b) Turkmenistan other forcing material upwards to form mountain belts
(c) Uzbekistan (d) China such as Alps or Himalayas or Causing one plate to be
Ans. (d) : Takla Makan is a temperate and largest desert subducted below the other resulting in volcanic
of China and 2nd largest desert of the world. mountain chains such as Andes.
2. Which strait connects Red Sea and Indian 7. Amritsar and Shimla are almost on the same
Ocean? latitude, but their climate difference is due to
(a) Bab-el-mandeb (b) Hormuz (a) the difference in their altitudes
(c) Bosphorus (d) Malacca (b) their distance from sea
Ans. (a) : (c) snowfall in Shimal
Strait Connects (d) pollution in Amritsar
Bab - el - mandeb - Connects Red sea to Indian Ans. (a) :, Amritsar and Shimla are almost on the same
ocean. latitude but due to difference in their Altitudes they
Hormuz - Connects Persian Gulf to differs in climatic condition.
Gulf of Oman 8. Coal mines in Jharkhand are located at
Bosphorus - Black sea to Aegean sea. (a) Jharia (b) Jamshedpur
Malacca - Andaman sea (Indian Ocean) (c) Ranchi (d) Lohardaga
to South China sea (Pacific Ans. (a) : Coal mines in Jharkhand are located at Jharia.
Ocean) Jharia Coalfield is one of the largest coal field in India.
3. Where is 'Ninety East Ridge' situated? 9. Major source of oceanic salinity is
(a) Pacific Ocean (b) Indian Ocean (a) River
(c) Atlantic Ocean (d) Arctic Ocean (b) Land
Ans. (b) : Ninety East Ridge is a mid ocean ridge (c) Wind
situated in Indian Ocean. (d) Ash from volcanoes
4. Which of the following is not a capital city? Ans. (a) :Salt in the ocean comes from two sources: run
(a) Canberra (b) Sydney off from the land and openings in the seafloor. Rocks
(c) Wellington (d) Riyadh on land are the major source of salts dissolved in
Ans. (b) : In the given options Canberra is the capital seawater. Rainwater that falls on land is slightly acidic
city of Australia, Wellington is the capital city of New so it erodes rocks. This releases ions that are carried
Zealand and Riyadh is the capital city of Saudi Arabia away to streams and rivers that eventually fell into
while Sydney is a city in Australia, it is not capital city. ocean. Surface salinity is greatly influenced in coastal
regions by fresh water flow from rivers.
5. The highest coal-producing country in the
world is 10. The large States of India in the order of area
(a) India (b) USA are
(c) China (d) Russia (a) Rajasthan, Madhya Pradesh, Maharashtra
(b) Madhya Pradesh, Rajasthan, Maharashtra
Ans. (c) : China is leading coal producer in the world
followed by India and U.S.A. (c) Maharashtra, Rajasthan, Madhya Pradesh
(d) Madhya Pradesh, Maharashtra, Rajasthan
6. Folding is the result of
(a) Epeirogenetic force (b) Coriolis force Ans. (a) : Rajasthan is the largest state in terms of area
(c) Orogenic force (d) Exogenic force followed by Madhya Pradesh and Maharashtra.

53th-55th BPSC (Pre) Exam. 2011 164 YCT


11. Bihar is the -- largest State of India according Ans. (b) : The Headquarter of East Central Railway
to Census 2001. Zone is located at Hajipur in Bihar.
(a) Second (b) Third 18. Where is Industries Commissioner Office of
(c) Fourth (d) Fifth Bihar situated?
Ans. (b) : As per census report 2001, and 2011, Bihar is (a) Patna (b) Gaya
the third largest state after Uttar Pradesh and (c) Hajipur (d) Muzaffarpur
Maharashtra in terms of population. Ans. (a) : Industries Commissioner Office of Bihar is
12. The highest wheat-producing State of India is situated at Patna.
(a) Haryana (b) Punjab 19. North Bihar is famous for
(c) Bihar (d) Uttar Pradesh (a) Agricultural prosperity
Ans. (d) : In terms of Production of wheat Uttar (b) heavy industries
Pradesh stands first while in terms of productivity of (c) flood
wheat Punjab ranks first. Hence option (d) is the right (d) famine
answer.
Ans. (c) : North Bihar is famous for floods. When rivers
13. TISCO plant is located near originating from Central Nepal and Nepal receives
(a) Patna (b) Darbhanga heavy water from rainfall and enters Bihar, they flow in
(c) Dhanbad (d) Tatanagar plains and lowlands of Bihar and Break their banks.
Ans. (d) : TISCO (Tata Iron and Steel Company) River Narayani, Bagmati and Kosi are famous for
Limited is located at Tatanagar, Jamshedpur in shifting their course.
Jharkhand. 20. Which one is not located on National Highway-
14. Which States in India are the largest producers 2 (NH-2)?
of sugarcane? (a) Aurangabad (b) Sasaram
(a) Bihar and Uttar Pradesh (c) Mohania (d) Patna
(b) Uttar Pradesh and Rajasthan Ans. (d) : Patna is not located at National Highway - 2
(c) Andhra Pradesh and Jammu and Kashmir (NH-2), Whereas, Aurangabad, Sasaram and Mohania
(d) Punjab and Himachal Pradesh are located at NH-2.
Ans. (a) : Among the given combination of states 21. Congress passed the 'Swaraj' resolution in the
largest producers of sugarcane are Bihar and Uttar year 1905. The purpose of the resolution was
Pradesh. (a) Right to make a constitution for themselves
In India Uttar Pradesh stands first in the production of but it was not done
sugarcane followed by Maharashtra, Tamil Nadu and (b) To secure self-rule
Karnataka. (c) Responsible
15. Which of the following States has the largest (d) Self-government
number of people belonging to Scheduled Ans. (d) : In calcutta session of 1906. Under the
Caste? leadership Dadabhai Naoroji. Congress adapted Swaraj
(a) Bihar (b) West Bengal as the goal of Indian People. At this session, resolution
(c) Uttar Pradesh (d) Punjab partition of Bengal, self Government (Swaraj).
Swadeshi, national education, Boycott were passed.
Ans. (c) : Population of Scheduled Caste is highest in
This question was deleted by BPSC.
Uttar Pradesh and population of Scheduled tribe is
highest in Madhya Pradesh. 22. The Provincial governments were formed
under which of the following Act?
16. According to 2001 Census, what is the total
(a) 1935 (b) 1932
population of Bihar?
(c) 1936 (d) 1947
(a) 82998509 (b) 76210007
(c) 96878627 (d) 80176197 Ans. (a) : The Government of India Act 1935 had
established Provincial Government.
Ans. (a) : As per census report 2001 the total
Some other important features of Government of India
population of Bihar was 82,998,509 and as per census
report 2011 the total population of Bihar is Act 1935 as below -
104,099,452. • It abolished diarchy at provinces and introduced it
at Centre.
17. The East Central Railway zone headquarters is
located at • Bicameral legislature is introduced in Provinces.
(a) Patna (b) Hajipur • It established Federal Court
(c) Muzaffarpur (d) Katihar • Establishment of RBI.

53th-55th BPSC (Pre) Exam. 2011 165 YCT


23. The Indian Constitution comprises of (c) The Rajya Sabha has to pass Bills within 14
(a) 395 Articles, 22 Parts and 12 Schedules days after it is passed by the Lok Sabha
(b) 317 Articles, 21 Parts and 11 Schedules (d) The Rajya Sabha may pass orreturn the
(c) 372 Articles, 20 Parts and 7 Schedules Money Bill with some recommendations to
(d) 381 Articles, 23 Parts and 8 Schedules the Lok Sabha
Ans. (a): The India Constitution comprises of 395 Ans. (b) : A money bill is defined in Article 110 of the
Indian Constitution. It originates in Lok Sabha only.
Articles, 22 Parts and 12 Schedules.
The power of Rajya Sabha related to money bill is
24. The Rajya Sabha consists of limited. e.g. Rajya Sabha may pass or return the money
(a) 280 members of which 20 members are bill with some recommendations to Lok Sabha but Lok
nominated by the President of India Sabha has the power to accept or reject these
(b) 275 members of which 18 members are recommendations. Rajya Sabha can only withhold this
nominated by bill for a maximum of 14 days.
(c) 250 members of which 12 members are 28. Which of the following rights conferred by the
nominated by the President of India Constitution of India is also available remedies
(d) 252 members of which 12 members are (a) Right to constitution remedies
nominated by the President of India (b) Freedom to speech
Ans. (c) : As per Article 80 of the Indian Constitution (c) Freedom to move and settle in any part of the
the council of states shall consist of 238 representatives country
of the states and the Union territories and 12 members (d) Freedom to acquire property
are to be nominated by the President who have special Ans. (a) : Article 32 of the Indian Constitution is a
knowledge or practical experience in respect of fundamental right conferred by the Constitution of India.
literature, Science, Art and Social Service. Under this any individual has the right to approach
directly Supreme Court seeking enforcement of other
25. January 26 was selected as the date for the fundamental rights recognized by the Constitution.
inauguration of the Constitution, because
29. Who is the head of the National Security
(a) the Congress had observed it as the Council?
Independence Day in 1930 (a) Home Minister (b) Prime Minister
(b) on that day the Quit India Movement was (c) President (d) Vice-President
started in 1942
Ans. (b) : National Security Council (NSC) is the
(c) it was considered to be an auspicious day Principal advisory body on the proper coordination and
(d) None of the above integration of plans and Policies affecting national
Ans. (a) : January 26 was selected as the date for the Security. NSC is headed by the Prime Minister.
inauguration of Indian constitution because this was the National Security Advisor (NSA) presides over the
day in 1930 on which the Indian National congress had NSC and is also the primary advisor to the Prime
demanded complete independence or Purna Swaraj as Minister. It is headed by Prime Minister. This question
opposed to the Dominion status. was deleted by BPSC.
26. The Public Accounts Committee submits its 30. According to which Article, 'National
report to Emergency' can be declared by the President of
(a) the comptroller and Auditor General India.
(b) the Speaker of the Lok Sabha (a) Article 352 (b) Article 370
(c) the Minister of Parliamentary Affairs (c) Article 371 (d) Article 395
(d) the President of India Ans. (a) : Under Article 352 of Indian Constitution, the
Ans. (b) : The Public Accounts Committee is formed President can declare a national emergency when the
every year to examine the audit report of Comptroller security of India or part of it is threatened by war or
and Auditor General after it is laid in the Parliament. external aggression or armed rebellion.
The strength of the committee is 22 member of which 31. Which of the following statement is not
15 are from Lok Sabha and 7 from Rajya Sabha. The correct?
committee submits it's report to the Speaker of Lok (a) Supreme Court was constituted in 1950
Sabha. (b) Supreme Court is the highest court of appeal
27. Of the following statements, which one is not in the country
correct? (c) Supreme Court can hear from any high
(a) The Rajya Sabha is powerless in money Court/Tribunals except from Court-martial
matter (d) Supreme court can hear from any High
(b) Money Bills originate in the Rajya Sabha Court/Tribunals as well from Court-martial
53th-55th BPSC (Pre) Exam. 2011 166 YCT
Ans. (d) : From the above given options, all options are II. Transmission and distribution losses are
correct except (d) because Supreme Court can not hear very high
appeal from Court - martial. However High Court and III. Lack of commercial autonomy for SEBs.
Supreme Court have the power of Judicial review over IV. State Governments have implemented
social subsidy policies through SEBs
Court martial proceeding, if these have resulted in
Which of the above are correct?
denial of fundamental rights.
(a) I, II and III (b) I, II, III and IV
32. Who is the constitutional head of the State (c) I, III and IV (d) II, III, and IV
Governments? Ans. (b) : Sale of power to agricultural and domestic
(a) Chief Minister (b) Governor consumers below the cost of production, transmission
(c) Speaker (d) High Court Judge and distribution losses, Lack of Commercial autonomy
Ans. (b) : Governor of the State is the constitutional for SEBs and State Government have implemented
social subsidy policies through SEBs. All these reasons
head of the State Government. Article 154 of
are behind for financial sickness of State Electricity
Constitution of India says that the executive power of Boards (SEBs) in India.
the State shall be vested in the Governor and shall be
37. Which type economy does India have?
exercise by him either directly or through officers (a) Socialist (b) Gandhian
subordinate to him in accordance with the Constitution (c) Mixed (d) Free
of India.
Ans. (c) : India is considered a mixed economy because
33. Commercial sources of energy purely consist of public and private Sectors both co-exist in India.
(a) power, coal, oil, gas, hydro-electricity and 38. When was 'Speed Post Service' launched by the
uranium Indian Postal Department as competition to the
(b) coal, oil, firewood, vegetable waste and 'Courier Service'?
agricultural waste (a) 1988 (b) 1987
(c) power, coal, animal dung and firewood (c) 1989 (d) 1986
(d) coal, gas, oil and firewood Ans. (d) : 'Speed Post Service' launched by the Indian
Postal Department in 1986 in competition to the courier
Ans. (a) : There are Commercial and Non-commercial service and for the purpose of the faster delivery of
sources of energy. Commercial Sources are coal, parcels, letters, cards document and other important
petroleum and electricity and Non - Commercial stuff.
sources are fuel wood, agricultural waste and dried 39. We witnessed the first telephones in India in
dung. This question was deleted by BPSC which year?
34. In India, per capita consumption of energy in (a) 1951 (b) 1981
1994 was (c) 1851 (d) 1861
(a) 300 kg of oil equivalent Ans. (*) : On 28th January 1882, the first in India formal
(b) 360 kg of oil equivalent telephone service was established with a total 93
(c) 243 kg of oil equivalent susbscribers. This question was deleted by BPSC
(d) 343 kg of oil equivalent 40. Consider the following problems being faced by
Ans. (*) : In India, per capita Energy consumption in the Indian Coal Industry:
1994 was 371.23 kg of oil equivalent. This question was I. Poor quality of coal and bottlenecks in the
deleted by BPSC coal movement
II. Low utilisation capacity of washeries
35. When did the first oil crisis/energy crisis occur
III. Growing dependence on the import of
in India? coking coal
(a) During 1950's and 1960's IV. Administered prices.
(b) During 1930's and 1940's Which of the above are correct?
(c) During 1990's and 2000's (a) II, III and IV (b) I, II, III and IV
(d) During 1970's and 1980's (c) I, III and IV (d) I, II and III
Ans. (d) : In India first oil energy crisis occurred during Ans. (b) : Problems being faced by Indian coal industry
1970's after Arab-Israel conflict and then in 1980's are as follows:-
when Iraq invaded Kuwait. 1) Poor quality of coal
2) Dependence on the import of coking coal.
36. Consider the following reasons for financial
3) Low utilization capacity of washeries
sickness of State Electricity Boards (SEBs) in 4) Administered price
India: 5) Use of domestic coal for power generation etc.
I. Sale of power to agricultural and domestic Hence all statements are correct and option (b) is the
consumers below the cost of production right answer.

53th-55th BPSC (Pre) Exam. 2011 167 YCT


41. Name the committee which was set up by the 44. In which year was the 'Planning Commission'
NDC in 1957 for the reorganisation of CDP and set up in India?
NES, which suggested a three tier system of (a) 1950 (b) 1947
rural local government (c) 1948 (d) 1951
(a) Balwant Rai Mehta Committee
Ans. (a) : The Planning Commission was setup in 1950
(b) Ashok Mehta Committee
by a resolution of the Government of India. Now it has
(c) Maharashtra Committee on Democratic
replaced by NITI Ayog.
Decentralisation
(d) Rural-Urban Relationship Committee 45. Patna International Airport is directly
connected with
Ans. (a) : Balwant Rai Mehta committee was setup in
1957 by the NDC for reorganization of Community (a) Kathmandu (Nepal), New Delhi, Kolkata,
Development Programme (CDP) and National Mumbai, Lucknow, Varanasi and Ranchi
Extension Service (NES). The committee suggested the (b) Bengaluru, Hyderabad and Dhaka
establishment of a democratic decentralized local (c) Islamabad, Dhaka and Bengaluru
government and suggested a three tier system of local (d) Washington, Dhaka and Chennai
government known as Panchayti Raj System. Ans. (a) : Patna International Airport is directly
42. Ashok Mehta recommended connected with New Delhi, Kolkata, Mumbai Lucknow,
(a) three-tier government of Panchayati Raj Varanasi, Ranchi and Kathmandu (Nepal)
Committee
46. In Bihar, air services are provided by
(b) two-tier government of Panchayati Raj
(a) only Indian Airlines
(c) single-tier government of Panchayati Raj
(b) only Sahara Airlines
(d) multiple-tier government of Panchayati Raj
(c) only Royal Nepal Airlines
Ans. (b) : Ashoka Mehta committee was setup in 1977
(d) Indian Airlines, Sahara Airlines and Royal
and recommended two tier government in Panchayati
Nepal Airlines
Raj.
Ans. (*) : When question was asked Indian Airlines
43. Consider the following about the 'Rolling
Nepal Airlines Provided air Services in Bihar. Sahara
Plan':
Airlines was taken over by Jet Airways in 2007. This
I. A plan for the current year which includes question was deleted by BPSC
the annual budget
47. For Bihar, in 1993-94, the Planning
II. A plan for a fixed number of years, say 3, 4
Commission estimated that percentage of
or 5
below poverty line (BPL) was
III. It is revised every year as per requirements
of the economy (a) 55 (b) 65
IV. A perspective plan for 10, 15 or 20 years (c) 45 (d) 35
Which of the above are correct? Ans. (a) : In 1993 - 94 the Planning Commission
(a) I and II estimated that percentage of Below Poverty Line (BPL)
(b) I and III in Bihar was 55%.
(c) II and III 48. In Bihar, though 'Zamindari' was statutorily
(d) I, II, III and IV abolished In 1952, the social base of land
Ans. (d) : The Rolling Plan consisted of three kinds of control remained in the grip of
Plans :– (a) Middle Caste Hindus
(i) The first Plan was for the present year, which (b) Scheduled Caste Hindus
comprised the annual budget. (c) Dominant Caste Hindus
(ii) The Second plan was for a fixed number of years (d) Scheduled Tribe Hindus
may be 3, 4 or 5 years. The Second plan kept
changing as per the requirements of the Indian Ans. (c) : Zamindari was statutorily abolished in 1952
economy. however the social base of land control remained in
(iii)The third Plan was a perspective plan for long terms hands of dominant caste Hindus.
maybe 10, 15 or 20 years. 49. Bihar State (before separation) had how many
Rolling plans are revised every year as per 'Industrial Area Development Authorities'?
requirements of the economy. (a) Six, viz., Adityapur, Bokaro, Patna, Ranchi,
Thus, there is no fixation of dates in respect of Darbhanga and Muzaffarpur
commencement and end of the rolling plans. This (b) Five, viz., Adityapur, Bokaro, Darbhanga,
question was deleted by BPSC Muzaffarpur and Patna

53th-55th BPSC (Pre) Exam. 2011 168 YCT


(c) Seven, viz., Adityapur, Darbhanga, Ranchi Ans. (a) : C. Rajagopalachari became the first and only
and Bokaro, Patna, Muzaffarpur, Bhojpur Indian Governor General of Independent India.
(d) Four, viz., Adityapur Bhojpur and 55. When was the National Council of Education
Muzaffarpur, Khagaria established?
Ans. (a) : Before the separation of Bihar there were 6 (a) 15th August, 1903 (b) 15th August, 1904
industrial development authorities they were - (c) 15th August, 1905 (d) 15th August, 1906
Adityapur, Bokaro, Patna, Ranchi, Darbhanga and
Ans. (d) : National Council of Education was
Muzaffarpur.
established on 15th August, 1906 with the objectives to
50. In Bihar, name the scheme which aims at promote science and technology as part of Swadeshi
building up infrastructural facilities of high
industrialization movement.
standard, required for establishment of 'Export
oriented Units' 56. The Non-Cooperation Movement was started
(a) Integrated Infrastructural Development (IID) in
(b) Export Promotion Industrial Park (EPIP) (a) 1918 (b) 1920
(c) Concept of Build-Operate-Transfer (BOT) (c) 1921 (d) 1922
(d) Software / Hardware Technology Park Ans. (b) : Non Co-operation movement was started in
(STP/HTP) 1920 by Indian National congress under the leadership
Ans. (b) : Export Promotion Industrial Park (EPIP) is a of Mahatma Gandhi. It's goal was to resist British
multiproduct export processing zone in Bihar which authority in non - violent manner.
aims at building up high standard export oriented 57. The Montagu-Chelmsford Report formed the
infrastructural facilities. basis of
51. "Swaraj is my birthright and I shall have it." (a) the Indian Independence Act, 1947
Who said? (b) the Indian councils Act, 1909
(a) M.K. Gandhi (c) the Government of India Act, 1919
(b) Jawaharlal Nehru (d) the Government of India Act, 1935
(c) Bal Gangadhar Tilak Ans. (c) : The Montagu-Chelmsford Report formed the
(d) Bhagat Singh basis of the Government of India Act, 1919. The
Ans. (c) : "Swaraj is my birthright and I shall have it". Government of India Act 1919 also known as Montagu
this is a famous slogan given by Bal Gangadhar Tilak. - Chelmsford reform.
52. The Madras Mahajan Sabha was established in 58. Who is popularly known as 'Sher-e-Punjab'?
the year (a) Rajguru (b) Bhagat Singh
(a) 1880 (b) 1881 (c) Lala Lajpat Rai (d) Udham Singh
(c) 1882 (d) 1883 Ans. (c) : Lala Lajpat Rai was an Indian freedom
Ans. (*) : Madras Mahajan Sabha was an early fighter. He played on important role in freedom
nationalist organization based in the Madras presidency. struggle. He was popularly known as "Punjab Kesari"
Madras Mahajan Sabha was established by S. and 'Sher-e-Punjab'. Bhagat Singh Popularly known as
Ramaswami Mudaliar and P. Anandacharlu in 1884. Its Shaheed-e-Azami
first conference was held on December 29, 1884. This
59. Gandhi-Irwin Pact took place in
question was deleted by BPSC
(a) 1930 (b) 1931
53. How many representatives took part in the
(c) 1932 (d) 1933
First Session of the Indian National Congress?
(a) 52 (b) 62 Ans. (b) : Gandhi - Irwin Pact took place in the year of
1931. By this pact Government agreed to release most
(c) 72 (d) 82
of civil disobedience volunteers, against whom there
Ans. (c) : Indian National Congress was established in was no allegation of violence and Congress agreed to
year 1885 and conducted it's first session with 72 participate in Second Round Table Conference.
representative at Bombay under the Presidenship of 60. Which Round Table Conference held in 1932?
W.C. Bannerjee. (a) First (b) Second
54. The first Indian Governor General of (c) Third (d) Fourth
Independent India was Ans. (c) : The Third Round Table Conference was held
(a) Raja Gopalachari (b) Surinder Nath in 1932. Congress did not participate in this conference
(c) Rajendra Prasad (d) B.R.Ambedkar because of failure of Second Round Table Conference.
53th-55th BPSC (Pre) Exam. 2011 169 YCT
61. Who is associated with 'Abhinav Bharat'? 67. Raj Kumar Shukla was resident of the village
(a) V.D. Savarkar (b) C.R. Das (a) Murli Bharhawa (b) Murli Bheet
(c) B. G. Tilak (d) S.C. Bose (c) Murli Dheer (d) Murli Kher
Ans. (a) : Abhinav Bharat society was founded by Ans. (a) : Raj Kumar Shukla was a freedom fighter who
Vinayak Damodar Savarkar and his brother Ganesh convinced Mahatma Gandhi to visit champaran to see
Damodar Savarkar in 1904. It was a revolutionary
miseries of Indigo Cultivators. He was born at Murli
organisation and purpose of foundation of this
Bharawa, a village in Champaran district of Bihar.
organisation was to overthrow of British rule through
armed rebellion. 68. Shri Nivaranchandra Das Gupta belonged to
62. 'Do or Die' is associated with the following (a) Patna (b) Purulia
movement (c) Hazipur (d) Meenapur
(a) Dandi (b) Non-Cooperation Ans. (b) : Shri Nivaranchandra Das Gupta was
(c) Khilafat (d) Quit India honorary District Magistrate of Purulia. He left his Job
Ans. (d) : Quit India movement also known as August in 1921 to participate in the Non - Co-operation
movement started in 1942 under the leadership of movement launched by Mahatma Gandhi for which he
Mahatma Gandhi at Gowalia Tank Maidan in Mumbai. served an imprisonment sentence.
Here Mahatma Gandhi gave a Do or Die' Call to the
people of India in a final push to make the British Quit 69. Kunwar Singh was the King of
India but very soon all legend leaders were arrested and (a) Hamirpur (b) Dheerpur
jailed. This movement became a leaderless movement. (c) Jagdishpur (d) Rampur
63. With whose efforts was Second Bihar Ans. (c) : Kunwar singh was a prominent freedom
Provincial Congress Session held? fighter. He was the King of Jagdishpur.
(a) G.K. Gokhale 70. Lord Sinha had resigned in 1921 from the post
(b) Shri Deepnarayan Singh of Governor of
(c) Shri Krishna (a) Bihar-Orissa (b) Bengal
(d) Surendranath Banerjee (c) Punjab (d) Madras
Ans. (b) : With the efforts of Deepnarayan Singh the Ans. (a) : Lord Sinha had resigned in 1921 from the
second Bihar provincial congress session was held at post of Governor of Bihar-Orissa.
Bhagalpur, Bihar in 1909 under presidentship of
71. The value of x − [y − {z − (x − y − z)}] is
Sachidanand Sinha.
(a) x + y + z (b) x – y – z
64. At which place was the 27th Session of the
Indian National Congress held? (c) 1 (d) 0
(a) Bhagalpur (b) Patna Ans. (d) : x − [y − {z − (x − y − z)}]
(c) Ranchi (d) Bankipur
= x − [ y − {− x + y}]
Ans. (d) : The 27th session of Congress was held at
Bankipur in 1912 under the presidentship of Shri = x − [ y + x − y]
Raghunath Narasinha Mudholkar.
65. The 16th Session of Bihari Students' =x–x=0
Conference at Hazaribag took place in Therefore option (d) is the correct Answer.
(a) 1921 (b) 1922 72. If the function f : I+→R, f(x)= log x then the
(c) 1923 (d) 1924 value of f (x)+f (y) is
Ans. (a) : The 16th session of Bihari Students (a) f (x y) (b) f (x + y)
Conference held at Hazaribagh in the year of 1921. (c) f (x/y) (d) f (y/x)
66. The editor of 'Bihar' a leading newspaper of Ans. (a) : f(x) = logx
Bihar, was f(y) = logy
(a) Babu G. Prasad f(x) + f(y) = logx + logy = logxy
(b) Babu Ram Prasad f(x) + f(y) = f(xy)
(c) Babu Gopal Prasad Therefore option (a) is the correct answer.
(d) Babu Maheshwar Prasad
73. The median of 2, 4, 6, 8, 10, 12, 14, 16 is
Ans. (d) : 'Bihar' was a leading newspaper of Bihar.
Babu Maheshwar Prasad was the editor of this (a) 8 (b) 9
newspaper. (c) 10 (d) 11

53th-55th BPSC (Pre) Exam. 2011 170 YCT


Ans. (b): Here n = 8 (even) 77. The coordinates of the mid-point on joining the
The median for n (even) number is - points P (4, 6) and (-4, 8) will be
(a) (2, 7) (b) (7, 2)
 n th  n th  
 term +  + 1 term  (c) (7, 0) (d) (0, 7)
2  2 
=  Ans. (d): The coordinates of the mid-point on joining
 2  the points (x1, y1) and (x2, y2)
 
   x1 + x 2 y1 + y 2 
 2 , 2 
4th term + 5th term  
= coordinates of the mid-point
2
 4−4 6+8
=
8 + 10 18
= =9  ,  = ( 0, 7 )
2 2  2 2 
Therefore option (d) is the correct Answer.
5 2
74. The inverse of the matrix   is 78. The solution of the differential equation
7 3 d2 y dy
 3 2 3 2 − 3 + 2y = e5x , is
(a)   (b)   dx 2 dx
 −7 5  7 5  b
(a)
 3 −2   −3 2  a
(c)   (d)  
 −7 5   7 −5 1
(b) y = C1 ⋅ e − x + C2 ⋅ e2x + e5x
12
5 2
Ans. (c) : A =   1
7 3 (c) y = C1 ⋅ e x + C2 ⋅ e −2x + e5x
12
 3 −2  1
AdjA =   (d) y = C1 ⋅ e x + C2 ⋅ e 2x + e5x
 −7 5  12
5 2 d2 y dy
A = = 15 − 14 = 1 Ans. (d) : − 3 + 2y = e5x
7 3 dx 2
dx
Adj ( A )  3 −2 
A −1 = = 
(D 2
)
− 3D + 2 y = e5x
A  −7 5 
( D − 1)( D − 2 ) y = e5x
Therefore option (c) is the correct Answer.
C.F. is y = c1ex + c2e2x
75. The length of the minute hand of a clock is 12
cm. Then, the area swept by the minute hand in 1
P! = e5x
one minute will be the area swept by the ( D − 1)( D − 2 )
minute hand in one minute will be
(a) 22.12cm2 (b) 23.10cm2  1 1  5x
= − e
(c) 24.12cm 2
(d) None of these  D − 2 D − 1 
Ans. (d) : r = 12 cm, The angle formed by minute 1 5x 1 5x
= e − e
needle in 1 minute = 360º/60º = 6º D−2 D −1

area =
πr 2 θ 22 ×122 × 6
= = 7.54 cm 2 ∫ ∫
= e 2x e5x e−2x dx − e x e5x e − x dx
360 7 × 360
e3x e 4x e5x
Therefore option (d) is correct Answer. = e 2x − ex =
76. A player completes 2200 m race in any circular 3 4 12
path with 7 revolutions. Then the radius of the y = C.F. + P!
 22  1 5x
circular path is  π =  y = C1ex + C2e2x + e
 7  12
Therefore option (c) is the correct Answer.
(a) 30m (b) 40m
(c) 50m (d) 60m 79. In triangle PQR if ∠ P = 120° and PQ = PR, the
∠ Q and ∠ R will be respectively
2200 (a) 60°, 30° (b) 30°, 40°
Ans. (c) : Circumference = = 2πr
7 (c) 30°, 30° (d) 20°, 40°
2200 × 7 Ans. (c) : PQ = PR
r= = 50m
7 × 2 × 22 ∠P + ∠Q + ∠R = 180º (Here PQ = PR)
Therefore option (c) is the correct Answer. 120º + θ + θ =180º (therefore ∠Q = ∠R = θ )

53th-55th BPSC (Pre) Exam. 2011 171 YCT


θ = R = 30º (∵ Angle opposite to the equal (a) Stems (b) Branch
side of a △ are equal) (c) Leaves (d) Roots
Therefore option (c) is the correct Answer. Ans. (c) : Leaves are component of plants, which
receive stimulus for flowering. Apart from light and
a x − bx
80. The value of is lim is chemical stimulus for flowering, temperature is also
x →0 x important stimulus.
a 86. Minerals are
(a) 0 (b) loge
b (a) Liquids
b (b) Inorganic solids
(c) loge (d) a – b
a (c) Gases
(d) All of the above
a x − bx
Ans. (b) : lim Ans. (b) : A mineral is a naturally occurring inorganic
x →0 x solids with a definite chemical composition and an
 a x − 1 bx − 1  ordered atomic arrangement. As liquid and gases have
lim  −  not crystalline structure they are not considered as
x →0
 x x 
minerals. Hence option (b) is the correct answer.
loge(a) – loge(b) 87. Development of the natural systems is
a described as
log e
b (a) Action of the systems
Therefore option (b) is the correct Answer. (b) evolution of the systems
81. The visible range of solar radiation is (c) Self-sustained process of the systems
(a) 100-400nm (b) 400-700nm (d) None of the above
(c) 740-10000nm (d) None of the above Ans. (c) : Self - Sustained process of the systems is
known as development of the natural system. In Self
Ans. (b) : The visible range of Solar radiation is 380 nm
sustained process organism nourishes itself rather than
to 720 nm Thus option (b) is the correct answer.
relying on another organism.
82. Plants which grow on saline soils are
88. Genetics deals with
(a) xerophytes (b) hydrophytes
(c) halophytes (d) succulents (a) Mendel's laws
(b) organic evolution
Ans. (c) : Halophytes are salt tolerant or salt resistant
plants. These plants grow in highly saline water. (c) DNA structure
(d) heredity and variations
83. Environment is a composite state of
Ans. (d) : Genetics is a branch of biology in which we
(a) biotic factors
study about genes, genetic variation and heredity in
(b) Physiographic factors organism.
(c) abiotic factors
89. Mendel's principles of inheritance are based on
(d) All of the above (a) vegetative reproduction
Ans. (d) : Environment is the natural component in (b) asexual reproduction
which biotic (living) and abiotic (non living) factors (c) sexual reproduction
interact among themselves and with each other. These (d) All of the above
interactions shape the habitat and ecosystem of an Ans. (c) : Mendel's Principles of Inheritance are based
organism. on sexual reproduction. G Mendel, through his work on
Hence, environment is a composite state of all biotic pea plants, discovered the fundamental law of
factors, physiographic factors and abiotic factors. inheritance. He deduced that genes come in pairs and
are inherited as distinct units, one from each parent.
84. Water is conducted in vascular plants by
90. Embryo is found in
(a) Phloem tissue (b) parenchyma tissue
(a) Flowers (b) Leaves
(c) meristems (d) xylem tissue (c) Seeds (d) Buds
Ans. (d) : The two primary vascular tissues are xylem Ans. (c) : The plant embryo, sometimes called the seed
which transports water and dissolved minerals from embryo is the part of seed or bud that contains the
roots to the leaves and phloem which transports food earliest forms of a plants roots, stem and leaves.
from the leaves to all parts of the plant. 91. What soil particles are present in loamy soils?
85. Which component of plants receives Stimulus (a) Sand particles
for flowering? (b) Clay particles
53th-55th BPSC (Pre) Exam. 2011 172 YCT
(c) Silt particles 98. Table salt (NaCI) is a product of
(d) All types of particles (a) Weak acid and weak base
Ans. (d) : In a loamy soils all types of particles like (b) Strong acid and strong base
sand particles, clay particles and silt particles are (c) Weak acid and strong base
present. (d) Strong acid and weak bases
92. A single type of atom is found in Ans. (b) : The table salt (NaCl) is the product of strong
acid, Namely Hydrochloric acid (HCL) and strong base,
(a) Compounds of minerals
namely Sodium Hydroxide.
(b) Mixture of minerals
99. Sexual reproduction causes genetic variation
(c) Native elements because of
(d) None of the above (a) blending of genes
Ans. (c) : Native element are those elements in nature (b) chromosomal changes
that are uncombined with other elements. These are (c) Shuffling genes
minerals that are composed of atoms from a single (d) All of the above
element. Ans. (c) : Gene shuffling refers to the creation of
93. Soil water available to plants is maximum in different combinations of alleles during meiosis and due
(a) Clayey soil (b) Silty soil to shuffling of genes the sexual reproduction causes
(c) Sandy soil (d) Loamy soil genetic variation.
Ans. (a) : Water holding capacity is highest in clayey 100. Average salinity of water of Arabian Sea is
soil due to the large surface area as well as tightly (a) 25 ppt (b) 35 ppt
packed particles that restrict the water from percolating. (c) 45 ppt (d) 55 ppt
BPSC had accepted (d) as right answer. Ans. (b) : Salinity is amount of salt dissolved in body
of water. It is measured in number of grams of salts per
94. Capillaries are most effective in
kilogram of seawater. It is expressed as parts per
(a) clayey soil (b) silty soil Thousand (PPT). The average salinity of water of
(c) sandy soil (d) loamy soil Arabian sea is 35ppt.
Ans. (a) : Capillarity is the phenomenon by which 101. Who among the following is the winner of the
water rises in a cylindrical column. The narrower the Mahatma Gandhi International Award for
column the higher the capillarity; similarly the denser Peace and Reconciliation, 2009?
the substratum present in the column the higher the (a) Yukia Amano (b) Aung San Suu Kyi
capillary effect. (c) Bill Gates (d) Hillary Clinton
Thus capillarity effect is most effective in clayey soil Ans. (b) : Mahatma Gandhi International Award for
followed by loamy soil. BPSC had accepted (d) as right Peace and Reconciliation 2001 was awarded to
answer. Aung San Suu Kyi.
95. Photosynthesis occurs in The award is provided by South Africa based Mahatma
(a) Nucleus (b) Mitochondria Gandhi Foundation.
(c) Chloroplast (d) Peroxisome 102. 'Durand Cup' is associated with the game of
Ans. (c) : Photosynthesis is a process by which green (a) Football (b) Polo
plants use sunlight, water and carbon dioxide to create (c) Cricket (d) Hockey
oxygen and energy in the form of sugar. The process of Ans. (a) : 'Durand Cup' is associated with Football. The
photosynthesis occurs in chloroplast. Durand cup was started in 1888.
96. Continents have drifted apart because of 103. For which word, the letter 'G' has been used in
(a) Volcanic eruptions '2G Spectrum'?
(b) Tectonic activities (a) Global (b) Government
(c) Folding and faulting of rocks (c) Generation (d) Google
(d) All of the above Ans. (c) : Based on the speed of the internet, different
Ans. (b) : Due to tectonic activities, continent have generation internet service has been started, in which 'G'
drifted apart. is the generation.
104. Former Central Minister for Communication
97. Which of the following do not belong to solar
and Information Technology, Mr. A. Raja
system?
belongs to which of the following parties?
(a) Asteroids (b) Comets
(a) Congress
(c) Planets (d) Nebulae (b) BJP
Ans. (d) : Asteroids, Comets and Planets belong to the (c) Communist Party of India
solar system while Nebulae exist in interstellar space. (d) DMK
53th-55th BPSC (Pre) Exam. 2011 173 YCT
Ans. (d) : Mr. A Raja is an Indian politician. He was Ans. (c) : Party-wise performance in 2010 Bihar
member of 15th Lok Sabha. He belongs to Dravida assembly elections compared to 2005 elections is as
Munnetra Kazhagam (DMK) Party. follows:-
105. Which one of the following numbered Rajdhani JDU BJP RJD LJP CONG OTHERS
trains covers the longest distance? +27 +36 -32 -7 -5 -19
(a) 12429 Bangalore City Junction 110. Till 2010, which State Governments have
(b) 12431 Trivandrum Central provided 50 percent reservation for women in
(c) 12433 Chennai Central local bodies?
(d) 12435 Dibrugarh Town (a) Bihar, Uttar Pradesh, Rajasthan
(b) Bihar, Madhya Pradesh, Himachal Pradesh
Ans. (b) : 12431 Trivendrum Central Rajdhani Train
covers the longest distance, which covers about 2845 (c) Bihar, Madhya Pradesh, Kerala
km. (d) Bihar, Himachal Pradesh, Uttar Pradesh
Ans. (b) : Till 2010, the state governments of Bihar,
106. What is the minimum age of a person whose
Madhya Pradesh and Himachal Pradesh have provided
photographing, fingerprinting and iris
50 percent reservation for women in local bodies.
mapping will be done for the Census of 2011?
111. 'AVAHAN', the Indian segment of 'Bill and
(a) 12 years (b) 15 years
Melinda Gates Foundation', is engaged in the
(c) 18 years (d) 21 years
prevention of
Ans. (b) : In the 2011 census, the minimum age of a (a) dengue (b) polio
person for photographing, finger printing and eye
(c) HIV/AIDS (d) filariasis
mapping was 15 years.
Ans. (c) : 'AVAHAN' is an initiative in India by 'Bill
107. The Union Cabinet, on August 2010, decided to and Melinda Gates Foundation for "HIV/AIDS' disease
form institutes like the All India Institute of prevention.
Medical Sciences (AIIMS) in which six States?
112. A recently held underwater meeting has drawn
(a) Bihar, Chhattisgarh, Madhya Pradesh, Orissa,
attention world over. The meeting was held by
Rajasthan, Uttarakhand
(a) Some Somali pirates to make a new strategy
(b) Bihar, Uttar Pradesh, Himachal Pradesh,
for capturing ships for ransom
Haryana, Kerala, Gujarat
(b) Some environmental scientists for promoting
(c) Bihar, Uttarakhand, Jharkhand, Orissa,
awareness for saving water
Andhra Pradesh, Maharashtra
(c) Some activists environmental awareness for
(d) Bihar, Tamil Nadu, Karnataka, Uttar Pradesh, promoting against air pollution
Jammu and Kashmir, Assam (d) The cabinet of a country to highlight the
Ans. (a) : In August 2010, the Union Cabinet had threat of rising sea level for any country
decided to establish six All India Institute of Medical Ans. (d) : To show the problem of increase in the water
Sciences (AIIMS) in following states - Bihar, level of the sea and to attract the attention of the world
Chhattisgarh, Madhya Pradesh, Odisha, Rajasthan and around this problem, the meeting was held in the water
Uttarakhand. by the Cabinet of the country Maldives.
108. Which was reported the richest party 113. Which one of the following cities is not located
according to the 'National Election Watch' on on the bank of river Ganga?
August 2010? (a) Fatehpur (b) Bhagalpur
(a) BSP (b) BJP (c) Uttarkashi (d) Kanpur
(c) Socialist Party (d) Congress Ans. (a) : Fatehpur is located at bank of the Sacred river
Ans. (d) : In August 2010, the Congress Party was Ganga and Yamuna. Bhagalpur, Kanpur, Varanasi etc
declared the richest party by the 'National Election are located at the bank of rivers Ganga. BPSC had
Watch'. accepted (a) as right answer.
109. Which party-wise performance incorrect for 114. Which one of the following human right is a
Bihar Assembly Elections, 2010 in comparison fundamental right under the Constitution of
to the elections of 2005? India?
JDU BJP RJD WP CONG OTHERS (a) Right to Information (b) Right to Work
(a) + 40 + 25 - 30 - 10 -7 - 19 (c) Right to Education (d) Right to Housing
(b) + 25 + 34 - 20 - 5 -3 - 11 Ans. (c) : Right to Education is a human right which is
(c) + 27 + 36 - 32 - 7 - 5 - 19 also a fundamental right as per Article 21 A of the
(d) + 17 + 46 - 15 - 10 - 10 - 10 Constitution of India.

53th-55th BPSC (Pre) Exam. 2011 174 YCT


115. The time at Cairo is 2 hours ahead of Ans. (b) : French Parliament banned 'Burqa' by making
Greenwich. Hence, it is located at law on September 14, 2010.
(a) 300W longitude (b) 300E longitude 122. Mr. Arjun Munda was on September 11, 2010
(c) 280E longitude (d) 300W longitude sworn in as the Chief Minister of Jharkhand.
Ans. (b) : Greenwich is located at 0º longitude, moving How many MLAs in the 81-member Assembly
towards east, every 15º longitude time increases by an supported him?
hour. In the case of 2 hours, ahead of Greenwich it will (a) 41 (b) 50
be at 30º East longitude. Therefore option (b) is the (c) 45 (d) 55
correct answer. Ans. (c) : In Jharkhand, Arjun Munda took oath as
116. The headquarters of International Court of Chief Minister on September 11, 2010. He got support
of 45 legislators of the 81 - member legislative
Justice is at
Assembly.
(a) Hague (b) New York
123. Mr. P.J. Thomas has been appointed as the
(c) Geneva (d) Paris
central Vigilance Commissioner on September
Ans. (a) : The headquarters of the International Court of 7, 2010. He is an IAS Officer of the following
Justice located at 'The Hague' city of Netherlands. State's cadre
117. In the world's best countries list, complied by (a) Assam (b) Karnataka
the reputed 'News Week' magazine in 2010, the (c) Kerala (d) Tamil Nadu
first position went to Finland. What was the Ans. (c) : Mr. P.J. Thomas was appointed as the Central
rank of India? Vigilance Commissioner in September 2010. He was an
(a) 17 (b) 48 IAS officer of Kerala cadre.
(c) 66 (d) 78 124. What is the maximum amount upto which
Ans. (d) : In the survey conducted by 'News Week' in financial autonomy has been provided to
2010 India was ranked 78th in the list of the best public Sector under takings under Mini Ratna
countries in the world. Category I by Ministry of Heavy Industries
and Public Enterprises
118. Which university was ranked first in the list of (a) ` 250 crore (b) ` 350 crore
'2010 Academic Rankings of the World
(c) ` 400 crore (d) ` 500 crore
Universities'?
Ans. (d) : Public Sector Undertakings (PSU) Under
(a) Jawaharlal Nehru University, India
Mini Ratna Category I are allowed to invest Rs 500
(b) Cambridge University, Britain Crore without government approval. It is the maximum
(c) Harvard University, USA limit upto which financial autonomy has been
(d) Massachusetts Institute of Technology, USA provided to Mini Ratna Category I PSU.
Ans. (c) : Harvard University, USA was ranked first in 125. When was the Right to Education added
the list of 2010 academic Rankings of the world through the amendment in the Constitution of
Universities. India.
119. Which date is known as 'Diabetes Day'? (a) 1st April, 2010
(a) 14th February (b) 14th May (b) 1st August, 2010
(c) 14th September (d) 14th November (c) 1st October, 2010
(d) 1st December, 2010
Ans. (d) : The world 'Diabetes Day' is celebrated on
14th November under the aegis of the World Health Ans. (a) : The Constitutional 86th Amendment Act,
Organization. 2002 inserted Article 21 A in the Indian Constitution to
provide free and compulsory education of all children in
120. In which country, Ms. Julia Gillard was made the age group of 6 to 14 years as a fundamental right.
the first woman Prime Minister in the year
The Right of Children to Free and Compulsory
2010? Education Act, 2009 represents the consequential
(a) Australia (b) Canada legislation envisaged under Article 21 A. Article 21 A
(c) Germany (d) Poland and R.T.E Act came into effect on April 1, 2010.
Ans. (a) : Julia Gillard became Australia's first woman 126. Match the achievements of India in the XVI
Prime Minister in 2010. Asain Games, 2010:
121. Which country's parliament has banned 1. Rank P. 14
'Burqa' by passing a law on 14th September, 2. Gold Q. 33
2010?
3. Silver R. 06
(a) USA (b) France
4. Bronze S. 17
(c) Italy (d) Holland
53th-55th BPSC (Pre) Exam. 2011 175 YCT
(a) 1 2 3 4 131. Which of the following is a Harappan port?
Q R P S (a) Alexandria (b) Lothal
(b) 1 2 3 4 (c) Mahasthangarh (d) Nagapattanam
R P S Q Ans. (b) : The site of Lothal is the only port town of the
(c) 1 2 3 4 Indus valley civilization. The city is a metropolis with
P Q R S an upper and a lower town. There is a basin on its
(d) 1 2 3 4 northern side with vertical wall, inlet and outlet
S R P Q channels. The basin has been identified as a tidal
Ans. (*) : 16th Asian Games were held in Guangzhou dockyard.
(China). the XVI Asian Games, India won 14 gold, 17 132. The Gupta emperor who defeated the 'Hunas'
silver and 34 bronze medals. India is ranked in medal was
tally was 6th. (a) Samudragupta (b) Chandragupta II
127. To which institute, Shubha Tole, the first (c) Skandagupta (d) Ramgupta
woman awardee of prestigious Shanti Swarup Ans. (c) : The Huna invasion took place at a time when
Bhatnagar Prize, 2010 in the field of Science, Gupta dynasty had been ruling over a greater part of
belongs? India. Gupta emperor Skandgupta fought the battle
(a) Tata Institute of Fundamental Research, against the invading Hunas and defeated them.
Mumbai 133. What is the name of Kalhana's book?
(b) Indian Institute of Science Education and (a) Arthashastra (b) Indica
Research, Pune (c) Purana (d) Rajtarangini
(c) Indian Institute of Technology, Kanpur Ans. (d) : The Rajtarangini is an epic poem in Sanskrit
(d) Indian Institute of Science, Bengaluru written by Kalhana in 12th century. The Rajatrangini is
Ans. (a) : Shubha Tole, who received the prestigious an account of the many royal dynasties that ruled the
Shanti Swarup Bhatnagar award in 2010, is associated ancient kingdom of Kashmir from its origin till poet's
with Tata Institute of Fundamental Research, Mumbai. own time.
128. Which Indian was honoured by the 'World 134. Which inscription tells about the various
Statesman Award, 2010?' achievements of Rudradaman?
(a) Mrs. Sonia Gandhi (a) Junagarh (b) Bhitari
(b) Atal Bihari Vajpayee (c) Nasik (d) Sanchi
(c) Dr. Manmohan Singh Ans. (a) : Junagarh's inscription tells about the various
(d) Lal Krishna Advani achievements of Rudradaman e.g. Restoration work of
Ans. (c) : Dr. Manmohan Singh was awarded the World Sudarshana lake, war with Satvahana dynasty, marriage
Statesman Award 2010. of daughter of Rudradaman etc.
129. India's Third Research Centre at Antarctica is 135. Who was the first ruler of the Slave dynasty?
name as (a) Qutubuddin Aibak (b) Iltutmish
(a) Bharti (b) Swagatam (c) Razia (d) Balban
(c) Hindustan (d) Maitri
Ans. (a) : Slave dynasty also known as Mumluk
Ans. (a) : India 's third research station at Antarctica is dynasty was established by Qutubuddin Aibak in 1206.
Bharti. The first and second research station at He was the slave of Mohammad Ghori. Hence the
Antarctica are Dakshin Gangotri and Maitri dynasty established by Qutubuddin Aibak is called
respectively.
slave dynasty.
130. In Commonwealth Games, 2010, India won 101
136. Who was the ruler of Devgiri at the time of
medals. Out of them, how many medals have
Alauddin Khalji's invasion?
been received by team events, women and men
respectively? (a) Prataprudra Dev (b) Ramchandra Dev
(a) 0, 37, 64 (b) 2, 35, 64 (c) Mali Kafoor (d) Rana Ratan Singh
(c) 1, 37, 63 (d) 1, 36, 64 Ans. (b) : When Alauddin Khalji's commander in-chief
Malik Kafur invaded Devgiri, the ruler of Devgiri was
Ans. (b) : In 2010 Commonwealth Games India had
Ramchandra Dev.
ranked second place with a total of 101 medals,
including 38 Gold, 27 Silver and 36 Bronze medals. Out 137. Who of the following Muslim rulers abolished
of the total 101 medals, 2 medals had been received by the pilgrimage tax?
team events, 35 medals had been won by women and 64 (a) Bahalol Lodi (b) Sher Shah
medals had been won by men. (c) Humayun (d) Akbar

53th-55th BPSC (Pre) Exam. 2011 176 YCT


Ans. (d) : In the given options Mughal ruler Akbar 143. Who founded the 'Theosophical Society'?
abolished the pilgrimage tax in 1563. (a) Madame H.P. Blavatsky
(b) Raja Ram Mohan Roy
138. What was 'Dar-ul-Shafa' established by Firoz
(c) Mahatma Gandhi
Tughlaq?
(d) Swami Vivekananda
(a) An alms house
Ans. (a): Theosophical Society was founded by
(b) A free hospital Madame H.P. Blavatsky and Colonel Olcott in 1875 in
(c) A library New York. In India the Society's headquarters was
(d) A guesthouse for pilgrims established at Adyar near Madras.
Ans. (b) : Dar-ul-Shafa was a free hospital established 144. Which one of the following books was written
by Firoj Shah Tughalq for the poor. He also created by Tilak?
department of charity to take care of orphans and (a) India in Transition
widows. (b) Gita Rahasya
(c) Gokhale-My political Guru
139. The 'Permanent Settlement' was made with (d) Discovery of India
(a) Zamindars
Ans. (b) : The book 'Gita Rahasya' was composed by
(b) Village communities Lokmanya Bal Gangadhar Tilak in the Mandalay Jail
(c) Muqaddamas (Rangoon).
(d) Peasants 145. Where was Mahavira Swami born?
Ans. (a) : A Permanent tax settlement (Zamidari (a) Kundagram (b) Pataliputra
System) was done with the landlords (Zamindars) by (c) Magadh (d) Vaishali
the British, under which fixed tax was imposed on Ans. (a) : Mahavira Swami was the 24th and last
landlords. It was started by Lord Cornowallis In Tirthankara of Jainism. He was born in 540 BC to the
Mahalwari system land revenue settlement was made king Siddharth and Queen Trishala at Kundagram near
Vaishali.
with Village Communities. Under Ryotwari System
land revenue settlement was made with Peasants. 146. Where did Mahatma Buddha's
'Mahaparinivana' take place?
140. Who was the founder of 'Prarthana Samaj'? (a) Lumbini (b) Bodh Gaya
(a) Atmaram Panduranga (b) Tilak (c) Kushinara (d) Kapilavastu
(c) Annie Besant (d) Rasbihari Ghosh Ans. (c) : In Buddhism 'Mahaparinirvana' implies death.
Ans. (a) : The Prarthana Samaj was established by Mahaparinirvana of Mahatma Buddha took place in
'Atmaram Panduranga in 1867 at Bombay with the help Kushinara in Uttar Pradesh.
of Keshav Chandra Sen. 147. At which place did Mahatma Buddha give his
141. Who was the father of Indian Renaissance? first 'Dharmachakrapravartan'?
(a) Bal Gangadhar Tilak (a) Lumbini (b) Sarnath
(c) Pataliputra (d) Vaishali
(b) Dayananda Saraswati
(c) Shraddhananda Ans. (b) : Mahatma Buddha give his first sermon at
'Sarnath'. The event is known as
(d) Raja Ram Mohan Roy
'Dharmachakrapravartan'.
Ans. (d) : Raja Ram Mohan Roy is considered as the
148. What was the early capital of Magadh?
father of the Indian Renaissance due to the remarkable
(a) Pataliputra (b) Vaishali
changes he instituted in the 18th and 19th century India.
He is known for his efforts to abolish the practices of (c) Rajagriha (Girivraja) (d) Champa
sati and child marriage. Ans. (c) : Rajagriha (Girivraja) was the early capital of
142. 'Swaraj Dal' was founded by Magadha.
(a) Tilak and Chittaranjan Das 149. What was the name of the dynasty of
(b) Gandhi and Motilal Nehru Ajatshatru?
(a) Maurya (b) Haryanka
(c) Gandhi and Tilak
(c) Nanda (d) Gupta
(d) Chittaranjan Das and Motilal Nehru
Ans. (b) : Ajatshatru was the ruler of the Haryanka
Ans. (d) : Swaraj Dal or the Congress Khilafat Swaraj Dynasty. He was the son of Bimbisara.
Party was established by C.R. Das and Motilal Nehru
on 1 January 1923. In the Gaya Session of Congress in 150. The Third Buddhist council was convented at
1922, C.R. Das moved a proposal to enter the (a) Takshashila (b) Sarnath
legislature but it was defeated. C.R. Das and other (c) Bodh Gaya (d) Pataliputra
leaders broke away from the Congress and formed Ans. (d) : In the reign of Ashoka, the third Buddhist
Swaraj Party. meeting was held in 255 BC at 'Pataliputra'.
53th-55th BPSC (Pre) Exam. 2011 177 YCT
56th–59th Bihar Public Service Commission
Preliminary Examination, 2015
GENERAL KNOWLEDGE & GENERAL SCIENCE
(Solved Paper with Detail Explanation)
1. The Union Health Minister launched Mental Other Exercises between India and the USA are-
Health Policy on Exercise Tiger Triumph (Humanitarian Assistance and
(a) 20th October, 2014 (b) 10th October, 2014 Disaster Relief exercise)
(c) 5th October, 2014 (d) 11th July, 2014 Exercise Vajra Prahar (Special Forces’ Exercise)
Ans. (b) : On 10th October, 2014 the Health Minister Cope India (Air Force).
Dr. Harsh Vardhan launched India’s first-ever National Malabar Exercise (Quadrilateral naval exercise of India,
Mental Health Policy to provide universal psychiatric USA, Japan and Australia).
care, e.g. to prevent mental illness, promote mental Red Flag (US’ multilateral air exercise).
health etc.
5. Which of the following groups of political could
2. Prime Minister Narendra Modi launched not win a single seat in the 16th Lok Sabha
Saansad Adarsh Gram Yojana on Election?
(a) 11th October, 2014
(a) MNS, HSP, SP, RJD
(b) 10th November, 2014
(b) SP, BSP, National Conference, RLD
(c) 26th August, 2014
(c) JDU, BSP, NCP, RJD
(d) 15th August, 2014
(d) DMK, RLD, National conference, BSP
Ans. (a) : Saansad Adarsh Gram Yojana is a rural
Ans. (d) : In the given options DMK, RLD, National
development programme broadly focusing on
Conference and BSP could not manage to win even a
development in village eg. social development cultural
development, economic development etc and also single seat in 16th Lok-Sabha election.
spread motivation among the people on social 6. President Pranab Mukherjee on 7th October,
mobilization of the village community. This programme 2014 presented the Lal Bahadur Shashtri
was launched by Prime Minister Narendra Modi on 11th National Award for excellence in Public
October, 2014. Administration, Academics and Management
3. The 11th Metropolis Congress was organized in for the year 2014 to
Hyderabad in (a) V. Sampath (b) Dr. A.S. Pillai
(a) September 2013 (b) January 2014 (c) Arvind Mayaram (d) Ajit Dobhal
(c) October 2014 (d) November 2014 Ans. (b) : President Dr. Pranab Mukharjee on 7th
Ans. (c) : The 11th Metropolis Congress was organized October, 2014 presented the Lal Bahadur Shashtri
in Hyderabad from 6-10 October 2014 and 13th edition National Award for excellence in public administration
of Metropolis Congress meet which was held during academics and management to Dr. A.S. Pillai. In 2021
November 2021 was organised in Guangzhou China. this award won by Dr. Randeep Guleria.
4. In which area was the Indo-US military 7. Who has been appointed as the Chairman of
exercise 'Yudh Abhyas 2014' held? the 7th Central Pay Commission constituted by
(a) Solan (Himachal Pradesh) the Government of India in February 2014?
(b) Gairsen (Uttarakhand) (a) Justice B.K. Singh
(c) Ranikhet (Uttarakhand) (b) Justice Swatanter Kumar
(d) Poonch (Jammu and kashmir)
(c) Justice Ashok Kumar Mathur
Ans. (c) : Yudh Abhyas is a military excercise between
(d) Justice Mridula Singh
India and U.S.A. in 2014 it was held in Ranikhet.
Ans. (c) : Justice A.K. Mathur is the chairman of 7th
The exercise was started in 2004 under the US Army
Pacific Partnership Program. The exercise is hosted Central Pay Commission. It was constituted by
alternately between both countries. Government of India in February 2014.

56th-59th BPSC (Pre) Exam. 2015 178 YCT


8. Justice J.S. Verma committee's Report, 2013 14. What is the theme of Global Financial
deals with Development Report, 2014?
(a) Centre-State relations (a) Hunger and poverty
(b) Change in law pertaining to offences against (b) Corruption and fraud in corporate sector
women (c) Financial inclusion
(c) The Fundamental Rights (d) Global recession and the Third World
(d) Rights of children Ans. (c) : The theme of Global Financial Development
Report, 2014 was Financial inclusion. This report is
Ans. (b) : Justice J.S. Verma Committee Report 2013 published by World Bank. The theme of Global
deals with change in law pertaining to offences against Financial Report 2019/20 was Bank regulation and
women. The committee recommended amendments to Supervision a Decade after the Global financial crisis.
the criminal law so as to provide for quicker trial and 15. Who among the following has/have figured in
enhanced punishment for crimanls accused for 'Time' magazine's list of 25 most influential
committing sexual assult against women. teens of 2014?
9. How many members are elected from Bihar to (a) Malia and Sasha obama
the Lok Sabha? (b) Kendall and Kylie Jenner
(a) 20 (b) 30 (c) Malala Yousafzai
(c) 35 (d) 40 (d) All of them
Ans. (d) : From Bihar total 40 members are elected to Ans. (d) : In the given options Malia , Sasha obama,
Kendall , Kylie Jenner and Malala Yousafzai figured as
the Lok Sabha and 16 members are elected to the Rajya
25 most influential teen of 2014 by Time Magazines.
Sabha.
16. The Seventh BRICS Summit is proposed to be
10. What is the unique festival of Bihar? held in 2015 at?
(a) Deepawali (b) Bisu (a) Durban, South Africa (b) Ufa, Russia
(c) Vinayak Chaturthi (d) Chhath Puja (c) New Delhi, India (d) Sanya, China
Ans. (d) : Chhath Puja is a folk festival that lasts four Ans. (b) : BRICS is the association of five major
days. It starts with Kartik Shukla Chaturthi and ends emerging economics viz. Brazil, Russia, India-China
with Kartik Shukla Saptami. It is a unique festival of and South Africa. It is headquartered in Shanghai. The
th
Bihar specifically along with the state West Bengal, 7 BRICS summit was held in Ufa, Russia and 14
Uttar Pradesh, Jharkhand. BRICS summit in 2022 was held in China in virtual
mode on June 20,2022.
11. Who among the following has been awarded
17. The International Day of Persons with
the Nobel Prize for Literature in 2014?
Disabilities is celebrated on?
(a) Kailash Satyarthi (b) Malala Yousafzai
(a) 10th December (b) 20th December
(c) Jean Tirole (d) Patrick Modiano th
(c) 19 November (d) 3th December
Ans. (d) : Patrick Madiano won Nobel Prize for Ans. (d) : The International day of persons with
literature in 2015. In 2021, Abdulrazak Gurnah, a disabilities is celebrated on 3rd December of every year.
Tanzanian novelist awarded to this award. 18. Amnesty International is an organization
12. Which among the following European Union which is associated with
countries is the first to officially recognize the (a) Protection of women's rights
State of Palestine? (b) Protection of human rights
(a) Sweden (b) Denmark (c) Abolition of untouchability
(c) Norway (d) Italy (d) None of the above
Ans. (a) : Swedan became first European Union Ans. (b) : Amnesty International is a non governmental
Country which recognized the state of Palestine organization that works for the upliftment and
officially in 2014. protection of human rights. It is headquartered in the
13. Which one of the following countries has United Kingdom.
decided in January 2014 to make mandatory 19. In October 2014, India has been rechecked for
service compulsory for all its adults males? three years as a member to?
(a) North Korea (b) UAE (a) The UN Security Council
(c) Israel (d) Turkey (b) The UN General Assembly
Ans. (b) : In January 2014, UAE had decided to make (c) The UN Economic and Social Council
mandatory service compulsory for as its adults males. (ECOSOC)
(d) The World Bank
56th-59th BPSC (Pre) Exam. 2015 179 YCT
th
Ans. (c) : In October 2014, India had been rechecked 24. What was India's position in the 17 Asian
for three year as a member to the UN Economic and Games in 2014?
Social Council (ECOSOC). In 2022 India again elected (a) 8th (b) 6th
as member of UN Economic and Social Council for (c) 3rd (d) 4th
2022-2024. th
Ans. (a) : In 17 Asian Games which was held in South
20. Angola Malaysia, New Zealand, Spain and Korea India managed at 8th position with 10 Gold, 10
Venezuela were elected as Non-permanent Silver and 36 Bronze. In 2022 the Asian games is going
members on 16th October, 2014 to serve in? to held in Hangzhou, China.
(a) The UN General Assembly 25. Who among the following was the Gold Medal
(b) The UN Trusteeship Council winner in the 17th Asian games Men's
(c) The UN Human Rights Commission Freestyle 65kg Wrestling?
(d) The UN Security Council (a) Jittu Rai (b) Sandeep Kumar
Ans. (d) : In 2014, Angola, Malaysia, New Zealand, (c) Rajat Chauhan (d) Yogeshwar Dutt
Spain and Venezuela were elected as Non-permanent Ans. (d) : In 17th Asian Games the winner of Gold
members to served in the UN Security Council. medal in Men’s Freestyle 65 kg wrestling event was
21. Which one of the following countries won the Yogeshwar Dutt.
highest number of Gold Medals at the Winter 26. Which country achieved the fifth position in
Olympic Games held in Sochi in February the 20th Commonwealth Games?
2014? (a) Sri Lanka (b) Nepal
(a) Canada (b) Russia (c) England (d) India
(c) Norway (d) USA Ans. (d) : India was at 5th position in 20th
Ans. (b) : Russia topped the medal tally/table with 11 Commonwealth Games held in 2014. In 2022, the
Gold, 9 Silver and 9 Bronze medals at the Winter Commonwealth Games held in Birmingham 2022.
Olympic Games held in Sochi in February 2014. In 27. Who won her first Grand Slam Australian
2022 the Winter Olympic games was held in Beijing, Open Tennis Competition, 2014 (Women's
China. In this event Norway topped the medal table Singles)?
with 16 Gold, 8 Silver and 13 Bronze medals. India did (a) Li Na
not win any medal. (b) Dominika Cibulkova
22. Who among the following cricket all-rounders (c) Sara Errani
cracked the fastest century in one-day (d) None of them
international cricket history in January 2014? Ans. (a) : In 2014, Chinese Tennis player ‘Li Na’ won
(a) Darren Bravo (b) James Faulkner her first Grand Slam Australian Open Tennis
(c) Glenn Maxwell (d) Corey Anderson Competition in women’s singles event.
Ans. (d) : When the question was asked Corey No. Grand Courts Timing
Anderson, a New zealand’s all rounder cracked the slam
Sahid Afridi’s fatest century record which was in 37 1. Australian Hard courts (Mid January)
balls, Corey Anderson cracked this record by hitting Open
century in just 36 balls. Later South African batsmen
2. French Clay Court May- June
AB de Villiers cracked this record by hitting a century
Open
in just 31 balls in 2015 against the West-Indies.
3. Wimbledon Grass Courts June–July
23. Who was the winner of the following
Champions League T-20, 2014? 4. US Open Hard courts August-
(a) Kolkata Knight Riders (India) September
(b) Rajasthan Royals (India) Tennis Grand Slam 2022 Winners
(c) Sydeny Sixers (Australia) 1.Australian Open 2022 Winners
(d) Chennai Super Kings (India) Men’s Singles Rafael Nadal (Spain)
Ans. (d) : In 2014, the winner of T-20 Champion’s Women’s Singles Ashleigh Barty (Australia)
League was Chennai Super Kings. It was the last edition
of this league. 2. French Open Tennis 2022 Winners
Gujarat Titans won IPL 2022 against Rajasthan Men’s Singles Rafael Nadal (Spain)
Royals which was held in Ahmadabad. Women’s Singles Iga Swiatek (Poland)

56th-59th BPSC (Pre) Exam. 2015 180 YCT


28. The winner of the SAFF Tournament, 2013 Ans. (d) : Historian V. A. Smith called Samudragupta
was? as Napoleon of the ancient India. Because of his great
(a) India (b) Maldives military conquests which are known from the ‘Prayag
(c) Afghanistan (d) Nepal Prashati’ written by his courtiers and poet Harisena.
Ans. (c) : The South Asian Football Federation 34. Which Chinese traveller visited India during
Championship commonly known as SAFF Harshavardhana's rules?
Championship. In this tournament total 7 teams viz. (a) Fa-Hien (b) Hiuen-Tsang
Bangaldesh, Bhutan, India, Maldives, Nepal, Pakistan (c) I-tsing (d) Taranath
and Sri Lanka participated. In 2013, SAFF Ans. (b) : Fa-Hien – He came to India during the reign
Championship was won by Afghanistan by defeating of Chandragupta-II Vikramaditya. He was the first
Nepal. India won it’s eighth SAFF Championship in Chinese pilgrim who visited to India between (405-411)
2021. AD.
29. Which award is given for excellence in sports? Hiuen-Tsang (630-645 AD) - He visited India during
(a) Jamnalal Bajaj Award the reign of Harshvardhana.
(b) Arjuna Award I-tsing - I-tsing visited India 673 AD.
(c) Tagore Award Hence, (b) is the right answer.
(d) Moortidevi Award 35. What is 'Dhai Din Ka Zhonpda'?
Ans. (b) : The correct answer is Arjuna Award which (a) Mosque (b) Temple
was started from 1961 . Arjuna Award is given for (c) Saint's hut (d) Tower
excellence in sports given to individuals for their
Ans. (a) : 'Dhai Din Ka Zhonpda' or ‘Adhai Din Ka
exemplary performance in various sports. Zonpda’ is one of the oldest mosque located at Ajmer,
30. 'Prince of Wales Cup' is associated with the in Rajasthan built by Abu Bakr of Herat first and again
game of? by Qutb-ud-din-Aibak in 1192 CE.
(a) Hockey (b) Cricket 36. In which language was Tuzuk-i-Baburi
(c) Football (d) Golf written?
Ans. (d) : 'Prince of Wales Cup' is associated with the (a) Persain (b) Arabic
Golf. (c) Turkish (d) Urdu
31. Who among the following was not associated Ans. (c) : Tuzuk-i-Baburi is an autobiography of Babur
with the excavation of Harappa and Mohenjo- wrote it in Turkish lanaguage.
Daro?
37. Who was the commander of Rana Pratap's
(a) R.D. Banerjee (b) K.N. Dikshit army in the Battle of Haldighat?
(c) M.S. Vats (d) V.A. Smith (a) Amar Singh (b) Man Singh
Ans. (d) : In the given option R.D. Banerjee, K.N. (c) Hakim Khan (d) Shakti Singh
Dikshit and M.S Vats was associated with the
Ans. (c) : In the Battle of Haldighat or Haldighati which
exacavation of Harappa and Mohenjodaro while V.A
is fought between Maharana Pratap under the command
Smith is not associated with excavation of Harappa and of Hakim Khan and the Mughal emperor Akbar’s forces
Mohenjodaro. He was a renowned historian of Ireland
led by Man Singh on 18 June, 1576.
and published the Early History of India and The
Oxford History of India. 38. Where was the capital of Shivaji?
(a) Rajgarh (b) Sindhudurg
32. Gautama Buddha gave his first sermon at?
(c) Poona (d) Kolhapur
(a) Vaishali (b) Kaushambi
Ans. (a) : Rajgad/Rajgarh was the capital of Shivaji. In
(c) Sarnanth (d) Pawanpuri
Rajgarh fort Shivaji was crowned as Chhatrapti in 1674.
Ans. (c) : Gautama Buddha gave his first sermon at
It is located in the Pune district of Maharashtra.
Sarnath. This event is known as Dharma-Chakra-
Pravartana (learning of the wheel of law). 39. In which year was the Regulating Act passed?
(a) AD 1757 (b) AD 1765
33. Who is called 'the Napoleon of Ancient India'? (c) AD 1773 (d) AD 1793
(a) Chandragupta Maurya
Ans. (c) : Regulating Act was the first step taken by the
(b) Pushyamitra British government to control and regulate the affairs of
(c) Kanishka East India company in India. This Act was passed in
(d) Samudragupta 1773 AD.
56th-59th BPSC (Pre) Exam. 2015 181 YCT
40. In 1857 who was the Governor-General? 45. Who was the founder of Nalanda University?
(a) Wellesley (b) Dalhousie (a) Chandragupta Vikramaditya
(c) Canning (d) Minto (b) Kumargupta
Ans. (c) : Lord Canning was the Governor-General of (c) Dharmapal
India in 1857. After Government of India Act 1858 (d) Pushyagupta
passed, the name of post Governor General of India Ans. (b) : Kumaragupta (Shakraditya) of the Gupta
changed to Viceroy of India. The Viceroy was dynasty founded Nalanda University at modern Bihar in
appointed directly by the British Government. The first the early 5th century, and it flourished for 600 years
Viceroy of India was Lord Canning. until the 12th century. During the era of
41. After which incident Mahatma Gandhi had Harshavardhana and the Pala monarchs, it rose to
called Non-Cooperation Movement as his popularity. Until the 12th century. In 1193 AD, when
'Himalayan Blunder'? Turkish ruler Qutbuddin Aibak's general Bakhtiyar
(a) Chauri-Chaura (b) Kheda Satyagraha Khilji demolished Nalanda University. Dharmpal
revived Nalanda University.
(c) Nagpur-Satyagraha (d) Rajkot Satyagraha
It is a UNESCO World Heritage Site and listed as an
Ans. (a) : After the incident of Chauri-Chaura Mahatma
"Institute of National Importance" by the government.
Gandhi had called Non-Cooperation movement as his
Himalayan Blunder. Chauri-Chaura was a violent 46. At which place in Bihar Gandhiji had first
incident where a large crowd of peasants set on fire a Satyagraha?
police station that killed 22 policeman and due to this (a) Champaran (b) Chhapra
incident Gandhi Ji called off the Non-Cooperation (c) Bettiah (d) Patna
movement and called Non-cooperation movement as hisAns. (a) : The Champaran Satyagraha of 1917 was the
Himalayan Blunder. first Satyagraha movement led by Gandhi ji in India. It
42. Which movement started after the partition of was a farmer’s uprising that took place in Champaran
Bengal? district of Bihar, India during the British colonial
period.
(a) Civil Disobedience Movement
(b) Swadeshi Movement 47. Who was the author of 'Indica'?
(c) Quit India Movement (a) Vishnugupta (b) Megasthenes
(d) Non-Cooperation Movement (c) Diamachus (d) Pliny
Ans. (b) : Swadeshi Movement was started as an Ans. (b) : Indica was written by Megasthenes. He was a
antipartition movement which was started to oppose Greek historian and an ambassador of Seleucus at the
Lord Curzon’s decision of dividing the province of Mauryan Court in Pataliputra.
Bengal. Hence option (b) is the right answer. 48. Which Sikh Guru was born at Patna?
43. Who was the first Indian Woman President of (a) Nanak (b) Teg Bahadur
the Indian National Congress? (c) Hargobind (d) Gobind Singh
(a) Mrs. Annie Besant (b) Sucheta Kripalani Ans. (d) : Guru Gobind Singh was the 10th Sikh Guru
(c) Sarojini Naidu (d) Indira Gandhi and considered as last Sikh Guru. He was born in Patna
Ans. (c) : Sarojini Naidu became the first Indian woman in 1666. His introduced the five ‘K’s (Kesh, Kangha,
president when she presided over 40th session of Indian Kara, Kripan, Kaccha) that the Sikhs abide by.
National Congress held at Kanpur in 1925. Where as 49. Who was the President of Gaya Session of the
Mrs. Anni Besant was remembered as first woman Indian National Congress held in 1922?
president of INC when she presided over Calcutta (a) Chittaranjan Das (b) S.N. Banerjee
session in 1917. (c) Dr. Rajendra Prasad (d) Hakim Ajmal Khan
44. Where did Madam Cama hoist the first Ans. (a) : The Gaya Session of Indian National
Tricolour flag in 1907? Congress which is held in 1922 presided by
(a) London (b) Paris Chittaranjan Das.
(c) Moscow (d) Stuttgart 50. By which name is Jay Prakash Narayan
Ans. (d) : On August 22, 1907 Madam Bhikaji Cama Known?
hoisted the Indian flag in Stuttgart in Germany and (a) Lokmanya (b) Loknayak
became the first Indian to do so on foreign soil. (c) Lokhitvadi (d) Lokneta

56th-59th BPSC (Pre) Exam. 2015 182 YCT


Ans. (b) : Jayaprakash Narayan was known by Ans. (c) : London city is known for the largest metal
Loknayak. Which means people’s hero or leader of the trading centre. London Metal Exchange (LME) is the
people. Due to leading so many movements for the world's largest industrial-trading Metal and cost-rist
interest of the people. People had given him the epithet management center.
of Lok Nayak. • This question was deleted by commission.
51. Which country has the largest coastline? 56. Which of the following drainage systems fall
(a) USA (b) Australia into bay of Bengal?
(c) Canada (d) India (a) Ganga, Brahmaputra and Godawari
Ans. (c) : Canada's coastline is the world's longest, (b) Mahanadi, Krishna and Cauvery
measuring 243,042 km (includes the mainland coast and (c) Luni, Narmada and Tapi
the coasts of offshore islands). This compares with (d) Both (A) and (B)
Indonesia (54,716 km), Russia (37,653 km), the United Ans. (d) : The drainage system of Ganga, Brahmaputra
States (19,924 km) and China (14,500 km). Godwari, Mahandi, Krishna and Cauvery falls into bay
52.The largest producer of aluminum in the world is of Bengal while the drainage system of Luni, Narmada
(a) France (b) India and Tapi fall into Arabian sea. Hence option (d) is
(c) China (d) Italy correct option.
Ans. (c) : China is largest producer of Aluminium in the 57. The oldest oil field in India is
world followed by India, and Russia. [US Geological (a) Bombay High, Maharashtra
Survey’s 2021 data] (b) Ankleshwar, Gujarat
• However this question was deleted by Commission. (c) Navagam, Gujarat
53. The Pennines (Europe), the Appalachians (d) Digboi, Assam
(America) and the Aravallis (India) are Ans. (d) : The oldest oil field in India is Digboi, Assam.
example of The Digboi refinery Commissioned in December 1901.
(a) Young Mountains (b) Old Mountains 58. In India, 'Yarlung Zangbo River' is known as
(c) Block Mountains (d) Fold Mountains (a) Ganga (b) Indus
Ans. (d) : The Pennines (Europe), The Appalachians (c) Brahmaputra (d) Mahanadi
(America) and the Aravallis (India) are example of fold Ans. (c) : Yarlung Zangbo or Yarlung Tsangpo is a
mountains. river of Tibet when it enters in India, It is known as
• However this question was deleted by Commission. river Brahmaputra and when enters in Bangladesh, it is
54. Match the rivers with the cities through which known as Jamuna.
they are flowing and select the correct answer 59. Which one of the following is the driest region?
using codes given below: (a) Mumbai (b) Delhi
City River (c) Leh (d) Bengaluru
(A) Rotterdam 1. Seine Ans. (c) : Leh is the Driest region in among given
(B) Paris 2. Potomac option following. The reason behind this is Leh has a
(C) Budapest 3. Rhine Cold desert climate with long Cold winters from late
(D) Washington 4. Danube November to early March, with minimum temperatures
fell below freezing for most of the winter. Thus option
(a) A-2 B-3 C-1 D-4 (b) A-1 B-3 C-4 D-2
(c) is the correct answer.
(c) A-3 B-1 C-4 D-2 (d) A-4 B-3 C-2 D-1
60. Who amongst the following States has not
Ans. (c) : The correct match is - identified tribal community?
City - River (a) Maharashtra (b) Chhattisgarh
Rotterdam - Rhine (c) Haryana (d) Karnataka
Paris - Seine Ans. (c) : Haryana was carved out of Punjab on 1st
Budapest - Danube November 1966, as the 17th Indian State. Haryana has
Washington - Potomac not Identified any tribal community. Thus option (c) is
the correct answer.
55. Which is the largest metal trading centre?
61. Sultanpur Bird Sanctuary is located at
(a) Johannesburg (b) New York (a) Chandigarh (b) Bharatpur
(c) London (d) Singapore (c) Gurgaon (d) Gandhinagar
56th-59th BPSC (Pre) Exam. 2015 183 YCT
Ans. (c) : Sultanpur National Park & Bird Sanctuary is Ans. (b) : In the given options Madhya Pradesh and
located in the Gurgaon (now Gurugram) district of Odisha both states of India are most richly and endowed
Haryana. with iron ore.
68. What is chiefly found at Jharia in Jharkhand?
62. The forest area in India per person (in average
(a) Thorium (b) Silk
hectare) is (c) Gold (d) Coal
(a) 0.23 (b) 0.34 Ans. (d) : Jharia is famous for their Coal production
(c) 0.20 (d) 0.06 located at Jharkhand, India.
Ans. (*) : The forest cover per capita means the area 69. Which of the following towns is located
covered by forest per total land on earth thus, in Indian easternmost of Bihar?
context when the forest cover calculated per capita it (a) Bhagalpur (b) Patna
comes around 0.29 hectare. (c) Katihar (d) Purnia
Ans. (c) : Among the following districts, Katihar is the
63. Which of the following States has agricultural easternmost district of Bihar
area less than 60 percent? Westernmost District - Kaimur
(a) West Bengal (b) Punjab Northernmost District - West Champaran
(c) Uttar Pradesh (d) Bihar Southernmost District - Gaya
Ans. (a) : In the given option West Bengal has an 70. Which of the following lakes is situated in
agricultural area of less than 60 percent. West Bengal is Bihar?
situated in eastern India and shares its borders with (a) Anupam Lake (b) Sambhar Lake
(c) Sukhna Lake (d) Kama Lake
Jharkhand, Bihar, Odisha, Sikkim and Assam. Option
Ans. (a) : In the given options Lake Anupam is located
(a) is the correct answer. However BPSC has deleted
at Kaimur district (In the Kaimur wildlife sanctuary of
this question. Bihar)
64. 'Nagarjuna Sagar Multipurpose Project' is on 71. The Decentralization System was
which river? recommended by?
(a) Tapti (b) Kosi (a) C. Rajagopalchari (b) J.B. Kripalani
(c) Godawari (d) Krishna (c) Balwant Rai Mehta (d) Ashok Mehta
Ans. (c) : The Decentralisation system in India firstly
Ans. (d) : Nagarjuna Sagar Dam is a masonry dam
was recommended by Balwant Rai Mehta Committee.
across the Krishna River at Nagarjuan Sagar which
72. Mahalanobis Plan Model adopted in India in
straddles the border between Guntur district in Andhra the Mid-fifties aimed at?
Pradesh and Nalgonda district in Telangana. (a) Building a strong defence industry base
Therefore option (d) is the correct answer. (b) Setting up heavy industries which were
65. Dalmianagar of Bihar is famous for capital intensive
(c) Curbing inflation in the economy
(a) Silk (b) Cement
(d) Removing unemployment within a short
(c) Leather (d) Jute period
Ans. (b) : Dalminagar of Bihar is famous for cement. Ans. (b) : Mahalanobis plan model was the India’s 2nd
Therefore option (b) is the correct answer. five year plan proposed by P.C. Mahalanobis. This plan
66. How much square metres forest area is there in gave priority to setting up heavy industries which were
capital intensive.
Bihar?
73. Match the Column-A with Column-B and
(a) 2812 Square metres (b) 3612 Square metres selected the correct answer using the codes
(c) 2461 Square metres (d) 2612 Square metres given below:
Ans. (*) : The question was abandoned by the Column-A Column-B
commission because of all incorrect options. (A) 1955 1. Export-Import Bank of
India
67. The two States of India, most richly endowed
(B) 1964 2. Industrial Development
with iron ore, are
Bank of India
(a) Bihar and West Bengal (C) 1982 3. Industrial Credit and
(b) Madhya Pradesh and Odisha Investment
(c) Bihar and Odisha (D) 1987 4. Board of Industrial and
(d) Madhya Pradesh and West Bengal Financial Reconstruction

56th-59th BPSC (Pre) Exam. 2015 184 YCT


(a) A-1 B-2 C-3 D-4 (b) A-2 B-4 C-1 D-4 Ans. (a) : The Interest payment liability of government
(c) A-3 B-2 C-1 D-4 (d) A-4 B-1 C-2 D-3 comes under revenue expenditure. Revenue expenditure
Ans. (c) : The correct match is - is that part of government expenditure that does not
1955- Industrial credit and investment corporation of result in creation of assets.
India. e.g. Payment of Salaries, Wages, Pensions Subsidies
and interest etc.
1964 - Industrial development bank of India.
Therefore, option (a) is the correct answer.
1982 – Export-Import bank of India.
1987 – Board of Industrial and financial reconstruction. 78. The HDI rank of India as power the HDR-2014
is
74. Match the Column-A with Column-B and
(a) 137 (b) 128
selected the correct answer using the codes
(c) 135 (d) 147
given below:
Ans. (c) : Human Development Report- 2014 released
Column-A Column-B
by the United Nations Development Programme in
(A) Open general license 1. Employment
Tokyo on July 24 has ranked India 135 in a list of 187
(B) TRYSEM 2. External trade countries. Therefore option (c) is the correct answer.
(C) Wholesale price index 3. Credit control
79. Consider the following consume price indices:
(D) Cash-reserve ratio 4. Inflation I. Consumer Price Index for Industrial Workers
(a) A-2 B-1 C-4 D-3 II. Consumer Price Index for Agricultural
(b) A-2 B-4 C-3 D-1 Labourers
(c) A-4 B-3 C-2 D-1 III. Consumer Price Index for Rural Labourers
(d) A-3 B-2 C-1 D-4 IV. Consumer Price Index for Urban Non-Manual
Ans. (a) : The correct match is - Employees which of the above indices is are
Open general license - External trade complied by Central Statistical Organization
TRYSEM - Employment (CSO)?
Wholesale price index - Inflation (a) III and IV only (b) I, II and III only
Cash - reserve ratio - Credit control (c) IV only (d) I, II, III and IV

75. The controlling authority of government Ans. (c) : C.P.I. measures price changes from the
perspective of retail buyers. It is released by N.S.O.
expenditure is?
(formally known as C.S.O.). The types of C.P.I. as
(a) The Reserve Bank of India
follows –
(b) The Planning Commission
1. CPI for industrial workers (IW).
(c) The Finance Ministry
2. CPI for Agricultural labourer (AL).
(d) The Finance Commission 3. CPI for Rural Labourer (RL).
Ans. (c) : The Ministry of Finance, Government of 4. CPI for (Rural/Urban) combined of these, the first
India, is responsible for controlling expenditure and three are compiled by Labour and Employment and
revenue collection. Hence the correct option is (c). Fourth is compiled by the NSO in the Ministry of
76. Effective Revenue Deficit was introduced in the Statistics and Programme Implementation.
Union Budget of 80. What is the total production of oilseeds in India
(a) 2010-11 (b) 2011-12 in 2013-14?
(c) 2009-10 (d) 2012-13 (a) 34.32 million tonnes
Ans. (b) : Effective Revenue Deficit was introduced in (b) 43.34 million tonnes
the Union Budget of 2011-12. The concept has been (c) 20.89 million tonnes
introduced to ascertain the actual deficit in the revenue (d) 30.72 million tonnes
account after adjusting revenue the expenditure of Ans. (d) : The total production of oilseeds in India in
Capital nature. Hence, option (b) is the correct answer. 2013-14 was 30.72 million tonnes. However BPSC has
77. Interest payment is an item of deleted this question.
(a) Revenue expenditure 81. India earns maximum foreign exchange by the
(b) Capital expenditure export of?
(c) Plan expenditure (a) Iron (b) Tea
(d) None of the above (c) Textile (d) Rubber
56th-59th BPSC (Pre) Exam. 2015 185 YCT
Ans. (c) : Foreign exchange reserves are assets held on (c) Allowing the value of the rupee to be fixed by
reserve by a Central Bank in foreign currencies, which market forces
can include bonds, treasury bills, and other government (d) Developing an international market for
securities. India earns maximum foreign exchange by currencies in India
the export of Textile. Therefore option (c) is the correct
Ans. (b) : Convertibility of currency means when the
answer.
currency of a country free can be converted into the
82. The currency which has a tendency of quick foreign exchange at the market determined rate of
migration is called? exchange. The exchange rate is determined by the
(a) Hot currency (b) Gold currency demand for supply of a currency.
(c) Soft currency (d) Hard currency In simple word the convertibility of rupee means that
Ans. (a) : The currency which has a tendency of quick who have a foreign exchange. (eg. US. Doller, Pound
migration is called hot currency. Hence option (a) is the Starlings etc. ) can get them converted into rupees and
correct answer. vice versa at the market determined rate of exchange.
83. Which one of the following is not included in The rupee is both convertible on current account and
the National Food Security Missions? capital account.
(a) Oilseeds (b) Wheat 88. Consider the following important sources of tax
(c) Rice (d) Pulses revenue for the Central Government. (I).
Ans. (a) : When the question was asked. Oilseeds is not Corporation tax (II). Taxes on income other
included in the National Food Security Missions. The than corporation tax (III). Customs (IV). Union
mission aims to increase the production of rice, wheat, excise duties which one of the following in the
pulses, coarse, cereals and commercial crops, through correct descending order in terms of gross tax
area expansion and productivity enhancement. Thus revenue?
option (a) is the correct answer. But in 2021 The (a) I—II—IV—III (b) I—II—III—IV
Government of India is promoting the production and (c) III—I—II—IV (d) II—III—I—IV
productivity of Oilseeds through the National Food Ans. (a) : In financial year 2013-14 the government
Security Mission (Oilseeds) in all districts of India. receive maximum tax revenue form Corporation Tax
84. Ad hoc Treasury Bill System of meeting budget followed by Income Tax, Union excise duty and
deficit in India was abolished on Custom duty.
(a) 1st April, 1992 (b) 1st April, 1994 89. Which one of the following is the prescribed
(c) 31st March, 1996 (d) 31st March, 1997 investment limit for medium enterprises in the
manufacturing sector as power the Micro,
Ans. (d) : Ad hoc Treasury Bill System was completely Small and Medium Enterprises Development
closed in 1997-98. The system of ad hoc Treasury Bills Act. 2006?
to finance budget deficit will be discontinued with (a) More than `10 lakhs and less than ` 2 crores
effect from April 1, 1997. (b) More than ` 2 lakhs and less than ` 5 crores
85. R.N. Malhotra Committee is Associated with? (c) More than ` 5 lakhs and less than ` 10 crores
(a) Sick industries (b) Tax reforms (d) More than ` 10 crores
(c) Insurance sector (d) Banking Sector Ans. (a) : When the question was asked more than `10
Ans. (c) : In 1993 under the chairmanship R.N. lakhs and less than ` 2 crores prescribed investment
limit for medium enterprises in the manufacturing
Malhotra a former governor of R.B.I. a committee was
sector as power the Micro, Small and Medium
setup named as R.N. Malhotra Committee to make
Enterprises Development Act. 2006.
recommendations for reforms in the Insurance sector.
New definition of Micro, Small and Medium enterprise
Therefore option (c) is the correct answer. is now revised. The revised criteria is now based on
86. Service tax in India was introduced in the year investment in plant and machinery/equipment and
(a) 1994-95 (b) 1996-97 annual turnover which is as follows-
(c) 1998-99 (d) 1991-92 Micro Small Medium
Ans. (a) : The Service Tax was introduced in India on Investment in Not more Not more Not more
July 1, 1994 at the recommendations of Dr. Raja plant than Rs. 1 than Rs. than Rs. 50
Chelliah Committee on tax reforms. machinery or crore 10 crore crore
87. Convertibility of the rupee implies equipment
(a) Being able to convert rupee notes into gold Annual Not more Not more Not more
(b) Freely permitting the conversion of rupee to turnover than Rs. 5 than Rs. than Rs.
other major currencies and vice versa crore 50 crore 250 crore

56th-59th BPSC (Pre) Exam. 2015 186 YCT


90. What does infant mortality rate refer to? SAARC preferential trading arrangement (SAPTA)
(a) The ratio of number of children who die was negotiated in the year 1993.
before their fifth birthday out of every 1000 The Comprehensive Economic Cooperation Agreement
live births (CECA) is a free trade agreement between Singapore
(b) The ratio of number of children who die and India to strengthen bilateral trade. It was signed on
before their first birthday out of every 1000 29 June 2005.
live births 93. During which year was the annual average
(c) The ratio of number of children who die growth rate (at constant prices) of agriculture
before their fifth birthday out of every 100 and allied sectors negative?
live births (a) 2002-03 (b) 2003-04
(d) The ratio of number of children who die (c) 2005-06 (d) 2006-07
before their first birthday out of every 100 Ans. (a) : The contribution of the agriculture sector in
live births GDP was 20% in 2003-04. This was also the year when
Ans. (b) : The infant mortality rate is the number of the sector clocked 9.5% GDP growth, after the severe
infant deaths for every 1000 live births for calculation drought of 2002 when the growth rate was negative.
purpose death of under age of 1 years are considered. 94. Consider the following Statements?
91. In which of the following years was the trade I. Varsha Bima, the rainfall insurance
balance favourable to India? scheme, is managed by the Agriculture
(a) 1970-71 and 1974-75 Insurance Company of India Limited
(b) 1972-73 and 1976-77 (AICIL)
(c) 1972-73 and 1975-76 II. The scheme was introduced during the
(d) 1971-72 and 1976-77 2007 south-west monsoon period.
Ans. (b) : India had a favourable balance of trade only Which of the statements given above is/are
in 1972-73 (Rs. 104 Crore) and 1976-77 (Rs. 68 crore) . correct?
These two financial years are the only two financial (a) I only (b) II only
years in the history of Post independence India, in (c) Both I and II (d) Neither I nor II
which we had a favorable trade balance. Ans. (a) : Agriculture Insurance Company of India Ltd.
92. Consider the following agreements: (AICIL) introduced Rainfall Insurance Scheme known
I. ISLFTA (India-Sri Lanka Free Trade as “Varsha Bima” during the 2004 southwest monsoon
Agreement) period.
II. SAFTA (South Asian Free Trade Area) 95. Consider the following Statements?
III. CECA (Comprehensive Economic I. Bank rate is the rate of interest which RBI
Corporations Agreement between India charges its clients on their short-term
and Singapore) borrowing.
IV. SAPTA (South Asian Preferential Trade II. Repo rate is the rate of interest which RBI
Arrangement) charges its clients on their long-term
borrowing.
Which one of the following is the correct
Which of the statements given above is/are
Chronological order of the above agreements?
correct?
(a) I - III - II - IV (b) IV - I - II - III (a) I only (b) II only
(c) II - I - IV - III (d) I - II - III - IV (c) Both I and II (d) Neither I nor II
Ans. (b) : The commercial relations between Sri Lanka Ans. (d) : Repo rate are lending rates at which Central
and India has marked a historical milestone when it was Bank of India lends to commercial banks and other
signed as India-Sri Lanka Free Trade Agreement financial institutions.
(ISLFTA) on 28th December 1998, as the 1st bilateral Bank rate caters to long term financial requirements
free trade agreement of Sri Lanka. whereas Repo rate focuses on short term financial
needs.
The South Asia Free Trade Area (SAFTA) Agreement
96. Financial sector reforms in India consist of?
was signed on January 6, 2004 at the 12th SAARC
(a) Lowering down of CRR and SLR
Summit held in Islamabad. The treaty has come into
(b) Entry of private firms in Insurance sector
force on January 1, 2006 and will be fully implemented
(c) Deregulation of rate of interest
by December 31, 2015 (d) All of the above
56th-59th BPSC (Pre) Exam. 2015 187 YCT
Ans. (d) : Financial sector reforms in India consist of 100. Match the Column-A with Column-B and
Lowering down of Cash Reserve Ratio, Statutory select the correct answer using the codes given
Liquidity Ratio, allowing private sector in insurance and below:
a deregulated interest rate to increase efficiency of Column-A Column-B
economy. (A) National Agricultural Policy 1. 2004
97. Which of the following is not a component of (B) Marine Fishing Policy 2. 1978
Bharat Nirman? (C) New Foreign Trade Policy 3. 2000
(a) Rural housing (D) Seventh Finance Commission 4. 2014
(b) Rural electrification (a) A-2 B-1 C-3 D-4
(c) Agro-based industries (b) A-4 B-3 C-1 D-2
(d) Rural Telephony (c) A-1 B-4 C-2 D-3
Ans. (c) : Bharat Nirman was a plan for creating rural (d) A-3 B-1 C-4 D-2
infrastructure. It comprises projects on irrigation, roads, Ans. (d) :
housing, water supply, electrification and
Policy/Institution Year
telecommunication connectivity.
National Agricultural Policy - 2000
While the agro based industries is not component of
Marine Fishing Policy - 2004
Bharat Nirman.
New Foreign Trade Policy - 2014
98. According to the Planning Commission of
India, which of the following are correct for the Seventh Finance Commission - 1978
poverty line? 101. Who founded the Pagal Panth?
I. Rs. 42 per capita per day in urban area (a) Bulleh Shah
II. Rs. 26 per capita per day in rural area (b) Karam Shah
III. Rs. 32 per capita per day in urban area (c) Yaduvendra Singh
IV. Rs. 32 per capita per day in rural area (d) Swami Sahajananda
Choose the correct option form the Ans. (b) : Pagal Panthis were a mixture of the
following. Hinduism, Sufism and Animism, which became
(a) I and II (b) I and III prominent in Bengal in initial years of 19th century. The
(c) II and III (d) III and IV sect was founded by Karam Shah, and his son Tipu
Ans. (c) : As per planning commission of India poverty Shah led these people to uphold the religion and rights
line defined as - of the peasants in Bengal.
If a people who lives in rural area earns less than Rs. 26 102. Who were the Faraizis?
per capita per day and a people who lives in urban area (a) Followers of Haji Shariatullah
earns less then Rs. 32 per capita per day is considered as
(b) Followers of Dadu
below poverty line.
(c) Followers of the Arya Samaj
99. Consider the following schemes:
(d) Followers of the Muslim League
I. EAS II. TRYSEM
Ans. (a) : Faraizis were the followers of Haji
III. JRY IV. RLEGP
Shariatullah. Faraizi movement was religious reform
The correct chronological sequence of the
movement led by Haji Shariatullah Bengal.
launching of these schemes is
(a) II - IV - I - III (b) IV - II - III - I 103. The correct geographical location for the
(c) IV - III - I - II (d) II - III - IV - I Ramosi Uprising?
(a) Western India (b) Eastern Ghats
Ans. (*) : The Employment Assurance Scheme (EAS)
nd
introduced on 2 October. 1993 for rural area. (c) Eastern India (d) Western Ghats
Training to Rural Youth for Self Employment Ans. (d) : The Ramosis were the hilly tribes of the
(TRYSEM) Scheme was launched in 1979. Jawahar Western Ghats.
Rozgar Yojana (JRY) was launched in 1989. Rural The Ramosis served in the lower ranks of the Maratha
Landless Employment Guarantee Programme army and police, revolted in Satara in 1822, under the
(RLEGP) was launched in 1983. leadership of Chittur Singh in protest against the heavy
Thus, chronological sequence (II > IV > III > I) is assessment of land revenue and the harsh methods of its
correct answer. collection.

56th-59th BPSC (Pre) Exam. 2015 188 YCT


104. The Waghera Uprising happened in (a) Motilal Nehru and Jawaharlal Nehru, India's
(a) Surat (b) Poona relationship with the British Empire
(c) Calicut (d) Baroda (b) Jawaharlal Nehru, Local Self-Government in
Ans. (d) : The Waghera Uprising lasted from 1818 until India
1820 took place in Baroda. Aside from hostility of (c) Motilal Nehru, Constitutional arrangements in
foreign domination, the Waghera chiefs of Okha India
Mandal were obliged to take up arms by the exactions (d) Jawaharlal Nehru, Constitutional
of Gaekwad of Baroda, which were supported by the
arrangements in India
British government.
105. In order to control the media in India, 'Acts' Ans. (c) : When Lord Birkenhead, secretary of State for
were passed in? Indian Affair, challenged Indians to draft a constitution
(a) 1835, 1867, 1878, 1908 which is acceptable to all, then Indian political parties
(b) 1854, 1864, 1872, 1910 accepted the challenge. In all parties conference, 1928 a
(c) 1854, 1872, 1908, 1910 committee to draft a constitution was formed under
(d) 1867, 1908, 1910, 1919 leadership Motilal Nehru which submitted its report
“Constitutional Arrangement in India”.
Ans. (a) : Following acts were passed in British India -
Press Act - 1835 109. What was the common relationship between
Katherine Mayo, Aldous Huxley, Charles
Registration Act - 1867
Andrews and William Digby?
Vernacular Press Act - 1878
(a) They wrote commentaries on the condition of
Newspaper (Incitement to Offences) Act - 1908
India during the British rule
Thus, the correct option is (a).
(b) They were supporters of the Indian National
106. The process of split in the Congress in the early Movement
years of the twentieth century began over? (c) They were opponents of the India National
(a) Strategies of the Congress Movement Movement.
(b) Objectives of the Congress Movement (d) They were friends of Mahatma Gandhi
(c) Participation of the people in the congress
Ans. (a) : Katherine Mayo, Aldous Huxley, Charles
Movement
Andrews and William Digby wrote commentaries on
(d) All of the above
condition of India during British rule.
Ans. (d) : The process of split in the Congress in the
110. Who penned the following lines "Sarfaroshi ki
early years of 20th century began over the differences
tamanna ab hamare dil me hai, dekhna hai zor
between extemists and moderates which originated
kitna baju-e-qatil me hai"?
during Swadeshi movement.
(a) Bismil (b) Rajguru
Extremists wanted to extend the Swadeshi and the
(c) Bhagat Singh (d) Azad
boycott movement from Bengal to the rest of the
country and they wanted to gradually extend the boycott Ans. (a) : “Sarfaroshi ki tamanna ab hamare dil me hai,
from foreign goods to every form of association or dekhna hai zor kitna baju-e-qatil me hai” is a line of an
cooperation with the colonial government. Moderates Urdu patriotic poem written by Indian revolutionary
were of opposite views. Ram Prasad Bismil.
Thus, currect option is (d). 111. Who raised the demand of 'Complete
107. 'New Lamps for Old' was a series of articles Independence' for the first time in 1921?
(1893-94) that criticized the Congress for being (a) Maulana Muhammad Ali
out of touch with the 'proletariat'. Who was the (b) Pandit Jawaharla Nehru
author of these articles?
(c) Mahatama Gandhi
(a) Aurobindo Ghose (b) A.O. Hume
(d) Maulana Hasrat Mohani
(c) G.K. Gokhale (d) B.G. Tilak
Ans. (a) : ‘New Lamps for Old’ was a series of articles Ans. (d) : Maulana Hasrat Mohani and Swami
published in 1893-1894 was authored by Aurbindo Kumaranand joint moved resolution of complete
Ghose. He was leader of extremist fraction of Indian independence in 1921 Allahabad session of All India
National Congress. Congress Committee.
108. Fill in the blanks: The Nehru Report was 112. Identify the years those are closest to the
drafted by a committee headed by ….. and the founding of the Communist Party of India and
subject was ….. the RSS respectively?
56th-59th BPSC (Pre) Exam. 2015 189 YCT
(a) 1915, 1914 (b) 1925, 1925 118. In which area was Rahul Sankritayan active in
(c) 1928, 1925 (d) 1925, 1929 the Non-Cooperation Movement of 1920?
Ans. (b) : The Communist Party of India and RSS both (a) Chhapra (b) Delhi
were formed in 1925. (c) Lucknow (d) Patna
Thus correct answer is (b). Ans. (a) : Rahul Sankrityayan was an independence
113. Swami Sahajananda Saraswasti formed the All activist as well as a writer, during Non Cooperation
India United Kisan Sabha with the demand for Movement he was active in Chhapra, Bihar.
the 'nationalization of land and waterways'
(a) Just before his death 119. Which of the following statements is correct?
(b) At a very Young age (a) In 1857, the Maharajas of Darbhanga,
(c) In the 1930s Dumraon and Hatwa and their fellow
(d) In the 1920s landlords helped the English with men and
Money.
Ans. (c) : Swami Sahajananda Saraswasti formed the
(b) In 1857, the Maharajas of Darbhanga,
All India United Kisan Sabha in Lucknow (1936).
Dumraon and Hatwa and their fellow
114. Fill in the blank: In …..., the rights of the Landlords helped the Enligh with men but not
tenants on land in Bengal and Bihar were given with Money
by the Bengal Tenancy Act.
(c) In 1857, the Maharajas of Darbhanga,
(a) 1885 (b) 1886 Dumraon and Hatwa and their fellow
(c) 1889 (d) 1900 landlords helped the English with money but
Ans. (a) : Bengal Tenancy Act 1885 was an enactment not with men.
of the Bengal government defining the natural rights (d) In 1857, the Maharajas of Darbhanga,
and liabilities of Zamindars and tenants in response to Dumraon and Hatwa and their fellow
widespread peasant discontent. landlords opposed the English.
115. Which Round Table Conference held in 1932? Ans. (a) : During 1857 AD, the Maharajas of
(a) First (b) Second Darbhanga, Dumraon and Hatwa and their fellow
(c) Third (d) Fourth landlords helped the English with men and Money.
Ans. (c) : Round Table Conference were a series of Hence the correct option is (a).
meetings in three sessions called by the British 120. Who highlighted the Plight of the peasants of
government to consider the future constitution of India.
Champaran by Writing under pseudonyms like
third Round Table conference was held in 1932.
'Dukhi', 'Dukhi Atma', 'Dukhi Hridaya'?
116. One of the popular beliefs in Bihar during the
(a) Peer Muhammad Moonis
Non-Cooperation Movement of 1920-22 was
that victory would come to them because (b) Rajendra Prasad
Gandhi? (c) Sahajananda Saraswati
(a) Was the epitome of Dharma (d) S.N. Sinha
(b) Was an accomplished politician Ans. (a) : Peer Muhammad Moonis highlighted the
(c) Knew the ways to defeat the English plight of the peasants of champaran by writing articles
(d) Knew the ways to defeat the English of ‘Dukhi’, ‘Dukhi Atma’, ‘Dukhi Haridaya’ in January
Ans. (a) : There was a popular belief in Bihar during 1916. Hence, the correct answer is (a). Peer Muhammad
Non-cooperation movement in 1920-22 that Gandhi was Moonis was founder of Bihar Hindi Sahitya Sammelan
the symbol of Dharma. Thus they would eventually win. which was founded in Sonepur on 1919.
117. Who among the following was/were peasant 121. Let f : R → R be defined by f(x) = x2 - 3x + 2.
leaders (s) from Bihar? I. Swami Vivekananda
II. Swami Sahajananda III. Sardar Then the value of f(f(5)) is
Vallabhbhai Patel Choose the correct option (a) 90 (b) 100
from the following: (c) 110 (d) 80
(a) I and II (b) II and III 2
Ans. (c) : f(x) = x - 3x + 2
(c) I and III (d) II only
Ans. (d) : Swami Sahajananda Saraswati was peasant So, f(f(5)) = ?
leader of India. He was born in Ghazipur of Uttar f(5) = 25 – 15 + 2 = 12
Pradesh. However his social and political activities were f(f(5)) = f(12) = 144 – 36 +2 = 110
mostly focused on Bihar in initial days and gradually
spread to rest of India. Hence, option (c) is the correct answer.

56th-59th BPSC (Pre) Exam. 2015 190 YCT


122. If 2nC3 : nC2 = 12:1, then the value of n is?
Here, a= 1 , b= 5 , c=7
(a) 6 (b) 5 2 6 6
(c) 4 (d) 3 5 1 7
Hence, 2× = +
Ans. (b) : 2nC3 : nC2 = 12:1 6 2 6
2n ( 2n – 1)( 2n – 2 ) n ( n – 1) 12 10 3 + 7
=
: = 6 6
3 × 2 ×1 2 ×1 1
10 = 10
 n n!  6 6
∵ c r = 
 r!( n − r ) !  Hence the series is in arithmetic progression.
127. A triangle has vertices (1,6), (3,0) and (-3,-7).
(2n–1) = 9 Its area in square units is?
n=5 (a) 10 (b) 25
Hence, option (b) is the correct answer. (c) 30 (d) 40
123. The variance of 7, 7, 7, 7, 7 is Ans. (b) : Area of Triangle ∆
(a) 7 (b) 0 1 6 1
1
(c) 3 (d) 4 = 3 0 1
2
Ans. (b) : –3 –7 1
Variance = Summation ( x – x ) / number of observation
2
1 6 1
1
So, Mean = (7+7+7+7+7)/5 = 2 –6 0
2
x = 35/5 = 7 –4 –13 0
All observations are same then summation ( x – x ) = 0
2
R 2 = R 2 – R1 
 
Variance = 0/5 = 0 R 3 = R 3 – R1 
Hence, option (b) is the correct answer.
124. Given, log2 = 0.30103. Then log 5 is
=
1
2
{24 – ( –26 )}
(a) 0.2301 (b) 0.5302 1
= {24 + 26} = 25
(c) 0.720 (d) 0.69897 2
Ans. (d) : Given, log2 = 0.30103 Therefore option (b) is the correct answer.
According to the question, Second Method -
1
10 Area = =  x1 ( y 2 − y3 ) + x 2 ( y3 − y1 ) + x 3 ( y1 − y 2 ) 
log5 = log
2 2
1
= log10 – log2 = 1( 0 + 7 ) + 3 ( −7 − 6 ) + ( −3)( 6 − 0 ) 
= 1 – 0.30103 2
1
= 0.69897 = [ 7 − 39 − 18]
125. The solution of the equation 2x+10 ≥0 is given 2
1
by = × ( −50 ) = 25 square unit
(a) x ≥ -8 (b) x ≥-5 2
(Area can not be negative)
(c) x ≥-9 (d) x ≥-10
128. The angle between two lines 3x + y - 7 = 0 and x
Ans. (b) : According to the question, + 2y + 9 = 0 will be
2x + 10 ≥ 0 (a) 600 (b) 450
0
2x ≥ –10 (c) 30 (d) 900
x ≥ –5 Ans. (b) : Let m1, m2 represent the slope of the given
lines.
Therefore option (b) is the correct answer.
−coefficient of x
1 5 7 Then, m1 = = −3
126. The series + + is: coefficient of y
2 6 6
1
(a) Arithmetic series (b) Geometric series m2 = –
2
(c) Harmonic series (d) Exponential series
If θ are acute angles between the given lines,
Ans. (a) : Common Difference (r) then
5 1 7 5 2
= – = – = = 1 (r)  m – m2 
6 2 6 6 6 3 tan θ =  1 
for arithmetic series –  1 + m1 m 2 
Condition - 2b = a + c

56th-59th BPSC (Pre) Exam. 2015 191 YCT


vigorously with air as well as water. Therefore option
1
−3 +   (d) is the correct answer.
 2  =1
134. Who synthesized the DNA in Vitro?
1
1 + ( 3)   (a) Arthur Kornberg (b) Robert Hooke
 2
(c) Edward Jenner (d) Joseph Lister
θ = 45º
Therefore option (b) is the correct answer. Ans. (a) : Arthur Kornberg was a American scientist
who was awarded Nobel Prize in Medicine for his
129. If tan 600 = 3 , then the value of sec 600 is - discovery of Synthesizing DNA in Vitro.
(a) 4 (b) 3
135. What was the fissionable material used in the
(c) 2 (d) 1
bombs dropped at Nagasaki (Japan) in the
Ans. (c) : Sec θ – tan θ = 1
2 2
years 1945?
Sec2 60o – tan260 = 1 (a) Sodium (b) Potassium
( 3) = 1
2
Sec2 60o – (c) Plutonium (d) Uranium
Sec2 60o = 1 + 3 Ans. (c) : Plutonium 239 was used as Fissisonable
Sec 60o = 2 material in bombs dropped at Nagasaki (Japan) in year
Therefore option (c) is the correct answer. 1945.
130. A circle has area 154 square units. Its 136. The scientist who first discovered that the earth
diameters have equations 2x - 3y + 12 = 0 and x revolves round the sun was?
+ 4y - 5 = 0. Then the radius of the circle is ? (a) Newton (b) Dalton
(a) 8 units (b) 7 units (c) Copernicus (d) Einstein
(c) 6 units (d) 5 units Ans. (c) : Scientist Copernicus gave heliocentric
Ans. (b) : Explanation :- astronomical model and discovered that the earth
πr 2 = 154 revolves around the Sun.
22 2 137. Cosmic rays
r = 22 × 7
7 (a) Are charged particles
r2 = 7×7 (b) Are uncharged particles
r = 7 Units (c) Can be charged as well as uncharged
Therefore option (b) is the correct answer.
(d) None of the above
131. Cloves are obtained from which of the
following parts of the plant? Ans. (a) : Cosmic rays are high energy charged
(a) Dried leaves (b) Dried Stems particles which are mainly proton and atomic nuclei.
(c) Dried seeds (d) Dried flower buds 138. Which of the following elements is found in all
Ans. (d) : Cloves are obtained from Dried flower buds organic compounds?
of Syzygium aromaticum. Thus option (d) is the correct (a) Carbon (b) Calcium
answer. (c) Nitrogen (d) Oxygen
132. Oncogenes are associated with? Ans. (a) : Carbon is found in all organic compounds.
(a) Tuberculosis (b) Hepatitis
The Carbon atoms are bound to on another and to other
(c) Cancer (d) Typhoid
atoms by covalent bonds.
Ans. (c) : A mutated (Changed) form of a type of gene
called a proto-oncogene, which is involved in normal 139. Which of the following can cause cancer as well
cell growth and division. When a proto-oncogene is as cure it depending upon its intensity and use?
changed so that too many copies are made or it becomes (a) Tobacco (b) Alcohol
more active than normal, it is called an oncogene. An (c) Ionized radiation (d) Ultraviolet rays
oncogene is a when mutated has the potential to cause
Ans. (c) : Hige energy radiation, such as x-rays gamma
cancer.
rays, alpha particles, beta particles and neutrons can
133. Which is the most reactive metal?
damage DNA and cause cancer and it is also used in
(a) Sodium (b) Calcium
(c) Iron (d) Potassium breaking off cancer cells to cure cancers and tumers.
Ans. (d) : Among the given metals, potassium Hence option (c) is correct answer.
exceptionally reactive. Potassium is the most 140. The total volume of blood in a normal adult
electropositive and thus, most reactive metal. It reacts human being is
56th-59th BPSC (Pre) Exam. 2015 192 YCT
(a) 5-6 litres (b) 3-4 litres 146. The smallest organism, capable of autonomous
(c) 8-10 litres (d) 10-12 litres growth and reproduction, is
Ans. (a) : The amount of blood circulating within an (a) Virus
individual depends on their size and weight, but average (b) Bacteria
(c) Mycoplasma
human adult has nearly 5 liters of circulating blood.
(d) Bacteriophage
Thus (a) is correct answer.
Ans. (c) : Virus and viroids are obligate parasites that
141. Typhoid and cholera are typical examples of cannot exhibit functions of the living organisms as
(a) Infectious diseases growth and reproduction. Mycoplasma is the smallest
(b) Airborne diseases bacteria that does not possess a cell wall. The organism
(c) Waterborne diseases is living and can perform functions like growth and
reproduction through binary fission.
(d) None of the above
Thus, the correct answer is option C.
Ans. (c) : Typhoid and Cholera are waterborne disease
147. The first human heart transplant was
caused by water that has been contaminated by animal performed in
or human waste. (a) America
142. Energy is stored in liver and muscles in the (b) England
form of (c) South Africa
(a) Carbohydrate (b) Fat (d) France
(c) Protein (d) Glycogen Ans. (c) : On December 3, 1967, 53-year-old Louis
Washkansky receives the first human heart transplant at
Ans. (d) : Glucose is the main source of fuel for our
Groote Schuur Hospital in Cape Town, South Africa.
cells. When the body doesnot need to use the glucose
148. The average blood flow through kidneys per
for energy, it stores it in the liver and muscles. This minute is
stored form of glucose is made up of many connected (a) 1000 cc
glucose molecules and is called glycogen. (b) 1200 cc
143. Who discovered bacteria? (c) 200 cc
(a) Fleming (b) Lamble (d) 500 cc
(c) Temin (d) Leeuwenhock Ans. (b) : The correct answer is 1200 cc. The average
blood flow through kidneys per minute is 1200 cc.
Ans. (d) : Antoni Van Leeuwenhock is credited with
149. A plant bud is
discoery of bacteria in 1676.
(a) An embryonic shoot
144. The reason of large-scale diversity among the (b) An embryonic leaf
organisms is (c) An endosperm
(a) Adaptation (b) Cooperation (d) A seed
(c) Mutation (d) Polyploidy Ans. (a) : Buds are small outgrowth present terminally
on the stem or in the axil of leaves. They are derived
Ans. (a) : The reason for large scale diversity among
from meristematic tissues and develop into leaf, shoot
the organisms is adaption. Adaption is Behavioural or flower.
change to suit natural environment. 150. Which of the following gases is used in cigarette
145. What is vermiculture? lighters?
(a) The science of raising worms (a) Butane
(b) Methane
(b) The science of studying animals (c) Propane
(c) The science of studying fishes (d) Radon
(d) The science of killing worms Ans. (a) : Butane is a hydrocarbon and a highly
Ans. (a) : Vermiculture means artificial rearing or flammable, colourless, odourless, easily liquefied gas. It
is typically used as fuel for cigarette lighters and
cultivation of worms (Earthworms) and the technology portable stoves, a propellant in aerosols, a heating fuel,
is the scientific process of using them for the betterment a refrigerant, and in the manufacture of a wide range of
of human beings products.
56th-59th BPSC (Pre) Exam. 2015 193 YCT
60th–62th Bihar Public Service Commission
Preliminary Examination, 2016-17
GENERAL KNOWLEDGE & GENERAL SCIENCE
(Solved Paper with Detail Explanation)
1. Who was the leader of revolutionaries in Bihar (c) Lord Curzon
during the Revolt of 1857? (d) Lord Minto
(a) Namdar Khan (e) None of the above/More than one of the
(b) Babu Kunwar Singh above
(c) Birsa Munda Ans. (c) : In 1900, Lord Curzon wrote that "In my
(d) Shankar Shah belief, Congress is to tottering to its fall and one of my
(e) None of the above/ great ambitions while in India is to assist it to a peaceful
More than one of the above demise" to the British foreign Minister in his Letter.
Ans. (b) : In Bihar, The revolt of 1857 was led by Babu 5. Who drew Mahatma Gandhi's attention
Kunwar Singh, the Zamidar of Jagdishpur. He passed towards the exploitation of the peasants by the
away on 26th April 1858. European Indigo planters?
2. Birsa was captured, while asleep, on: (a) Baba Ram Chandra
(a) 1February, 1900 (b) Raja Kumar Shukla
(b) 2 February, 1900 (c) Swami Sahajananda Saraswati
(c) 3 February, 1900 (d) Sri Krishna Sinha
(d) 4 February, 1900 (e) None of the above/More than one of the
(e) None of the above/More than one of the above
above Ans. (b) : Gandhi Ji was requested by Rajkumar
Ans. (e) : In the 1890s, Birsa Munda emerged as the Shukla, a local man, to look into the problems of the
head of a movement of the Munda tribes of Singhbhum farmers in context of Indigo planters of Champaran in
and Ranchi districts of Chotanagpur region. He was Bihar. The European Planter had been forcing the
called 'Dharti Aaba.' On 3rd March 1900, Birsa Munda peasants to grow indigo of 3/20 part of the total land
was arrested by British Army, while he was sleeping (called Tinkathiya System).
with his tribal guerrila army at Jamkopai forest in 6. Who was elected the first President of All India
Chakradharpur. He died on 9th June 1900 while lodged Kisan Sabha at Lucknow in 1936?
at the Ranchi Jail. (a) N.G. Ranga
3. "The Muslims, if contented and satisfied, (b) E.M.S. Namboodripad
would become the greatest bulwark of British (c) Swami Sahajananda Saraswati
power in India". Who wrote it? (d) Acharya Narendra Dev
(a) Herbert Risley (e) None of the above/More than one of the
(b) Lord Lytton above
(c) W.W. Hunter Ans. (c) : The All India Kisan Sabha was founded in
(d) H.N. Brailsford Lucknow in April 1936 with Swami Sahajanand
(e) None of the above/More than one of the Saraswati as the President and N.G. Ranga as the
above General Secretary. A kisan manifesto was issued and a
Ans. (c) : Scottish historian, Sir W.W. Hunter had periodical under Indulal Yagnik started.
written in his book named "The Indian Musalmans'' that 7. Sir Michael O' Dwyer was shot dead on 13th
The Muslim, if contented and satisfied, would become March 1940 in London by:
the greatest bulwark of British power in India. (a) Madan Lal Dhingra
4. "In my belief, Congress is to tottering to its fall (b) M.P.T. Acharya
and one of my great ambitions while in India is (c) V.D. Savarkar
to assist it to a peaceful demise." Who wrote it? (d) Udham Singh
(a) Lord Lytton (e) None of the above/More than one of the
(b) Lord Dufferin above
60th-62th BPSC (Pre) Exam. 2016-17 194 YCT
Ans. (d) : Udham Singh had assassinated Michael O' Ans. (c): Bihar Freedom fighter, Sheel Bhadra Yajee
Dwyer, the Lieutenant Governor who presided over the joined Subhas Chandra Bose to found All India Forward
brutal British suppression of the 1919 protest in Punjab Bloc and he also took an active role in INA movement.
on 13th March 1940. Udham Singh was hanged in 1940 12. Who is regarded as "The Mother of Indian
for his deed. Revolution"?
8. Who was the Prime Minister of England when (a) Mrs. Annie Besant
the Montague-Chelmsford Act was passed in (b) Snehlata Wadekar
1919?
(c) Sarojini Naidu
(a) Lloyd George
(d) Madam Bhikhaji Rustam Cama
(b) George Hamilton
(e) None of the above/More than one of the
(c) Sir Samuel Hoare
above
(d) Lord Salisbury
Ans. (d) : Madam Bhikhaji Rustam Cama is referred to
(e) None of the above/More than one of the
as the "Mother of the Indian Revolution. On 22nd
above
August 1907, She became the first person to hoist
Ans. (a) : David Llyod George (1916-1922) was the Indian flag in Stuttgart in Germany.
Prime Minister of United Kingdom, when the Montague
Chelmsford Act was passed. 13. Which jornal was not associated with the
revolutionary activities?
9. Who left Patna College with just 20 days
(a) Sandhya
remaining for his examination during the Non-
Cooperation Movement? (b) Yugantar
(a) Rajendra Prasad (c) Ghadar
(b) Braj Kishore (d) Young India
(c) Jai Prakash Narayana (e) None of the above/More than one of the
(d) Sri Krishna Sinha above
(e) None of the above/More than one of the Ans. (d) : 'Young India' was a weekly Journal in
above English started by Mahatma Gandhi to spread his
Ans. (c) : Jai Prakash Narayan had left the Patna college ideology of non violence and resistance against the
with just 20 days remaining for his examination during British while Sandhya, Yugantar and Ghadar Journals
the Non Cooperation Movement. were associated with the revolutionary activies
10. Who became champion of socialism and 14. First Indian elected to the British House of
wanted to overthrow of British rule, Princely Commons was Dadabhai Naoroji who
states, Landlordism and Capitalism in 1933? contested on the ticket of,
(a) Rajendra Prasad (a) Liberal Party
(b) Jawaharlal Nehru (b) Labour Party
(c) Bhulabhai Desai (c) Conservative Party
(d) Sardar Patel (d) Communist Party
(e) None of the above/More than one of the (e) None of the above/More than one of the
above above
Ans. (b) : Jawaharlal Nehru became the champion of Ans. (a) : Dada Bhai Naoroji was the first Indian to be
Socialism and wanted to overthrow British rule, elected to the membership in the British Parliament. He
Princely States, Landlordism and Capitalism in 1933. was elected on the ticket of Liberal Party for finsbury
11. Who joined Subhas Chandra Bose to found All Central at the 1892 general election.
India Forward Bloc and actively associated 15. Swaraj Party was formed following the failure
with the INA movement? of:
(a) Jai Prakash Narayan (a) Non-Cooperation Movement
(b) Baikunth Shukla (b) Civil Disobedience Movement
(c) Sheel Bhadra Yajee (c) Rowlatt Bill Satyagraha
(d) Ram Narain Prasad (d) Champaran Satyagraha
(e) None of the above/More than one of the (e) None of the above/More than one of the
above above
60th-62th BPSC (Pre) Exam. 2016-17 195 YCT
Ans. (a) : The Swaraj Party was formed in the after Ans. (b) : The Indian Association was the precursor of
math of important event such as withdrawal of the Non- Indian National Congress. Indian Association was
Cooperation Movement by Gandhiji, The Government founded by Surendranath Banerjee and Ananda Mohan
of India Act of 1919 and the 1923 election. On 1st Bose in 1876. It Later merged with INC in 1886.
January 1923, Swaraj Party was established, with C.R. 20. Who organized the "Khudai Khidmatgar" in
Das as President and Motilal Nehru as one of the 1929?
(a) Abdul Gaffar Khan
Secretaries.
(b) Ali Brothers
16. The Chittagong Armoury Raid had been (c) Ansari Brothers
planned by: (d) Maulana Abul Kalam Azad
(a) Surya Sen (e) None of the above/More than one of the
(b) Chandan Dutta above
(c) Vidhan Ghosh Ans. (a) : 'Khudai Khidmatgar' was founded by Khan
Abdul Gaffar Khan in 1929, also known as 'Bacha
(d) Jaitn Das Khan' and Frontier Gandhi'. Khudai khidmatgar was a
(e) None of the above/More than one of the non violent movement against British occupation of the
above Indian Sub continent.
Ans. (a) : Chittagong armoury raid was an attempt by 21. A,B,C invested 20 lac rupees in a business in
armed Indian independence fighters led by Surya Sen the ratio 7:2:1. The total profit is 20%. A has to
on 18th April 1930 to raid the Chittagong armoury of pay 30% tax and B has to pay 20% tax. A's net
police and auxiliary forces in the Bengal presidency of profit is what percent of B's net profit.
(a) 118.8%
British India. Surya Sen was arrested in February 1933
(b) 180.0%
and hanged in January 1934. (c) 306.25%
17. The Cabinet Mission came to India in: (d) 304.5%
(a) February 1942 (e) None of the above/More than one of the
(b) March 1942 above
(c) April 1942 Ans. (c) Let us take the amount invested by A, B, C be
7x, 2x, x, respectively.
(d) May 1942 Total amount invested by A,B, C = Rs. 20 lac
(e) None of the above/More than one of the According to question,
above ⇒ 7x + 2x + x = 20
Ans. (b) : The Cabinet Mission reached Delhi on 24th ⇒ x = Rs. 2 lac
March 1946. This Mission had three British Cabinet A investment = 7x = Rs. 14 lac
members - Pethick Lawrence, Stafford Cripps and A.V. B investment = 2x = rs. 4 lac
Alexander. This mission's aim was to discuss the C investment = x = Rs. 2 lac
transfer of power from British to Indian leadership. Total profit = 20% of 20 lac ⇒ Rs. 4 lac
Now, the profit share of A, B, C
18. A leader of Bihar who left his lucrative practice ⇒ A + B + C = 10x
during Non-Cooperation Movement was:
⇒ 4 = 10 x ⇒ x = Rs. 40000
(a) Jai Prakash Narayan A's Profit = 7x = Rs. 280000
(b) Rajendra Prasad B's Profit = 2x = Rs. 80000
(c) Sahajanand Saraswati C's Profit = x = Rs. 40000
(d) Raj Kumar Shukla New Profit = Profit – Tax % of Profit
(e) None of the above/More than one of the A's Net Profit = 280000 – 30% of 280000 = Rs. 196000
above B's Net Profit = 80000 – 20% of 80000 = Rs. 64000
A's Net Profit = % of B's Net Profit
Ans. (b) : Dr. Rajendra Prasad was the leader of Bihar
who left his lucrative practice during Non - Cooperation 196000
⇒ × 100 = 306.25%
Movement. 64000
19. The precursor of Indian National Congress ∴ A's Net profit is 306.25% B's net profit.
was: 22. If SURGICAL-STRIKE is coded as 13979313-
(a) British Indian Association 129925, then METRO-TRAIN is coded as
(b) Indian Association (a) 15295-29195
(b) 45296-29195
(c) The Indian National Union
(c) 45295-29194
(d) Indian League (d) 15296-29195
(e) None of the above/More than one of the (e) None of the above/More than one of the
above above
60th-62th BPSC (Pre) Exam. 2016-17 196 YCT
old Gross salary = 7700+ 9625 = Rs 17325
Ans. (b) Here, the sum of the code digits of the letters is
being done. Old TDS = 20% of Gross salary
S U R G I C A L →S T R I K E ⇒ (20/100)×17325 = Rs. 3465
Old Net Salary = 17325 – 3465 = Rs. 13860
↓↓↓ ↓↓↓↓↓ ↓↓ ↓↓↓↓
New DA = 132% of basic salary
1 3 9 7 9 31 3 1 2 9 9 2 5 ⇒ (132/100) × 7700 = Rs. 10164
Similarly, New Gross Salary = 7700 + 10164 = Rs. 17864
METRO – TRAIN → 45296 – 29195 New TDS = 22% of Gross Salary
23. x is a consecutive whole number and the first 22
four values of expression x3+3y-3 are 7, 20, 45, ⇒17864 × 100 = Rs. 3930
88 then the fifth value is New net salary = 17325 – 3930 = Rs. 13934
(a) 137 (b) 155
Increase in Net Salary = 13934 – 13860
(c) 158 (d) 143
= Rs. 74
(e) None of the above/More than one of the
∴ The salary got increased by Rs. 74
above
Ans. (b) Substitute the value of x in given expression 26. Inspect the following figure and find the region
which represents the government employee
⇒ 1 + 3y – 3 = 7 who live in own house and compulsorily
⇒ y=3 graduate.
for x = 2 (second value)
⇒ 8 + 3y – 3 = 20
⇒ y=5
Here
x = 1, then y =3
x = 2, then y = 5
so x is increasing by 1 and y is increasing by 2
Then for x = 5 (fifth value), then y = 11
Substitute x = 5 and y = 11 in equation (a) EGHD (b) BED
53 + 3 × 11 – 3 = 155 (c) BHG (d) HGI
∴ The fifth value is 155 (e) None of the above/More than one of the
24. In the series of numbers and alphabets above
A26H,C24F, G20B, the next term is Ans. (c) The region BHG represents the government
(a) M13D (b) O11C employee who live in own house and compulsorily
(c) M12B (d) M14E gradutate.
(e) None of the above/More than one of the 27. If sign +represents multiplication, × addition, -
above division, ÷ difference, > equal and = represents
Ans. (e) The sequence of the given letter greater than, then which of the following
numbers is as follows - relation is true?
+2 +4 +6 (a) 12 - 2 + 3 ÷ 8 × 1 > 12
A → C → G → M
−2 −4 −6
(b) 11 × 2 ÷ 4 - 2 + 1 = 11
26  → 24  → 20  →14 (c) 7 ÷ 2 × 5 - 5 + 2 = 7
−2 −4 −6
H  → F  → B  →V (d) 5 × 6 - 3 ÷ 3 + 1 > 4
So, the next term is M14V which is not given in (e) None of the above/More than one of the
options. above
Hence, the correct answer is "None of the above/more Ans. (d) On Checking options,
than one of the above". (a) 12 – 2 + 3 ÷ 8 × 1 > 12
25. The dearness-allowance of a person with basic Interchanging the sign,
salary ` 7700 is increased to 132% from 125% 12 ÷ 2 × 3 – 8 + 1 = 12
and the tax deduction on both in increased to 18 – 8 + 1 = 12
22% from 20%. He got salary increased by 11 ≠ 12
(a) ` 74.00 (b) ` 77.00 (b) 11 × 2 ÷ 4 - 2 + 1 = 11
(c) ` 385.00 (d) ` 369.00 Interchanging the sign,
(e) None of the above/More than one of the 11 + 2 – 4 ÷ 2 × 1 >11
above 11 + 2 – 2 × 1 > 11
Ans. (a) Old DA = 125% of basic salary 11 > 11 (not true)
⇒ (125/100) × 7700 = Rs. 9625 (c) 7 ÷ 2 × 5 - 5 + 2 = 7
60th-62th BPSC (Pre) Exam. 2016-17 197 YCT
on interchanging the sign, 90
Ans. (c) Expenditure for labour = 3000000 ×
7 – 2 + 5 ÷5 × 2 > 7 120
7–2+2>7 = ` 2250000
7 > 7 (not true) Expenditure for labour increased by 5%
(d) 5 × 6 – 3 ÷ 3 + 1 > 4 New increased expenditure of labour
On interchanging the sign,
105
5+6÷3–3×1=4 = 2250000 ×
100
5+2–3=4
= ` 2362500
4=4
Increases amount of labour from old to new expenditure
Hence option (d) is true.
= 2363500 – 2250000
28. How many times the digit 9 occurs in the
= ` 112500
numbers from 100 to 999?
(a) 280 70
other expenditure = 3000000 ×
(b) 218 120
(c) 229 = ` 1750000
(d) 228 Expenditure on other expenses can be reduced
(e) None of the above/More than one of the 112500
above = × 100 = 6.43%
1750000
Ans. (a) In the range of 100 to 200, ∴ 6.43% expenditure on other expenses be reduced to
There are 20 9's maintain the profit.
109, 119, 129, 139, 149, 159, 169, 179, 189, 190, 191,
30. In the bottom of a water tank, there are two
192, 193, 194, 195, 196, 197, 198, 199.
drains A and B. If only A is open, it takes 30
Similarly from range of 100 to 899 minutes to empty a full tank and if only B is
Number of 9's = 20 × 8 = 160 open, it takes 20 minutes. If for 10 minutes
In the range of 900 to 999 both drains are open, then B is closed, how
Number of 9's = 100 + 10 + 10 = 120 much time it takes to empty a full tank?
Total number of 9's occur = 160 + 120 (a) 18 minutes
= 280 (b) 15 minutes
∴ 280 times the digits 9 occurs in the numbers from (c) 17 minutes
100 to 999. (d) 20 minutes
29. In the following pie-chart, the various (e) None of the above/More than one of the
expenditures of an industry are presented. above
Total expenditure on raw material is ` 30 lac.
Ans. (b) Let us take LCM for (30, 20) = 60
If labour expenditure is increased by 5% How
Total work = 60 unit
much expenditure on other expenses be
reduced to maintain the profit? 60
Efficiency of A = unit
30
60
Efficiency of B = unit
20
Efficiency of both A and B = 2 + 3 = 5 unit
Total work = Efficiency × Time taken to complete the
work
A and B does work for 10 min
work done for 10 min = 5 × 10 = 50 unit
Remaining work = 60 – 50 = 10 unit
The remaining 10 unit work is done by A alone so,
Total work = Efficiency × Time taken to complete the
(a) 5.9%
work
(b) 12.86%
10
(c) 6.43% Time taken by A = = 5 mins
(d) 6.21% 2
(e) None of the above/More than one of the Total time = 10 + 5 = 15 mins
above ∴ 15 mins to empty a full tank.

60th-62th BPSC (Pre) Exam. 2016-17 198 YCT


31. Which of the following pairs is not correctly (c) Cyanocobalamin-Anaemia
matched? (d) Ergocalciferol- Rickets
(a) Vitamin B1- Oranges (e) None of the above/More than one of the
(b) Vitamin D- Cod-liver oil above
(c) Vitamin E-Wheat germ oil Ans. (b) : Vitamin E is also known as Tocopherol.
(d) Vitamin K- Alfalfa Vitamin E deficiency causes mild hemolytic anemia and
(e) None of the above/More than one of the nonspecific neurologic deficits. Whereas Beriberi is
above caused by the deficiency of Vitamin B1 or Thiamene.
Ans. (e) : Oranges are a rich source of Vitamin C, in Rest all pairs are correctly matched.
addition to Vitamin C, oranges provide smaller amount 36. Bakelite is formed by the condensation of
of other nutrients, including calcium, potassium and (a) Urea and formaldehyde
Vitamin B1 (Thiamine). Alfalfa contains Vitamin K, (b) Phenol and formaldehyde
Cod liver Oil contains Vitamin D and Wheat germ Oil (c) Phenol and acetaldehyde
is the richest natural source of Vitamin E. Hence all the (d) Melamine and formaldehyde
above given pairs are correctly matched. (e) None of the above/More than one of the
32. Teflon is a polymer of which of the following above
monomers? Ans. (b) : Bakelite is a polymer made up of the
(a) Tetrafluoroethylene monomers phenol and formaldehyde. It is the worlds'
(b) Vinyl chloride first man-made polymer. It was invented by Leo
(c) Chloroprene Hendrik Baekeland.
(d) Acetylene dichloride 37. Which one of the following is an antibiotic
(e) None of the above/More than one of the drug?
above (a) Quinine
Ans. (a) : Teflon is a polymer of tetrafluoroethylene. It (b) Sulphaguanidine
is an excellent plastic material for films and coatings. (c) Chloramphenicol
33. Which of the following is responsible for (d) Aspirin
controlling hereditary characters of the living (e) None of the above/More than one of the
cells? above
(a) Enzyme Ans. (e) : Chloramphenicol is an antibiotic. It is useful
(b) Hormone for the treatment of a number of bacterial infection.
(c) RNA Sulphagunidine is also an antibiotics, it is used for the
(d) DNA treatment of Bacillary dysentery. Hence option (e) will
be correct. However BPSC had considered option (c) as
(e) None of the above/More than one of the
correct.
above
38. Match List-I with List-II and select the correct
Ans. (d) : DNA is responsible for controlling the
ans using the codes given below the lists:
hereditary character of living cells. DNA is the
List-I List-II
hereditary material found in almost in living organisms.
(Applications) (Substances)
It in made up of molecules called nucleotides.
A. Aspartame 1. Synthetic rubber
34. Which of the following vitamins is known as B. Freon 2. Anti-histamine
ergocalciferol? C. Neoprene 3. Artificial Sweetener
(a) Vitamin D2 D. Benadryl 4. Refrigerant
(b) Vitamin D3 (a) A-1, B-3, C-4, D-2
(c) Vitamin B12 (b) A-2, B-4, C-1, D-3
(d) Vitamin B6 (c) A-3, B-4, C-1, D-2
(e) None of the above/More than one of the (d) A-3, B-1, C-2, D-4
above (e) None of the above/More than one of the
Ans. (a) : Vitamin D2 is also known Ergocalciferol. It is above
used to treat hypoparathyroidism and rickets. Ans. (c) : The correct matches are as :-
Ergocalciferol is obtained from plant source or dietary List - I List - II
supplements derived from irradiated fungi. (Applications) (Substances)
35. Which of the following pairs is not correctly Aspartame Artificial Sweetener
matched? Freon Refrigerant
(a) Retinol - Xerophthalmia Neoprene Syntheic rubber
(b) Tocopherol -Beriberi Benadryl Anti histamine

60th-62th BPSC (Pre) Exam. 2016-17 199 YCT


39. Insulin is 44. Match List-I with List-II and select the correct
(a) Fat (b) Vitamin Ans using the codes given below the lists:
(c) Carbohydrate (d) Protein List-I List-II
(e) None of the above/More than one of the A. Skin Cancer 1. Chlorofluorocarbons
above B. Noise Pollution 2. Ultraviolet light
Ans. (d) : Insuline is a protein. It is a peptide hormone C. Global Warming 3. Decibel
produced by beta cells of the pancreatic islets. There are D. Ozone Hole 4.Carbon dioxide
51 amino acid in an insuline molecule. (a) A-1, B-2, C-3, D-4
40. The gas used in the manufacture of Vanaspati (b) A-2, B-3, C-4, D-1
Ghee from Vanaspati Oil is (c) A-2, B-3, C-1, D-4
(a) Hydrogen
(d) A-4, B-3, C-2, D-1
(b) Oxygen
(c) Nitrogen (e) None of the above/More than one of the
(d) Carbon dioxide above
(e) None of the above/More than one of the Ans. (b) : The correct matches are as :-
above List - I List - II
Ans. (a) : Hydrogen (H2) gas is used in the manufacture Skin cancer Ultraviolet light
of Vanaspati Ghee from Vegitable Oil. This process is Noise pollution Decibel
called 'hydrogenation'. At high pressure, in the presence Global warming Carbon dioxide
of nickel catalyst, hydrogen is mixed with vegetable oils Ozone Hole Chlorofluorocarbons
which turn them into Vegetable ghee. 45. In human body, Vitamin A is stored in
41. LPG used as domestic fuel chiefly contains (a) Liver
(a) Methane (b) Skin
(b) Propane (c) Lung
(c) Ethylene (d) Kidney
(d) Butane (e) None of the above/More than one of the
(e) None of the above/More than one of the above
above Ans. (a) : Most of the human body's Vitamin A is
Ans. (d) : LPG (Liquefied Petroleum Gas) used as stored in the liver. Vitamin A is fat soluble Vitamin. It
domestic fuel chiefly composed of propane and butane, is also known as retinol.
it also contain propane. LPG is an odourless and
colourless gas. Ethyl Mercaptan is the odurance added 46. Which of the following vitamins is used as an
to make LPG and natural gas smell. antidote to anticoagulant poisons?
42. Soap removes grease by (a) Vitamin A
(a) Coagulation (b) Vitamin D
(b) Adsorption (c) Vitamin E
(c) Emulsification (d) Vitamin K
(d) Osmosis (e) None of the above/More than one of the
(e) None of the above/More than one of the above
above
Ans. (d) : Vitamin K is used as an antidote to
Ans. (c) :Soap removes grease by the process of anticoagulant poisons. Vitamin K facilitaes in blood
emulsification. Emulsification is defined as the process
clotting.
of dispersing one immiscible liquid in another
immiscible liquid. 47. Iodised salt contains
43. Which of the following hormones play a role in (a) Free iodine
release of milk from mammary glands? (b) Calcium iodide
(a) Adrenaline (b) Thyroxine (c) Magnesium Iodide
(c) Progesterone (d) Oxytocin (d) Potassium Iodide
(e) None of the above/More than one of the (e) None of the above/More than one of the
above above
Ans. (d) :Oxytocin hormone plays a role in release of
Ans. (d) : Iodised salt contains potassium iodide or
milk from mammary glands. Oxytocin hormone is also
known as love hormone. It is secreted by the pituitary Sodium iodide. Potassium iodide is added to iodised salt
glands of mammals during childbirth, sex, lactation or to prevent iodine deficiency and associated thyroid
social bonding. disease.

60th-62th BPSC (Pre) Exam. 2016-17 200 YCT


48. When sulfur dioxide is released into the air it 52. The Iran-Pakistan Gas pipeline is also called as
reacts with water and oxygen to form sulfuric (a) Friendship pipeline
acid, H2SO4. Sulfuric acid is a strong acid. (b) Future pipeline
Which of the following is present in maximum (c) Peace pipeline
in acid rain? (d) Unity pipeline
(a) HCI (e) None of the above/More than one of the
(b) H2NO3 above
(c) H2SO4 Ans. (c) :Iran - Pakistan gas pipeline is also known as
(d) H2CO3 the peace pipeline. It is an under construction 2775 Km
(e) None of the above/More than one of the pipeline to deliver natural gas from Iran to Pakistan.
above 53. What is the India's rank in the 2016
Ans. (c) :Sulphuric Acid (H2SO4) is present in Sustainable Development Goal Index?
maximum amount when sulfur dioxide is released into (a) 110th
the air it reacts with water and oxygen to form sulfuric (b) 88th
acid, H2SO4. Sulfuric acid is a strong acid. 'Acid Rain'. (c) 63rd
H2SO4 represents about 60% of all acid rain with HNO3
(d) 129th
at about 25% and HCl 15%.
(e) None of the above/More than one of the
Acid rain is caused by a chemical reaction that begins
above
when compounds like sulfur dioxide and nitrogen
oxides are released into the air. Ans. (a) : India had been ranked at 110th position in the
Sustainable Development Goal Index 2016.
49. 'God Particle' is
The 7th edition of Sustainable Development Goal index
(a) Neutrino
2.22 ranked India at 121th out of 163 countries with a
(b) Higgs Boson country score of 60.32.
(c) Meson
54. The United Nations Framework Convention on
(d) Positron
Climate Change deals with
(e) None of the above/More than one of the
(a) Reduction in fossil fuel usage
above
(b) SO2 emissions mitigation
Ans. (b) : Higgs Boson is also known as God Particle. (c) reduction in Uranium production
These particles are the fundamental particles that
(d) Greenhouse gases emission mitigation
provide mass to other particles. It is the elementary
(e) None of the above/More than one of the
particle that decays quickly. It is found in Higgs field.
above
50. Which of the following radio isotopes is used in
Ans. (d) :United Nations Framework Convention on
the treatment of blood cancer [leukemia]?
Climate Change (UNFCCC) came into force on 21st
(a) Iodine-131
March 1994. It is primary multilateral treaty governing
(b) Sodium-24 actions to combat climate change through adoption and
(c) Phosphorus-32 mitigation efforts directed at control of emission of
(d) Cobalt-60 Green House Gases that cause global warming.
(e) None of the above/More than one of the 55. The 2016 Nobel Prize in Literature was won by
above
(a) Alice Munro
Ans. (e) : Cobalt 60 and Phosphorus 32 are radio (b) Svetlana Alexievich
isotopes. These both are used to treat blood cancer (c) Bob Dylan
called leukemia.
(d) Angus Deaton
51. The elected US President Donald Trump was (e) None of the above/More than one of the
the Presidential Nominee of the above
(a) Democratic Party
Ans. (c) :The Nobel Prize in Literature 2016 was
(b) Republican party awarded to Bob Dylan for having created new poetic
(c) Libertarian Party expressions within the great American song tradition.
(d) Green party The Nobel Prize in Literature for 2021 is awarded to
(e) None of the above/More than one of the Abdulrazak Gurnah.
above
56. Which country has the largest reserves of oil?
Ans. (b) : The 45th President of United States of (a) United States (b) China
America was Donald Trump. He was the presidential (c) Russia (d) Venezula
Nominee of Republican Party. He had defeated the
(e) None of the above/More than one of the
Democratic Presidential Candidate Hillary Clinton.
above
60th-62th BPSC (Pre) Exam. 2016-17 201 YCT
Ans. (d) :Venezuela has the largest oil reserves in the 61. The head of state of the United Kingdom is
world with more than 300 billion barrels in reserve (a) Queen Elizabeth I
followed by Saudi Arabi'a, Canada and Iran. (b) Queen Elizabeth II
57. Who is the new Secretary General of the (c) Queen Elizabeth III
United Nations? (d) Queen Elizabeth IV
(a) Matthew Rycroft (e) None of the above/More than one of the
(b) Anibal Cavaco Silva above
(c) Antonio Guterres Ans.(b):Queen Elizabeth II has been the UK's Head of
(d) Vitaly Churkin the state since 1952 when her father king George VI died.
(e) None of the above/More than one of the 62. India's first and Asia's longest cycle highway
above has been opened in which State of India?
Ans. (c) : Antonio Guterres is the 9th Secretary General (a) Uttar Pradesh
of United Nations. He took office on 1st January 2017. (b) Andhra Pradesh
58. Which of the following countries has recently (c) Bihar
joined the Hague Code of Conduct [HCOC], a (d) Assam
global initiative to prevent ballistic missile (e) None of the above/More than one of the
proliferation? above
(a) Palestine Ans. (a) :India's first and Asia longest cycle highway
(b) Israel has been opened in Uttar Pradesh. It has a length of 207
(c) India km long cycle track and it run between Etawah and
(d) Iran Agra in U.P.
(e) None of the above/More than one of the 63. India's first textile university will be set up in
above which of the following states?
Ans. (c) :Hague Code of Conduct (HCOC) is a political (a) Madhya Pradesh (b) Bihar
initiative aimed at globally curbing ballistic missile (c) Gujarat (d) Odisha
proliferation globally. The original HCOC was signed (e) None of the above/More than one of the
by 9 countries including the US, UK, France, Russia, above
Germany and China. The 9th Country signed, later on, Ans. (c) : India's first textile University will be setup in
in 2016 was India. Surat, Gujarat.
59. Which State has been declared India's first 64. What is the fiscal deficit target of India for the
'Open Defecation Free State'?
financial years 2016-17?
(a) Bihar
(a) 3.9% of GDP
(b) Assam
(b) 3.5% of GDP
(c) Hinachal Pradesh
(c) 4.9% of GDP
(d) Sikkim
(d) 4.5% of GDP
(e) None of the above/More than one of the
(e) None of the above/More than one of the
above
above
Ans. (d) : On 27th May 2016, Sikkim had been declared
Ans. (b) : The fiscal deficit target of India for financial
India first 'Open Defecation Free State'.
years 2016 - 17 was 3.5% of GDP. The Fiscal deficit in
60. Which of the following is the newest member of 2022 - 23 is estimated at 6.4% of GDP.
the Eurozone?
(a) Lithuania 65. The 2016 joint military exercise "Maitree" has
(b) Croatia been conducted between India and which of the
following countries?
(c) Bulgaria
(a) Indonesia
(d) Cyprus
(b) Thailand
(e) None of the above/More than one of the
above (c) Malaysia
(d) Maldives
Ans. (a) : Lithuania is the newest member of Eurozone.
(e) None of the above/More than one of the
Euro zone consist of 19 members of European Union,
which uses the Euro as their official currency. 8 EU above
members (Bulgaria, Poland, Hungry, Romania, Sweden, Ans. (b) : From 15 - 30 July 2016, The joint military
(Zech Republic, Demark and Croatia) do not use the exercise " Maitree" had been conducted between Indian
Euro. Croatia is set to become 20th member of euro zone Army and Royal Thai Army at Krabi, Thailand. It is
on 1st January 2023. also called Indo - Thailand Joint Exercise.
60th-62th BPSC (Pre) Exam. 2016-17 202 YCT
66. The nation-wise breastfeeding programme Ans. (d) : In 2016, Union Government and Reserve
"MAA" has been launched by which Union Bank of India had formed Neeraj Kumar Gupta
Ministry? Committee to enhance cashless transaction. The main
(a) Ministry of Tribal Affairs aim of the committee is to develop a card acceptance
(b) Ministry of Health and Family Welfare Infrastructure and fix interchange fees for credit cards.
(c) Ministry of Women and Child Development 70. Which country has won the 2016 Davis Cup
Tennis Tournament?
(d) Ministry of AYUSH
(a) Argentina
(e) None of the above/More than one of the
(b) Switzerland
above
(c) Serbia
Ans. (b) :MAA - Mother's Absolute Affection is an (d) Croatia
intensified Programme with an attempt to bring (e) None of the above/More than one of the
undiluted focus on promotion of breastfeeding, in above
addition to on going efforts through health system. It is Ans. (a) : Argentina had won the 2016 Davis Cup title
a nationwide programme of the Ministry of Health and by defeating Croatia in the final at Zagreb on 25 - 27
Family Welfare. November 2017. The Davis Cup 2021 was won by the
67. Which of the following is a bond through which Russian Tennis Federation.
Indian entities can raise money from foreign 71. India's first Laser Interferometer
markets in rupees, and not in foreign Gravitational-Wave Observatory (LIGO)
currency? laboratory will be set up in which State?
(a) Corporate Bonds (a) Uttar Pradesh
(b) Masala Bonds (b) Maharashtra
(c) Municipal Bonds (c) Andhra Pradesh
(d) Zero-coupon Bonds (d) Bihar
(e) None of the above/More than one of the (e) None of the above/More than one of the
above
above
Ans. (b) : Laser Interferometer Gravitational Wave
Ans. (b) : Indian entilies can raise money from foreign
Observatory (LIGO) is a large scale physics experiment
market in rupees, and not in foreign currency through carried out to detect Gravitational waves.
masala bonds. Masala bonds are rupee denominated
India's LIGO will be set up at Aundha Nagnath in the
bonds. It is a debt instrument issued by an Indian entity Hingoli district in Maharashtra.
in foreign markets to raise money in Indian currency
72. In Rio Olympics 2016 which country finished
instead of dollars or local denomination.
at the top of the table with gold medals?
68. Which train safety system has recently been (a) United States
launched by the Indian Railways to avoid train (b) China
accidents? (c) Great Britain
(a) Tri-Safety (d) Russia
(b) Tri-Netra (e) None of the above/More than one of the
(c) Tri-Network above
(d) Tri-Veil Ans. (a) : The 2016 Summer Olympics Game was held
(e) None of the above/More than one of the from 5 to 21 August 2016 in Rio de Janerio, Brazil.
above United States of America topped the medal table,
Ans. (b) : Indian Railway had introduced TRI - NETRA winning the most gold medals (46) and highest number
(terrain imaging for diesel drivers infrared, enhanced of medals overall (121).
optical and radar assisted) system on its trains to reduce 73. The FIFA World Cup 2022 will be held in
trains accidents. (a) Argentina
69. Which committee has been constituted by the (b) Germany
Government of India to boost cashless (c) Qatar
transaction? (d) Russia
(a) Nachiket More Committee (e) None of the above/More than one of the
(b) Shanta Kumar Committee above
(c) H.R. Khan Committee Ans. (c) : The 2022 FIFA world Cup is scheduled to
(d) Neeraj Kumar Gupta Committee take place in Qatar from 21 November to 18 December
(e) None of the above/More than one of the 2022. The 2026 FIFA world Cup will be jointly hosted
above by Canada, Mexico & United States.
60th-62th BPSC (Pre) Exam. 2016-17 203 YCT
74. As per Census 2011, what is the rank of Bihar Ans. (b) : Six Machine : I don't like cricket ----I love it
State in terms of population in the country? is the autobiography of Chris Gayle.
(a) I 79. Who is the first Indian to win two Paralympic
(b) II Gold Medals?
(c) III (a) Devendra Jhajharia
(d) IV (b) Deepa Malik
(e) None of the above/More than one of the (c) Mariyappan Thangavelu
above (d) Varun Singh Bhati
Ans. (c) : As per Census 2011, Bihar is the third most (e) None of the above/More than one of the
populous state of India with a total population of above
104,099,452 (54,278,157 Male and 49,821,295 Female) Ans. (a) : Devendra Jhajharia is the first Indian
75. Which country has won the 2016 Kabaddi Paralympics player to win two gold medals at
World Cup? paralympics. He had won first paralympics gold medal
(a) Srilanka at the 2004 Summer Paralympics in Athens and second
(b) Iran at the 2016 Summer Paralympics in Riode Janerio.
(c) India 80. What was the theme of the 2016 Rio Olympics?
(d) China (a) Peace for inner soul
(e) None of the above/More than one of the (b) World peace and Environment
above (c) Nothing can be achieved without peace
Ans. (c) : The 2016 Kabaddi was held fron 7 to 22 (d) Live sports, live freedom
October 2016 in Ahmedabad, India. India had won this (e) None of the above/More than one of the
tournament by defeating Iran 38-29 in the Final. above
76. Which country's women Badminton team has Ans. (b) : The 2016 Olympics was held from 5 to 21
won the 2016 Uber Cup Tournament? August 2016 in Rio de Janerio, Brazil. The theme of
(a) Thailand 2016 Rio Olympics was "World Peace and
(b) India Environment."
(c) South Korea 81. Which among the following Harappan site
(d) China terracotta of 'plough' was found?
(e) None of the above/More than one of the (a) Dholavira
above (b) Banawali
Ans. (d) : The 2016 Uber Cup had been won by China (c) Kalibangan
by defeating South Korea in the final. (d) Lothal
77. Who has been conferred with the 2015 (e) None of the above/More than one of the
Jyanpith Award? above
(a) Pannalal Patel Ans. (b) : The terracotta of 'Plough' was found in the
(b) Umashankar Joshi Harappan site of Banawali (Haryana). Archaeologist
(c) Raghuveer Chaudhari have also found evidence of a ploughed field at
(d) Rama jain Kalibangan (Rajasthan).
(e) None of the above/More than one of the 82. Kalinga's King Kharvela was associated with:
above (a) Mahameghvahana dynasty
Ans. (c) : Gujarati novelist, poet and critic Raghuveer (b) Chedi dynasty
Chaudhari has been awarded 2015 Jnanpith Award. The (c) Satvahana Dynasty
most recent Jnanpith Award winners in 2021 and 2022 (d) Rath-Bhojak dynasty
were Assamese poet Nilmani Phookan Jr. and a (e) None of the above/More than one of the
Konkani novelist Damodar Mauzo respectively. above
78. The book Six Machine : I Don't Like Cricket Ans. (e) : Kharvela was a king of Kalinga. He was the
…. I Love It is the autobiography of which ruler of Chedi Dynasty also known as
cricketer? Mahameghavahana dynasty. He ruled in Kalinga after
(a) Yuvraj Singh the decadence of the Mauryas. He was the third ruler of
(b) Chris Gayle Chedi dynasty.
(c) Virendra Sehwag 83. Chinese traveler 'Sung Yun' was came to India
(d) AB de Villiers in:
(e) None of the above/More than one of the (a) 515 AD to 520 AD
above (b) 525 AD to 529 AD
60th-62th BPSC (Pre) Exam. 2016-17 204 YCT
(c) 545 AD to 552 AD 88. Which among the following King's reign
(d) 592 AD to 597 AD Persian traveller 'Abaur Razzaq' came to
(e) None of the above/More than one of the India?
above (a) Dev Raya I
Ans. (a) : Chinese traveler, Sung Yun came to India in (b) Krishna Dev Raya I
from 515 AD to 520 AD. (c) Dev Raya II
84. Which type of land was called 'Aprahat'? (d) Krishna Raya II
(a) Without cultivated forest land (e) None of the above/More than one of the
above
(b) Irrigated land
(c) Dense forest land Ans. (c): Persian traveller Abdur Razzaq came to India
during the reign of Deva Raya II. Devaraya II acquired
(d) Cultivated land
the title of Gajabetegara. He extended and held
(e) None of the above/More than one of the territories upto the Krishna River.
above
89. 'Jaswant and Dasawan' famous painters were
Ans. (a): The land of Gupta period can be classified courtier of Mughal emperor.
into following categories :- (a) Akbar (b) Jehangir
Kshetra :- Cultivated land (c) Shahjahan (d) Aurangzeb
Khila :- Waste land (e) None of the above/More than one of the
Aprahat :- Without cultivated jungle or forest. above
Vasti :- Habitable land.
Ans. (a): Mughal Emperor Akbar had organised
Gopata Sarah :- Pasture land.
painting in imperial Karkhans and also introduced
85. Which among the following state 'Odantipur' European style. He invited a large number of painters
Education Centre was situated? from different part of the country to his court. Jamshed
(a) Bengal Basawan, Dasawan (painted Razmnama, the Persian
(b) Bihar Mahabharata), Mir Sayyid Ali, Abdul Samad, Farukh
(c) Gujarat Beg, Bhusraukuli, and Miskina were the prominent
(d) Tamil Nadu painters of his court.
(e) None of the above/More than one of the 90. Akhil Bhartiya Kisan Congress was founded
above
in:
Ans. (b) : The ancient three universities of India, (a) 1936 AD
Nalanda University (Nalanda District), Odantpuri
(b) 1939 AD
University (Bihar Sarif District) and Vikramshila
University (Bhagalpur District) were situated in Bihar. (c) 1942 AD
86. 'Diwan-E-Arz' department was associated with: (d) 1945 AD
(a) Royal Correspondence (e) None of the above/More than one of the
(b) Foreign above
(c) Defence Ans. (a): Akhil Bhartiya Kisan Sabha (also known as
(d) Finance All India Kisan Sabha) was founded in 1936 at Indian
(e) None of the above/More than one of the National Congress. Lucknow session of the All India
above Kisan Sabha held in 1936 under the presidentship of
Swami, Sahajanand Saraswati.
Ans. (c): Diwan-E-Arz was established by Balban. It
was the department of military managed by Ariz - i - 91. Which among the following year 'Santhal
Mumalik. revolt' was occurred?
87. Who was the founder of Bahamani Kingdom? (a) 1831-32 AD
(a) Alauddin Hasan (b) 1844-46 AD
(b) Feroz Shah (c) 1851-52 AD
(c) Mahmud Gaonwa (d) 1855-56 AD
(d) Asaf Khan (e) None of the above/More than one of the
(e) None of the above/More than one of the above
above Ans. (d): Santhal Rebellion took place in 1855 - 56.
Ans. (a): Alauddin Hasan Bahman Shah was the This revolt was led by four Brothers Sidhu, Kanu,
founder of the Bahmani Sultanate. He began his carrer Chand and Bhairav, against the oppressive Zamidari
as a general serving under the Sultan Muhammad bin System setup by British East India Company. The
Tughlaq. His original name was Hasan Gangu. rebellion was suppressed by 1856.

60th-62th BPSC (Pre) Exam. 2016-17 205 YCT


92. Which among the following year Surendra Ans. (c): Bakhtiyar Khalji was the first Muslim
Nath Banerjee was eliminated from Indian conqueror of Bihar. He was a Turk Afghan military
Civil Services? general who was the leader of the Muslim conquests of
(a) 1874 AD Bihar and Bengal region and later on established him
(b) 1877 AD self as the ruler of this region.
(c) 1885 AD 97. Who was the last King of Karnata dynasty?
(d) 1892 AD (a) Harisimha
(e) None of the above/More than one of the (b) Ramsimha
above
(c) Matisimha
Ans. (a): Surendranath Banerjee was Indian nationalist
(d) Shyamsimha
during the British rule. He was the second Indian after
Satyendranath Tagore to qualify Indian Civil Service (e) None of the above/More than one of the
examination. In 1874, he was dismissed from his above
service for a minor and apparently inadvertent Ans. (a): Karnata dynasty was founded by Nanyadeva.
procedural error. Harisimhadeva was the last ruler of this dynasty. He
93. Who is known as father of Dyarchy? rulled from 1304 to 1325.
(a) Lord Clive 98. The author of 'Rajaniti Ratnakara' is
(b) Hector Munaro (a) Chandesvara
(c) Lord Macaulay (b) Vidyapati
(d) Sir Lionel Curtis (c) Jyotiresvara
(e) None of the above/More than one of the (d) Haribrahmadeva
above (e) None of the above/More than one of the
Ans. (d): Sir Lionel Curtis is known as the father of the above
Dyarchy. Dyarchy was a system of double government, Ans. (a): 'Rajniti Ratnakara' was published by Bihar and
introduced by Government of India Act 1919 for the Orissa research society in 1924. The author of this book
provinces of British India. is Chandesvara Thakur.
94. Which among the following viceroy period the
99. The author of 'Udwant Prakash' is
title of 'Rai Bahadur' and 'Khan Bahadur'
(a) Mauli Kavi
were to conferred to Indian?
(a) Lord Ripon (b) Bodhraj
(b) Lord Lytton (c) Paramala
(c) Lord Meyo (d) Vidyapati
(d) Lord Dufferin (e) None of the above/More than one of the
(e) None of the above/More than one of the above
above Ans. (a): Udwant Prakash is written by Mauli Kavi in
Ans. (b): The title of Rai Bahadur and Khan Bahadur 1747. This book has the history of Ujjainiya Rajput of
began to conferred to Indians during the reign of lord Jagdishpur, Bihar.
Lytton. He was the viceroy of India from 1876 to 1880. 100. Nalanda Vihar was destroyed by :
95. Who was the founder of Karnata dynasty? (a) Bakhtiyar Khalji
(a) Nanyadev (b) Qutubddin Aibak
(b) Narsimhadev (c) Muhammad Bin Tughlaq
(c) Vijaydev (d) Alauddin Khalji
(d) Haridev (e) None of the above/More than one of the
(e) None of the above/More than one of the above
above Ans. (a): Bakhtiyar Khalji had destroyed Vikramshila
Ans. (a): Karnata Dynasty was a Maithil Dynasty and the Nalanda University of Bihar. Nalanda
established by Nanyadeva in 1097 CE. This dynasty University was completely distroyed and it had millions
ruled in Tirhut or Mithila in Bihar. of books that were burned.
96. Who was the first Muslim Conqueror of Bihar? 101. The Great Victorian Desert is located at
(a) Malik Ibrahim (a) Australia
(b) Iltumish (b) India
(c) Bakhtiyar Khalji (c) Egypt
(d) Ali Mardan Khalji (d) North Africa
(e) None of the above/More than one of the (e) None of the above/More than one of the
above above
60th-62th BPSC (Pre) Exam. 2016-17 206 YCT
Ans. (a): The Great Victorian desert is the largest desertAns. (d): The correct matches are as :-
in Australia and 7th largest desert in the world. Place Industry
102. Which of the following activities is performed Bengaluru Aircraft
at Coolgardie? Korba Aluminium
(a) Coal mining Jamshedpur Iron & Steel
(b) Copper mining Malajkhand Copper
(c) Gold mining
106. Which of the following seas has the highest
(d) Forestry
average salinity?
(e) None of the above/More than one of the
(a) Black Sea
above
(b) Yellow Sea
Ans. (c): Coolgardie is famous for gold mines. It is
located in Australia. Coolgardie is the birthplace of the (c) Mediterranean Sea
great gold rush of 1892. (d) Dead Sea
103. What is Particulate matter? (e) None of the above/More than one of the
(a) Solid residue above
(b) Air pollutant Ans. (d): Dead Sea has the highest average salinity, the
(c) Water pollutant dead sea seawater has a density of 1.240 kg/L , which
(d) Soli pollutant makes swimming in its waters similar to floating. It is
(e) None of the above/More than one of the the deepest hyper saline lake in the world.
above 107. The Dravadian races are mainly confined at
Ans. (b): Particulate Matter is air pollutant. It is made which of the following parts of India?
up of tiny pieces (particles) of solid or liquid that are (a) South India
suspended in the air. (b) North-Western India
104. What is the name of mid latitude grass land in (c) North-Eastern India
South America? (d) North India
(a) Prairie (e) None of the above/More than one of the
(b) Pampas above
(c) Veld Ans. (a): Dravidian races are mainly confined at South
(d) Steppes India. Dravidians peoples migrated from Africa and
reached South India via southern route. Approx 28% of
(e) None of the above/More than one of the Indian are Dravidians and reside in S. India with one of
above the Dravidian languages as their main language, which
Ans. (b): Pampas is mid latitude grassland in South include, Tamil, Malayalam, Telugu, Kannada and Tulu.
America. The grasslands are found in the middle 108. At which of the following hills the Eastern
latitudes of S.America, N. America, Africa and Asia. In Ghats join the Western Ghats?
N. America, they are known as Prairies, in S. America (a) Palani Hills
they are known as Pampas. Central Eurasian grasslands (b) Anaimudi Hills
are refered to as steppes. (c) Nilgiri Hills
105. Match the following palces with industries and (d) Shervoroy Hills
select the correct answer using the code given (e) None of the above/More than one of the
below: above
Place Industry
Ans. (c): Eastern Ghats and Western Ghats meets at the
A. Bengaluru 1. Iron & Steel Nilgiri hills in Tamil Nadu. Some part of the Nilgiri
B. Korba 2. Copper hills lie in Tamil Nadu, Karnataka and Kerala as well.
C. Jamshedpur 3. Aircraft 109. At which of the following places the Copper
D. Malajkhand 4. Aluminium Industry is located?
(a) A-1, B-2, C-3, D-4 (a) Tarapur
(b) A-2, B-1, C-4, D-3 (b) Titagarh
(c) A-4, B-3, C-2, D-1 (c) Ranchi
(d) A-3, B-4, C-1, D-2 (d) Khetri
(e) None of the above/More than one of the (e) None of the above/More than one of the
above above
60th-62th BPSC (Pre) Exam. 2016-17 207 YCT
Ans. (d): Khetri is a town in JhunJhunu district of prayag. Pindari river Joins the Alaknand river at
Rajasthan. It is famous for its copper mines. Other Karnaprayag.
major copper mines in India are :- Singhbhum copper Dhauli Ganga Joins Alaknanda at Vishnuprayag.
belt in Jharkhand and Malanjkhand copper belt in 114. Which of the landforms occupy the North-
Madhya Pradesh. Western parts of Bihar State?
110. What is the Golden Quadrilateral? (a) Someshwar Hilly Range
(a) Rail lines joining metros (b) Kaimur Plateau
(b) Major Air Routes (c) Nawada Hilly Region
(c) National Highway Project (d) Rajgir Hilly Region
(d) Gold Trade Routes (e) None of the above/More than one of the
(e) None of the above/More than one of the above
above Ans. (a): Someshwar hilly range landforms occupy the
Ans. (c): The Golden Quadilateral project was started in north-western parts of Bihar State.
2001 by the then Prime Minister Atal Bihari Vajpayee.
115. River Tel is a tributary of which of the
It is a national highway network that connects the
following rivers?
majority of India's main economic, agricultural and
(a) Bagmati
cultural centers from east to west and north to south.
(b) Ghaghara
111. Which of the following Iron ores is mined at
(c) Gandak
Bailadila?
(a) Haematite (d) Kamla
(b) Siderite (e) None of the above/More than one of the
above
(c) Limonite
(d) Magnetite Ans. (e): Tel river originates in the Nabarangpur district
(e) None of the above/More than one of the of Orissa. It is an important tributary of Mahanadi river.
above 116. Which of the following canal systems irrigate
Ans. (a): Haematite iron ore is mined at Bailadila. areas of Bihar?
Bailadila is famous for producing iron which is used for (a) Upper Ganga Canal
steel making. (b) Triveni Canal
112. The only state which produces saffron in India (c) Sharda Canal
is (d) Eastern Yamuna Canal
(a) Himachal Pradesh (e) None of the above/More than one of the
(b) Assam above
(c) Jammu Kashmir Ans. (b): Triveni Canal system irrigate areas of Bihar.
(d) Meghalaya This canal system has been built on the river Gandak.
(e) None of the above/More than one of the 117. Which of the following is a silk textile
above producing centre in Bihar?
Ans. (c): At present Saffron is only produced in Jammu (a) Motipur
and Kashmir in India. Pampore region, in India, is (b) Bajari
commonly known as saffron bowl of Kashmir. It is the (c) Bhagalpur
main contributor to saffron production. (d) Dalmianagar
113. At which of the following towns the Alaknanda (e) None of the above/More than one of the
and the Bhagirathi combines to form River above
Ganga? Ans. (c): Bhagalpur district of Bihar is silk textile
(a) Haridwar producing centre in Bihar. Bhagalpur is the main center
(b) Rishikesh of tassar weaving.
(c) Rudraprayag 118. At which of the following places Bharat Wagon
(d) Devprayag and Engineering Co. Ltd. is situated?
(e) None of the above/More than one of the (a) Madhepura
above (b) Jamalpur
Ans. (d): Alaknanda and Bhagirathi rivers merge to (c) Harnat
form the holy river Ganga at Devprayag. (d) Mokama
Mandakini Joins the Alaknanda at Rudraprayag. (e) None of the above/More than one of the
Alaknanda river joins the Nandakini river at Nanda above
60th-62th BPSC (Pre) Exam. 2016-17 208 YCT
Ans. (d): Bharat Wagon and Engineering Co. Ltd. is Ans. (c): Vote on Account meant for appropriating
situated at Makoma, Bihar. It is a Public Sector funds pending passing of budget. It is a grant in advance
Undertaking (PSU) of Government of India. for the central government to meet the short term
119. Which of the following National Highways expenditure needs from the Consolidated Fund of India.
connect Chhapara and Gopalganj? 123. Constitution Day of India is celebrated on
(a) National Highway No. 77 (a) 26th October
(b) National Highway No. 84 (b) 26th November
(c) National Highway No. 85 (c) 26th January
(d) National Highway No. 80 (d) 15th August
(e) None of the above/More than one of the (e) None of the above/More than one of the
above above
Ans. (c): National Highway no. 85 old numbering is Ans. (b): Constitution Day also known as Samvidhan
about 95 km long highway that connects Chhapara and Divas. It is celebrated in our country on 26th November
Gopalganj districts of Bihar. every year to commemorate the adoption of the
120. Which of the following districts record highest Constitution of India.
growth of population during 2001-11? 124. Which officer of the government of India has
(a) Kishanganj the right to take part in the proceedings of
(b) Araria Parliament of India even though he is not a
(c) Madhepura member?
(d) Khagdia (a) Vice-President
(e) None of the above/More than one of the (b) Attorney General of India
above (c) Comptroller and Auditor General
Ans. (c): Among the above given options, Madhepura (d) Election Commissioner
district of Bihar record highest growth of population (e) None of the above/More than one of the
during 2001 - 11. above
121. Which of the following is/are listed among the Ans. (b): Article 76 of Indian Constitution is related to
Directive Principles in Part-IV of the Attorney General of India. He has right to Speak and to
Constitution of India? take part in the proceedings of both the Houses of
I. Equal Pay for Equal work Parliament or their joint sitting and any committee of
II. Uniform Civil Code the parliament of which he may be named a member,
III. Small family norm but without a right to vote.
IV. Education through mother tongue at 125. The Supreme Court has held that hoisting the
primary level National Flag a top the private buildings is a
(a) I, II and III fundamental right to every citizen under:
(b) I and II (a) Article 14 of the Constitution
(c) II and III (b) Article 19(1)(A) of the Constitution
(d) I, II and IV (c) Article 21 of the Constitution
(e) None of the above/More than one of the (d) Article 25 of the Constitution
above (e) None of the above/More than one of the
Ans. (b): Article 36-51 Under Part - IV of the Indian above
Constitution deal with the Directive Principles of State Ans. (b): Article 19(1)(a) of the Indian Constitution
Policy (DPSP). Indian Constitution recognised the
guarantees to all its citizens the right to freedom of
principle of Equal pay for Equal work for both men and
speech and expression. Under this Article, the Supreme
women through Article 39(d). Article 44 of Indian
Court has held that hoisting the National flag a top the
Constitution secure for all citizens a uniform civil code
throughout the country. private buildings is a fundamental right of every
citizens.
122. Vote on Account is meant for
(a) Vote on the report of CAG 126. Choose the fundamental rights available to
Indian Citizen but not to aliens:
(b) To meet unforeseen expenditure
I. Freedom of Speech and Expression
(c) Appropriating funds pending passing of
II. Equality before the Law
budget
III. Right of minorities
(d) Budget
IV. Protection of Life and Liberty
(e) None of the above/More than one of the
above (a) I and III (b) I and IV

60th-62th BPSC (Pre) Exam. 2016-17 209 YCT


(c) II and IV (d) II and III 130. Creation of a new state requires a majority for
(e) None of the above/More than one of the Constitutional Amendment.
above (a) Simple
Ans. (a): Under Indian Constitution, certain (b) Two-third
fundamental rights are available only to the citizens of (c) Three-fourth
India, They are :- Prohibition of discrimination on (d) Two-third plus ratification by half of all states
grounds of race, religion, caste, gender or place of birth (e) None of the above/More than one of the
(Article 15), Equality of opportunity in matter of public above
employment (Article 16), Freedom of speech and Ans. (a): Article 3 of Indian Constitution empowered
expression, assembly, association, movement, residence the Parliament to make law relating to the formation of
and profession (Article 19), Protection of the culture,
new states and alteration of existing states. A simple
language and script of minorities (Article 29), Right of
majority of the parliament is required for the creation of
minorities to establish and administer educational
new states.
institutions (Article 30).
131. The expression "Socialist" was introduced in
127. A Panchayat Samiti at the block level in India the Preamble of Indian Constitution by which
is only a/an amendment?
(a) Advisory Body (a) 42nd amendment
(b) Consultative Committee (b) 44th Amendment
(c) Co-ordinating and supervisory Authority (c) 46th Amendment
(d) Administrative Authority (d) 74th Amendment
(e) None of the above/More than one of the (e) None of the above/More than one of the
above above
Ans. (d): Panchayat Samiti at the block level in India is Ans. (a): The Preamble of India has been amended by
only an administrative authority. the 42nd Constitutional Amendment Act 1976, which
128. A seat of M.P. can be declared vacant if he added three new words, - Socialist, Secular and
absents himself from the house for a Integrity.
continuous period of 132. President of India exercises his powers
(a) Six months (a) either directly or through officers subordinate
(b) Two months to him
(c) Three months (b) through Ministers
(c) through Prime Minister
(d) One years
(d) through cabinet
(e) None of the above/More than one of the
(e) None of the above/More than one of the
above
above
Ans. (b): A House can declare the seat of a member
Ans. (a): Article 53 of Indian Constitution states that
vacant if he is absent from all its meetings for a period
the President of Indian can exercise his powers directly
of sixty days without permission.
or by subordinate authority, with few exceptions, all of
129. Match the following: the executive powers vested in the president are, in
List I List II practice, exercised by the Prime Minister with the help
I. Union List (a) 97 entries of Council of Minister.
II. State List (b) 47 entries 133. The "Ease of Doing Business Index" is
III. Concurrent List (c) 66 entries prepared and published by
(a) I-a, II-b, III-c (a) World Trade Organisation
(b) I-a, II-c, III-b (b) World Bank Group
(c) I-c, II-a, III-b (c) United Nations
(d) I-c, II-b, III-a (d) European Nations
(e) None of the above/More than one of the (e) None of the above/More than one of the
above above
Ans. (a): The correct matches are as :- Ans. (b): Ease of Doing Business was an index
List -I List - II published by the World Bank Groups. In September
Union List 97 entries (Now 99) 2021, the World Bank scrapped publication of Ease of
State List 66 entries (Now 61) Doing Business report after an external review of data
Concurrent List 47 entries (Now 52) irregularities in the 2018 and 2020.

60th-62th BPSC (Pre) Exam. 2016-17 210 YCT


134. Which one of the following measures is not (a) 21.54
likely to aid in improving India's Balance of (b) 17.64
Payment position? (c) 15.59
(a) Promotion of Import Substitution Policy
(d) 19.21
(b) Devaluation of rupee
(e) None of the above/More than one of the
(c) Imposition of higher tariff on imports above
(d) levying of higher duties on exports Ans. (b): The percentage of decadal growth of
(e) None of the above/More than one of the population during 2001 - 2011 as per census 2011 is
above 17.64%.
Ans. (d): Among the above given options, levying of 139. Which of the following is not a cash crop?
higher duties on exports is not likely to aid in improving
(a) Jute
India's Balance of Payment positions because it will
negatively impact exports. (b) Groundnut
(c) Jowar
135. Service tax was introduced in India on the
recommendation of: (d) Sugarcane
(a) Kelkar Committee (e) None of the above/More than one of the
above
(b) Raja J. Challiah Committee
(c) Manmohan Singh Committee Ans. (c): Jowar is not a Cash crop rest all the other are
examples of cash crop. A cash crop is an agricultural
(d) Yashwant Sinha Committee
crop which is grown for selling to make profit.
(e) None of the above/More than one of the
above 140. First five year plan of India was based on the
Ans. (b): The service tax was introduce in India on 1st (a) Harrod Domar Model
July 1994 on the recommendations of Dr. Raja J. (b) Mahalanobis Model
Chelliah Committee on tax reforms. (c) Dadabhai Naroji Model
136. As per Census 2011, which state has the lowest (d) J.L. Nehru Model
sex-ratio?
(e) None of the above/More than one of the
(a) Punjab above
(b) Haryana
Ans. (a): The first five year plan was based on the
(c) Uttar Pradesh
Harrod Domar Model. First five year plan was launched
(d) Bihar for the duration of 1951 to 1956. Its main focus was on
(e) None of the above/More than one of the the agricultural development of the country.
above
141. Black revolution is related to the
Ans. (b): As per census 2011, Haryana has the lowest
sex ratio in India with just 879 women per 1000 males. (a) Fish Production
137. The economist who for the first time (b) Coal production
scientifically determined National Income in (c) Crude oil production
India: (d) Mustard Production
(a) D.R. Gadgil (e) None of the above/More than one of the
(b) V.K.R.V. Rao above
(c) Manmohan Singh
Ans. (c): Major agricultural revolutions that took place
(d) Y.V. Alagh in India are as :-
(e) None of the above/More than one of the Revolution Products/Aim
above
Black Revolution Crude Oil Production
Ans. (b): The first scientific method to calculate
national income of India was made by Professor Blue Revolution Fish Production.
V.K.R.V. Rao. Yellow Revolution Mustard Production
138. The percentage of decadal growth of Silver Revolution Egg Production
population during 2001-2011 as per Census Pink Revolution Onion Production/meat
2011 is : production

60th-62th BPSC (Pre) Exam. 2016-17 211 YCT


142. Which Indian state has the largest number of (c) Registered Manufacturing
Cotton Textiles Mills? (d) Transport
(a) Madhya Pradesh (e) None of the above/More than one of the
(b) Maharashtra above
Ans. (c): During 2005 - 06 to 2014 - 15, the
(c) Gujarat
manufacturing sector recorded highest growth rate.
(d) West-Bengal
147. "SPUR" project in Bihar relates to :
(e) None of the above/More than one of the (a) Health
above (b) Poverty
Ans. (b): Maharashtra has the largest number of cotton (c) Banking
textile mills in India. (d) Municipal Finances
143. Which district in Bihar is poorest according to (e) None of the above/More than one of the
per capita income criteria? above
Ans. (b): SPUR (Support Programme for Urban
(a) Begusarai
Reforms) is a 6 year technical assistance programme
(b) Madhepura
designed to accelerate economic growth and poverty
(c) Supaul reduction in the Indian State of Bihar.
(d) Sheohar 148. Total length of National Highways in Bihar is :
(e) None of the above/More than one of the (a) 3410 kms
above (b) 3587 kms
Ans. (d): Sheohar district in Bihar is poorest according (c) 4595 kms
to per capita income criteria. (d) 4707 kms
(e) None of the above/More than one of the
144. Which of the following Tax is levied by Union above
and collected and appropriated by States?
Ans. (c): The total length of the National Highways in
(a) Stamp Duites Bihar in 4595 km. The total length of NH in Bihar in
(b) Passenger & Goods Tax 5301 km. which was 4554 km during 2014. According
(c) Estate Duty to Ministry of Road Transport & Highways annual
(d) Taxes on Newspapers report 2021-22, the total length of National Highway in
Bihar is 5,940 Km.
(e) None of the above/More than one of the
above 149. Per capita energy consumption in the state of
Bihar is:
Ans. (a): As per Article 268 of Indian Constitution, (a) 20 kWh
Stamp Duties are mentioned in the Union List shall be (b) 187 kWh
levied by the Government of India but collected and (c) 161 kWh
appropriated by the States. (d) 145 kWh
145. Bank rate means (e) None of the above/More than one of the
(a) Interest rate charged by moneylenders above
(b) Interest rate charged by Scheduled Banks Ans. (e): As of 2018 - 19, The per capita energy
consumption in the state of Bihar was 311 Kilo Watt.
(c) Rate of profit of banking institution
150. ''JEEVIKA'' is an initiative of Bihar Govt. for :
(d) Official rate of interest charged by central
(a) Employment Generation
Bank
(b) Financial Inclusion
(e) None of the above/More than one of the (c) Poverty Attenuation
above (d) Public Distribution
Ans. (d): Bank rate is the official rate of interest (e) None of the above/More than one of the
charged by the central bank for lending funds to above
commercial banks. Ans. (a): In 2006, Bihar Government inauguarated the
146. In Bihar, which sector recorded highest growth JEEVIKA project with the objective of social and
during 2005-06 to 2014-15? economic empowerment of the rural poor. The scheme
(a) Agriculture aims at employment generation, financial inclusion,
(b) Banking poverty attenuation, Public distribution etc.
60th-62th BPSC (Pre) Exam. 2016-17 212 YCT
63th Bihar Public Service Commission
Preliminary Examination, 2017-18
GENERAL KNOWLEDGE & GENERAL SCIENCE
(Solved Paper with Detail Explanation)
1. Quartz crystals used in clocks are Ans. (a) : Galvanizing, or galvanization, is a
(a) sodium silicate manufacturing process where a coating of zinc is
(b) silicon dioxide applied to steel or iron to offer protection and prevent
(c) germanium dioxide rusting. There are several galvanizing processes
(d) titanium dioxide available, but the most commonly offered and used
(e) None of the above/More than one of the method is called hot-dip galvanizing.
above 5. The wavelength of visible spectrum is in the
Ans. (b) : Quartz crystals used in clocks are silicon range
dioxide. Almost every modern clock that's made (a) 1300 Å–3900 Å (b) 3900 Å–7600 Å
nowadays has a quartz crystal-based oscillator inside (c) 7800 Å–8200 Å (d) 8500 Å–9800 Å
them. Quartz has piezoelectric property, which means (e) None of the above/More than one of the
that it generates a small voltage when pressure is above
applied on it. The reverse of this is also true, where a Ans. (b): The visible light spectrum is the segment of
voltage applied to the crystal causes it to vibrate at a the electromagnetic spectrum that the human eye can
specific frequency view. The wavelength of the visible spectrum is in the
range of 3800Å-7000Å (A is Angstrom). The visible
2. Bromine is a
light lies in between the infrared and ultraviolet range of
(a) Colourless gas
wavelengths. Among the options, option (b) is most
(b) Brown Solid appropriate.
(c) Highly inflammable gas
6. Detergents used for cleaning clothes are
(d) Red liquid (a) Carbonates
(e) None of the above/More than one of the (b) Bicarbonates
above (c) Bismuthates
Ans. (d): Bromine is a chemical element as of halogen (d) Sulfonates
group. It is brown-red liquid at room temperature which (e) None of the above/More than one of the
evaporates readily to form similarly colored vapour. It above
is the only non-metallic element that stays in a liquid Ans. (d): Detergents used for cleaning clothes are
state at room temperature. sulfonates. The surfonates in detergents improve water's
3. Gases used in welding are ability to wet things, spread over surfaces and seep into
(a) Oxygen and hydrogen dirty clothes fibers. Sulfonates are sodium salt of long
(b) Oxygen and nitrogen chain benzene sulphonic acid or the sodium salt of a
(c) Oxygen and acetylene long chain alkyl hydrogen sulphate..
(d) Hydrogen and acetylene 7. The element excreted through human sweat is
(e) None of the above/More than one of the (a) Sulfur (b) Iron
above (c) Magnesium (d) Zinc
(e) None of the above/More than one of the
Ans. (c): Gases used in welding process include: above
• Shielding gases such as carbon dioxide, argon, helium
Ans. (e): Sweating is the release of liquid from body's
etc.
sweat glands. Sweat is made up of several chemicals
• Fuel gases such as acetylene, propane, butane etc. mainly consists of water, minerals, lactic and Urea. The
• Oxygen used with fuel gases and also in small sodium potassium, calcium and magnesium are the
amounts in same shielding gas mixture. minerals composition of sweat. Some traces of metals
Thus, among the opting oxygen and acetylene are that the body excretes in sweat are zinc, copper, iron,
correct answer. chromium, Nickel and Lead.
4. Galvanized iron pipes have a coating of 8. Blue vitriol is chemically
(a) Zinc (b) Mercury (a) Sodium sulphate (b) Nickel sulphate
(c) Lead (d) Chromium (c) Copper sulphate (d) Iron sulphate
(e) None of the above/More than one of the (e) None of the above/More than one of the
above above
63th BPSC (Pre) Exam. 2017-18 213 YCT
Ans. (c): The blue vitriol is also known as the Copper of inflection. Penicillin was first ever 'true' antibiotic
Sulphate pentahydrate or crystalline copper sulphate. and it was discovered by Alexander Fleming in 1928.
The chemical formula for blue vitriol is CuSO4.5H2O. Paracetamol is not an antibiotic It is used to treat fever
Applications of blue vitriol: and mild to moderate pain. Aspirin is an antipyretic. It
It is used as a herbicide and a fungicide. is not an antibiotic, as It does not kill any micro-
It is used as an analytical reagent. organisms.
It is used as battery electrolytes. 13. Which of the following is mostly carbohydrate
It is used as a mordant during a vegetable dyeing ?
process (a) Wheat (b) Barley
Iron Sulphate is also known as Green vitriol. (c) Rice (d) Maize
9. The positively charged part at the centre of an (e) None of the above/More than one of the
atom is called as above
(a) Proton (b) Neutron Ans. (c): Rice is primarily composed by Carbohydrate.
(c) Electron (d) Nucleus Carbohydrates are macro nutrients. These are
(e) None of the above/More than one of the compounds of oxygen, hydrogen and carbon.
above Carbohydrates are essential nutrients which include
Ans. (d): The positively charged parts at the centre of sugars, fiber and starches.
an atom is nucleus. A nucleus of an atom has proton and 14. The heaviest metal among the following is
neutrons. As we know that a proton has charge of +1 (a) Gold (b) Silver
while a neutron has no charge, or zero. So the nucleus (c) Mercury (d) Platinum
of an atom will always have positive charge. (e) None of the above/More than one of the
10. The conversion of a solid directly into gas is above
called as Ans. (d): Heavy metal refers to any metallic chemical
(a) Sublimation (b) Condensation element that has a relatively high density. Among the
(c) Evaporation (d) Boiling above given options Platinum is the heaviest metal.
(e) None of the above/More than one of the Metal Atomi Atomi Grou Perio Densit
above c c No. p d y
Ans. (a): Sublimation is defined as the transition of a Symbo g/cm3
substance from the solid phase to the gaseous phase l
without changing into the liquid phase. examples are:- Gold Au 79 11 6 19.32
Dry Ice, Naphthalene etc. Silver Ag 47 11 5 10.49
Condensation is the process where water vapor becomes Mercur Hg 80 12 6 13.56
liquid. y
Evaporation is the process in which liquid turns into Platinu Pt 78 10 6 21.45
vapour. m
11. In human system, insulin controls the 15. Stainless steel is an alloy of
metabolism of (a) Iron and nickel
(a) Fats (b) Carbohydrates (b) Iron and chromium
(c) Proteins (d) Nucleic acids (c) Copper and chromium
(e) None of the above/More than one of the (d) Iron and zinc
above (e) None of the above/More than one of the
Ans. (b): Insulin is an anabolic hormone which is used above
for promoting the uptakes of glucose, glycogenesis, Ans. (e): Stainless steel is a corrosion - resistant alloy
lipogenesis, and protein synthesis that take place in the of iron, chromium and in some cases, nickel and other
skeletal muscle and fat tissue. In human system, insulin metals. Chromium play an important role in production
controls the metabolism of carbohydrates. In inhibits of stainless steel. It gives the hardness to the stainless
steel and makes it tough.
hepatic glucose production, promotes glucose storage in
the liver and glucose uptake and storage in muscles and 16. To protect broken bones, plaster of Paris is
used. It is
adipose tissues. (a) Slaked lime (b) Calcium carbonate
12. The antibiotic among the following is (c) Calcium oxide (d) Gypsum
(a) Penicillin (b) Aspirin (e) None of the above/More than one of the
(c) Paracetamol (d) Sulfadiazine above
(e) None of the above/More than one of the Ans. (d): Plaster of Paris is a white powdery chemical
above compound which is hydrated calcium sulphate that is
Ans. (c): Antibiotics are antimicrobial drugs obtained usually obtained from calcining gypsum. Plaster of
from the organisms (such as moulds, fungous & some Paris is also referred to as Gypsum plaster. It is a
popular chemical substance that is utilized most
soil bacteria) to combat harmful micro-organisms.
commonly for sculpting materials and in gauze
Sulfadiazine and Penicillin are examples of antibiotic bandages. The chemical formula of Plaster of Paris is
Sulfadiazine is used to treat and prevent a wide variety written as CaSO4·1/2H2O.
63th BPSC (Pre) Exam. 2017-18 214 YCT
17. A hydrocarbon in which two carbon atoms are 22. Mahmoud Abu Zeid has won the 2018
joined by a double bond is called as an UNESCO/Guillermo Cano World Press
(a) Alkane (b) Alkene Freedom Prize. He hails from which country ?
(c) Alkyne (d) Ionic bond (a) Israel (b) Iraq
(e) None of the above/More than one of the (c) Iran (d) Egypt
above (e) None of the above/More than one of the
Ans. (b) The alkenes comprise a series of compound above
that are composed of carbon and hydrogen atom with at Ans. (d): Egyptian photojournalist Mahmoud Abu Zeid
least one double bond in carbon chain. This group of has won the 2018 UNESCO/ Guillermo Cano World
compounds comprises a homologous series with a Press Freedom Prize. The Prize recognizes a person,
general molecular formula of CnH2n where 'n' equals organization or institution that has made an outstanding
any Integer greater than 1. contribution to the defence or promotion of press
The alkynes is an unsaturated hydrocarbon containing at freedom especially in the face of danger.
least one carbon triple bond'. Alkanes are organic 23. The bilateral military exercise 'Harimau Shakti
compounds that consist of single-bonded carbon and 2018' has been conducted between India and
hydrogen atoms. which country ?
18. The chemical name for vinegar is (a) Indonesia (b) Malaysia
(a) Methanol (b) Ethanol (c) New Zealand (d) South Korea
(c) Acetic acid (d) Ehtyl acetate (e) None of the above/More than one of the
(e) None of the above/More than one of the above
above
Ans. (b): During April 30th to May 13th, 2018, a joint
Ans. (c) The chemical name for vinegar is acetic acid.
training exercise HARIMAU SHAKTI 2018 was
Acetic acid is also known as ethanoic acid, ethylic acid,
vinegar acid and methane carboxylic acid, it has the conducted as a part of India-Malaysia defence
chemical formula of CH3COOH. Acetic acid is a by cooperation.
product of fermentation and gives vinegar its 24. Which Union Ministry has launched the
characteristic odor. Second Edition of the 'Unnat Bharat Abhiyan'?
19. The acid used in a car battery is (a) The ministry of Home Affairs
(a) Acetic acid (b) Hydrochloric acid (b) The ministry of Agriculture and Farmers
(c) Nitric acid (d) Sulfuric acid Welfare
(e) None of the above/More than one of the (c) The ministry of human resource Development
above (d) The ministry of Rural Development
Ans. (d): The acid in car battery uses Sulphuric acid as (e) None of the above/More than one of the
electrolyte. above
20. The gas, which comes out on opening a soda Ans. (c) : In 2014, the Ministry of Education had
water bottle is launched the Unnat Bharat Abhiyan. It aims to link the
(a) Carbon dioxide (b) Hydrogen higher education institution with a set of at least five
(c) Nitrogen (d) Sulfur dioxide villages. These institutions can contribute to the
(e) None of the above/More than one of the economic and social betterment of the village
above communities using their knowledge base.
Ans. (a): The gas which comes out on opening a soda 25. Which Union Ministry in collaboration with
bottle is carbon dioxide.Soda water, like other carbonate Google India has launched a 360° Virtual
beverage, contains carbon dioxide that has dissolved Reality (VR) video on ' Incredible India'?
under pressure upto 1200pounds per square inch. When (a) The Ministry of Electronics and Information
the pressure is released by opening the soda container, Technology
the liquid cannot hold as much carbon dioxide, so the (b) The Ministry of Information and
excess bubbles out of the solution. Broadcasting
21. Which of the following smart cities has become (c) The Ministry of social Justice and
the India's first city to run on 100% renewable Empowerment
energy during daytime? (d) The Ministry of Tourism
(a) Bengaluru (b) Jaipur (e) None of the above/More than one of the
(c) Indore (d) Diu above
(e) None of the above/More than one of the Ans. (d): Google Arts and culture in partnership with
above Ministry of Tourism had launched 360º Virtual Reality
Ans. (d): In April 2018, Diu Smart City has become (VR) experience video for global 'Incredible India'
first city in India that runs on 100% renewable energy tourism campaign. The immersive VR experience
during day time setting a new benchmark for other showcases the best elements from different parts of the
cities to become clean and Green. country and its varied traditions.

63th BPSC (Pre) Exam. 2017-18 215 YCT


26. Which city hosted India's first-ever nationwide 30. Which district has become the India's first
food archaeology conference 'ArchaeoBroma'? district to have 100% solar-powered health
(a) Mumbai (b) New Delhi centers?
(c) Indore (d) Surat (a) Kolkata (b) Chennai
(e) None of the above/More than one of the (c) Bengaluru (d) Surat
above (e) None of the above/More than one of the
Ans. (a) India first ever national conference on above
archaeology, anthropology and sociology of food was Ans. (d): In March 2018, the Surat district of Gujarat
held in Mumbai, Maharashtra on 5th and 6th May 2018. has becomes the India’s, first district to have 100%
It was organized by the India Study Centre Trust and solar powered primary health centre. There are a total of
the Centre for Extra-Mural Studies of the University of 52 PHCs in the district and all of them are powered by
Mumbai. The conference title 'Archaeo Broma' was solar system.
touted as India's first ever national meet on "Food as 31. Which Indian sports person has bagged Gold
Culture". in the Men's 25 m Rapid Fire Pistol Event at
27. What is the India's rank in the 2018 World the 2018 ISSF Junior Shooting World Cup in
Press Freedom Index (WPFI) ? Sydney?
(a) 136th (b) 138th (a) Rajkanwar Singh (b) Anhad Jawanda
(c) 135th (d) 137th (c) B. Sainath (d) Anish Bhanwala
(e) None of the above/More than one of the (e) None of the above/More than one of the
above above
Ans. (b): India ranked 138th in the World Press Ans. (d) : In 2018, ISSF Junior Shooting World Cup in
Freedom Index 2018.It is being published every year
Sydney, Anish Bhanwala had won individual gold
since 2002 by Reporters Without Borders
medal in 25 m Rapid Fire Pistol event on 27th March
20th edition of the World Press Freedom Index 2022
was published by Reporters Without Borders. 2018. Rhythm Sangwan and Anish Bhanwala won gold
India ranked 150th among the 180 countries. medal in the 25m rapid fire pistol mixed team in ISSF
Theme for 2022: world cup 2022.
Journalism under digital siege 32. Which state Government has singned a pact
Top Performers: Norway (1st) Denmark (2nd) with Airport Authority of India (AAI) to
Worst Performers: North Korea develop villages in Kamrup?
(a) West Bengal (b) Mizoram
28. Which institute has been awarded the 2018
(c) Nagaland (d) Assam
National Intellectual Property Award in the
(e) None of the above/More than one of the
category top R & D Institution /Organization
above
for Patents and Commercialization ?
(a) The Council of Scientific and Industrial Ans. (d) : Airport Authority of India and Assam State
Research Government had signed a Memorandum of
(b) The Indian Space Research Organization Understanding. Under its Corporate Social
(c) The Bhabha Atomic Research Center Responsibilities initiative for sustainable development
(d) The Defence Research and Development of human settlements at villages in Kamrup district of
Organization Assam.
(e) None of the above/More than one of the 33. Which state has the highest number of fully
above covered tribal habitations with the safe
Ans. (a) The Council of Scientific and Industrial drinking water?
(a) Madhya Pradesh (b) Odisha
Research had been awarded the National Intellectual
(c) Rajasthan (d) Jharkhand
Properly Award 2018 in the category "Top R&D
(e) None of the above/More than one of the
Institution/Organization for Patents and above
Commercialization. These awards are conferred every
year by office of Controller General of Patents, Design Ans.(a) : Madhya Pradesh has the highest number of
and Trade Marks, Department for Promotion of Industry 55,770 fully covered tribal habitations with safe
and Internal trade, Ministry of Commerce and Industry. drinking water in the country followed by Odisha
52427, Jharkhand 53476 number of habitations.
29. Which state has topped the list of states with
most number of beggars, as per data released 34. ISRO has successfully launched GSAT-6A
by the Union Ministry of Social Justice? satellite from which rocket vehicle?
(a) GSLV-F07 (b) GSLV-F09
(a) West Bengal (b) Andhra Pradesh (c) GSLV-F05 (d) GSLV-F08
(c) Bihar (d) Uttar Pradesh (e) None of the above/More than one of the
(e) None of the above/More than one of the above
above
Ans. (d): ISRO had successfully launched GSAT-6A
Ans. (a) As per data released by the Union Ministry of satellite from GSLV-F08 rocket vehicle on 29th March
Social Justice in 2021, the total number of beggars is 2018 from Satish Dhawan Space Centre, Shriharikota,
4,13,670. West Bengal ranked top with 81, 224 beggars Andhra Pradesh. GSAT-6A is a high power S-band
followed by 65, 835 beggar in Uttar Pradesh. communication Satellite configured around I-2K bus.

63th BPSC (Pre) Exam. 2017-18 216 YCT


35. What is the theme of the 2018 International Ans. (c) : West Bengal Government had launched
Day of Forests? "Rupashree Scheme" in 2018 to ensure safe future for
(a) Forests and Water young women. Under this scheme West Bengal
(b) Forests and Energy government provides one time financial grant of Rs.
(c) Forests and Sustainable cities 25000 for economically stressed families at the time of
(d) Forests and Life their adult daughter's marriage.
(e) None of the above/More than one of the 40. NITI Aayog has launched the 'Atal New India
above
Challenge' that will provide grants of up to
Ans. (c) : International Day of Forest or world forest how much amount to innovative products and
day is celebrated every year on 21st March. The day is solutions?
observed to raise awareness of the importance of all (a) ` 2 crore (b) ` 1 crore
types of forests. The theme for 2018 International Day (c) ` 1.5 crore (d) ` 2.5 crore
of Forest was "Forests and sustainable cities". The (e) None of the above/More than one of the
theme for 2022 is "Forests and Sustainable Production
above
and Consumption".
Ans.(b) : The National Institution for Transforming
36. Who has been elected as the new President of
India, NITI Aayog has launched the Atal New India
Egypt?
(a) Maoussa Mostafa Moussa Challenge under the Atal Innovation Mission (AIM)
(b) El-Sayyid el-Badawi with the aim of bringing in innovations and
(c) Abdel Fattah el-Sisi technologies, relevant to the masses. The applicant
(d) Ahmed shafiq Sisi displaying the capability and intent to manufacture
(e) None of the above/More than one of the marketable products under the 17 pre-identified focus
above areas will be granted grant of up to 1 crore rupees. The
Ans. (c) : Abdel fattah el-Sisi has served as the sixth 2nd edition of the Atal New India Challange (ANIC 2.0)
and current President of Egypt since 2014. launched on 28th April 2022.
37. Who has been chosen for the prestigious J.C. 41. Who chaired the first meeting of the think tank
Daniel Award for 2017? on the Framework for National Policy on E-
(a) Sathyan Anthikkad commerce?
(b) K.G.George (a) Narendra Modi (b) Arun Jaitley
(c) Sreekumaran Thampi (c) Suresh Prabhu (d) Smriti Irani
(d) Adoor Gopalakrishnan (e) None of the above/More than one of the
(e) None of the above/More than one of the above
above Ans. (c) : In April 2018, The then Commerce and
Ans. (c) : The J.C. Daniel Award is the Kerala's Industry Minister Suresh Prabhu had chaired the first
government highest honor for outstanding contribution meeting of the Think Tank set up to frame a national
to Malayalam cinema. The award includes a cash prize
policy on e- commerce. The think tank was with
of 5 lakh a statuette, a citiation. Sreekumaran Thampi
had been chosen for J.C. Daniel Award for 2017. He is objective formed to recommend a comprehensive and
a prolific scriptwriter of Malayalam. In 2021, P overarching national policy on E-commerce.
Jayachandran was awarded the JC Daniel Award. In 42. Which of the following districts of Bihar does
2022, Malayalam filmaker K P Kumaran was awarded not share border with Nepal?
the J.C. Daniel Awards. (a) Madhubani (b) Kishanganj
38. According to recent data released by Indian (c) Sitamarhi (d) Purnia
Cellular Association (ICA), India has acquired (e) None of the above/More than one of the
what position in the world in producing mobile above
phones? Ans. (d) : There are 7 districts in Bihar that shares the
(a) First (b) Second
(c) Third (d) Fourth border with Nepal. The districts are:- West champaran,
(e) None of the above/More than one of the East champaran, Sitamarhi, Madhubani, Supaul, Araria,
above and Kishanganj. Purnia district is sharing border with
Ans.(b) : As per data released by Indian Cellular Araria to the North, Bhagalpur to the south, Katihar to
Association (ICA), India acquired second position in the the east and Madhepur district to the west.
world after china in production of mobile phone’s. As 43. Which of the following is the only UNESCO
per data shared by ICA, annual production of mobile World Heritage Site present in Bihar State of
phone in India increased from 3 millions unit in 2014 to India?
11 million on units in 2017. (a) Madhubani
39. Which State Government has rolled out (b) Buddhist Monastery in Nalanda
'Rupashree Scheme' of marriage of poor girls?
(c) Ancient site of Vikramashila
(a) Tamil Nadu (b) Odisha
(c) West Bengal (d) Assam (d) Tomb of Sher Shah Suri
(e) None of the above/More than one of the (e) None of the above/More than one of the
above above
63th BPSC (Pre) Exam. 2017-18 217 YCT
Ans. (e): A World Heritage site is a place that is listed Ans.(d) : India Council of Forestry Research and
by UNESCO for its special cultural and physical Education (ICFRE) is an apex body in national forestry
significance. Bihar is the home of two UNESCO world research system that promotes and undertakes need
heritages sites and quite a few on the tentative lists. based forestry research extension. It was created in
1986, under the central Ministry of Environment and
Nalanda university is one of the 2 UNESCO world
Forests. It is headquartered at Dehradun, Uttarakhand.
Heritage site in Bihar listed in 2016. It is listed as the In 2018, ICFRE had signed MoU with a TIFAC for
archaelogical site of Nalanda Mahavira at Nalanda, supporting forest based communities.
Bihar. Mahabodhi, Temple complex or the Mahabodhi 47. Which of the following teams has won the
Mahavihar was inscribed in the UNESCO world Newton-Bhabha Fund for the project on
Heritage list 2002 as a unique property list of cultural Groundwater Arsenic Research in Ganga
and archaelogical importance. River Basin?
44. The Union Cabinet has approved MoU between (a) India-Russia Joint Team
India and SACEP for cooperation on oil and (b) India-UK Joint Team
chemical pollution in the South Asian seas (c) India-Japan Joint Team
(d) India-Israel Joint Team
region. What does SACEP stand for? (e) None of the above/More than one of the
(a) South Asia Compatibility Environment above
Programme Ans. (b) : In March 2018, an India- UK Joint team had
(b) South Asia Cooperative Environment won the Newton- Bhabha fund for project on ground
Programme water arsenic research in Ganga River Basin. The
(c) South Asia Connectivity Environment Department of Science and Technology had undertaken
Programme the project with the Natural Environment Research
(d) South Asia Coercive Environment Council of United Kingdom, to find solutions to the
water challenged faced in the pervasively arsenic-
Programme
affected Ganga River Basin. Research and innovators
(e) None of the above/More than one of the from UK and India to tackle the biggest challenges the
above face globally.
Ans. (b) : SACEP stands for - South Asia Co-operative 48. The Barh Super Thermal Power Station
Environment Programme which is an (BSTPS) is Located in which State?
intergovernmental organization, established in 1982 by (a) Bihar (b) Karnataka
the governments of South Asia to promote and support (c) Rajasthan (d) Punjab
(e) None of the above/More than one of the
protection, management and enhancement of the
above
environment in the region. It is headquartered at Ans. (a) : The Barh Super Thermal Power Station
Colombo, Sri Lanka. (BSTPS) is located in Barh, Bihar and is owned by
The members of SACEP includes Afghanistan, National Thermal Power Corporation (NTPC). The
Bangladesh, Bhutan, India, Maldives, Nepal, Pakistan mega power project was established by former P.M.
and Sri Lanka. Atal Bihari Vajpayee.
45. Which State Government has recently signed Hence, the correct option is (a).
MoU South Korea to enhance tourism 49. The International Solar Alliance (IAS) has
recently signed a pact with which international
cooperation ? organization to promote solar energy
(a) Kerala (b) Mizoram deployment in Asia and Pacific?
(c) Tamil Nadu (d) Uttar Pradesh (a) Asian Infrastructure Investment Bank
(e) None of the above/More than one of the (b) New Development Bank
above (c) African Development
Ans.(d) : In December 2017, UP Government had (d) Asian Development
(e) None of the above/More than one of the
signed a MoU with South Korea to enhance cooperation above
in the field of tourism, skill development, culture and Ans.(d) : The Asian Development Bank (ADB) and
farming. International Solar Alliance (ISA) signed a cooperation
46. The Indian Council of Forestry Research and agreement on 10th March 2018 to promote solar energy
Education (ICFRE) has signed MoU with development in the Asia and Pacific region .
TIFAC for supporting Forest-based 50. India has signed a tripartite MoU with which
Communities. Where is the headquarters of countries for civil nuclear cooperation?
(a) USA and Nepal
ICFRE located?
(b) Japan and Myanmar
(a) Hyderabad (b) Bengaluru (c) Russian and Bangladesh
(c) Allahabad (d) Dehradun (d) Germany and Sri Lanka
(e) None of the above/More than one of the (e) None of the above/More than one of the
above above
63th BPSC (Pre) Exam. 2017-18 218 YCT
Ans.(c) : In March 2018, India had signed a tripartite The Sutta Pitaka consists of the main teaching or
MoU with foreign partners Russia and Bangladesh for Dhamma of Buddha.
civil nuclear co-operation. The agreement was signed in The Abhidamma Pitaka is a philosophical analysis
Moscow. This MoU is helpful for cooperation in and systematization of the teaching and the scholarly
construction of Roopur Nuclear Power Plant in Pabna, activity of the monks.
Bangladesh. Other important Buddhist texts include Divyavadana,
51. What is the most significant feature of the Dipavamsa, Mahavamsa, Milind Panha etc.
Indus Valley Civilization? 55. The central point in Ashoka's Dhamma is
(a) Burnt brick Buildings (a) Loyalty to the king
(b) First true arches (b) Peace and non-violence
(c) Respect to elders
(c) Buildings of worship
(d) Religious toleration
(d) Art and architecture (e) None of the above/More than one of the
(e) None of the above/More than one of the above
above
Ans.(e) : The central point in Ashoka’s Dhamma are all
Ans. (a) : Indus Valley Civilization is also known as the given options. The Ashoka’s dhamma was not only
Harappan Civilization. It flourished around 2500 BC. the philosophical teaching of Buddha, but it also
Jhon Marshal was the first scholar to use the term 'Indus contained the idea of humanism, the universal
Valley Civilization'. Brunt Brick Building are the most brotherhood and Ahimsa. Hence, the correct option is
significant feature of Indus Valley Civilization other (e).
features of this Civilization are: personal Cleanliness, Ashoka’s Dhamma:-
town planning, ceramics, casting, forging of metals, Ashoka’s Dhamma cannot be regarded as a sectarian
manufacturing of catton and woolen textiles etc. faith. Its broad objective was to preserve the social
52. Megasthenes was the ambassador of order.
(a) Seleucus (b) Alexander It ordained that:-
(c) Daarius (d) The Greeks People should obey their parents.
(e) None of the above/More than one of the Pay respect to Brahmans.
above Buddhist monks show mercy to slaves and servants
Ans. (a) : Megasthenes was the ambassador of Seleucus 56. The division of Mauryan society into seven
Nicator. He wrote a book called Indica which is an classes is
(a) Kautilya's Arthashastra
account of Mauryan India. The book, Indica provides
(b) Ashokan edicts
valuable information on Maurya Administration, social (c) The Puranas
classes and economic activities. Megasthenes states that (d) The Indica of Megasthenes
Indian society was divided into 7 classes. (e) None of the above/More than one of the
53. Kautilya's Arthashastra is a book on above
(a) Economic relations Ans. (d) : Megasthenes states that Indian society was
(b) Principles and practice of statecraft divided into 7 classes during the Mauryan period.
(c) Freign policy Megasthenes was the ambassador of Seleucus Nicator.
(d) Welth accumulation He wrote a book called Indica which is an account of
(e) None of the above/More than one of the Mauryan era. The fragments of Indica, provide valuable
above information on Maurya Administration, social classes
and economic activities.
Ans.(e) : Kautilya also known as Vishnugupta and
Chanakya. He was the minister in the reign of Chandra 57. The famous Chinese pilgrim Fa-Hien visited
India during the reign of
Gupta Maurya during 317-293 BC. He has been
(a) Chandragupta I (b) Chandragupta II
considered as one of the shrewdest minister of that (c) Ramagupta (d) Srigupta
times and has explained his views on State, war, social (e) None of the above/More than one of the
structures, diplomacy, ethics, politics and statecraft very above
clearly in his book called Artha Shastra, Arthashastra Ans.(b) : Fa-hien was a Chinese Buddhist monk, who
has wide influence on Sanskrit literature. came to India in the 5th century AD during the reign of
54. Tripitakas are the sacred books of the Chandragupta II to acquire Buddhist texts. He wrote a
(a) Jains (b) Hindus book titled as ‘Fo-Kwo-Ki (the travels of Fa-hien)’. He
(c) Parsees (d) Buddhists visited the major sites associated with the Buddha, as
(e) None of the above/More than one of the well the renowned centres of education and Buddhist
above monasteries.
Ans.(d) : Tripitakas are the sacred books of the 58. The Chola age was most famous for
(a) Village assemblies
Buddhist So correct answer is (d).
(b) War with the Rashtrakutas
Three Pitakas (c) Trade with Ceylon
The Vinaya Pitaka consists of rules of conduct and (d) Advancement of Tamil culture
discipline applicable to the monastic life of the monks (e) None of the above/More than one of the
and nuns. above
63th BPSC (Pre) Exam. 2017-18 219 YCT
Ans.(a): The founder of Chola Empire was Vijayalaya, principle of his theory of kingship was the divine right
who was earlier feudatory of the Pallavas of Kanchi. He of kings. Balban said that the king was the
captured Tanjore in 850 A.D. The greatest Chola rulers representative of God on the earth and kingship is the
were Rajaraja-1, and his son Rajendra I. The most divine institution. He was also famous for his policy of
important feature of the Chola administration was local blood and iron. He introduced the Persian customs of
self government or village assemblies. Sijadah and Paibos. Balban himself was a member of
Other facts of Chola Empire : Chalisa or Chahalgani but he broke the power of
Cholas maintained a strong navy. ‘Kasu’ or Kalaju’ was Chahalgani and restored the prestige of the crown.
their gold coin. He created
Rajaraja built Vrihadeshwar/Rajarajeshwar temple 1. Diwan-i-Arz - Strong centralised army
(attributed to Shiva) at Tanjore. 2. Diwan-i-Wazarat - Separation of military affairs from
Rajendra I assumed the title of ‘Gangaikonda Cholam’ the finance department.
and built a city called Gangaikonda Cholapuram.
62. Paper came to be used in India in the
Rajendra I conquered complete Sri Lanka and made
(a) 12th century (b) 13th century
Anuradhapura his capital.
(c) 14th century (d) 15th century
59. The mighty gateways found at the temples of (e) None of the above/More than one of the
South India are called above
(a) Shikhars (b) Gopurams
Ans. (a) : Paper making was invented by Chinese. Tsai
(c) Devalayas (d) Mandaps
Lun, a Chinese official working in the Imperial Court
(e) None of the above/More than one of the
during the Han Dynasty, is associated with the
above
invention of paper. Paper came to be used in India in the
Ans.(b) : Two board orders of temple in the country are 12th century.
known as Nagara in the North India and Dravida in the
63. European paintings were introduced in the
South India. The Dravidian temple is enclosed within a
court of
compound wall, the front wall has an entrance gateway
(a) Humayun (b) Akbar
in its centre which is known as Gopuram. In the South
(c) Jahangir (d) Shah Jahan
India temple, the word 'Shikhara' is used for the
(e) None of the above/More than one of the
crowing element at the top of the temple which is
above
usually shaped like a small stupika. Vimana is the shape
of main temple tower. Ans. (b) : Europeans artist came to India along the
The entrance to temple which may be portico or British traders, who brought with them new style and
colonnaded hall that incorporates space for a large they began producing pictures which became widely
number of worshippers is known as Mandapa. popular in Europe and helped shape western perceptions
of India European Painting were introduced in the court
60. The remains of Vijayanagara empire are found
of Akbar by the Portuguese priests. Under their
in
influence, the foreshortening, where by near and distant
(a) Bijapur (b) Golconda
people and things could be placed in perspective was
(c) Hampi (d) Baroda
quietly adopted.
(e) None of the above/More than one of the
above 64. Who was the ruler of India when the English
East India Company was formed?
Ans.(c) : Vijayanagar or "City of Victory" was the
(a) Aurangzeb (b) Akbar
name of both city and an empire. Vijayanagar empire
(c) Jahangir (d) Humayun
was founded in the 14th century (1336 AD) by Harihara
(e) None of the above/More than one of the
and Bukka of the Sangama dynasty. They made Hampi
above
the Capital city. Vijaynagar Empire was ruled by four
importent dynasties and they are: Sangama, Saluva, Ans. (b) : The British East India Company was
Tuluva & Aravidu. The Ruins of Vijayanagar Empire established as a trading company. On 31st December
was found at Hampi (Karnataka) The temple town of 1600, the British East India Company received a Royal
Hampi was declared the UNESCO World Heritage Site charter from Elizabeth I to trade with the East Indias.
in 1986. The company went on to colonies the India sub
continents. Mughal Emperor Akbar was the ruler of
61. Who was the first Muslim ruler to formulate
India when British East India Company was formed in
the 'Theory of kingship' similar to the 'theory
Landon. Sir Thomas Roc reached the court of Mughal
of divine right of the kings?
Emperor, Jahangir as emissary of King James I in 1615,
(a) Aibak (b) Iltutmish gained for the British the right to establish a factory at
(c) Balban (d) Alauddin Surat.
(e) None of the above/More than one of the
65. Which of the following Princely States was not
above annexed by the British?
Ans. (c) : The first Muslim ruler to formulate the (a) Sind (b) Gwalior
‘theory of kingship' similar to the "theory of divine right (c) Awadh (d) Satara
of the kings” was Balban. Ghiyasuddin Balban was the (e) None of the above/More than one of the
ninth Sultan of the Mamluk dynasty of Delhi. The main above
63th BPSC (Pre) Exam. 2017-18 220 YCT
Ans. (b) : The Doctrine of Lapse was an annexation In Punjab under Lala Lajpat Rai and Ajit Singh
policy followed widely by Lord Dalhousie for the In Delhi under Syed Haider Raza
extension of British Paramountcy . Under the terms of In Madras under Chidambaram Pillai.
Doctrine of Lapse the company annexed many Princely 69. Bihar Provincial Kisan Sabha was formed in
States viza Satara (1848), Jaitpur and Sambalpur (a) 1929 (b) 1930
(1849), Jhansi (1853), Nagpur (1854) Tanjor and Arcot (c) 1931 (d) 1932
(1855), Udaipur and Awadh (1856). (e) None of the above/More than one of the
Sind was rulled by Baluchi chiefs collectively known as above
Amir of Sindh. Baluchis were defeated at the Battle of Ans. (a) : Sahajanand Saraswati established the Bihar
Miani & Battle of Dobo in 1843 & Sind was annexed to Provincial Kisan Sabha in 1929. The motive was to
the British empire in India. However Gwalior was not mobilise peasant grivances against the zamindari attacks
annexed by the British. on their occupancy rights and thus sparking the farmer's
66. What was the ultimate goal of Gandhi ji's Salt movement in India. Mohammad Zubair and Shree
Satyagraha? Krishna Singh had organised a Kisan Sabha in Munger
(a) Repeal of Salt laws in 1922.
(b) Curtailment of the government's power 70. By which Act, Bihar become a separate
(c) Economic relief to the common people province?
(d) 'Purna Swaraj" for India (a) The Government of India Act of 1935
(e) None of the above/More than one of the (b) The Government of India Act of 1919
above (c) The Indian Councils Act of 1909
(d) The Indian Independence Act of 1947
Ans. (e) : On 12th March 1930, Gandhi set out from (e) None of the above/More than one of the
Sabarmati with 78 follower on a 241 mile march to the above
coastal town of Dandi on the Arabian Sea, to break the Ans. (a) : On 1st April 1936, Bihar and Orissa became
salt law by producing salt from sea water. The Salt separate provinces by the Government of India Act,
March or Salt Satyagraha began on 12th March 1930 1935. Earlier, Bihar, Orissa and Bengal were single
and continued till 6th April 1930. Salt Satyagraha led the provinces. On 1st April 1936, Sir James David Sifton
mass Civil Disobedience Movement. The ultimate goal appointed the first governor of Bihar whereas
of Gandhiji Salt Satyagraha was:- Repeal of Salt laws, Muhammad Yunus was declared the first Prime
economic relief for common people. The declaration of Minister of state.
Purna Swaraj came after the Dandi March. 71. Which one of the following Committees/
67. The Moplah Rebellion of 1921broke out in Commissions recommended the creation of
(a) Assam (b) Kerala Nyaya Panchayats?
(c) Punjab (d) Bengal (a) Balwant Rai Meahta Committee
(e) None of the above/More than one of the (b) Ashok Mehata Committee
above (c) G.V.K. Rao committee
Ans. (b) : Mopalah Rebellion also known as the (d) Sarkaria Commission
Mopalah Riots of 1921, was the culmination of series of (e) None of the above/More than one of the
riots by Mappila Muslim of Kerala in the 19th and early above
20th centuries against the British and the Hindu Ans. (b) : Ashok Mehta Committee was appointed in
landlords in Malabar (Northern Kerala). This armed 1977 to suggest measures to revive and Strengthen the
rebellion was led by Variyamkunnath Kunjahammed declining Panchayati Raj System in India. The
Haji. committee recommended that the Nayaya Panchayats
should be kept as separate bodies from that of
68. The Swadeshi Movement took Place in India
department Panchayats. Balwant Rai Mehta is credited
during
for pioneering the concept the Panchayati in India.
(a) The champaran Satyagraha of Gandhi ji
LM Singhvi committee is best known for
(b) The anti-Bengal partition agitation recommending the constitutional status for Panchayats.
(c) The protest against the Rowlatt Act
(d) The Non-Cooperation Movement 72. Which one of the following functions is not the
concern of Local Self- Government?
(e) None of the above/More than one of the (a) Public Health
above (b) Sanitation
Ans. (b) : The Indian National Congress (INC) in a (c) Public Utility Services
meeting in 1905 resolved to condemn the partition of (d) Maintenance of Public Order
Bengal and support the Anti- Bengal partition and (e) None of the above/More than one of the
Swadeshi Movement. In the 1906 Congress Session above
held at Calcutta, the INC under the Presidentship of Ans. (d) : The Maintenance of Public Order is not
Dadabhai Naoroji declared self-government or Swaraj concern of local self government. It has been mentioned
as the goal of INC. Although the movement was in State List under schedule 7th of the constitution of
confined majorly to Bengal, it spread to other parts of India. Public health, Public utility services, sanitation,
India discussed as follows: roads and bridges, development scheme etc are related
to local self government.
In Poona and Bombay under Bal Gangadhar Tilak
63th BPSC (Pre) Exam. 2017-18 221 YCT
73. The system of Panchayti Raj is mentioned in 77. Who appoints the Judges of the Supreme Court
(a) The Union list (b) The State list of India?
(c) The Concurrent List (d) None of the above (a) The Prime Minister
(e) More than one of the above (b) The President
Ans. (b) : Panchayati Raj Institutions (PRI) were (c) The Chief Justice of India
constitutionalized through 73rd Constitutional (d) The Ombudsman
Amendment Act 1992 to build democracy at the grass (e) None of the above/More than one of the
roots level and was entrusted with the task of rural above
development in the country. Panchayati Raj is a state Ans. (b) : Article 124 (2) of Indian constitution states
subject and part of State List in the 7th Schedule of the every Judge of Supreme Court shall be appointed by
constitution of India. the President by warrant under his hand and seal after
74. 50% reservation for women in Panchayati Raj consultation with such of the Judges of Supreme Court
Institutions is Legalized in certain States such as & of the High Courts in the states as the President may
(I) Bihar deem necessary for the purpose and shall hold office
(II) Uttarakhand
until he attains the age of 65 years.
(III) Madhya Pradesh
(IV) Himachal Pradesh 78. Who is the Ex-officio Chairman of the Rajya
(a) Only III (b) Only II and III Sabha?
(c) II,III and IV (d) I,II,III,and IV (a) The President (b) The Vice President
(e) None of the above/More than one of the (c) The Prime Minister (d) None of the above
above (e) More than one of the above
Ans. (d) : At present, 20 States namely Andhra Pradesh, Ans. (b) : Article 64 States that the Vice President of
Assam, Bihar, Chhattisgarh, Gujarat, Himachal Pradesh, India is the Ex officio Chairman of Rajya Sabha. He is
Jharkhand, Karnataka, Kerala, M.P., Maharashtra. expected to run the affairs of the Rajya Sabha. When
Odisha, Punjab, Sikkim, Rajasthan, Tamil Nadu, the V. President acts as the President or discharges
Telangana, Tripura, Uttarakhand & West Bengal have function of President under Article 65 of the Indian
made Provisions, of 50% reservation for women in Constitution, he cannot perform duties of office of
Panchayati Raj Institution in their respective state chairman of Rajya Sabha.
Panchayati Raj legislations. 79. The Prime Minister of India is responsible to
75. Money Bill can be presented Whom?
(a) Only in the Lok Sabha (a) The Cabinet (b) The President
(b) Only in the Rajya Sabha (c) The Lok Sabha (d) The Rajya Sabha
(c) In both the Houses
(d) In joint meeting of the Houses (e) None of the above/More than one of the
(e) None of the above/More than one of the above
above Ans. (c) : As per article 75(3) Prime Minister and
Ans. (a) : Article 110 of the Indian Constitution defines Council of Ministers is collectively responsible to the
money bill. A money bill can be introduced only in Lok Lok Sabha. This is because India has adopted the
Sabha on the recommendation of the President of India. British model of Parliamentary Democracy rather than
Speaker of Lok Sabha decides whether a bill is money the Presidential form .In other words, PM stays in the
bill or not. A money bill after having been passed by office only as long as he enjoys majority support in the
Lok Sabha & sent to Rajya Sabha for its Lok Sabha.
recommendations has to be returned to Lok Sabha by
the Rajya Sabha within a period of 14 days from the 80. What is the minimum age for a Member of the
date of its receipt with or without recommendations.The Parliament?
amendments by Rajya Sabha may or may not be (a) 18 years (b) 21 years
accepted by the Lok Sabha. (c) 25 years (d) 30 years
76. What is the nature of the Indian Constitution? (e) None of the above/More than one of the
(a) Federal above
(b) Unitary Ans. (e) : As per the provisions envisaged in our
(c) Parliamentary Constitution's Article 84 (B) :
(d) Federal in nature but unitary in spirit The minimum age criteria for being elected as a
(e) None of the above/More than one of the Member of Parliament in Lok Sabha (Lower House of
above Parliament) is 25 Years and in Member of Parliament in
Ans. (d) : B.N. Banerjee has said that "The Constitution Rajya Sabha (Upper House of Parliament) is 30 Years.
of India is federal in nature but unitary in spirit". In fact Similarly for State Legislaturey, Article 173 (B)
Indian constitution has many federal as well as unitary states that:
features. This means that both central and state The minimum age criteria for being elected as a
government have their own powers defined by the
Member of Legislative Assembly (MLA) in Vidhan
constitution itself. However for some situations there
are provisions in constitution when state governments Sabha is 25 Years and in Legislative council (MLC) in
become subordinate to central government. Vidhan Parishad is 30.

63th BPSC (Pre) Exam. 2017-18 222 YCT


81. What is the retirement age of the Judges of the 85. According to the 14th Finance Commission,
Supreme Court? The percentage share of States in the net
(a) 60 years (b) 62 years proceeds of the shareable central tax revenue
(c) 65 years (d) 70 years should be
(e) None of the above/More than one of the (a) 32 -percent (b) 35 -percent
above (c) 40- percent (d) 42- percent
Ans. (c) : Every Judge of the Supreme court shall be (e) None of the above/More than one of the
appointed by the President of India. Judges of Supreme above
Court shall hold office until he/she attains the age of 65 Ans. (d) : The 14th Finance Commission had increased
years. The retirement ages for Judges of High Courts is the states share in central taxes from 32% to 42% of the
62 years. total divisible pool of central tax revenues.
82. Who is the highest Civil Services Officer of the For 2021-26, the 15th Finance Commission has set 41%
Central Government share for states in the divisible pool of the centre's taxes.
(a) The Attorney General of India 86. Which sector in India attracts the highest FDI
(b) The Cabinet Secretary equity flow?
(c) The Finance Secretary
(a) Construction Sector (b) Energy Sector
(d) The Home Secretary
(c) Automobile sector (d) Service Sector
(e) None of the above/More than one of the
above (e) None of the above/More than one of the
above
Ans. (b) : Cabinet Secretary is the highest Civil
Services officer of the Central Government. Ans. (d) : During 2018-19, Services sector in India had
The Attorney General of India is the highest law officer attracted the highest FDI equity flow followed by
of the country and also the chief legal advisor to the computer software and hardware.
Government of India. • Top five sectors receiving highest FDI equity inflow
The Finance Secretary is the administrative head of the F.Y. 2021-22 are Computer Software & Hardware
Minister of Finance. (24.60%), Services sector (12.13%), Automobile
The Home Secretary is the administrative head of the Industry (11.89%), Trading (7.72%) and Construction
Ministry of Home Affairs. (Infrastructure) Activities (5.52%)
83. One of the problems in calculating National • FDI Equity inflow in Manufacturing Sectors have
Income in India is increase by 76% in F.Y. 2021-22 (USD 21.34 billion)
(a) Underemployment compared to previous F.Y. 2020-21 (USD 12.09
(b) Inflation Billion).
(c) Low level of savings 87. Who amongst the following is the present
(d) non-organized sector Governor of the RBI?
(e) None of the above/More than one of the
(a) Bimal Jalan
above
(b) Arvind Subramanian
Ans. (d) : The Non- organized Sector is the biggest
(c) Raghuram Rajan
problem in calculating National Income in India.
(d) Urjit Patel
National Income means the value of goods and services
produced by a country during a financial year. Some (e) None of the above/More than one of the
other problem while calculating National Income are:- above
(i) Exclusion of Real Transations Ans. (d) : When the question was asked, Urjit Patel was
(ii) The value of Leisure the Governor of the Reserve Bank of India. Shaktikanta
(iii) Cost of Environmental Damages Das currently serving as the 25th Governor of the
(iv) Self- Consumption Reserve Bank of India.
(v) Imputed Income 88. The size of marginal land- holding in India is
(vi) Transfer Payments and Capital Gains. (a) more than 5 hectares
84. The apex bank for providing agricultural (b) 2 hectares to 4 hectares
refinance in India is (c) 1 hectares to 2 hectares
(a) RBI (b) NABARD (d) less than 1 hectares
(c) SBI (d) Imperial Bank (e) None of the above/More than one of the
(e) None of the above/More than one of the above
above Ans. (d) : In agriculture census, the Indian Government
Ans. (b) : National Bank for Agriculture and Rural classifies land holding into five groups:-
Development (NABARD) came into existence on 12th (i) Marginal land holding (less than 1 hectare)
July 1982. It is the apex banking institution to provide (ii) Small land holding (1-2 hectare)
finance for agriculture and rural development. Its (iii) Semi-Medium land holding (2-4 hectare)
headquarter is located in Mumbai. It Supervises (iv) Medium land holding (4-10 hectare)
Cooperative Banks and Regional Rural Banks (RRBs) (v) Large land holding (10 hectare and above)

63th BPSC (Pre) Exam. 2017-18 223 YCT


89. Globalization does not include- Ans. (b) : Gullak Bachcha Bank is a bank based in
(a) Reduction in import duties Patna, Bihar. This bank has been opened with the
(b) Abolition of import licensing objectives of teaching the youngsters - habit of saving,
(c) Free flow of FDI use of the cash and its proper management.
(d) Disinvestment of Public Sector Equity 94. The base year for All-India Wholesale Price
(e) None of the above/More than one of the Index (WPI) has been changed by the
above Government of India from 2004-05 to
Ans. (d) : Globalisation refers to the integration of the (a) 2010-11 (b) 2011-12
economy of the national with the world economy. It (c) 2012-13 (d) 2013-14
aims to create a borderless world. Since Globalization (e) None of the above/More than one of the
include free flow of trade, It also involves reduction in above
import duties abolition of import licensing, free flow of Ans. (b) : The Government of India revised the base
FDI etc. It does not include the disinvestment of public year for All- India Whole Sale Price Index (WPI) from
sector equity. Disinvestment is the process in which a 2004-05 to 2011-12. WPI measures the average change
certain percentage of shares of public sector units are in prices of goods at the wholesale level. The base year
disinvested to the private sector. of WPI was revised in order to align it with base year of
90. The Prevention of Money Laundering Act other micro economic indicators like the GDP and
came into force in India during Index of Industrial production.
(a) 1998 (b) 1999 95. According to Indian Population census, 2011,
(c) 2001 (d) 2005 the percentage of literacy among males and
(e) None of the above/More than one of the female both is lowest for the State of
above (a) Arunachal Pradesh (b) Andhra Pradesh
Ans. (d) : Prevention of Money Laundering Act (c) Bihar (d) Jammu and Kasmir
(PMLA) was enacted by the Indian Parliament in 2002 (e) None of the above/More than one of the
and came into force in 2005. It was enacted to curb above
money laundering (Process of converting black money Ans. (c) : As per census 2011, the percentage of literacy
into white) & to provide for seizure of property derived among males and females both is lowest for the state of
from money laundering. Bihar. Literacy Rate of Bihar is 61.80% of that male
91. TRIMs stands for literacy is 71.20% and female literary rate is 51.50%.
(a) Trade-Related Income Measures 96. The average size of landholdings in Bihar in
(b) Trade-Related Incentives Measures comparison with other State is
(c) Trade-Related Investment Measures (a) Lowest (b) Second Lowest
(d) Trade-Related Innovative Measures (c) Highest (d) Second highest
(e) None of the above/More than one of the (e) None of the above/More than one of the
above above
Ans. (c) : The agreement on Trade-Related Investment Ans. (b) : As per Agriculture Census, the average size
Measure (TRIMs) recognizes that certain investment of land holding in Bihar in comparison with other states
measures can restrict & distort trade. It state that WTO is the second lowest. The average size of land holding is
members may not apply any measures that highest in Nagaland while lowest is in Kerala.
discriminates against foreign product or that leads to 97. The latest per capita income at current prices is
quantitative restrictions, both of which violate basic lowest for the Indian State of
WTO principles. (a) Bihar (b) Uttar Pradesh
92. NITI Aayog came into existence on (c) Odisha (d) Nagaland
(a) 1st January, 2014 (b) 1st June, 2014 (e) None of the above/More than one of the
(c) 1st January, 2015 (d) 1st June, 2015 above
(e) None of the above/More than one of the Ans. (a) : As per data released by Ministry of Statistics
above and Programme Implementations, Bihar had the lowest
Ans. (c) : Planning Commission was replaced by a new per capita income Rs. 44652 in Fiscal Year 2018-19
institution - NITI AAYOG on 1th January 2015. NITI while Goa has the highest NSDP per capita income at
stands for "National Institution for Transforming India" current prices.
It is country's premier policy making institution that is 98. The latest annual; per capita consumption of
expected to bolster the economic growth of the country. electricity in Bihar is
It is an executive body. It is neither constitutional nor (a) Lowest (b) second lowest
statutory body. Prime Minister of India is the chairman (c) highest (d) second lowest
of NITI Aayog. (e) None of the above/More than one of the
93. 'Gullak Bachcha Bank' is a bank based in above
(a) Delhi (b) Patna Ans. (a) : The States with lowest annual per capita
(c) Bhopal (d) Jaipur consumption of electricity in F.Y. 2019-20 is Bihar at
(e) None of the above/More than one of the 332 Kwh, followed by the Assam 348 Kwh, Nagaland
above 367 Kwh, Manipur 385 Kwh and Tripura 425 Kwh.

63th BPSC (Pre) Exam. 2017-18 224 YCT


99. The average yield of cotton is highest in India Ans. (b) : Capability Approach is associated with Prof.
for State of Amartya Sen. Capability Approach is a process of
(a) West Bengal (b) Bihar development by enhancing people's capability there by
(c) Assam (d) Andhra Pradesh expanding their real freedoms. The capability to
(e) None of the above/More than one of the function effectively is what matters the most and it goes
above well beyond the availability of commodities.
Ans. (d) : Cotton in an important crop and commodity 105. Which one of the following is known as the
produced in India. As per data provided by RBI 'Coffee Port' of the world?
productivity of cottons is highest in Punjab with 827 (a) Sao Paulo (b) Santos
kg/hectare. Among the options Andhra Pradesh has (c) Rio de Janerio (d) Buenos Aires
highest productivity with 649 kg/hectare. (e) None of the above/More than one of the
100. The road length per lakh of population in above
Bihar is Ans.(b) : Santos port of Brazil is known as the 'Coffee
(a) Lowest (b) second lowest Port' of the world.
(c) highest (d) second highest 106. Which one of the following States has the
(e) None of the above/More than one of the highest percentage of area under forests?
above (a) Himachal Pradesh (b) Assam
(c) Andhra Pradesh (d) Arunachal Pradesh
Ans. (b) : Road density in Bihar is second lowest
(e) None of the above/More than one of the
followed by Jharkahand.
above
101. During 10000 BC, the population of the world
Ans. (d) : As per India State of Forest Report, 2021,
was
The Total forest and Tree cover is 24.62% of the
(a) 2 million (b) 3 million
geographical area of the country. Mizoram (84.53%)
(c) 4 million (d) 5 million
Arunachal Pradesh (79.33%)> Meghalaya (76.00%)>
(e) None of the above/More than one of the
Manipur (74.34%)> Nagaland (73.90%).
above
107. Which one of the following soils is deposited by
Ans. (e) : The population of the world during 10000 BC rivers?
was approximately 1 million. (a) Red soil (b) Black soil
102. Cocos Plate lies between (c) Alluvial soil (d) Laterite soil
(a) Central America and Pacific Plate (e) None of the above/More than one of the
(b) South America and Pacific Plate above
(c) Red Sea and Persian Gulf Ans. (c) : Alluvial deposit, Material deposited by rivers.
(d) Asiatic Plate and Pacific Plate It consists of silt, sand, clay, and gravel, as well as
(e) None of the above/More than one of the much organic matter. Alluvial deposits are usually most
above extensive in the lower part of a river’s course, forming
Ans. (a) : Cocos Plate lies between the Central America floodplains and deltas, but they may form at any point
and Pacific plate. Cocos plate is a young oceanic where the river overflows its banks or where the flow of
tectonic plate, named for Cocos Island which rides upon a river is checked. It widespread in the northern plains
it. and river valleys.It covers about 40% of the total land
103. 'Rust Bowl' of the USA is associated with which area of the country.They are rich in potash but poor in
one of the following regions? phosphorus. Two different types of alluvial soils have
(a) Great Lakes region (b) Alabama region developed in the Upper and Middle Ganga plains –
(c) California region (d) Pittsburg region Khadar and Bhangar.
(e) None of the above/More than one of the Khadar is the new alluvium and occupies the flood
above plains of the rivers. Khadar is enriched with fresh silt
Ans. (d) Rust Bowl of the USA is associated with deposits every year.
Pitttsburg region. Bhangar is the old alluvium, deposited away from the
Rust Bowl is a geographic region that was formerly a flood plains.
manufacturing or industrial power house but now facing 108. What is the share of hydroelectric power in the
industrial decline. It is also known as Rust Belt. total electricity produced in India?
104. Which one of the following approaches is (a) 10 percent (b) 12 percent
associated with Prof. Amartya Sen? (c) 20 percent (d) 22 percent
(a) Basic needs Approach (e) None of the above/More than one of the
(b) Capability Approach above
(c) Income Approach Ans. (a) : As of recent data about 65% of electricity
(d) Welfare Approach consumed in India is generated by thermal power
(e) None of the above/More than one of the plants, 22% by hydroelectric plants and 3% by nuclear
above power plant & rest 10% by alternative source of energy.
63th BPSC (Pre) Exam. 2017-18 225 YCT
109. Which Census year is called 'Demographic Ans. (a) : Khetri Belt of Rajasthan is famous for copper
Divide' in the population study of India? mining other important copper mine is Malanjkhand
(a) 1911 (b) 1921 Copper belt in Madhya Pradesh.
(c) 1931 (d) 1941 114. Which one of the following rivers joins the
(e) None of the above/More than one of the Ganga at Fatuha?
above (a) Son (b) Punapun
Ans. (b): The year 1921 is known as the year (c) Sakri (d) Balan
demographic divide because it is the only census year (e) None of the above/More than one of the
when there was a decrease in the growth of population. above
The census year 1911- 1921 registered a negative Ans. (b) : Punpun river join Ganga at Fatuha.
growth rate of-0.31 per cent which happened only once Punpun is the second most important river in South
throughout the demographic history of India. Bihar. It originates from Chhotanagpur Plateau region
110. The largest number of cotton mills in Tamil of Palamu district in Jharkhand.
Nadu are found in 115. According to Koppen's climatic classification,
(a) Chennai (b) Coimbatore the climate of North Bihar may explained as
(c) Madurai (d) Salem (a) Cwg (b) Aw
(c) CA'w (d) CB'w
(e) None of the above/More than one of the
(e) None of the above/More than one of the
above above
Ans. (b): The largest number of cotton mills in Tamil Ans. (a) : According to Koppen's climatic classification,
Nadu are found in Coimbatore. Combatore is often the climate of North Bihar may be explained as Cwg.
referred to as 'Manchester of South India' due to its The climate in North Bihar is monsoon type with dry
cotton production and textile industries. Tamil Nadu has winters, the summer received an ample amount of
dual distinction of holding the maximum number of rainfall while there is very little rain in winters.
textile mills and maximum number of shutdowns. 116. Which soil is predominantly found in he
111. Which one of the following States is the leading districts of Muzaffarpur, Darbhanga and
producer of rubber in India? Champaran?
(a) Tamil Nadu (b) Kerala (a) Black soil (b) Newer alluvium
(c) Karnataka (d) Andhra Pradesh (c) Older alluvium (d) Red soil
(e) None of the above/More than one of the (e) None of the above/More than one of the
above above
Ans. (b) : Kerala tops rubber cultivation, followed by Ans. (b) : Newer Alluvium soil is predominately found
Tripura. Other major natural rubber producers are in districts of Muzaffarpur, Darbhanga and Champaran.
Karnataka, Tamil Nadu, Assam and other Northeastern Khadar is the newer deposits of alluvial soil, Khadar has
more fine particles & is more fertile than the Bangar.
States.
Thailand is the largest producer of rubber in world 117. The largest jute- producing district of Bihar is
followed by Indonesia & Vietnam. India stands at 4th (a) Siwan (b) Gaya
(c) Vaishali (d) Purnia
Position in Rubber Production. (e) None of the above/More than one of the
112. National Waterway-1 connects above
(a) Kottapuram to kollam(b) Sadiya to Dhubri Ans. (d) : Bihar is second largest producer of Jute in
(c) Haldia to Allahabad (d) Haldia to Kolkata country after West Bengal. The major jute producing
(e) None of the above/More than one of the districts of the states are Katihar, Purnia, Saharsa,
above Supaul and Mathepura. As per data of 2017-18 Supaul
Ans. (c) : There are 111 National Waterways (NWs) district was largest producer. Thus among the options
Purnia is correct answer.
declared under the National Waterways Act, 2016.
118. Triveni Canal has been constructed on the
Details of some National Waterways.
river
1. NW-1: Ganga-Bhagirathi-Hooghly River System
(a) Kosi (b) Son
(Haldia - Allahabad)
(c) Gandak (d) Kamla
1620Km (e) None of the above/More than one of the
Uttar Pradesh, Bihar, Jharkhand, West Bengal above
2. NW-2: Brahmaputra River (Dhubri - Sadiya) 891Km
Ans. (c) : Triveni canal is built on Gandak river. It has
Assam been taken out from left bank of Gandak river. It is used
3. NW-3: West Coast Canal (Kottapuram - Kollam), for irrigations in Champaran district of Bihar.
Champakara and Udyogmandal Canals 205 Km- Kerala
119. The salvation place of Mahavir ji, the
113. Khetri Belt of Rajasthan State is famous for originator of Jain religion, is located at
(a) Copper mining (b) gold mining (a) Maner (b) Rajgir
(c) mica mining (d) iron ore mining (c) Pavapuri (d) Jalan Fort
(e) None of the above/More than one of the (e) None of the above/More than one of the
above above
63th BPSC (Pre) Exam. 2017-18 226 YCT
Ans. (c) : Mahavira was the 24th Tirthankara of Jainism, (c) Rambriksh Benipuri
in search of the truth, he abandoned the home at the age (d) karpoori Thakur
of 30 & became ascetic. At the age of 42 the attained (e) None of the above/More than one of the
perfect infinite knowledge known as Kaivalya. He above
conquered misery & happiness, he is known as Ans. (b) : Bihar Socialist Party was formed with
Mahavira or Jina. He passed away at the age of 72 in Acharya Narendra Dev as a President and Jay Prakash
527 BCE at Pavapuri near Patna. Narayan as General Secretary in 1934. It was
established at Anjuman Islam Hall in Patna on 17th May
120. Kaimur plateau is famous for 1934.
(a) Copper (b) Limestone
(c) Lithium (d) Bauxite 125. Who was the member of the Constituent
(e) None of the above/More than one of the Assembly of India from Bihar?
(a) A.N.Sinha
above (b) Rajendra Prasad
Ans. (b) : Kaimur Plateau or Rohtas plateau is situated (c) Jagjivan Ram
in Bihar. This Plateau is famous for limestone. The (d) Shayam Nadan Prashad Mishra
plateau is located in Bihar. (e) None of the above/More than one of the
121. Swami Sahajanand Saraswati published a above
journal named Ans.(e) : Personalities from Bihar in constituent
(a) Janakranti (b) Hunkar assembly were as follows:-
(c) Krishak Samachar (d) Vidrohi 1) Dr. Rajendra Prasad
(e) None of the above/More than one of the 2) Shri Anugrah Narayan Sinha
above 3) Shri Krishna Sinha
Ans. (b) : Swami Sahajanand Saraswati is recognized as 4) Maharaja Kameshwar Singh
one of the principle and significant leaders of Peasantry 5) Shri Jagat Narayan Lal
movement in Bihar in 1929, he established the Bihar 6) Shir Shyam Nantan Sahay
Provincial Kisan Sabha. He along with other ran the 7) Shri Satyanarayan Sinha
Hindi literary weekly Hunkar from Patna. 8) Shri Jaipal singh
9) Babu Jaggiyan Ram
122. Who established a branch of Anushilan Samiti 10) Shri Ram Narayan Singh
at Patna 11) Shri Brajeshwar Prasad
(a) Rewati Nag 126. The first Indian Ministry in Bihar in 1937 came
(b) Yadunath Sarkar into existence under whose leadership?
(c) Sachindra Nath Sanyal (a) Shri Krishna Singh (b) Md. Yunus
(d) Mazharul Haque (c) G.S. Lal (d) Wahab Ali
(e) None of the above/More than one of the (e) None of the above/More than one of the
above above
Ans. (c) : Sachindra Nath Sanyal was a Indian Ans. (b) : The first Indian Ministry in Bihar in 1937
revolutionary. He was the founder of Hindustan came into existence under the leadership of Md. Yunus.
Republican Association. He had founded a Branch of He took the oath as the first Prime Minister of Bihar
the Anushilan Samiti in Patna in 1913. He was Province on 1st April 1937 under the Government of
sentenced to life for his involvement in the Kakori Train India Act 1935.
Action & was imprisoned at Cellular Jail where he
wrote his book titled Bandi Jeevan in 1922. He & 127. Whom did Rajendra Prasad consider as the
Gandhi ji engaged in a famous debate published in Father of Pakistan ?
Young India between 1920 & 1924. (a) Md. Jinnah (b) Lord Mountbatten
123. Who organized Kisan Sabha at Munger in (c) Lord Minto (d) Liaquat Ali khan
1922-23? (e) None of the above/More than one of the
(a) Swami Sahajanand Saraswati above
(b) Shri Krishna Singh Ans. (c) : Lord Minto also came to be renowned as the
(c) Mohammed Jubair father of the communal electorate. He was the part of
(d) K.N. Singh the Morley Minto Reform in 1909. It introduced a
(e) None of the above/More than one of the system of communal representation for Muslims due to
above this, Rajendra Prasad considered Lord Minto as the
Ans. (e) : Kisan Sabha was organized in 1922-23 by Father of Pakistan.
Mohammad Zubair and Sri Krishna Singh in Munger. 128. When was Patna Lawn renamed as Gandhi
Swami Sahanjnand Saraswati established Bihar Maidan?
(a) During Quit India Movement
Provincial Kisan Sabha in 1929. (b) During Anti-Simon commission Rally
124. Bihar Socialist Party was born in 1934 with its (c) During Champaran Satyagraha
Secretary (d) On the eve of Independence
(a) Acharya Narendra Dev (e) None of the above/More than one of the
(b) Jayaprakash Narayan above
63th BPSC (Pre) Exam. 2017-18 227 YCT
Ans. (e) : Patna Lawn was renamed Gandhi Maidan in 133. The post of the Indian High Commission was
1948 after his assassination as a tribute. The statue of created by which Act?
Mahatma Gandhi in Gandhi Maidan is tallest bronze (a) The Indian Councils Act,1909
statue in the world. How ever BPSC had considered (b) The Government of India Act,1919
option (d) as correct to this question. (c) The Government of India Act,1935
(d) The Indian Independence Act,1947
129. Bombay Presidency Association was
(e) None of the above/More than one of the
established by above
(a) Ferozshah Mehta (b) K.T. Tailang
(c) W.C. Banerjee (d) Tayabjee Ans. (b) : The post of the Indian High Commission was
created by the Government of India Act,1919.
(e) None of the above/More than one of the
The first High Commissioner for India was Indian civil
above service officer Sir William Stevenson Meyer, and the
Ans. (e) : Bombay Presidency Association was formed first of Indian origin was Sir Dadiba Merwanji Dalal.
by Ferozshah Mehta, K. T Telang, Badruddin Tyabji 134. The Indian National Congress did not take part
and others in 1885. The reactionary policies of Lytton in which of the following Round Table
and the Ilbert Bill controversy led to the formation of Conferences ?
this association. (a) First (b) Second
130. Who established Imperial Cadet Corps? (c) third (d) Participated in all
(a) Lord Minto (b) Lord Curzon (e) None of the above/More than one of the
(c) Lord Lytton (d) Lord Ripon above
(e) None of the above/More than one of the Ans. (e) : The Round Table Conference were a series of
above three conference organised by the British Government
Ans. (b) : Imperial Codet Crops (ICC) was established to deliberate upon & bring about constitutional reforms
by Lord Curzon in 1901, with the aim of providing in British India during 1930-32. First Round Table
military education and special officer training to young conference was held in London in 1930, but INC did
men from royal aristocratic families in India. The ICC not participated in it. In March 1931, Mahatma Gandhi
was disbanded in 1917 owing to ambiguity in its & Lord Irwin entered into a Pact called Gandhi-Irwin
purpose. Pact, by which INC called of Civil Disobedience
131. The Declaration of Partition of Bengal was Movement & agreed to participate in Second Round
Table Conference held in London on 7th of September
done on
1931.
(a) 19th July, 1905 (b) 7th August, 1905
Third Round Table Conference was held in London on
(c) 15th August, 1905 (d) 16th October, 1905 17th November 1931 to considered the reports of
(e) None of the above/More than one of the various subcommittee appointed from time to time. The
above INC did not participate in it.
Ans. (a) : The decision to partition of Bengal was 135. Mahatma Gandhi presided over which Session
announced on July 19th, 1905 by Viceroy Lord Curzon. of the Indian National Congress?
The Partition took effect on 16th, October 1905. The (a) 1922 (b) 1924
idea was to divide Bengal into two part Western-Bengal (c) 1928 (d) 1930
& Eastern Bengal. The Western Bengal, Bihar & Orissa (e) None of the above/More than one of the
were retained under Bengal while the remaining above
partition of Bengal & Assam formed the Eastern Ans. (b) : The Belgaum session of 1924 was the only
Bengal. The day of partition was considered as the day session of INC Presided over by Mahatma Gandhi.
of Mourning throughout Bengal. Due to the relentless Gaya session of 1922 was presided over by Chittaranjan
anti partition movement 1911 the partition of Bengal Das.
was annulled by King George . Calcutta session of 1928 was presided over by M.L.
132. Who was given life punishment for the murder Nehru.
of Colonel Wyllie? Karachi session of 1930 was presided over by J.L.
(a) Madan Lal Dhingra (b) Udham Singh Nehru.
(c) Bhagat Singh (d) Manmathanath 136. Who was not the member of the Cabinet
(e) None of the above/More than one of the Mission?
above (a) Patrick-Lawerence (b) Wavell
Ans. (a) : Colonel Wyllie was assassinated by Madan (c) Stafford Cripps (d) A.V. Alexander
Lal Dhingra at the Imperial Institute while attending an (e) None of the above/More than one of the
event organized by the National Indian Assocation in above
1909 in London. Madan Lal Dhingra was a member of Ans. (b) : Cabinet Mission was sent to India in March
India House in London. 1946 . The sole purpose of the Cabinet Mission was to
Udham Singh killed Michel O' Dwyer, the former find out the ways and means for the peaceful transfer of
Lieutenant - Governor of the Punjab. power to India. The members of the Cabinet Mission
Bhagat Singh, Sukhadev & Rajguru were hanged for were: Lord Patrick-Lawrence, Secretary of State for
killing of British officer J.P. Saunders on 23rd March. India, Sir Stafford Cripps, President of the Board of
1931 Trade, and A.V Alexander, First Lord of Admiralty.
63th BPSC (Pre) Exam. 2017-18 228 YCT
137. Who wrote the pamphlet 'Now or Never'? 342p + 47 47
(a) Md.Jinnah (b) Md. Iqbal ⇒ 18p +
19 19
(c) Rahmat Ali (d) Khafi Khan
47
(e) None of the above/More than one of the = 9 ⇒ {2 ×19} + 9
above 19
‘so remainder = 9’
Ans. (c) : 'Now or Never' Pamphlet was written and
published by Choudhry Rahmat Ali in 1933. In this 2
142. A person who spends 66 % of his Income is
pamphlet the concept of a separate muslim state was not 3
only highlighted but the names. "Pakistan" was also able to save ` 1,200 per month. His monthly
proposed for it. expenses (in `) are
(a) 2,400 (b) 3,000
This Pamphlet was circulated to the delegate of the 3rd (c) 2,000 (d) 3,600
Round Table Conference in 1932. (e) 2,800
138. Who declared 'Sati' illegal? Ans. (a): Let his total monthly income = 3 unit
(a) Warren Hastings (b) William Bentinck 2
(c) Cornwallis (d) Curzon His expense = 3 × 66 %
3
(e) None of the above/More than one of the
2
above = 3 × = 2 unit
Ans. (b) : The Bengal Sati Regulation, 1829 had banned 3
the Sati Practice in entire jurisdiction of British India. It His saving = 3 – 2 = 1 unit
1 unit = Rs. 1200
was passed on 4th December 1829 by the Governor
His expense (2 unit) = Rs. 2400
General Lord William Bentinck.
143. A man gains 20% by selling an article for a
Raja Rammoham Roy was a vociferous campaigner certain price. If he sells it at double the price,
against sati. the percentage of profit will be
139. When was the Punjab Land Alienation Act (a) 140 (b) 200
Passed? (c) 100 (d) 160
(a) 1850 (b) 1895 (e) 120
(c) 1900 (d) 1905 Ans. (a): Let the C.P= x
(e) None of the above/More than one of the A man gains 20% by selling an article.
above 120x 6x
Then S.P= =
Ans. (c) : The Punjab Land Alienation Act was enacted 100 5
by the Government of Punjab in 1900. It was a piece of 2 × 6x 12x
Legislation introduced by the British Government with New S.P = =
5 5
the aim of limiting the transfer of land ownership in 12x 7x
Punjab Province. Thus profit = –x=
5 5
140. The Department of State was established under
7x
whose headship? Therefore, the profit percent is ×100 = 140%
(a) Mahatma Gandhi (b) J.L. Nehru 5x
(c) G.B. Pant (d) Sardar Patel 144. 10 women can complete a work in 7 days and
(e) None of the above/More than one of the 10 children take 14 days to complete the same
above work. How many days will 5 women and 10
children take to complete the work?
Ans. (d) : Department of State was formed in May 1947 (a) 6 (b) 5 (c) 3 (d) 7
to replace the Political Department of the British (e) 4
Government. It primary responsibility was to deal with Ans. (d): M1 D1 = M2 D2
the princely states and manage their relationship with 10 W × 7 = 10 C × 14
independent India. It was established under the W 2
leadership of first Deputy Prime Minister and Home =
C 1
Minister of India, Vallabh Bhai Patel. Total work = 10 × 2 × 7 = 140
141. A number when divided by 342 gives a 140
remainder 47. When the same number is Required time =
divided by 19, what would be the remainder? ( 5 × 2 + 10 × 1)
(a) 0 (b) 9 140
= = 7 days
(c) 18 (d) 8 20
(e) None of the above/More than one of the 145. How many numbers between 11 and 90 are
above divisible by 7 ?
Ans. (b) : Let number will be of form (a) 10 (b) 9
a = 342p+47 (c) 13 (d) 12
When ‘a’ will be divided by ‘19’ (e) 11
63th BPSC (Pre) Exam. 2017-18 229 YCT
Ans. (e): 14, 21, 28 ....... 84 Ans. (b): Dividend on Rs. 20
Let first term a = 14  9  9
last term l = 84 = Rs. × 20  = Rs.
 100  5
‘n’ be the number of terms Rs. 12 is an income on Rs. 100.
In arithmetic progression, last term l=a+(n–1)×d
∴ Rs. is an income on = Rs.
9 100 9 
84 = 14 + (n–1)7 ×  = = Rs.15
84 – 14 = (n–1) × 7 5  12 5 
70 149. The following pie-chart shows the spendings of
= n –1 a country on various sports during a
7 particular year. Study the chart carefully and
n – 1= 10 answer the question:
n= 10+1=11
n=11
Therefore, 11 numbers are there in between 11 to 90
which are divisible by 7.
1
146. A man can row 7 km per hour in still water.
2
1
If in a river running at 1 km an hour, it takes
2
him 50 minutes to row to a place and back, how
far off is the place?
(a) 3 km (b) 4 km How much percent less is spent on football than
(c) 2 km (d) 5 km that on cricket?
(e) 7 km 1
(a) 35 (b) 29
Ans. (a) Let the distance covered in one side =x 2
speed in still water = 7.5 km/h 1
speed of stream = 1.5 km/h (c) 32 (d) 31
2
50 1
Time of row up and down = hour. (e) 33
60 3
distance
Time = 81o − 54o
speed Ans. (e) = ×100
Thus, 81o
x x 50 27
+ = = × 100
( 7.5 –1.5 ) ( 7.5 + 1.5 ) 60 81
x x 5 5x 5 90 15 1
+ = ⇒ = ⇒ 5x = ⇒ x = ⇒ x=3 = 33 %
6 9 6 18 6 6 5 3
so place is 3 km far. 150. A invested `76,000 in a business. After few
147. A sum of money invested at compound interest months, B joined him with `57,000. At the end
amounts to ` 4,624 in 2 years and to `4,913 in 3 of the year, the total profit was divided between
years. The sum of money is ? them in the ratio 2:1 After how many months
(a) `4,240 (b) `4,280
(c) `4,096 (d) `4,346 did B join?
(e) `4,406 (a) 6 (b) 4
Ans. (c) S.I. on Rs. 4624 for 1 year= Rs. (4913–4624) (c) 3 (d) 8
(e) 5
 100 × 289  1
= Rs. 289 ∴Rate =  % = 6 % Ans.(b): Suppose B joined the business for x months.
 4624 ×1  4
Then,
Now,
2 Ratio of profit of A and B = (76000×12) : (57000 × x)
 25  Therefore,
x 1 +  = 4624
 400  (76000×12) : (57000×x ) = 2:1
76 × 12 2
⇒ x × × = 4624 ⇒ x =  4624 × × 
17 17 16 16
=
16 16  17 17  57x 1
⇒ x = 4096 76 × 12
148. A man buys `20 shares paying 9% dividend. x=
2 × 57
The man wants to have an interest of 12 % on
his money. The market value of each share is x=8
(a) `18 (b) `15 Hence B joined business for 8 months, or joined after
(c) `21 (d) `25 12–8= 4 months
(e) `20
63th BPSC (Pre) Exam. 2017-18 230 YCT
64th Bihar Public Service Commission
Preliminary Examination, 2018-19
GENERAL KNOWLEDGE & GENERAL SCIENCE
(Solved Paper with Detail Explanation)
1. Defence Minister of India Smt. Nirmala 5. Where did Foreign Minister Smt. Sushma
Sitharaman made a three-day visit to which Swaraj inaugurate 'Panini Language
nation in the first week of October 2018? laboratory' in August 2018?
(a) Russia (b) Kazakhstan (a) Sri Lanka (b) South Africa
(c) Ukraine (d) China
(c) Mauritius (d) Indonesia
(e) None of the above/More than one of the
above (e) None of the above/More than one of the
above
Ans. (b) In October 2018, erstwhile the Defence
Minister of India Smt. Nirmala Sitharaman made a Ans. (c) The 'Panini Language Laboratory' was
three-day visit to Kazakhstan. inaugurated at Mahatma Gandhi Institute (MGI) at Port
2. The Parliament of which of the following Louis, Capital of Mauritius. The lab will help the
nations dismissed the honorary citizenship of institute in teaching Indian languages in Mauritius. It
Aung San Suu Kyi in October 2018? was launched by Former External Affairs Minister
(a) USA (b) United Kingdom Sushma Swaraj on the sidelines of 11th world Hindi
(c) Norway (d) Canada conference in Mauritius.
(e) None of the above/More than none one of the
6. Which nation enacted 'Human Organ
above
Transplantation Law' for migrants from
Ans. (d) Aung San Suu Kyi, Myanmar’s civilian leader,
Indian origin in August 2018?
was stripped of her honorary Canadian citizenship in
October 2018 over her inaction on military violence (a) United Kingdom (b) Canada
against the country’s Rohingya Muslims. (c) USA (d) Singapore
(e) None of the above/More than one of the
3. Which of the following is not associated with
NAFTA? above
(a) Great Britain (b) Canada Ans. (a) United Kingdom enacted 'Human Organ
(c) Mexico (d) USA Transplantation Law' for migrants from Indian origin in
(e) None of the above/More than one of the August 2018.
above 7. Emmerson Mnangagwa was elected as the
Ans. (a) The 'NAFTA' Stands for, "North American President of which country in presidential
Free Trade Agreement" is an agreement signed by election held in November 2017?
United States, Canada and Mexico that create a trilateral (a) Uganda (b) Kenya
Trade bloc in North America. NAFTA came into effect (c) Zimbabwe (d) Ghana
on January 1,1994. (e) None of the above/More than one of the
4. With which nation, India has signed an above
agreement 'Mobilize Your City' in September Ans. (c) Emmerson Mnangagwa is a Zimbabwean
2018? revolutionary and politician, who is serving as the third
(a) Sweden (b) Germany
and current president of Zimbabwe since November,
(c) Japan (d) France
2017 .
(e) None of the above/More than one of the
above 8. On 20thJuly, 2018, which nation ended
Ans. (d) India and France has signed an implementation emergency after two years?
agreement on 'Mobilise Your City' (MYC) to support (a) Iraq (b) Syria
three pilot cities as-Nagpur, Kochi and Ahmedabad for (c) Turkey (d) Yemen
reducing Green House Gas (GHG) emissions related to (e) None of the above/More than one of the
urban transport of Housing and Urban Affairs. Mobilise above
Your City (MYC) is part of an international initiative
Ans. (c): The emergency was imposed in Turkey after a
which is supported by the French and the German
Governments and was launched at COP-21 (Conference failed 2016 coup and it came to an end on 20th July
of Parties) of UNFCCC metting in December, 2015. 2018.

64th BPSC (Pre) Exam. 2018-19 231 YCT


9. Which of the following nations declared Ans. (e): Atal Bhujal Yojana is a Central Sector
emergency due to polio outbreak in June 2018? Scheme for facilitating sustainable ground water
(a) Papua New Guinea (b) Fiji management with an outlay of INR 6000 crore, out of
(c) Philippines (d) Mali this, 3,000 crore will be a loan from the World Bank
(e) None of the above/More than one of the and INR 3000 crore as contribution from the
above Government of India.
Ans. (a): A National Emergency was declared by the 14. According to the report of IIT, Kharagpur
Government of Papua New Guinea, on 26th June 2018. study group, continuous low rainfall for how
many years was the cause of fall of the Indus
10. For how many times, India won the Cricket
Valley Civilization?
Asia Cup on 29th September, 2018 defeating
(a) 600 years (b) 700 years
Bangladesh? (c) 800 years (d) 900 years
(a) Six times (b) Seven times (e) None of the above/More than one of the
(c) Eight times (d) Nine times above
(e) None of the above/More than one of the Ans. (d): In 2018, an IIT Kharagpur Study claimed that
above the continuous low or negligible rainfall Which is
Ans. (b): The Asia Cup is One-Day International drought like Phase that lasted over 900 years was the
cricket tournament involving Asian teams. In 2018 cause of fall of the India Valley Civilization.
Indian team won the championship 7th time. 15. According to Indian Forest Status Report
11. Which of the following is the correct order of released in February 2018, how much area of
medals won by India in the 18th Asian Games, India is covered by forests?
2018 (Jakarta)? (a) 23.00% (b) 23.40%
Gold Silver Bronze Total (c) 24.00% (d) 24.40%
(a) 15 21 33 69 (e) None of the above/More than one of the
(b) 15 32 22 69 above
(c) 15 30 24 69 Ans. (d) According to the 16th Indian Forest Status
(d) 15 24 30 69 Report (IFSR), Which was published in February 2018,
(e) None of the above/More than one of the as:
above The total forest cover is 7,08,273 sq. km. which is
Ans. (d): The 18th Asian Games were held in Jakarta 21.54% of the total geographical area of the country.
and Palembang of Indonesia. India's final tally at the • Forest and Tree cover combined is 8,02,088 sq km. or
18th Asian Games were: 24.39% of the total geographical area.
• There was an increase of 8,02I sq. km. in the total
Gold Silver Bronze Total
forest and tree cover of the country. compared to the
15 24 30 69 previous assessment in 2015.
• India ranked 10th globally with a total of 24.40% of
12. Which is the first Indian airline company that land area under forest and tree.
used biofuel as ATF in August 2018? • Madhya Pradesh has the highest area under forest
(a) Jet Airways (b) Vistara • Lakshadweep has the highest percentage of the total
(c) Air India (d) Spice Jet geographical area.
(e) None of the above/More than one of the NOTE: In IFSR 2017 only forest covered 21.54% and
above forest and Tree covered 24.39% of total Geographical
Ans. (d): Spice Jet is the first airline company that area. According to BPSC (64th) correct.
operated biofuel as ATF. The mixture was powered by a Source: Forest survey of India Report.
blend of 75% ATF and 25% biojet fuel. The fuel was a 16. In January 2018, which Indian State allowed
mixture of Jatropha seeds and ATF (Aviation Turbine people to establish a humanitarian relationship
Fuel). The test flight was conducted between with trees, i.e., brother-sister?
Deharadun and New Delhi. (a) Assam (b) Sikkim
(c) Nagaland (d) Manipur
13. In June 2018, how much amount the World (e) None of the above/More than one of the
Bank has approved for 'Atal Bhujal Yojana' an above
ambitious scheme of India? Ans. (b): The Government of Sikkim came up with a
(a) ` 5,000 crore (b) ` 6,000 crore unique war of preserving trees by encouraging people to
(c) ` 7,000 crore (d) ` 8,000 crore forge a relationship of brotherhood or sisterhood with
(e) None of the above/More than one of the trees through a practice locally known as Mitini.
above
64th BPSC (Pre) Exam. 2018-19 232 YCT
17. Which district achieved the first position as per Ans. (b): JIMEX-2018, the 3rd edition of Japan-India
Swachh Survekshan Grameen, 2018? Maritime Exercise (JIMEX) Commenced at
(a) Gaya-Bihar Visakhapatnam, Andhra Pradesh.
(b) Tuticorin-Tamil Nadu "JIMEX is a biennially conducted naval exercise
(c) Satara-Maharashtra between the Indian Navy and Japanese Maritime Self
(d) Meerut-UP Defence Force (JMSDF). The JIMEX series is designed
(e) None of the above/More than one of the to create a better interoperability, ensure better
above understanding and share best practices between the two
Ans. (c): As per 'Swachh Survekshan Grameen, 2018 navies. Other Military Exercise with Japan:
Satara district of Maharashtra achieved the first → SHINYUV Maitri
position. → Dharma Guardian
Haryana was ranked as the best state while Uttar 21. In which city/State was air pollution control
Pradesh was rewarded for maximum citizen's system 'VAYU' installed in September- 2018?
participation. (a) Chennai (b) Amritsar
18. Which scheme was recognized as world's (c) Delhi (d) Varanasi
largest insurance scheme launched in India in (e) None of the above/More than one of the
September 2018? above
(a) Pradhan Mantri Jan Aarogya Yojana Ans. (a): ⇒ 'WAYU' which stands for' Wind
(b) Pradhan Mantri Jan Swasthya Yojana Augmentation Purifying Unit' is an air pollution control
(c) Pradhan Mantri Fasal Bima Yojana device and it was developed by National Environmental
(d) Pradhan Mantri Jan Dhan Yojana Engineering Research Institute (NEERI-CSIR).
(e) None of the above/More than one of the It was installed in September 2018 for traffic junctions
above at ITO Intersection and Mukarba Chowk in New Delhi.
Ans. (a) The 'Pradhan Mantri Jan Aargya Yojana' was This question has been deleted by the 64th BPSC official
recognized as world's largest insurance scheme Answer key (Final), because in the question
launched in India in September, 2018. The Scheme is abbreviation given 'VAYU' instead of 'WAYU'.
widely known as Ayushman Bharat, and this scheme is 22. Under which of the following Chief Minister
set to cover some 500 million poor people. PM-JAY Shri Nitish Kumar launched more than 750
offers a sum insured 5 lakh per family per year. This development schemes at Begusarai and
cover will take care of almost all secondary care and Khagaria districts on 6th January, 2018?
most of tertiary care procedures. (a) Vikas Yatra
19. In which of the following institutes was the first (b) Vikas Survekshan Yatra
Indian Railway University established in (c) Nav-Srijan Yojana, 2018
September 2018? (d) Vikas Samiksha Yatra
(a) Indian National Rail Academy-Vadodara (e) None of the above/More than one of the
(b) Indian Railway School-Pune above
(c) Indian National Rail Research Institute- Ans. (d): Chief Minister of Bihar Mr. Nitish Kumar
Mysore launched more than 750 development scheme under
(d) Indian Rail Coach Institute Kapurthala Vikas Samiksha Yatra, at Begusarai and Khagaria
(e) None of the above/More than one of the district on 6th January 2018.
above 23. In which city, Deputy Chief Minister Shri
Ans. (a): The National Rail and Transportation Institute Sushil Modi inaugurated Bihar's first 'Start-up
(NRTI), a university in Vadodara, Gujarat, is India's Hub' on 16th March, 2018?
first rail university and the third in the world after (a) Motihari (b) Munger
Russia and China. This Institute is located in the (c) Patna (d) Arrah
National Academy of Indian Railway (NAIR). (e) None of the above/More than one of the
20. JIMEX 2018 relates to above
(a) Japan-India Joint Military Exhibition Ans. (c) Sushil Kumar Modi, Deputy Chief Minister, of
(b) Japan-India Maritime Exercise Bihar inaugurated Bihar's first 'Start-up Hub' in Patna,
(c) Japan-India Joint Motor-vehicle Exhibition on 16th March, 2018. The new start-up hub aimed to
(d) Japan-India Joint Missile Programme provide work station and office space equipped with
(e) None of the above/More than one of the free of cost "Plug and play" facility to 31 Startup
above companies.

64th BPSC (Pre) Exam. 2018-19 233 YCT


24. On 22nd March, 2018 who unveiled the book, Ans. (a): According to the report open signal, the
Neel Ke Dhabbe on the occasion of 106th Bihar credible firm on 4G availability in 20 of India's largest
Day in Patna? cities, Stated that Patna has the highest LTE availability
(a) Vice President Shri Venkaiah Naidu in the country with 92.61% followed by Kanpur,
(b) Prime Minister Shri Narendra Modi Allahabad, Kolkata. Bhopal and Jaipur respectively.
(c) Home Minister Shri Rajnath Singh • LTE stands for- ' Long Term Evolution.
(d) Speaker Smt. Sumitra Mahajan 29. In which city of Bihar, Sulabh International
(e) None of the above/More than one of the launched cheapest drinking water project
above 'Sulabh Jal?
Ans. (a) The 13th and former vice president of India. (a) Patna (b) Bhagalpur
Shri Venkaiah Naidu unveiled the book, "Neel ke (c) Darbhanga (d) Purnea
Dhabbe" at the inaugural session of the 106th Bihar (e) None of the above/More than one of the
Diwas celebration in Patna. above
25. The capacity of India's most powerful electric Ans. (c) The Sulabh International launched the cheapest
locomotive engine made in Madhepura drinking water project 'Sulabh Jal'. It is an innovative
Locomotive Factory is project that converts contaminated pond water and well
(a) 9000 HP (b) 10000 HP water into bottled pure drinking water.
(c) 11000 HP (d) 12000 HP The founder of Sulabh International is Dr. Bindeshwar
(e) None of the above/More than one of the Pathak.
above 30. Which of the following is not true about Bihar's
Ans. (d): First 12000 HP Made in India Locomotive Mukhyamantri Kanya Utthan Yojana?
manufactured by Madhepura Electric Locomotive (a) Proposed annual expenditure of the scheme is
Factory situated in Bihar was put into operation in May ` 7,221 crore
2020. (b) Education, Health and Social Welfare trio
26. On 10th April, 2018, Prime Minister Shri departments ran the scheme
Narendra Modi addressed 'Swachhagrahis (c) Every girl child will receive total ` 60,000
National convention' in from her birth to graduation level education
(a) Motihari (b) Patna (d) At the time of a girl child's birth, her family
(c) Kishanganj (d) Katihar receives ` 2,000
(e) None of the above/More than one of the (e) None of the above/More than one of the
above above
Ans. (a): The Prime Minister, Shri Narendra Modi Ans. (c): Mukhyamantri Kanya Utthan Yojana aims to
addressed the 'Swachhagrahis National convention at inspire girls towards education and will provide them
Motihari on 10th April. 2018. The event was organized, financial help right from their birth till they get
as part of the centenary celebrations of the Champaran graduated through which the girls may get a better life.
Satyagrah (1917) led by Mahatma Gandhi. On birth of a girl child Rs. 2000; On 1st Vaccination
Rs. 1000; On passing intermediate or +2 Exam Rs.
27. In which airport, NDMA conducted a mock
10000; On completing Graduation degree Rs. 25000.
drill based on CBRN emergency in April 2018?
Hence, a total of 40,000 will be given to families who
(a) IGI-New Delhi (b) LNJP-Patna
have a girl child.
(c) CSI-Mumbai (d) DABH-Indore
(e) None of the above/More than one of the 31. The earliest evidence of animal domestication
above and agriculture in India has come from
(a) Anjira
Ans. (b): A Five day training programme aimed at (b) Damb Sadat
enhancing the preparedness of Airport Emergency (c) Kili Gul Muhammad
Handlers to respond to CBRN emergencies at the (d) Mehrgarh
airports was conducted at the Lok Nayak Jayaprakash (e) None of the above/More than one of the
Airport in Patna in April 2018. above
28. According to a report, what is the rank of Ans. (d): The earliest evidence of animal domestication
Patna city among 20 cities as per availability of and agriculture in India comes from the ancient site of
4G signals between December 2017 to Mehrgarh. Research shows that people here lived in
February 2018? houses and were involved in hunting, domestication of
(a) First (b) Second animals and cultivation of cereals like barley and
(c) Nineteenth (d) Twentieth Wheat, Mehrgarh is an archaeological site situated at
(e) None of the above/More than one of the Baluchistan in Pakistan.
above
64th BPSC (Pre) Exam. 2018-19 234 YCT
32. After the Mahaparinirvana of Lord Buddha, 36. Who wrote Periplus of the Erythraean Sea?
the first Buddhist council was held at (a) Ctesias (b) Pliny
(a) Rajgriha (Rajgir) (b) Gaya (c) Ptolemy (d) Strabo
(c) Pataliputra (d) Vaishali (e) None of the above/More than one of the
(e) None of the above/More than one of the above
above
Ans. (e): The 'Periplus of the Erythraean Sea' is a
Ans. (a) The four Buddhist Council were as follows: Greco-Roman Periplus written in Koine Greek that
Buddhist Place Patronage Presidency describes navigation and trading opportunities from
council Roman Egyptian ports to different ports of the world. A
First Rajgriha Ajatashatru Mahakashyapa periplus is a kind of logbook recording sailing
Second Vaishali Kalasoka Sabakami itineraries and commercial, political, and ethnological
Third Pataliputra Ashoka Moggaliputta details about the port visited. There has been a debate
Tissa about the authorships of this book. some historian refers
Fourth Kashmir Kanishka Vasumitra William H. Schoff as the author.
33. Who among the following first issued gold
37. The earliest description of Bakhtiyar Khalji's
coins on a large scale?
invasion of Bihar is in
(a) Pushyamitra Shunga
(a) Taarikh-I Hind
(b) Menander
(b) Tabaqat-I Nasiri
(c) Vima Kadphises
(d) Gautamiputra Satakarni (c) Taj-ul Maasir
(e) None of the above/More than one of the (d) Tarikh-I Mubarak Shahi
above (e) None of the above/More than one of the
Ans. (c) Vima Kadphises of Kushan dynasty first above
introduced gold coins on large scale, in addition to the Ans.(b): The earliest reference of the Afghan military
existing copper and silver coinage. Many Indian god Chief Bakhityar Khalji's invasion of Bihar in 1200 CE
and goddesses depicted on Kushana coins besides is found in the Tabaqat-I-Nasiri written by Minhaj-I-
Persian and Greek deities. The coins also bear the figure Siraj Juzjani. Khalji invaded Bihar and destroyed the
of Shiva standing beside a bull. Nalanda, Vikramshila and Odantapuri Universities.
34. Who among the following was the first to 38. Which Sultan demanded half the crops on land
explain that the rotation of the earth on its own after measurement as revenue?
axis accounts for the daily rising and setting of (a) Iltutmish
the sun? (b) Balban
(a) Aryabhata (b) Bhaskara (c) Alauddin Khalji
(c) Brahmagupta (d) Varahamihira
(d) Muhammad bin Tughluq
(e) None of the above/More than one of the
(e) None of the above/More than one of the
above
above
Ans. (a): Aryabhatta, India's famous Mathematician
and astronomer was the first to explain that the rotation Ans. (c) Alauddin Khalji was sultan of Delhi from 1296
of the earth on its own axis accounts for the daily rising to 1316. He measured the cultivable land & fixed the
and setting of the sun. land revenue. The market control department was also
etablished under a minister called Diwan-i-Riyasat. The
35. The ruler of Suvarnabhumi, who founded a
sultan's administrators measured the land and kept
Buddhist monastery at Nalanda and requested
careful accounts. There were three type of Taxes:
Devapala through his ambassador to grant five
1. Kharaj- amounting to about 50% (half the crops) of
villages for its maintenance, was
the peasant's produce.
(a) Dharanindra
2. On Cattle
(b) Sangramadhananjaya
3. On House
(c) Balaputradeva
(d) Chudamanivarmana 39. Sheikh Bahauddin Zakaria belonged to which
(e) None of the above/More than one of the sect?
above (a) Suhrawardi Silsilah (b) Rishi Silsilah
(c) Chisti Silsilah (d) Firdausi Silsilah
Ans. (c): Balputradeva was a king of Sailendra
(e) None of the above/More than one of the
dynasty. Balputradeva ruler of Suvarnabhumi sent an
ambassador to Devapala asking for a grant of five above
villages for the construction and maintenance of a Ans. (a) Hazrat Sheikh Bahauddin Zakaria Suhrawardi
monastery at Nalanda. was the leading Sufi Saint of his time in North Western
Devapala was the most powerful ruler of Pala dynasty. India and belong to well known Suhrawardi Silsilah
the founder of Pala dynasty was Gopal in 750 AD. originating in Baghdad.

64th BPSC (Pre) Exam. 2018-19 235 YCT


40. Who among the following rulers divided his 44. Who wrote the book, Poverty and un-British
troops into unit of two hundred, two hundred Rule in India?
fifty and five hundred? (a) R.C. Dutt (b) Dadabhai Naoroji
(a) Bahlul Lodi (b) Sikandar Shah (c) Bipin Chandra Pal (d) Lajpat Rai
(c) Sher Shah (d) Islam Shah (e) None of the above/More than one of the
(e) None of the above/More than one of the above
above Ans. (b): Dadabhai Naoroji was a politician and a
Ans. (d): Islam Shah was the successor of his father, writer. He was the author of the two famous books,
‘Poverty and Un-British Rule in India’ and ‘Poverty of
Shershah Suri. He divided his troops into units of 50,
India’. He was also founder of the Indian National
200, 250 and 500. The army was organized into larger
Congress and was elected thrice as its President.
divisions of 5000, 10000 and 20000 soldiers. Islam
45. When were the rights of the tenants on land in
Shah also added to the defence of the empire by
Bengal and Bihar recognized by the Bengal
building 5 more fortresses, viz, Shergarh, Islamgarh, Tenancy Act?
Rashidgarh, Firozgarh and Mankot. (a) 1868 (b) 1881
Hence fourth option is correct but 64 BPSC marked as (c) 1885 (d) 1893
(e) as answer. (e) None of the above/More than one of the
41. Who stated that five thousand to twenty above
thousand men worked for forty years to build Ans. (c): The Bengal Tenancy Act. of 1885 was passed
the palace of forty pillars at Allahabad in by the Bengal legislative Council on the
Akbar's time? recommendations of the Rent Commission (1880). The
(a) Manucci Act defined rights and obligations of Intermediate
(b) Tavernier tenancies and raiyati tenancies on land in Bihar and
(c) William Finch Bengal. The permanent Settlement gave right to the
Zamindars but was Silent about the rights of tenants.
(d) Abdul Hamid Lahori
The peasants stood up for their rights and Bengal saw
(e) None of the above/More than one of the several revolts such as Pabna Revolt (1873-76).
above
46. When did Bihar become a separate province in
Ans. (c) William Finch was an English traveller during India?
the reign of Jahangir stated that 5000 to 20000 men (a) 1897 (b) 1905
worked from 40 years to build the palace of 40 pillars at (c) 1907 (d) 1912
Allahabad in Akbar's time. (e) None of the above/More than one of the
42. Which Sultan was reputed to be 'Akbar of above
Kashmir? Ans. (d): On 22 March 1912 Bihar and Orissa carved
(a) Sultan Shamsuddin Shah out as separate provinces through the Bengal, Bihar and
(b) Sultan Qutbuddin Orissa and Assam Laws act 1912.
(c) Sultan Sikandar Bihar Diwas is observed every year on March 22,
marking the formation of the state of the Bihar. Bihar
(d) Sultan Zainul Abidin
Diwas was started and celebrated on large scale by
(e) None of the above/More than one of the
Bihar government in tenure of Nitish Kumar.
above
47. Who invited Gandhiji Champaran?
Ans. (d) Sultan Zainul Abidin, the eighth Sultan of (a) Rajendra Prasad
Kashmir is known as 'Akbar of Kashmir. He is one of (b) Raj Kumar Shukla
the very few rulers who ruled for about 50 years. He (c) Mazharul Huq
also look several steps for the welfare of his subjects. (d) Krishna Sahay
He had liberal religious policy, promoted literature, art (e) None of the above/More than one of the
and architecture due to his peaceful reign he was called above
the 'Akbar of Kashmir'. Ans. (b): Raj Kumar Shukla invited Gandhiji to
43. Who began the Faraizi Movement? Champaran to fight against the injustice meted out to
(a) Haji Shariatullah (b) Syed Ahmed the peasants in Champaran.
(c) Salimullah (d) M.A. Jinnah 48. Who raised the Flag of independence at the
(e) None of the above/More than one of the congress session of 1929?
above (a) Maulana Muhammad Ali
(b) Pandit Jawaharlal Nehru
Ans. (a) Faraizi Movement was started and led by Haji
(c) Vallabhbhai Patel
Shariatullah in 1818. The movement extolled Muslims
(d) Subhas Chandra Bose
of Bengal to give up un-Islamic practices and act upon (e) None of the above/More than one of the
their duties as Muslims. above
64th BPSC (Pre) Exam. 2018-19 236 YCT
Ans. (b): The Congress session began in Lahore in 53. In which of the following areas, Mediterranean
1929 and Pt. Nehru was the president of the session. In climate does not prevail?
this session the Indian tricolor Flag was hoisted by Pt. (a) Central Chile (b) Cape Town
Jawharlal Nehru on the bank of the Ravi River. (c) Adelaide (d) Pampas
49. When was the Bihar Socialist Party formed? (e) None of the above/More than one of the
(a) 1912 (b) 1927 above
(c) 1931 (d) 1934 Ans. (d): Mediterranean climate is found between 30-
(e) None of the above/More than one of the 40 degrees N-S Latitudes and get its name from climate
above of the Mediterranean sea.
Ans. (b) Bihar Socialist Party was formed in July 1931 The Climate of the Pampas is generally temperate, and
by Ganga Sharan Sinha, Rambriksh Benipuri, Phulan the Cape Town has a Mediterranean-Style Climate, also
Prasad Verma and Ramanand Mishra within congress climate of Central Chile and Adelaide has a
by ideologies of Marxist Leninist socialists. Mediterranean Climate.
50. Who among the following was the leader of the 54. The country, which is the largest silk producer
first Congress Ministry in Bihar? in the world, is
(a) Anugrah Narayan Sinha (a) India (b) China
(b) Abdul Bari (c) Brazil (d) Japan
(c) Jayaprakash Narayan (e) None of the above/More than one of the
(d) Shri Krishna Sinha above
(e) None of the above/More than one of the Ans. (b): The major silk producing countries in the
above world are, China, India, Uzbekistan, Brazil and the
Ans. (d) : Sri Krishan Sinha known as Shri Babu and major silk consumer of the world are, USA. Italy,
Japan.
Bihar Kesari was the first Chief Minister of Bihar
Karnataka is the largest silk producing state in India.
(1946-61). He was the first Congress Minister in Bihar.
There are five major type of silk of commercial
51. Which one of the following is a landlocked importance obtained from different species of
country? silkworms. There are Mulberry, Oak Tasar & Tropical
(a) Belgium (b) Hungary Tasar, Muga and Eri.
(c) Romania (d) Ukraine There are 5 types of silk-:
(e) None of the above/More than one of the • Mulberry
above • Oak Tasar
Ans. (b): A landlocked country is a country that is not • Tropical Tasar
connected to an ocean or sea or does not have any • Munga
coastline. Hungary in a landlocked country is bordered • Eri
by Slovakia, Ukraine, Romania, Serbia, Croatia Karnataka is the largest producer is India.
Slovenia and Austria. Romania and Ukraine is bordered
55. Which one among the following countries of
by black sea. Belgium is bordered by north sea. the world, except United States of America, is
52. Which one among the following is the largest the largest crude oil producer?
island in area? (a) Russia (b) China
(a) Borneo (b) Great Britain (c) Saudi Arabia (d) Canada
(c) Madagascar (d) Sumatra (e) None of the above/More than one of the
(e) None of the above/More than one of the above
above Ans. (c): Saudi Arabia is one of the major oil producer
countries. As of 2019 data, USA is the largest oil
Ans. (a): Among the following Borneo has the largest
producer with 17 million bpd (barrels per day) an 18%
island in area of the entire global output.
Sr Island Area (Sq.km) Saudi Arabia occupied second place on this list with
1. Green land 2,166,086 national output of around 11.8 million bpd in 2019 with
2. New Guinea 786,000 12.4% of the global output.
3. Borneo 743,330 56. Which one of the following is not a part of the
4. Madagascar 592,800 Meghalaya Plateau?
(a) Bhuban Hills (b) Garo Hills
5. Sumatra 473,605
(c) Khasi Hills (d) Jaintia Hills
6. Honshu 230,966 (e) None of the above/More than one of the
Great Britain 229,979 above

64th BPSC (Pre) Exam. 2018-19 237 YCT


Ans. (a): The Meghalaya plateau is further sub-divided Ans. (b): NITI Aayog which stands for 'National
into three Hills, as: The Garo, Khasi and Jaintia Hills, Institution for Transforming India, formed in 1st January
named after the tribal groups inhabitating this region. 2015 to replace the Planning Commission.
Bhuban hills, are located in Assam. The Bhuban Hills, NITI Aayog is the premier policy 'Think Tank' of the
located in the south east of Barak valley connected with Government of India. Providing both directional and
Manipur and Mizoram. policy inputs, NITI Aayog also provides relevant
57. The Pass, which is situated at the highest technical advice to the center and states. Prime Minister
elevation, is serves as the Ex-officio chairman of NITI Aayog.
(a) Zoji La (b) Rohtang 61. After Independence, in which year were the
(c) Nathu La (d) Khyber Indian States reorganized on the linguistic
(e) None of the above/More than one of the basis?
above (a) 1947 (b) 1951
Ans. (c): Nathu La pass is located at an elevation of (c) 1956 (d) 2000
4310 in Sikkim. (e) None of the above/More than one of the
• Zoji La pass is located at an elevation of 3528 m above
above the sea level in the Kargil district of Ladakh. Ans. (c) After Independence, The state organization
• Rohtang pass is located at an altitude of 3978 m. Act. 1956 reformed the boundaries of Indian states and
above sea level in Himachal Pradesh. territories, systematizing them on Language base.
• Khyber pass is a high mountain pass at an elevation of 62. Which one among the following states of India
1070 m above the sea level in Pakistan. is called 'Sugar Bowl'?
58. Karewas soils, which are useful for cultivation (a) Uttar Pradesh (b) Maharashtra
of Zafran (a local variety of saffron), are found (c) Bihar (d) Haryana
in (e) None of the above/More than one of the
(a) Kashmir Himalaya (b) Garhwal Himalaya above
(c) Nepal Himalaya (d) Eastern Himalaya Ans. (a) Muzaffarnagar, located in the northern part of
(e) None of the above/More than one of the Uttar Pradesh is popularly known as ‘The Sugar Bowl
above of India. Uttar Pradesh is referred as the Sugar-Bowl of
Ans. (a): Karewas soils, are found in Kashmir India as it is the largest producer of Sugarcane in India.
Himalaya and it is useful for cultivation of Zafran. Cuba is known as the 'Sugar-Bowl' of world.
Zafran is a local variety of saffron in Kashmir valley. 63. In which one of the following minerals, India
The Karewas soil are also important for the cultivation leads in production in the world?
(a) Sheet mica (b) Copper
of Almond, walnut, apple, etc.
(c) Gypsum (d) Iron ore
59. As per 2011 census, the urban population (e) None of the above/More than one of the
percentage to total population of India was above
about Ans. (c) Mica is known as a shiny Silicate mineral the
(a) 21 (b) 31 most prominent industry to use mica is as a thermal or
(c) 36 (d) 40 Electrical insulator in the electrical and electronic
(e) None of the above/More than one of the industry.
above Andhra Pradesh is the largest mica producing state of
Ans. (b): As per 2011, census: India which leads 93% of total production. India has a
• Total urban population% – 31.16% nearly-monopoly in the production of mica in the world
• Tamilnadu is the most Urbanized state– 48.4% and largest exporter of Mica also.
• Himanchal Pradesh is the most rural state – 89.97% 64. Bihar as an Indian State was formed in
(a) 1911 (b) 1912
60. In Government of India's NITI Aayog, which is
(c) 1936 (d) 2000
an economic policy-making think tank, what is
(e) None of the above/More than one of the
the full form of NITI?
above
(a) National Internal Trade Information
(b) National Institution for Transforming India Ans. (c) Bihar and Orissa was carved out of Bengal
(c) National Integrated Treaty Institute province as a separate provinces on 22nd March 1912.
(d) National Intellectual Training Institute Bihar and Orissa become separate provinces on 1 April
(e) None of the above/More than one of the 1936. Thus, Bihar Become on Indian state exclusively
above in 1936.

64th BPSC (Pre) Exam. 2018-19 238 YCT


65. Which one of the following parts of Bihar is Ans. (a) According to the 2011 census, Bihar is the least
geologically older in age relatively? literate state. Hence, Bihar ranks first in terms of the
(a) Rohtas Plateau lowest literacy rate as per the 2011 census with 69.83%
(b) North-Western Hills of total literacy, male literacy stands at 70.32% and
(c) Kharagpur Hills female literacy is at 53.57% The National Literacy rate
(d) North Ganga Plains in 2011 is 74.04% as total and 82.14% male and
(e) None of the above/More than one of the
65.46% for females, Kerala retains its top position at
above
93.91% of literacy rate.
Ans. (a) The Rohtas Plateau are geologically older in
69. What is the rank of Bihar in terms of
age relatively to other geological division.
production of jute among the states of India?
Physiographically Bihar is divided into three regions :
1. Shivalik Region (a) First (b) Second
2. Bihar Plain (c) Third (d) Fourth
3. Southern plateau Region (e) None of the above/More than one of the
The correct answer is ‘Rohtas Plateau’. above
Rohtas Plateau lies in South western part of Bihar, Ans. (b) : The Ganges delta contributes more than 80%
Significant waterfalls along Rohtas Plateau are Devdari of Jute production in the world. West Bengal is the
Falls (58 m) in the Karmanasa River , Telharkund Falls largest producer of Jute and Bihar is second. Purnia is
(80 m) on the Suara West River, Suara Falls (120 m) on the largest jute producing district of Bihar.
the Suara East River, Durgawaati Falls (80 m) on the 70. Which type of industries has potentials and
Durgavati River etc. prospects in Bihar?
The Oraon or Kurukh tribe resides in this Plateau. (a) Oil refinery
66. By how many districts of Bihar was Patna (b) Forest-based industry
district bounded till July 2018? (c) Sand-mining industry
(a) 7 (b) 8 (d) Agro-based industry
(c) 9 (d) 10 (e) None of the above/More than one of the
(e) None of the above/More than one of the
above
above
Ans. (d) : Bihar is the third most populous state in India
Ans. (c): Patna is the capital of Bihar state and it is
with majority of its population depending on
surrounded by the 9 districts namely:
agriculture. According to the 2011 census. 88.71%
In North– Saran, Vaishali, Samastipur.
In South– Jehanabad, Lakhisarai, Nalanda and Arwal. population of Bihar lives in rural areas were agriculture
In East – Begusarai. is the main occupation. Bihar is endowed with fertile
In West – Bhojpur. gangetic alluvial soil categories associated with
Bihar consist of total 38 districts and 9 divisions. different agro-Climatic zones.
67. Most of the area in Bihar is covered by 71. A Judge of the Supreme Court may resign his
(a) Mountainus soil (b) Alluvial soil office by writing to
(c) Regur soil (d) Terai soil (a) The President
(e) None of the above/More than one of the (b) The Prime Minister
above (c) The Law Minister
Ans. (b): Most of the area in Bihar is covered by (d) The Attorney General of India
Alluvial soil. There are three major types of soil in (e) None of the above/More than one of the
Bhiar. above
A. Gangetic Alluvium: The Gangetic Alluvium covers
the plain of Bihar (Both new as well as old.) Ans. (a) Article 124(2) Provides that, a Judge of
B. Piedmont Swamp Soil: found in northwestern part of Supreme Court may by writing under his hand
west Champaran district. addressed to the President, resign his office.
C. Terai Soil: found in northern part of the state along 72. Who was the first Chairman of the Backward
with the border of Nepal. Class Commission?
68. Among the States of India, Bihar's rank in (a) Jagjivan Ram
terms of lowest literacy rate (2011 Census) is (b) Kaka Saheb kalelkar
(a) First (b) Second (c) B.D. Sharma
(c) Third (d) Fourth (d) B.R. Ambedkar
(e) None of the above/More than one of the (e) None of the above/More than one of the
above above
64th BPSC (Pre) Exam. 2018-19 239 YCT
Ans.(b) : The First Backward Classes Commission was 76. In which Article of the Constitution of India
setup in 1953 under the chairmanship of Kaka Kalelkar. was the provision for reservation of scheduled
Its report, however, was never implemented. castes in the lok Sabha made?
Second Backward Classes Commission was setup 1979 (a) Article 330 (b) Article 331
under B.P. Mandal. The commission decided to give (c) Article 332 (d) Article 333
27% reservation in Jobs and education to the OBC’s. In (e) None of the above/More than one of the
1990, the Union Government decided to implement the above
proposals of the Commission. Ans. (a) Article 330 of the Indian constitution provides
The Constitution (102nd Amendment act), 2018 made for the reservation of seats in the Lok Sabha for the
NCBC Constitutional and inserted a new Article 338B scheduled castes and scheduled tribes. The number of
in the Constitution. It was placed under the Ministry of seats reserved in any state of Union Territories for such
the Social Justice an Empowerment. caste and tribes will be made on the population basis.
73. The Chairman of the state public service 77. The President
commission is appointed by (a) Is not a part of the Parliament
(a) The Chairman of the Union Public Service (b) Is a part of the Parliament
Commission (c) Is a part of the Parliament and sits in the
(b) The President of India Parliament
(c) The Governor of the State (d) Can vote in the Parliament
(d) The Chief Minister (e) None of the above/More than one of the
(e) None of the above/More than one of the above
above
Ans. (b): The President of India is a constituent part of
Ans. (c) The Chairman and other members of the State
Parliament, he does not sit or participate in the
Public Service Commission are appointed by the
discussions in either of the Houses. There are certain
governor of the state to hold office for a term of six
constitutional functions which he has to perform with
years or till the age of 62 years, whichever is earlier.
respect to Parliament. The President summons and
The chairman or any other member of SPSC shall only
prorogues the two Houses of Parliament from time to
be removed from his/her office by order of the President
of India. time. While the Rajya Sabha is a continuing body, the
Article 316 related to appointment and term of office of power to dissolve the Lok Sabha vests in the President.
members of UPSC as well as State Service Commission His assent is essential for a Bill passed by both Houses
and Article 317 are related to removal and suspension of Parliament. When the Parliament is not in Session
of a member of both the UPSC or SPSC. and he is satisfied that circumstances exist which render
74. In order to be chosen a member of the Lok it necessary for him to take immediate action, the
Sabha, a person must not be less than the age President can promulgate Ordinances having the same
of force and effect as laws passed by parliament.
(a) 18 years (b) 21 years 78. Which one of the following Articles directs the
(c) 25 years (d) 30 years state Governments to organize the Panchayats?
(e) None of the above/More than one of the (a) Article 33 (b) Article 40
above (c) Article 48 (d) Article 50
Ans. (c) According to the Article 84 (b) of the Indian (e) None of the above/More than one of the
constitution, the minimum age for becoming a candidate above
for the Lok Sabha election shall be 25 years. Ans. (b): Article 40 directs the state government to
75. The Governor of a State is appointed by organize village Panchayats and endow them with such
powers and authority as may be necessary to enable
(a) The Prime Minister
them to function as unit of self-government.
(b) The Chief Justice of India
79. In which year did the state Jharkhand come
(c) The President
into existence?
(d) The Chief Justice of the High Court
(a) 1998 (b) 1999
(e) None of the above/More than one of the (c) 2000 (d) 2001
above (e) None of the above/More than one of the
Ans. (c) Article 155: The governor of a State shall be above
appointed by the President by warrant under his hand Ans. (c): Jharkhand was carved out of Bihar as a
and seal. Article 156: The governor shall hold office separate state on 15 Nonvember 2000. after the
during the pleasure of the President. parliament passed the Bihar reorganization Act. 2000.

64th BPSC (Pre) Exam. 2018-19 240 YCT


80. In which one of the following years was Ans. (e): The 'Make in India' initiative was launched in
Sarkaria Commission, which was empowered September 2014 with the objective of promoting India
to recommend changes in Center-state as an important investment destination India a global
relations, submitted its report? hub in manufacturing - design and innovation. This
(a) 1983 (b) 1984 initiative aims to create a conductive environment for
(c) 1985 (d) 1987 investment, development of Modern and efficient
(e) None of the above/More than one of the infrastructure, opening up new sector for foreign
above investment and forging a partnership between
Ans. (e) Sarkaria Commission was constituted in 1983 government and industry through a positive mind set.
by Central Government. It’s mandate was to review the 85. Which one of the following is not an initiative
working of existing arrangements between the Union for industrial development?
and States in regard to powers, functions and (a) Make in India
responsibilities in all spheres and recommend such (b) Ease of Doing Business
changes or other measures as may be appropriate. (c) Start-up India
The Commission submitted its report in January 1988. (d) Digital India
81. Under which one of the following Articles is (e) None of the above/More than one of the
the formation of Finance Commission laid above
down? Ans. (d): Make in India , Ease of Doing Business, Start
(a) Article 280 (b) Article 269 up India leads to Industrial Development.
(c) Article 268 (d) Article 265 Make in India– to promote India as a manufacturing and
(e) None of the above/More than one of the innovation hub.
above
Start up India- it is a flagship initiative launched by the
Ans. (a) Article 280 of the constitution provides that, government to build a strong ecosystem for nurturing
President of India can constitute the Finance
innovation and startups in the country.
Commission and give its recommendations on
distribution of tax between the State Governments and Digital India:- It aims to transform the country into a
Union Government and also amongst the State digitally empowered society and knowledge economy.
themselves. Ease of Doing Business: It is an annual report
82. The Attorney General of India is appointed by published by World Bank.
(a) The Law Minister 86. PURA (Providing Urban Amenities to Rural
(b) The President of India Areas) model was advocated by
(c) The Speaker of the Lok sabha (a) A.P.J. Abdul Kalam
(d) The Prime Minister (b) Manmohan Singh
(e) None of the above/More than one of the (c) Lal Krishan Advani
above (d) Rajiv Gandhi
Ans. (b): Article 76 of the constitution of India provides (e) None of the above/More than one of the
for the appointment of Attorney General of India, the above
highest law officer in country. The Attorney General is
Ans. (a) Provision of Urban Amenities in Rural Areas
appointed by the president.
(PURA) is a strategy for rural development in India
83. During Twelfth five-Year plan, which one of
given by former President of India, Dr. A.P.J. Abdul
the following average annual growth rate
targets was envisaged for agriculture and allied Kalam. The objective of the scheme is to provide urban
sector? amenities and livelihood opportunities in rural areas to
(a) 3.0 percent (b) 3.5 percent bridge to rural – urban divide , thereby reducing
(c) 4.0 percent (d) 4.5 percent migration from rural to urban areas.
(e) None of the above/More than one of the 87. As per Census 2011, the female literacy rate in
above India was
Ans. (c) The 12th five year plan (2012-17) envisaged 4% (a) 60.0 percent (b) 63.0 percent
average annual growth for agriculture and allied sector. (c) 65.5 percent (d) 68.5 percent
84. The Government policy 'Make in India' aims at (e) None of the above/More than one of the
(a) Removal of bureaucratic sloth above
(b) Elimination of red tapism Ans. (c) According to the census 2011
(c) Reduction in cost of manufacturing
Literacy rate in the country – 74.04%
(d) Making the competitive product
(e) None of the above/More than one of the Literacy rate of males – 82.14 %
above Literacy rate of females – 65.46%

64th BPSC (Pre) Exam. 2018-19 241 YCT


88. Who among the following has given the idea of 92. Which one of the following was not included in
Self-Help Groups as an effective tool for the intended objectives of the Union Budget,
poverty alleviation? 2017-18?
(a) Amartya Sen (b) Md. Yunus (a) Transform India (b) Clean India
(c) S. Chakravarti (d) Venkaiah Naidu (c) Educate India (d) Energize India
(e) None of the above/More than one of the (e) None of the above/More than one of the
above above
Ans. (b): Md. Yunus is a Bangladeshi Social Ans. (c) The agenda for the union Budget 2017-18:
entrepreneur, banker. He was awarded the Nobel Peace "Transform, Energise and clean India".
Prize in 2008 for founding Gramin Bank and pioneering The union Budget 2017-18 contains three major refoms:
concepts of microcredit and microfinance . He gave the • The merger of railway budget with general budget
idea of Self-Help groups as effective tool for poverty • Advancement of Date of presentation-
alleviation. • Done away with plan and non-plan expenditure.
89. At present, who is the Governor of the Reserve Hence, Educate India was not included in the intended
Bank of India? objectives of the union budget 2017-18.
(a) Urjit Patel (b) Raghuram Rajan 93. Which one of the following most appropriately
(c) Shanta Kumar (d) Lalita D. Gupte describes the nature of Green Revolution of
(e) None of the above/More than one of the late sixties of 20th century?
above (a) Intensive cultivation of green vegetable
Ans. (e) Shaktikanta Das, is a current (25th) Governor of (b) Intensive agriculture district programme
Reserve Bank of India. (c) High-yielding varieties programme
Reserve Bank of India is India's Central Bank and it (d) Seed-Fertilizer-Water technology
controls the monetary policy. The RBI was established (e) None of the above/More than one of the
based on the Hilton Young Commission' on 1st April, above
1935, and RBI was nationalized in 1949. Ans. (d) : The Green Revolution in India was an effort
90. Which one of the following is the pathway to to increase Agricultural production with the help of
increase productivity in agriculture? industrial agriculture technologies such as:
(a) Efficient irrigation (b) Quality seeds • High-yielding varieties of seeds.
(c) Use of pesticides (d) Use of fertilizers • Increased the use of fertilizers
(e) None of the above/More than one of the • Controlled water-Supply, involving irrigation.
above Norman Borlaug is known as father of Green
Ans. (e) Investments in irrigation, combined with the Revolution. M.S. Swaminathan is known as the father
better quality seeds can increase the productivity, of “Indian Green Revolution".
among regions as for as irrigation is concerned. Some 94. Who among the following is the CEO of the
part have more water supply as compared to other, The NITI Aayog?
(a) Amitabh kant
state should endeavour to increase the irrigation
(b) S.S. Mundra
methods. Unscientific use of pesticides and fertilizers
(c) Cyrus Mistry
can decrease the productivity of soil. (d) Soumya Kanti Ghosh
91. Which one of the following is the objective of (e) None of the above/More than one of the
MGNREGA? above
(a) To build assets Ans. (a) NITI Aayog stands for. "National Institute for
(b) To encourage micro irrigation Transforming India." The prime minister of India is the
(c) Water management chairman of the NITI Aayog.
(d) To enhance rural income When question was asked CEO of NITI Aayog was
Amitabh Kant. Presently CEO of NITI Aayog is Shri
(e) None of the above/More than one of the
Parameswaran Iyer.
above
95. Which one of the following states has the
Ans. (e) The main objective of Mahatma Gandhi highest density of population according to the
National Rural Employment Guarantee Act Census of India, 2011?
(MGNREGA), 2005 is to provide at least 100 days of (a) Kerala (b) Haryana
wage employment in a financial year to every rural (c) Bihar (d) Uttar Pradesh
household whose adult member volunteers to do (e) None of the above/More than one of the
unskilled manual work. above

64th BPSC (Pre) Exam. 2018-19 242 YCT


Ans. (c) Population density of mentioned States as per 100. What was the economic growth rate of the state
census 2011: of Bihar during the year 2016-17?
Total population density of India – 382 (a) 6.3 percent (b) 7.3 percent
Population density of Bihar – 1106 (c) 8.3 percent (d) 9.3 percent
Population density of Kerala – 860 (e) None of the above/More than one of the
Population density of Haryana – 573 above
Population density of Uttar Pradesh 829. Ans. (e) Bihar recorded 8.9% GDP growth rate in 2016-
96. What is the sex ratio in Bihar state as per the 17 which was higher than national GDP growth rate of
Census of India, 2011? 8.3%.
(a) 893 (b) 961 101. Which of the following was not a reason for
(c) 918 (d) 925 making the sepoys of the East India Company
(e) None of the above/More than one of the rebellious?
above (a) The efforts of the officers of the Company to
Ans. (c) As per census 2011. The sex ratio of Bihar is spread Christianity
918 for each 1000 male. The top five districts of Bihar (b) The order to the sepoys to travel on ships
in terms of sex ratio are: (c) The stoppage of Bhatta
(i) GopalGanj – (1021) (d) The inefficiency of the officers
(ii) Siwan – (988) (e) None of the above/More than one of the
(iii) Saran – (949) above
(iv) Kishanganj – (946) Ans. (d): The inefficiency of the officers is not reason
(v) Nawada – (939) for making the sepoys of the East India company
The lowest sex ratio of Bihar is in Munger district – rebellious.
(879) Reason for making the sepoys of the East India
97. 'Bihar Economic Survey' For the first time, Company rebellious are:
was published for the Financial Year → Christian missionaries
(a) 2004-05 (b) 2006-07 → The Religious Disabilities Act of 1850.
(c) 2008-09 (d) 2009-10 → General Service Enlistment Act, 1856, which made
(e) None of the above/More than one of the it compulsory for the sepoys to cross the seas,
above whenever required.
Ans. (b) : Bihar Economic Survey was first published → The foreign service allowance (Bhatta) of Indian
for the 2006-07 financial year. soldiers for service in Sindh was also stopped.
98. In Bihar, during April-June 2018, which sector 102. About Mughal Emperor Bahandur Shah Zafar,
has attracted the highest FDI equity inflow? it was said that his empire extended from
(a) Service sector (a) Chandni Chowk to Palam
(b) Steel industry (b) Delhi to Bihar
(c) Processing industry in agriculture (c) Peshawar to Bihar
(d) Cement industry (d) Peshawar to Varanasi
(e) None of the above/More than one of the (e) None of the above/More than one of the
above above
Ans. (a) : Highest FDI equity inflow in Bihar during Ans.(a): Bahadur Shah Zafar (1775-1862) was the last
April-June 2018 was by Service Sector. Education Mughal emperor. The authority of the Mughal empire
received the highest allocation of Rs. 25,251 Crore in started declining after the death of Aurangzeb in 1707.
2017-18, which was 11% higher than the revised Thus, by beginning of 19th Century, the Mughal ruler
estimated of 2016-17. was only a nominal head and his authority was limited
99. What is the percentage of population in Bihar only to the walled city of Shahjahanabad (old Delhi).
State employed in agriculture sector in 2017- 103. Who among the following did not rebel against
18? the British East India Company's control over
(a) 65 (b) 67 them?
(c) 68 (d) 70 (a) Raja of Vizianagaram
(e) None of the above/More than one of the (b) Nizam of Hyderabad
above (c) Polygar of Tamil Nadu
Ans. (e) As per Bihar Government data 76% of the (d) Dewan Velu Thampi of Travancore
work force in Bihar depend on agricultural and allied (e) None of the above/More than one of the
activities for their livelihood. above
64th BPSC (Pre) Exam. 2018-19 243 YCT
Ans. (b): The Nizam of Hyderabad always remained a 107. Which of the following was not a 'nationalist'
subsidiary of the British East India Company and he newspaper?
never rebelled against them. Rest of all the rebelled (a) The Hindu (b) Bengalee
(c) The Mahratta (d) The Times of India
against the British East India Company's control over (e) None of the above/More than one of the
them. above
104. The Indigo Rebellion was about Ans. (d) : 'The Times of India' was not a ‘nationalist’
(a) The peasant not wanting to grow indigo but newspaper. Originally paper was established, as ‘The
being forced to Bombay Times’ and Journal of Commerce to serve the
(b) The peasant wanting to grow indigo but being British residents of Western India. It’s name was
forced not to changed to The ‘Times of India’ in 1861.
(c) The peasant not wanting to grow Indigo but • The Hindu → It was started as a weekly in 1878 and
become a daily newspaper in 1889, founded by, G.
being forced to grow at a price that was
Subramania Iyer.
unacceptable
• Mahratta → It was an English newspaper started by
(d) A protest movement that carried an indigo- Bal Gangdhar Tilak.
coloured flag 108. Identify the correct combination from the
(e) None of the above/More than one of the options given below for Prarthana Samaj,
above Young India, Lokhitvadi, Satyashodhak Samaj,
Ans. (c) The Indigo Rebellion was a revolt led by the Rehnumai Mazdayasan Sabha.
peasants of Bengal in 1859-60. The peasants were (a) Gopal Hari Deshmukh, Atmaram Pandurang,
forced to grow indigo in place of food crops by the Mohandas Karamchand Gandhi, Jyotiba
planters as its cultivation was more profitable for the Phule, Naoroji Furdonji
planters. The peasants were also not paid properly for (b) Atmaram Pandurang, Mohandas Karamchand
the produce and the system was very exploitative. Gandhi, Gopal Hari Deshmukh, Jyotiba
Nil Darpan, the play written by Dinabandhu Mitra in Phule, Naoroji Furdonji
1858-59, portrayed the farmer's situation in Bengal. (c) Atmaram Pandurang, Jyotiba Phule,
Mohandas Karamchand Gandhi, Gopal Hari
105. The immediate cause of the Deccan Riots of
Deshukh, Naoroji Furdonji
1875 was (d) Naoroji Furdonji, Atmaram Pandurang,
(a) The shadow of a famine Mohandas Karamchand Gandhi, Gopal Hari
(b) High interest rates charged by Mahajans Deshmukh, Jyotiba Phule,
(c) High land revenue rates (e) None of the above/More than one of the
(d) Protest against imposition of religious above
reforms Ans. (b): The correct combination is:
(e) None of the above/More than one of the (i) Prarthna Samaj → Atmaram Pandurang
above (ii) Young India → Mahatama Gandhi
Ans. (b): The immediate cause of the Deccan Riots in (iii) Lokhitvadi → Gopal Hari Deshmukh
1875 was the high interest rates charged by the (iv) Satyashodhak Samaj → Jyotiba Phule
Mahajans. (v) Rehnumai Mazdayasan Sabha → Naoroji Furdonji
106. Rajani Palme Dutt said, "They both fought and 109. From the options given below, find the correct
collaborated with imperialism" Which political combination of the names of Editors of the
nationalist newspapers The Hindu, Kesari,
grouping was he referring to?
Bengalee, Hindustani, Sudharak.
(a) The India National Congress
(a) Surendra Nath Banerjea, G Subramanina Iyer,
(b) The Communist Party of India Bal Gangadhar Tilak, Ganga Prasad Varma,
(c) The Swaraj Party Gopal Krishna Gokhale
(d) Hindustan Socialist Republican Association (b) Bal Gangadhar Tilak, G. Subramania Iyer,
(e) None of the above/More than one of the Surendra Nath Banerjee, Ganga Prasad
above Varma, Gopal Krishan Gokhale
Ans. (a) Rajani Palme Dutt was a journalist affiliated to (c) G. Subramainia Iyer, Bal Gangadhar Tilak,
the Communist Party of Great Britain (CPGB). He Ganga Prasad Varma, Gopal Krishna
Gokhale, Surendra Nath Banerjee
founded and edited a monthly magazine, Labour. In his
(d) G. Subramania Iyer, Bal Gangadhar Tilak,
book, he mentioned that the Congress was instituted by
Surendra Nath Banerjee, Ganga Prasad
the British Government. Further he noted that the Varma, Gopal Krishna Gokhale
Congress both fought and collaborated with (e) None of the above/More than one of the
Imperialism. above

64th BPSC (Pre) Exam. 2018-19 244 YCT


Ans. (d) : The correct combination of newspaper and 113. Where did Kunwar Singh join the Rebellion of
their editor's 1857 against the British?
(i) The Hindu – G. Subramania Iyer (a) Arrah (b) Patna
(ii) Kesari – Bal Gangadhar Tilak (c) Bettiah (d) Varanasi
(iii) Bengalee – Surendra Nath Banerjee (e) None of the above/More than one of the
(iv) Hindustani – Ganga Prasad Verma
above
(v) Sudharak – Gopal Krishna Gokhale
110. Bal Gangadhar Tilak came to be known as Ans. (b): Maharaja Kunwar Singh (1777-1858) of
'Lokmanya Tilak' when Jagdishpur led the Revolt of 1857 in Bihar. The sepoys
(a) He become a popular teacher of Danapur cantonment revolted against the British and
(b) He started a popular newspaper Kunwar Singh assumed command of the soldiers on
(c) The government accused him in the Rand 25th July 1857 at Patna. On 27th July, he occupied the
Murder Case district headquarters of Arrah.
(d) He started the Shivaji and Ganpati festivals His Army was defeated by Major Vincent Eyre, he died
(e) None of the above/More than one of the
on 26th April 1858 after sustaining injuries during war.
above
Note :- As per official answer key of BPSC option B is
Ans. (e) Bal Gangadhar Tilak was popularly known as
"Lokmanya" which means accepted by the people as correct but according to numerous sources, Kunwar
their leader. There is an ambiguity about the exact time Singh joined the Rebellion of 1857 against the British in
when he began to be referred as, "Lokmanya' Arrah of Bihar.
The official answer key of 64th BPSC mentions (E) as 114. In which territory did tribal's rebel against the
correct Answer. British?
111. How old was Bhagat Singh when he threw a (a) Bihar (b) Punjab
bomb in the assembly in protest against the (c) Sindh (d) Kathiawar
government?
(e) None of the above/More than one of the
(a) 21 years
(b) A little more than 21 years above
(c) 25 yaers Ans. (e) During the 19th and 20th centuries, tribal's rebel
(d) A little more than 25 years against the Britishers.
(e) None of the above/More than one of the The kols in chhotanagpur plateau (Jharkhand) rebelled
above in (1831-32.)
Ans. (b): Bhagat Singh was born in 1907 in Lyallpur Santhals in Jharkhand in 1855. Bastar (Chhattisgarh)
district of Punjab. On 8th April 1929, Bhagat Singh and
Rebellion in 1910.
Batukeshwar Dutt, threw two bombs into the assembly
chamber from its public gallery while it was in session 115. "So always with the assent and often to the
and were arrested. dictates of the chamber of commerce, the
Hence, at that time Bhagat Singh was a little more than Government of India is carried on, and this is
21 years. the 'White Man's Burden". Who said this?
112. The Jallianwala Bagh Massacre happened in (a) Bankim Chandra Chatterjee
the context of which Gandhian Satyagraha? (b) Mahatma Gnadhi
(a) Swadeshi Satyagraha (c) Sachchidananda Sinha
(b) Rowlatt Satyagraha (d) Rajendra Prasad
(c) Bardoli Satyagraha (e) None of the above/More than one of the
above
(d) Individual Satyagraha
Ans. (c) Sachchidanand, Sinha was critique of British
(e) None of the above/More than one of the
economic policies followed in India. The above
above mentioned phrase was written by him in his publication
Ans. (b) : The Jallianwala Bagh Massacre took place on “Indian Nation”.
13th April 1919 at Jallianwala Bagh, Amritsar, when the 116. Which of the following was the first satyagraha
police on the order of acting Brigadier-General R.E.H. of Mahatma Gnadhi in India?
Dyer open fire on a peaceful gathering of unarmed (a) Ahmedabad (b) Bardoli
people. The massacre happened during protest by (c) Champaran (d) Individual
peoples against the arrest of leaders Dr. Satya Pal and (e) None of the above/More than one of the
Dr. Saifuddin Kitchlew. above

64th BPSC (Pre) Exam. 2018-19 245 YCT


Ans. (c) Champaran Satyagraha was the first satyagrah Ans. (e) The Kisan Sabha movement in Bihar was
initiated by Mahatma Gandhi in India. Mahatma Gandhi started under leadership of Swami Sahajananda. He
returned to India in 1915 from South Africa. He formed Bihar Kisan Sabha in 1929. Later in 1939
travelled for one year around the country on the Yadunandan Sharma also joined Kisan Sabha.
suggestion of his political Guru, Gopal Krisha Gokhale. 121. The length of the longest pole that can be
Raj Kumar Shukla persuaded Gandhi to come to
placed in a room 12 meters long, 9 meters wide
Champaran. The peasants of Champaran were forced to
and 8 meters high is
grow Indigo on their cultivable lands and were paid
(a) 864 meters (b) 10 meters
very less for products.
(c) 17 meters (d) 43 meters
117. Gandhiji's Non-Cooperation movement urged
(e) None of the above/More than one of the
people to avoid alcohol. This resulted in a
above
serious loss of revenue for the government. The
government of a province circulated a list of Ans. (c) The length of the longest pole that can be
prominent individuals who drank alcohol as a placed in a room-
device to persuade people to start drinking diagonal of room (cuboid) = l 2 + b 2 + h 2
again. Name the province. here, l = length of the room = 12m
(a) Andhra Pradesh (b) Bihar
b = Breadth of the room = 9m
(c) Bombay (d) Gujarat
h = Height of the room = 8m
(e) None of the above/More than one of the
diagonal = (12 ) + ( 9 ) + ( 8 )
2 2 2
above
Ans. (b): The Government of Bihar and Orissa
circulated a list of prominent individual great men in = 289
history who drank liquor, to regain the lost revenue = 17 m.
during non-cooperation movement . Hence, the longest pole that can be placed in a room is
118. During Salt Satyagraha, in Bihar, in addition 17 meter.
to making salt, the people chose to oppose the 122. A train of length 150 meters, moving towards
government by opposing which tax? north direction at a speed of 144 km/hr, can
(a) Chowkidari (b) Haathi cross a 250 meters long bridge in
(c) Development (d) Malba (a) 20 seconds (b) 100 seconds
(e) None of the above/More than one of the (c) 45 seconds (d) 10 seconds
above (e) 28 seconds
Ans. (a) During Salt Satyagraha, there was widespread Ans. (d) Length of the Train = 150m
Civil Disobedience carried on by the people. Apart from Length of Bridge = 250m
the Salt tax, unpopular tax laws were being defied like
 144 × 5 
the Forest laws, Chowkidar tax, Land tax etc. Speed of train = 144km/hr =  
119. The elected Government of Bihar resigned in  18 
February 1938 for which reason? = 40 m/s.
(a) The participation of India in war Time taken by train to cross the bridge.
(b) Satyagraha against the British at the call of length of train + length of bridge
Gandhi t=
Speed of train
(c) High taxes being imposed by the British
150 + 250
(d) For the release of political prisoners = = 10 seconds
(e) None of the above/More than one of the 40
above 123. How many prime numbers are there between 1
Ans. (d): When Congress formed ministry in 1937, the and 50?
ministry demanded the release of political prisoners. (a) 17 (b) 15
The government of Bihar resigned to put pressure on (c) 14 (d) 16
the viceroy. The viceroy later conceded the demands (e) None of the above
and prisoners were released. Ans. (b) The prime numbers between I and 50 are:
120. Who was connected with the Bihar Kisan 2, 3, 5, 7, 11, 13, 17, 19, 23, 29, 31, 37, 41, 43 and 47.
Sabha? ∴ Required number of prime numbers = 15
(a) Swami Sahajananda
2
(b) Karyananda Sharma 124. If x = - , then 9x2 – 3x – 11 is equal to
(c) Rahul Sankrityayan 3
(d) Yadunandan Sharma (a) –13 (b) 13
(e) None of the above/More than one of the (c) –5 (d) –17
(e) 17
above
64th BPSC (Pre) Exam. 2018-19 246 YCT
−2 Ans. (b)
Ans. (c) Here, x = and the given equation is- 1 1 1
3 x+ = 1, y + = 1, z + = ?
9x2 – 3x – 11. y z x
So put the value of x in given equation
1 1 1
 −2 
2
 −2  Here, x + = 1, ⇒ = 1 − x,so y =
∴ 9 ×   − 3   − 11 y y 1− x
 3   3  1
4 2 Similarly, y + = 1
= 9 × + 3 × −11 = 4 + 2 – 11 = – 5 z
9 3 1
125. In an examination, every candidate took Hindi so = 1 − y
or History or both, 66% took Hindi and 59% z
took History. The total number of candidates 1
= 1−
was 3000. How many candidates took both 1− x
Hindi and History? −x
(a) 500 (b) 750 =
(c) 542 (d) 738 1− x
(e) 830 1− x
z=
Ans. (b) Let A and B the sets of candidates who opt for −x
Hindi and History respectively. −1
The- z= +1
x
1
z + =1
x
128. The average age of 4 sisters is 7 years. If we
add the age of mother, then the average
increases by 6 years. Find the age of the
n (A ∪ B) = 3000 mother.
3000 × 66 (a) 46 years (b) 39 years
n (A) = (c) 37 years (d) 47 years
100
= 1980 (e) 57 years
3000 × 59 sum of Numbers
n (B) = = 1770 Ans. (c) Average =
100 Total Number
∴ n (A ∩ B) = n (A) + n (B) – n (A ∪ B)
= 1980 +1770 – 3000= 750 Sum of ages of 4 sisters = 4 × 7 = 28
∴ The number of candidates who took both Hindi and Sum of ages of 4 sisters and their mother = 5 × 13 = 65
History = 750. age of mother = 65 – 28 = 37
0.00001225 129. If 3 x + 8 = 27 2x +1 then the value of x is
126. If x = , then the value of x is (a) 9 (b) 1
0.00005329 (c) –1 (d) 10
35 525 (e) –10
(a) (b)
73 933 Ans. (b) Here, 3x+8 = 272x+1
205 135 3x+8 = (33)2x+1 = 36x+3
(c) (d) ∴ x + 8 = 6x + 3
403 233 8 – 3 = 6x – x
(e) None of the above
5x = 5
Ans. (a) x=1
0.00001225 10
1
x= 130. If S = ∑ (2n + ), then S is
0.00005329 n=1 2
1225 35 1
= = (a) 55 (b) 56
5329 73 2
(c) 111 (d) 115
∴ (Here, the value of 1225 = 35 and 5329 = 73 )
1
1 1 (e) 100
127. If x + = 1 and y + = 1, then the value of 2
y z 10
 1
1 Ans. (d) Here, S = ∑  2n +  .
z + is n =1  2
x 10
 1 10 10
1
(a) x – y then, S = ∑  2n +  = ∑ 2n + ∑ =
(b) 1 n =1  2  n =1 n =1 2
(c) Not known/Not countable 1
(d) 2 2 (1 + 2 +……+10) + (1 + 1 + ......10 times )
(e) None of the above 2

64th BPSC (Pre) Exam. 2018-19 247 YCT


2 ×10 ×11 1 136. What is triglyceride?
⇒ + ×10 (a) Protein (b) Carbohydrate
2 2 (c) Fat (d) Mineral

= 110 + 5 = 115 ∵1 + 2 + ..... + n = n
( n + 1)  (e) None of the above/More than one of the
 above
 2 
Ans. (c) : Triglycerides are a type of fat. They are most
131. Which of the following has pH value 7? common type of fat found in body.
(a) Pure water (b) Neutral solution 137. Electroencephalogram (EEG) is used in
(c) Basic solution (d) Acidic solution monitoring
(e) None of the above/More than one of the (a) Heart (b) Liver
above (c) Pancreas (d) Brain
Ans. (b): pH scale indicates the acid or basic nature of a (e) None of the above/More than one of the
solution. On the pH scale, a pH value of less than 7 is above
acidic in nature and pH value of greater than 7 and up to Ans. (d): An Electro Encephalogram (EEG) is a test
14 is basic in nature. used to evaluate the electrical activity in the brain. It is
The pH value exact 7 is neutral in nature (i.e pure water used to diagnose sleep disorders.
& neutral solution). The pH of blood is 7.35 to 7.45. Some other instruments:
132. Which of the following elements does not ECG (Electro Cardio Gram)– To check the human
contain neutrons? heart's rhythm.
(a) Oxygen (b) Nitrogen MRI (Magnetic Resonance Imaging)– Medical imaging
(c) Hydrogen (d) Copper technique used in radiology to form pictures of the
(e) None of the above/More than one of the anatomy and physiological processes of body.
above 138. Body temperature is regulated by
Ans. (c) : The Hydrogen (H) atom does not have any (a) Thalamus (b) Hypothalamus
neutrons in its nucleus. It has only one electron and one (c) Cerebellum (d) Medulla
proton. (e) None of the above/More than one of the
Neutrons are the particles in an atom that has a neutral above
charge. Ans. (b): Human body temperature is regulated by the
Oxygen has– 8 electrons (2, 6) part of the brain, Hypothalamus. The Hypothalamus
8 protons checks our current temperature and compares it with the
8 neutrons normal temperature of 37ºC. If our temperature is too
Nitrogen has– 7 electrons (2, 5) low, the hypothalamus makes sure that the body
7 protons generates and maintain heat.
7 neutrons
Copper has– 29 electrons (2, 8, 18, 1) 139. Choose the correct statement.
29 protons (a) Wavelength of red light is less than violet
35 neutrons. light.
(b) Wavelength of red light is more than violet
133. Angstrom is a unit of light.
(a) Wavelength (b) Energy (c) Wavelength of violet light is more than green
(c) Frequency (d) Velocity light.
(e) None of the above/More than one of the (d) Wavelength of violet light is more than
above yellow light.
Ans. (a) : Angstrom is a unit of length convenient on the (e) None of the above/More than one of the
atomic scale. It is denoted by Å, 1 Å = 10-10 meter. above
Wavelength is measured in Angstrom. Ans. (b): Red colour has the most wavelength and the
134. Frequency is measured in least frequency and violet colour has the least
(a) Hertz (b) Meter/second wavelength and the most frequency.
(c) Radian (d) Watt The sequence of colours of the rainbow in terms of
(e) None of the above/More than one of the increasing wavelength and decreasing frequency:
above ⇒ VIBGYOR (Violet→ Indigo → Blue → Green →
Ans. (a) : Frequency is the number of occurrences of a Yellow → Orange → Red).
repeating event per unit of time. Its SI unit is Hertz. 140. The value of 40 degrees Celsius in Fahrenheit
135. Who discovered DNA? scale is
(a) James Watson and Francis Crick (a) 104 ºF (b) 100 ºF
(b) Gregor Mendel (c) 102 ºF (d) 75 ºF
(c) Johannsen (e) None of the above/More than one of the
(d) Hargovind Khorana above
(e) None of the above/More than one of the Ans. (a) : The S.I unit of temperature is Kelvin. Degree
above and Fahrenheit are also used as unit of temperature.
Ans. (e) : Francis Crick and James Watson are most The formula conversion between Fahrenheit and Celsius
often associated with discovery of DNA. However C ( F − 32 )
= , putting C = 40, we get 104ºF.
DNA was discovered by Friedrich Miescher. 5 9
64th BPSC (Pre) Exam. 2018-19 248 YCT
141. The unit of electric power is 146. Einstein got the Nobel Prize for
(a) Ampere (b) Volt (a) Relativity
(c) Coulomb (d) Watt (b) Bose-Einstein condensation
(e) None of the above/More than one of the (c) Mass-energy equivalence
above (d) Photoelectric effect
Ans. (d) (e) None of the above/More than one of the
• Ampere is the unit of electric current. above
• Volt is the unit of electric potential. Ans. (d): Albert Einstein received Nobel Prize in the
• Coulomb is the unit of electric charge. 1921, for his discovery of law of the" photoelectric
• Watt is the unit of electric power. effects". He is also famous for his theory of relativity.
142. In electrical motor Relativity showed us that matter and energy are just two
(a) Heat is converted into electrical energy different forms of the same thing which he expressed
(b) Electrical energy converted into heat as-
(c) Electrical energy is converted into mechanical E = MC2, here E = energy
energy M = mass
(d) Mechanical energy is converted into electrical C = speed of light.
energy 147. The powerhouse of cell is
(e) None of the above/More than one of the (a) Cell wall (b) Mitochondrion
above (c) Ribosome (d) Nucleus
Ans. (c) The electric motor is a machine which converts (e) None of the above/More than one of the
electrical energy into mechanical energy. Electric above
motors can be powered by alternating Current (AC) Ans. (b): Mitochondria is called as power house of the
sources, such as grid power or electrical generators. cell. Mitochondria produces energy by cellular
143. Human body contains respiration that is taking in glucose and oxygen.
(a) About 70% water (b) 20%–30% water 148. AIDS is caused by
(c) 10%–20% water (d) 30%–40% water (a) Water (b) Bacteria
(e) None of the above/More than one of the (c) Virus (d) Fungus
above (e) None of the above/More than one of the
Ans. (a) : On an average the human body contains 60- above
70% of water. Water in the body is stored in the Ans. (c) : Acquired Immuno Deficiency Syndrome
intercellular fluid, fluid within the cells and in the (AIDS) is a viral disease caused by the Human
extracellular fluids, fluids outside the cells. Immunodeficiency Virus (HIV). HIV is a retrovirus that
144. The device to measure electric current is primarily infects components of the human immune
(a) Voltmeter (b) Ammeter system, making a person more vulnerable to other
(c) Voltammeter (d) Potentiometer infections and diseases.
(e) None of the above/More than one of the 149. Which instrument is used to measure
above humidity?
Ans. (b) : The electric current in a circuit is measured (a) Hydrometer (b) Hygrometer
by Ammeter. The S.I unit of electric current is the (c) Pyrometer (d) Lactometer
Ampere. (e) None of the above/More than one of the
Other devices: above
Device Measured Ans. (b): Humidity is the measure of amount of
Voltmeter Potential Difference moisture in the air. A Hygrometer is an instrument used
Voltameter Electric Charge to measure relative humidity.
Potentiometer Potential Difference • Hydrometer – Hydrometer measures a ratio of specific
145. The main component of greenhouse gases in gravity of density of a liquid to the density of water.
(a) Carbon dioxide (b) Methane • Pyrometer – Pyrometer measures the temperature.
(c) Nitrous oxide (d) Ozone
(e) None of the above/More than one of the • Lactometer – Lactometer measures the specific
above gravity of Milk, it is used to find out the amount of
Ans. (e) : Green house gases are essential for the water in the milk.
survival of humans and millions of other living things. 150. What is the unit of pressure?
Due to industrializations, deforestation and large scale- (a) Newton/sq. meter (b) Newton-meter
agriculture, quantities of green house gases in the (c) Newton (d) Newton/meter
atmosphere have risen to threatening levels. Large (e) None of the above/More than one of the
quantity of these gases leads to global warming. above
The Main Green house gases are: Ans. (a) : Pressure is the amount of force applied
→ Water vapour perpendicular to the surface of an object per unit area.
→ Carbon dioxide (CO2) The SI unit of Pressure is Pascal (P)
→ Methane F
→ Ozone (O3) So = P = ⇒ 1 Pascal = Newton/meter2
→ Nitrous oxide (N2O) A
→ Chlorofluro carbon (CFC) [Here, F = Force, A = Area]

64th BPSC (Pre) Exam. 2018-19 249 YCT


65th Bihar Public Service Commission
Preliminary Examination, 2019
GENERAL KNOWLEDGE & GENERAL SCIENCE
(Solved Paper with Detail Explanation)
1. Who wrote the book Desher Katha? 5. Who said, '' Swaraj is my birhtright and I shall
(a) Sakharam Ganesh Deuskar have it"?
(b) Rajendra Prasad (a) Bipin Chandra Pal
(c) Nivaran Chandra (b) Aurobindo Ghose
(d) Murali Mohan Prasad (c) Bal Gangadhar Tilak
(e) None of the above/More than one of the (d) Subhash Chandra Bose
above (e) None of the above/More than one of the
Ans. (a) : Desher Katha book was written by Sakharam above
Ganesh Deuskar. This book describes in exhaustive Ans. (c) : Bal Gangadhar Tilak said that Swaraj is my
detail the British commercial and industrial exploitation birthright and I shall have it, (Belgaum, 1916).
of India. Bal Gangadhar Tilak was born in 23 July 1856. He was
2. Who was the Editor of Searchlight news paper an Indian nationalist, teacher, and an Independence
in Bihar? activist.
(a) Abdul Bari 6. Rabindranath Tagore gave up his 'Knighthood'
(b) Lambodar Mukharjee because of
(c) Murali Mohan Prasad (a) brutal suppression of Civil Disobedience
(d) Ramanada Chatterjee Movement
(e) None of the above/More than one of the (b) Execution of Bhagat Singh
above (c) Jallianwala Bagh Tragedy
Ans. (c) : The Editor of Searchlight Newspaper in Bihar (d) Chauri Chaura Incident
was Murali Mohan Prasad. He started searchlight as a (e) None of the above/More than one of the
bi-weekly newspaper. above
3. Who formed the Bihar 'Socialist Party' in Ans. (c) : The Jallianwala Bagh massacre, also known
1931? as the Amritsar massacre took place on 13 April 1911.
(a) Phulan Prasad Verma After Jallianwala Bagh Tragedy Rabindranath Tagore
(b) Swami Yoganand gave up his Knighthood title in condemnation of the
(c) Narhari Parikh tragedy.
(d) Dadabhai Naoroji 7. Who established the 'Servants of India
(e) None of the above/More than one of the Society'?
above (a) Lala lajpat Rai
Ans. (a) : Phulan Prasad Verma formed the 'Bihar (b) Bipin Chandra Pal
Socialist party' in 1931. Bihar Socialist Party later (c) Gopal Krishna Gokhale
provided the foundation for Congress Socialist Party in (d) Bhagat Singh
1934. The party had the Following objectives- (e) None of the above/More than one of the
(1) Propagation of socialist ideas. above
(2) Formation of labour and peasant's organization. Ans. (c) : Gopal Krishna Gokhale established the
4. Who was the founder of the Indian National servants of India society in Pune on 12th June 1905. The
Congress? society organized many campaigns to promote
(a) William Adam (b) A.O. Hume education, sanitation, healthcare and fight the social
(c) Rash Behari Bose (d) Motilal Nehru evils of untouchability and discrimination, oppression
(e) None of the above/More than one of the of women and domestic abuse.
above 8. Which revolutionaries were hanged in the
Ans. (b) : A.O. Hume was the founder of the Indian 'Kakori Train Robbery Case'?
National Congress. (a) Ram Prasad Bismil and Ashfaqulla
In December 1884, after a theosophical convention in (b) Veer Savarkar and Vansudev Chapekar
Madras, Sir Allan Octavian Hume presided over a (c) Prafulla Chandra Chaki and Khudiram Bose
private meeting with 17 men. In this meeting, the idea (d) Surya Sen and Udham Singh
of forming the Indian National Congress was (e) None of the above/More than one of the
formulated. above

65th BPSC (Pre) Exam. 2019 250 YCT


Ans. (a) : Ram Prasad Bismil and Ashfaqulla were (e) None of the above/More than one of the
hanged in the 'Kakori Train Robbery' case. This robbery above
was carried out by 10 revolutionaries including Ans. (a) : India House in London, was founded by
Hindustan Republican Association's Ram Prasad Shaym ji Krishna Verma is 1905. It was opened as a
Bismil, Ashfaqullah Khan, Rajendra Lahiri, Keshav student hostel to provide accommodation for upto 30
Chakraborty, Mukundi Lal, Banwari Lal. Ram Prasad students. It also served as the headquarters for several
Bismil, Ashfaqullah Khan and Roshan Singh were organisations including Indian Home Rule Society.
hanged on December 19, 1927 for involvement in the 14. Who led the salt Satyagraha in Bhagalpur?
Kakori conspiracy. (a) Shri Krishna Singh
9. Who wrote the book, India for Indian? (b) Mahadev Lal Saraf
(a) C.R. Das (b) M.G. Ranade (c) Kumar mishra
(c) V.D. Savarakr (d) S.N. Banerjee (d) Satyanarayan
(e) None of the above/More than one of the (e) None of the above/More than one of the
above above
Ans. (a) : Chittaranjan Das was the author of the book, Ans. (b) : The Salt Satyagraha in Bhagalpur was led by
India for Indians. Mahadev Lal Saraf in the year 1930. He was highly
10. 'Satyshodhak Samaj'was founded by influenced by Gandhiji's thoughts. The movement
(a) Dayanand Saraswati witnessed the refusal to pay chaukidari tax, protest
(b) Jotibha Phule infront of liquor shops, promotion of use of Khadi etc.
(c) Gandhi ji 15. Who led the 'Ambari Satyagraha' in 1939 in
(d) Dr. Baba Saheb Ambedkar Bihar?
(e) None of the above/More than one of the (a) Rahul Sankrityayan
above (b) Yadunanadan Sharma
Ans. (b) : 'Satyashodhak Samaj' was a social reform (c) Swami Sahajanand
society founded by Jyotiba Phule in Pune, Maharashtra (d) Swami Yoganand
on 24 September 1873. It espoused a mission of (e) None of the above/More than one of the
education and increased social rights and political above
access for underprivileged group, focused especially on Ans. (a) : Rahul Sankrityayan led the 'Ambari
women, shudras and Dalit in Maharashtra. Satyagraha' in 1939 in Bihar. He was a unique
11. Who was the first woman President of the personality of Hindi literature.
Indian National Congress ? During this movement he was implicated in false case
(a) Kasturba Gandhi and sentenced to 6 months imprisonment.
(b) Mrs. Annie Besant 15

∑  n + 3  then the value of S is


(c) Sarojini Naidu 1
16. If S =
(d) Bhakti Laxmi Desai
n =1
(e) None of the above/More than one of the
above 1
(a) 125 (b) 120+
Ans. (b) : Mrs. Annie Besant was the first woman 3
President of the Indian National Congress. She presided 1
(c) 135+ (d) 130
over in the 1917 Calcutta session of Indian National 3
Congress. (e) None of the above/More than one of the
12. Who was the first satyagrahi of the Individual above
Satyagraha Movement ? 15
 1
(a) Sarojini Naidu
(b) C. Rajagopalchari
Ans. (a) ∑  n + 3 
n =1
(c) Vinoba Bhave
(d) Subhash Chandra Bose 15 15
1
(e) None of the above/More than one of the
above
= ∑
n =1
n+
3 ∑1
n =1
Ans. (c) : Vinoba Bhave was the first satyagrahi of the 1
Individual satyagraha movement. The Individual = (1 + 2 + 3 + ..........15 ) + (1 + 1 + 1 + ........15 times )
satyagraha was not only to seek independence but to 3
affirm the right of speech. 15 ×16 1
= + ×15
13. Who was the founder of the 'India House' in 2 3
London? = 120 + 5 = 125
(a) Shyam ji Kridhnavarma 17. Two trains, each of length 150 meters, are
(b) Rash Behari Bose moving in opposite directions with equal speed
(c) Ramchandra of 90 km per hour. The time taken by the trains
(d) Taraknath Das to cross each other will be
65th BPSC (Pre) Exam. 2019 251 YCT
(a) 3 sec. (b) 4.5 sec. Ans. (d)
(c) 6 sec. (d) 9 sec.
(e) None of the above/More than one of the
above
Ans. (c) Relative speed of trains = 90 + 90 = 180 km/h
150 + 150 300
required time= = =6 seconds
5 50
180 × 22. If x2 – y2 = 7 and x – y = 1, then the length of a
18
18. The number of ways in which 12 identical pens diagonal of a rectangle with length and width
can be distributed between two students, if respectively x cm and y cm will be
each student is to get at least two pens, is (a) 5 cm
(a) 8 (b) 9 (b) 6 cm
(c) 10 (d) 11 (c) 7 cm
(e) None of the above/More than one of the (d) 8 cm
above (e) None of the above/More than one of the
Ans. (b) Let us now see the possible groups- above
= (2, 10), (3, 9), (4, 8), (5, 7), (6,6), (7, 5), (8,4) (9, 3), Ans. (a) x2 – y2 = 7
(10, 2) (x + y) (x – y) = 7 .......... (1)
∴ Here are the 9 possibilities, so the answer is 9. x–y=1
19. If the radii of circles A and B are in the ratio From equations (1) and (2),
1.5 : 1, then the areas of the circles A and B will x+y=7
be the ratio on Solving the equations (2) and (3),
(a) 1.5:1 (b) 2:1 x = 4, y = 3
(c) 2.25:1 (d) 2.5:1 diagonal = x 2 + y 2 = 42 + 32 = 5cm
(e) None of the above/More than one of the
above 23. In a class of 80 students, 60 % students play
Ans. (c) Ratio of radii of two circles A and B = 1.5 : 1 carrom, 45 % play chess and 10 % students
neither play carrom nor chess. The number of
Ratio of Areas of two circles = πr12 : πr22
students who play only chess is
= r12 : r22 (a) 36
= (1.5)2 : (1)2 = 2.25:1 (b) 24
20. One tap can fill a water tank in 3 hours and (c) 12
another tap can empty it in 4 hours. If the tank (d) 8
is one-third full and both the taps are opened (e) None of the above/More than one of the
together, then the time taken to fill the tank above
will be Ans. (b)
(a) 8 hours (b) 9 hours
(c) 10 hours (d) 11 hours
(e) None of the above/More than one of the
above
Ans. (a)

percentage of students who play either of the games =


100 –10 = 90%
2 60 – x + x + 45 –x = 90
Time taken to fill the rd part of the tank 105 – x = 90
3
x = 15
2 Percentage of students who play only chess
× 12
= 3 = 8 hours = 45 – 15 = 30%
4−3 Required number of students
21. The next term in the sequence 30
1, 3, 9, 15, 25, 35, 49...... = 80 ×
100
will be = 24
(a) 80 (b) 64
117 3
(c) 81 (d) 63 24. If x 3 - y 3 = and x - y =
(e) None of the above/More than one of the 8 2
above then the value of x2 + xy + y2 will be
65th BPSC (Pre) Exam. 2019 252 YCT
5 27. Which of the following does not have any
(a) 1 (b) enzyme in its cells?
8
39 39 (a) Lichen (b) Virus
(c) (d) (c) Bacteria (d) Algae
4 8 (e) None of the above/More than one of the
(e) None of the above/More than one of the above
above Ans. (b) : Virus does not have any enzyme in its cells.
Ans. (c) ∵ x3 – y3 = (x – y) [(x – y)2 + 3xy] Viruses are non- cellular microscopic infection agents
117 3  3   that can only replicate in host cell.
2
=   + 3xy  Viruses lack most of the basic cellular structures that
8 2  2  
 are responsible for enzyme formation.
39 9 28. In which organ of the human body are the
− = 3xy lymphocytes formed?
4 4
(a) Liver (b) Bone marrow
30 (c) Spleen (d) Pancreas
= 3xy
4 (e) None of the above/More than one of the
5 above
xy = Ans. (b) : Lymphocytes are formed is the Bone marrow
2
x2 + xy + y2 = x2 –2xy + y2 + 3xy and Thymus (central lymphoid organs). Lymphocytes
= (x –y)2 + 3xy is a type of white blood cells and is responsible for
2 immunity system in most vertebrates.
3 5
=   + 3× 29. Which of the following hormones contains
 
2 2 iodine?
9 15 39 (a) Testosterone (b) Adrenaline
= + = (c) Thyroxine (d) Insulin
4 2 4 (e) None of the above/More than one of the
x 2 +2x+7
If ( 5 ) = (125 ) above
2x+1
25. then the value of x is
Ans. (c) : There are two iodine containing hormones in
(a) 5 (b) 4
the human body Tri-iodothyronine (T3) and Thyroxine
(c) 3 (d) 2 (T4). Thyroxine plays a crucial role in heart and
(e) None of the above/More than one of the digestive function, metabolism, brain development and
above muscle control. It affects almost all of the body's
x 2 + 2x + 7 2x +1
Ans. (d) ( 5) = (125 ) function.
30. AIDS is caused by
( )
2
+ 2x + 7 2x +1
( 5)x = 53 (a) helminth (b) bacteria
(c) fungus (d) virus
2
5x + 2x + 7 = 56x +3 (e) None of the above/More than one of the
By the law of indices, above
x2 + 2x + 7 = 6x + 3 Ans. (d) : AIDS is caused by a virus (HIV) which is a
x2 – 4x + 4 = 0 retrovirus that was first found in 1983. The HIV infects
(x – 2)2 = 0 the cell of the immune system that form the body's
x–2=0 defence system and makes them unable to fight off
x=2 infection.
26. The study of bones is done under the branch of 31. The unit of pressure is
science called (a) kg/cm2 (b) kg/cm
(a) geology (b) serology (c) kg/mm (d) kg/cm3
(c) orology (d) osteology (e) None of the above/More than one of the
(e) None of the above/More than one of the above
above Ans. (a) : The basic unit of pressure is pascal which is
Ans. (d) : Ostrology is the detailed study of the defined as the pressure exerted by a force of one
structure of bones skeletal elements, teeth etc. Newton perpendicularly upon an area of one square
Geology is the study of rocks, and of the way they are meter. Out of the given options Kg/cm2 is a unit of
formal. pressure 1 kg/cm2 = 98066.5 pascals.
Orology is the study of mountain especially the study of 32. The sunlight from the sun to the earth reaches
how mountains are formed. in
Serology is the scientific study or diagnostic (a) 5 minutes (b) 6 minutes
examination of blood serum, especially with regard to (c) 8 minutes (d) 10 minutes
the response of the immune system to pathogen or (e) None of the above/More than one of the
introduced substances. above
65th BPSC (Pre) Exam. 2019 253 YCT
Ans. (c): The sunlight from the sun to the earth reaches checks our current temperature and compares it with the
in an average of 8 minutes and 20 seconds. normal temperature of about 37ºC. If our temperature is
33. Which one of the following is a scalar quantity? too low, the hypothalamus make sure that the body
generates and maintains heat.
(a) Force (b) Pressure
(c) Velocity (d) Acceleration 39. The device to measure electric current is
(e) None of the above/More than one of the (a) voltmeter (b) ammeter
(c) voltmeter (d) potentiometer
above
(e) None of the above/More than one of the
Ans. (b) : A scalar quantity is defined as the physical above
quantity with magnitude and no direction as- Mass, Ans. (b) : Ammeter, is an instrument for measuring
Speed, Distance, Time, Area, Volume, Density, either direct or alternating electric current in amperes.
Temperature, Pressure, etc. An ammeter can measure a wide range of current values
34. Which one of the following quantities does not because at high values only a small portion of the
have unit? current is directed through the meter mechanism.
(a) Stress (b) Force 40. The unit of electric power is
(c) Strain (d) Pressure (a) ampere (b) volt
(e) None of the above/More than one of the (c) coulomb (d) watt
above (e) None of the above/More than one of the
Ans. (c) : The strain is a dimensionless quantity. It is above
the ratio between the length shift and the initial length, Ans. (d) : Electric current → Ampere
so it is unit less. Electrical potential → Volt
35. Sound wave in air is Electric power → Watt
(a) transverse (b) longitudinal Electric charge → Coulomb.
(c) electromagnetic (d) Polarized The unit of electric power is called watts, which is equal
(e) None of the above/More than one of the to one ampere under the pressure of one volt (P = V×I).
above The name Watt is given to honor James Watt.
Ans. (b) : Sound waves in any fluid medium or in air 41. Who is regarded as the father of Modern
one longitudinal waves. In longitudinal waves particles Chemistry?
(a) Rutherford (b) Einstein
of the medium through which the waves travelled
(c) Lavoisier (d) C.V.Raman
vibrate parallel to the direction of the wave movement.
(e) None of the above/More than one of the
36. The value of 50 °C in Fahrenheit scale is above
(a) 104°F (b) 122°F Ans. (c) : Valuable contribution to modern chemistry is
(c) 100°F (d) 75°F made by Antoine Lavoisier, is regarded as the father of
(e) None of the above/More than one of the modern chemistry.
above He discovered that mass is conserved in a chemical
Ans. (b) : [Here F = Fahrenheit & C = Celsius] reaction and also discovered the role of oxygen in
According to Formula:- combustion. He gave the names oxygen and hydrogen.
F = (c × 1.8) + 32 42. The number of electrons and neutrons in an
F = (50 × 1.8) + 32 element is 18 and 20 respectively. Its mass
∵ = 90 + 32 number is
F = 1220 (a) 22 (b) 2
37. What is measured in hertz? (c) 38 (d) 20
(a) Frequency (b) Energy (e) None of the above/More than one of the
(c) Heat (d) Quality above
(e) None of the above/More than one of the Ans. (c) : Formula ⇒ Mass Number = The number of
above Electrons + The number of Neutrons.
∵ = 18 + 20
Ans. (a) : Hertz is the SI unit of frequency. One hertz is
Mass Number = 38.
equal to one cycle per second.
43. Which of the following is also known as
1 laughing gas?
Formula – F=
T (a) Nitric oxide (b) Nitrous oxide
38. Body temperature is regulated by (c) Nitrogen pentoxide (d) Nitrogen
(a) Medulla (b) thalamus (e) None of the above/More than one of the
(c) hypothalamus (d) cerebellum above
(e) None of the above/More than one of the Ans. (b) : Nitrous oxide, is commonly known as
above laughing gas or happy gas. It is colourless, non-
Ans. (c) : Our body temperature is regulated by a part flammable gas. This gas is used in medical and dental
of our brain called the hypothalamus. The hypothalamus procedures as a sedative.
65th BPSC (Pre) Exam. 2019 254 YCT
44. Which of the following is not a type of element? Ans. (c): Formula of glucose is - C6 H12 O6. It is
(a) Metals (b) Nonmetals simple sugar Fructose also has same chemical
(c) Gases (d) Metalloids formula as C6 H12 O6. One molecule of sucrose
(e) None of the above/More than one of the
above
breaks into one molecule each of Glucose &
Fructose.
Ans. (c) : Gas is a state of matter, it is not type of
element. C12 H12 O12 + H2O  invertase
→ C6 H12 O6 (glucose)
Note - Elements are chemically the simplest substances + C6 H12 O6 (fructose).
and hence cannot be broken down using chemical Formula of sources - C12 H22 O11.
reactions; element can only be changed into other 50. What is the bond order of CO group
elements using nuclear methods. (a) 1 (b) 2.5
45. The pH value of water is (c) 3.5 (d) 3
(a) 4 (b) 7 (e) None of the above/More than one of the
(c) 12 (d) 18 above
(e) None of the above/More than one of the Ans. (d) : Bond order is the number of chemical bonds
above present between a pair of atoms in the compound.
Ans. (b): The pH value of water is 7 pH is a Bond order =
quantitative, measure of the acidity or basicity of No. of bonding electron – No. of anti bonding electron
aqueous or other liquid solution. A solution with a pH 2
less than 7 is considered acidic, a solution with a pH in CO the number of bonding electron is 10 and no. of
greater than 7 is considered basic, or alkaline. anti bonding electron is 4.
46. The main component of greenhouse gases is 10 – 4 6
(a) carbon dioxide (b) methane ∴ Bond order = = =3
(c) nitrous oxide (d) ozone 2 2
(e) None of the above/More than one of the 51. The book, the Origin of Species was written by
above (a) Linnaeus (b) Lamarck
Ans. (a) : The main component of green house gas is (c) Mendel (d) Darwin
carbon dioxide which accounts for about 79% of the (e) None of the above/More than one of the
total green house gases emission. Other green house above
gases are Methane (11%), Nitrogen oxide (7%), and Ans. (d) : The book, The Origin of Species was written
Fluorinated gases such as Chlorofluorocarbon (3%), as by Charles Darwin.
per US Greenhouse Gas Emission 2020. Charles Robert Darwin (1809-1882) was an english
47. Oxygen is absent in naturalist whose scientific theory of evolution by
(a) kerosene (b) glass natural selection become the foundation of modern
(c) soil (d) cement evolutionary studies.
(e) None of the above/More than one of the 52. The interaction between algae and fungi to
above form lichen is called
(a) Parasitism (b) Mutualism
Ans. (a) : Oxygen is absent in kerosene. It is called
(c) Commensalism (d) Conversion
paraffin or paraffin oil. It is a flammable hydrocarbon
(e) None of the above/More than one of the
liquid commonly used as a fuel. Kerosene, is typically
above
pale yellow or colourless liquid and has a not-
unpleasant characteristic odour. It is obtained from Ans. (b) : The interaction between algae and fungi to
petroleum and used for burning in kerosene lamps and form lichen is called mutualism.
domestic heater or furnances. Mutualism is an association between organism of two
different species in which each benefits from the other.
48. Natural rubber is a polymer of
Mutualistic arrangements are most likely to develop
(a) Isoprene (b) styrene
between organisms with widely different living
(c) vinyl acetate (d) propene requirement.
(e) None of the above/More than one of the
53. The actual location or place where an organism
above lives is called
Ans. (a) : Natural rubber is a polymer of isoprene. (a) Habitat (b) Ecosystem
Isoprene, also called 2- methyl - 1,3-butadiene, a (c) Niche (d) Biome
colourless, volatile liquid hydrocarbon obtained only in (e) None of the above/More than one of the
processing petroleum or coal tar and used as a chemical above
raw material. Ans. (a) : The actual location or place where an
49. What is the formula of glucose? organism lives is called habitat.
(a) C6H22O11 (b) C6H10O6 In ecology, the term habitat summarises the array of
(c) C6H12O6 (d) C6H6O6 resources, physical and biotic factors that are present in
(e) None of the above/More than one of the an area, such as to support the survival and reproduction
above of a particular species.

65th BPSC (Pre) Exam. 2019 255 YCT


54. Solar energy is converted into ATP in (a) Bihar
(a) Mitochaondria (b) Chloroplast (b) Sikkim
(c) Ribosome (d) Peroxisome (c) Himachal Pradesh
(e) None of the above/More than one of the (d) Jammu and Kashmir
above (e) None of the above/More than one of the
Ans. (b) : In chloroplast, the electromagnetic energy of above
sunlight is converted into chemical energy. In Ans. (a) : Special Category Status is a classification
chloroplast, chlorophyll absorbs the sunlight. given by the centre to assist development of states that
Chlorophyll here is packed into grana. In grana sunlight face geographical and socio-economic disadvantages.
is converted into chemical energy is the form of ATP Out of the given options, Bihar was never granted the
(adenosine triphosphate) which is the main energy special category status by the central government.
storing molecule in living organisms. 59. Yousuf Shah Chak, the last Muslim ruler of
55. The by-product of photosynthesis is kasmir valley, who was exiled to Bihar by the
(a) CO2 (b) O2 Mughal emperor Akbar, was buried in
(c) energy (d) sugar (a) Patliputra (b) Rajgir
(e) None of the above/More than one of the (c) Munger (d) Nalanda
above (e) None of the above/More than one of the
above
Ans. (b) : Oxygen (O2) is the by-product of
photosynthesis. Ans. (d) : Yousuf Shah Chak was a ruler of kashmir.
Photosynthesis, is the process by which green plants He was successor of his father Ali Shah Chak and ruled
Kashmir from 1579 to 1586 AD. He was the last
and certain other organisms transform light energy into
independent Mulsim ruler of Kashmir Valley. He was
chemical energy. During photosynthesis in green plants,
exiled to Bihar by the Mughal emperor Akbar. He was
light energy is captured and used to convert water,
buried in Biswak village of Nalanda district.
carbon dioxide, into oxygen and energy rich organic
compound i.e. glucose. 60. What is the tagline of Bihar State Tourism
Development Corporation?
56. Who was the Chief Justice of the Patna High (a) Bright Bihar
court who administered the oath of office to (b) Hum Sabka Bihar
Mr. Fagu Chauhan as the Governor of Bihar in (c) Blissful Bihar
July 2019 ? (d) Bihar is the Best
(a) Hon. Mukesh Shah (e) None of the above/More than one of the
(b) Hon. A.P.Sahi above
(c) Hon. Rekha Manharlal Doshit
Ans. (c) : Blissful Bihar is the tagline of Bihar State
(d) Hon. Dipak Mishra
Tourism Development Corporation.
(e) None of the above/More than one of the
61. Prime Minister Narendra Modi recently laid
above
the foundation of Mandal Dam Project to be
Ans. (b) : Hon. A.P. Sahi was the Chief of Justice of the built on which river?
Patna High Court who administered the oath of office to (a) Koshi (b) Burhi Gandak
Mr. Fagu Chauhan as the Governor of Bihar in July (c) Phalgu (d) North Koel
2019. (e) None of the above/More than one of the
57. What is the significance of 'Bihar Diwas' that is above
celebrated on March 22 every year? Ans. (d) : Prime Minister Narendra Modi laid the
(a) On this day, the State Reorganization foundation of Mandal Dam project to be built on North
Commission created Bihar Koel River. The project aim to provide irrigation to
(b) Bihar was carved out of United Provinces on drought prone areas of Palamu & Garhwa districts in
this day in 1873 Jharkhand and Aurangabad & Gaya district in Bihar.
(c) To commemorate Bihar's separation from 62. In July 2019, experts visited for the first
Bengal Presidency in 1912 documentation of a Megalithic site of Bihar in
(d) To celebrate naming of Bihar in 12th century which of the following districts?
by Muslim rulers (a) Patna (b) Kaimur
(e) None of the above/More than one of the (c) Madhubani (d) Banka
above (e) None of the above/More than one of the
Ans. (c) : Bihar Diwas is observed every year on March above
22 to commemorate the Separation of Bihar from Ans. (e) : In July 2019, for the first time, archaeological
Bengal Presidency in 1912. exploration of megalithic cultural site has taken place in
58. Which one of the following States was never Kaimur hill of Rohtas district of Bihar. Researcher Dr.
granted the Special Category Status (SCS) by Shyam Sundar Tiwary discovered the site in the Kaimur
the Central Government? Hills under Nauhatta block of Rohtas district.
65th BPSC (Pre) Exam. 2019 256 YCT
63. Which country was invited by the host UAE as (c) Third (d) Fourth
the 'Guest of Honour' to attend 46th Session of (e) None of the above/More than one of the
the Councill of Foreign Ministers of the above
organization of Isalmic Cooperation (OIC) held Ans. (b) : The second edition of the Indian Sign
in Abu Dhabi recently? Language Dictionary was released by the government in
(a) Pakistan (b) Oman march 2019. It is provided to the hearing impaired
(c) India (d) Turkey people with 6,000 words in Hindi & English along with
(e) None of the above/More than one of the their corresponding graphic representation.
above The first edition of the dictionary was launched with
Ans. (c) : India was invited by the host UAE as the 3000 words on March 23, 2018.
Guest of Honour" to attend the 46th session of the 68. Who succeeded Manohar Parrikar as the Chief
Council of Foreign Ministers of Organisation of Islamic Minister of Goa on his death recently?
Cooperation. Even though Pakistan is founding member (a) Sudin Dhavalikar (b) Nilesh Cabral
of OIC. It absented itself during the session. (c) Pramod Sawant (d) Rohan Khaunte
64. Recently, ISRO and CNES signed as agreement (e) None of the above/More than one of the
to set up a joint maritime surveillance system. above
CNES is a space agency of which country? Ans. (c) : Pramod Sawant is an Indian politician who is
(a) Russia (b) Germany the 13th and current Chief minister of Goa who
(c) France (d) Canada succeeded Monohar Parrikar as Chief Minister of Goa
(e) None of the above/More than one of the after his death.
above 69. Who is the winner of the International Gandhi
Ans. (c) : CNES is the French Government Space Peace Prize for 2018?
Agency founded in 1961. ISRO and CNES signed an (a) Yohei Sasakawa
agreement in March 2019, to set up a joint maritime (b) Akshaya Patra Foundation
surveillance system. France and India will work in order (c) Sulabh international
to develop a constellation of low-earth orbiting satellite (d) Ekal Abhiyan trust
that will identify and track movement of ships globally (e) None of the above/More than one of the
and in particular those moving in the Indian Ocean. above
65. What is the capital of Paraguay to which the Ans. (a) : International Gandhi Peace Prize for 2018
Vice president of India paid a visit in march was conferred on Yohei Sasakawa who is Goodwill
2019? Ambassador of the World Health Organization for
(a) Asuncion (b) Zagreb Leprosy Elimination.
(c) San Jose (d) Managua 70. World's first camel hospital is located in which
(e) None of the above/More than one of the city?
above (a) Tehran (b) Jaipur
Ans. (a) : Asuncion city is the capital of Paraguay. The (c) Bikaner (d) Dubai
city is bordered by the Paraguay River. It is the (e) None of the above/More than one of the
principle part and main industrial and cultural center of above
Paraguay. Ans. (d) : Dubai camel hospital the word's first camel
66. Where was the 4th Session of the UN hospital, is a veterinary hospital located in Dubai, UAE.
Environment Assembly (UNEA-4) held The camel hospital opened its doors in 2017 to meet the
recently ? overwhelming demand in the UAE for an advanced
(a) Paris-France medical facility dedicated to treating camels.
(b) Ulaanbaatar - Mongolia 71. Who was the winner of the Mens's Singles of
(c) Benijing - China the YONEX All England Open Badminton
(d) Nairobi - Kenya Championships, 2019?
(e) None of the above/More than one of the (a) Viktor Axelsen (b) Lin Dan
above (c) Kento Momota (d) Shei Yuqi
Ans. (d) : The fourth UN Environment Assembly was (e) None of the above/More than one of the
held in Nairobi (Kenya's capital). The theme of the above
Assembly was "Innovation Solutions for Environmental Ans. (c) : The winner of the men's single, YONEX All
Challenges and Sustainable Consumption and England Open Badminton Championships, 2019, is
Production". Kento Momota. Momota became the first Japanese man
The United Nations Environment Assembly was created to win the Title All England Open Badminton
on June 2012. The UN Environment Assembly is now Championships is held annually in England and it is
composed of 193 member States. world's oldest Badminton tournament.
67. Which edition of the Indian Sign Language 72. The five rhino range nations, who signed a
Dictionary was released by the Government in declaration The New Delhi, Declaration on
March 2019? Asian Rhinos, 2019, are India, Nepal, Malaysia,
(a) First (b) Second Indonesia and
65th BPSC (Pre) Exam. 2019 257 YCT
(a) Vietnam (b) Thailand Ans. (a): The Ministerial Meeting of the Coordinating
(c) Myanmar (d) Bhutan Bureau of Non- Aligned Movement (NAM) was held on
(e) None of the above./ More than one of the 21st July, 2019 in Caracas, capital of Venezuela. The
above theme of the meeting was "Promotion and
Ans. (d) : India and four Rhino range nations, i.e. Consolidation of Peace through Respect for
International Law.
Bhutan, Nepal, Indonesia and Malaysia, have signed a
77. Who was the Permanent Representative of
declaration 'The New Delhi Declaration on Asian
India to The UN before Syed Akbaruddin?
Rhinos 2019' for the Conservation and Protection of the (a) Hardeep Singh Puri
Species held in New Delhi at Second Asian Rhino (b) Shashi Tharoor
Range countries meeting. (c) Asoke Kumar Mukerji
73. Who is the second fastest batsman to score 25 (d) Nirupam Sen
centuries in the test format after Sir Don (e) None of the above/More than one of the
Bradman? above
(a) Virat Kohli Ans. (c) : The permanent representative of India to the
(b) Mahela jayawardene UN is the India's foremost diplomatic Representative to
(c) Kane Williamson the United Nation (UN). Syed Akbaruddin was the
Permanent Representative of India to UN from January
(d) Steve Smith 2016 to April 2020, the Preceder of Syed Akbaruddin
(e) None of the above/ More than one of the was Asoke Kumar Mukerji. The current permanent
above representative of India to UN is Ruchira Kamboj (July
Ans. (d) : Steve Smith has became the second fastest 2022 till now).
batsman to score 25 centuries in the test format after Sir 78. Who is the Democratic Presidential candidate
Don Bradman. Steve Smith achieved this feat during the and the first Hindu member of the US
second inning of first Ashes test against England. Sir Congress, who sued Google for at least $50 M
Don Bradman Still holds the record with 25 centuries in for its 'discriminatory actions' against his/her
68 innings. Sir Don Bradman was an Australian 2020 election campaign?
cricketer. (a) Ro Khanna
(b) Bobby Jindal
74. In July 2019, The spacecraft Chandrayaan-2 (c) Raja Krishnamoorthi
was launched from which Indian State? (d) Tulsi Gabbard
(a) Gujarat (e) None of the above/More than one of the
(b) Andhra Pradesh above
(c) Karnataka Ans. (d) : Tulsi Gabbard, the Democratic presidential
(d) Maharashtra candidate and the first Hindu member of the US
(e) None of the above/ More than one of the congress, had sued Google for at least 50m for Googles
above "discriminatory actions" against her 2020 election
campaign and stifling her free speech rights.
Ans. (b) : The Indian Space Research Organization
(ISRO) launched the spacecraft. Chandrayaan- 2' from 79. Who is the author of the book, A Gallery of
Rascals?
Sriharikota, Andhra Pradesh on 22 July 2019. (a) Arundhati Roy (b) Ruskin Bond
75. After Theresa May, who amongst the following (c) Vikram Seth (d) Toni Morrison
was not among the last seven in the contest (e) None of the above/More than one of the
after the first round of voting in the above
Conservative Party to be the Prime Minister of Ans. (b) : The Author of the Book, 'A Gallery of
the UK? Rascals' is Ruskin Bond. Ruskin Bond is one of India's
(a) Boris Johnson Most well-known writers. He received the Padma Shri
(b) Matthew Hancock in 1999 and the Padma Bhushan in 2014.
(c) Rory Harper 80. The first showroom in India of the retail
(d) Mark Harper furniture giant 'Ikea' was opened in which city
(e) None of the above/More than of the above in 2018?
(a) Bengaluru (b) Hyderabad
Ans. (d) : The Mark Harper was not among the last (c) New Delhi (d) Mumbai
seven in the contest after the first round of voting in the (e) None of the above/More than one of the
conservative party to be the P.M. of UK. above
76. Where was the Ministerial Meeting of the Ans. (b) : The Swedish furniture, IKEA was opened
Coordinating Bureau of Non-Aligned their first store in Hyderabad India.
Movement (NAM) held in 2019? IKEA is a MNG (Multinational Group) of Sweden, that
(a) Venezuela (b) Azerbaijan designs and sells ready to assemble furniture, Home
(c) Serbia (d) Ethiopia accessories and other valuable goods and home
(e) None of the above/More than one of the services. IKEA has received the Indian Government's
above nod for 100% FDI in Indian Single brand retail.
65th BPSC (Pre) Exam. 2019 258 YCT
81. Where will the Summer Olympic Games be 85. Who became the Editor of the newspaper, The
held in 2020? Hindu, after Mukund Padmanabhan?
(a) Berlin (b) Paris (a) N. Ram
(c) Tokyo (d) Los Angeles (b) Siddharth Varadarajan
(e) None of the above/ More than one of the (c) Suresh Nambath
above (d) Malini Parthasarathy
Ans. (c) : The Summer Olympic Games, 2020 were (e) None of the above/More than one of the
held in Tokyo, Japan. It was scheduled to be held in above
(July-August, 2020) but the games were postponed due Ans. (c) : Suresh Nambath was appointed the editor of
to the Global Pandemic Covid-19. After that the 2020 the newspaper 'The Hindu', after Mukund Padmanabhan
Summer Olympics were held in 23 July to 8 August by the Board of Directors of the Hindu Publishing
2021 In Tokyo. The motto of the Games was “Faster, Group.
Higher, Stronger - Together”. That was the second time 86. Who is considered as the creator of Modern
the Olympics games were held in Tokyo. Earlier it was Bihar?
held in 1964. (a) Sachchidananda Sinha
82. UNESCO inaugurated the celebration of 2019 (b) Kumar Kalika Prasad Sinha
as the International Year of the Periodic Table (c) Sir Ganesh Singh
of Chemical Elements to celebrate its (d) Acharya Narendra Dev
completion of how many years? (e) None of the above/More than one of the
(a) 100 (b) 150 above
(c) 75 (d) 50 Ans. (a) : Sachchidananda Sinha is considered as the
(e) None of the above/ More than one of the creator of modern Bihar. Mr. Sinha was the first Vice-
above Chancellor of Patna University. He was the leading
Ans. (b) : The International year of the Periodic Table journalist and an educationist and a famous advocate
of Chemical Elements launched at UNESCO and an active member of the INC (Indian National
headquaters. The events and activities will be held Congress). Sachchidananda Sinha also participated in
throughout the year to celebrate the 150th Anniversary the Home Rule League Movement.
of the organization of the Periodic Table by Russian 87. Who formed Bihar Provincial Kisan Sabha?
scientist Dmitri Mendeleev, one of the father of modern (a) Swami Sahajanand Saraswati
chemistry. (b) Ram Sundar Singh
83. By Passing 'Tripal Talaq Law' recently, the (c) Ganga Sharan Sinha
Parliament of India endorsed which year's (d) Ramanand Mishra
ruling of the Supreme Court of India that had (e) None of the above/More than one of the
made Triple Talaq ('Talaq-e-Biddat) as above
unconsitutional? Ans. (a) : Swami Sahajanand Saraswati formed Bihar
(a) 2011 (b) 2013 Provincial Kisan Sabha in 1929 in Bihar. Swami
(c) 2015 (d) 2017 Sahajanand Saraswati was the pioneer of Bihar Kisan
(e) None of the above/More than one of the Sabha movement and patron of Bihar Provincial Kisan
above Sabha. He was the first president of the All India Kisan
Ans. (d) : The Supreme Court of India had declared the Sabha. The Major aims of Bihar Provincial Kishan
'Triple Talaq ' (Talaq - e - Biddat) as Sabha (BPKS) was, to abolish the Zamindari system
unconstitutional in the Shayara Bano case (2017). The and reduce land revenue.
parliament has passed the Muslim Women (Protection 88. Dutch East India Company established its
of Rights on Marriage) Bill, 2019, thereby, factory at Patna in which years?
criminalizing the practice of instant Triple Talaq. In (a) 1601 (b) 1632
Triple Talaq or Talaq -e-Biddat, any Muslim husband (c) 1774 (d) 1651
who gives instant divorce (Talaq) orally or in written (e) None of the above/More than one of the
will be void and illegal and he may face the punishment above
of up to three year in Jail. Ans. (b) : The Dutch East India Company was created
84. Where is the BRICS Summit, 2019 scheduled in 1602 as "United East India Company" and it's first
to be held? permanent trading post was in Indonesia. In India, they
(a) Brazil (b) India established the first factory in Masulipattanam in 1605,
(c) China (d) Russia followed by Pulicat in 1610 and 1616 in Surat.
(e) None of the above/More than one of the Dutch East India Company also established their factory
above at Patna in 1632, where presently Patna College is
Ans. (a) : The 11th BRICS Summit 2019, was held at situated.
Brasilia, Brazil. The theme of the 11th BRICS Summit 89. Who established Swaraj Dal in Bihar?
2019, was- "BRICS: Economic Growth for an (a) Shri Krishna Singh
Innovative future". 14th BRICS Summit hosted by (b) Ramala Shah
China in virtual mode on 23-24 June, 2022. (c) Bankim Chandra Mitra
65th BPSC (Pre) Exam. 2019 259 YCT
(d) Sachindra Nath Sanyal Ans. (a): Mughal paintings reached its zenith during the
(e) None of the above/More than one of the reign of Jahangir. He patronized a number of painters
above like Abul Hasan, Bishan Das, Madhu, Anant, Manohar,
Ans. (a) : Shri Krishna Singh established 'Swaraj Dal' Govardhan and Ustad Mansur.
in 1923 in the Bihar. Swaraj Dal, also known as Swaraj Mughal Painting is a particular style of South Asian
Party, was a political party formed in India on 1 January painting confined to miniatures either as book
1923 after the Gaya annual conference of the National illustrations or as single works to be kept in album.
Congress in 1922. The Swaraj Party was founded by 95. In medieval India, why was Mansabdari system
CR. Das and Motilal Nehru. was introduced?
90. The name by which Ashoka is generally (a) For revenue collection
referred to in his inscriptions is (b) Facilitating recruitment to the army
(a) Chakravarti (b) Priyadarshi (c) To establish religious harmony
(c) Dharmadeva (d) Dharmakirti (d) Ensuring clean administration
(e) None of the above/More than one of the (e) None of the above/More than one of the
above above
Ans. (b) : Ashoka was generally referred to as Ans. (b) : Akbar introduced a well-organised system in
Devanampriya Piyadarshi in his few inscriptions. the military and civil service of the empire which came
Maski edict of Ashoka in Raichur district of Karntaka to be known as mansabadari system. Later on, this
confirmed the name of Ashoka as Devanampriya system became the main basis of the military and civil
Piyadarshi. administration of the Mughals.
91. Who among the following is known for his 96. Who established Ashta Pradhan?
work on medicine during the Gupta period? (a) Chandragupta (b) Ashoka
(a) Sushruta (b) Saumilla (c) Harshavardhana (d) Shivaji
(c) Shudraka (d) Shaunaka (e) None of the above/More than one of the
(e) None of the above/More than one of the above
above Ans. (d) : Ashta Pradhan was the administrative and
advisory council set up by the maratha leader
Ans. (a) : Sushruta is known for his work on medicine
Chhatrapati Shivaji, which contributed to his successful
during the gupta period. Sushruta was the first Indian
military attacks on the Mulsim Mughal Empire and to
surgeon of 6th century AD. The Sushruta Samhita is a
the good governance of the territory over which he
sanksrit text on surgery. The Gupta Era (320 AD to 550
established his rule. The council was formed in 1674.
AD) considered to be the golden era of India in terms of The term Ashta Pradhan literally means, Council of
Art, Medical science, Literature, Astronomy etc. Eight.
92. Which one of the following scripts of ancient 97. Which Delhi Sultan built maximum number of
India was written from right to left? canals?
(a) Brahmi (b) Sharada (a) Firoz Shah Tughlaq (b) Iltutmish
(c) Kharosthi (d) Nandanagari (c) Balban (d) Sikandar Lodi
(e) None of the above/More than one of the (e) None of the above/More than one of the
above above
Ans. (c) : Kharosthi script was originally developed in Ans. (a) : The sultan of Delhi who is reputed to have
present day northern Pakistan and eastern Afghanistan built the biggest network of canal in India was Firoz
between 4th and 3rd century BCE. It is written from right Shah Tughlaq. To support the newly founded city of
to left.
Hisar-e-Firoza in 1354 AD, he constructed a double
93. Which one of the following ports was in use for system of canals from Yamuna to Sutlej. That Yamuna
the north Indian trade during the Gupta canal was repaired for irrigation purposes during the
Period?
time of Akbar.
(a) Kalyan (b) Tanralipti
(c) Broach (d) Cambay 98. Who used Hooghly as a base for piracy in the
(e) None of the above/More than one of the Bay of Bengal?
above (a) The Dutch (b) The French
Ans. (b) : Tamralipti port controlled the North Indian (c) The Portuguese (d) The British
trade during the Gupta Period. The major port included (e) None of the above/More than one of the
Tamralipti, Broach, Kalyan, and Cambay. It is above
considered that Tamralipti was the exit port of the Ans. (c) : Hooghly is one of the main economically
Mauryan trade route for the south and south east. growing districts in West Bengal. At that time Hooghly
94. Mughal painting reached its zenith under was in a good position both in Industry and Agriculture.
(a) Jahangir (b) Humayun The Portuguese used Hooghly as a base for piracy in the
(c) Shahjahan (d) Akbar Bay of Bengal. In 1631-32, Qasim Khan who was the
(e) None of the above/More than one of the Governor of Bengal during Shah Jahan's reign finally
above subdued and imprisoned thousand of Portuguese.

65th BPSC (Pre) Exam. 2019 260 YCT


Hooghly was used by the Portuguese as a base for 103. Who was the President of the Indian National
piracy in the Bay of Bengal. In the 16th century Congress at the time of Partition of India?
Portuguese had official base in Hooghly and used it as a (a) J.B. Kriplani
base for piracy and slave trade. The base as well as the (b) Jawaharlal Nehru
piracy business was crushed by Qasim Khan, Governor (c) Maulana Abul Kalam Azad
of Bengal, during the period of Shah Jahan. (d) C. Rajagopalchari
99. Under whose leadership was suppression of (e) None of the above/More than one of the
Thugs achieved? above
(a) Lord Clive (b) Captain Sleeman Ans. (a) : Jivatram Bhagwandas Kriplani was the
(c) Lord Minto (d) Alexander Burnes President of the Indian National Congress at time of
(e) None of the above/More than one of the partition of India.
above 104. Who led the 1857 Revolt in Bihar?
Ans. (b) : Captain Sleeman was a british soldier and (a) Babu Amar Singh
administrator in British India. He is best known for his (b) Hare Krishna Singh
work in 1830 in suppressing the organised criminal (c) Kunwar Singh
gangs known as' Thug'. During 1830s, the Thugs were (d) Raja Shahzada Singh
targeted for eradicating by Governor-General of India, (e) None of the above/More than one of the
William Bentinck and his chief captain, Willian Henry above
Sleeman. Ans. (c) : Babu Kunwar Singh was one of the most
The rigorous operations under Sleeman led to the important freedom fighter during the revolt Mutiny of
capture of 1400 Thugs who were executed by the 1857. He was nearly eighty and in falling health when
government or imprisoned for life. A special prison was in was called upon to lead the rebellion. On 23rd April,
established at Jabalpur for thugs. Kunwar Singh had a victory near Jagdishpur over the
100. The Viceroy who followed aggressive policy force led by Captain Le Grand, but the following day he
towards Afghanistan was died in his village due to injuries.
(a) Lord Mayo (b) Lord Lytton
105. Who led cultivators in Bihar during the Non-
(c) Lord Dufferin (d) Lord Canning
cooperation Movement?
(e) None of the above/More than one of the
(a) Swami vidyanand (b) Raj Kumar Shukla
above
(c) Shri Krishna Singh (d) J.B. Sen
Ans. (b) : Lord Lytton followed aggressive policy (e) None of the above/More than one of the
towards Afghanistan. Lytton was appointed Viceroy in above
1876 by Conservative Prime Minister Benjamin
Disraeli. In November 1876 he launched the invasion of Ans. (a) : Swami Vidyanand led cultivators in Bihar
Afghanistan from British India by an Anglo-Indian during the non-cooperation Movement.
Force with the aim of replacing the Afghan Amir, Sher Swami Vidyanand was a prominent leader of
Ali, who was reputed to harbor pro-Russian sentiments, Darbhanga peasant movement at that time (1919-20).
with a ruler more favorable to Britain. The non-cooperation Movement was endorsed at the
Nagpur session (1920) of INC. The non-cooperation
101. In 1930, from where Mahatma Gandhi started
movement was called off by Mahatma Gandhi in
the Civil Disobedience Movement?
(a) Wardha (b) Dandi February 1922 after an angry mob in the Chauri-Chuara
(c) Savegram (d) Sabarmati incident.
(e) None of the above/More than one of the 106. Among the following districts, which one has
above larger area under dense deciduous forest
Ans. (d) : On 12 March, 1930, Mahatma Gandhi started cover?
the Civil Disobedience Movement from Sabarmati (a) Paschim Champaran
Ashram at Ahmedabad with his 78 followers and (b) Gaya
reached Dandi on 5 April 1930, Gandhi ji broke the salt (c) Kaimur
law at Dandi on 6 April 1930 and started the Civil (d) Nawada
Disobedience Movement. (e) None of the above/More than one of the
102. The Sarabandi Campaign of 1922 was led by above
(a) Chittaranjan Das Ans. (a) : Paschim Champaran in Bihar has larger area
(b) Sardar Vallabhbhai Patel under dense deciduous forest cover. The Someshwar
(c) Rajendra Prasad and Dun mountain ranges in west Champaran are
(d) Lala Lajpat Rai covered with dense moist deciduous forest.
(e) None of the above/More than one of the The forest cover in the state is 7,381 square km which is
above 7.84% of the states geographical area as per ISFR,
Ans. (b) : The 'Sarabandi' which means No tax, was a 2021.
campaign of 1922 led by Sardar Vallabhbhai Patel. In 107. The district of Bihar which has a longer length
the campaign of Sarabandi the peasants decided not to of Ganga river is
pay the taxes. (a) Bhagalpur (b) Katihar
65th BPSC (Pre) Exam. 2019 261 YCT
(c) Patna (d) Begusarai 112. Among the following countries, which one is
(e) None of the above/More than one of the the largest producer of saffron in the world?
above (a) Spain (b) Greece
Ans. (c) : The longest strength of Ganga River in Bihar (c) New Zealand (d) Iran
is in Patna district and the length of the river in Patna is (e) None of the above/More than one of the
around 99 kilometeres. above
Ans. (d) : Iran is the world's leading producer of
108. Which district of Bihar has recorded the
saffron. it produces about 90% of worlds total saffron.
highest density of population (per sq. km) as Besides Iran, India, Spain and Greece are other major
per the 2011 Census? producer of saffron.
(a) Sheohar (b) Vaishali
113. Which one of the following countries is the
(c) Patna (d) Darbhanga
largest producer of uranium in the world?
(e) None of the above/More than one of the (a) Kazakhstan (b) Canada
above (c) Australia (d) France
Ans. (a) : Sheohar district of Bihar has recorded the (e) None of the above/More than one of the
highest population density per sq. km as per the 2011 above
census. Ans. (a) : Kazakhstan with 45% of the world's uranium
The Density of given district of Bihar are as: Sheohar - resources was world's largest uranium producer in 2021.
1882, Vaishali - 1717, Patna - 1823 and Darbhanga- Kazakhstan followed by Namibia (12%) and Canada
1728 (10%) with respect to share of uranium production.
109. Which one of the following countries is the 114. The hills situated closer to Kanyakumari are
largest country without borders in terms of (a) Anaimalai Hills (b) Nilgiri Hills
geographical area? (c) Cardamom Hills (d) Shevaroy Hills
(a) New Zeeland (b) Philippines (e) None of the above/More than one of the
(c) Japan (d) Cuba above
(e) None of the above/More than one of the Ans. (c) : Kanyakumari, also known as Cape Comorin
above is a district in state of Tamil Nadu.
Ans. (c) : Among the given option Japan is the largest Cardamom Hills are situated closer to Kanyakumari.
country without borders in terms of Geographical area. The Cardamom Hill is situated to the south of
Annamalai hills and is separated from them by the
Area of given countries are as follows-
Ealaimalai. These hills are famous for cardamom
Japan - 377835 Km2
cultivation.
New Zealand - 269055 Km2
115. Among the following tributaries, which one is
Philippines - 300000 Km2
the part of Ganga river basin?
Cuba - 110860 Km2 (a) Sankh (b) North Koel
110. The country which has the longest north-south (c) South Koel (d) Barakar
(latitudinal) extension of its territory is (e) None of the above/More than one of the
(a) Russia (b) Chile above
(c) China (d) Brazil Ans. (b) : The Ganga River rises from the Gangotri
(e) None of the above/More than one of the glacier near Gomakh Tal in the Uttarkashi district of the
above Uttarakhand. The total length of Ganga is about 2525
Ans. (b) : Chile has the longest north-south (latitudinal) km. The right tributaries of Ganga are Yamuna, Son,
extension of its territory. The geography of Chile is Chambal, Mahananda, etc. and left tributaries, Karnali,
extremely diverse as the country extends from a latitude mahakali, Gandak, Kosi, Ramganga, Ghagra, etc.
of 17º South to the cape Horn at 56º South. North Koel is a part of the Ganga River basin. The
111. Which one of the following countries has the North Koel rises on the Chhota Nagpur Plateau, It flows
in the Jharkhand and falls into son River, which is a part
highest number of islands?
of Ganga River Basin.
(a) Philippines (b) Indonesia
(c) Maldives (d) Cuba 116. Which one of the following hills does not have
tea plantations?
(e) None of the above/More than one of the
(a) Kanan Devan (b) Nilgiri
above
(c) Darjeeling (d) Girnar
Ans. (b) : According to the Embassy of the Republic of (e) None of the above/More than one of the
Indonesia, Washington, DC there are total number of above
17,508 islands of which about 6000 are inhabited. It is Ans. (d) : Kanan Devan, Nilgiri and Darjeeling's
also the world's most populous Island country with a climate is favorable for tea plantation but Girnar hills is
population of 279,526,342 as in August, 2022 and has located in semi-arid region of Gujarat where tea
ranked 4th in the world. It shares a border with Malaysia plantations are not possible. Gir range is a low mountain
in the northern part of Borneo and Papua New Guinea. range. The climate of Girnar hills around mostly dry

65th BPSC (Pre) Exam. 2019 262 YCT


and warm and the favorable climate for tea cultivation (b) Mumbai-Bengaluru
is warm and wet as tropical monsoon climate and (c) Delhi-Mumbai
temperature in between 16º-30ºC. (d) Amritar-kolkata
117. In India, the State with the largest area under (e) None of the above/More than one of the
dense deciduous forest cover is above
(a) Odisha (b) Maharashtra Ans. (c) : Delhi- Mumbai Industrial/Economic Corridor
(c) Madhya Pradesh (d) Chhattisgarh of India is being developed in collaboration with Japan.
(e) None of the above/More than one of the
The DMIC project was launched in pursuance of an
above
Memorandum of Understanding (MoU) signed between
Ans. (c) : Madhya Pradesh state of India has the largest
the Government of India and the Government of Japan
area under dense deciduous forest cover. Tropical moist
in December 2006.
deciduous forest covers 8.97% of total area of Madhya
Pradesh distributed in the district of Siddhi, Mandla, 122. Which one of the following districts does not
Belaghat, Seoni etc. have dharwar geological formations?
118. Coral reefs are not found in which one of the (a) Munger (b) Rohtas
following regions? (c) Jamui (d) Nawada
(a) Gulf of Cambay (e) None of the above/More than one of the
(b) Gulf of Mannar above
(c) Gulf of Kachchh Ans. (b) : Munger, Jamui and Nawada of Bihar are the
(d) Lakshadweep and Minicoy Island part of Dharwar geological formation but Rohtas district
(e) None of the above/More than one of the of Bihar does not have Dharwar Geological formation.
above Dharwar region is economically the most important
Ans. (a) : Corals are marine invertebrate formed of rock system because they possess valuable minerals like
groups of coral polyps held together by calcium high grade Iron-ore, manganese, copper, lead, gold, etc.
carbonate. Coral reefs in India found in the gulf of 123. The number of districts situated on the bank of
Kachchh, Gulf of Mannar, Lakshadweep and Minicoy Ganga river in Bihar state is
Island. (a) 21 (b) 17
Coral reefs are not found in Gulf of Combay. (c) 12 (d) 6
119. Among the following religious groups, which (e) None of the above/More than one of the
one has recorded the highest percentage above
literacy rate as per the 2011 Census?
(a) Christians (b) Hindus Ans. (c) : Total 12 Districts of Bihar State, are located
(c) Sikhs (d) Jains on the bank of River Ganga. These 12 districts are:
(e) None of the above/More than one of the Patna, Samastipur, Buxar, Saran, Bhojpur, Vaishali,
above Begusarai, Munger, Katihar, Khagaria, Bhagalpur and
Ans. (d) : As per the census 2011 data Jains have the Lakhisarai. The total Length of Ganga which flows in
highest percentage of literates above 7 years of age Bihar is around 445 km.
among India's religious communities, with census 2011 124. Which one of the following wildlife sanctuaries
finding 86.73% of them as literate and only 13.27% as is situated in Munger district of Bihar?
illiterate. Muslims on the other hand, named as the (a) Valmiki (b) Rajgir
highest percentage of illiterates aged beyond 7 years at (c) Bhimbandh (d) Gautam Buddha
42.72%. (e) None of the above/More than one of the
120. Which one of the following pairs of Tribe and above
state is not matched? Ans. (c) : Bhimbandh Wildlife Sanctuary is situated in
(a) Bhils-Gujarat Munger district of Bihar, India. This wildlife Sanctuary
(b) Gaddis-Himachal Pradesh is famous for its hot water springs. It is located in the
(c) Kotas-Tamil Nadu south west towards Haveli Kharagpur of Munger. Tiger,
(d) Todas-Kerala Leopard, chital, wolf, peacock, common kestrel and
(e) None of the above/More than one of the crocodiles are the main wildlife animal and birds of
above
Bhimbandh wildlife sanctuary.
Ans. (d) : Todas Tribal groups are found in Nilgiri hills
125. The district of Bihar which has recorded the
of Tamil Nadu. Toda people are a Dravidian Ethinic
group who live in the Nilgiri mountains. highest sex ratio as per the 2011 Census is
(a) Siwan (b) Gopalganj
121. Which one of the above following
(c) Saran (d) Kishangang
industrial/economic corridors of India is being
developed in collaboration with Japan? (e) None of the above/More than one of the
(a) Chennai-Vizag above

65th BPSC (Pre) Exam. 2019 263 YCT


Ans. (b): According to the Census 2011, among the 130. Which one of the following constitutional
given districts, Gopalganj has registered the highest sex remedies is also known as 'postmortem'?
ratio with 1021 female against 1000 males in 2011. The (a) Prohibition
lowest sex ratio was recorded in Bhagalpur (880) and (b) Mandamus
Munger (876) districts. The highest sex ratio in child (c) Certiorari
population category was recorded by Nawada district at (d) Quo warranto
945. The sex ratio of Bihar state in 2011 was 918. (e) None of the above/More than one of the
126. Who amongst the following is a land record above
officer? Ans. (d) : The constitutional remedy of Quo warranto
(a) Patwari (b) Lambardar is commonly known as 'Postmortem'. Quo warranto a
(c) Zamindar (d) Zaildar writ or legal action requiring a person to show by what
(e) None of the above/More than one of the warrant an officer or franchise is held, claimed or
above exercised. It is known as "Postmortem" because it
Ans. (a) : Patwari is the land record officer of a village. examines the person after his complete appointment to a
He is a link between the village and the government. He public office.
provides information about the crops, based on harvest
inspection to the government. His main functions are as 131. What is the version of the machine VVPAT
follows-maintaining record of crops grown at every used for voting in India?
harvest, maintaining a record of all the land and its (a) M1 (b) Z1
owners in the village and collecting the land revenue. (c) M3 (d) Z3
127. Which one of the following characteristics is (e) None of the above/More than one of the
not true for unitary form of Government? above
(a) Swift decision Ans. (c) : The Voter Verifiable Paper Audit Trail
(b) Flexible (VVPAT) is a tool used by a ballotless voting system to
(c) Ideal for large countries provide feedback to voters.
(d) Uniformity of laws Currently M3 version of the(VVPAT) in being working
(e) None of the above/More than one of the in India. This method was introduced in all 543 Lok
above Sabha constituencies in 2019 Indian general election.
Ans. (c) : Unitary form of government is not Ideal for 132. Which one of the above following Indian States
large countries. A unitary state is a state governed as a has the largest number of members in its state
single power in which the central government is legislature?
ultimately supreme. A unitary government is also (a) Arunchal Pradesh (b) Himachal Pradesh
known as a 'centralised government'. Therefore for a (c) Manipur (d) Meghalaya
large country it would be difficult to administer the (e) None of the above/More than one of the
whole country from a single seat of power. above
128. What is the objective of Community Ans. (b) : Among the given options Himachal Pradesh
Development Programmes? has the largest number of members in its state
(a) Providing educational facilities
legislative Assembly. Himachal Pradesh has 68
(b) Improving standards of living
members in its State Legislature. Arunachal Pradesh 60,
(c) Political training
(d) Helping villages in planning Manipur has 60 and Meghalaya also has 60 its member
(e) None of the above/More than one of the in State Legislature.
above 133. In the Indian order of precedence, who
Ans. (e) : Community Development Porgammes were amongst the following comes first?
launched on 2 October, 1952. Its objective were: (a) The Chairman of UPSC
providing educational facilities, Improving standards of (b) The Chief Election Commissioner
living, helping villages in planning, involving villages (c) The Comptroller and Auditor General
in national reconstruction etc. (d) The Chief Justice of High court
129. Aam Admi Party is a (e) None of the above/More than one of the
(a) State party above
(b) National party Ans. (e) : The Chief Election Commissioner, The
(c) Regional party Comptroller and Auditor General and the Chairman of
(d) Registered party UPSC are at the same order of precedence. Whereas
(e) None of the above/More than one of the Chief Justice of High Court are at 14 place with
above Chairman and Speakers of states legislatives within
Ans. (a) : Aam Admi Party is a state party recognized their representative states.
by Election Commission on Novermber, 2012. Aam 134. Which one of the following statements is
Admi Party has held the majority in the Delhi correct regarding parliamentary form of
legislative Assembly since the 2015 election. Government?
65th BPSC (Pre) Exam. 2019 264 YCT
(a) The Legislature is responsible to Judiciary. 139. Which one of the following programmes was
(b) The Legislature is responsible to Executive. initiated during the Sixth Five-Year Plan?
(c) The Legislature and the Executive are (a) Integrated Rural Development
independent. (b) Rural Literacy Development
(d) The President is responsible to Judiciary. (c) Rural Railways
(e) None of the above/More than one of the (d) Advanced Communication Links for Rural
above People
Ans. (e) : A parliamentary from of Government is that (e) None of the above/More than one of the
in which the Executive have direct or indirect control above
over Parliament. The head of the Government is Prime
Minister, who enjoys the real power. Ans. (a) : The Sixth Five Year Plan in India was
undertaken during the period between 1980 to 1985,
135. Which of the following was constituted under
the Panchayati Raj system? with the main aim of attaining objective like elimination
(a) Khap Panchayat of poverty through the rural development schemes such
(b) Caste Panchayat as IRDP- (Integrated Rural Development Programme),
(c) Gram Panchayat NREP (National Rural Employment Programme), and
(d) Jan Panchayat RLEGP (Rural Landless Employment Guarantee
(e) None of the above/More than one of the Programme).
above 140. Antyodaya Programme was started first of all
Ans. (c) : Gram Panchayat has been constituted under in the state of
the Panchayati Raj system. In India, the Panchayati Raj (a) Bihar (b) Tamil Nadu
now functions as a system of governance in which gram (c) Andhra Pradesh (d) Rajasthan
panchayats are the basic unit of local administration. (e) None of the above/More than one of the
There are three-tier Panchayati Raj system in India. above
Gram Panchayat at the village level, Panchayat system
Ans. (d) : The Antyadaya (Antyodaya Ann Yojana -
at Block level and Zila Parishad at district level.
AAY) was instituted for the upliftment of the poorest of
136. As per the Census 2011, the child sex ratio in
the poor in the country. The programme was launched
Bihar was
by the government on 25th December 2000.
(a) 935 (b) 934
(c) 933 (d) 932 Note: The Antyodaya Programme was firstly launched
(e) None of the above/More than one of the in the mid 70s in Rajasthan by the state government.
above 141. India's share in meat and meat preparation
Ans. (a) : The child sex ratio is defined as the number exports in the year 2017 was
of female per 1000 males in the age group o-6 year. (a) 5% (b) 6%
According to the census 2011, the child sex ratio in (c) 2% (d) 3%
Bihar was 935. (e) None of the above/More than one of the
137. The growth rate of the tertiary sector in Bihar above
during the year 2017-18 was Ans. (e) : India's share in meat and meat preparation
(a) 14.2% (b) 14.6% exports in the year 2017 was less than 10% was at the
(c) 15.6% (d) 15.2% lowest level in 6 year.
(e) None of the above/More than one of the According to BPSC the answer of this question as none
above of the above.
Ans. (b) : As per the Economic Survey Report of India
142. The system under which the peasant himself
2018-19, Bihar's growth rate of the tertiary sector in
owns the land and is responsible for payment of
during the years 2017-18 was recorded at 14.6%.
land revenue to the Government is known as
138. The Gross State domestic Product (GSDP) of
(a) Zamindari system
Bihar at current prices in the year 2017-18 was
(a) Rs. 4,87,628 crores (b) Ryotwari system
(b) Rs. 3,61,504 crores (c) Mahalwari system
(c) Rs. 1,50,036 crores (d) Dahasala system
(d) Rs. 5,63,424 crores (e) None of the above/More than one of the
(e) None of the above/More than one of the above
above Ans. (b) : There were three major system of land
Ans. (a) : The Gross state Domestic Product (GSDP) of revenue collection existed in India. They were:
Bihar for 2022-23 (at current prices) in projected to be Zamindari, Ryotwari and Mahalwari system. In
Rs. 7,45,310 crores. While in the year 2017-18. Gross Ryotwari system the ownership rights were handed over
State Domestic Product (GSDP) of Bihar at current to the peasants and British Government collected taxes
prices was Rs. 4,87,628 crores. directly from the peasants.
65th BPSC (Pre) Exam. 2019 265 YCT
The Ryotwari System was introduced by Thomas receive a minimum assured pension of Rs 3000/-Per
Munro in 1820. This was the primary land revenue month after attaining the age of 60 years, and the
system in South India. subscriber dies, the spouse of the beneficiary shall be
143. In the fiscal year 2018-19, the total foreign entitled to receive 50% of the pension as a family
exchange reserves are pension.
(a) Rs. 34,55,882 crores 147. Who determines the minimum support price in
(b) Rs. 30,55,882 crores India?
(c) Rs. 32,55,882 crores (a) The Commission for Agricultural Costs and
(d) Rs. 28,55,882 crores Prices
(e) None of the above/More than one of the (b) The Agriculture Ministry
above (c) The Finance Commission
Ans. (d) : As per economic survey for the financial year (d) NABARD
2018-19, India's foreign reserves was about 400 billion (e) None of the above/More than one of the
US dollars. Therefore none of the given options is above
correct. Data given by RBI shows that India’s forex
Ans. (a) : Minimum Support Price (MSP) is the
reserves stood at $601.057 billion in the week ending 3
minimum price which the government pays for the
June, 2022.
farmer's produce at the time of procurement. The MSP
144. Which one of the following is not a source of
is determined by the Commission for Agricultural Costs
direct finance?
and Prices. MSP is aimed at saving the crops from price
(a) NABARD
(b) Regional Rural Bank fluctuations in the market. MSPs were firstly introduced
(c) State Bank of India in 1966-67 when the country adopted Green Revolution
(d) Allahabad Bank Technologies.
(e) None of the above/More than one of the 148. Hindu growth rate is related to
above (a) Money
Ans. (a) : Direct finance refers to a financial transaction (b) GDP
between a lender and a borrower which takes place (c) Population
without the presence of financial intermediary like a (d) GNP
commercial bank or an insurance company or mutual (e) None of the above/More than one of the
fund. NABARD is a development bank not commercial above
bank so it, is not source of Direct finance. NABARD Ans. (b) : Indian economist professor Raj Krishna used
provides refinance support for building rural the term 'The Hindu Rate of Growth' to described the
infrastructure. slow growth rate of GDP between 50's and 80's.
BPSC considered options (e) as the correct answer to 149. In Bihar, 'Krishi Kumbha Mela, 2019' was held
the question. in
145. What is the full form of EPCG? (a) Champaran (b) Motihari
(a) Export Promotion Consumer Goods (c) Rajgir (d) Gaya
(b) Exchange programm for consumer Goods (e) None of the above/More than one of the
(c) Export Promotion Capital Goods
above
(d) Expert Programme for Cedit Generation
(e) None of the above/More than one of the Ans. (b) : A three day 'Krishi Kumbh Mela, 2019 was
above held in Motihari, Bihar. The Mela was inaugurated by
Ans. (c) : The term EPCG stands for 'Export Promotion Union Agriculture Minister Radha Mohan Singh along
Capital Goods'. EPCG is a scheme of export promotion with Bihar Governor Lalji Tandon. It was organised
under which an exporter can import a certain amount of with the help of ICAR.
capital goods at zero duty for enhance technology 150. In NITI Aayog's health Index 2019, Bihar
related to exports. scored
146. In Pradhan Mantri Sharam Yogi Maan-Dhan (a) 30.12 (b) 30.13
.
Scheme, each Subscriber shall receive the (c) 32 11 (d) 32.12
minimum assured pension of _____per month (e) None of the above/More than one of the
after attaining the age of 60 years. above
(a) Rs. 3,500 (b) Rs. 2,000 Ans. (c) : The NITI Aayog was released the second
(c) Rs. 3,000 (d) Rs. 1,500 edition of health index in 2019 for year 2017-18. In
(e) None of the above/More than one of the which score of Bihar was 32.11, NITI Aayog release
above fourth version of health index in December, 2021 in
Ans. (c) : The Pradhan Mantri-Shram Yogi Maan-Dhan which top 3 states were- Kerala, Tamil Nadu and
Scheme (PM-SYM) is a voluntary and contributory Telangana and bottom 3 were U.P., Bihar and M.P. The
pension scheme, under which the Subscriber would title of the report was: Healthy States, Progressive India.
65th BPSC (Pre) Exam. 2019 266 YCT
65th Bihar Public Service Commission
Preliminary (PH) Re-Examination, 2019
GENERAL KNOWLEDGE & GENERAL SCIENCE
(Solved Paper with Detail Explanation)
1. Which of the following statements are correct? Ans.(b): Gautam Buddha was born in Kapilvastu
I. Harappa was excavated in 1921 (Lumbini) in 563 BCE. At the age of 29 he left the
II. Currently Harappa is in Pakistan home for asceticism. Gautam Buddha got Bodhi
III. Harappa was on the banks of the River (enlightenment) under a Pipal tree at Bodh Gaya in
Vyas. Bihar. He gave his first sermon in Sarnath, near
IV. R.D. Banerjee was the Harappan Varanasi Uttar Pradesh. This event is known as
Excavator. Dhammachakrapravartana (turning of the wheel of law).
V. The copper chariot with umbrella has The ancient name of Sarnath is- Mrigadava and
been found from Harappa. Rishipattana.
(a) Only I and II 4. Who was the president of first jain council
(b) Only I, II and III convened at Pataliputra?
(c) Only II and IV (a) Vasumitra
(d) Only I, III and IV (b) Sthiramati
(e) None of the above/more than one of the (c) Sthulbhadra
above (d) Sudharman
Ans. (a) : In 1921 AD, under the direction of sir John (e) None of the above/more than above
Marshall, Rai Bahadur Daya Ram Sahani (Director Ans. (c) : The First Jain Council was organized in 300
General of Archaeological Survey of India) got the first BCE in Pataliputra under the reign of Chandragupta
Harappa site excavated. (The site is located on the bank Maurya. The president of First Jain Council was
of Ravi River in Montgomery district in the Punjab Sthulabhadra.
province, Pakistan). Rakhal Das Banerjee got the 5. What did Chandragupta Maurya give as a
excavation of Mohenjo-daro site in the year 1922. A blessing to Seleucus Nicator?
copper chariot of Harappa times was found at
(a) 500 war elephants
Daimabad.
Hence only statements 1 and 2 are correct. (b) 1000 Horses
(c) 2000 Bulls
2. Which one of the following River was not
included in Sapta-Saindhav ? (d) 4000 Cows
(a) Jhelum (e) None of the above/more than above
(b) Chenab Ans. (a) : Seleucus I Nicator was the son of Antiochus
(c) Ravi and one of the Generals of Alexander. Chandra Gupta
(d) Sabarmati Maurya presented 500 war elephants to Seleucus
(e) None of the above/more than above Nicator and Seleucus got his daughter married to
Chandragupta. Seleucus sent one of his ambassador
Ans. (d) : During the Rig-Veda Period, the term 'Sapta- named Megasthenes to the mauryan court.
Saindhav' refered to as the seven Indus Rivers. These
seven rivers are : Sindhu, Saraswati, Vitasta (Jhelum), 6. Which ruler shifted his capital to Pataliputra?
Parushni (Ravi), Askini (Chenab), Sutudri (Sutlej) and (a) Bimbisar
Vipasa (Beas). Saraswati was considered as a Sacred (b) Ajatashatru
river at that time. (c) Udayin
3. Where did Gautam Buddha give his first (d) Chandragupta Maurya
sermon? (e) None of the above/more than above
(a) Pataliputra Ans. (c) : Magadha was one of 16 Mahajanpadas of
(b) Sarnath ancient India. The first dynasty ruled over Magadha was
(c) Gaya the Haryanka dynasty. Udayin was one of the rulers of
(d) Vaishali Haryanka dynasty. He shifted the capital of Magadha
(e) None of the above/more than above from Rajgriha to Pataliputra.

65th BPSC (Pre) (PH) Re-Exam. 2019 267 YCT


7. Which of following pair is not correct? Ans. (b) : Humayu-nama is the biography of Humayun
(Author) (Book) and it was written by Mughal Emperor Babur's younger
(a) Dandi Dashkumarcharita daughter and Humayun's sister, Gulbadan Begum after
(b) Vishnu Sharma Panchtantra the death of Humayun. It is written in Persian language.
(c) Aryabhattha Brihat Samhita Hence statement v is incorrect and rest of all are correct.
(d) Bhas Swapnavasvadata 11. In Shivaji's reign the name of the foreign
(e) None of the above/more than above minister ........... was known as?
Ans. (c) : Brihat-Samhita was an influential (a) Secretary
encyclopedic text in Sanskrit, written by Varahamihira. (b) Minister
The other work of Varahamihira is 'Panchasiddhantika' (c) Amatya
Aryabhattha wrote the book - 'Aryabhatiya'. (d) Sumant
8. Which was built by Vigraha Raj-IV before the (e) None of the above/ more than one of the
mosque named "Adhai Din ka Jhopara" in above
Ajmer? Ans. (d) : During the reign of Maratha Emperor Shivaji,
(a) College the foreign minister was known as Sumant. Shivaji had
the council of ministers namely Asht-Pradhan, who
(b) Temple
helped him in the administration. These Asht-Pradhans
(c) Garib-khana were :
(d) Dharamshala 1. Peshwa – Prime Minister
(e) None of the above/more than one of the 2. Sumant – Foreign Minister
above 3. Amatya – Finance minister
Ans. (a) : Adhai Din Ka Jhopra is a large and imposing 4. Sachiv – Secretary
structure in the City of Ajmer in Rajasthan, India. It was 5. Sar-i-Naubat – Senapati or chief of army
a originally a Sanskrit College built by Vigraharaja IV, 6. Waqia-Navis – Home Minister
a famous King of the Shakambhari Chauhan dynasty. 7. Nyayadhish – Chief Justice
According to a tablet found at the Site, the original 8. Pandit Rao – Priest
College building must have been constructed before 12. With whose help Hyder Ali established a
1153 CE. modern armoury at Dindigul in 1755?
(a) Dutch experts (b) British experts
9. Who founded the Chishti sect of Sufi in India?
(c) Spanish experts (d) French experts
(a) Baba Farid (e) None of the above/ more than one of the
(b) Sheikh Bahauddin Zakaria above
(c) Moinuddin Chishti Ans. (d) : Haider Ali was the king of Mysore. He
(d) Khwaja Baqi Billah established a modern armoury at Dindigul in 1755 with
(e) None of the above/ more than one of the the help of French experts.
above 13. Where was the first Portuguese factory
Ans. (c) : The Chishti sect was established in India established in India?
around 12th century AD, By Khwaja Moinuddin Chisti. (a) Surat (b) Goa
He was also known as Gharib Nawaz. The Chishti sect (c) Calicut (d) Chandan Nagar
is known for its emphasis on love, openness and (e) None of the above/ more than one of the
tolerance. above
10. Which of the following statements are/is Ans. (c) : Vasco-De-Gama was the first European who
correct? discovered the sea route from Europe to India via Cape
of Good Hope. He reached the port of Calicut in 1498
I. Kalhana's book 'Rajatarangini is in
and was received by Hindu ruler of Calicut Zamorin
sanskrit (Title). The first factory of the Portuguese was
II. Ibn Battuta was appointed Qazi of Delhi established in Calicut in 1500 which was later closed by
III. Padshahnama of Abdul Hamid Lahori is Zamorin. They (Portuguese) established their second
about Shah Jahan's governance. factory in 1501 at Cannanore. Francisco De Almeida
IV. Amir Khusro was the inventor of sitar. (1505-09) was the first Portuguese governor in India.
V. Humayunama was written by Hamida 14. Home charges or the expenses of the Indian
Banu Begum. Government in Britain included?
(a) Only I and II (a) Pension of British Indian officers
(b) Military and other stores purchased in India.
(b) Only I, II, III and IV
(c) Guaranteed interest of Railways.
(c) Only II, III and V (d) All of the above
(d) I, II, III, IV and V (e) None of the above/more than one of the
(e) None of above/more than above above
65th BPSC (Pre) (PH) Re-Exam. 2019 268 YCT
Ans. (e) : During the British rule in India. Britain's Ans. (e) : In 1876, Surendranath Banerjee and Anand
trade surplus in India helped to pay the home charges or Mohan Bose founded Indian Association. It was the
domestic expenses that included private remittances first Indian Nationalist organization. Initially it was
sent to home by British officials and traders, Pensions established as Bharat Sabha and its first annual
of British officials of India and interest payment on conference held in Calcutta. Later it was known as
India's external debt. Indian Association.
Hence, option (e) is the correct answer. Hence, option (e) more than one option is correct.
15. Who started the Ryotwari system in Madras? 19. In which year an Indian woman became the
(a) Lord cornwallis president of Indian National congress (INC) ?
(b) Thomas Munro (a) 1917
(c) Lord William Bentinck (b) 1918
(d) Lord Wellesley
(c) 1920
(e) None of the above/more than one of the
(d) 1924
above
(e) None of the above/more than one of the
Ans. (b) : Ryotwari system was devised by Thomas
above
Munro and Captain Read at the end of the 18th century
and introduced by latter when he was governor of Ans. (e) : Sarojini Naidu was the first Indian women
Madras presidency (1819-26). There were mainly three President of Indian National Congress (INC) at Kanpur
types of land revenue system in existence during the session (1925). She was also the first women Indian
British India. These are – The Ryotwari system, The governor after independence. She was the governor of
Zamindari system and The Mahalwari system. Under Uttar Pradesh. The first woman president of INC was
the Ryotwari system, the land revenue was paid by the
Annie Besant who presided at Calcutta session in 1917.
farmers directly to the state.
20. Which of the following statements are correct?
16. Who said that "the event of 1857, started as a
I. Laiq Ali was the president of the executive
struggle of religion and that event ended in the
council of the Nizam.
form of freedom struggle"?
II. The 'Operation Polo' lasted for 48 hours
(a) Dr. S. N. Sen
against Hyderabad.
(b) Ashok Mehata
III. "The Instrument of Accession" was signed
(c) V. D. Sawarkar
by the state of Kashmir on 15 August 1947.
(d) Dr. R. C. Majumdar IV. Shah Nawaj Bhutto was the last Diwan of
(e) None of the above/more than one of the Junagarh state.
above V. K. M. Munshi was appointed by the
Ans. (a) : Dr. Surendra Nath Sen wrote in his book, Government of India as its agent in
"Eighteen Fifty Seven" that, the event of 1857, started Hyderabad.
as a struggle of religion and ended in the form of (a) Only I, IV and V
freedom struggle". Dr. S. N. Sen concluded that the (b) Only I, II and III
revolt of 1857 was "Not an organized national revolt". (c) Only II, III and IV
17. The peasant leader Motilal Tejawat was related (d) Only I, II, III and IV
to? (e) None of the above/more than one of the
(a) Bijolia movement above
(b) Bheel Tribal movement of Mewar Ans. (a) : Hyderabad became the part of India in 1948
(c) Peasant movement of Marwar with the help of militarily operation named as 'operation
(d) Peasant movement of U.P. polo’. It was a 5 days operation, initiated on 13
(e) None of the above/more than one of the September and lasted on 17 September 1948, so
above statement II is incorrect. 'The Instrument of Accession'
Ans. (b) : The prominent peasant leader Motilal is a legal document executed by Maharaja Hari Singh
Tejawat was related to Bhil Tribal Movement of Mewad (Ruler of Jammu & Kashmir) on 2 October 1947, so
region of Rajasthan. He was also leader of Eki statement III is incorrect. Rest of all are correct.
movement which was agitated in the 1920s in the Hence. Option (a) is correct.
Aadivasi areas presently in Rajasthan and Gujarat. 21. Who was the president of Indian National
18. Who was the leader of Indian association? Congress of Gaya Session in 1922?
(a) Surendranath Banerjee (a) C. R. Das
(b) Anand Mohan Bose (b) S. P. Sinha
(c) Dwarkanath Ganguly (c) Dr. Rajendra Prasad
(d) R. C. Dutta (d) Ras Bihari Bose
(e) None of the above/more than one of the (e) None of the above/more than one of the
above above
65th BPSC (Pre) (PH) Re-Exam. 2019 269 YCT
Ans. (a) : The Indian National Congress was (c) Ashwani Kumar Dutta
established in 28 December 1885 by A.O. Hume. The (d) Harkishan Lal
1st session of INC was held in Bombay in 1885 under (e) None of the above/more than one of the
the presidency of Womesh Chandra Bannerjee. The 38th above
INC session was held at Gaya in 1922 and the president Ans. (c) : In 1897, at Amravati session of INC, Ashwini
of that session was Chittaranjan Das. Chittaranjan Das Kumar Dutt said that, "it is a three-days Buffoonery''.
is popularly known as Deshbandhu. C.R. Das and
Motilal Nehru formed the Swaraj Party in January 1923. 26. Who was/were the person related to
'Zimmerman' scheme?
22. Who raised the question of the Indigo growers
(a) Lala Hardayal
of Bihar in the Lucknow session of Indian
National congress, 1916? (b) Bhoopendra Nath Dutta
(a) Raj Kumar Shukla (c) Virendra Nath Chatopadhyaya
(b) Dr. Rajendra Prasad (d) All of the above
(c) J B. Kripalani (e) None of the above/more than one of the
(d) Dr. Sampurna Nand Sinha above
(e) None of the above/more than one of the Ans. (d) : Bhupendranath Dutt, Lala Hardayal and
above Viredra Nath Chatopadhyay established the 'Berlin
Ans. (a) : The Lucknow Session of Indian National Committee for Indian Independence under the
Congress was held in 1916 in Lucknow (U.P) and the Zimmerman scheme in 1915. The aim of the
president was Ambika Charan Mazumdar. In 1916, revolutionaries was to inspire Indians to conduct
Lucknow session of INC, Raj Kumar Shukla raised the revolutionary activities by staying in India, supplying
question of the Indigo growers of Bihar. After this arms and somehow gain Independence of India from
session, the Bihar delegate, Raj Kumar Shukla colonial rule.
particularly urged Gandhiji to visit Champran to witness 27. Simon Commission was appointed under whose
the miseries of the riots. chairmanship?
23. Who was the editor of Hindoo Patriot? (a) Sir John Simon
(a) Harish Chandra Mukherji (b) Clement Attlee
(b) Deenbandhu Mani (c) Harry Levy-Lawson
(c) Babul Mani (d) Edward Cadogan
(d) Naveen Chandra Mani (e) None of the above/more than one of the
(e) None of the above/more than one of the above
above Ans. (a) : Simon commission was appointed by the
Ans. (a) : The 'Hindoo Patriot' was an English weekly British government in 1927 under the chairmanship of
magazine founded in 1853 by Madhusudhan Ray, under Sir John Simon-to review the working of government of
the editorship of Girish Chandra Ghosh in (1853-55). India Act 1919 and suggest future measures. Simon
Harish Chandra Mukherjee was the editor of 'Hindoo Commission consisted of seven British members.
Patriot from 1855 to 1861.
28. Who started the 'Hindustani Sewa-Dal' in the
24. When did the Muslim league accept the mid of 1920's ?
objective called self-government under the (a) Jawahar Lal Nehru
British Empire?
(b) Subhas Chandra Bose
(a) 1905
(c) N. S. Hardikar
(b) 1913
(d) N. G. Ranga
(c) 1930
(e) None of the above/more than one of the
(d) 1946
above
(e) None of the above/more than one of the
above Ans. (c) : The Hindustani Seva Dal was one of the
organization of the INC. It was founded by Dr. Narayan
Ans. (b) : The Muslim league was founded in 1906 at
Subharao Hardikar (Dr. N.S. Hardikar)
Dhaka. In 1913, the league amended its constitution to
accept the objective of self-government under the aegis 29. What was proposed in the 'Bombay Plan' ?
of the British crown as the ideal of the Muslim League. (a) Rapid economic growth
25. Who called 'three-day Buffoonery' of the 1897, (b) Equitable distribution
Amaravati session of Indian National (c) Abandonment of certain capitalist features.
Congress? (d) Employees/workers welfare scheme
(a) Jatindra Nath Banerjee (e) None of the above/more than one of the
(b) Satish Mukherjee above
th
65 BPSC (Pre) (PH) Re-Exam. 2019 270 YCT
Ans. (e) : The Bombay Plan was proposed in 1944. It 34. In which period the Deccan Trap plateau was
was also called as Industrialist Plan. This plan was formed?
drafted by Eight industrialists of Bombay. Such as (a) Tertiary Period
J.R.D Tata, GD Birla, Purshottamdas Thakurdas, etc. It (b) Permian Period
was a brief memorandum of economic development for
(c) Cretaceous Period
India. Main focus in this plan was rapid economic
growth and doubling the agricultural sector's equitable (d) Triassic Period
distribution and some welfare schemes. The plan (e) None of the above/more than one of the
published in 1944 by eight leading Indian industrialists, above
proposed state intervention in economic development of Ans. (c) : The Deccan Trap plateau was formed in the
the nation after independence. Cretaceous period. Deccan Traps are special type of
Hence, correct option is (e), more than one options are structure. They are formed from basaltic rocks.
correct.
Basically, they developed in 'Cretaceous to the Eocene
30. Who established the Jamsedpur labour Era'. Black soil is formed by the erosion of these rocks.
association in 1920?
Their expansion has increased immensely in the state of
(a) S. N. Haldar
Maharastra and Gujarat.
(b) Byom Kesh Chakraborty
(c) Ramesh Paul 35. Consider the following statements and identify
(d) Swami Vishwanand the correct statement?
(e) None of the above/more than one of the I. Greater Himalaya is known for its
above continuity.
Ans. (a) : Jamshedpur Labour Association was started II. The lower Himalaya is known for its
in 1920 by S. N. Haldar. It was initiated in Jamshedpur broadness and transverse valleys.
with the workers of TATA Steel Industry. III. Shivalik is the outer Himalaya which is
31. Who wrote 'Kisaan kya karein ? latest in the origin.
(a) Swami Sahajanand Saraswati (a) Only I and II
(b) Karya Nand Sharma (b) Only II and III
(c) Baba Ram Chandra (c) Only I and III
(d) All of the above (d) All of the above
(e) None of the above/more than one of the (e) None of the above/more than one of the
above above
Ans. (a) : Swami Sahajanand Saraswati was a Ans. (c) : The lower Himalayan Mountains are 60 to 80
nationalist and peasant leader of India. He wrote and km broad and 2400 km long. The lower Himalayas have
published 'Kisan Kya Karein'. Sahajanand Saraswati steeps, barren southern slopes and forested northern
established 'Bihar Provincial Kisan Sabha' in 1929. slopes. Hence, statement II is incorrect and rest of all
32. Who presided the 27th Indian National congress are correct.
session? 36. At which of following place River Koshi merge
(a) S. N. Sinha in the River Ganga?
(b) R. N. Mudholkar (a) Kursela
(c) Majrul-Haque (b) Vaishali
(d) Rajendra Prasad (c) Hajipur
(e) None of the above/more than one of the (d) Jamalapur
above (e) None of the above/more than one of the
Ans. (b) : The 27th session of Indian National Congress above
was presided by Raghunath Narasinha Mudholkar. It Ans. (a) : The river Koshi merges in the river Ganga
was held at Bankipur (Patna) in 1912. near Kursela (Katihar, district of Bihar). River Koshi is
33. Who published ''Mother Land" ? also known as Saptkoshi for its seven tributaries. It is
(a) Rajendra Prasad called as Sorrow of Bihar because it causes severe flood
in Bihar.
(b) Mohammad Safi
(c) Majrul-Haque 37. Select the forests found in the Western Ghats,
(d) Braj Kishore Prasad the hills of the North-Eastern region and the
Andaman and Nicobar Islands.
(e) None of the above/more than one of the
(a) Tropical thorn forest
above
(b) Tropical deciduous forest
Ans. (c) : "The Mother land" was the weekly magazine (c) Tropical evergreen forest
in English language. Its publication was started in 1921 (d) Mountain forest
from Sadaqat Ashram, Patna, Bihar by Maulana (e) None of the above/more than one of the
Mazharul Haque. above
65th BPSC (Pre) (PH) Re-Exam. 2019 271 YCT
Ans. (c) : Tropical Evergreen Forest are usually found Ans. (d) : The value of the gravity on Moon is 1/6th of
in areas receiving more than 200 cm of rainfall and the gravity of the Earth. Therefore, the object whose
having a temperature of 15 to 30 degree Celsius. In weight comes 100N when measured on the surface of
India they are mainly found in western slopes of 2
Western Ghats, hilly areas of North-East states and the Earth, then its weight will be 16 or 16.666 N or
Andaman Nicobar Islands. Tropical Evergreen Forests 3
about 16.7 N when measured of the surface of the
have trees-like rosewood, Mahogany and ebony etc. moon.
38. In which of the following states the forest cover Hence, option (d) is correct answer.
is not more than 75%? 42. Which of the following is the brightest planet in
(a) Mizoram the solar system?
(b) Nagaland (a) Earth
(c) Meghalaya (b) Uranus
(d) Madhya Pradesh (c) Neptune
(d) Venus
(e) None of the above/more than one of the
(e) None of the above/more than one of the
above above
Ans. (d) : As per the Forest Survey Report 2021, in Ans. (d) : Venus, the second planet from the Sun, is the
terms of area, Madhya Pradesh has the largest forest hottest and brightest planet in the Solar System.
cover in the India but in terms of percentage it is less 43. Which of the following is the largest mountain
than 75% of its area. The percentage coverage of – range?
(a) The Himalayas
Mizoram (84.53%), Nagaland (73.90%) and Meghalaya (b) The Alps
(76%) (c) The Rockies
39. The Tummalapalle mine, which is considered (d) The Hindukush range
to be the world's largest Uranium reserves, is (e) None of the above/more than one of the
located at? above
(a) Andhra Pradesh Ans. (c) : Among the given options, The Rockies
mountain (4839 km) is the largest mountain range that
(b) Tamil Nadu stretch from Canada to central new Mexico. The Andes,
(c) Karnataka South American continent mountain range is the world's
(d) Kerala largest mountain range.
(e) None of the above/more than one of the 44. Which is the driest Desert in the world?
above (a) Sahara
Ans. (a) : As per the research of Atomic Energy (b) Sonora
Commission of India in 2011, the Tummalapalle mine (c) Taklimakan
(d) Atacama
might have one of the largest reserves of Uranium in the
(e) None of the above/more than one of the
world. This mine is located at Kadapa in the state of above
Andhra Pradesh, India. Ans. (d) : World's driest desert is Atacama. It is situated
40. Under which of the following pass, Jawahar in South American continent between Andes mountain
Tunel has been constructed? range and Chilean coast mountain range, the Atacama is
(a) Shipki La Pass also the largest fog desert in the world. Sahara is the
(b) Banihal Pass largest desert in the world.
(c) Baralacha La Pass 45. Which country has about half of the world's,
(d) Rohtang Pass total number of large dams?
(a) India
(e) None of the above/more than one of the (b) Russia
above (c) United states of America
Ans. (b) : The Jawahar Tunnel has been constructed (d) Canada
under Banihal pass. It is called as Banihal tunnel. It is (e) None of the above/more than one of the
situated in the Range of Pir-Panjal, Jammu and above
Kashmir. The length of the tunnel is 2.8 km. Ans. (e) : China has the largest number of dams which
is about one half of the total numbers of dams in the
41. The weight of an object when measured on the world. China has total 23,841 dams followed by USA
surface of the earth is 100N. What would be its (9263) second and India (5202) respectively.
weight when measured on the surface of the 46. The most abundant gas in the Earth's
Moon? atmosphere is?
(a) 1.67 N (a) Hydrogen
(b) 100 N (b) Oxygen
(c) 10 N (c) Carbon dioxide
(d) 16.7 N (d) Nitrogen
(e) None of the above/more than one of the (e) None of the above/more than one of the
above above
65th BPSC (Pre) (PH) Re-Exam. 2019 272 YCT
Ans. (d) : Nitrogen (N2) gas is the most abundant gas in Ans. (d) : In the Constitution of India the procedure of
the Earth's atmosphere, it is around 78.08%. In the presidential election is by the electoral college system.
Earth's atmosphere the composition of gases is as : The electoral college is made up of all the elected
• Nitrogen – 78.08% members (not appointed) of Parliament (Lok Sabha and
• Oxygen – 20.95% Rajya Sabha) and the elected members of the
• Carbon dioxide – 0.036% Legislative Assemblies of States and Union Territories
(MLA's). The election of President is conducted by
47. Kraal is a type of Built structure from? Election Commission of India.
(a) South Africa 52. Who presides over the joint session of the
(b) South American Parliament?
(c) Eastern Europe (a) President
(d) Western Australia (b) Vice-President
(e) None of the above/more than one of the (c) Speaker of Lok Sabha
above (d) Prime Minister
Ans. (a) : Kraal is built structure basically for the cattle (e) None of the above/more than one of the
or live stock, built by South African tribal groups. above
48. When was the provision of Article 35-A Ans. (c) : Article 108 of the Indian Constitution,
included in the Indian constitution? provides joint session of both Houses. The joint session
(a) 1947 of Parliament is called by the President of India and
(b) 1950 presided over by the Speaker or, in his absence, by the
(c) 1953 Deputy Speaker of the Lok Sabha or in his absence the
(d) 1954 Deputy Chairman of the Rajya Sabha. The Chairman of
(e) None of the above/more than one of the Rajya Sabha doesn’t preside over the joint session at
above any means/cost.
Ans. (d) : Article 35A, related to special rights and 53. The executive power of the central government
is vested in the constitution by?
privileges to citizens of Jammu and Kashmir was (a) Constitution
included in the constitution of India in may 1954 by an (b) President
order of the then President Rajendra Prasad on the (c) Governor
advice of Jawarharlal Nehru Cabinet. (d) Prime Minister
49. The grounds of Judicial review in India is? (e) None of the above/more than one of the
(a) Due Process of Law above
(b) Procedure established by Law Ans. (b) : As per the article 53(1), of the Indian
(c) Rule of law Constitution, the executive power of the central
(d) Procedure established by Parliament government is vested mainly in the President of India.
(e) None of the above/more than one of the The President can exercise the power directly or
above through subordinate officers to him in accordance with
Ans. (b) : The Judicial review in Indian Constitution is this constitution.
based on the "procedure established by Law". It is 54. At Present the number of judges in the
adopted from the constitution of America. The Judicial Supreme Court is?
review in American constitution is based on, "due (a) 15 (b) 25
process of law". (c) 31 (d) 20
50. The special election to fill the vacant seat of (e) None of the above/more than one of the
Legislature is called? above
(a) General election Ans. (e) : At present the number of Judges in the
(b) By-election Supreme Court, including the Chief Justice of India, is
(c) Mid-term election 34. It has been increased by the Supreme Court
(d) Referendum (Number of Judges) Amendment Act, 2019, from 31 to
(e) None of the above/more than one of the 34. The current Chief Justice of India is Nuthalapati
Venkata Ramana (N.V. Ramana). He is the 48th Chief
above Justice of India.
Ans. (b) : The special election which is held to fill the 55. 124th Constitutional Amendment is related to?
vacant seat of legislature is called as By-election. By- (a) To the Higher Education
elections are held if a member of Parliament resigns or (b) To the Reservation
dies or is disqualified to be member of the house under (c) To N.R.C.
the tenth schedule. (d) To the Triple Talak
51. The election of president in India is carried by? (e) None of the above/more than one of the
(a) By Rajya Sabha above
(b) By the people of India Ans. (b) : The 124th Constitutional Amendment Bill,
(c) By the member of both the houses of the 2019, is related to the 10 percent reservation in jobs and
Educational Institutions to the Economically Weaker
Parliament Section (EWS) by inserting Article 15(6) and Article
(d) By the Parliament and state legislature 16(6) in the Indian Constitution. It became the 103rd
(e) None of the above/more than one of the Constitutional Amendment Act after the signing of
above President on the bill.

65th BPSC (Pre) (PH) Re-Exam. 2019 273 YCT


56. On the basis of which constitutional 60. The unit of Electric Current is?
amendment Panchayati Raj was given a (a) Col
constitutional status? (b) volt
(a) 74th Constitutional Amendment (c) watt
(b) 73rd Constitutional Amendment (d) Ampere
(c) 72nd Constitutional Amendment (e) None of the above/more than one of the
(d) 71st Constitutional Amendment above
(e) None of the above/more than one of the Ans. (d) : The SI unit of electric current is ampere (A).
above Electric current is a rate of flow of electric charges.
Ans. (b) : On 24th April, 1993, the Panchayati Raj was Charge(q)
given constitutional status by effecting the 73rd Current(I) =
Constitutional Amendment Act, which was passed in time(t)
1992. It inserted Part-IX consisting of Article 243 to 61. Visual Impairment "Myopia ......., is known as?
243(O), and the Eleventh Schedule enumerating 29 (a) Hyper Metropia
functional items. (b) Presbyopia
57. Which of the following political organisation (c) Glaucoma
seats are reserved for women in India? (d) Cataract
(a) Panchayati Raj (e) None of the above/more than one of the
(b) State Legislature above
(c) Rajya Sabha Ans. (e) : The visual impairment or Myopia, also
(d) Lok Sabha known as nearsightedness, is a very common vision
(e) None of the above/more than one of the disorder that is usually diagnosed before age 20. In
above Myopia the eyeball is too long for the normal focusing
Ans. (a) : Women Reservation is provided in power of the eye, as a result images of distant objects
Panchayats. As per provisions contained in Article appear blurred.
243D of the constitution, 1/3rd of the seats of 62. With reference to LED (Light Emitting Diode)
Panchayati Raj and 1/3rd offices of the chairperson at which statement is not Correct?
all level of Panchyati Raj covered by Part IX of the (a) LED light sources consume less Energy
Constitution are reserved for women. (b) LED are more durable than light bulbs
58. Who takes decision for conducting Panchayat (c) LED contain mercury which is harmful to
Election? humans
(a) Central Government (d) LED is used in traffic signal lights
(b) State Government (e) None of the above/more than one of the
(c) District Magistrate above
(d) Election Commission Ans. (c) : The term LED stands for Light Emitting
(e) None of the above/more than one of the Diode. It is a source of less energy consuming. It is
above more durable and has extended life span than light
bulbs. LED's are mostly used in Automotive lightening
Ans. (b) : Panchayat elections are independent from the and traffic signal lights. LED is a semiconductor device
Election Commission of India. Article 243(K) of the that emits light when electric current is passed through
Indian Constitution established under the 73rd it. It does not contain mercury. Hence, option C is not
Constitutional Amendment Act, provides that all correct.
elections to the Panchayats shall be vested in the state 63. Which of the following is used in Solar furnace?
Election Commissions of each state. (a) Plane mirror
59. When did the community Development (b) Concave mirror
programme start? (c) Convex mirror
(a) 2nd October 1952 (d) Concave Lens
(b) 2nd October 1954 (e) None of the above/more than one of the
(c) 18th May 2016 above
(d) 2nd October 1959 Ans. (b) : Concave mirror converges the parallel sun
(e) None of the above/more than one of the rays at a point. It helps to increase the temperature of
the furnace that can reach up to 3500°C. So concave
above
mirror is basically used in the designing of solar
Ans. (a) : Community Development Programme was furnace.
the first major development programme which was
64. What is the optical instrument used to see
launched on 2nd October 1952, In India. The CDP minimal objects?
(Community Development Programme) was the Multi- (a) Microscope
Project Programme with the aim of the overall (b) Electroscope
development of rural people. It consisted of agriculture, (c) Spectroscopy
animal husbandry, irrigation, public health, education (d) Stethoscope
etc. (e) None of the above/more than one of the
above
65th BPSC (Pre) (PH) Re-Exam. 2019 274 YCT
Ans. (a) : A microscope can be used to see minimal Ans. (b) : Physical Changes are the reversible process.
(Small/micro) objects. It gets a clear magnified view of In Physical change, the substances involved do not
microorganisms like cells, bacteria and amoeba. Two change their original form. They can be easily returned
Dutch spectacle makers, Hans and his son Zacharias to their original form by some physical changes for
Janssen, create the first microscope in the late 16th example- freezing of water, boiling of water, making a
century. Antonie van Leeuwenhoek was the first to solution, etc.
observe bacteria under microscope. 70. Which of the following is a non-ferrous metallic
65. Ultrasonic waves are? mineral?
(a) Frequency of Sound wave, less than 20 kHz (a) Manganese
(b) Frequency of Sound wave, more than 20 kHz (b) Nickel
(c) Frequency of Sound wave, between 20 Hz (c) Cobalt
and 20kHz (d) Tin
(d) Electro-magnetic waves (e) None of the above/more than one of the
(e) None of the above/more than one of the above
above Ans. (e) : There are several kind of mineral, basically
Ans. (b) : Ultrasonic waves are the frequency of sound ferrous minerals and non-ferrous minerals. Ferrous
wave which are greater than the audible range for minerals are metallic minerals containing iron-example-
human (more than 20 kHz). Manganese, Nickel, cobalt, Tin, Iron etc, and Non-
66. Which of the following metals has the highest ferrous Metallic minerals are that do not contain iron
electrical conductivity? example - Gold, copper, zinc, etc.
(a) Gold 71. The glass which is used in tube lights and test
(b) Silver tubes, is made up of ?
(c) Copper (a) Silica
(d) Iron (b) Calcium Carbonate
(e) None of the above/more than one of the (c) Sodium Carbonate
above (d) Pieces of Glass
Ans. (b) : Silver has the highest electrical conductivity. (e) None of the above/more than one of the
It is the most electrically conductive metal on the Earth. above
Second most electrically conductive metal is Copper. Ans. (e) : The tube lights and test tubes are made up
67. The element commonly used to generate from expansion resistant type glasses, mostly
electricity from nuclear fission in nuclear Borosilicate (Silica + boron trioxide) glass or fused
reactor is? quartz. Borosilicate glasses are more durable and
(a) Radium resistant to heat. It is used in higher temperature lamp.
(b) Plutonium 72. Alcohol can be converted into carboxylic acid
(c) Uranium by?
(d) Deuterium (a) Only by Oxidation
(e) None of the above/more than one of the (b) Only by Reduction
above (c) By esterification
Ans. (c) : The fuel pellets of Uranium (U235) is used to (d) Both Oxidation and Reduction
produce electricity in nuclear reactor by nuclear fission. (e) None of the above/more than one of the
U235 is bombarded with neutron particles and nucleus above
breaks down into two nuclei then more neutron particles Ans. (a) : Alcohols are converted into carboxylic acid
are used to break more uranium atoms resultantly chain by the process of oxidation. Primary alcohols are
reaction occurs which helps to generate electricity. oxidized to carboxylic acids in two stages, first to an
68. Boiling point of liquids at high altitude? Aldehyde and then to the acid.
(a) Increases
(b) Decreases
(c) Remain constant
(d) Decrease after increasing
(e) None of the above/more than one of the
above
Ans. (b) : At higher altitude where the atmospheric 73. Which one of the following is the best fire-
pressure is lower, the Vapour pressure is also lower extinguisher?
which in turn creates a lower boiling point (decrease). (a) Soil
Cooking at high altitude takes more time than cooking (b) Water
at surface (sea level). (c) Oxygen
69. According to chemistry, which of the following (d) Carbon dioxide
is an example of physical change? (e) None of the above/more than one of the
(a) Rusting of Iron above
(b) Boiling of water Ans. (d) : Carbon dioxide is known as the best fire
(c) Burning of wood extinguisher because it stops the influence of oxygen
(d) Green leveling on Copper utensils and decreases the oxygen present in that fire which
(e) None of the above/more than one of the leads to the put off the fire.
above CO2 is a non-combustible gas.
65th BPSC (Pre) (PH) Re-Exam. 2019 275 YCT
74. Fullerene or Bucky balls are, composed of how (d) Carrot
many carbon atoms? (e) None of the above/more than one of the
(a) 20 (b) 40 above
(c) 45 (d) 50
(e) None of the above/more than one of the Ans. (a) : The plants which store food in their stem are
above ginger, onion, banana, potato, bamboo, pineapple,
Ans. (e) : Fullerenes belong to the carbon-only cactus etc.
molecules family, and it is composed of 60 carbon Note:- Plants that produce underground modifications
atoms (C60) in form of a ball like structure. They are of stem utilize it for permeation and storage of food.
cluster of carbon. Fullerene was discovered by Richard 79. Which of the following cell organelles destroy
E. Smalley and F. Curl of Rice University, Houston and bacteria entering the cell?
Harold W. Kroto of University of Sussex, USA in 1985. (a) Ribosome
Fullerenes are also known as Bucky ball and carbon (b) Golgi body
nanotubes. (c) Lysosome
75. Which of the following phenomenon increase (d) Endoplasmic Reticulum
oxygen level in the carbon cycle taking place on (e) None of the above/more than one of the
earth? above
(a) Respiration Ans. (c) : A Lysosome is a membrane-bound cell
(b) Volcanic Eruption organelle that contains digestive enzyme. It involves in
(c) Decay of Organic Matter various cell processes. Lysosome destroys invading
(d) Photosynthesis viruses and bacterias. Lysosomes are known as Suicide
(e) None of the above/more than one of the Bags of the cell.
above 80. Iron deficiency causes in the body?
Ans. (d) : The leading producers of oxygen are plants (a) Anaemia
and sunlight by the process of photosynthesis. It is a (b) Scurvy
biological process in which green plants synthesize their (c) Osteoporosis
food in presence of Sunlight and produce energy and (d) Skin disease
oxygen. (e) None of the above/more than one of the
76. Which of the following is a vascular plant? above
(a) Moss Ans. (a) : Anaemia is a condition in which the number
(b) Liverwort of Red Blood Cells or their oxygen carrying capacity is
(c) Hornwort reduced. Basically Anaemia is caused by the deficiency
(d) Fern of Iron. Some other factors also causes Anaemia i.e.
(e) None of the above/more than one of the deficiency of vitamin B12, folate and vitamin B9.
above 81. Ideal temperature of Human body is
Ans. (d) : Vascular Plants are also called (a) 99.6°F
Tracheophytes, which means duct or vessel in Plants. (b) 97.4°F
The vascular plants have vascular tissues including (c) 98.6°F
xylem (Conducting water) and Phloem (integrating (d) 95.6°F
food). Vascular plants include all the seed containing (e) None of the above/more than one of the
plants as: Ferns, Angiosperms and Gymnosperms. above
77. Which of the following is a perfect example of Ans. (c) : In general, average normal body temperature
Organic fertilizer: of human body is considered as 98.6°F or 37°C. In
I. Mycorrhizal fungi adult, the average of human body temperature is 97°F to
II. Blue Green Algae 99°F.
III. Bacillus thuringiensis 82. In which organ of Human body is epithelial
(a) Only I cells are found?
(b) Only I and II (a) Hair
(c) Only II (b) Nose
(d) Only III (c) Lungs
(e) None of the above/more than one of the (d) Pancreases
above (e) None of the above/more than one of the
Ans. (b) : Organic fertilizers are the by-product of plant above
and animal manures. Organic fertilizers are substances
Ans. (e) : Epithelium is one of the body cell that forms
that contain microbes. It Comprises of living organisms
which include Mycorrhizal fungi, Blue Green Algae and the covering on all internal and external surfaces of
Cyno bacteria. Bacillus thuringiensis is Pesticide. body. There are various kinds of epithelial tissue
Hence, option (b) only statement I and II are correct. throughout the body. Which are - outer layer of skin
78. Which of the following plants has modified its (epidermis), Intestine, Lining of respiratory tract,
stem to store food materials? abdominal cavity and etc,
(a) Ginger Hence, the correct answer is option (e).
(b) Radish 83. Which organ of the Human body affected by
(c) Beet Typhoid disease?
65th BPSC (Pre) (PH) Re-Exam. 2019 276 YCT
(a) Lung 87. In which year Hon'ble Judge Shri Rakesh
(b) Spinal cord Kumar took over as a permanent Judge in
(c) Body Joints Patna High Court?
(d) Intestine (a) 2003
(e) None of the above/more than one of the (b) 2011
above (c) 2013
Ans. (d) : Typhoid is a bacterial disease that mainly (d) 2015
spreads by contaminated water and food. Intestine (e) None of the above/more than one of the
(small intestine) of the human body is affected by above
Typhoid disease. Ans. (b) : Hon'ble Judge Mr. Rakesh Kumar was
84. Organic farming is a farming system in which? appointed as an additional Judge of Patna High court on
(a) Maximum chemicals are used in the form of 25th December 2009, and took oath as a Permanent
fertilizers, herbicides and pesticides. judge on 24th October 2011.
(b) Minimum use of organic manures, recycled 88. Who is the Union Minister of state for Home
farm wastes and bio-factors. Affairs from Bihar?
(a) R. K. Singh
(c) Maximum use of organic manures, recycled
(b) Nityanand Rai
farm wastes and bio-factors. (c) Ashwani kumar
(d) Cultivation of two or more than two crops (d) R. V. Paswan
together in the same field. (e) None of the above/more than one of the
(e) None of the above/more than one of the above
above Ans. (b) : Nityanand Rai is an Indian Politician and
Ans. (c) : Organic farming is a system of farming that current Union Minister of state for Home Affairs from
creates an ecosystem of agriculture production without Bihar.
the use of chemical fertilizers, pesticides, and synthetic 89. In which movement did the farmers of Bihar
hormones. In this farming method, the maximum use of refused to pay 'Chowkidari Tax"?
organic manures and biological fertilizers are preferred. (a) Champaran Satyagraha
85. Abbreviation of the ISRO is? (b) Non-Cooperation movement
(a) Indian Satellite Research Organization (c) Swadeshi Movement
(b) Indian Scientific Research Organization (d) Civil-disobedience movement
(c) Indian Space Research Organization (e) None of the above/more than one of the
(d) International Scientific Research above
Organization Ans. (d) : The Chowkidari Tax was collected by the
(e) None of the above/more than one of the secretary of the state. It was collected from the farmers
above for the compensation of village watchmen and that
affected the poor farmers. In 1930, the farmers refused
Ans. (c) : The ISRO, stands for, Indian Space Research to pay this tax during the Civil Disobedience
Organisation. It was formed on 15 August 1969 and its Movement.
headquarters is in Bangalore. It operates under 90. Which of the following is the latest in the
Department of Space and which is directly over seen by Geological history of Bihar?
the Prime Minister of India. The current (January 2022) (a) Dharwar-region of south-east
chairman of ISRO is S.P. Somanath. (b) Plateau region
86. Which global conference was the first step (c) Plains of Bihar
towards a global agreement for the reduction (d) Vindhya region of South-east
(e) None of the above/more than one of the
of greenhouse gas emissions? above
(a) Vienna (1985)
Ans. (c) : The plain of Bihar is the latest in the
(b) Montreal (1987)
Geological history of Bihar. Bihar has been divided into
(c) Rio (1992) three parts on the basis of physical and structural
(d) Kyoto (1997) condition, i.e. the Southern Plateau Region, the Shivalik
(e) None of the above/more than one of the Region and the Gangetic Plain.
above
91. Old alluvial soil or Bangar soil is found in
Ans. (c) : The first Rio Earth Summit was organized by which of the following areas of Bihar?
the United Nations in 1992 in Rio de Janeiro, the capital of (a) Patna and Gaya
Brazil. In this, a comprehensive action plan 'Agenda-21' (b) Purania and Gaya
for sustainable development was approved, which is (c) Purania and Patna
considered to be the first step towards a global agreement (d) Darbhanga and Purania
to reduce the emission of green house gases and control of (e) None of the above/more than one of the
biodiversity. Kyoto protocol was held in 1997. above
65th BPSC (Pre) (PH) Re-Exam. 2019 277 YCT
Ans. (a) : Old alluvial or Bangar soil is found in the (GSDP) estimate at constant price (base year
Patna and Gaya region of Bihar. Two different types of 2011-12) ?
alluvial soils have developed in the upper and middle (a) `31,316
Gangetic plain i.e. - Bangar and Khadar. Bangar is (b) `30,000
known as old alluvial soil, and Khadar is the new (c) `21,316
alluvial soil. (d) `20,000
92. Barauni refinery in Bihar was established with (e) None of the above/more than one of the
whose co-operation? above
(a) Germany Ans. (a) : In Economic survey 2018-19, the per capita
(b) South Korea Gross State Domestic Product (GSDP) of Bihar in
(c) Soviet Union 2017-18 at current prices was Rs. 42,242 and the per
(d) France capita income (GSDP) at the constant price (2011-12)
(e) None of the above/more than one of the was estimated `31,316.
above 97. Which district of Bihar at constant prices is one
Ans. (c) : Barauni Refinery is located in the Begusarai of the three most prosperous districts (in terms
district of Bihar. It was built in 1964 as a dream project of per capita income) in 2018-19 (base year
of first Chief Minister of Bihar, Shri Krishna Sinha, in 2011-12)?
collaboration with the Soviet Union. It was (a) Madhepura
(b) Supaul
Commissioned with a refining capacity of 1 million (c) Begusarai
tonnes per year (d) Sheohar
93. The famous cattle fair (Pashu Mela) of Bihar is (e) None of the above/more than one of the
held on Kartik Purnima at ? above
(a) Gaya Ans. (c) : The growth rate of Bihar in 2019-20 fiscal
(b) Madhubani year has been 10.5%. According to the economic
(c) Bhagalpur survey, three most prosperous districts (in terms of per
(d) Sonepur capita income in 2018-19) are Patna, Munger and
(e) None of the above/more than one of the Begusarai (at base year-2011-12). As per 2019-20, the
above per capita income in Begusarai is Rs. 45540.
Ans. (d) : Sonpur Mela, a famous cattle fair (Pashu 98. What is the approximate percentage increase in
Mela) is organized at Sonpur, Bihar on Kartik Purnima, revenue surplus in Bihar in 2018-19 as
in the month of November which is situated on the compared to 2017-18?
confluence of the holy rivers Ganga and Gandak. It is (a) 40%
one of the Asia's largest cattle fair. (b) 50%
94. The traditional water conservation of irrigation (c) 54%
system popular in South Bihar is called? (d) 44%
(a) Ahar-Pyne (e) None of the above/more than of the above
(b) Iron-Pyne Ans. (d) : In Economic Survey of Bihar 2018-19, the
(c) Cast-Pyne revenue surplus has increased by 44% (approximately)
(d) Gahan-Pyne in 2018-19, as compared to fiscal year 2017-18.
(e) None of the above/more than one of the 99. The total production (In lakh tonnes) of coarse
above cereals in Bihar in 2017-18 and its growth rate
Ans. (a) : A traditional water harvesting irrigation between 2013-14 and 2017-18 were,
method prevalent in South Bihar is known as Ahar- respectively?
Pyne. It is an indigenous irrigation technology which (a) 2.15%, 5%
continue from the time of the Magadh dynasty to (b) 4%, 6%
irrigate substantial areas even today in South Bihar. (c) 3.15%, 6%
95. In which district of Bihar is Vikramshila (d) 5.15%, 4%
Gangetic Dolphin Sanctuary located? (e) None of the above/more than one of the
(a) Bhagalpur above
(b) Patna Ans. (c) : As per Economic survey of Bihar in 2017-18,
(c) Buxar the total production (In lakh tonnes) of coarse cereals
(d) Nalanda and its growth rate between 2013-14 and 2017-18 were
(e) None of the above/more than one of the 3.15% and 6% respectively.
above 100. 'Jeevika' has achieved unprecedented success
Ans. (a) : Vikramshila Gangetic Dolphin Sanctuary, in promoting social development in Bihar. The
estimated number of self-help groups formed
notified in 1991, is the only Dolphin sanctuary in the till September 2018, was?
country. It is located in the Bhagalpur district of Bihar. (a) 7.17 lakh
The Gangetic dolphin has been declared the National (b) 9.01 lakh
Aquatic Animal of India in 2009. (c) 9.17 lakh
96. The Per Capita Gross State Domestic Product (d) 8.17 lakh
(GSDP) of Bihar in 2017-18 at current prices (e) None of the above/more than one of the
was Rs. 42,242. What is the per capita Income above
65th BPSC (Pre) (PH) Re-Exam. 2019 278 YCT
Ans. (d) : Bihar Rural Livelihoods Promotion Society 104. The average age of a father and a mother in a
(BRLPS), scaled up the "Jeevika" model in all 534 family is 36 years. The average age of father,
Blocks of 38 districts in Bihar. The model has achieved mother and their only daughter is 28 years.
unprecedented success in promoting social development How old is the daughter?
in Bihar. Around 8.17 lakh Self-Help Groups (SHGS) (a) 11 years
have been formed till September 2018. (b) 12 years
(c) 13 years
101. Patna Youth Association was established in? (d) 14 years
(a) 1917 (e) None of the above/more than one of the
(b) 1925 above
(c) 1927 Ans. (b) : Total age of father and mother = 36 × 2
(d) 1930 = 72 years
(e) None of the above/more than one of the Total age of mother and daughter and father = 28 × 3
above = 84 years
Ans. (c) : Patna Youth Association (Patna Yuvak Then the age of daughter = 84 – 72
Sangh) was established in 1927 by revolutionary leader = 12 years
Shri Manindra Narayan Rai. 105. If the product of three consecutive number is
102. A number if divided by 114 leaves 21 120, then the sum of those numbers is?
remainder. If the same number is divided by (a) 12
19, the remainder is? (b) 13
(a) 3 (c) 16
(b) 4 (d) 15
(c) 2 (e) None of the above/more than one of the
(d) 1 above
(e) None of the above/more than one of the Ans. (d) : Let the three consecutive no. is –
above (x – 1), x, (x + 1)
Ans. (c) : Assume the number is x. Then – x × (x – 1) × (x + 1) ;
As. Per formula Dividend = Divisor × Quotient + ⇒ (x – 1) (x + 1) = (x2 – 12)
Remainder ⇒ x × (x2 – 1) = 120
Then x = 114 × 1 + 21 ⇒ x3 – x = 120, assume that [x = 5]
x = 114 + 21 = 135 Then (5)3 – 5 = 120 ⇒ 125 – 5 = 120
x = 5 then the numbers are x – 1 = 4, x = 5, x + 1 = 6
135 2 Sum = 4 + 5 + 6 = 15
Now = 7 , then the Remainder = 2
19 19 3
9n × 35 ( 27 )
old remainder 106. If = 243 , then the value of n is?
Method 2 : Now Remainder = 4
Divisor 3 ( 81)
21 (a) 3
= =2 (b) 4
19 (c) 5
103. If 1/8th part of a pencil is black, remaining 1/2 (d) –4
1 (e) None of the above/more than one of the
part is white and the remaining 3 cm is blue, above
2 Ans. (b) : In the given equation –
then find the total length of the pencil?
(a) 8 cm 9n × 35 (27)3 (32 ) n × 35 × (33 )3
= 243 ; = 243
(b) 6 cm 3(81) 4 3 × (34 ) 4
(c) 10 cm 32n × 35 × 39
(d) 7 cm = = 35 then, 32n = 35 × 33
(e) None of the above/more than one of the 317
above ⇒ 2n = 8 ; n = 4
107. If the ratio of the cost price and selling price of
Ans. (a) : Let the total length of Pencil = x cm an article is 10 : 11 , then the percentage of
x profit will be?
From the given data, is black (a) 7
8
(b) 14
x 7x (c) 9
Remaining white = x – =
8 8 (d) 13
(e) None of the above/more than one of the
7x 1 7x above
= × =
8 2 16 Ans. (e) : The ratio of cost price and selling price is
7x 1 7 10x:11x
Remaining blue = = 3 or The profit of an article = 1 x
16 2 2
Total length of pencil (x) 1x
Profit percentage = × 100 = 10%
x = 8cm 10x

65th BPSC (Pre) (PH) Re-Exam. 2019 279 YCT


108. Some people get a work completed in 12 days. Ans. (d) : From the given question-
By this, half of the work will be done twice by a a
these people? x + y = 2a, ∴ +
(a) In 3 days x −a y−a
(b) In 2 days
a(y − a) + a(x − a) a(x + y) − 2a 2
(c) In 4 days ⇒ =
(d) In 5 days (x − a) (y − a) (x − a) (y − a)
(e) None of the above/more than one of the
a ⋅ 2a − 2a 2
above ⇒ [because x + y = 2a]
Ans. (a) : As per the question – (x − a) (y − a)
M1 = X, D1 = 12 Days, W1 = 1 2 a2 − 2 a2
1 =
M2 = 2X, D2 = ? Day, W2 = (x − a) (y − a)
2
0
M1D1 M 2 D 2 Χ × 12 2X × D 2 ⇒ =0
Then – = ⇒ = (x − a) (y − a)
W1 W2 1 1/ 2
⇒ 12X = 4X × D2 112. Name of the part of a Lander which was
12x recently launched by Indian spacecraft
D2 = = 3, D2 = 3 Chandrayann 2 ?
4x
(a) Vikram
109. A train runs from Chandigarh at 4:00 pm and
reaches Panipat at 7:15 pm. The average speed (b) Pragyan
of the train is 40 km/hr. What is the distance (c) Rover
between Chandigarh and Panipat? (d) Satish
(a) 125 km (e) None of the above/more than one of the
(b) 110 km above
(c) 130 km Ans. (a) : Chandrayaan 2 is an Indian Lunar Mission
(d) 145 km that will explore the Moon's south polar region. It was
(e) None of the above/more than one of the
launched on 22 July 2019. The launch vehicle carried an
above
orbiter, a rover named Pragyan and a lander named
Ans. (c) : Time taken by train = 4 pm to 7:15pm.
Speed of the train = 40 km/hr. Vikram.
Distance 113. Citizenship of Julian Assange is?
∵ Speed = ⇒ Distance = speed × time (a) British
Time (b) American
⇒ Distance = 40 × 3.25
(c) French
= 130km (d) Australian
5.32 × 56 + 5.32 × 44 (e) None of the above/more than one of the
110. If X = , then the value of X
above
( 7.66 )2 – ( 2.34 )2
is? Ans. (d) : Julian Paul Assange is an Australian activist
(a) 7.66 and publishers. He founded wiki Leaks in 2006.
(b) 10 114. The number of members in the National
(c) 5.32 Human Rights Commission increased through
(d) 9.5 the Protection of Human Rights (Amendment)
(e) None of the above/more than one of the act, 2019 ?
above (a) One
Ans. (b) : As given, (b) Two
5.32 × 56 + 5.32 × 44 (c) Three
X= 2 2 (d) Four
( 7.66 ) − ( 2.34 ) (e) None of the above/more than one of the year
5.32 ( 56 + 44 ) 5.32 × 100 Ans. (c) : The Protection of Human Rights
Now = (Amendment) Bill, 2019 was introduced in Lok Sabha
( 7.66 + 2.34 )( 7.66 − 2.34 ) 10 × 5.32
on 8th July 2019. The Amendment Bill proposed to
Hence, X = 10 allow three persons to be appointed, as one who has
a a been Chief Justice of Supreme Court, or a Judge of the
111. If x + y = 2a, then the value of + is ?
x–a y –a Supreme Court will be the chairperson of the Nation
(a) 3 (b) –1 Human Rights Commission (NHRC) and two persons
(c) –2 (d) 0 having knowledge of human right, will be appointed as
(e) None of the above/more than one of the members of the NHRC. One of the three appointed
above persons, will be a woman.

65th BPSC (Pre) (PH) Re-Exam. 2019 280 YCT


115. Recently, G-20 summit was held in ? (c) Gujarat
(a) Osaka (d) Rajasthan
(b) Paris (e) None of the above/more than one of the
(c) New Delhi above
(d) New York Ans. (d) : The Rajasthan Assembly passed bill against
(e) None of the above/more than one of the mob lynching and Honour killing in August 2019. Now
above Rajasthan has become the 2nd state after Manipur which
passed a law to curb lynching cases.
Ans. (a) : The 14th G-20 Summit was held in Osaka,
Japan from 28th to 29th June 2019. 17th G-20 Summit 121. The Name of India's first Private Train ?
will be held in Bali, Indonesia on 15th-16th November (a) Marudhar
2022. (b) Bhuj
(c) Rajdhani
116. 'VARUNA' Naval exercise' was conducted ? (d) Shatabdi
(a) Between India and China (e) None of the above/more than one of the
(b) Between Russia and India above
(c) Between India and USA
(d) Between Pakistan and China Ans. (e) : Under the Bharat Gaurav Scheme, India's first
(e) None of the above/more than one of the private train 'Tejas Express' is being run by its
above subsidiary IRCTC, on Lucknow-Delhi route.
Ans. (e) : The Indian and French Navies have been 122. The winner of T-20 physical Disability world
conducting Bilateral Martime Exercise since 1993. Cricket Series, 2019 Trophy is ?
Since 2001 this exercise have been called VARUNA'. (a) England
VARUNA - 2021 had been conducted in the Arabian (b) Nepal
sea. 'VARUNA - 2022' was also conducted in the (c) India
Arabian Sea from 30 March to 3 April 2022. (d) Bangladesh
(e) None of the above/more than one of the
117. Indian Air Force has received the first batch of above
four AH-64E Apache Helicopter from ?
(a) Israel Ans. (c) : T-20 Physical Disability World Cricket Series
(b) France 2019, was held in England. India defeated England in
(c) USA final and won the series.
(d) Britain 123. The Newly appointed Cabinet Secretary of
(e) None of the above/more than one of the India is ?
above (a) Rajiv Gauba
Ans. (c) : India contracted 22 Apache Helicopters from (b) Rajiv Kumar
Boeing through the Foreign Military Sales Programme (c) P.K. Goyal
of the USA government in September 2015. (d) Rajiv Maha
118. The first Lokpal of India is ? (e) None of the above/more than one of the
(a) P. C. Ghose above
(b) U. U. Lalit
(c) P. S. Banerjee Ans. (a) : Rajiv Gauba is current Cabinet Secretary of
(d) Rajiv Kumar India since 2019. He is a 1982 batch (Jharkhand cadre)
(e) None of the above/more than one of the IAS officer.
above 124. From which constituency of Bihar R.K. Singh
Ans. (a) : Retired Supreme Court Judge, Justice Pinaki won the Lok Sabha seat in the year 2019?
Chandra Ghose was appointed the first Lokpal of India (a) Begusarai
in 2019. Justice Pradip Kumar Mohanty is current (b) Buxar
Lokpal of India from May 2022. (c) Arrah
119. Recently passed 'Tripal Talak Bill' 2019 (d) Patna Sahib
provides for maximum jail term? (e) None of the above/more than one of the
(a) four years above
(b) Three years Ans. (c) : Raj Kumar Singh (R.K. Singh), won the 10th
(c) Two years
(d) Five years and 17th Lok Sabha election seat from Arrah
(e) None of the above/more than one of the Constituency, Bihar. He is 1975th batch (Bihar cadre)
above IAS officer and current union cabinet minister.
Ans. (b) : Any Muslim husband who communicates the 125. Which country will host the G-20 summit in the
‘Talaq orally’ or in writing may face punishment upto year-2022?
three years in jail. The punishment may be also (a) Nepal
extended. (b) Russia
120. Which state has recently passed an Act against (c) Britain
'mob lynching and Honour killing ? (d) Japan
(a) Haryana (e) None of the above/more than one of the
(b) Bihar above
65th BPSC (Pre) (PH) Re-Exam. 2019 281 YCT
Ans. (e) : The 17th G-20 Summit 2022 will be organized Ans. (e) : The Umbrella Movement was started as a
in Bali, Indonesia from 15 to 16 November 2022. Moto spontaneous social movement for democratic
of the summit is "Recover Together, Recover stronger". development in Hong Kong in 2014. In this movement
The 18th G-20 Summit will be held in India. the umbrellas were used as a tool for defence against
126. What is the name of the IIT Bombay student police pepper spray so it is called as Umbrella
who recently took charge in his new job as Movement.
trackman in Dhanbad ? 131. Which Indian has been selected for the Raman
(a) Rajiv Kumar Magsaysay Award, 2019?
(b) Ravi Shankar (a) Rajendra Singh
(c) Shashi Kumar (b) Ravish Kumar
(d) Shravan Kumar (c) Ram Sharan Aggarwal
(e) None of the above/more than one of the (d) Parvesh Kumar
above (e) None of the above/more than one of the
above
Ans. (d) : Shravan Kumar is working as a trackman in
Ans. (b) : The winner of Ramon Magsaysay Award
the Dhanbad railway station. He holds B.Tech and
2019, is the Indian journalist Ravish Kumar. He is a
M.Tech degree from IIT Bombay. senior executive editor of NDTV Indian news channel.
127. The Padamshri Award was given to Bhagirathi Ramon Magsaysay award is also called as Asia's Nobel
Devi in which field in January 2019? Prize.
(a) Sports 132. The "Meghdoot App" launched by the
(b) Acting Government of India is related to?
(c) Public Affairs (a) Agriculture
(d) Agriculture (b) Medical Science
(e) None of the above/more than one of the (c) Education
above (d) Pension
Ans. (c) : Bhagirathi Devi got Padma Shree 2019, in (e) None of the above/more than one of the
field of Public Affairs. above
128. Who won the best director's award at the 66th Ans. (a) : Ministries of Earth Science and Agriculture
National Film Awards? launched 'Meghdoot App'. It will provide weather based
(a) Aditya Singh agro advisories to farmers in local language. It provides
forecast information to farmers relating to temperature,
(b) Aditya Dhar rainfall, humidity which play crucial roles in
(c) Karan Johar agriculture.
(d) Anil Kapoor 133. To make India a $5 trillion economy by 2024-
(e) None of the above/more than one of the 25. What should be the constant rate of growth
above of GDP?
Ans. (b) : In the 66th National Film Award, Aditya Dhar (a) 8%
won the best director award for 'Uri ; The Surgical (b) 7%
(c) 6%
Strike' film. (d) 9%
129. From which school Mr. Pappu Harijan belong (e) None of the above/more than one of the
to a teacher selected from Bihar for National above
Teacher Award-2018 ? Ans. (a) : In 2019, Indian Prime Minister Narendra
(a) Kendriya vidyalaya Modi envisioned making India as a $5 Trillion
(b) Michal School Economy by 2024-25, which requires 8% GDP growth
rate. After this India would become the third largest
(c) Delhi public School economy in the world.
(d) Kulhariya School
134. What is the goal of "Pradhan Mantri Karam
(e) None of the above/more than one of the Yogi Maan-Dhan Yojana", announced in
above Budget?
Ans. (d) : Shri Pappu Harijan, the teacher from Bihar (a) To give education allowance for single girl
was selected for the National Teacher Award 2018. He child.
belongs to Kulhariya school (Banka district, Bihar). He (b) To give allowance for unemployed Youth.
(c) To give Pension benefits to retail traders and
was honored by the President on 5 September small shopkeepers.
(Teachers' day). (d) Disability allowance
130. The main demand for umbrella protesters in (e) None of the above/more than one of the
Hong Kong was? above
(a) Reservation Ans. (c) : Prime Minister Narendra Modi Launched
(b) Freedom Pradhan Mantri Karam Yogi Maandhan Yojana (PM-
(c) Fundamental Rights KYM) in 2019. This scheme provides assured monthly
pension to retail traders and small shopkeepers. It
(d) Economic growth provides Rs. 3000 per month to traders and shopkeepers
(e) None of the above/more than one of the upon reaching the age of 60 as announced in Union
above Budget 2019-20.
65th BPSC (Pre) (PH) Re-Exam. 2019 282 YCT
135. 'Dwarf enterprises' (establishments with less (b) 9.3%
than 100 personnel's) have a percentage (c) 9.8%
contribution to employment and productivity
(d) 8.4%
in India in 2018-2019, respectively of?
(a) 14%; 8% (e) None of the above/more than one of the
(b) 15%; 5% above
(c) 12%; 4% Ans. (b) : According to the economic survey report
(d) 10%; 10%
2018-19, the growth in fixed investment picked up from
(e) None of the above/more than one of the
above 8.3% in 2016-17 to 9.3% in 2017-18 and further to 10%
Ans. (a) : According to the economic survey 2018-19, in 2018-19. Further net FDI inflows grew up by 14.2%
the Dwarf enterprises (firms with less than 100 workers) in 2018-19.
account for more than half of all organized firm in 140. According to Gems and Jewellery Export
manufacturing by number, their contribution to
employment is only 14% and 8% in productivity. Promotion Council of India, how much export
136. In fiscal year 2018-19, the percentage of rural development rate has decreased in April-July
households in India that have been found to 2019 compared to be same time in 2018?
have access to toilets and practice it? (a) 7%
(a) 80.5% (b) 5%
(b) 96.5%
(c) 8.5%
(c) 100%
(d) 99% (d) 6%
(e) None of the above/more than one of the (e) None of the above/more than one of the
above above
Ans. (b) : According to National Annual Rural Ans. (c) : As per Gems and Jewellery Export promotion
Sanitation Survey (NARSS), in the financial Year 2018-
19, 96.5% of rural house holds in India have found council of India report 2019, the export development
access to toilets and use them. Government of India rate has decreased around 8.5% in April-July 2019 as
Launched Swachh Bharat Abhiyan in 2014, aiming to compared to the same time period in 2018.
completely eliminate open defecation in country by 2nd, 141. In August 2019, the ministry of Finance,
October 2019
Government of India announced how much
137. The Share of renewable energy in total power
generation (above 25 Mw except hydro power amount (in crores) to be invested to improve
source) in India has increased from 5% in the liquidity and lending system of public
2014-15 to what percentage in 2018-2019? sector banks?
(a) 12% (a) `30,000
(b) 15%
(b) `45,000
(c) 10%
(d) 7% (c) `70,000
(e) None of the above/more than one of the (d) `80,000
above (e) None of the above/more than one of the
Ans. (c) : The share of renewable energy in total power above
generation (above 25 Mw, except hydro power source)
in India has increased from 5% in 2014-15 to 10% in Ans. (c) : The Ministry of Finance, Government of
2018-19 India, announced 70,000 crore amount to be invested to
138. In India in 2017-18 the growth rate of GDP has improve the liquidity and lending system of public
increased to 7.2% vis-a-vis what percentage in sector banks in August, 2019.
2018-2019? 142. To ensure, 'Har Ghar Jal' for all rural families,
(a) 5.2%
(b) 6.8% the target upto which year has been set by 'Jal
(c) 7.0% Jeevan Mission'?
(d) 6.2% (a) 2022
(e) None of the above/more than one of the (b) 2025
above
(c) 2026
Ans. (b) : In fiscal year of 2018-2019 the growth rate of
GDP was 6.8%. It was increased (resolved to) 7.2% in (d) 2024
financial year 2017-18. Under the first revision released (e) None of the above/more than one of the
in 2019, real GDP or GDP at constant (at base year above
2011-12) prices for 2017-18 was at 131.80 Lakh Crore. Ans. (d) : Under Jal Jeevan Mission (JJM), the target to
139. In India by what percentage has the growth ensure assured tap water supply or 'Har Ghar Jal' to all
rate of fixed or fixed investment in 2017-18 rural households is by 2024. It comes under Jal Shakti
increased to 10% in 2018-2019 ?
(a) 8% Ministry.

65th BPSC (Pre) (PH) Re-Exam. 2019 283 YCT


143. Deepa Malik is related to which sports? 147. How many billion dollars will India give to
(a) Hockey Russia for the development of far east?
(b) Cricket (a) Four billion
(c) Kabbaddi (b) One billion
(d) Badminton (c) Five billion
(e) None of the above/more than one of the (d) Two billion
above (e) None of the above/more than one of the
Ans. (e) : Deepa Malik is an Indian Para athlete. She is above
the first Indian woman who won a silver medal in Ans. (b) : Prime Minister Narendra Modi launched 'Act
Paralympics 2016 in the shot put. She is also a player of Far East' policy during his two-day visit to Russia and
Javelin throw. She was honored with Arjuna award in boosted India's engagement with Russia's Far East
2012, Padma Shri in 2017. Region. He announced 1 billion dollar credit for the
144. Currently Shaksgam Valley is ? development of Russia's Far East Region.
(a) Under the territory of Pakistan 148. Romila Thapar is a?
(b) Under the territory of Japan (a) Doctor
(c) Under the territory of India (b) Engineer
(d) Under the territory of China (c) Historian
(e) None of the above/more than one of the (d) Economist
above (e) None of the above/more than one of the
Ans. (d) : The Shaksgam valley is also known as Trans- above
Karakorum valley. The Shaksgam valley is located in Ans. (c) : Romila Thapar is an Eminent Historian. She
the northern most part of Jammu and Kashmir, India, attained a doctorate in Indian history from School of
which is expanded on both sides of the Shaksgam river. Oriental and African Studies, University of London in
It was part of India but Pakistan fraudulently took it 1958.
under its control. The Shaksgam valley was ceded to 149. The state of Manipur was established?
China by Pakistan in 1963. Now it is part of China's (a) In 1972
territory. (b) In 1987
145. Which country does Aramco Oil Company (c) In 1985
belong to? (d) In 1963
(a) Iran (e) None of the above/more than one of the
(b) Oman above
(c) Saudi Arabia Ans. (a) : Manipur is one of the seven sister states of
(d) USA Northeast India. In October, 1949 Manipur became the
(e) None of the above/more than one of the part of India. First it became a union territory in 1956
above and then became a full fledged state in 1972 under
Ans. (c) : Aramco Oil Company is a Saudi Arabian oil North- east Area (Re-organisation) Act 1971. Imphal is
company namely, Saudi Aramco. As of 2019-20 report, the capital of Manipur.
it is one of the largest company in the world by revenue. 150. A state of India has a following characteristics :
It was founded in May 1933. I. It's Northern part is arid and semi-arid.
II. Cotton is cultivated in it's central part
146. From which year did the 'Arjun Award begin?
III. Cash crops are preferred over food crops
(a) 1989
Choose the exact state
(b) 1997 (a) Andhra Pradesh
(c) 1961 (b) Gujarat
(d) 1899 (c) Rajasthan
(e) None of the above/more than one of the (d) Karnataka
above (e) None of the above/more than one of the
Ans. (c) : Arjuna Award is India's highest sports award above
bestowed by the Government of India to the Ans. (b) : State of Gujarat has the following
sportspersons for their outstanding performance in characteristics :
national and international games. It was launched in
1. It's Northern part is arid and semi-arid.
1961. The sportspersons are honored with a bronze
2. In it's central part cotton is cultivated in about 15.19
statue of Arjuna, a scroll and cash. Krishna Das was the
first sportsperson who was awarded Arjun award in lakh hectares.
Archery (1961). 3. Cash crops are preferred over food crops.

65th BPSC (Pre) (PH) Re-Exam. 2019 284 YCT


66th Bihar Public Service Commission
Preliminary Examination, 2020
GENERAL KNOWLEDGE & GENERAL SCIENCE
(Solved Paper with Detail Explanation)
1. When was the Citizenship (Amendment) Act Ans. (b) : Dutch East India Company was founded in
passed? 1602 in Dutch Republic (Presently Netherlands) to trade
(a) 11th December, 2018 in East Indies including India. The Dutch company
(b) 11th December, 2019 established its first Indian factory in 1605 at
(c) 11th October, 2019 Masulipatnam (Machilipatnam) and a second at Surat in
(d) 11th October, 2020 1616. The company had also established their factory at
Patna in 1632.
(e) None of the above/More than one of the
above 4. Which one of the following is not the
characteristic of decentralization?
Ans. (b) : The Citizenship Amendment Act 2019 was (a) Autonomy
passed by Lok Sabha on 10th December 2019. The (b) People's participation
Rajya Sabha also passed the Act on 11th December (c) To instill non-confidence among local
2019. The Act amends the Citizenship Act, 1955 so as communities
to grant a certain class of illegal migrants a path to (d) To empower local communities
Indian citizenship. The Citizenship Amendment Act, (e) None of the above/More than one of the
2019 seeks to make illegal migrants who are Hindus, above
Sikhs, Buddhist, Jains Parsis and Christians from Ans. (c) : Decentralization is defined as the practice of
Afghanistan, Bangladesh, Pakistan eligible for Indian taking power away from the union and state government
Citizenship. and giving it to local governments. Instill non-
2. The legislative origins of the 73rd confidence among local communities is not a
Constitutional Amendment Act can be traced characteristic of decentralization.
back to which Constitutional Amendment Bill? 5. Which Act is not the landmark in the
(a) 61st Constitutional Amendment Bill development of the Constitution during the
(b) 62nd Constitutional Amendment Bill British Rule?
(c) 63rd Constitutional Amendment Bill (a) The Regulating Act, 1773
(d) 64 Constitutional Amendment Bill (b) The Charter Act, 1833
(e) None of the above/More than one of the (c) The Government of India Act, 1919
above (d) The Protection of Civil Rights Act, 1955
Ans. (d) : The Legislative origins of the 73rd (e) None of the above/More than one of the
above
Constitutional Amendment Act can be traced back to
the 64th Constitutional Amendment Bill. Rajiv Gandhi Ans. (d) : Regulating Act 1773, The Charter Act 1833,
had introduced 64th Constitutional Amendment Bill and The Government of India Act, 1919 are the
landmark in the development of the constitution during
1989, to provide constitutional status to Panchayati Raj
the British Rule whereas "The Protection of Civil
institutions but this bill was not passed by Rajya Sabha.
Rights Act, 1955 was enforced on 01-06-1955 (after
Then through 73rd Amendment Act, 1992, Panchayati
independence). The Act prescribe punishment for the
Raj was constitutionalised. The 73rd Amendment Act,
preaching and practice of Untouchability for the
1992 has added a new Part IX and 11th Schedule to the enforcement of any disability arising therefrom and for
Indian Constitution. matters connected therewith.
3. Which company established its factory in 1632 6. Who won the bid to construct the new
at Patna, Bihar? Parliament building recently?
(a) British East India Company (a) L&T Limited
(b) Dutch East India Company (b) Reliance Projects Limited
(c) Portuguese East India Company (c) Tata Projects Limited
(d) French East India Company (d) The National Highways Authority of India
(e) None of the above/More than one of the (e) None of the above/More than one of the
above above

66th BPSC (Pre) Exam. 2020 285 YCT


Ans. (c) : Tata Projects won the contract to build the Ans. (d) : The 73rd Amendment Act of 1992 has added
new Parliament building as a part of the government's a new Part-IX to the Indian Constitution. This part is
Central Vista redevelopment plans. As per the project entitled as " The Panchayats' and consist of provision
schedule, the new Parliament building should be from Article 243 to 243-O; The Act also added a new
completed by November 2022. 11th Schedule to the Constitution. This schedule
7. Which Article in the Indian Constitution is contains 29 functional items of the Panchayats. It deals
related with the establishment of the Election with Article 243-G.
Commission of India? 11. Which of the following commercial banks of
(a) Article 324 India comes in top 100 global banks?
(b) Article 148 (a) ICICI Bank
(c) Article 342 (b) SBI
(d) Article 325 (c) HDFC Bank
(e) None of the above/More than one of the (d) Kotak Mahindra Bank
above (e) None of the above/More than one of the
Ans. (a) : Article 324 in the Indian Constitution is above
related to the establishment of the Election Commission Ans. (b) : State Bank of India (at the 55th position) is
of India. The Election Commission is a permanent and the only bank of India in the global top 100 List.
independent body established by the Indian Constitution 12. The Government of India announced a new
to ensure free and fair elections in the country. It is scheme 'NIRVIK' in the Budget for 2020-21.
responsible for conducting free & fair elections of Which of the following sectors of economy will
Parliament , State legislature, the office of President and take the benefit from this scheme?
Vice President of India. (a) Agriculture sector
8. Who is the Head of the National Commission (b) Industrial sector
for Women at present? (c) Health sector
(a) Mamta Sharma (d) Export sector
(b) Lalitha Kumaramangalam (e) None of the above/More than one of the
(c) Rekha Sharma above
(d) Smriti Irani Ans. (d) : The Government of India announced a new
(e) None of the above/More than one of the scheme 'NIRVIK' (Niryat Rin Vikas Yojana) in the
above Union Budget for 2020-21. The NIRVIK Scheme will
Ans. (c) : The National Commission for Women was boost the export segment of Indian Economy. NIRVIK
formed to establish an equal and just livelihood for is also known as Niryat Rin Vikas Yojana. This Scheme
woman by making legal & constitutional amendments aimed at providing high insurance coverage for
for women in India. It was setup in 1992 under the exporters while reducing the premiums on small scale
National Commission for Women Act, 1990. Mrs. exporters.
Rekha Sharma is the current chairperson of this 13. Bihar Government has introduced the
commission and he held the additional charge as Comprehensive Financial Management System
Chairperson, NCW with effect from September 29 , th (CFMS) on 1st April, 2019. This system
2017 before becoming its Chairperson on 7th August (a) will make all financial activities in the State
2018. online and paperless
(b) will solve the problem of NPA of banks
9. What is meant by 'Rule of Law'?
(c) will ensure effective implementation of State
(a) One act for all and one judiciary for all
projects
(b) One act for all and one State for all
(d) will manage State Government finances
(c) One State for all and one judiciary for all
including local bodies
(d) All act for one and one judiciary for all
(e) None of the above/More than one of the
(e) None of the above/More than one of the above
above
Ans. (e) : Bihar Government had introduced the
Ans. (a) : Rule of law is a legal principle. It means that Comprehensive Financial Management System (CFMS)
all laws apply equally to all citizens of the country and on 1st April 2019. This system will make financial
no one can be above the law. It also states that no activities in the state online and paperless and will
person shall be subject to harsh, uncivilized or ensure effective implementation of state project.
discriminatory treatment even for the sake of
maintaining law and order. 14. The Government of India conferred the Krishi
Karman Award to Bihar State on 2nd January,
10. Which Article gives the list of 29 functions to 2020. This award is given for
be performed by the Panchayati Raj (a) production and productivity of maize and
Institutions? wheat
(a) Article 243(H) (b) for grain production
(b) Article 243(E) (c) production of rice
(c) Article 243(F)
(d) production of oilseeds
(d) Article 243(G)
(e) None of the above/More than one of the (e) None of the above/More than one of the
above above
66th BPSC (Pre) Exam. 2020 286 YCT
Ans. (a) : On 2nd January 2020, the Government of Ans. (d) : The Food Corporation of India FCI) was
India conferred the Krishi Karman Award to Bihar (setup in 1965. It plays a significant roll in India's
Government for excellent production & productivity of success in transforming the crisis management oriented
maize & wheat. food security into stable security stem. The objective of
15. Bihar Government launched a new scheme food management in India are :- Procurement of food
'Satat Jivikoparjan Yojana' in August 2018. grains from farmers at remunerable prices, distribution
The objective of this scheme is of these food grains, Maintenance of a Buffer stock,
(a) to provide unemployment allowance to youth Maintenance of food safety & Price stability. Export of
(b) to provide employment in rural areas through food grains is not an objective of food management in
local bodies India.
(c) to provide sustainable income generating 19. Revenue deficit in India implies that
assets to extremely poor households (a) the Indian Government needs to borrow in
(d) to provide free training for skill upgradation order to finance its expenses which will create
of youth capital assets
(e) None of the above/More than one of the (b) the Indian Government needs to borrow in
above order to finance its expenses which do not
create capital assets
Ans. (e) : In August 2018, Bihar government had (c) the Indian Government needs to borrow from
launched 'Satat Jivikoparjan Yojana'. The objective of the Reserve Bank of India against
this programme is to empower ultra poor households government securities
traditionally engaged in production, transportation & (d) the Indian Government needs to borrow from
selling of country liquor/Foddy, ultra poor from SC/ST international financial institutions
and other communities through diversification of (e) None of the above/More than one of the
livehood, capacity building and improved access to above
finance. Ans. (b) : A revenue deficit measures the difference
16. Which of the following is not included in Seven between the projected amount of income and the actual
Resolves (Saat Nishchay) of Bihar amount of income. It arises when government revenue
Government? expenditure exceeds the total revenue receipts. It
(a) Women employment implies the Government's need to borrow in order to
(b) Clean drinking water finance it expenses which do not create capital assets.
(c) Supply of electricity to all households 20. Which of the following countries is India's top
(d) Child welfare trading partner in 2019-20?
(e) None of the above/More than one of the (a) USA
above (b) China
Ans. (d) : Bihar Government had landed Seven Resolve (c) UAE
(Saat Nishchay ) in 2015. The seven resolve are:-(1) (d) Saudi Arabia
Aarthik Hal, Yuvaon Ko Bal (ii) Aarakshit Rozgar, (e) None of the above/More than one of the
Mahilaon ko Adhikar (iii) Har Ghar Bijli, Lagatar, (iv) above
Har Ghar Nal ka Jal (v) Ghar Tak Pakki Gali-Naaliyan Ans. (a) : USA was the India's top trading partner in
(vi) Shauchalay Nirman, Ghar ka Samman.(vii) Avsar 2019-20 followed by China, UAE. In 2019-20 the
Badhe, Aage Padhein. Child Welfare is not included in bilateral trade between USA & India was 88.75 billion
Saat Nishchay USD.
17. The per capita Net State Domestic Product at 21. Which of the following infrastructure sectors of
constant prices for Bihar is less than the India is related with Bharatmala Project?
country. In the year 2018-19, it was (a) Telecom sector
(a) 75 percent of the national average (b) Railways
(b) 60 percent of the national average (c) Road infrastructure
(c) 50 percent of the national average (d) Port sector
(d) 33 percent of the national average (e) None of the above/More than one of the
(e) None of the above/More than one of the above
above Ans. (c) : The Bhartmala Project was launched on 25
Ans. (d) : The Per Capita Net State Development October 2017 for the construction of better road
Product at constant Prices for Bihar is less than the transport system in India. It is an umbrella project under
country. In the year 2018-19 it was 33% of the national the Ministry of Road Transport and Highways.
average. 22. Which of the following agencies releases the
18. Which one among the following is not an index of industrial production to measure
objective of food management in India? industrial performance in India?
(a) Distribution of food grains (a) The National Sample Survey Office (NSSO)
(b) Procurement of food grains (b) The Reserve Bank of India (RBI)
(c) Maintenance of food grains buffer stock (c) The Central Statistics Office (CSO)
(d) The Indian Statistical Institute (ISI)
(d) Export of food grains
(e) None of the above/More than one of the (e) None of the above/More than one of the
above above
66th BPSC (Pre) Exam. 2020 287 YCT
Ans. (c) : Index of Industrial Production is compiled & Ans. (b) : Chittaranjan Das was popularly called as
released monthly by the Central Statistical Office, Deshbandhu. He along with Motilal Nehru formed
Ministry of Statistic and Programme Implementation. It Swaraj Party in 1923. The second individual satyagrahi
is an indicator to measure the change in volume of was Jawaharlal Nehru. Ras Bihari Ghosh has presided
production of industrial products during a given period. the Surat session of INC in 1907.
23. According to the Ease of Doing business India House was founded by Shyamji Krishna Verma in
Report, 2020, India improved its rank from 1905 in London UK.
(a) 77 in previous year to 63rd position 27. Which day was declared as 'Purna Swaraj Day'
(b) 130 in previous year to 100th position by the Indian National Congress?
(c) 100 in previous year to 77th position (a) 26-01-1930 (b) 15-08-1947
(d) 100 in previous year to 67th position (c) 30-01-1948 (d) 31-12-1950
(e) None of the above/More than one of the (e) None of the above/More than one of the
above above
Ans. (a) : Ease of Doing Business Report was launched Ans. (a) : On 19th December 1929, Indian National
in 2003 by World Bank to provide assessment of Congress had passed "Purna Swaraj'' resolution at its
objective measures of business regulations and their Lahore session. Public declaration was made on 26
enforcement across 190 countries on 10 Parameters January 1930 - a day which the INC urged Indians to
which affects a business through its life cycle. As per celebrate as 'Independence Day'.
the Ease of Doing Business Report 2020, India has 28. When was Bihar established?
improved their rank from 77 in 2019 to 63rd in 2020 (a) 1911 (b) 1912
24. To improve institutional agricultural credit (c) 1913 (d) 1914
flow, what credit target for 2020-21 has been (e) None of the above/More than one of the
fixed in the Union Budget of India? above
(a) ` 10 lakh crore Ans. (b) : Bihar was established on 22 March 1912. On
(b) ` 13.5 lakh crore this day, the British carved out the state from Bengal in
(c) ` 15 lakh crore 1912. In 1912 Bihar and Orissa were separated from
(d) ` 16.5 lakh crore Bengal Presidency.
(e) None of the above/More than one of the 29. In Tinkathia System in Bihar, how much land
above was to be reserved for indigo cultivation?
Ans. (c) : To improve institutional agricultural credit (a) 01/10 (b) 01/03
flow, 15 Lakh crore credit target for 2020-21 had been (c) 03/20 (d) 03/25
fixed in the Union Budget of India. (e) None of the above/More than one of the
25. The objective of PM-KUSUM Scheme is above
(a) to reduce farmer's dependence on monsoon Ans. (c) : Tinkathiya System was the system under
for irrigation which the native farmers of Champaran in Bihar were
(b) to reduce farmer's dependence on forced to cultivate indigo in 3 parts of land out of every
moneylenders for credit 20 parts of land. This gave berth to several revolts and
(c) promotion of floriculture in India movements.
(d) to remove farmers dependence on diesel and 30. Who was the first Chief Minister of Bihar?
kerosene and to link pump sets to solar (a) Shri Krishna Sinha
energy (b) Satya Pal Malik
(e) None of the above/More than one of the (c) Nitish Kumar
above (d) Rabri Devi
Ans. (d) : PM-KUSUM Scheme was launched in 2019. (e) None of the above/More than one of the
It is a scheme for farmers for installation of solar pumps above
& grid connected solar and other renewable power Ans. (a) : Shri Krishna Sinha also known as Shri Babu
plants in the country. The solar pumps will save the was the the first Chief Minister of Bihar. Nitish kumar
expenditure incurred on diesel for running diesel pumps is the present Chief Minister of Bihar.
and provide the farmers a reliable source of irrigation 31. Where was the first Congress Session in Bihar
through solar pumps apart from preventing harmful held?
pollution from running diesel pumps. (a) Patna (b) Gaya
26. Which of the following pairs is correct? (c) Muzaffarpur (d) Darbhanga
(a) Vinoba Bhave–Second Individual Satyagrahi (e) None of the above/More than one of the
(b) C.R. Das–Deshbandhu above
(c) William Wedderburn–Congress President in Ans. (a) : Indian National Congress held its 27th
1907 Session at Bankipore Patna in 1912. It was the first
(d) Shyamji Krishna Verma–Founder of India INC Session in Bihar which was presided over by Rao
House in Paris Bahadur Raghunath Narasinha Mudholkar. Second
(e) None of the above/More than one of the Session of INC in Bihar was held in Gaya in 1922. It
above was presided by Chittaranjan Das.
66th BPSC (Pre) Exam. 2020 288 YCT
32. Which title was given to Jayaprakash 36. For how many days did Dandi March last?
Narayan? (a) 10 days (b) 20 days
(a) Praja Hitechhu (c) 24 days (d) 30 days
(b) Lok Nayak (e) None of the above/More than one of the
(c) Lokmanya above
(d) Rashtra Nayak Ans. (c) : The Dandi March also known as Salt
(e) None of the above/More than one of the Satyagraha , was an act of nonviolent civil disobedience
above led by Gandhi ji. The 24 day March from 12 March to 5
Ans. (b) : Jayaprakash Narayan was an Indian socialist April 1930 was a tax resistance campaign against the
pro-independence activist, theorist and political leader. British salt monopoly. Gandhi ji and 78 other from
He led so many movements for the interest of the Sabarmati Ashram had embanked on the Dandi Yatra
people and people gave him epithet of 'Lok Nayak'. on 12 March 1930 to break the law which had been
33. In which of the following colleges, Gandhiji had imposed tax on salt.
studied? 37. Who is the author of the popular song,
(a) Samaldas College, Bhavnagar Sarfaroshi Ki Tamanna Ab Hamare Dil Mein
(b) Dharmendrasinhji College, Rajkot Hai?
(c) Gujarat College, Ahmedabad (a) Surya Sen
(d) Bahauddin College, Junagadh (b) Chandra Shekhar Azad
(e) None of the above/More than one of the (c) Sardar Bhagat Singh
above (d) Ram Prasad Bismil
(e) None of the above/More than one of the
Ans. (a) : Mahatma Gandhi had studied at Samaldas Art above
College, Bhavnagar, Gujarat. Samaldas Art College was
Ans. (e) : Ram Prasad Bismil was an Indian freedom
established in 1885 by Late Maharaja Sir Takhtasinji of
fighter and the Chief founder of HRA. He is perhaps
Bhavnagar. most remembered for the Kakori Conspiracy case
34. Which of the following persons had occurred on 9th August 1925. He was hanged on 19th
participated actively in the Revolt of 1857? December 1927 at Gorakhpur Jail.
(a) Nana Saheb (Kanpur) Sarfaroshi Ki Tamanna is an Urdu Patriotic poem
(b) Begum Hazrat Mahal (Lucknow) written by Bismil Azimabadi of Patna in 1921. Later the
(c) Maulavi Ahmadullah (Faizabad) poem "Sarfaroshi Ki Tamanna" was immortalized by
(d) Begum Zeenat Mahal (Delhi) Ram Prasad Bismil.
(e) None of the above/More than one of the 38. Which movement was started as a reaction to
above the Partition of Bengal?
Ans. (e) : The revolt of 1857 was the conscious (a) Non-Cooperation Movement
beginning of the Independence struggle against the (b) Civil Disobedience Movement
colonial tyranny of the British. Many leaders took part (c) Swadeshi Movement
in the revolt of 1857. Some are as follows. (d) Purna Swaraj Movement
Place Leader of Revolt of 1857 (e) None of the above/More than one of the
Lucknow Begum Hazrat Mahal above
Delhi Bahadur Shah, General Bakht Khan Ans. (c) : Lord Curzon officially announced Partition
Barrackpore Mangal Panday of Bengal on July 20, 1905 and on October 16, 1905
Bihar Kunwar Singh Partition came into effect. Against the Partition of
Faizabad Maulavi Ahmadullah Bengal, Swadeshi Movement was started on 7 August
Kanpur Nana Saheb and Tantia Tope 1905.
35. About which Act, Jawaharlal Nehru had said, 39. Who among the following advocates had
"We were provided with a car with all brakes appeared for the INA trials at Delhi in 1945-
and no engine"? 46?
(a) Act of 1858 (a) Dr. Rajendra Prasad
(b) Bhulabhai Desai
(b) Act of 1909
(c) K.M. Munshi
(c) Act of 1919 (d) Sardar Patel
(d) Act of 1935 (e) None of the above/More than one of the
(e) None of the above/More than one of the above
above Ans. (b) : INA trials was the British Indian trial by court-
Ans. (d) : The Government of India Act, 1935 marked a martial of a number of officers of INA between November
major step towards conferring 'Dominion Status' on 1945 & May 1946 On various charges of treason, torture,
India but it fell short on several counts so that Jawahar murder & abetment to murder during second World War.
Lal Nehru had said about the Government of India Act The trial began at Red Fort. Shah Nawaz Khan, PK Sehgal
1935 that we were provided with a car with all break & G S Dhillon were three senior INA officer against
and no engines. He mentioned this Act as the "Charter whom the INA trial was initiated. Bhulabhai Desai,
of Slavery" Jawahar Lal Nehru, Tej Pratap Sapru were the advocates
appointed by INC for the INA trial.
66th BPSC (Pre) Exam. 2020 289 YCT
40. Where did Madam Cama unfurl India's 55. The sensitive area of the human tongue to
tricolour flag of freedom? bitterness is
(a) Paris (b) London (a) Tip
(c) Stuttgart (d) Geneva (b) Middle part
(e) None of the above/More than one of the (c) Posterior part
above (d) Edge
Ans. (c) : Madam Bhikaji Rustom Cama became the (e) None of the above/More than one of the
first person who hoisted Indian flag on foreign soil in above
Stuttgart in Germany on 22nd August 1907. Ans. (c) : Human tongue mainly contains 4 types of
41. The vitamin which is effective in blood clotting taste buds :- Sour, Sweet, Salty & Bitter. The tip of the
is tongue contains the sweet taste buds while the bitter
(a) Vitamin A taste buds are present in posterior part. Sour at the sides
(b) Vitamin B of the tongue.
(c) Vitamin D
(d) Vitamin K 56. Ginger is a stem and not root because
(e) None of the above/More than one of the (a) It stores food material
above (b) It has nodes and internodes
Ans. (d) : Vitamin K, the generic name for a family of (c) It grows horizontally in the soil
compounds with a common chemical structure of 2- (d) It lacks chlorophyll
methyl –1, 4- naphthoquinone, is a fat soluble Vitamin (e) None of the above/More than one of the
that is naturally present in some foods and is available above
as a dietary supplement. Ans. (b) : Ginger is a stem not root because it has nodes
It helps to make various proteins that are needed for and internodes. It is an example of rhizome because the
blood clotting and building of bones. Prothrombin is a stem exist below the ground level.
vitamin K -dependent protein directly involved with 57. The sweetest sugar among the following is
blood clotting. Scientific Name of Vitamin-K is (a) Fructose (b) Glucose
Phytonadione. (c) Maltose (d) Sucrose
52. The female sex hormone is (e) None of the above/More than one of the
(a) Estrogen above
(b) Androgen Ans. (a) : Fructose is a type of Sugar, it is also known
(c) Insulin
as fruit sugar because it is primarily found in fruits.
(d) Oxytocin
(e) None of the above/More than one of the Fructose is the sweetest Sugar. It in 1.2-1.8 times
above sweeter than sucrose.
Ans. (a) : Estrogen, Progesterone & small amounts of 58. Among the following, which is not a true fruit?
Testosterone are main sex hormones in females. The (a) Apple (b) Grape
Estrogen and Progesterone are secreted for the influence (c) Date (d) Plum
on woman's reproductive health and are termed as (e) None of the above/More than one of the
female hormones. above
53. The Nobel Prize for developing treatment of Ans. (a) : All the true fruits develop from ovaries where
Parkinson's disease was given to as all the false fruits develop from any part of the plant
(a) Arvid Carlsson except ovary. Apple does not grow from ovary, so it is a
(b) John F. Enders false fruit. Where as Grapes, Date, Plum are true fruit.
(c) Robert B. Laughlin 59. Legumes are highly nutritious because they are
(d) Walter Kohn rich in
(e) None of the above/More than one of the (a) Fat (b) Protein
above (c) Oil (d) Starch
Ans. (a) : Dr Arvid Carlsson, a Swedish Scientist was (e) None of the above/More than one of the
awarded the Nobel Prize in Medicine in 2000 for above
research on the brain's chemical signals and the Ans. (b) : Legumes are highly nutritious because they
resulting treatment for parkinson's disease. are particularly rich in proteins, carbohydrates, iron,
54. The malfunctioning of thyroid gland is due to copper, zinc etc., and are generally low in fat.
the deficiency of. 60. Clove, a spice, is obtained from which part of
(a) Vitamin A (b) Calcium the plant?
(c) Iodine (d) Iron (a) Fruit (b) Stem
(e) None of the above/More than one of the (c) Root (d) Flower bud
above (e) None of the above/More than one of the
Ans. (c) : The thyroid gland produce hormones that above
regulate the body's metabolic rate controlling heart , Ans. (d) : Cloves are generally used as spice. Cloves
muscle and digestive function, brain development and are obtained from flower bud of the plant. These are
bone maintenance. The malfunctioning of thyroid gland aromatic dried flower buds. It belongs to the family
is due to the deficiency of Iodine. Myrtaceae.

66th BPSC (Pre) Exam. 2020 290 YCT


61. The radiations used in the treatment of muscle Ans. (d) : Faraday constant represent the magnitude of
ache are electric charge per mole of electron. It is universal
(a) Infrared (b) Microwave constant. It has currently universally accepted value.
(c) UV (d) X-ray Faraday constant (F) = 96485.3328959 C mol-1
(e) None of the above/More than one of the 65. 'Light-year' is a unit of
above (a) Time
Ans. (a) : Infrared radiations are used in the treatment (b) Speed
of muscles ache. Infrared are also used for acceleration (c) Distance
of healing, tissue repairs pressure sores, in reducing (d) Intensity of light
pain, muscle ache etc. A part from medical field it is (e) None of the above/More than one of the
also used in thermal imaging cameras, short range above
communication etc. Ans. (c) : A light year is defined as the measurement of
62. The total resistance of a circuit having two length or the distance travelled by light in vacuum in a
parallel resistors is 1.403 kilo-ohm. If one of the year. So light year is a unit of distance.
resistors is 2.0 kilo-ohm, then the other resistor 66. Which of the following does not change when
will be light travels from one medium to another?
(a) 1.403 kilo-ohm (a) Velocity
(b) 2.0 kilo-ohm (b) Wavelength
(c) 3.403 kilo-ohm (c) Frequency
(d) 4.70 kilo-ohm (d) Refractive index
(e) None of the above/More than one of the (e) None of the above/More than one of the
above above
Ans. (d) : Total Resistance (R) = 1.403 kilo-ohm Ans. (c) : Frequency is the property of light that does
R1 =2.0 kilo-ohm. not change when light travels from one medium to
1.403kilo ohm =1403 ohm another whereas, wavelength , speed of light, Velocity
2 kilo ohm =2000 ohm and Refractive index changes when light travels from
Now for parallel combination: one medium to another .
1/R= 1/R1 + 1/R2 67. The velocity of electromagnetic waves is
1 1 1 (a) 3 × 108 ms–1 (b) 3 × 107 ms–1
6 –1
= = + (c) 3 × 10 ms (d) 3 × 105 ms–1
1403 2000 R2 (e) None of the above/More than one of the
1 1 above
R2 = −
1403 2000 Ans. (a) : Velocity of an electomagnetic wave is
R2 = 4700.16 ohm property which is dependent on the medium in which it
Or is travelling. Electromagnetic waves are the
R2 = 4.70016 Kilo-ohm. composition of oscillating electric and magnetic field.
These waves travel with a constant velocity of 3×108
63. On heating, the resistance of a semiconductor ms-1 in a vacuum.
(a) Increases
68. The first man who placed his foot on the moon
(b) Decreases is
(c) Remains same (a) Leonov
(d) First increases and then decreases (b) Neil Armstrong
(e) None of the above/More than one of the (c) Michael Collins
above (d) James Van Allen
Ans. (b) : As the temperature rises, the valence band (e) None of the above/More than one of the
electrons in semiconductor jump into conduction band above
to gain in thermal energy and become capable of Ans. (b) : Neil Armstrong, became the first man who
conducting current so that on heating, it increases the walked on the moon on 20th July 1969. Neil Armstrong,
conductance of semiconductor and decreases the Edwin Buzz Aldrin and Michael Collins were the
resistance. Hence the resistance of semiconductor astronauts in Apollo 11 space mission conducted by
U.S.A.
material decreases with increase in temperature
69. The number of neutrons in the nucleus of
64. Faraday constant plutonium nuclide (94Pu242) is
(a) Depends on the amount of the electrolyte (a) 94 (b) 148
(b) Depends on the current passed in the (c) 242 (d) 336
electrolyte (e) None of the above/More than one of the
(c) Depends on the volume of the solvent in above
which the electrolyte is dissolved Ans. (b) The number of neutrons in the nucleus of
(d) Is a universal constant plutonium nuclide (94Pu242) is 148. No. of Neutrons =
(e) None of the above/More than one of the Mass Number – No of protons. (No. of Neutron = 242 –
above 94 = 148)

66th BPSC (Pre) Exam. 2020 291 YCT


70. The highest viscosity among the following is of (a) Nitric oxide
(a) Water (b) Air (b) Nitrogen dioxide
(c) Blood (d) Honey (c) Carbon dioxide
(e) None of the above/More than one of the (d) Carbon monoxide
above (e) None of the above/More than one of the
Ans. (d) : Viscosity is a measure of a fluid's resistance above
to flow. As the viscosity increases, the flow rate Ans. (d) : Due to incomplete combustion of petrol &
decreases. All fluids, gases having low density have low diesel carbon is not completely oxidised and produces
viscosities and those having high density have the high carbon monoxide.
viscosity. Among the above given options, honey has 76. The pH of a solution changes from 3 to 6. The
highest Viscosity. H+ ion concentration will be
71. Milk is a poor source of (a) Increased 3 times
(a) Calcium (b) Decreased 3 times
(c) Decreased 10 times
(b) Protein
(d) Decreased 1000 times
(c) Vitamin C (e) None of the above/More than one of the
(d) Carbohydrate above
(e) None of the above/More than one of the
Ans. (d) : Given pH = 3
above
So, Hydrogen ion (H+) = 10-3
Ans. (c) : Milk contains high quantities of vitamins A & Now pH = 6 so (H+) = 10-6
B such as thiamine, nicotinic acid and riboflavin, Hence, the H+ ion concentration will decrease
although it is low source of vitamin C & E. Calcium, 1000 times.
Sodium, Phosphorus & Potassium are major minerals in 77. A mixture of sand and naphthalene can be
milk. It is colloidal suspension of proteins. Lactose is separated by
the main carbohydrate. Lactose is disaccharide found (a) Sublimation
only in milk & dairy products. (b) Distillation
72. The breath test conducted by police to check (c) Chromatography
drunken driver has which one of the following (d) Fractional distillation
on the filter paper? (e) None of the above/More than one of the
(a) Potassium dichromate-sulfuric acid above
(b) Potassium permanganate-sulfuric acid Ans. (a) : Sublimation is a process in which solid
(c) Silica gel coated with silver nitrate converts into gas directly without converting into liquid.
(d) Turmeric Examples of sublimation are-Dry ice, Naphthalene
(e) None of the above/More than one of the Mothballs etc. A mixture of sand and naphthalene can
above be separated by sublimation, when the Naphthalene is
Ans. (a) : A breath test used by traffic police to check heated, it sublimates easily.
drunken driving uses potassium dichromate-sulphuric 78. The number of sigma and pi bonds in benzene
acid. When alcohol vapor makes contact with the are
dichromatic crystals, the colour changes from yellow to (a) 3, 3 (b) 3, 6
green. The degree of colour change is directly related to (c) 12, 3 (d) 12, 6
the level of alcohol in person's breath. (e) None of the above/More than one of the
above
73. Glucose is converted to ethyl alcohol by the
enzyme Ans. (c) : Benzene is a hydrocarbon composed of six
(a) Maltase (b) Invertase carbons connected via covalent bond & exists in the
form of ring. Its chemical formula is C6H6.
(c) Zymase (d) Diastase
The structure of Benzene is-
(e) None of the above/More than one of the
above
Ans. (c) : Zymase enzyme converts glucose into ethanol
through process of fermentation. This enzyme is used in
the preparation of alcoholic beverages commercially.
74. Limewater is turned milky by
(a) CO (b) CO2
(c) O2 (d) O3 From the benzene structure we can see there are 6 C-H
(e) None of the above/More than one of the bonds and 6 C-C bonds, hence there are total of 12
sigma bonds in benzene and we can see there are 3
above bonds. Hence there and 3 pi-bonds in benzene.
Ans. (b) : Limewater's turned milky, when CO2 gas is 79. The elements present in urea are
passed into it. CO2 turns Limewater into milky due to (a) C, H, O (b) C, N, O
formation of insoluble calcium carbonate (CaCO3). (c) C, H, N (d) C, H, N, O
75. The incomplete burning of petrol and diesel (e) None of the above/More than one of the
produces above
66th BPSC (Pre) Exam. 2020 292 YCT
Ans. (d) : Urea is nitrogenous compound containing a (c) Joseph Biden's election campaign
carbonyl group attached to two amine groups with (d) 'make army great again' campaign
osmotic diuretic activity. It has 4 elements which are (e) None of the above/More than one of the
Carbon, Nitrogen, Oxygen and Hydrogen. above
80. The poorest conductor of heat among the Ans. (b) : Operation MAGA is related to Donald
following is Trump's re-election campaign. Donald Trump has
(a) Copper (b) Lead launched Operation Maga in 2020. The Operation
(c) Mercury (d) Zinc MAGA, which it describes as a full marshalling of top
(e) None of the above/More than one of the level surrogates, campaign coalitions and Trump
above supporters " to rally behind the President until he can
Ans. (c) : The materials which cannot transfer heat are return to the trail.
poor conductors. Poor conductors are also called 85. Which Arab state has started first nuclear
insulators, copper, lead, zinc are the good conductor of power plant?
heat. Mercury is the poor conductor of heat but a good (a) Iraq
conductor of electricity. (b) Saudi Arabia
Thermal conductivity of mercury is 8 & thermal (c) United Arab Emirates
conductivity of copper is 385 (w/m.k). (d) Egypt
81. Who represented India's perspective in the (e) None of the above/More than one of the
Fifth BRICS Culture Ministers Meeting held above
through a video conference in September 2020? Ans. (c) : Barakah Nuclear Power Plant in the Abu
(a) Prahlad Singh Patel Dhabi Emirate (UAE) was the first Nuclear Power
(b) Ramesh Pokhriyal Nishank Station in Arab world and part of the oil producing
(c) Nitin Gadkari State's efforts to diversify its energy mix. This power
(d) Ravi Shankar Prasad plant got operational on 31st July 2020. It will produce
(e) None of the above/More than one of the 5600 Megawatts of electricity.
above 86. Which was the first Arab state to sign peace
Ans. (a) : In September 2020, the 5th BRICS Culture deal with Israel?
Ministers Meeting was held through video conference (a) Egypt (b) Jordan
under the Chairpersonship of Russian Federation. Union (c) Bahrain (d) Sudan
Minister of State for Culture and Tourism Mr. Prahlad (e) None of the above/More than one of the
Singh Patel had represented India's perspective in this above
meeting. Ans. (a) : Egypt was the first Arab State to sign a peace
82. Where was the 36th ASEAN Summit virtually deal in Washington D.C., the United States On 26th
held in June 2020? March 1979 while Jordon signed a peace deal with
(a) Thailand (b) Singapore Israel in 1994.
(c) Indonesia (d) Vietnam 87. Which country assumed the role of Chairman
(e) None of the above/More than one of the of the International Labour Organization's
above Governing Body for the period of October 2020
Ans. (d) : In June 2020, the 36th ASEAN Summit was till June 2021?
held via video conference to focus on the Covid-19 (a) Japan
Pandemic response, post pandemic recovery and further (b) New Zealand
cooperation with partner countries. This Summit was (c) Australia
chaired by Vietnam. (d) India
83. Which of the following has topped in the (e) None of the above/More than one of the
Human Capital Index, 2020? above
(a) Japan
Ans. (d) : India has assumed the chairmanship of the
(b) South Korea
(c) Singapore Governing Body of the International Labour
(d) Hong Kong Organization (ILO) for the period of October 2020 -
(e) None of the above/More than one of the June 2021.
above 88. 'Operation My Saheli' launched in September
Ans. (c) : Human Capital Index was released by World 2020 aims
Bank. It includes health and actuation data of 174 (a) To encourage self-defence education among
countries. Singapore with a score of .088 had topped in girls
the Human Capital Index 2020 followed by Hongkong (b) To boost the security of women passengers in
& Japan. India with score of 0.49 had ranked at 116th. trains
84. Recently in USA, 'Operation MAGA' is related (c) To explore new job opportunities for women
to (d) To boost awareness against girl child labour
(a) campaign against COVID-19 (e) None of the above/More than one of the
(b) Donald Trump's re-election campaign above
66th BPSC (Pre) Exam. 2020 293 YCT
Ans. (b) : Indian Railways has launched 'Operation My 93. At the international airport of which city, India
Saheli' on 29th October 2020. It aims to provide security launched its first COVID-19 testing facility?
to women passengers travelling in trains from the (a) Kolkata (b) Delhi
source station to the destination Station. (c) Mumbai (d) Bengaluru
89. On the occasion of the first anniversary of the (e) None of the above/More than one of the
Fit India Movement, who gave the Mantra, above
'Fitness ki Dose, Aadha Ghanta Roz (Fitness Ans. (b) : Delhi's Indira Gandhi International Airport
Dose, Half an Hour Daily)'? had launched its first COVID-19 testing facility for
(a) Prime Minister Narendra Modi International passengers in September 2020.
(b) Union Home Minister Amit Shah
(c) Union Health Minister Dr. Harsh Vardhan 94. Rafale fighter aircraft has been formally
(d) Yoga Guru Baba Ramdev inducted into the Indian Air Force at which Air
(e) None of the above/More than one of the Force Station?
above (a) Hindon (b) Sarsawa
Ans. (a) : Fit India Movement was launched on 29th (c) Ambala (d) Amritsar
August 2019, with a view to make fitness an integral (e) None of the above/More than one of the
part of our daily lives. On the occasion of the first above
anniversary of the Fit India Movement Prime Minister Ans. (c) : Five Rafale fighter jets were formally
of India Narendra Modi gave the Mantra, Fitness ki inducted into the Indian Air Force on 10th September
Dose, Aadha Ghanta Roz (fitness Dose, Half an Hour 2020 at a glittering ceremony in the Ambala air base.
daily). The Dassault Rafale jet is a multirole fighter jet
90. On the one hundredth birth anniversary of designed & built by Dassault Aviation, a French aircraft
Rajmata Vijaya Raje Scindia, Prime Minister manufacturer.
Narendra Modi released a 95. The name of India's first indigenous anti-
(a) Fifty rupees coin radiation missile is
(b) Fifty rupees note (a) Tandav (b) Trinetra
(c) One hundred rupees coin (c) Saksham (d) Rudram
(d) One hundred rupees note (e) None of the above/More than one of the
(e) None of the above/More than one of the above
above Ans. (d) : Rudram -I is India's first indigenous anti
Ans. (c) : On 12th October 2020, Prime Minister radiation missile. Rudram-I is an air to surface missile.
Narendra Modi released commemorative coin of Rs.100 It is designed & developed by DRDO.
rupee denomination to celebrate the birth centenary of
Rajmata Vijaya Raje Scindia through video conference. 96. Who has been appointed by the Board of
Control for Cricket in India as the Head of All
91. Which state's short documentary film, Chi
India Women's Selection Committee?
Lupo has won Dada Saheb Phalke Award,
(a) Mithu Mukherjee
2020?
(a) Manipur (b) Neetu David
(b) Nagaland (c) Renu Margate
(c) Meghalaya (d) V. Kalpana
(d) Arunachal Pradesh (e) None of the above/More than one of the
(e) None of the above/More than one of the above
above Ans. (b) : Neetu David has been appointed by the BCCI
Ans. (d) : Arunachal Pradesh filmmaker Kezang D as the head of All India Women's Selection Committee.
Thongdok had bagged the Dada Saheb Phalke Award Other member of All India women's selection
2020 for a short documentary 'Chi Lupo' . Chi Lupo is committee includes Mithu Mukherjee, Renu Margrs,
based on the custom of honey hunting among the Arati Vaidya & V. Kalpana.
Sherdukpen tribe of West Kameng district of Arunachal 97. Who has won the Women's Singles US Open
Pradesh. Tennis Tournament, 2020?
92. Which private sector bank has launched 'e- (a) Naomi Osaka
Kisaan Dhan' app for farmers? (b) Bianca Andreescu
(a) Axis Bank (c) Sofia Kenin
(b) HDFC Bank (d) K. Pliskova
(c) IDBI Bank (e) None of the above/More than one of the
(d) Kotak Mahindra Bank above
(e) None of the above/More than one of the Ans. (a) : Japan's Naomi Osaka had defeated Victoria
above Azarenka in the finals of the US Open Tennis
Ans. (b) : HDFC Bank had launched 'e-Kissan Dhan' Tournament, 2020. Dominic Thiem had won the US
app for farmers all over India. Through this app, Open 2020 men's single title.
farmers will be able to access the services of banking 98. Which athlete scored first position in the
and agriculture. London Marathon held on 4th October, 2020?
66th BPSC (Pre) Exam. 2020 294 YCT
(a) Eliud Kipchoge 103. Who has been selected for the Nobel Peace
(b) Shura Kitata Prize, 2020?
(c) Vincent Kipchumba (a) European Union
(d) Sisay Lemma (b) World Food Programme
(e) None of the above/More than one of the (c) Robert B. Wilson
above (d) Paul R. Milgrom
Ans. (b) : Ethopian athlete Shura Kitata had secured (e) None of the above/More than one of the
first position in the London Marathon held on 4th above
October 2020. Vincent Kipchumba of Kenya was Ans. (b) : The UN World Food Programme (WFP) has
runner-up in the event. been awarded the 2020 Nobel Peace Prize. WFP
99. Which Indian cricketer along with M.S. Dhoni provides food assistance to millions across the world -
announced retirement from the international often in extremely dangerous & hard to access
cricket on 15th August, 2020? condition. The Nobel Peace Prize 2021 was awarded
(a) Suresh Raina jointly to Maria Ressa and Dmitry Andreyevich
(b) Harbhajan Singh Muratov.
(c) Bhuvneshwar Kumar 104. Which Indian film actor has been included in
(d) Rohit Sharma the Time Magazine's list of hundred most
(e) None of the above/More than one of the influential people of 2020?
above (a) Ayushaman Khurana
Ans. (a) : On 15th August 2020, former Indian cricket (b) Shahrukh Khan
team captain M.S Dhoni along with Suresh Raina (c) Ajay Devgan
announced his retirement from International Cricket. (d) Sunny Deol
100. Who has been appointed as the Chairperson of (e) None of the above/More than one of the
the National School of Drama recently? above
(a) Mahesh Bhatt
(b) Akshay Kumar Ans. (a) : Indian Prime Minister Narendra Modi, Actor
(c) Anupam Kher Ayushman Khurana, Shaheen Bagh protester Bilkis,
(d) Paresh Rawal Alphabet CEO Sundar Pichai, Ravindra Gupta had been
(e) None of the above/More than one of the featured in the time magazine's list of 100 Most
above Influential people of 2020.
Ans. (d) : Paresh Rawal had been appointed 105. How many Indian beaches have been
chairperson of the National School of Drama (NSD). He recommended for the Blue Flag Certification
was appointed for four year term. recently?
101. Who is the Chairman of the Defence Research (a) Six (b) Seven
and Development Organization? (c) Eight (d) Nine
(a) Surjeet Singh Deswal (e) None of the above/More than one of the
(b) Dr. G. Satheesh Reddy above
(c) Arvind Saxena Ans. (c) : The Blue flag certificate is an internationally
(d) Charanjit Singh Attra recognized eco-label given by Foundation for
(e) None of the above/More than one of the Environment Education, Denmark. Blue flag beaches
above are considered the benchmark for clean and hygienic
Ans. (b) : Defense Research and Development beaches in the world. As of December 2020, eight
Organization. (DRDO) works under the administrative Indian beaches had been recommended for the Blue flag
control of Ministry of Defense, Government of India. certification.
Dr. G. Satheesh Reddy is the incumbent Chairman of However as of September 2021, Ten beaches in India
DRDO. have been awarded the Blue flag certification. Namely:
102. For which discipline, Dr. Bushra Ateeq and Dr. Golden Beach (Odisha), Shivajpur Beach (Gujarat),
Riten Agarwal have been selected for Shanti
Kappad Beach (Kerala), Ghoghla Beach (Diu),
Swarup Bhatnagar Prize, 2020?
(a) Chemical Sciences Radhanagar (Andaman and Nicobar), Kasaragod Beach
(b) Physical Sciences (Karnataka), Padubidri (Karnataka) Rushikonda Beach
(c) Medical Sciences (Andhra Pradesh), Kovalam Beach (Tamil Nadu),
(d) Mathematical Sciences Golden Beach (Puducherry).
(e) None of the above/More than one of the 106. Ram Vilas Paswan started his political career
above from which political party?
Ans. (c) : Shanti Swarup Bhatnagar Prize is given every (a) Janta Party
year by CSIR in the fields of Biological sciences, (b) Bharatiya Lok Dal
chemical science & Earth, Atmosphere, Engineering (c) Samyukta Socialist Party
sciences. Dr. Bushra Ateeq and Dr. Ritesh agarwal had (d) Praja Socialist Party
been Awarded Shanti Swarup Bhatnagar Prize 2020 for (e) None of the above/More than one of the
Medical Sciences. above
66th BPSC (Pre) Exam. 2020 295 YCT
Ans. (c) : Ram Vilas Paswan started his political career 111. Who among the following participated in the
from Samyuakta Socialist Party. He was elected to the Parliament of Religions held at Chicago in
Bihar State Legislative Assembly in 1969 from Alauli, a 1893?
reserved constituency. He formed Lok Janshakti Party (a) Dayananda Saraswati
in 2000. He died on 8 October 2020. (b) Swami Vivekanand
107. When did the Election Commission of India (c) Mahatma Gandhi
issue a Press Note for the General Elections to (d) Raja Rammohan Roy
(e) None of the above/More than one of the
the Legislative Assembly of Bihar, 2020?
above
(a) 23rd September, 2020
(b) 24th September, 2020 Ans. (b) :The World's Parliament of religions was
(c) 25th September, 2020 convened in Chicago in 1893. Its goal was to bring
(d) 26th September, 2020 together world religious leaders on common ground.
(e) None of the above/More than one of the Swami Vivekananda had participated at the Parliament
above of Religions held in Chicago, USA in September 1893.
He stressed upon world harmony & how religions,
Ans. (c) : The Election Commission of India had while co-existing with one another, must preserve its
announced a schedule for General Election to the individuality and grow.
Legislative Assembly of Bihar on 25th September 2020.
112. Who established Swaraj Party in 1923?
This election was conducted in three phases. (a) Mahatma Gandhi
108. In which state, 'Ghar Tak Fiber' Scheme has (b) Vallabhbhai Patel
been launched on 21st September, 2020 (c) C.R. Das and Motilal Nehru
through a video conference? (d) B.R. Ambedkar
(a) Uttar Pradesh (e) None of the above/More than one of the
(b) Madhya Pradesh above
(c) Bihar Ans. (c) : The Swaraj Party or the Congress Khilafat
(d) Karnataka Swarajya Party was formed by C.R. Das and Motilal
(e) None of the above/More than one of the Nehru on 1st January 1923. It was created in the
above aftermath of important events such as Non Cooperation
Ans. (c) :In 2020, Prime Minister Narendra Modi had Movement's withdrawal, the Government of India Act
launched ‘Ghar Tak Fibre’ scheme in Bihar. This 1919 and the 1923 elections. C.R. Das was the President
scheme aims at connecting all 45,945 villages of Bihar & Motilal Nehru was the Secretary of this party.
with high speed optical fibre internet by 31st March 113. Who wrote the famous play, Neel Darpan in
2021. This scheme is implemented by the Ministry of which oppression of indigo farmers was
Electronics and Information Technology. displayed?
109. Bihar politician Late Raghuvansh Prasad (a) Sharat Chandra Chatterjee
Singh was for the first time elected to which (b) Rabindranath Tagore
Lok Sabha? (c) Barindra Ghosh
(a) Tenth Lok Sabha (d) Dinabandhu Mitra
(b) Eleventh Lok Sabha (e) None of the above/More than one of the
(c) Twelfth Lok Sabha above
(d) Thriteenth Lok Sabha Ans. (d) : Neel Darpan was written by Dinabandhu
(e) None of the above/More than one of the Mitra in 1858-59. The drama was written in context of
above Indigo Revolt of Bengal. The play examines treatment
Ans. (b) : Raghuvansh Prasad Singh was an Indian of the peasants by Indigo planters. It showed how
Politician. He was the first time elected to the 11th Lok farmers were coerced into planting indigo without
Sabha (1996). He died from the complications from adequate payment.
COVID-19 at AIIMS, New Delhi on 13th September 114. The famous Battle of Wandiwash in 1760 was
2020 at the age of 74. fought by the British against whom?
110. Which among the following is not a member of (a) The French (b) Spain
the Quadrilateral Security Dialogue–an (c) Mysore (d) Carnatic
informal strategic forum? (e) None of the above/More than one of the
(a) India (b) Japan above
(c) China (d) Australia Ans. (a) : Battle of Wandiwash was fought on January
(e) None of the above/More than one of the 22nd, 1760 between the British and the French army.
The battle was part of the third Carnatic War fought
above between the British and the French army. The decisive
Ans. (c) : QUAD is known as the Quadrilateral Security Battle of Wandiwash was won by the British army.
Dialogue. It is an informal strategic forum comprising General Eyre Coote of the British Army totally routed
four nations, namely-United States of America, India, the french army under Count De Lally and took
Australia and Japan. The main objectives of QUAD is Marquis de Bussy as prisoner. After this Battle the
to work for a free, open, prosperous & inclusive in British became supreme European power in Indian
Indo-Pacific region. subcontinent.

66th BPSC (Pre) Exam. 2020 296 YCT


115. Which of the following Acts introduced Ans. (e) : As per Buddhist text Anguttara Nikay there
separate electorate system in India? were 16 Mahajanpadas. Magadha gained sovereignty
(a) The Regulating Act, 1773 and became the most powerful Mahajanpada. Rajagriha
(b) The Charter Act, 1833
or modern-day Rajgir historically known as Girivraja
(c) The Pitt's India Act, 1784
(d) The Indian Councils Act, 1909 was the first capital of Magadha.
(e) None of the above/More than one of the 120. Swami Sahajananda was related to which of
above the following?
Ans. (d) Indian Council Act 1909 is more commonly (a) Tribal Movement in Bihar
known as Morley-Minto Reforms. It introduced (b) Labour Movement in Bihar
separate electorates on the basis of religion in India and (c) Peasant Movement in Bihar
introduced a few reforms in the legislative councils and (d) Caste Movement in Bihar
the Act increased the involvement of Indians in the (e) None of the above/More than one of the
government of British India. It considerably increased
above
the size of legislative councils at both Central &
Provincial level. Ans. (c) : Swami Sahajanand Saraswati is recognized as
116. Kol Mutiny of 1831 under Buddho Bhagat took one of the principle and significant leaders of peasantry
place in which of the following regions? in Bihar. He had established the Bihar Provincial Kisan
(a) Kutch Sabha in 1929. He was the first president of All India
(b) Singhbhum Kisan Sabha, which was formed with some congress
(c) Western Ghats socialists at the Lucknow session of INC on 11th April
(d) Satara 1936.
(e) None of the above/More than one of the
above 121. Which of the following was Gandhiji's first
Satyagraha Movement in India in which he
Ans. (b) : Kol uprising is also known in British records
as the Kol mutiny. It took place in 1831-32 in Chota used Civil Disobedience?
Nagpur and Singhbhum region of Bihar and Jharkhand. (a) Champaran
Kol revolted under the leadership of Buddho Bhagat, (b) Kheda
Joa Bhagat , Madara Mahato and Bindrai Manki. (c) Ahmedabad
117. Who spearheaded the 1857 Revolt in Bihar? (d) Rowlatt Satyagraha
(a) Nana Saheb (e) None of the above/More than one of the
(b) Tatya Tope above
(c) Kunwar Singh Ans. (a) The Champaran Satyagrah of 1917 was the
(d) Maulavi Ahmadullah first Civil Disobedience Movement launched by
(e) None of the above/More than one of the
above Mahtma Gandhi to protest against the injustice meted
Ans. (c) : Kunwar Singh was a leader during the Indian out to tenant farmers in Champaran district of Bihar.
Rebellion of 1857. He led the Indian Rebellion of 1857 122. In which of the following Harappan cities,
in Bihar. He was nearly the 80-year-old, when he took furrows of ploughed fields have been found?
charge of the sepoys who were stationed at Danapur on (a) Kalibangan
25th July 1857. He gave a tough fight to British. (b) Dholavira
118. Who drew Gandhiji's attention to the plight of (c) Mohenjo-daro
indigo peasants in Champaran? (d) Lothal
(a) Rajendra Prasad (e) None of the above/More than one of the
(b) Anugarh Narayan Sinha above
(c) Acharya Kripalani
Ans. (a) : The furrows of the pre-Harappan phase have
(d) Raj Kumar Shukla
been discovered at Kalibangan, Rajasthan. It indicates
(e) None of the above/More than one of the
that the fields were ploughed during the Harappan
above
period. The major archaeological finding in Kalibangan
Ans. (d) : Raj Kumar Shukla was a Indigo cultivator of are: Seven fire altars, Decorated brick, wheels of a toy
Champaran , Bihar. He had convinced Mahtma Gandhi
cart, Mesopotamian cylindrical seal.
to visit Champaran in Bihar to see the plight of the
cultivators under an oppressive system established by 123. Triratna or Three Jewels, i.e., right knowledge,
European Indigo planters. This led to the launch of right faith and right action are related to which
Champaran Satyagraha in 1917 by Gandhi ji. of the following?
119. Which was the first capital of ancient (a) Buddhism
Mahajanapada Magadh? (b) Hinduism
(a) Pataliputra (b) Vaishali (c) Jainism
(c) Champa (d) Anga (d) Christianity
(e) None of the above/More than one of the (e) None of the above/More than one of the
above above
66th BPSC (Pre) Exam. 2020 297 YCT
Ans. (c) : Jainism is one of worlds oldest religion. Ans. (b) : The Chola empire was very famous for their
Jainism provides threefold path know as Triratna or local self-government. Vijayalaya was the founder of
Three Jewels for the attainment of salvation (Moksha) this dynasty. In many villages, the administration was
Triratnas are:-(1)Right faith (2) Right knowledge & (3) carried out not by the government officials but by the
Right conduct. The three jewels of Jains ethics must be villagers themselves. The system of village autonomy
with assemblies (Sabhas, Urs and Nagaram) and their
followed to achieve the liberation of soul.
committees (Variyams) was developed through the ages
124. Which of the following rulers convened the and reached its culmination during the Chola rule.
Fourth Buddhist Council in Kashmir? 128. Pietra dura is related to which of the following?
(a) Ashoka (b) Ajatashatru (a) Decorating the walls with floral designs made
(c) Kanishka (d) Kalashoka of semi-precious stones
(e) None of the above/More than one of the (b) Building sloping walls in Minars
above (c) Use of arch in construction
Ans. (c) : Fourth Buddhist Coundil was held in 72 AD (d) Use of marble in buildings
at Kundalvana , Kashmir under the patronage of king (e) None of the above/More than one of the
Kanishka above
Following are the major Buddhist councils. They are Ans. (a) : Pietra dura is a decorative art. It is an art of
as follows:- carving the stone. It is related to decorating the walls
with floral designs made of semi precious stones. This
Council Venue Chairman Patron
art was most popular during Sahjahan's reign.
First Saptaparni Mahakassapa Ajatshatru
129. Which Mughal ruler established Karkhanas
cave, Rajgir for painting
Second Chullavana, Sabakami Kalashoka (a) Humayun (b) Akbar
Vaishali (c) Jahangir (d) Shah Jahan
Third Patliputra Moggaliputta Ashoka (e) None of the above/More than one of the
Tissa above
Fourth Kundalvana Vasumitra Kanishka Ans. (b) : Mughal emperor Akbar had established
125. Which of the following philosophies of India Karkhana or Tasvirkhana for painting. He had
propounded the atom theory? established separate department for painting. Most of
(a) Yoga (b) Nyaya the paintings during Akbar's reign were miniature
(c) Sankhya (d) Vaisheshika paintings.
(e) None of the above/More than one of the 130. The dual system of governance in Bengal was
above enforced by
(a) Warren Hastings
Ans. (d) : Vaisheshika is one of the 6 Schools of Indian (b) William Bentinck
philosophy. It was proposed by Maharishi Kanada. (c) Robert Clive
Vaisheshika philosophy of India has propounded the (d) Lord Curzon
atom theory. (e) None of the above/More than one of the
126. Which Delhi Sultan is known for adopting the above
'blood and iron' policy? Ans. (c) : Dual System of Governance in Bengal was
(a) Iltutmish established by Robert Clive in 1765. In this system
(b) Balban company had Diwani rights, that is the rights to collect
revenue and the Nizam or Indian chief had power on
(c) Ala-ud-din Khalji administrative authority. This system had been
(d) Muhammad bin Tughlaq abolished by Warren Hastings in 1772.
(e) None of the above/More than one of the 131. Which one of the following pairs of industry
above and place is not correctly matched?
Ans. (b) : Balban, the Sultan of Delhi has pursued the (a) Oil refinery–Barauni
policy of 'Blood & Iron'. This policy implied being (b) Cement–Banjari
ruthless to the enemies, use of sword, harshness and (c) Fertilizer–Bhaurahi
strictness and shedding blood. This policy was adopted (d) Wagon and Engineering–Bhagalpur
(e) None of the above/More than one of the
to safeguard Sultnate and to keep an eye on enemies.
above
127. Which medieval Indian empire was famous for Ans. (d) : Correct matches are as:-
elaborated local self-government? Industry Place
(a) Chalukya Oil refinery - Barauni
(b) Chola Cement - Banjari
(c) Solanki Fertilizer - Bhaurahi
(d) Parmar Bharat wagon & - Muzaffarpur &
(e) None of the above/More than one of the Engineering Mokama
above Silk industry - Bhagalpur

66th BPSC (Pre) Exam. 2020 298 YCT


132. Piedmont Swamp Soil is found only in which Ans. (c) : Valmiki National Park in West Champaran
one of the following districts of Bihar? district of Bihar is the only national park in Bihar.
(a) Madhubani 137. The largest island of Japan in terms of the
(b) Bhagalpur geographical area is
(c) West Champaran (a) Hokkaido (b) Honshu
(d) Sitamarhi (c) Shikoku (d) Kyushu
(e) None of the above/More than one of the (e) None of the above/More than one of the
above above
Ans. (c) : Piedmont Swamp Soil is found in Ans. (b) : Hokkaido, Honshu, Shikoku & Kyushu are
Northwestern part of West Champaran district of Bihar. four major islands of Japan. Honshu is the largest and
Here abundance of swamps are found, which favour the most populated among of these. Honshu is the 7th
cultivation of Paddy in this region. largest island of the world.
133. In which one of the following pairs of 138. Greenland is a part of which one of the
Administrative Divisions of Bihar, Ganga river following countries?
does not flow? (a) Denmark
(a) Darbhanga–Munger (b) Finland
(b) Purnea–Bhagalpur (c) Canada
(c) Tirhut–Saran (d) United Kingdom
(d) Kosi–Magadh (e) None of the above/More than one of the
(e) None of the above/More than one of the above
above Ans. (a) : Greenland is an autonomous region within
Ans. (d) : The Ganga river originates from Gangotri the kingdom of Denmark. It is world's biggest island. It
glacier in Uttrakhand. It flows in Uttarakhand, UP, is politically linked to Europe but is geographically part
Bihar, Jharkhand and West Bengal. It drains into the of N. America.
Bay of Bengal. In Bihar Ganga river flows through the 139. Which one of the following countries of the
middle of North Bihar & South Bihar from west to east. world has the largest Indian population as on
It flows from Darbhanga-Munger, Purnea-Bhagalpur, December, 2018?
and Tirhut-Saran. It does not flow from Kosi-Magadh. (a) United Arab Emirates
134. Among the following pairs of tribe and district, (b) Malaysia
which one is not correctly matched? (c) United Kingdom
(a) Santhal–Banka (d) Unites States of America
(b) Munda–Jamui (e) None of the above/More than one of the
(c) Oraon–Supaul above
(d) Kharwar–Bhagalpur Ans. (d) : As per data of Ministry of External Affairs,
(e) None of the above/More than one of the USA has the largest Indian population followed by
above UAE. The number of Indians in United States of
Ans. (b) : There are 33 tribes that have been notified as America (USA) is about 45 lakhs.
schedule tribes in Bihar. The correct matches are as :- 140. Among the following countries, which country
Tribes - District (Bihar) has recorded the highest annual gold output (in
Santhal - Banka, Jamui tonnes) in 2019?
Munda - Katihar (a) Russia
Oraon - Supaul
(b) Australia
Kharwar - Bhagalpur
(c) China
135. What is the percentage of all types of forest (d) Unites States of America
area of the total geographical area of Bihar
(e) None of the above/More than one of the
state?
(a) 7.27 (b) 6.87 above
(c) 3.21 (d) 12.77 Ans. (c) : China was the largest producer of Gold in
(e) None of the above/More than one of the world in 2021. About 332 metric tons of gold has been
above produced in China in 2019. Russia was the second
Ans. (e) : As per forest report 2021, Bihar has 7.84% largest producer of gold while Australia was the 3rd
forest cover, of total geographical area of Bihar. largest producer of gold.
136. Which one of the following districts is having 141. Among the following continents, which one has
the only National Park of Bihar? the highest number of countries?
(a) Nalanda (a) Europe
(b) Bhagalpur (b) Asia
(c) West Champaran (c) Africa
(d) East Champaran (d) North America
(e) None of the above/More than one of the (e) None of the above/More than one of the
above above
66th BPSC (Pre) Exam. 2020 299 YCT
Ans. (c) : Africa continent has the highest number of Ans. (e) :All the rivers given in the options are south
countries. It has 54 countries. where as Asia consists of flowing river in Bihar. Where as Phalgu, Son, Punpun
48 countries. and Kiul are the north flowing rivers in Bihar.
142. The State of India with the highest percentage 147. Atal Tunnel is across which one of the
of tribal population to its total population as following Himalayan ranges?
per the 2011 Census is (a) Zanskar
(a) Arunachal Pradesh (b) Western Pir Panjal
(b) Nagaland
(c) Ladakh
(c) Mizoram
(d) Meghalaya (d) Eastern Pir Panjal
(e) None of the above/More than one of the (e) None of the above/More than one of the
above above
Ans. (c) : As per the 2011 census, the state of Mizoram Ans. (d) : Atal Tunnel is world's longest highway
has highest percentage of tribal population to its total tunnel above 10,000 Feet. It is located at Rohtang Pass
population with 94.4% followed by Nagaland with in the Eastern Pir Panjal Range, and connects Manali
86.5%. with Lahaul. Spiti Valley in Himachal Pradesh.
143. Which one of the following coffee-growing area 148. India's 13th major port is going to be set up in
is not in Karnataka? which state?
(a) Chikmagalur (a) Kerala
(b) Coorg (b) Gujarat
(c) Baba Budangin (c) Maharashtra
(d) Pulneys (d) Tamil Nadu
(e) None of the above/More than one of the (e) None of the above/More than one of the
above above
Ans. (d) : Karnataka is the one of the largest coffee Ans. (c) : India has 12 major ports and 205 notified
producing states in India. Coorg, Chikmagalur, Hassan, minor and intermediate ports. In February 2020 The
Kodagu, Baba Budan Giri are the main coffee union cabinet had approved the setting up of India's 13th
producing area in Karnataka. Whereas, Pulney is coffee major port at Vadhavan in Maharashtra.
producing region in Tamilnadu.
149. Which one of the following states is a leading
144. Which one of the following districts of India is producer of solar energy in India?
the largest in terms of geographical area? (a) Telangana
(a) Leh (b) Kutch (b) Karnataka
(c) Jaisalmer (d) Barmer (c) Andhra Pradesh
(e) None of the above/More than one of the (d) Rajasthan
above (e) None of the above/More than one of the
Ans. (b) : Kutch is he largest district of India in terms above
of geographical area followed by Leh district. Mahe is Ans. (b) : As per report released by Ministry of New
the smallest district of India. Renewable Energy (MNRE) in 2020, Karnataka was the
145. The State of India with maximum number of leading producer of solar energy in India
wildlife sanctuaries is However, as per Data released by MNRE in March
(a) Karnataka 2022, Rajasthan has overtaken Karnataka to rank first in
(b) Tamil Nadu the country with an installed capacity of 10,000 MW of
(c) Maharashtra solar energy
(d) Madhya Pradesh 150. Among the following pairs of mineral and
(e) None of the above/More than one of the district, which one is correctly matched?
above (a) Limestone–Kaimur
Ans. (c) :The Union Territory of Andaman & Nicobar (b) Mica–Bhagalpur
Islands in India has highest number of wildlife (c) Quartzite–Madhubani
sanctuaries (94). There are 565 wildlife sanctuaries in (d) Lead-zinc–Gaya
India. Among States, Maharashtra has maximum (e) None of the above/More than one of the
number of wildlife sanctuaries (49) followed by above
Karnataka (35). Ans. (a) : The correct matches are as:
146. Which of the following rivers flowing in Bihar District (in Bihar) Mineral
is a north flowing river? Kaimur, Rohtas, Limestone
(a) Bagmati (b) Kamla Nawada Mica
(c) Kosi (d) Gandak
Lakhisarai, Monghyr Quartzite
(e) None of the above/More than one of the
above Banka & Rohtas Lead.

66th BPSC (Pre) Exam. 2020 300 YCT


66th Bihar Public Service Commission
Preliminary Re-Examination, 2020
GENERAL KNOWLEDGE & GENERAL SCIENCE
(Solved Paper with Detail Explanation)
1. In the following which in fundamental physical Therefore at -40oC, Celsius and Fahrenheit scale
quantity? coincides.
(a) Forces (b) Velocity 5. The power of lens is measured in–
(c) Electric current (d) Work (a) Watt (b) Ampere
(e) None of the above/More than one of the above (c) volt (d) dioptre
Ans. (c) : Fundamental physical quantities are those (e) None of the above/More than one of the above
basic physical quantities that do not depend on other
quantities. There are 7 fundamental quantities these are- Ans. (d) : Power of a lens is defined as the
Fundamental Quantities Unit measurement of bending strength of the lens. The power
Length Meter (m) of lens in measured in Dioptre (D).
Time Second (s) 6. To diagnose the medical conditions MRI
amount of substance mole (mole) technique is used. Which is not used in MRI?
Electric current ampere (A) (a) Magnetic filed (b) X-ray
Temperature Kelvin (k) (c) Radio wave (d) Gamma ray
Mass kilogram (kg) (e) None of the above/More than one of the above
Luminous intensity candela (cd) Ans. (b) : Magnetic Resonance imaging (MRI) is a non-
A fundamental Quantity can be measured but cannot be invasive imaging technology that produces three
defined. We define a fundamental quantity either by dimensional anatomical images. It is used for disease
specifying how it is measured or by stating how it is detection, diagnosis and treatment monitoring. It is based
calculated from other measurement. on sophisticated technology that excites and detects the
2. An example of longitudinal wave in– changes in the direction of rotational axis of protons found
(a) Radio wave (b) Sound wave in the living tissues which contains Water. MRI Technique
(c) X-ray (d) gamma ray generally uses, magnetic field, gamma rays and radio
(e) None of the above/More than one of the above waves. X-rays are not used in MRI.
Ans. (b) : Longitudinal waves are mechanical waves 7. When a sound wave goes from one medium to
consisting of a periodic disturbance or vibration that another, the quantity that remains unchanged
takes place in the same direction as the advancement of is-
the wave. Out of the given options sound waves are (a) frequency (b) amplitude
example of longitudinal wave.
(c) wavelength (d) speed
3. Dispersion of light is possible by–
(e) None of the above/More than one of the above
(a) prism (b) convex lens
(c) concave lens (d) simple mirror Ans. (a) : When sound waves travel from one medium
(e) None of the above/More than one of the above to another, the frequency never changes. If the wave
travel into the dense medium, the speed and wavelength
Ans. (a) : When white light is passed through a glass
decreases and amplitude increases.
prism it get divided into its spectrum of colors, (which
are violet, Indigo, Blue, Green, Yellow, Orange and 8. Light-year is a unit of-
Red) and this process is known as dispersion of light. (a) time (b) distance
Out of the given option prism is the correct answer to (c) speed (c) force
the question. (e) None of the above/More than one of the above
4. At which temperature, the Celsius and Ans. (b) : Light year is a unit of astronomical distances.
Fahrenheit scales coincides? It is equal to the distance travelled by light in one year.
(a) 00 (b) 100 (c) –400 (d) 400 It is equivalent to about 9.46 trillion kilometers.
(e) None of the above/More than one of the above 9. Which particle is free of charge?
C F − 32 (a) α-particle (b) Electron
Ans. (c) : If C = F then by formula → =
5 9 (c) Neutron (d) proton
C C − 32 (e) None of the above/More than one of the above
= ⇒ 9C = 5C − 160
or, 5 9 Ans. (c) : Out of the given option neutron is free of any
or,4C = −160 ⇒ C = −40 charge. Charges on the given particles are-

66th BPSC (Pre) Re-Exam. 2020 301 YCT


α-particle +2 16. The component(s) of acid rain is/are-
Electron -1 (a) HNO3 (b) H2SO4
Proton +1 (c) CO2 (d) Both (a) and (b)
Neutron 0 (e) None of the above/More than one of the above
10. Ohm-meter is unit of Ans. (d) : Acid rain results when Sulfur Dioxide (SO2)
(a) resistance (b) conductance and Nitrogen Oxides (NOx) are emitted into the
atmosphere and transported by wind and air currents.
(c) resistivity (d) charge
The SO2 and NOx reacts with water, oxygen and other
(e) None of the above/More than one of the above chemicals to form sulfuric acid and nitric acid. These
Ans. (c) : Ohm-meter is the SI unit of electrical then mix with water and other materials before falling
resistivity, It is represented by (Ω.M) to the ground. Sulfuric acid (H2SO4), Nitric Acid
11. The chemical formula of sapphire- (HNO3) and Carbonic acid (H2CO3) are the major
(a) Al2O3 (b) Al3O2 components of acid rain.
(c) N2O (d) NO2 17. The chemical formula for heavy water is-
(e) None of the above/More than one of the above (a) H2O (b) N2O
Ans. (a) : Sapphire is a variety of Corundum or (c) D2O (d) CuO
Aluminum oxide. Its chemical formula is Al2O3. (e) None of the above/More than one of the above
12. The quality of petrol is expressed by- Ans. (c) : Heavy water is basically Deuterium oxide
(a) cetane number (D2O). Deuterium is an isotope of hydrogen with a mass
double that of ordinary hydrogen.
(b) octane number
(c) butane number 18. The pH value of water is
(d) Reynolds number (a) 4 (b) 7
(e) None of the above/More than one of the above (c) 12 (d) 18
(e) None of the above/More than one of the above
Ans. (b) : The ignition quality of petrol is expressed by
octane number. It is a value used to rate the fuel's Ans. (b) : Pure water has a PH value of about 7. It is
resistance to knocking. Higher the octane number, the considered as neutral because it has neither acidic nor
more resistant the petrol is to knocking. basic properties.
13. Which of the following has higher melting 19. The Nobel Prize in Chemistry, 2019 was
point? awarded for the development of-
(a) LED (b) LCD
(a) Boron (b) Iron
(c) Lithium-ion batteries (d) MRI
(c) Silicon (d) Aluminium
(e) None of the above/More than one of the above
(e) None of the above/More than one of the above
Ans. (c) : The Nobel Prize in chemistry 2019 had been
Ans. (a) : Out of the given options Boron has higher
awarded jointly to John B. Goodenough, M. Stanley
melting point (2076oC ). Due to small atomic size boron
Whittingham and Akira Yoshino for the development of
forms strong covalent bonds with the neighboring atoms
lithium-ion batteries.
that is why it has high melting and boiling points.
Melting Point of Iron - 15380C 20. Natural rubber is a polymer of-
(a) Isoprene (b) Vinyl acetate
Melting Point of Silicon - 14100C
(c) Styrene (d) Propene
Melting point of Aluminium - 660.30C
(e) None of the above/More than one of the above
14. An example of antibiotic medicine's
Ans. (a) : Natural rubber is a polymer of an organic
(a) aspirin (b) paracetamol
compound, isoprene with minor impurities. Chemical
(c) chloroquine (d) penicillin formula of isoprene is C5H5.
(e) None of the above/More than one of the above
21. The chemical used to destroy fungi in water
Ans. (d) : Antibiotics are medicines that fight infections tank is-
caused by bacteria. They work by killing the bacteria or (a) Nitric acid (b) Zinc sulphate
by making it hard for the bacteria to grow or multiply. (c) Magnesium sulphate (d) Copper sulphate
Out of the given options penicillin is an antibiotic.
(e) None of the above/More than one of the above
Aspirin and Paracetamol are antipyretic while
chloroquine is antimalarial. Ans. (d) : Copper Sulphate (CuSO4) is a fungicide used
to destroy fungi in water tank. It is also used as
15. The major component of CNG is- algaecide root killer and herbicide in agricultural and
(a) CO2 (b) N2 non agricultural settings.
(c) H2 (d) CH4 22. Which of the following is not component of
(e) None of the above/More than one of the above chlorophyll?
Ans. (d) : Major component of Compressed Natural (a) Calcium (b) Carbon
Gas (CNG) is Methane. Chemical formula of methane (c) Magnesium (d) Hydrogen
is CH4. (e) None of the above/More than one of the above
66th BPSC (Pre) Re-Exam. 2020 302 YCT
Ans. (a):The chlorophyll consists of magnesium, Ans. (a): Lymphocytes are a type of white blood cells
nitrogen, carbon and hydrogen. Its molecular structure that are made in bone marrow and found in the blood
consists of a central magnesium atom surrounded by and lymph tissue. They are part of lymphatic system
porphyrin ring (contains Nitrogen) attached to it, that collect excess fluid from cells and tissues and
carbon hydrogen side chain (phytol chain). returns it to blood stream, helping in the easy circulation
23. Water in plants is transported by- of blood. Therefore out of the given option lymphocytes
(a) Xylem (b) Epidermis help in blood circulation.
(c) Phloem (d) Cambium 30. Angora wool is extracted from-
(e) None of the above/More than one of the above (a) Rabbit (b) Fox
Ans. (a) : Xylem is a fundamental tissue for (c) Goat (d) Sheep
transportation of water. The water from the soil is (e) None of the above/More than one of the above
absorbed by root hairs of the plant and through osmosis Ans. (a) : Angora wool is obtained from the fur of
the water reaches xylem through which it reaches to Angora rabbits. There is a variety of goat called Angora
leaves and other parts of the plants goat too but the wool produced from Angora goat is
24. During photosynthesis, green plants absorb- called mohair.
(a) Oxygen (b) Nitrogen 31. Ancestral village of Kamla Harris is situated in
(c) C (d) CO2 which district of Tamil Nadu?
(e) None of the above/More than one of the above (a) Dindigul (b) Madurai
(c) Tiruvarur (d) Theni
Ans. (d) : During photosynthesis green plants absorb
(e) None of the above/More than one of the above
carbon dioxide (CO2) from the air and in presence of
sunlight, this CO2 and water is converted into glucose Ans. (c) : The village Thulasendrapuram from where
and oxygen. the maternal grandparents of Kamala Harris belonged is
a remote village in the Tiruvarur district of Tamil Nadu.
25. Oxygen is absent in- Kamala Harris is first female Vice-president of the
(a) Glass (b) Soil United states of America.
(c) Cement (d) Kerosene
32. Where is Salim Ali Bird Sancturary located?
(e) None of the above/More than one of the above (a) Goa (b) Srinagar
Ans. (d) : Out of the given options kerosene does not (c) Odisha (d) Maharashtra
have oxygen. Its chemical formula is C12H26-C15H32 and (e) None of the above/More than one of the above
is a hydrocarbon while glass [which is based on silica
(SiO2)], soil and cement contain oxygen. Ans. (a) : Salim Ali Bird sanctuary is one of smallest
protected area of Goa. It is located on the island of
26. The source of the enzyme 'lipase' is- Chorao in the Mondovi river.
(a) Kidney (b) Pancreas
33. Which portal has been launched by the
(c) Liver (d) Heart ministry of Human Resource Development to
(e) None of the above/More than one of the above improve the education system in India?
Ans. (b) : Lipase enzyme is a naturally occurring (a) Ekta (b) Shagun
enzyme found in the stomach and pancreatic juice. Its (c) Shikasha (d) Gyan
function is to digest fats and Lipids, helping to maintain (e) None of the above/More than one of the above
correct gallbladder function. Ans. (b) : The word Shagun here is coined from two
27. BCG vaccine is used for the prevention of- different words-‘Shala’ for schools and ‘Gunvatta’
(a) smallpox (b) typhoid meaning quality. The Shagun portal was launched in
(c) tuberculosis (d) plague August 2019 as an overreaching initiative to improve
(e) None of the above/More than one of the above the school education system. The initiative involves
Ans. (c) : BCG, or Bacilli Callmette-Guerin is a vaccine creating a junction in the form of a platform for all
for tuberculosis (TB disease.) postals and websites of the department of school
education in the India.
28. Which hormone is produced in pancreas?
(a) Thyroxin (b) Insulin 34. Rupsi Airport is situated in which state of
India?
(c) Galanin (d) Gastrin
(a) Arunachal Pradesh (b) Assam
(e) None of the above/More than one of the above
(c) Goa (d) Port Blair
Ans. (b) : Pancreas is a large gland located behind the (e) None of the above/More than one of the above
stomach in human body. The main hormone produced by
pancreas is Insulin. However it also produce other hormones Ans. (b) : Rupsi Airport is situated in Kokrajhar district
such as glucagon. Insulin helps in maintaining glucose level in of Assam. It became operational again in 2021 It was
the body. closed in 1984.
29. Which among the following helps circulation of 35. Which State in India has enacted law to make
blood? wearing of facial masks compulsory?
(a) Lymphocytes (b) Monocytes (a) Rajasthan (b) Bihar
(c) Eritherocytes (d) Blood platelets (c) Delhi (d) Maharashtra
(e) None of the above/More than one of the above (e) None of the above/More than one of the above
66th BPSC (Pre) Re-Exam. 2020 303 YCT
Ans.(a): Rajasthan, by the Rajasthan Epidemic Disease Ans. (c):Addu Atoll is the southernmost atoll of the
(Amendment) Bill, 2020 made wearing face masks archipelago of Maldives. It is situated in Indian ocean.
mandatory at public places to prevent the spread of corona 42. India has 'Free Movement regime' with which
virus. Rajasthan is the first state to pass such a bill. country?
36. Beej Bacho Andolan (Save the seed was started (a) Myanmar (b) Nepal
in which State? (c) Bhutan (d) Bangladesh
(a) Bihar (b) Uttarakand (e) None of the above/More than one of the above
(c) Jharkhand (d) Uttar Pradesh Ans. (a) : The India-Myanmar border has a free
(e) None of the above/More than one of the above movement Regime which allows people living along the
Ans. (b) : Beej Bachao Andolan was started by Vijay border (specially tribes), to travel 16 Km across either
Jardhari and his follow farmers, from a village called side of border without visa restriction for up to 72
Jardhargaon in Tehri district of Uttarakhand in the year hours.
1986. Jardhari has dedicated his life to conserving 43. The first informal summit between India and
traditional seeds, which otherwise would have China was held at-
disappeared due to wide acceptability of hybrid seeds (a) Delhi (b) Wuhan
among farmers. (c) Fujian (d) Chennai
37. Presentation of cruelty to animal law listed in (e) None of the above/More than one of the above
which list of the Indian Constitution- Ans. (b) : On, 28th April 2018, Prime Minister of India,
(a) Union List (b) State List Shri Narendra Modi and the president of China Mr. Xi
(c) Concurrent List (d) Preamble Jinping held their first informal summit in Wuhan
(e) None of the above/More than one of the above China, to exchange views on overarching issues of
bilateral and global Importance.
Ans. (c) : Prevention of cruelty to animal is listed in
concurrent list, which means that a law on the subject 44. Who amongst the following is not a Member of
Regional Comprehensive Economic
matter of prevention of cruelty can be made by state as
Partnership?
well as Union Government.
(a) Australia (b) Japan
38. Khetri Mines situated in Rajasthan is famous for- (c) China (d) Pakistan
(a) copper (b) iron (e) None of the above/More than one of the above
(c) diamond (d) coal Ans. (d) : The Regional Comprehensive Economic
(e) None of the above/More than one of the above Partnership (RCEP) is a proposed agreement between
Ans. (a) : Khetri mines situated in Jhunjhunu district of the member states of the ASEAN and its free trade
Rajasthan is famous for its copper mines. The copper agreement partners. Negotiations started in Nov. 2012
mine at Khetri is under the control of Hindustan copper Following countries are its members-Brunei, Cambodia,
limited, a public sector undertaking under the Indonesia, Laos, Malaysia, Myanmar, the Philippines,
Government of India. Singapore, Thailand, Vietnam, China, Japan, South
39. Who is the Chairman of Competition Korea, Australia and New Zealand. Pakistan is not a
member of RCEP. India was one of its founding
Commission of India?
member, however in 2019 it withdrew from the
(a) Ajay Tyagi (b) P.K. Gupta partnership.
(c) Ashok Gupta (d) Ghanendra Kumar
45. India hold 2+2 Meeting with which country-
(e) None of the above/More than one of the above
(a) Japan (b) USA
Ans. (c) : The Competition Commission of India (CCI) (c) Australian (d) All of the above
was established by the Government of India with effect (e) None of the above/More than one of the above
from 14th October 2003. CCI consists of a chairman and
6 members appointed by the Government of India. The Ans. (d) : The 2+2 dialogue is held between the foreign
and defence ministers of two countries. It enables both
current chairperson of the CCI is Shri Ashok kumar
countries to understand each other's strategic
Gupta. sensitivities. India has a 2+2 format dialogue with
40. When is National Statistics Day celebrated in Russia, U.S.A., Australia and Japan.
India? 46. What is Track 1.5 Diplomacy?
(a) 1 may (b) 20 April (a) Government Diplomacy
(c) 29 June (d) 20 December (b) Non-Government Diplomacy
(e) None of the above/More than one of the above (c) Government Diplomacy and non-Government
Ans. (c) : The Government of India has designated 29th Diplomacy
June, birth anniversary of Prasanta Chandra (d) The United Nations Diplomacy
Mahalanobis, as National Statistics Day. (e) None of the above/More than one of the above
41. Addu Atoll are situated in which ocean? Ans. (c) : Track 1.5 diplomacy involves dialogues that
(a) Atlantic Ocean (b) Archtic Ocean include a mix of government officials, who participate
(c) Indian Ocean (d) Pacific Ocean in an unofficial capacity, and non-governmental experts,
(e) None of the above/More than one of the above all sitting around the same table.

66th BPSC (Pre) Re-Exam. 2020 304 YCT


47. Mekong Ganga Cooperation was launched in Ans. (c): First Integrated check post between Bihar
2000 at- (India) and Nepal is along the Raxaul-Birgunj border
(a) Laos (b) Thailand point. It was inaugurated in 2018.
(c) Cambodia (d) India 53. The Headquarters of International Boxing
(e) None of the above/More than one of the above Federation is in–
Ans. (a) :The Mekong-Ganga cooperation, that (a) New Mexico (b) New York
comprises 6 member countries viz. India, Thailand, (c) New Jersey (d) Ohio
Vietnam, Laos, Cambodia and Myanmar, was (e) None of the above/More than one of the above
established on 10th November 2000 at the capital of Ans. (c) : The Headquarters of International Boxing
Laos i.e. Vientiane. federation is situated in Springfield, New jersey of
48. Which country has Sister-state Relationship USA. IBF (International Boxing federation) was
with Ahmadabad? organized in 1983.
(a) USA (b) UK 54. Shane Watson has announced his retirement
(c) China (d) Japan from all forms of cricket he played for which of
(e) None of the above/More than one of the above the following teams?
Ans. (d) : India and Japan had signed a MOU for a (a) Kings XI Punjab
sister-state agreement for Gujarat and Hyogo prefecture. (b) Chennai Super kings
Along with it Ahmadabad-Kobe sister-city agreement (c) Mumbai Indians
was also signed by the two countries. It is notable that (d) Royal Challengers Bangalore
Kobe is a city in Hyogo prefecture of Japan . These (e) None of the above/More than one of the above
agreements were signed during G-20 summit held in
Osaka in June 2019. Ans. (e) : Shane Watson is a former Australian
cricketer. He announced his retirement from all forms
49. Who has become New Zealand first Minister of of cricket on 2nd November .2020. He had played with
Indian origin? Royal challengers Bangalore (2016) and Chennai Super
(a) Priyanca Radhakrishnan Kings (2018) in Indian Premier League (IPL). Hence
(b) Paramjit Parmar more than one of the given options are correct.
(c) Kanwaljit Bakshi However BPSC has considered option B as correct
(d) Lisa Singh answer to the question.
(e) None of the above/More than one of the above 55. India's first Dolphin Observatory is being built in-
Ans. (a) : In November 2020, Priyanca Radhakrishnan (a) Delhi (b) Mumbai
became New Zealand's first Indian origin minister
Currently She is minister for the community and (c) Bihar (d) Odisha
voluntary sector is New Zealand. She was born in (e) None of the above/More than one of the above
Chennai. Ans. (c) : India's first dolphin observatory was being
50. Who publishes the World Economic Outlook? built at the Vikramshila Gangetic Dolphin sanctuary in
(a) WTO (b) World Bank Bhagalpur District of Bihar. It is being developed on
(c) IMF (d) UNDP Sultanganj-Aguwani Ghat bridge over the Ganga river.
(e) None of the above/More than one of the above 56. Bihar's first Ramsar Site is located in-
Ans. (c) : The World Economic Outlook is a report by (a) Begusarai (b) Banka
the International Monetary Fund (IMF). It is a survey by
(c) Bhagalpur (d) Bhojpur
the IMF staff that presents analysis of global economic
development for the near and medium term. (e) None of the above/More than one of the above
51. Nepal's first Transnational Petrol pipeline Ans. (a) : Kabartal wetland of Begusarai district of
starts from which place is Bihar? Bihar is the first Ramsar site of the state. It is India's
(a) Motihari (b) Raxaul 39th Ramsar site.
(c) Jogbani (d) Patna 57. Who are the Champions of FIDE in ever
(e) None of the above/More than one of the above Online Chess Olympiad 2020.
Ans. (a) : Nepal's first transnational petroleum pipeline (a) India-Amarica (b) India-China
is Motihari-Amlekhganj pipeline. It is between (c) India-Canada (d) India-Russia
Amlekhganj oil Depot in Parasa of Nepal and Motihari
(Bihar) of India. This pipeline transports petroleum (e) None of the above/More than one of the above
from Indian Oil Corporation to Nepal Oil corporation. Ans. (d): India and Russia were the co-champions of
52. First Integrated Check post in Bihar was set up the first ever FIDE chess Olympiad 2020.
in- 58. Who in the Winner of the Italian Grand Prix, 2020?
(a) 2012 (b) 2014 (a) Lauise Hamilton (b) Pierre Gasly
(c) 2018 (d) 2020 (c) Bottas (d) Hamilton
(e) None of the above/More than one of the above (e) None of the above/More than one of the above
66th BPSC (Pre) Re-Exam. 2020 305 YCT
Ans. (b): The 2020 Italian Grand Prix was held On 4-6 65. Who started Public Distribution system in
September 2020 at Monza of Italy. This formula one Sultanate period?
race was won by Pierre Gasley of France. (a) Ala-ud-din Khalji
59. Who hosted the event called 'One More Thing'. (b) Sikandar Lodhi
(a) Google (b) Intel (c) Muhammad bin Tughlag
(c) Microsoft (d) Apple (d) Firoz Shah Tughlag
(e) None of the above/More than one of the above (e) None of the above/More than one of the above
Ans. (d) : One more thing was a virtual event hosted by Ans. (a) : Alauddin Khalji through his market control
Apple Inc. was an event where apple was expected to system & rationing system introduced concept of the
unveil a new line-up of Apple silicon-powered Mac public distribution system during the Sultanate period.
notebooks and computers. 66. Which famous war took place in 1565 AD?
60. Who is the author of 'I am No Messiah'. (a) Panipat I War (b) Panipat II War
(a) Anil Kapoor (b) Sonu Sood (c) Khanwa War (d) Talikota War
(c) Sonu Nigam (d) Sanjay Kapoor (e) None of the above/More than one of the above
(e) None of the above/More than one of the above Ans. (d) : The Battle of Talikota was fought between
Ans. (b) : 'I am No Messiah' is a memoir authored by Vijaya Nagar kingdom and Sultanates of Deccan on 26
Sonu Sood and Meena Iyer. January 1565. It was a watershed battle which
61. The first Human Fossil was found froms which ultimately destroyed the Vijaynagar empire.
River Valley of India? 67. Who promulgated Din-E-Ilahi?
(a) Ganga Valley (b) Yamuna Valley (a) Babur (b) Akbar
(c) Narmada Valley (d) Tapti Valley (c) Jahangir (d) Shah Jahan
(e) None of the above/More than one of the above (e) None of the above/More than one of the above
Ans. (c) : The first Human Fossil in India was found at Ans. (b) : Din-e-Illahi was started by Mughal Emperor
Hathnora Village in Narmada Valley, in 1982. It was Akbar in Late 16th century AD. It was an elite eclectic
discovered by geologist-Arun Sonakia. religious movement with very few followers.
62. The first Empire was established by which 68. 'Ashta Pradhan' was helping in Administration
ruler in India? of which ruler?
(a) Chandragupta Maurya (a) Babur (b) Akbar
(b) Ashoka (c) Aurangeb (d) Shivaji
(c) Kanishka (e) None of the above/More than one of the above
(d) Chandragupta II Vikramaditya Ans. (d) : Ashta Pradhan was administrative and
(e) None of the above/More than one of the above advisory council setup by Chhatrapati Shivaji Maharaj.
Ans. (a) : Chandragupta Maurya is considered to The Ashta Pradhan were 8 prominent and main officials
establish first Empire in India. He was able to bring of Maratha polity. They are-
almost whole India under his rule. He was founder of Peshwa, Amatya Sumant, Sacheev/Shurunavis, Waqia
Mauryan Empire. Navis, Senapati, Nyayadhish and Pandit Rao.
63. The oldest sculpture of Buddha is made in 69. Who was the first Governor General of India?
which style? (a) Lord Mayo (b) Lord Lytton
(a) Gandhara Style (b) Mathura Style (c) Lord Canning (d) Lord Dufferin
(c) Mauryan Style (d) Gupta Style (e) None of the above/More than one of the above
(e) None of the above/More than one of the above
Ans. (e) : By the Charter Act of 1833, the designation
Ans. (a) : The seated Buddha from Gandhar is one of of governor general of Bengal was changed to the
the oldest Sculpture of Buddha. It is considered to be Governor General of India, making Lord William
sculpted during 2nd or 3rd century A.D. Therefore by its Bentinck the first governor general of India.
style it can be concluded that oldest sculpture of
Buddha was made in Gandhar style. 70. Who was the founder of Empire in India?
(a) C. Sleeman (b) Lord Minto
64. In which inscription, Kalinga Vijaya is
mentioned? (c) Lord Clive (d) Lord Mayo
(a) Maski Inscription (e) None of the above/More than one of the above
(b) Rudradaman Inscription Ans. (c) : Robert clive is regarded as the founder of
(c) Junagarh Inscription British rule in India. He commanded the British Forces
(d) Hathigumpha Inscription of East India company at the Battle of Plassey that
(e) None of the above/More than one of the above decided the fate of British rule in India.
Ans. (e) : The Kalinga Vijay of Ashoka is mentioned in 71. Where from the Revolution of 1857 started first-
major Rock Edict XIII. It also mentions Ashoka's (a) Lucknow (b) Allahabad
Dhamma victory over Greek king Antiochus of Syria (c) Jhansi (d) Meerut
(Amtiyoko), Ptolemy of Egypt (Turamage) etc. (e) None of the above/More than one of the above
66th BPSC (Pre) Re-Exam. 2020 306 YCT
Ans.(d):The revolt of 1857 began on 10 May 1857 in (c) Neolithic culture (d) Chalcolithic culture
the form of a sepoy mutiny of the company's army in (e) None of the above/More than one of the above
the garrison town of Meerut. Soon it spread to Delhi, Ans. (c) : Chirand is an important archaeological site in
Agra, Kanpur and Lucknow. the Saran district of Bihar. It is 1st known site of
72. Who was the leader of Munda Revolution? Neolithic culture in India. However Chirand has also
(a) Sindhu (b) Birsa revealed evidences of Chalcolithic and Iron Age
settlements.
(c) Kanhu (d) Tilka Manjhi
(e) None of the above/More than one of the above 79. Baba Thakur Das established which society in
Patna?
Ans. (b) : Birsa Munda, a fearless young man from the (a) Servants of india society
Munda tribe, spearheaded the Munda rebellion (1874-
(b) Ramakrishna Mission Soeciety
1907) against the British in the areas now bordering
(c) Seva Samiti
Bengal, Bihar and Jharkhand. The Munda rebellion is
also known as Ulgulan. (d) Social Service League
(e) None of the above/More than one of the above
73. Who established the 'Arya Samaj'
Ans. (b) : Baba Thakur Das established the
(a) Ranade (b) Dayanand Ramakrishna mission society at Patna in 1906-07 with
(c) Dayananda Saraswati (d) Swami the aim of social upliftment of women, to help poor and
Vivekananda to eradicate untouchability and orthodoxy from society.
(e) None of the above/More than one of the above 80. Who was the first Governor of Bihar after
Independence?
Ans. (b) : Arya samaj was founded by Swami
(a) Jagannath Mishra
Dayanand Saraswati in 1857 at Mumbai to reestablish
the Vedas as revealed truth. Later on its headquarters (b) Lalit Narayan Mishra
was shifted to Lahore. (c) Jairamdas Daulatram
(d) Jaiprakash Narayan
74. Who was to receive first Bharat Ratna? (e) None of the above/More than one of the above
(a) Rajendra Prasad (b) Jawaharlal Nehru Ans. (c) : Shri Jairamdas Daulatram was first governor
(c) Lal Babadur Sastri (d) S. Radhakrishnan of Bihar after independence.~
(e) None of the above/More than one of the above 81. Niagara Falls in North America is located
Ans. (d): Bharat Ratna is India's highest civilian award between
in recognition of exceptional service/performance of (a) Lake Superior and Lake Michigan
highest order. It was instituted on 2nd January 1954. C. (b) Lake Michigan and Lake Huron
Rajagopalachari, S. Radhakrishnan and C.V. Raman (c) Lake Huron and Lake Ontario
were its first recipients in 1954. (d) Lake Ontario and Lake Erie
75. Which city of Bihar is aurrounded by five hills? (e) None of the above/More than one of the above
(a) Gaya (b) Patna Ans. (d) : Niagara falls is situated between two Great
(c) Bhagalpur (d) Bodhgaya lakes of Erie and Ontario at the border of USA and
(e) None of the above/More than one of the above Canada.
Ans. (e) : The city of Rajgir of Bihar is surrounded by 5 82. One of the following pairs is a mismatch find it.
hills viz. Ratnagiri, Swarngiri, Vaibhavgiri, Vipulgiri/ (a) Masai-Central Eastern Africa
vipulachal and Udaygiri. (b) Sakai-Malaysia
(c) Bedouin-Arabian Peninsula
76. The first President of India Rajendra Prasad
beionged to which State? (d) Kyrgyz-central Asia
(a) Bihar (b) Haryana (e) None of the above/More than one of the above
(c) Delhi (d) Uttar Pradesh Ans. (e) : None of the given pairs are correctly
matched. Maasai are ethnic group inhabiting Kenya and
(e) None of the above/More than one of the above
Tanzania (central Eastern Africa), Sakai are indigenous
Ans. (a) : Dr. Rajendra Prasad, the first president of ethnic group of Malay peninsula (Malaysia), Bedouin
India, was born at Ziradei in the Siwan district of Bihar are Arabian ethnic group and Kyrgyz are Turkic
on 3rd December 1884. speaking people of central Asia.
77. The 16th session of Bihari students conference 83. Which of the following is not a coalfield of
was held at Germany?
(a) Hazaribag (b) Patna (a) Ruhr Valley (b) Lorraine Basin
(c) Ara (d) Chhapra (c) Saar Basin (d) Aachen Basin
(e) None of the above/More than one of the above (e) None of the above/More than one of the above
Ans. (a) : The 16th session of Bihar Students Ans. (b) : Ruhr, Saar, and Aachen are important
Conference was held at Hazaribagh in 1921. coalfields of Germany, where as Lorrain basin is
78. The oldest culture from Chirand of Bihar is– Industrial region of France. Lorrain is a coal basin of
(a) Palaeolithic culture (b) Mesolithic culture France.

66th BPSC (Pre) Re-Exam. 2020 307 YCT


84. Name the city of Russia that is known as Ans. (a): Madhya Pradesh is Largest copper producing
'Venice of North'. state of India. It is accountable for nearly 51% of copper
(a) Vladivostok (b) St. Petersburg concentrates production of India. However in terms of
(c) Novosibirsk (d) Moscow reserves Madhya Pradesh stood 3rd among Indian states
(e) None of the above/More than one of the above Rajasthan has largest reserve of copper in India.
Ans. (b) : St. Petersburg is inhabited on 101 islands and 91. Pahalgam in Kashmir is located on the bank of
has about 350 bridges that criss-cross the city, and has river-
an intensive network of canals that's why it is often (a) Shyok (b) Jhelum
called as 'Venice of the North.' (c) Lidder (d) Chenab
85. Pyrenees Mountain Ranges in Europe separate (e) None of the above/More than one of the above
the countries:- Ans. (c) : Pahalgam is a city of Anantnag district of
(a) Spain and France Union Territory of Jammu & Kashmir. It is one of the
(b) Spain and Portugal five tehsil of Anantnag district. It is situated on the
(c) Francs and Germany banks of Lidder river.
(d) Germany and Switzerland 92. Dandakaranya region is situated in the State
(e) None of the above/More than one of the above of-
Ans. (a) : The Pyrenees Mountains form a natural (a) Odisha (b) Chhattisgarh
border between France and Spain. The mountain range (c) Andhra Pradesh (d) Telangana
extends in a North west -South east direction from the (e) None of the above/More than one of the above
Atlantic Ocean to the Mediterranean sea. Ans. (e) : Dandakaranya region includes parts of
86. Chandra and Bhaga rivers flow through the Chhattisgarh, Odisha, Telangana and Andhra Pradesh. It
region:- is a physical region of East-central India.
(a) Spiti (b) Ladakh 93. Which of the following is not a Cattle breed of
(c) Lahaul (d) Kargil Rajasthan-
(e) None of the above/More than one of the above (a) Tharparkar (b) Rathi
Ans. (c) : Chandra and Bhaga rivers meet at Tandi, in (c) Hallikar (d) Mewati
the Lahaul and spiti district of Himachal Pradesh, to (e) None of the above/More than one of the above
form Chandrabhaga river. Ans. (c): Hallikar breed of cattle belonged to
87. Rana Pratap Sagar Hydro-electrical Project on Karnataka. Hallikar cattle are usually grey or greyish
Chambal river is located in the state of– black in colour and have a pronounced hump. The name
(a) Madhya Pradesh (b) Rajasthan came from Hallikar people of Mysore belt traditionally
known for their cattle rearing. Tharparkar, Rati and
(c) Uttar Pradesh (d) Chhattisgarh
Mewati are cattle breeds of Rajasthan.
(e) None of the above/More than one of the above
94. How many districts of Bihar had less than 10
Ans. (b) : Rana Pratap sagar hydroelectric project on Lakhs population in 2011?
Chambal river is located at Rawatbhata of Chittorgarh (a) 2 (b) 3
District of Rajasthan.
(c) 4 (d) 5
88. Bharmour tribal region is located a- (e) None of the above/More than one of the above
(a) Sikkim (b) Himachal Pradesh
Ans. (b) : As per census 2011, Out of 38 district of
(c) Uttarakhand (d) Ladakh Bihar only 3 district have less than 10 lakhs population.
(e) None of the above/More than one of the above These districts are- Arwal (700843), Sheohar (656246)
Ans. (b) : Bharmour Tribal region is situated in and Sheikhpura (636342).
Chamba district of Himachal Pradesh. It is a historical 95. Which one of the following has the highest per
region which once was capital region of Chamba. Gaddi hectare yield in Bihar?
tribes are main ethnic group of this tribal region. (a) Rice (b) Wheat
89. Jawaharlal Nehru Port is located in the State (c) Makka (d) Barley
of– (e) None of the above/More than one of the above
(a) Goa (b) Gujarat Ans. (c) : Per hectare yield of given cereals in Bihar is
(c) Andhra Pradesh (d) Maharashtra as follows-
(e) None of the above/More than one of the above Rice - 938 kg/ha (2018)
Ans. (d) : Jawaharlal Nehru port or Nhava sheva port is Wheat - 2500-3000 kg/ha
India's largest container port. It is situated in Maize - 3975 kg/ha (2014-15)
Maharashtra (Raigad district). barley - less than 2000 kg/ha
90. Which state of India is the largest producer of Therefore out of the given option maize (makka) has the
copper? highest productivity (per hectare) in Bihar.
(a) Madhya Pradesh (b) Rajasthan 96. Leadng fish producer districts of Bihar are-
(c) Jharkhand (d) Chhattisgarh (a) Madhuband and East Champaran
(e) None of the above/More than one of the above (b) Madhubani and Darbhanga
66th BPSC (Pre) Re-Exam. 2020 308 YCT
(c) East Champaran and Darbhanga Ans. (c): First Muslim President of the Indian National
(d) East Champaran and Muzaffarpur Congress was Badaruddin Tyabji who presided INC in
(e) None of the above/More than one of the above 1887 at Madras and the second Muslim to Preside over
Ans. (b) : Highest fish producing districts of Bihar are INC was Rahimtulla Mohammad Sayani in 1896 at
Madhubani and Darbhanga. Calcutta. Nawab Sayed Muhammad Bahadur was 3rd
Muslim president of INC who presided over in 1913 at
97. The top ranking pulses in terms of acreage in
Karachi.
Bihar are-
(a) masoor and gram 102. The First Seassion of Constituent Assembly
(b) moong and gram was held on-
(c) moong and masoor (a) 9th December, 1946 (b) 15th August 1947
(d) masoor and khesari (c) 26th November (d) 26th January, 1946
(e) None of the above/More than one of the above (e) None of the above/More than one of the above
Ans. (c) : According to acreage top ranking pulses of Ans. (a) : The constituent Assembly met for the first
Bihar are- time on 9 December, 1946 in New Delhi, in the
Masoor (Lentil) - 159.7 Thousand hect. constitution Hall, which is now known as Central hall of
parliament house.
Mung beans - 103 Thousand hect.
Khesari - 73.8 Thousand hect 103. Electoral College for the election of the
Chikea (gram) - 61.3 Thousand hect President of India consists of–
* data- 2013-14 and 2011-12 (a) all the elected members of the Rajya Sabha
(b) all the elected members of the Lok Sabha
98. The leading sugarcane production district of
(c) All the members of the state legislative
Bihar in-
Assemblies
(a) West Champaran (b) Sitamarhi
(d) All the elected members of both the Houses
(c) Gopalganj (d) East Champaran of Parliament and elected members of the
(e) None of the above/More than one of the above state legislative Assemblies
Ans. (a) : West Champaran district of Bihar is largest (e) None of the above/More than one of the above
producer of sugarcane in the state. It accounted for Ans. (d) : The electoral college for the election of the
58.3% of total sugarcane production of the state during
president of India consists of-
2018-19.
(a) The elected members of both houses of parliament
99. The largest canal irrigated area in Bihar lies in and
the district of– (a) The elected members of the Legislative Assemblies
(a) West Champaran of the states including NCT of Delhi and UT of
(b) Aurangabad Puducherry.
(c) Rohtas The President of India is indirectly elected by the
(d) Bhojpur system of proportional representation single
(e) None of the above/More than one of the above transferable vote.
Ans. (c) : Rohtas district of Bihar has the highest area 104. What constitutes the
under Canal irrigation, As per 2017-18 data Rohtas (a) imposition, abolition and regulation of any
district has approximately 250 thousand hectare land Tax
under canal irrigation. (b) Regulation of the borrowing of money
100. Name the districts of Bihar which has the (c) Appropriation of money out of the
lowest rural female literacy rate in 2001 and consolidated fund of India
2011 respectively? (d) The declaring of any expenditure to be charge
(a) Kishanganj and Madhepura on the consolidated fund of India
(b) Supaul and Sahars (e) None of the above/More than one of the above
(c) Supaul and Madhepura Ans. (e) : Article 110 of the constitution of India
(d) Kisanganj and Saharsa defines the money bill. Following matters have been
(e) None of the above/More than one of the above included in the definition of a money Bill-
Ans. (d) : In 2001 lowest rural female literacy rate • The imposition, abolition, remission, alteration or
among districts of Bihar was in Kishanganj while in regulation of any Tax,
2011 Saharsa district has the lowest rural female • Regulation of the borrowing of money or giving of
literacy rate. any guarantee by the Government of India,
101. Who was the second Muslim President of the • The appropriation of money out of the consolidated
Indian National Congress? fund of India,
(a) Abul Kalam Azad • The declaring of any expenditure to be charge on the
(b) Zakir Hussain consolidated fund of India, or in rearing the amount of
(c) Rahimtulla Mohammad Sayani any such expenditure.
(d) Badruddin Tyabji • The receipt of money on account of the consolidated
(e) None of the above/More than one of the above fund of India etc.
66th BPSC (Pre) Re-Exam. 2020 309 YCT
105. Decision of Supreme Court in Indra Sawhney A B C D E
VS Union of India is related to which matter? (a) 3 4 2 1 5
(a) Reservation of other backward castes in (b) 5 3 2 1 4
government jobs (c) 2 3 5 4 1
(b) Doctrine of Basic Structure (d) 1 5 3 4 2
(e) None of the above/More than one of the
(c) Approval of reservation in promotion for above
Scheduled castes
(d) Extension of reservation in Scheduled castes. Ans. (e) : The correct Provisions with their respective
articles are as follows.
(e) None of the above/More than one of the above Provisions Article
Ans. (a) : The famous Indra Sawhney vs Union of India Formation of village panchayat – Articles 40
case is related to the reservation of other Backward
class in Government job. It is not related to the Uniform Civil code – Articles 44
promotions. The order of Supreme Court affected Separation of Judiciary from - Articles 50
article 15(4), and 16(4) of the Constitution of India. executive
106. For a party to be recognized as national party, Promotion of International peace and - Articles 51
how percentage of it should have get in evelopment
previous elections Agriculture and Animal Husbandry - Articles 48
(a) 10% of valid votes in four or more states 109. Which of the following Constitutional
(b) 4% of valid votes in four or more states Amendment Acts restricted the size of the
(c) 15% of valid votes in two states Council of Ministers to 15 Percent of the total
(d) 6% of valid votes in more than half of states. members of the Lok Sabha?
(a) 95th Constitutional Amendment Act, 2009
(e) None of the above/More than one of the above
(b) 93rd Constitutional Amendment Act, 2005
Ans. (e) : To be recognized as a national Party in India, (c) 91st Constitutional Amendment Act, 2003
a political party must fulfill any of the 3 condition listed (d) 90th Constitutional Amendment Act, 2002
below-
(e) None of the above/More than one of the above
• The party wins 2% of seats of Lok sabha from at least
3 different state. Ans. (c) : The constitution (Ninety-First Amendment)
Act, 2003 amended Articles 75 and Article 164 of the
• In the general election to Lok sabha or legislative constitution of India an inserted a new article 361B. The
Assembly, the party polls 6% of votes in any four or amendment also amended the 10th Schedule of the
more states and in addition it wins 4 Lok sabha seats. constitution of India. according to the amendment, total
Currently there are 8 recognized national parties in number of minister including the prime minister or chief
India. minister (as may be the case) should not exceed fifteen
107. What is the object of 'Cut Motion'? percent of the total number of the house of the people or
(a) To put check on the policies of the Legislative Assembly of the state
Government 110. Match List-I with List-II and select the correct
(b) To stop the functioning of ruling party answer using the codes given below the lists–
(c) To move a proposal to reduce expenditure in List-I List-II
the Budget proposal (Panchayati Raj (Year)
(d) Rejection of complete financial dealings of committees)
the Government A. C.H. Hanumantha 1. 1985
Rao
(e) None of the above/More than one of the above B. G.V.K. Rao 2. 1986
Ans. (c) : A cut motion is a special power vested in C. Ashok Mehta 3. 1984
members of the Lok Sabha to oppose a demand being D. L.M. Singhvi 4. 1957
discussed for specific allocation by the government in E. Balwant Rai Mehta 5. 1978
the finance bill as part of the Demand for Grants. If the A B C D E
motion is adopted, It amounts to a no-confidence vote. (a) 4 5 1 3 2
108. Match List-I with List-II and select the correct (b) 2 4 1 3 5
answer using the codes given below the lists: (c) 5 3 2 4 1
List-I List-II (d) 3 1 5 2 4
A. Formation of 1. Article-44 (e) None of the above/More than one of the above
Village Panchayat Ans. (d) : The correct match is as follows–
B. Uniform Civil 2. Article-48 Panchayate Raj Committees - Year
Code C.H. Hanumantha Rao - 1984
C. Separation 3. Article-50 G.V.K. Rao - 1985
Judiciary from
Executive Ashok Mehta - 1978 (Submitted
D. Promotion of 4. Article-51 report)
International peace L.M. Singhavi - 1986
E. Agriculture and 5. Article-40 Balwant Rai Mehta - 1957
Animal Husbandry Ashok mehta committee was constituted in 1977.
66th BPSC (Pre) Re-Exam. 2020 310 YCT
111. Which of the following is not a method to 115. Which one is correct in the ascending under of
calculate the Gross Domestic Product (GDP)? HDI 2019 ranking of the following countries?
(a) Product method (a) Brazil, India, South Africa, China
(b) Diminishing cost method (b) Brazil, China, South Africa, India
(c) Income method (c) South Africa, China, India, Brazil
(d) Expenditure method (d) India, China, South Africa, Brazil
(e) None of the above/More than one of the above (e) None of the above/More than one of the above
Ans. (b) : There are 3 methods to calculate G.D.P., Ans. (b) : Correct ascending order of HDI-2019 ranking
these are- Income Method, Expenditure method and of the given countries are-
Product or outcome method. Diminishing cost method Brazil 74th rank)- China (85th rank) - South Africa (114th
is not used to calculate GDP. However Diminishing rank)- India (131st rank).
Balance method is used to calculate depreciation.
116. When was gender budgeting initiated in India?
112. Which of the following offices of institutes
releases data of the national income in India? (a) Union Budget, 2005-06
(a) NSSO (b) Union Budget, 2006-07
(b) NITI Aayog (c) Union Budget, 2008-09
(c) CSO (d) Union Budget, 2004-05
(d) Prime Minister's Office (e) None of the above/More than one of the above
(e) None of the above/More than one of the above Ans. (a) : In the budget of 2005-06, India introduced
Ans. (c) : The National statistical office (NSO) [created gender budgeting Gender budgeting refers to gender
by merger of NSSO & CSO on 23rd may 2019] under based assessment of budgets, incorporating a gender
the ministry of Statistics and Programme perspective at all levels of the budgetary process and
Implementation, is responsible for releasing data of the restructuring revenues and expenditures in order to
National Income in India. promote gender equality.
Before the creation of NSO, these data were released by 117. Which one is not the true statement about the
CSO. [The commission has accepted CSO as the correct PM Cares Fund?
answer to the question] (a) Any person appointed a trustee shall act in a
113. Which one is not true about AYUSHMAN pro bono capacity
SAHAKAR Scheme? (b) The fund consists entirely of voluntary
(a) The Finance Ministry of India launched this contributions from individuals/organization.
scheme. (c) The primary objective of this is dealing with
(b) It is a scheme to assist cooperative in the any kind of emergency or distress situation
creation of health infrastructure and to provide relief to the affected
(c) It specifically covers establishment, (d) It does not get any budgetary support
modernization, expansion, repairs, renovation (e) None of the above/More than one of the above
of hospital and healthcare and education Ans. (e) : All the given statement regarding PM cares
infrastructure fund are correct. PM Cares fund is created by Prime
(d) It is formulated by the National Cooperative minister of India Shri Narendra Modi to undertake and
Development Corporation (NCDC). support relief or assistance of any kind of emergency or
(e) None of the above/More than one of the above distress situation to the affected. The fund consists
Ans. (a) : The Ayushman Sahakar scheme was entirely of voluntary contributions from individuals or
Launched by Ministry of Agriculture and Farmers organizations. It does not get any budgetary support,
Welfare, to assist cooperatives to play an important role and any person appointed as in trustee shall act in a pro
in creation of healthcare infrastructure in the country. It bono capacity.
is formulated by National cooperative Development Prime Minister is the ex-officio chairman of the fund
Corporation (NCDC). and Ministers of Defence, Minister of Home Affairs,
and Minister of Finance, of Government of India are ex-
114. In which budget the Commodity Transaction officio trustee of the fund.
Tax (CTI) was introduced in the Budget of
India? 118. Choose the correct sequence of the states of
India, according ascending order of rice
(a) 2013-14 (b) 2012-13
production in the year 2018-19.
(c) 2014-15 (d) 2017-18 (a) Punjab, Uttar Pradesh, Rajasthan, Haryana,
(e) None of the above/More than one of the above Madhya Pradesh
Ans. (a) : In the Budget of 2013-14, the Commodity (b) Uttar Pradesh, Punjab, Haryana, Madhya
Transaction Tax was introduced in the Budget of India. Pradesh, Rajasthan
The commodity Transaction Tax is a tax charged on (c) Rajasthan, Haryana, Madhya Pradesh,
exchange-traded non-agriculture commodity derivatives Punjab, Uttar Pradesh
in India. It is to be 0.01% of the price of the trade on (d) Punjab, Rajasthan, Haryana, Madhya
non-agricultural commodity future contracts, which is Pradesh, Uttar Pradesh
the same rate on equity futures. (e) None of the above/More than one of the above
66th BPSC (Pre) Re-Exam. 2020 311 YCT
Ans. (e) : As per economic survey of India (2020-21) 124. What was the power capacity availability in the
the largest producer of rice in India (in ascending order) Bihar State in he year 2019?
are- Punjab < Uttar Pradesh < West Bengal. (a) 4767 MW (b) 4867 MW
119. Lowering the cash reserve ratio, it will have the (c) 5767 MW (d) 5867 MW
following impact on the economy (e) None of the above/More than one of the above
i) Banks will have higher leverage to liquidity Ans. (a) : In the year 2018-19, available power capacity
ii) The economy may see increased investment of Bihar was 4767 MW, which increase by 27.4% to
6073 MW in 2019-20.
iii) Supply of currency in economy may broaden
iv) Real investment rate may decline. 125. For which food grain production, Bihar got the
Krishi Karman Award, 2017-18?
(a) i only (b) Both i and ii
(a) Maize (b) Rice
(c) i, ii, iii and iv (d) i, ii and iv
(c) Wheat (d) Pulse
(e) None of the above/More than one of the above (e) None of the above/More than one of the above
Ans. (c) : Lowering the cash reserve ratio will have the Ans. (c) : Bihar got Krishi Karman Award in 2017-18
following impact on the economy- for record production of wheat. The production of
It will provide higher leverage of liquidity, wheat in 2017-18 was 61.04 lakh Tonnes in 2017-18.
Supply of currency in economy may increase, Before that Bihar also got Krishi-Karman Award for
The economy may witness increase in investment, high production of wheat in 2011-12 and 2012-13 and
however real investment rate may decline for maize in 2015-16.
120. According to the Reserve Bank of India, what 126. In 1902, Lord Curzon appointed the University
was the total values of the Foreign Exchange Commission including two Indian members
Reserves of India in 2018-19? Who were they?
(a) 493560 million US dollar (a) Bal Gangadhar Tilak and Surendranath
(b) 4881078 million US dollar Banerjee
(c) 477807 million US dollar (b) Gopal Krishna Gokhale and Rashbehari Bose
(c) Syed Hussain Bilgrami and Surendranath
(d) 412871 million US dollar
Banerjee
(e) None of the above/More than one of the above (d) Syed Hussain Bilgrami and Justice Gurudas
Ans. (d) :According to RBI the total value of the Banerjee
Foreign Exchange Reserve of India, in 2018-19 was (e) None of the above/More than one of the above
412871 million US dollar. Ans. (d): Lord Curzon appointed a University
121. What is the share of Bihar in the divisible pool Commission on 27 January 1902 to enquire into the
of central Taxes on the recommendation of the conditions and prospect of universities in British India
15th Finance Commission? and to elevate the standard of university teaching and to
(a) 10.06% (b) 10.02% promote the advancement learning. The commission
(c) 13.02% (d) 15.03% was led by Thomas Raleigh. The commission also
(e) None of the above/More than one of the above included two Indians- Syed Hussain Bilgrami and
Justice Gurudas Banerjee.
Ans. (a) : As per the recommendations of 15th Finance
127. Kunwar Singh entered Jagdishpur on–
commission of India the share of Bihar in the divisible
pool of central Taxes is 10.06% (a) 22 April, 1858 (b) 23 April, 1858
(c) 20 April, 1858 (d) 25 April, 1858
122. What was the growth rate of Bihar economy at (e) None of the above/More than one of the above
constant prices in the year 2018-19?
Ans. (a) : Kunwar Singh led the rebellion of 1857 in
(a) 13.53% (b) 11.53%
Bihar. On 22-23 April 1858 he reentered Jagdishpur and
(c) 12.53% (d) 10.53% defeated the Britishers but due to battle injuries, he died
(e) None of the above/More than one of the above on 26th April 1858 in his palace in Jagdishpur.
Ans. (d) : In the year 2018-19 the growth rate of the 128. Poona Sarvajanik Sabha was founded in–
economy of Bihar at constant price was 10.53% and at (a) 1858 by Mahadav Govind Ranade
current price it was 15.01% (b) 1870 by Mahadev Govind Ranade
123. The contribution of agricultural and allied (c) 1870 by Pandit Ramabai Ranade
sectors in the Gross State Value Added (GSVA)
(d) 1870 by Surendranath Banerje
of Bihar was-in the financial year 2017-18.
(e) None of the above/More than one of the above
(a) 17 Percent (b) 18 Percent
(c) 19 Percent (d) 20 Percent Ans. (b) : Poona Sarvajanik Sabha was founded on 2nd
April 1870 by Mahadev Govind Ranade, Ganesh
(e) None of the above/More than one of the above
Yasudeao Joshi and S.H. Chiplunkar. It was a
Ans. (e) : The contribution of agricultural and allied Sociopolitical Organization during British India. Its
sectors in the Gross state value Added was 23% in objective was to work as a mediating body between the
2017-18, and that of Manufacturing and services sectors government and people of India and to popularize the
are 15% and 62% respectively. peasant's legal rights.
66th BPSC (Pre) Re-Exam. 2020 312 YCT
129. How many Muslim delegates participated in 133. Who founded Abhinav Bharat society in 1906
the Second Convention of the Indian National in London?
Congress? (a) Vinayak Damodar Savarkar
(a) 2 (b) 33 (b) Shyamiji Krishna Varma
(c) 30 (d) 41 (c) Sohan Singh
(e) None of the above/More than one of the above (d) P.N.Bapat
Ans. (b) : The 2nd Session of Indian National Congress (e) None of the above/More than one of the above
was held at Kolkata in 1886, It was presided over by Ans. (a) : Abhinav Bharat Society was founded by
Dada Bhai Naoroji. In that session 33 Muslim delegates Savarkar brothers i.e. Vinayak Damodar Savarkar and
Participated. Ganesh Damodar Savarkar in 1904 at London.
130. The Champaran Movement was basically 134. All Indian Azad Dasta at Bakri ka Tapu was
against: organized during the Quit India Movement by–
(a) Land revenue (a) Ram Manohar Lohia (b) Usha Mehta
(b) cash crops (c) Chitu Panday (d) Jayprakash Narayan
(c) agricultural policies of the British (e) None of the above/More than one of the above
Government Ans. (d) : Azad Dasta was organized by Jai Prakash
(d) Land grants Narayan while being underground during Quit India
(e) None of the above/More than one of the above Movement, from Tarai region of Nepal (Bakari ka
Tapu), to fight the tyranny of the British rule.
Ans. (b) : The Champaran movement of 1917, was
against the Tinkathia system of Indigo planters. So 135. The Bihar Herald was started in 1872 by–
basically it was against the cash crop. (a) Munshi Surajmal (b) Rai Roshan Lal
131. In 1904, the Golden League was established at (c) Babu Guru Prasad (d) Krishan Bhatt
Deoghar with the objective of boycotting the (e) None of the above/More than one of the above
British and advancing the Swadeshi Movement Ans. (c) : Bihar Herald was first english news paper of
in– Bihar. It was started in 1872 by Babu Guru Prasad Sen.
(a) Bengal (b) Avadh However the publication of the news paper was started
(c) Punjab (d) Bihar in 1875.
(e) None of the above/More than one of the above 136. The Muslim League started its annual
Ans. (d) : The Golden league was established at convention of regular basis from–
Devghar in 1904. Its main objective was to boycott the (a) 1916 AD (b) 1919 AD
British and foreign product and supporting the (c) 1924 AD (d) 1925 AD
Swadeshi movement in Bihar. (e) None of the above/More than one of the above
132. Which of the following was not objective of the Ans. (d) : All India Muslim league was established in
Indian National Congress as pointed out by 1906 at Dacca. Its first session was held in 1907.
W.C. Banerjee in 1885? However its regular annual sessions started since 1925.
(a) To promote contact and friendship among the 137. Interim Government under the leadership of
protectors of the interests of the country Raja Mahendra Pratap Singh was founded in
(b) To promote the feeling of nationalism among Kabul with the support of Germany during
fellow countrymen by removing the feeling of the–
casteism, communalism and regional biases (a) Gadar Movement
(c) To express the viewpoint on the necessary (b) Swadeshi Movement
social issues with due consent of the educated (c) Second World War
class
(d) Home Rule Movement
(d) To determine on which direction and on
which basis the work has to be done in the (e) None of the above/More than one of the above
coming years for the Indian public interest Ans. (a) : During the Ghadar Movement, Raja
(e) None of the above/More than one of the above Mahendra Pratap Singh founded the Interim
Government at Kabul with support of Germany, on 1st
Ans. (e) : In his presidential address W.C. Banerjee December 1915. He himself became the president and
declared following objectives of the Indian National Maulavi Barkatullah was made Prime Minister of the
Congress. Government in exile of free Hindustan.
To promote and consolidate the feeling of National
138. The Tebhaga Movement was started in 1946
unity from Bengal under the leadership of–
To formulate popular demands and present them (a) Muslim League
before the government.
(b) Kisan Sabha
To bring together leaders from deferent parts of the
country (c) Indian National Congress
To train and organize public opinion in the country (d) Trade Union
(e) None of the above/More than one of the above

66th BPSC (Pre) Re-Exam. 2020 313 YCT


Ans. (b) : Tebhaga Movement of 1946 was a 142. A started business with Rs. 48,000 B Joined
sharecroppers movement (peasants movement) against him after few months with an amount of Rs.
the owners of the lands demanding two third of the 36,000. If the profit at the end of the year was
produces from the land for themselves and one third for divided into the ratio 2:1, after how many
the land lords. Kisan Sabha provided leadership to the months B Joined the business?
movement. (a) 4 (b) 6
139. 'So Long as this third power, i.e., England is (c) 8 (d) 9
here, our communal differences is here, our (e) None of the above/More than one of the above
communal differences would keep on troubling
us. 'Who said this? Ans. (a) : Time invested by A in the business = 1 yrs =
12 months
(a) Dr. Rajendra Prasad
Let B joined the business after X months
(b) Maulana Abul Kalam Azad
(c) Jawaharlal Nehru then the time invested by B = (12 – x) months
(d) Vinayak Damodar Savarkar Profit = Investment × time period
(e) None of the above/More than one of the above According to question
Ans. (e) : The given statement i.e. "so long as this third 48000 × 12 2
=
power (England) is here, our communal differences is 36000 (12 − x ) 1
here.." was given by Mahatma Gandhi
[36000(12-X]×2 = 48000×12
140. The Congress Socialist Party was founded in
1934 by– 48000 × 6
=8
(a) Jawaharlal Nehru and Vinoba 12-x = 36000
(b) Jawaharlal Nehru and Jay Prakash Narayan 12 − x = 8 ⇒ x = 4months.
(c) Jayprakash Narayan and Acharya Narendra 143. The harmonic mean of the roots of the equation
Dev
(4 + 5)x 2 − (2 + 3)x + 6 + 3 3 = 0
(d) Ashok Mehta and Dr. Rajendra Prasad
(e) None of the above/More than one of the above (a) 4 (b) 2
Ans. (c) : In 1934, Jai Prakash Narayan, Acharya (c) 8 (d) 6
Narendra Dev and Ram Manohar Lohia founded the (e) None of the above/More than one of the above

( ) ( )
Congress Socialist Party. It was a socialist group within
the Congress party. the party became defunct in 1948. Ans. (d) : 4 + 5 x 2 − 2 + 3 x + 6 + 3 3 = 0
141. Father is aged 3 times more than his son, after  
8 years he should be two and half times of his −b  2 + 3 
sum of roots (α + β) = =
son's age. After further 7 years, how many
times would he be of his son's age?
a  4 + 5 
  ( )
39 33 c 6+3 3
(a) (b) product of roots (αβ) = =
15 15 a 4+ 5
47 47
(c) (d) 2αβ
33 28 Harmonic mean of roots =
α +β
(e) None of the above/More than one of the above
Ans. (e) : Ans. (e) : Let the age of son be x years

(6 + 3 3 )
then the age of the father = 3x years
4+ 5
Now According to question, after 8 years =
age of son = x+8 and age of father = (3x+8) 2+ 3
but as per question 4+ 5

(3x + 8) =
5
( x + 8) =
(
2× 6 + 3 3 )
2 2+ 3
6x + 16 = 5x + 40
=6
x = 24 year
144. If the sum of the first 11 terms of an arithmetic
after 8+ 7 years progression equals that of the first 19 terms,
age of son = 24+ 8+ 7 = 39 then sum of first 30 terms will be–
age of father = 24 × 3 +8+ 7 = 87 (a) 0 (b) –1
87 (c) 1 (d) 30
The ratio will be =
39 (e) None of the above/More than one of the above

66th BPSC (Pre) Re-Exam. 2020 314 YCT


Ans. (a) : According to Question Ans. (c) :
s11 = s19 Persons No. of units Total cost
n purchased
sn =  2a + ( n − 1) d  from
2
11 19 W B C
 2a + (11 − 1) d  =  2a + (19 − 1) d 
2 2  X 6 5 3 A
22a+110 d = 38a + 342d Y 7 9 5 1.5A
16a = 232d Z 8 11 7 1.75A
2a = –29d
Unit price n p r Y
2a + 29d = 0
amount paid by X = 6n + 5p + 3r = A––––(i)
30
s30 =  2a + ( 30 − 1) d  amount paid by Y = 7n + 9p + 5r = 1.5A–––(ii)
2 
amount paid by Z = 8n + 11p + 7r = 1.75A––(iii)
= 15[2a + 29 d] Subtracting equation (i) from (ii) we get
[value of 2a + 29d= 0] n + 4p + 2r = 0.5 A ––––– (iv)
s30 = 15 × 0 = 0 and by subtracting equation (ii) from equation (iii)
145. Suppose n is an integer such that the sum of the we get
digits of n is 2, and 1010 < n < 1011 . The number n + 2p + 2r = 0.25 A –––––(v)
of different values for n is– Now by subtracting equation (v) from equation (iv)
(a) 20 (b) 11 2p = 0.25 A
(c) 9 (d) 8 0.25A
or p =
(e) None of the above/More than one of the above 2
Ans. (b) : when the sum of digits is 2, then there are putting the value of p in equation (iv)
two options there are 2 ones, with one 1 fixed in the 0.25
n + 4× A + 2r = 0.5A
first position the question is now based on the 2
arrangement of 10 zeros and 1 one, so −n
10! or r =
no. of possibilities = = 10 ways 2
9!×1! by putting the value of p & r in equation (i)
and when 2 is the first digit no. of possibility = 1 0.25 n
Therefore total no. of possibilities = 10+1 = 11 6n + 5 × A − 3× = A
2 2
146. The number of units purchased by the persons 6n = 0.5A
X,Y and Z from three companies W,B and C Total amount paid by X for the unit from company
are given in the following table with total cost 0.5A
and unit price– W= × 100 = 50
A
Persons No. of units Total
147. A milkman makes 30 liters of water with 70
purchased cost
liters of milk. After selling one fourth of this
from
mixture, he adds water to replenish the
W B C Quantity that he has sold. What is the current
proportion of water to milk?
X 6 5 3 A
9 19
(a) (b)
Y 7 9 5 1.5A 21 21
Z 8 11 7 1.75A (c)
7
(d)
19
9 9
Unit price n p r Y
(e) None of the above/More than one of the above
What percent of total amount paid by X was Ans. (b) : quantity of water in the mixture = 30liters
paid for the units from company W?
quantity of milk in the mixture = 70 liters
(a) 43 (b) 45
ratio of water : milk = 3: 7
(c) 50 (d) 55
according to Question
(e) None of the above/More than one of the above

66th BPSC (Pre) Re-Exam. 2020 315 YCT


 3 Ans. (b) : given
 30 − 25 × 10  + 25 22.5 + 25 x3 = 256
=  
water
=
milk  7  52.5 x = 28/3
 70 − 25 × 
 10  According to Question
47.5 19 ∴ y = (log2x)3 – 6 log2 x + 12
= =
( )
3
52.5 21 = log 2 28 / 3 − 6 log 2 28 / 3 + 12
148. In lock and key factory, one machine produces 3
only locks at the rate of 100 locks per minute 8  8
=  log 2 2  − 6 × log 2 2 + 12
and needs to be cleaned for 5 minutes after  3  3
production of every 1000 locks. Another 3
machine produces only keys at the rate of 75 8
=   − 16 + 12
keys per minute and needs to be clearned for 3
10 minutes after production of every 1500 keys.
512
If both the machines start production at the = −4
same time, what is the minimum duration in 27
minutes required for produce 9000 pairs of 512 − 108 404
= =
locks and keys? 27 27
(a) 180 (b) 170
404
(c) 135 (d) 130 y=
27
(e) None of the above/More than one of the above
150. What will be the constant term in the
Ans. (b) : number of total locks manufactured by the 6
 1
machine in one minute = 100 expansion at  x +  = ?
time taken in cleaning the machine after making 1000  x
locks = 5 minutes (a) 20
Total time taken to make 9000 locks (b) 15
9000  9000  (c) 7
= + −1 × 5
100  1000  (d) 6
= 90 + 40 = 130 min (e) None of the above/More than one of the above
number of keys made by second machine in Ans. (a) :
minutes = 75 6
 1
time taken to clean the machine after 1500 keys = 10 x + x  = ?
 
minutes
Bionomial theorem-
time taken to make 9000 keys = r
1
=
9000  9000 
+ − 1 × 10 Tr+1 = 6 Cr .x 6− r  
75  1500  x
= 120 + (5) × 10 = 6 Cr x 6−r x −r
= 120 + 50 = 6
Cr x 6 − 2r
= 170 min
For constant term, power of x = 0
minimum time required to produce 9000 pairs of lock
6 – 2r = 0
and keys = 170 minutes
r=3
149. Let y = (log 2 x)3 − 6 log 2 x + 12, x is positive
constant term = 6 Cr .x 6 − 2r
number. Then the equation x 3 = 256, has-
= 6 C3 x 6 − 6
(a) no solution for x
(b) exactly one solution for x = 6 C3
(c) exactly three solutions for x 6 × 5 × 4 × 3! 6 × 5 × 4
(d) exactly two solutions for x = =
3!× 3! 3× 2
(e) None of the above/More than one of the above = 20
66th BPSC (Pre) Re-Exam. 2020 316 YCT
67th Bihar Public Service Commission
Preliminary (Cancelled) Examination, 2021-22
GENERAL KNOWLEDGE & GENERAL SCIENCE
(Solved Paper with Detail Explanation)
1. Which of the following is a parliamentary 4. The first 'Lokayukta' was established in which
Committee relating to delegation of power to of the following states?
make rules and regulation to the Executive? (a) Odisha
(a) Committee on Executive Legislation (b) Kerala
(b) Committee on Subordinate Legislation (c) Maharashtra
(c) Committee on Administrative Legislation (d) Punjab
(d) Committee on Delegated Legislation (e) None of the above/More than one of the
(e) None of the above/More than one of the above
above Ans. (c) : Maharashtra was the first state to introduce
Ans. (b) : Committee on Subordinate Legislation to the institution of Lokayukta through the Lokayukta and
scrutinize and report to the house whether the powers to Upa-Lokayuktas Act in 1971.Justice S.P.Kotwal was
make rules, regulations, bye-laws, schemes or other the first Lokayukta of Maharashtra .
statutory instruments conferred by the Constitution or 5. Which of the following posts was held by A. S.
delegated by Parliament have been properly exercised Anand in late 1990's?
within such conferment or delegation. (a) Comptroller and Auditor General
2. Article 300 of the Indian Constitution deals (b) Chief Justice of India
with (c) Chief Vigilance Commissioner
(a) suits and proceedings (d) Attorney General
(b) government contracts (e) None of the above/More than one of the
(c) Attorney General above
(d) restrictions on trade and commerce Ans. (b) : Justice Adarsh Sein Anand, was enrolled as
(e) None of the above/More than one of the an advocate on November 1964. In May 1975, he was
above appointed as an additional judge of the Jammu and
Ans. (a) : Article 300 in the Constitution is related with Kashmir High Court and was confirmed as a permanent
Suits and proceedings. Article 300 of constitution says judge in February 1976. Later on he become Chief
the Government of India may sue or be sued by the Justice of Jammu and Kashmir and Madras High Court.
name of the Union of India and the Government of a He served as the Chief Justice of India from October
State may sue or be sued by the name of the State and 10th October 1998 till 2001.
may, subject to any provisions which may be made by 6. Which of the following is not a part of the
Act of Parliament or of the Legislature of such State Eleventh schedule of the constitution?
enacted by virtue of powers conferred by this (a) Libraries
Constitution. (b) Fuel and Fodder
3. Judicial process in India is based on (c) Rural Sports
(a) the constitution (d) Technical Training
(b) the due process of routine law (e) None of the above/More than one of the
(c) conventions above
(d) the procedure established by law Ans. (c) : The 11th Schedule was added to the Indian
(e) None of the above/More than one of the constitution in 1992 by the 73rd Constitution
above Amendment Act. This schedule contains 29 subjects.
Ans. (d) : Judicial process in India is based on This schedule covers important subjects such as
procedure established by law. In India, a liberal Panchayat's powers, Minor forest produce, Safe water
for drinking, Khadi, village and cottage industries,
interpretation is continuously being made by the
Rural housing, Fuel and fodder, Rural electrification,
judiciary after 1978 and it has tried to make the term
including distribution of electricity etc. Rural sports are
'Procedure established by law' as synonymous with 'Due not covered under the Eleventh Schedule of the
process' when it comes to protecting individual rights. constitution.

67th BPSC (Pre) Cancelled Exam. 2021-22 317 YCT


7. How many (number) 'Demands for Grants' are 10. The 'split system' in the Indian administration
put before the Parliament as a part of the relates to
Annual Budget in pursuance to Article 113 of (a) Audit/Accounts
the constitution? (b) Union/States
(a) 98 (c) Policy/Implementation
(b) 96 (d) All India Services/Central Services
(c) 104 (e) None of the above/More than one of the
(d) 109
above
(e) None of the above/More than one of the
above Ans. (c) : The Functioning of secretariat in India has by
Ans. (e) : Demand for Grants is the form in which and large been based on principle of separation of
estimates of expenditure from the Consolidated Fund of policy from its implementation which is known as split
India, included in the annual financial statement and system.
required to be voted upon in the Lok Sabha, are 11. With reference of the Finance commission of
submitted in pursuance of Article 113 of the India, consider the following statements :
Constitution. Generally, one demand for grant is 1. The Finance Commission is a statutory
presented in respect of each ministry or department. body.
However, more than one demand may also be presented 2. The Finance Commission was set up under
for a ministry or Department depending on the nature of Article 280 of the Constitution.
expenditure. 3. The recommendation made by the Finance
8. In which year was the 'National Extension Commission are only advisory in nature.
Service' launched? 4. The first Finance Commission was set up
(a) 1953 in 1950.
(b) 1957 Which of the above statements are correct?
(c) 1960
(a) 1 and 4 only
(d) 1972
(b) 3 and 3 only
(e) None of the above/More than one of the
above (c) 2 and 3 only
(d) 2 and 4 only
Ans. (a) : The National Extension Service (NES) was
launched on 2nd October 1953. It was implemented in (e) None of the above/More than one of the
the areas which were not covered by community above
Development program (CDP), when compared to CDP, Ans. (c) : The Finance Commission is a constitutional
the NES was less intensive in character. body constituted by the President under article 280 of
Objectives of NES: the Constitution mainly to give its recommendations on
i. To change the outlook of village people; distribution of tax revenues between the Union and the
ii To make the people participate effectively in States and amongst the States themselves. The first
development programmes; finance commission of India was appointed in 1951, for
iii. To develop village leaders accepted by all; and the period 1952-57 by the President of India and was
iv. To increase the employment and production. Chaired by K.C.Niyogi.
9. The members of a State Legislative Council are The recommendation made by the finance commission
not chosen by which of the following categories are advisory nature only and therefore not binding upon
of persons? the government. It is upto the government to implement
(a) Members of the local bodies/Panchayats its recommendation on granting money to states.
(b) Teachers
(c) Graduates 12. With reference to the Bare Necessities Index
(d) Industrialists (BNI), consider the following statements :
(e) None of the above/More than one of the 1. The economic survey has come up with the
above Bare Necessities Index at rural and urban
Ans. (d) : The members of the Council are either levels.
nominated by the Governor of the state or are indirectly 2. The BNI has been created for all states for
elected from the following Categories of person. 2018 only.
One-third of the members of this House are elected by 3. The BNI is based on 26 indicators.
the Legislative Assembly. One-third are elected by the 4. The BNI consists of six dimensions.
member of local bodies like a municipality or other Which of the above statements are correct?
local authorities. One-twelfth of the members are (a) 1 and 3 only
elected by graduates. One-twelfth of the members are (b) 2 and 3 only
elected by teachers. About one-sixth of the members are
(c) 1 and 4 only
nominated by the Governor. The legislative Council
elects its Chairman, who plays the role of presiding (d) 2 and 4 only
officer and Deputy Chairman from amongst its (e) None of the above/More than one of the
members. above

67th BPSC (Pre) Cancelled Exam. 2021-22 318 YCT


Ans. (a) : Economic survey 2020-21 constructed Bare Ans. (e) : Under Digital Bihar Program, all students of
Necessities Index at the rural and urban and all India class VI and above will receive computer education and
level. The BNI summarises 26 indicators from five training from 2021-2022 onwards. The state will take
dimensions viz, water, sanitation, housing, micro- steps towards implementing the National Education
environment, and other facilities. The BNI has been policy 2020 and increasing the gross enrolment ratio in
created for all states in 2012 and 2018 using data from higher education to 50% by 2035.
two NSO rounds viz, 64th and 76th on Drinking water, 17. According to the 15th Finance Commission's
Sanitation, Hygiene and Housing condition etc. in India. recommendations, how much share will Bihar
13. Which type of banks is proposed to be receive in divisible pool of Central taxes from
established for agriculture and rural 2021-26?
development in Bihar? (a) 4.12 percent
(a) Krishi Vigyan Banks (b) 4.23 percent
(b) Krishi Yantra Banks (c) 4.89 percent
(c) Krishi Vikas Banks (d) 4.06 percent
(d) Krishi Utthaan Banks (e) None of the above/More than one of the
(e) None of the above/More than one of the above
above
Ans. (a) : The Finance Commission is a constitutional
Ans. (b) : As per the announcement of the Agriculture body formed by the President of India to give
Department, 328 Krishi Yantra Banks will be set up by suggestions on centre-state financial relations. The 15th
the government in 13 districts of the State. These Yantra Finance Commission has recommended 4.12 percent
banks will be set up in aspirational districts of Bihar like share to Bihar in divisible pool of central taxes from
Nawada, Katihar, Begusarai, Sheikhpura, Araria,
2021-26.
Khagaria, Purnia, Gaya, Aurangabad, Banka, Jamui,
Muzaffarpur, Sitamarhi. Yantra Bank will be set up 18. Who among the following is not a member of
through farmer groups and Agricultural machines will the 15th Finance Commission?
be given on rent by these groups to nearby farmers. The (a) Ashok Kumar Lahiri
government will give 80 percent subsidy for agricultural (b) Sudipto Mundle
machinery banks, the maximum limit of which has been (c) Ramesh Chand
fixed at Rs.8 lakh. (d) Ajay Narayan Jha
14. The value of Human Development Index of (e) None of the above/More than one of the
Bihar in 2019 was above
(a) 0.641 Ans. (b) : The Finance Commission is a
(b) 0.613 Constitutionally mandated body that is at the centre of
(c) 0.596 fiscal federalism, set up under Article 280 of the
(d) 0.574 Constitution. The Chairman of 15th finance commission
(e) None of the above/More than one of the was N. K. Singh and Members were Ajay Narayan Jha,
above Prof. Anoop Singh, Dr. Ashok Lahiri, Prof. Ramesh
Ans. (d) : The value of Human Development Index of Chand, (part-time) Arvind Mehta.
Bihar in 2019 was 0.574. 19. In the recent Union Budget, the FM has
15. The fiscal deficit in Bihar during 2021-22 is increased the Foreign Direct Investment (FDI)
estimated as limit in the insurance sector from the existing
(a) ` 22, 511 crores one to
(b) ` 27, 617 crores (a) 50%
(c) ` 20, 011 crores (b) 62%
(d) ` 21, 543 crores (c) 74%
(e) None of the above/More than one of the (d) 100%
above (e) None of the above/More than one of the
Ans. (a) : Revenue surplus in 2021-22 is estimated to above
be Rs 9,196 crore, which is 1.21% of the GSDP. In
Ans. (c) : After the recent decision of the Government
2020-21 (revised estimate), the state has estimated a
revenue deficit of Rs 5,187 crore (0.8% of GSDP). of India, the maximum permissible limit of FDI in the
Fiscal deficit in 2021-22 is estimated to be Rs 22,511 insurance sector has been set to 74%.
crore (2.97% of GSDP). 20. As per the Union Budget, 2021, the fiscal deficit
16. Under Digital Bihar Programme, which is estimated to be how much percent of the
students will receive computer education and GDP in 2021-22?
training from 2021-22 onwards? (a) 8.8%
(a) All students of Class V (b) 7.8%
(b) All students of Class VI (c) 7.6%
(c) All students of Class VII (d) 6.8%
(d) All students of Class VIII (e) None of the above/More than one of the
(e) None of the above/More than one of the above
above
67th BPSC (Pre) Cancelled Exam. 2021-22 319 YCT
Ans. (d) : Revenue deficit is targeted at 5.1% of GDP in 24. Match List-I with List -II :
2021-22, which is lower than the revised estimate of List-I List-II
7.5% in 2020-21 (3.3% in 2019-20). Fiscal deficit is a. Sarkaria 1. To Review the
targeted at 6.8% of GDP in 2021-22, down from the Commission Methodology for
revised estimate of 9.5% in 2020-21 (4.6% in 2019-20). Measurement of
The government aims to steadily reduce fiscal deficit to Poverty
4.5% of GDP by 2025-26. b. C. Rangarajan 2. Infrastructure
21. The best index of economic development is Committee Financing
provided by c. Parekh 3. Centre-State
(a) growth in national income at current prices Committee Relationship
(b) growth in per capita real income from year to d. Narasimham 4. Banking Sector
year Committee Reforms
(c) growth in savings ratio Choose the correct answer from the options
(d) improvement in balance of payments position given below-.
(e) None of the above/More than one of the a b c d
above (a) 3 1 2 4
Ans. (b) : Per capita Gross National Product (GNP) is (b) 2 1 3 4
the best index of development. It can be derived by (c) 4 3 2 1
dividing the GNP of a country with its population. (d) 1 2 4 3
Higher the level of per capita income, higher is the (e) None of the above/More than one of the
economic development. The World Bank, in its world above
development report 1998, classified the countries in the Ans. (a) : (1) Sarkaria Commission was set up in 1983
world on the basis of per capita GNP. to examine the centre-state relationship
22. The second phase of the Swachh Bharat (2) The Rangarajan committee raised the poverty line
Mission (Grameen) is to be implemented based on the average of monthly per capita expenditure
during which time period ? to Rs 972 in rural India and Rs 1,407 in urban India
(3) Deepak Parekh to head committee on financing
(a) 2020-21 to 2022-23
infrastructure.
(b) 2020-21 to 2023-24 (4) The Narasimham Committee had 9 members. This
(c) 2020-21 to 2024-25 Committee was appointed by the Government of India
(d) 2020-21 to 2025-26 in 1991 for Banking Sector reforms.
(e) None of the above/More than one of the 25. Consider the following states :
above 1. Punjab
Ans. (c) : The Union Cabinet, chaired by the Prime 2. Uttar Pradesh
Minister, Shri Narendra Modi has approved the Phase II 3. Andhra Pradesh
of the Swachh Bharat Mission (Grameen) [SBM (G)] 4. West Bengal
till 2024-25, which will focus on Open Defecation Free Choose the correct sequence of the above in
Plus (ODF Plus), which includes ODF sustainability ascending order of rice-producing States in
and Solid and Liquid Waste Management (SLWM). India.
23. Who among the following is not a part of the (a) West Bengal, Punjab, Andhra Pradesh, Uttar
National Development Council? Pradesh
(b) West Bengal, Uttar Pradesh, Andhra Pradesh,
(a) The Secretary of the NITI Aayog
Punjab
(b) The Secretary of the Planning and (c) Andhra Pradesh, Uttar Pradesh, West Bengal,
Programme Implementation Ministry Punjab
(c) The Vice Chairman of the NITI Aayog (d) Andhra Pradesh, Punjab, Uttar Pradesh, West
(d) The Chairman of the Finance Commission of Bengal
India (e) None of the above/More than one of the
(e) None of the above/More than one of the above
above Ans. (d) : As per the Economic Survey of India 2021-
Ans. (e) : National Development Council was set up on 22 the correct sequence of Rice producing states in
August 6, 1952, to strengthen and mobilize the effort ascending order are Andhra Pradesh < Punjab < Uttar
and resources of the nation in support of the Plan, to Pradesh < W. Bengal
promote common economic policies in all vital spheres, 26. Which of the following events happened first?
and to ensure the balanced and rapid development of all (a) Quit India Resolution
parts of the country. NDC comprises of Prime Minister (b) Arrival of Cripps Mission.
of India [chairman] and includes all Union Ministers, (c) Arrival of Lord Wavell as the Governor
Chief Ministers of all the States and Administrators of General
Union Territories and Members of the Planning (d) Arrival of the Cabinet Mission
Commission. (e) None of the above/More than one of the
above
67th BPSC (Pre) Cancelled Exam. 2021-22 320 YCT
Ans. (b): 1. The Quit India Movement, also known as 30. During Quit India Movement, who among the
the August Movement, was a movement launched at the following escaped from the Hazaribagh Jail?
Bombay session of the All India Congress Committee (a) Sachchidananda Sinha
by Mahatma Gandhi on 8 August 1942 (b) Jayaprakash Narayan
2. The Cripps Mission was a failed attempt in late (c) Jagjivan Ram
March 1942 by the British government to secure full (d) Rafi Ahmed Kidwai
Indian cooperation and support for their efforts in (e) None of the above/More than one of the
World War II. above
3. Lord Wavell was appointed the Viceroy of India in Ans. (b) : On 8 August 1942 at the All-India Congress
the year 1944 Committee session in Bombay, Mohandas Karamchand
4. The Cabinet Mission came to India in 1946 in order Gandhi launched the 'Quit India' movement. During this
to discuss the transfer of power from the British period, when the anti-British movement was raging
government to the Indian political leadership, with the throughout the country, Jayaprakash Narayan made a
aim of preserving India's unity and granting its successful escape bid from the Hazaribagh central jail in
independence. 1942.
27. Who was the president of Lahore Session of the 31. Who was the President of Gaya Session of the
Indian National Congress held in 1929 AD? Indian National Congress held in 1922 AD?
(a) Vallabhbhai Patel (a) Chittaranjan Das
(b) Motilal Nehru (b) Hakim Ajmal Khan
(c) Jawaharlal Nehru (c) Mahatma Gandhi
(d) Rajendra Prasad (d) Maulana Abul Kalam Azad
(e) None of the above/More than one of the
(e) None of the above/More than one of the
above
above
Ans. (c) : The 1929 Lahore session under the
presidency of Jawaharlal Nehru holds special Ans. (a) : Indian National Congress was founded on 28
significance as in this session "Purna Swaraj" was December 1885 and Chittaranjan Das was the President
declared as the goal of Congress. 26 January 1930 was of the Gaya Session of the Indian National Congress
declared as "Purna Swaraj Diwas". held in 1922.
28. Who is not related to the Champaran 32. What portfolio Rajendra Prasad held in the
Satyagraha of 1917 AD? Interim Government established in 1946 AD?
(a) J. B. Kripalani (a) Finance
(b) Raj Kumar Shukla (b) Defence
(c) Rajendra Prasad (c) Home
(d) Ram Manohar Lohia (d) Food and Agriculture
(e) None of the above/More than one of the (e) None of the above/More than one of the
above above
Ans. (d) : Gandhi ji arrived in Champaran, on 10 April Ans. (d) : On September 2, 1946, the interim
1917 and stayed at the house of Sant Raut in Amolwa government of India led by Pandit Jawaharlal Nehru
village with a team of eminent lawyers: Brajkishore
was formed. It was the only such cabinet in India’s
Prasad, Rajendra Prasad, Maulana Mazharul Haque,
history in which both Congress and the Muslim League
Anugrah Narayan Sinha, Babu Gaya Prasad Singh, Raj
Kumar Shukla, Ramnavmi Prasad, and others including shared power at the Centre. The Cabinet of the Interim
J. B. Kripalani. Government of India was composed of the following
29. One of the leaders of the Revolt of 1857 AD, members ministers.
Kunwar Singh was related to which place? 1. External Affairs and Commonwealth Relations:
(a) Gwalior Jawaharlal Nehru (INC)
(b) Jagdishpur 2. Home Affairs, Information and Broadcasting: Sardar
(c) Jhansi Vallabhbhai Patel (INC)
(d) Meerut 3. Agriculture and Food: Rajendra Prasad (INC)
(e) None of the above/More than one of the 4. Commerce: Ibrahim Ismail Chundrigar (ML)
above 5. Defence: Baldev Singh (INC)
Ans. (b) : Kunwar Singh was a leader and military 6. Finance: Liaquat Ali Khan (ML)
commander during the Indian Rebellion of 1857. He 7. Education and Arts: C Rajagopalachari (INC)
belonged to a family of the Ujjainiya clan of the Parmar 8. Health: Ghazanfar Ali Khan (ML)
Rajputs of Jagdispur, currently a part of Bhojpur 9. Labour: Jagjivan Ram (INC)
district, Bihar, India. Place of Revolt of Indian Leaders
10. Law: Jogendra Nath Mandal (ML)
and British Officials who suppressed the revolt
67th BPSC (Pre) Cancelled Exam. 2021-22 321 YCT
11. Railways and Communications, Post and Air: Abdur (b) Madan Mohan Malaviya
Rab Nishtar (ML) (c) B. R. Ambedkar
12. Works, Mines and Power: C H Bhabha (INC) (d) Mahatma Gandhi
(e) None of the above/More than one of the
33. Where was the Ghadar Party established?
(a) Germany above
(b) United States of America Ans. (c) : Three Round Table Conferences of 1930–
(c) France 1932 were a series of meetings organized by the British
(d) England Government, involving Indian political personalities to
(e) None of the above/More than one of the discuss constitutional reforms in India. B.R. Ambedkar
above attended all the three round table conferences.
Ans. (b) : The Ghadar Party was a political 37. Who established Indian Association in 1876
revolutionary organization which was originally known AD?
as the Pacific Coast Hindustan Association. The Ghadar (a) Surendranath Banerjee
party was founded by Indian migrants on July 15, 1913 (b) Chittaranjan Das
in the United States by Lala Har Dayal, Sant Baba (c) W. C. Banerjee
Wasakha Singh Dadehar, Baba Jawala Singh, Santokh (d) Aurobindo Ghosh
Singh, and Sohan Singh Bhakna. The party was multi- (e) None of the above/More than one of the
ethnic, but mostly Punjabi, with Sikh, Hindu, and above
Muslim members, but mostly Sikh leaders. Ans. (a) : The Indian Association was the first avowed
34. Who established the Servants of India Society nationalist organization founded in British India by
in 1950 AD? Surendranath Banerjee and Ananda Mohan Bose in
(a) Surendranath Banerjee 1876 with an aim to promote by every legitimate means
(b) Bipin Chandra Pal the political, intellectual and material advancement of
(c) Mahadev Govind Ranade Indian society.
(d) Gopal Krishna Gokhale 38. In which country was the honorific 'Netaji'
(e) None of the above/More than one of the applied to the name of Subhas Chandra Bose ?
above (a) India
Ans. (d) : Servants of India Society was founded on 12 (b) Italy
June 1905 by Gopal Krishna Gokhale accompanied by a (c) Germany
small group of educated Indians, as Natesh Appaji (d) Japan
Dravid, Gopal Krishna Deodhar, Surendra Nath (e) None of the above/More than one of the
Banerjee, and Anant Patwardhan who wanted to above
promote social and human development and overthrow Ans. (c) : The honorific title Netaji was first awarded to
the British rule in India . The main aim of 'The Servants Subhas Chandra Bose in Germany in early 1942-by the
of Indian Society' was to train national missionaries for Indian soldiers and the German and Indian officials in
service to India and promote the true interests of the the Special Bureau for India in Berlin.
Indian people by constitutional means alone. 39. Who is related to 'Chittagong Armoury Raid'?
35. Who has written the book, Unhappy India? (a) Rash Behari Bose
(a) Bal Gangadhar Tilak (b) Chandra Shekhar Azad
(b) Annie Besant (c) Batukeshwar Dutt
(c) Lala Lajpat Rai (d) Surya Sen
(d) A. O. Hume (e) None of the above/More than one of the
(e) None of the above/More than one of the above
above Ans. (d) : The Chittagong armoury raid, also known as
Ans. (c) : the Chittagong uprising, was an attempt by armed
Books Authors Indian independence fighters led by Surya Sen to raid
1. 'Unhappy India' Lala Lajpat Roy the Chittagong armoury of police and auxiliary forces in
2. 'Hind Swaraj’ Mahatma Gandhi British India's Bengal presidency. Surya Sen was
3. 'Why I am an Atheist’ Bhagat Singh arrested by the police in February 1933 as a result of a
4. 'Gitanjali’ Rabindranath Tagore tip from Netra Sen. In January 1934, he was tried and
5. 'The Golden Threshold’ Sarojini Naidu hanged
6. 'India Divided’ Rajendra Prasad 40. In Anandamath of Bankim Chandra
7. ‘The Discovery of India’ Jawaharlal Nehru Chatterjee, which revolt is mentioned?
8. 'Annihilation of Caste’ BR Ambedkar (a) Sanyasi
36. Who participated in all the three Round Table (b) Kuka
Conferences ? (c) Santhal
(a) Maulana Abul Kalam Azad (d) Neel (Indigo)
67th BPSC (Pre) Cancelled Exam. 2021-22 322 YCT
(e) None of the above/More than one of the 16 18 144
above Converting into Km/hr = × =
10 5 25
Ans. (a) : Ananda Math is a Bengali fiction, composed = 5.76 Km/hr
by Bankim Chandra Chattopadhyay and published in
43. A clock is set right at 8 a.m. The clock gains 10
1882. It is inspired by and set in the back ground of the
minutes in 24 hours. What will be the true time
Sanyasi Rebellion in the late eighteenth century and it is
when the clock indicates 1 p.m. on the following
viewed as one of the main text in the history of Bengali
day/
and Indian literature. (a) 12 noon
41. The perimeter of a rhombus is 52 m and its (b) 48 minutes past 12 noon
shorter diagonal is 10 m. The length of the (c) 1 p.m.
longer diagonal is (d) 2 p.m.
(a) 12 m (e) None of the above/More than one of the
(b) 18 m
above
(c) 10 m
(d) 24 m Ans. (b) :
(e) None of the above/More than one of the 8am  24hr
→ 8am + → 5hr
1pm = 29 hr
above 10
Ans. (d) : Perimeter = 52 24hr + hr → 24hr
60
Perimeter = 4a Showing time Real time
52 = 4a 1
a = 13 24 + hr → 24
6
d1 (Diagonal) = 10m , d2 = ?
145
→ 24
6
24
1hr → ×6
145
24 24 × 6 144
2
 d1   d 2 
2 So in 29 hr = × 6 × 29 = =
a =   +  145 5 5
2  2  4
2 2
28 ⇒ 28 hr + 48 min total Journey
d  d  5
132 = 52 +  2  ⇒ 169 = 25 +  2  So ⇒ 12:48 min
 2   2 
2 44. The value of the continued fraction
d 
169–25 =  2 
 2 
d
144 = 2 is
2 (a) 1 (b) – 1
d2 = 12×2 (c) 2 (d) 0
d2 = 24 (e) None of the above/More than one of the
42. In a 100 m race, A runs at 8 kmph. If A gives B above
a start of 4 m and still beats him by 15 seconds, 1
what is the speed of B? Ans. (d) : Let, x = 2 −
1
(a) 5 kmph 2−
1
(b) 5.76 kmph 2−
(c) 6 kmph ........
(d) 6.34 kmph 1
x = 2–
(e) None of the above/More than one of the x
above 2x − 1
Ans. (b) : x=
x
D 100 × 60S × 60S 2
= −1
A = Time = = = 45sec x 2x
S 8 × 1000m 2
x − 2x + 1 = 0
B Time = 45 + 15 = 60 sec
B = Distance = 96m (x − 1)2 = 0
96m x −1 = 0
B = Speed = = 1.6
60 sec ⇒ x =1

67th BPSC (Pre) Cancelled Exam. 2021-22 323 YCT


45. How many numbers between 200 and 600 are Ans. (c) :
divisible by 4, 5 and 6?
log 32 log25
(a) 4 (b) 5 log 32 = =
(c) 6 (d) 8
2
log 2 log 21/ 2
(e) None of the above/More than one of the 5log 2
above
1
= (∵ l og X n = n log X
Ans. (c) : Let the L.C.M of 4, 5, 6 ⇒ 60 log 2
According to the question number should be between 2
200 to 600 5
= = 10
So → 201 to 599 1
599 200 2

60 60 49. By selling 45 lemons for ` 40, a man loses 20%.
↓ ↓ How many should he sell for ` 24 to gain 20%
9 − 3=6 in the transaction?
46. If n is any positive integer, then (34n – 43n) is (a) 16 (b) 18
always divisible by (c) 20 (d) 22
(a) 7 (e) None of the above/More than one of the
(b) 17 above
(c) 112 Ans. (b) : S.P. of 45 Lemons = Rs – 40
(d) 145 40 8
(e) None of the above/More than one of the S.P of 1 Lemons = Rs = =
above 45 9
Ans. (b) : X = 34n – 43n , n∈2t CP of 1 Lemons =
100 8 10
× =
Smallest Positive integer = n = 1 ...(i) 80 9 9
X = 34×1 43×1 = 34 – 43 S.P of 1 Lemon when Profit is = 20%
= 81 – 64 = 17 10
n=2 ...(ii) = 20% of
X = 38 – 46 = 6561 – 4096 = 2465 9
= 17 × 145 120 10 4
So 17 is a common factor = × = Rs =
100 9 3
47. The difference between the squares of two 4
numbers is 256000 and the sum of the numbers 1 Lemon S.P. =
is 1000. The numbers are 3
(a) 600, 400 3
(b) 640, 360 So = × 24 = 18 Lemon
4
(c) 628, 372
50. A train 110 m long is travelling at a speed of 58
(d) 650, 350
(e) None of the above/More than one of the kmph. The time in which it will pass a passerby
above walking at 4 kmph in the same direction is
Ans. (c) : Let the number be, a and b 1
(a) 6 seconds (b) 7 seconds
According to the question, a2 – b2 = 256000 & a + b = 2
1000 1
a2 – b2 = (a + b) (a – b) = 256000 = 1000 (a – b) (c) 7 seconds (d) 8 seconds
3
a – b = 256
So → a + b = 1000 (e) None of the above/More than one of the
a – b = 256 above
2a = 1256 Ans. (c) : Length of Train = 110 m
1256 Train speed = 58 km/hr
a= = 628 ..........(i) Passerby speed = 4 km/hr
2 Relative speed = 58 – 4 = 54 km/hr
b = 1000 – 628 .............(ii)
= 372 5
54 × m/sec
48. The value of log 2 ( 32 ) is 18
= 15 m/sec
5 110 22 1
(a) (b) 5 time = = = 7 sec
2 15 3 3
1
(c) 10 (d) 51. Which among the following is an incorrect
10 option?
(e) None of the above/More than one of the (a) Atom is electrically neutral due to presence of
above protons and electrons inside the nucleus.
67th BPSC (Pre) Cancelled Exam. 2021-22 324 YCT
(b) Electrons are negatively charged and have Ans. (d) : The major components of LPG are propane
negligible mass. and butane. Some other hydrocarbons are also present
(c) Atomic number is the number of protons in but they are in small concentrations. The mixture of
an atom of an element. propane and butane is a flammable mixture. Due to
(d) Isotopes are useful as nuclear fuel, in medical higher percentage of butane is correct answer.
field, in carbon dating, etc.
55. Water passes from the soil into the roots by a
(e) None of the above/More than one of the
above physical process called
(a) Diffusion
Ans. (a) : Atom are electrically neutral because they
(b) Transpiration
have the same number of negatively charged electrons
and Positively charged protons. Proton is present in (c) Absorption
nucleus but electron revolves in orbit around nucleus. A (d) Osmosis
neutron is a neutral particle, carrying no charge which is (e) None of the above/More than one of the
present in the nucleus of the atom. above
52. Select the incorrect statement out of the Ans. (c) : Root hairs are single-celled extensions of
following. epidermal cells in the root. They grow between soil
(a) Cotton is suitable for use as clothing in particles and absorb water and minerals from the soil.
summer because it absorbs moisture. Water enters the root hair cells by osmosis. This
(b) Polycarbonate is used for making CDs. happens because the water in the soil has a higher water
(c) Acrylic is also called artificial silk as it is potential than the root hair cell cytoplasm.
prepared from cotton but has shine like silk. 56. Biodegradable wastes can usually be converted
(d) Teflon is used for coating non-stick into useful substance with the help of
kitchenware's. (a) Bacteria
(e) None of the above/More than one of the (b) Viruses
above (c) Nuclear proteins
Ans. (c) : Acrylic is a Synthetic fibre, commonly (d) Radioactive substances
known as synthetic wool because it mimics wool. (e) None of the above/More than one of the
Whereas, Rayon is called Artificial silk. It is obtained above
by chemical treatment of wood pulp.
Ans. (a) : Biodegradable wastes can usually be
53. Which of the following statements about
converted into useful substances with the help of
graphite and diamond is true?
(a) They have the same crystal structure. Bacteria. Biodegradable waste includes any organic
(b) They have the same hardness. matter in which waste can be broken down into carbon
(c) They have the same electrical conductivity. dioxide, water, methane or simple organic molecules by
(d) They can undergo the same chemical micro-organisms such as bacteria. Biodegradable waste
reaction. can be used for composting and are resource for heat,
(e) None of the above/More than one of the electricity and fuel by means of incineration or
above anaerobic digestion.
Ans. (d) : * Diamond and graphite both are the 57. In the nighttime, it is advised not to sleep under
allotrope of carbon. trees because
* Diamond has a crystalline structure but Graphite has (a) They liberate less amount of oxygen
a layered structure. (b) They liberate harmful gases at night
*Diamond is an insulator of electricity and Graphite is (c) They liberate carbon dioxide
a good conductor of electricity. (d) They liberate carbon monoxide
* Diamond is the hardest naturally occurring substance (e) None of the above/More than one of the
but graphite is soft and has a greasy touch. above
* Diamond and graphite are both made up of carbon and
Ans. (c) : Photosynthesis does not occur in plants
have same chemical properties as that of carbon and
during night time as the sunlight is absent At night
they both undergo same type of chemical reactions.
plants continue to respire releasing carbon dioxide. So
54. Which of the following is the major constituent
person feels suffocate in sleeping under tree. Thus it is
of the liquefied petroleum gas?
not advisable to sleep under tree at night. During night
(a) Methane
(b) Ethane the trees breathe in oxygen and release CO2. If one
(c) Propane sleeps under the trees, the amount of increased CO2 in
(d) Butane the air around will certainly affect the health. So it is
(e) None of the above/More than one of the inadvisable not to sleep under trees during night.
above 58. Insectivorous plants grow in soil deficient in

67th BPSC (Pre) Cancelled Exam. 2021-22 325 YCT


(a) water (b) magnesium (c) Ultrasonic waves
(c) nitrogen (d) calcium (d) Ultraviolet rays
(e) None of the above/More than one of the (e) None of the above/More than one of the
above above
Ans. (c) : Insectivorous plants trap insects and digest Ans. (c) : In Ultrasonography, or ultrasound, high-
them for nutrition. Insectivorous plants have frequency sound waves (Ultrasonic waves) are sent
modifications to their leaves to help them trap insects. through your abdomen by a device called a transducer.
They grow in nitrogen-deficient soil and obtain nitrogen The sound waves are recorded and changed into video
from different insects like drosera, nepenthes, dionaea, or photographic images of your baby.
etc. 63. The universal acceptor blood group is
(a) A
59. Stem of a plant helps in distributing food to all
(b) B
parts of the plant. It also helps in
(c) AB
(a) storing the food
(d) O
(b) shaping the plant (e) None of the above/More than one of the
(c) respiration above
(d) photosynthesis
Ans. (c) : Karl Landsteiner discovered the blood
(e) None of the above/More than one of the groups. There are 4 main blood groups (types of blood)
above – A, B, AB and O. Your blood group is determined by
Ans. (e) : Stems support buds and leaves and carry the genes you inherit from your parents. AB positive
water, minerals and food (photosynthates). The vascular blood type is known as the “universal recipient”
system inside the stem forms a continuous pathway because AB positive patients can receive red blood cells
from the root, through the stem, and finally to the leaves from all blood types. People with type O- blood are
and through this system water and food products move. called universal donors because their donated red blood
60. In transfusion, blood must be compatible not cells have no A, B or Rh antigens and can therefore be
only in blood type but also in safely given to people of any blood group.
(a) Rh factor 64. Pneumonia is an infection of
(b) the number of white cells (a) nerve
(c) the number of red cells (b) blood
(d) race of donor and recipient (c) skin
(e) None of the above/More than one of the (d) lungs
above (e) None of the above/More than one of the
above
Ans. (a) : In transfusion blood must be compatible not
only in blood type but also in Rh factor. Ans. (d) : Pneumonia is an infection that inflames air
sacs in one or both lungs (alveoli). The air sacs may fill
61. Pituitary gland is located in with fluid or pus, causing symptoms such as a cough,
(a) intestine fever, chills and trouble in breathing. It is life
(b) liver threatening to infants and old peoples.
(c) kidney
65. A person standing in front of a mirror finds his
(d) brain image larger than himself. This implies that the
(e) None of the above/More than one of the mirror is
above (a) concave
Ans. (d) : The pituitary gland is not larger than a pea, (b) plane
and is located at the base of the brain. The main (c) convex
function of your pituitary gland is to produce and (d) cylindrical with bulging side outwards
release several hormones that help, regulate several (e) None of the above/More than one of the
physiological processes which includes : above
1. Growth. Ans. (a) : Concave mirror gives magnified image
2. Metabolism whereas convex mirror gives diminished image and
3. Reproduction. plane mirror gives image of equal size.
4. Stress or trauma. 66. A fan produces a feeling of comfort during the
5. Water and sodium (salt) balance. hot weather because
6. Childbirth. (a) fan supplies cool air
7. Breastfeeding (b) our body radiates more heat in air
62. Which of the following rays/waves are used to (c) conductivity of air increases
know the growth of fetus in the womb? (d) our perspiration evaporates
(a) X-rays (e) None of the above/More than one of the
(b) Microwaves above
67th BPSC (Pre) Cancelled Exam. 2021-22 326 YCT
Ans. (d): The fast moving air increases the rate at Ans. (b) : The room is going to get warmer. A
which our bodies lose heat due to convection and refrigerator is a heat engine it extracts heat from low
evaporation. A fan produces a feeling of comfort temperature reservoir and transfer it to high temperature.
because our perspiration or sweating evaporates rapidly. If a refrigerator’s door is kept open, then room will
67. Twinkling of star in clear sky during nighttime become hot, because then refrigerator exhaust more heat
can be explained with into the room than earlier. In this way, temperature of the
(a) refraction of light room increases and room becomes hot.
(b) reflection of light 71. A piece of ice is floating in a beaker containing
(c) polarization of light water up to its brim. When whole of the ice
(d) interference of light melts
(e) None of the above/More than one of the (a) the water will spill on the floor
above (b) the level of water will come down in the
Ans. (a) : The twinkling of stars is due to atmospheric beaker
refraction of star-light. The Optical Density of Air (c) the level of water will first fall and then it will
varies continuously in the different layers of the spill out of the beaker
(d) the water level will not change
atmosphere. Due to the varying optical densities of air,
(e) None of the above/More than one of the
starlight is refracted multiple times through the
above
atmosphere before it reaches our eyes.
Ans. (d) : When a piece of ice is placed in a beaker
68. Hydrogen bomb is based on the principle of
containing water, some of its portion remains out of the
(a) controlled fusion reaction
water level. Since the volume of ice is greater than the
(b) uncontrolled fusion reaction
water hence after melting the volume of the ice will
(c) controlled fission reaction
decrease and the water level will remain the same.
(d) uncontrolled fission reaction
72. Which of the following is not correctly
(e) None of the above/More than one of the
matched?
above
(a) Voltmeter – Potential difference
Ans. (b) : Hydrogen bomb is based on the principle of (b) Ammeter – Electric current
uncontrolled fusion reaction. In hydrogen bomb (c) Potentiometer – Electromotive force
isotopes of hydrogen combine under extremely high (d) Galvanometer – Electric resistance
temperatures to form helium. (e) None of the above/More than one of the
69. Food gets cooked faster in a pressure cooker above
because Ans. (d) : Galvanometer is an instrument used for
(a) water starts boiling at low temperature due to measuring a small electrical current or a function of the
high pressure current by deflection of a moving coil.
(b) water starts boiling at high temperature due to 73. If a feather, a rubber ball and a wooden ball
high pressure are falling freely simultaneously from the same
(c) water starts boiling at low temperature due to height in vacuum, then
low pressure (a) the feather will reach at the ground first
(d) water starts boiling at high temperature due to (b) the rubber ball will reach at the ground first
low pressure (c) the wooden ball will reach at the ground first
(e) None of the above/More than one of the (d) all the three will reach at the ground together
above (e) None of the above/More than one of the
Ans. (b) : Principle utilized in pressure cooker is that above
boiling point of water increases with increase in Ans. (d) : All of the objects will fall with the same
pressure. Because of increase in boiling point heat speed, because there is no air resistance in the vacuum,
provided to cooker is utilised to increase its temperature and gravitational force acts same on all the objects.
rather than converting its state (latent heat) - water to Hence in the vacuum, every object will reach at the
steam. ground together.
70. If the doors of a refrigerator are left open for 74. When a soap film on the water is seen in the
few hours, then the room temperature will daytime, it shows beautiful colours. This
(a) decrease phenomenon is due to
(b) increase (a) diffraction of light
(c) remain the same (b) refraction of light
(d) decrease only in the area in the vicinity of the (c) polarization of light
refrigerator (d) interference of light
(e) None of the above/More than one of the (e) None of the above/More than one of the
above above
67th BPSC (Pre) Cancelled Exam. 2021-22 327 YCT
Ans. (d) : The beautiful colors we see on the soap film 79. The most important ore of aluminium is
is due to interference of light waves reflected from the (a) Bauxite
top and bottom surfaces of thin films. (b) Calamine
75. The mass number of a nucleus is (c) Calcite
(a) the sum of the numbers of neutrons and (d) Galena
protons (e) None of the above/More than one of the
(b) the total mass of neutrons and protons above
(c) always more than the atomic weight Ans. (a) : Bauxite is the most common aluminum ore.
(d) always less than its atomic Approximately 98% of primary aluminum production is
(e) None of the above/More than one of the based on bauxite.
above 80. Which among the following solutions has pH
value more than 7?
Ans. (a) : The mass number of an atom is its total
(a) Buffer solution
number of protons and neutrons. Atoms of different
(b) Basic solution
elements usually have different mass numbers , but they
(c) Acidic solution
can be the same. For example, the mass number of
(d) Neutral solution
argon atoms and calcium atoms can both be 40.
(e) None of the above/More than one of the
76. Which of the following is the main ingredient of above
cement?
Ans. (b) : pH is a measure of how acidic/basic water is.
(a) Limestone
The range goes from 0 - 14, with 7 being neutral. pHs of
(b) Silica clay
less than 7 indicate acidic, whereas a pH greater than 7
(c) Gypsum
indicates a base.
(d) Ash
(e) None of the above/More than one of the 81. On which date did India begin its two-year
above term as a Non-Permanent Member of the UN
Security Council?
Ans. (e) : Limestone is mainly calcium carbonate Ca
(a) 1 October, 2020
Co3, whereas line, the chief ingredient of cement is Cao.
(b) 1 November, 2020
Cement is most important construction material.
(c) 1 January, 2021
Its ingredient are as follows :
(d) 1 July, 2021
Lime- 60.65%
(e) None of the above/More than one of the
Magnesia 1–3%
above
Alkaline 0–1%
Silica 17–25% Ans. (c) : India will began its two-year tenure as a non-
Alumina 3–8% permanent member of the United Nations Security
Iron oxide 0.5–6% Council (UNSC) on January 1,2021. This is the eighth
Calcium Sulfate 0.1–0.5% time that the country has a seat in UNSC.
Sulfur Trioxide 0–1% 82. Which country exported Sputnik V COVID,
77. Isotopes of an element differ in vaccine to India?
(a) the number of protons (a) America
(b) the mass number (b) Russia
(c) the number of electrons (c) UK
(d) the atomic number (d) France
(e) None of the above/More than one of the (e) None of the above/More than one of the
above above
Ans. (b) : The atoms of different isotopes are atoms of Ans. (b) : In August 2020, Russia was the first country
the same chemical element, they differ in the number of to announce the development of a COVID-19 vaccine.
neutrons in the nucleus. Russia, exported sputnik to India.
78. The temperature point at which solid, liquid 83. Joe Biden has assumed office as the
and gaseous states may stay together is known (a) 46th President of America
as (b) 47th President of America
(a) Boiling point (c) 48th President of America
(b) Melting point (d) 49th President of America
(c) Freezing point (e) None of the above/More than one of the
(d) Triple point above
(e) None of the above/More than one of the Ans. (a) : Democratic candidate Joe Biden is the 46th
above and current President of the United States of America.
Ans. (d) : Triple point is that value of temperature and Kamala Harris is the first female Vice-President of
pressure at which all the three phases of matter coexist USA. She is also first African-American and Asian-
simultaneously. American Vice-President.

67th BPSC (Pre) Cancelled Exam. 2021-22 328 YCT


84. Who was the foreign Chief Guest in the (b) bringing IT revolution in Bihar
Republic Day parade in 2021? (c) facilitating e-administration in Bihar
(a) Boris Johnson (d) providing services to people stranded outside
(b) Vladimir Putin Bihar during COVID-19 lockdown
(c) Barack Obama (e) None of the above/More than one of the
(d) Sheikh Hasina above
(e) None of the above/More than one of the Ans. (d) : The Bihar government has won the Digital
above India Awards, instituted by the central government, for
Ans. (e) : For the 2021 Republic Day Parade UK Prime taking innovative steps in e-governance. Bihar has been
Minister Boris Johnson was invited to be the Chief awarded for its initiative to transfer financial assistance
guest. However later, on it was canceled due to rising directly to the accounts of beneficiaries during the
up of COVID-19 Cases in UK. COVID-19 pandemic.
85. Who is the Permanent Representative of India 90. How many seats are reserved for the SC
in the UN? candidates in the Bihar Legislative Assembly?
(a) Tarun Bajaj (a) 38
(b) T. S. Tirumurti (b) 40
(c) Ajay Seth (c) 44
(d) Harsh Vardhan Shringla (d) 46
(e) None of the above/More than one of the (e) None of the above/More than one of the
above above
Ans. (b) : When the question was asked, T.S. Tirumurti Ans. (a) : With the creation of a separate State of
was the permanent representative of India in the United Jharkhand, by an Act of Parliament titled as the Bihar
Nations. At present, Ruchirakamboj is the permanent Reorganisation Act, the strength of the Bihar
representative of India in the UN. Legislative Assembly was reduced from 324 to 243
86. According to 2011 Census, which district of members. Among 243 seats 38 are reserved for
Bihar has more women than men? Scheduled Castes and 2 are reserved for Scheduled
(a) Gopalganj
Tribes.
(b) Begusarai
(c) Patna 91. In which year was 50% reservation to women
(d) Siwan in Panchayat bodies provided in Bihar?
(e) None of the above/More than one of the (a) 2005
above (b) 2006
Ans. (a) : As per 2011 census Sex Ratio in Bihar is 918 (c) 2009
i.e. for each 1000 male, which is below national average (d) 2014
of 943. Gopalganj has registered the highest sex ratio (e) None of the above/More than one of the
with 1021 females against 1000 males in 2011. above
87. In which year was the first sitting of the Bihar Ans. (b) : Bihar is the first state to provide reservation
Legislative Council convened? to half the total number of seats to the women by
(a) 1911
enacting the “Bihar Panchayati Raj Act 2006”.
(b) 1913
(c) 1914 92. In which year was Bihar divided to create
(d) 1919 Jharkhand?
(e) None of the above/More than one of the (a) 2000
above (b) 2001
Ans. (b) : The first sitting of the Bihar legislative (c) 2004
Council was convened on 20th January, 1913 at (d) 2005
Bankipore. (e) None of the above/More than one of the
88. Rashmi Kumari from Bihar is a/an above
(a) international cricketer Ans. (a) : The state of Jharkhand was created on 15
(b) international carrom champion November 2000, by dividing Bihar. In the same year
(c) football player
state of Uttarakhand and Chhattisgarh were also created.
(d) chess player
(e) None of the above/More than one of the 93. The new ministry proposed in the Bihar
above Budget for the year 2021-22 focuses on
Ans. (b) : Rashmi Kumari is an International Carrom (a) child care
champion from Bihar, India, playing since 1992. She (b) women empowerment
has been the two times world champion. (c) infrastructure development
89. The 'Digital India Award, 2020' was given to (d) skill and entrepreneurship development
the Bihar Government for (e) None of the above/More than one of the
(a) establishing technical institutes in Bihar above
67th BPSC (Pre) Cancelled Exam. 2021-22 329 YCT
Ans. (c): Tar Kishore Prasad, Bihar Deputy chief (a) 2015
Minister and Finance Minister, presented a budget of (b) 2016
Rs. 2.37 Lakh Crore with prime focus on education, (c) 2017
health, infrastructure and social sectors. (d) 2018
94. How many departments are covered under the (e) None of the above/More than one of the
Human Development Sub-Mission is Bihar? above
(a) 6 Ans. (c) : In 2017, the Railway Budget was merged
(b) 7 with the Union Budget, ending a practice that began in
(c) 8 1924 under the British. A Niti Aayog commission
(d) 10 submitted a white paper recommendation to do away
(e) None of the above/More than one of the with the practice.
above 99. What is the purpose of the Citizenship
Ans. (b) : For the implementation of Good Governance (Amendment) Act, 2019?
program in mission mode 7 Sub-Mission have been (a) To remove Bangladeshi illegal immigrants
(b) To identify genuine Indian citizens
formed under Bihar Vikas Mission. Human
(c) To check border infiltration by foreigners
Development Sub-Mission is one of 7 Sub-Missions.
(d) To grant citizenship to persecuted minority
Seven departments are covered under Human
groups of Afghanistan, Bangladesh and
Development Sub-Mission.
Pakistan
95. From which district of Bihar was the Garib (e) None of the above/More than one of the
Kalyan Rojgar Abhiyan launched by the Prime above
Minister in 2020? Ans. (d) : The Citizenship (Amendment) Act, 2019
(a) Patna
seeks to amend the Citizenship Act, 1955.
(b) Banka
The CAA provides citizenship on the basis of
(c) Madhepura
religion to six communities (Hindus, Sikhs, Buddhists,
(d) Khagaria
Jains, Parsis and Christians) which are minority and
(e) None of the above/More than one of the
persecuted in Pakistan, Afghanistan and Bangladesh
above
and who entered India on or before 31st December,
Ans. (d) : Garib Kalyan Rojgar Abhiyaan' is a massive
2014.
rural public works scheme of Government of India to
empower and provide livelihood opportunities to the 100. The central vista project refers to the
returnee migrant workers and rural citizens. The (a) construction of an IT park in New Delhi
Abhiyaan was launched on 20th June, 2020 from (b) construction of a new parliament, the prime
Village – Telihar, Block- Beldaur of Khagaria District minister and vice president's residences along
of Bihar. with a common central secretariat
96. Covishield, the COVID vaccine of India which (c) construction of a commercial area in new
is approved by WHO, is manufactured by Delhi
(a) Serum Institute (d) construction of residences for all ministers
(b) Bharat Biotech (e) None of the above/More than one of the
(c) Panacea Biotec above
(d) Zydus Cadila Ans. (b) : The central vista project aims to strengthen
(e) None of the above/More than one of the governance infrastructure by building new facilities for
above India’s Parliament and an efficient and sustainable
Ans. (a) : Pune-based Serum Institute of India partnered central secretariat to house all the ministers of
with the Oxford University to manufacture their government of India.
Covishield vaccine in India
101. Which Union Territory formed after division of
97. What is the length of the Atal Tunnel, Rohtang,
Himachal Pradesh? Jammu and Kashmir state does not have a
(a) 8.02 km legislature of its own?
(b) 9.02 km (a) Jammu
(c) 10.02 km (b) Kashmir
(d) 11.02 km (c) Ladakh
(e) None of the above/More than one of the (d) Both Kashmir and Ladakh
above (e) None of the above/More than one of the
Ans. (b) : The 9.02 km long, strategically significant, above
Atal Tunnel, is built under the 'Rohtang Pass' on the Ans. (c) : The union territory of Jammu and Kashmir
Manali - Leh Highway under the challenging conditions has a unicameral legislature. The Jammu and Kashmir
of freezing temperatures in extremely difficult terrain. Legislative Council was formally abolished on 16
98. In which year was the Railway Budget merged October 2019.
with the General Budget in India? Ladakh does not have an elected legislative assembly.

67th BPSC (Pre) Cancelled Exam. 2021-22 330 YCT


102. Who is the Indian footballer who won the All 107. When the US evacuated its troops from
India Football Federation (AIFF) Men's Player Afghanistan, who was the last American soldier
of the Year Award for 2021? to leave Kabul?
(a) Suresh Singh Wangjam (a) General Frank McKenzie
(b) Sandesh Jhingan (b) David Brunnstrom
(c) Aniket Jadhav (c) Major General Chris Donahue
(d) Arindam Bhattacharya (d) Ross Wilson
(e) None of the above/More than one of the (e) None of the above/More than one of the
above above
Ans. (b) : Senior defender of Indian Football team Ans. (c) : US Army Major General Chris Donahue,
commander of the 82nd Airborne Division, became the
Sandesh Jhingan had won the AIFF Men's Player of the
last soldier to leave Kabul, Afghanistan on 31 August
year Award for 2021 2021
103. On which date was the Pradhanmantri Kisan 108. Which French company supplied Rafale fighter
Samman Nidhi Yojana started ? jets to India?
(a) 1 November, 2017 (a) LH Aviation
(b) 1 January, 2018 (b) Dassault Aviation
(c) 1 February, 2019 (c) Issoire Aviation
(d) 1 April, 2020 (d) Humbert Aviation
(e) None of the above/More than one of the (e) None of the above/More than one of the
above
above
Ans. (e) : The Government with a view to augment the Ans. (b) : French aerospace major Dassault Aviation is
income of the farm families implemented a Central the manufacturer of the Rafale jets while MBDA
Sector Scheme, namely, "Pradhan Mantri Kisan supplies the missile systems for the aircraft.
Samman Nidhi (PM-KISAN)". The Scheme came in
effect from 01.12. 2018. The scheme was officially 109. When did the clash between the soldiers of
launched on 24th February, 2019. India and china take place in the Galwan
Valley?
104. In which game was the Gold Medal for India
(a) December 2019
won in the Olympics, 2021?
(b) April 2019
(a) Gymnastics
(c) January 2020
(b) Javelin Throw
(d) June 2020
(c) Boxing
(e) None of the above/More than one of the
(d) Shooting
above
(e) None of the above/More than one of the
above Ans. (d) : On June 15 2020, Ladakh’s Galwan Valley
witnessed a violent clash between the Armies of India
Ans. (b) : Neeraj Chopra won India's first-ever athletics
and China.
gold medal in the javelin throw final in Tokyo Olympic
held in 2021. 110. In which country, the Pegasus spyware has
105. How many medals have been won by India in been developed?
the Tokyo Paralympics, 2021? (a) Israel
(a) 19 (b) Brazil
(b) 21 (c) Russia
(c) 22 (d) China
(d) 18 (e) None of the above/More than one of the
(e) None of the above/More than one of the above
above Ans. (a) : Pegasus is spyware developed by the Israeli
Ans. (a) : India recorded its best-ever campaign at the cyber-arms company, NSO Group that can be covertly
Tokyo Paralympics 2021 year with a total of 19 medals installed on mobile phones (and other devices) running
across five sports. The athletes returned home with five most versions of IOS and Android. Pegasus is perhaps
gold, eight silver and six bronze medals from the event. the most powerful spyware ever created.
106. On which date did India take over the 111. The Forward Bloc was founded by
chairmanship of the UN Security council? (a) Sardarsinhji Rana
(a) 1 January, 2021 (b) Subhas Chandra Bose
(b) 1 March, 2021 (c) Jayaprakash Narayan
(c) 1 April, 2021 (d) Lakshmi Sahgal
(d) 1 August, 2021 (e) None of the above/More than one of the
(e) None of the above/More than one of the above
above Ans. (b) : Subhash Chandra Bose resigned from the
Ans. (d) : India took over the chairmanship of UN Congress Presidentship in 1939 due to differences with
security council on 1 August 2021. M.K. Gandhi and organized the All India Forward Bloc
67th BPSC (Pre) Cancelled Exam. 2021-22 331 YCT
a faction within the Congress in Bengal. The purpose Ans. (b): Gautam Buddha attained enlightenment under
was to consolidate the political left and major support a pipal tree at Bodhgaya ( Bihar).
base in his home state Bengal. Buddha gave his first sermon in the village of Sarnath,
112. Who was the Finance Minister in the Interim near Varanasi in Uttar Pradesh. This event is known as
Government? Dharma Chakra Pravartana (turning of the wheel of
(a) Liaquat Ali Khan law). He died at the age of 80 in 483 BCE at
(b) Sardar Patel Kushinagar, Uttar Pradesh.
(c) M. A. Jinnah 116. Who established a branch of Anushilan Samiti
(d) Jawaharlal Nehru in Patna in 1913 AD?
(e) None of the above/More than one of the (a) Ramananda Sinha
above (b) Satish Jha
Ans. (a) : Interim Government was formed from the (c) Sachindra Nath Sanyal
Constituent Assembly which was elected in August (d) Bipin Jha
1946. Members of the Interim Government with their (e) None of the above/More than one of the
ministries are as follows : above
1. Vice President, Also in charge of External Affairs Ans. (c) : A branch of Anushilan Samiti was established
and Commonwealth Relations: Jawaharlal Nehru (INC) in Patna by Sachindranath Sanyal in 1913 and
2. Home Affairs, Information and Broadcasting: Sardar Bankimchandra Mitra of BN College was given
Vallabhbhai Patel (INC) responsibilities to lead the organization.
3. Agriculture and Food: Rajendra Prasad (INC) 117. Who had formed the Bihar Socialist Party in
4. Defence: Baldev Singh (INC) 1931 AD?
5. Finance: Liaquat Ali Khan (ML) (a) Phulan Chand Tiwari and Rajendra Prasad
6. Education and Arts: C Rajagopalachari (INC) (b) Phulan Prasad Varma and Jayaprakash
7. Health: Ghazanfar Ali Khan (ML) Narayan
8. Labour: Jagjivan Ram (INC) (c) Raj Kumar Shukla and Swami Agnivesh
113. Who took the lead of the Santhal Rebellion of (d) Swami Sahajananda and Swami Yogananda
(e) None of the above/More than one of the
1855 AD?
above
(a) Sidhu and Kanhu
(b) Budhu Bhagat and Teja Bhagat Ans. (b) : Bihar Socialist party was established in 1931
(c) Mulu Manek and Jodha Manek by Jayaprakash Narayan and Phulan Prasad Verma at
(d) Madari Pasi and Sahadev Patna.
(e) None of the above/More than one of the above 118. Where is Sher Shah's tomb located?
Ans. (a) : Santhals were the agricultural people settled (a) Sasaram
in Rajmahal Hills of Bihar. On 30th June 1855, two (b) Maner
(c) Sitamarhi
Santhal brothers Sidhu and Kanhu organised 10,000
(d) Pavapuri
Santhals and proclaimed a rebellion against the British.
(e) None of the above/More than one of the
114. Who invited Gandhiji to come to Champaran? above
(a) Raj Kumar Shukla
Ans. (a) : The tomb of Sher Shah Suri is in the Sasaram
(b) Rajendra Prasad
town of Bihar. The tomb was built by Sher Shah Suri, a
(c) Jayaprakash Narayan
Pathan from Bihar defeated the Mughal Empire and
(d) Krishna Sahay founded the Suri Empire in northern India. He died in
(e) None of the above/More than one of the above an accidental gunpowder explosion in the fort of
Ans. (a) : Mahatma Gandhi was invited to Champaran Kalinjar in 1545. This tomb is an example of Indo-
by Raj Kumar Shukla and Sant Raut. Champaran Islamic architecture, it was designed by the architect
Satyagrah of 1917 was the first Satyagrah Movement Mir Muhammad Aliwal Khan and built between 1540
lead by Mahatma Gandhi in India. The Satyagraha was and 1545.
against the exploitative Practices adopted by European 119. Which one of the following Harappan sites is in
Planters. Gujarat?
115. Who had attained enlightenment in Bodh (a) Lothal
Gaya? (b) Dabarkot
(a) Mahavira Swami (c) Kalibangan
(b) Gautam Buddha (d) Rakhigarhi
(c) Simandhar Swami (e) None of the above/More than one of the
(d) Parshvanath Swami above
(e) None of the above/More than one of the Ans. (a) : There were seven important sites in the Indus
above valley civilization:
67th BPSC (Pre) Cancelled Exam. 2021-22 332 YCT
1. Mohan Jodaro (Province of Sindh, Pakistan) Book Authors
2. Harappa (Punjab, Pakistan) Mudrarakshasah Vishakhdatta
3. Kalibangan (Hanumangarh, Rajasthan) Rajtarangini Kalhana
4. Lothal (Ahmedabad Gujarat) Kathasaritsagar - Somdeva
5. Chanhudaro (Sindh, Pakistan) Kamasutra - Vatsayana
6. Dholavira (Kutch, Gujarat) Indica - Megasthanese
Kiratarjuniya - Bharavi
7. Banawali (Fatehabad, Haryana) Lilawati - Bhaskara II
120. Who was the founder of the Chola Dynasty? Harshacharita, Kadambari,
(a) Vijayalaya Nagananda,Ratnavali - Harsha vardhan
(b) Karikala 123. The Qutub Minar was completed by
(c) Aditya I (a) Iltutmish
(d) Rajaraja I (b) Qutb-ud-din Aibak
(e) None of the above/More than one of the (c) Ulugh Khan
above (d) Raziya Sultana
(e) None of the above/More than one of the
Ans. (a) : Emergence of Cholas began with Vijayalaya above
(850–871 CE) conquering the Kaveri delta from
Ans. (a) : Qutub-ud-din Aibak, the first Muslim ruler of
Muttaraiyar. He built the city of Thanjavur and Delhi, commenced the construction of the Qutab Minar,
established the Chola kingdom in 850. Rajaraja I is the but could only finish the basement. His successor,
most celebrated of the Chola kings. He engaged in naval Iltutmish, added three more storeys, and in 1368, Firoz
expeditions and emerged victorious on the West Coast, Shah Tughlak carried out restoration work.
Sri Lanka and conquered the Maldives in the Indian 124. Who was the founder of the Vijayanagara
Ocean. Rajendra I defeated Mahipala 1 of Bengal. Empire?
Rajendra III was the last Chola king who was defeated (a) Deva Raya I
by Jatavarman Sundarapandya II (b) Krishnadeva Raya
(c) Harihara-Bukka
121. Who was the President of the First Buddhist
(d) Vira Narasimha Raya
Council? (e) None of the above/More than one of the
(a) Vasumitra above
(b) Mahakashyapa Ans. (c) : In 1336, Vijayanagar kingdom was
(c) Sangharaksha established by Harihara and Bukka, who were two
(d) Parshvaka brothers and served in the army of Muhammad-bin-
(e) None of the above/ Tughlaq. They broke away from the Delhi Sultanate and
than one of the above established an independent state in Karnataka and
Ans. (b) : 1. First Buddhist Council- 486 BC In established the capital city Vijayanagar on the banks of
Sattaparni guha (Cave) situated outside Rajgriha. King: river Tungabhadra in 1336. Niccolo de conti visited
Ajatasatru Presiding Priest: Venerable Maha Kasyapa Vijayanagar during times of Devaraya 1. Abdul Razzaq
with 500 monks. The First Buddhist Council collected from Persia visited during Devaraya 2. Domingo Paes
together and arranged the Buddhist Scriptures known as and Barbosa visited during Krishnadevraya Nuniz
the Pali Tipitaka visited during the times of achyuthdevaraya
2. Second Buddhist Council Venue: Vaishali. Year: 386 125. During the time of which Mughal Emperor did
BC King: Kalasoka (Shisunaga Dynasty). Presiding Sir Thomas Roe come to India?
Priest: Sabakami. (a) Babur (b) Akbar
3. Third Buddhist Council Venue: Pataliputra (today’s (c) Jahangir (d) Shah Jahan
Patna). Year: 250 BC. King: Ashoka (Maurya Dynasty). (e) None of the above/More than one of the
Presiding priest: Mogaliputta Tissa (Upagupta). above
4. Fourth Buddhist Council: Venue: Kundalavana, Ans. (c) : Sir Thomas Roe was an English ambassador
Kashmir. Year: 72 AD King: Kanishka Presiding and a member of the House of Commons, who lived in
Priest: Vasumitra; deputed by Asvaghosha. the court of Mughal Emperor Jahangir from 1615 to
122. Who wrote the book, Kiratarjuniya? 1619. His major goal was to provide safety for the
(a) Bhatti British East India Company's factory in Surat
(b) Shudraka 126. Ain-i-Akbari was written by
(c) Kalidasa (a) Abdul Qadir
(d) Bharavi (b) Akbar
(e) None of the above/More than one of the (c) Khwaja Nizamuddin
above (d) Abul Fazl
Ans. (d) : List of the Books and Authors in Ancient (e) None of the above/More than one of the
India above
67th BPSC (Pre) Cancelled Exam. 2021-22 333 YCT
Ans. (d): Abul Fazal was the author of Ain-i-Akabari, Ans. (b) : Annie Besant served as the first women
the third volume of the book "Ain-i-Akbar", Akbarnama President of the Indian National Congress in 1917.
consist of 3 volumes. Annie Besant joined the Congress in 1914. In 1916 she
127. The Ryotwari system was first implemented in along with Bal Gangadhar Tilak started the Home Rule
(a) Gujarat Movement. The Home Rule League demanded self
(b) Madras government to the Indians.
(c) Bombay 130. Who started the journal, Bahishkrita Bharat?
(d) Orissa
(a) Jyotiba Phule
(e) None of the above/More than one of the
above (b) Karsandas Mulji
(c) Bal Gangadhar Tilak
Ans. (b) : Before independence, there were three major
(d) Babasaheb Ambedkar
types of land tenure systems prevailing in the country:
1 . The zamindari system was introduced by Lord (e) None of the above/More than one of the
Cornwallis in 1793 through Permanent Settlement that above
fixed the land rights of the members in perpetuity Ans. (d) : On 3 April 1927, Ambedkar launched the
without any provision for fixed rent or occupancy right Marathi fortnightly ‘Bahishkrit Bharat’ his other
for actual cultivators. newspapers are Mooknayak (1920), Janata (1930) and
2. Ryotwari System, was devised by Captain Alexander Prabuddha Bharat (An awakenned India- 1956)
Read and Sir Thomas Munro at the end of the 18th 131. Pattiseema Project is associated with the
century and introduced by the latter when he was integration of which of the following rivers?
governor of Madras Presidency (1819–26). It was (a) Krishna and Kaveri
prevalent in most of southern India, first introduced in (b) Krishna and Godavari
Tamil Nadu (Then Madras). It was later extended to
(c) Godavari and Mahanadi
Maharashtra, Berar, East Punjab, Coorg and Assam
3. Mahalwari System was introduced in 1822 by Holt (d) Ganga and Brahmaputra
Mackenzie. Later the system was reformed during the (e) None of the above/More than one of the
period of William Bentic (1833). It was introduced in above
Central province, North-West frontier, Agra, Punjab, Ans. (b) : Pattiseema Lift Irrigation Project is a river
Gangetic valley, etc of British India. interlinking project which connects Godavari River to
128. The Uprising of 1857 was described as the Krishna River.
'First Indian War of Independence' by 132. Which of the following statements are
(a) V. D. Savarkar concerned with eskers and drumlins?
(b) Bal Gangadhar Tilak 1. Eskers are ridges of crudely bedded
(c) R. C. Majumdar gravels and sands.
(d) Dadabhai Naoroji 2. Drumlins are constituted mostly of boulder
(e) None of the above/More than one of the and clay.
above 3. Basket of eggs topography is the
Ans. (a) : There are some books about 1857 Revolt characteristic of terrains with eskers.
with its authors 4. While eskers are built by glacier streams,
1. Dr SN Sen- “Eighteen Fifty Seven drumlins result from glacier action.
2. John Kaye- “History of the Sepoy War in India Choose the correct answer from the
3. SB Chaudhary- “Civil Rebellion in the Indian options given below.
Mutinies 1857–59” (a) 1, 2 and 3
4. BC Majumdar- “The Sepoy Mutiny and the Revolt of (b) 1, 2 and 4
1857″ (c) 3 and 4 only
5. AT Embree- “1857 in India”
(d) 1 and 2 only
6. Eric Stokes- “The Peasant and the Raj”
(e) None of the above/More than one of the
7. HP Chattopadhyay - “The Sepoy Mutiny 1857″
above
8. P Joshi- “Rebellion 1857”
9. VD Savarkar- “The Indian War of Independence Ans. (b) : Drumlins and Eskers are both hilly glacial
1857″ land forms. However they differ in shape.
10. Ashok Mehta- “1857,The Great Revolt” Eskers are sand and gravel ridges formed by
129. The first woman President of the Indian glacial melt water channels on top of glaciers. While,
National Congress was Drumlins are elongated, oval-shaped or say teardrop-
(a) Sarojini Naidu hills of rock, sand, and gravel.
(b) Annie Besant 133. Combustion of underground coal occurs in the
(c) Kasturba Gandhi state of
(d) Aruna Asaf Ali (a) West Bengal
(e) None of the above/More than one of the (b) Bihar
above (c) Jharkhand
67th BPSC (Pre) Cancelled Exam. 2021-22 334 YCT
(d) Odisha 138. Brahmayoni Hill is located in which district of
(e) None of the above/More than one of the Bihar?
above (a) Gaya
Ans. (c) : Explanation. Jharkhand, one of India's largest (b) Rohtas
and most productive coal field is also home to some of (c) Nalanda
the longest-burning fires in the world. (d) Nawada
Coal-spontaneous combustion is a phenomenon in (e) None of the above/More than one of the
which coal catches fire without igniting. It is a above
complicated process involving a physical and chemical Ans. (a) : Brahmayoni is a sacred hill in Gaya in Bihar,
reaction, which is caused by heat energy releases from a
which is mentioned in Mahabharata and Buddhist
coal-oxygen reaction.
records. It is very close from the vishnupad Temple.
134. Which district of Bihar has mica deposits?
(a) Rohtas 139. Which agroclimatic region of Bihar has the
(b) Patna highest number of districts?
(c) Gaya (a) Northern East
(d) Saran (b) Northern West
(e) None of the above/More than one of the (c) Southern East
above (d) Southern West
Ans. (e) : The largest mica reserve in the country is in (e) None of the above/More than one of the
Andhra Pradesh (41 per cent) followed by Bihar, above
Jharkhand, Maharashtra, Odisha, Rajasthan and Ans. (b) : Northern West agro-climatic regions of Bihar
Telangana. Jharkhand and Bihar consisting of four has the highest number of district. It has 13 Districts. It
districts - Giridih and Koderma in Jharkhand, and Jamui includes West Champaran, East Champaran, Siwan,
and Nawada in Bihar - has rich reserves of mica. Saran, Sitamarhi, Sheohar, Muzaffarpur, Vaishali,
135. The confluence of the rivers Son and Ganga is Madhubani, Darbhanga, Samastipur, Gopalganj,
located in which district of Bihar? Begusarai.
(a) Bihar
(b) Patna 140. Geodesy is the science that deals with
(c) Bhojpur (a) dating of terrestrial rock
(d) Nalanda (b) measurement of dimension of the earth
(e) None of the above/More than one of the (c) measurement of elevation and depression of
above the earth
Ans. (b) : Patna the capital of Bihar state, is a historic (d) recording of the changes undergone by the
city located at the confluence of three rivers, the Ganga, crust
the Gandak, and the Son. The Ganga flows all along its (e) None of the above/More than one of the
northern boundary. above
136. The rivers Damodar, Koel and Subarnarekha Ans. (e) : Geodesy is the science of accurately
originate from which of the following? measuring and understanding the Earth's geometric
(a) Deccan Plateau shape, orientation in space, and gravity field.
(b) Central Highland 141. The theory of continental drift was developed
(c) Chota Nagpur Plateau by
(d) Meghalaya Plateau (a) J. J. Wilson
(e) None of the above/More than one of the
(b) A. Wegener
above
(c) Du Toit
Ans. (c) : The source of the Damodar river, Koel and
(d) H. Hess
Subarnarekha river is Chota Nagpur Plateau (Palamau
hills in Jharkhand) . (e) None of the above/More than one of the
above
137. Which is the first Ramsar Site in Bihar?
(a) Kaimur Range Ans. (b) : The theory of continental drift is most
(b) Kharagpur Lake associated with the scientist Alfred Wegener. In the
(c) Kanwar Lake early 20th century, Wegener published a paper
(d) Rajgir Hill explaining his theory that the continental landmasses
(e) None of the above/More than one of the were “drifting” across the Earth, sometimes plowing
above through oceans and into each other.
Ans. (e) : Kabartal in Bihar’s Begusarai district has 142. Sunda Trench lies parallel to the island of
been recognised as a wetland of international (a) Java (b) Maldives
importance - the first such wetland in the state - under (c) Sumatra (d) Mauritius
the Ramsar Convention, according to the Union (e) None of the above/More than one of the
Environment Ministry. above
67th BPSC (Pre) Cancelled Exam. 2021-22 335 YCT
Ans. (c): The Sunda Trench, earlier known as the Java Ans. (a): Rivers in India are divided into two
Trench, is an oceanic trench located in the Indian Ocean categories: Himalayan Rivers and Peninsular Rivers.
near Sumatra. It lies parallel to the Island to Sumatra. Himalayan rivers are perennial and flow throughout the
year. A large number of peninsular rivers are seasonal
143. An effective Coriolis force results from and flow during a certain period in a year. Himalayan
(a) solar system rivers receive water from rainfall and melting snow of
(b) earth rotation the mountains and glaciers while Peninsular river
(c) interior of the earth receive water from rainfall only. The slopes of
(d) Colorado and Gulf Streams Himalayan rivers is very steep in upper reaches crossing
and transporting huge Sediments on other had
(e) None of the above/More than one of the peninsular rivers flow on already eroded old topography
above and also hard rock's of the region are not susceptible to
Ans. (b) : Coriolis Force is an artifact of the earth's erosion. Thus only statement 1 & 2 are correct.
rotation. Once air has been set in motion by the pressure 147. Gangetic plain has been described as a
gradient force, it undergoes an apparent deflection from (a) Pediplain
its path, as seen by an observer on the earth. This (b) Peneplain
(c) Geosyncline
apparent deflection is called the "Coriolis force" and is (d) Karst plain
a result of the earth's rotation. (e) None of the above/More than one of the
144. Which of the following countries does not have above
a land border with the Dead Sea? Ans. (c) : Ganga Plain can be best described as a
(a) Lebanon (b) Jordan Geosyncline. Peneplain means a more or less level land
(c) Israel (d) Palestine surface produced by erosion over a long period.
Pediplain is an extensive plain formed in a desert by the
(e) None of the above/More than one of the coalescence of neighboring pediments. Karst plain is an
above area made of limestone.
Ans. (a) : Dead sea eastern shore is in Jordan, and the 148. Which of the following is a freshwater lake?
western shore is in Israel. However, the western shore's (a) Chilika
southern half belongs to Israel, while the shore's (b) Sambhar
northern half is in West Bank, (Palestine) an area (c) Wular
(d) Loktak
claimed by both Israel and Palestine. However, (e) None of the above/More than one of the
Lebanon do not share its boundary with Dead Sea. above
145. Ilmenite, which is widely distributed along the Ans. (c) : Wular Lake is the 2nd largest fresh-water
Indian coastline, is a mineral of lake of Asia, situated on the foothills of Haramuk
(a) tungsten (b) titanium Mountain. It is spread in a total area of 200 square km
(c) gallium (d) tin covering almost 24 km in length while its breadth is 10
(e) None of the above/More than one of the km.
above 149. Which of the following planets has the least
Ans. (b) : Ilmenite (FeOTiO3) is an important titanium density?
mineral. It is widely distributed along the Indian (a) Earth
coastline. As per department of Atomic Energy (b) Mars
presently India has reserves of 348.22 million tons at (c) Venus
Ilmenite. (d) Saturn
(e) None of the above/More than one of the
146. Compare the Himalayan river with the above
Peninsular river based on the following Ans. (d) : The eight planets of our Solar System vary
comparisons : widely, not only in terms of size but also in terms of
1. Most of the Himalayan rivers are mass and density. Earth has the highest density (5.5
Perennial, whereas most of the peninsular gm/cm3) of any planet in the Solar System. while Saturn
rivers are rain fed. has least density.
2. The gradient of the Himalayan river is 150. What is the most famous feature of Chiria
located in West Singhbhum?
steeper than the Peninsular river. (a) Iron Ore Mining
3. The Peninsular river causes more erosion (b) Dam
on its way in comparison to the Himalayan (c) Bird Sanctuary
river. (d) National Park
Choose the correct answer from the (e) None of the above/More than one of the
options given below. above
(a) 1 and 2 only Ans. (a) : Chiria located in West Singhbhum district in
(b) 2 and 3 only state of Jharkhand is famous for Iron Ore Mining.
(c) 1 and 3 only Chiria is a census town situated in Pashchimi
(d) 1, 2 and 3 Singhbhum district in the state of Jharkhand, India.
(e) None of the above/More than one of the India's largest iron ore mine with a reserve of 2 billion
above tonnes of iron ore is started there.
67th BPSC (Pre) Cancelled Exam. 2021-22 336 YCT

You might also like